ANZCA FINALS PAST MCQ

Ace your homework & exams now with Quizwiz!

In an infant, the intercristine line is at the level of A. L1-L2 B. L2-L3 C. L3-L4 D. L4-L5 E. L5-S1

ANSWER E CEACCP http://ceaccp.oxfordjournals.org/content/4/5/148.full

Severe pre-eclampsia. WORST treatment option: A. Magnesium B. Nifedipine C. Metoprolol D. SNP E.

ANSWER A

Desflurane vaporiser, heated because of A. High SVP B. High boiling point C. Low SVP D. High MAC E. Low MAC

ANSWER A Desflurane's SVP is so HIGH (~700mmHg). The other reason why a Tec-6 vaporiser is used with Desflurane is b/c of it's LOW boiling point (not HIGH boiling point - as per possible answers). As mentioned above, at 23C, that ambient temperature may cause the liquid to boil, causing significant variation in the vapour concentration. Read 'Understanding Vapourizers - CEACCP article

Doing an awake CEA. Patient becomes confused & combative after carotid clamped and opened. Priority is... a) tell surgeon to release clamp b) tell surgeon to place shunt c) induce GA d) give midazolam

ANSWER B In this case a shunt should be placed as the carotid is clamped and open.

In a normal pregnant woman laboratory tests would show: A. an arterial pH of 7.4 B. an increase in functional residual capacity (FRC) C. decreased oxygen consumption D. an arterial base excess of +5mmol.l-1 E. a PaCO2 of 50 mmHg

ANSWER A To quote KB: "This is the only example of full acid-base compensation in normal physiology." * pH increases to 7.41-7.46 A&IC 33:2 p168 table (2005). * FRC decreased during pregnancy * pCO2 decreased to 30-32 mmHg

AM14 Dantrolene sodium: A. Has muscle relaxant properties B. Has muscle stimulant properties C. Is used in the treatment of malignant hyperpyrexia D. Is more soluble in acidic solution E. Decreases intracellular Ca++

ANSWER A C E

A 23 year old man takes his buprenorphine patch off prior to surgery. When can you expect the plasma level to be half? a. 6 hours b. 12 hours c. 24 hours d. 36 hours e. 48 hours

ANSWER B

A post-op child being given 2.5%D + 1/2NS on the ward seizes, is intubated and ventilated and transferred to ICU. Sodium is 116. What do you do? A. Give phenytoin B. Give hypertonic saline C. Give normal saline D. Give frusemide E. Give demopressin

ANSWER B

Autonomic ?neuropathy/ ?dysfunction is not associated with A. Spinal cord transection B. Hypothroidism C. Diabetes mellitus D. Tetanus E. Alcoholism

ANSWER B

How far to insert PICC line in a kid beyond the carina A: At the carina B: 1cm below C: 1cm above

ANSWER B

Nerve block for anaesthesia over anterior 2/3 of ear? A. C2 B. Mandibular nerve C. Maxillary nerve D. Ophthalmic nerve E. Vagus

ANSWER B

Pre-eclamptic woman BP 180/110. Aim to drop BP to a) 150-160 b) 140-150 c) 120-130 d) 110-120 e) 100-110

ANSWER B

The carotid sinus derives its nerve supply from the A. vagus nerve B. glossopharyngeal nerve C. ansa cervicalis (hypoglossi) D. middle cervical ganglion E. stellate ganglion

ANSWER B

Useful tests in assessing secondary metastases to the liver: A. Serum bilirubin B. Serum alkaline phosphatase C. BSP excretion D. Cephalin flocculation tests

ANSWER B

What nerve supplies sensation to the larynx above the vocal cords: A. internal branch of superior laryngeal nerve B. external branch of superior laryngeal nerve C. recurrent laryngeal nerve D. glossopharyngeal nerve E. palatotonsillar nerve

ANSWER B

You see a patient in the pre-op clinic. He is on propranolol for treatment of long QT syndrome. Which of the following will give the best reassurance that his treatment is effective? a. normal QT interval on resting ECG b. no change in QT interval with valsalva c. HR less than 60 d. no arrhythmias on 24h holter monitor e. decreased QT interval in response to a valsalva manoeuvre

ANSWER B

What is the oxygen concentration in a standard bottle of heliox? A. 21% B. 25% C. 30% D. 33

%ANSWER A

AZ55 [Apr98] (type A) Cricoid pressure applied at 40 Newtons would: A. Halve the lower oesophageal sphincter (LOS) pressure B. Double the LOS pressure C. No effect on LOS pressure/tone D. Resist a gastric pressure of 40 mmHg. E. Resist a gastric pressure of 80 mmHg.

ANSWER ?A ?D 40N resists 38mmHg and 40mmHg OESOPHAGEAL pressure depending on study. Halves LOS tone from 24 to 12.

32 y/o male. Weakness distal and prox muscles, infection 10 days ago, no sensory involvement, temp 37.8, facial weakness. Cause: A. Guillian Barre B. Myasthenia Gravis D. Poliomyelitis E. ?Acute encephalitis F. ?Polymyositis

ANSWER A

Which of the following drugs does NOT significantly cross the placenta? A. Heparin B. Warfarin C. Propranolol D. Cimetidine

ANSWER A

What is the most accurate method of determining fetal heart rate in a neonate A. Palpation of an umbilical vein pulse B. Auscultation with a stethoscope C. Palpation of femoral pulse D. Pulse oximetry

ANSWER B Australian resus council. HR can be determined by listening to the heart with a stethoscope (most reliable) or in the fist few minutes after birth by feeling for pulsations at the base of the umbilical cord.

Cerebral palsy is associated with each of the following EXCEPT A. gastro-oesophageal reflux B. increased sensitivity to non-depolarising muscle relaxants C. malnutrition D. recurrent aspiration E. scoliosis

ANSWER B Cerebral Palsy associated with increased sensitivity to depolarising NMBDs and resistance to non-depolarising NMBDs.

An 18 yo with Fontan circulation undergoing exploratory laparotomy. On ICU ventilation, saturation is 70%. Which ventilator parameter would you INCREASE to improve his saturation? A. Bilevel pressure B. Expiratory time C. Inspiratory time D. Peak inspiratory pressure E. PEEP

ANSWER B During IPPV a DECREASE in inspiratory time (which means an INCREASE in expiratory time) will improve pulmonary blood flow - despite the increased peak airway pressures.

The estimated risk of infection following percutaneous exposure (needlestick injury) to human immunodeficiency virus (HIV) is approximately A. 1 in 30 B. 1 in 300 C. 1 in 3,000 D. 1 in 30,000 E. 1 in 300,000

ANSWER B HIV 0.3% HCV 3% HBV 30% Factors increasing risk of viral transmission: * higher viral titres (for example higher HIV if terminal AIDS, higher HCV if coinfection with HIV) * hollow bore rather than solid needle * deeper puncture * visible blood on needle

Flow with the O2 flush button pressed and volatile agent turned on will give you: A: 20-30l/min O2 B: 30-70l/min O2 C: volatile agent + 30l/m O2 D: volatile agent + 40l/m O2 E: volatile agent + 50l/min O2

ANSWER B The oxygen flush valve allows direct communication between the oxygen high-pressure circuit and the low-pressure circuit. Flow from the oxygen flush valve enters the low-pressure circuit downstream from the vaporizers and downstream from the Ohmeda machine outlet check valve. Actuation of the valve delivers 35 to 75 L/min at a presusre of 50psi to the breathing circuit.

Closed circuit anaesthesia with 70%N2O,70kg man (low flow i think) what is the uptake of N2O after 90 mins anaesthesia: A. less than 50ml/min B. 100ml/min C. 200ml/min D. 500ml/min E. 1000 ml/minut

ANSWER B uptake is 1000 divided by the square root of the time administered in minutes (which works out roughtly to be B

75 year old with non-valvular AF usually on warfarin has their warfarin stopped for one week. What is their daily risk of stroke? A: 1% B: 0.1% C: 0.01% D: 4% E: 10%

ANSWER C

You see a man in his 60s in clinic 1 week prior to laparoscopic cholecystectomy. He has dilated cardiomyopathy with an ejection fraction of 30%, but does not get dyspnoeic with normal activities of daily living. What is the most appropriate management of his heart failure? A. Frusemide B. Amiodarone C. ACE D. Digoxin E. Biventricular pacemaker

ANSWER C

For oral intubation in a neonate, what is the best length of the ETT at the lips? A. 7 cm B. 8 cm C. 9.5 cm D. 11 cm E. Age/2 + 12

ANSWER C The approximate depth of insertion measured from the centre of the lips for an oral tube: * in a newborn is 9.5 cm, * 11.5 cm for a 6 months old infant * 12 cm for a 1 year old. * Thereafter, the approximate depth of oral insertion is given by the formula: age (years)/2 + 12 cm

A patient undergoing suprapubic prostatectomy appears to be bleeding excessively. In an attempt to exclude primary hyperfibrinolysis as a possible cause the most useful test is A clot retraction time B plasma fibrinogen estimation C prothrombin time D thromboelastography E whole blood clotting time

ANSWER D Fibrinolysis -breakdown on clot Congential -deficiency in a2 antiplasmin Acquired -liver dysfunction -severe trauma -major surgery -fibrinolytics : streptokinase Diagnosis via TEG APTT and PT are poor Treatment -transaemic acid -aminicaproic acid

Clinical signs of a low serum magnesium level after prolonged I.V. nutrition does NOT include A. muscular weakness B. tetany C. hallucinations D. shortened PR interval E. hyperventilation

ANSWER D Hypomagnesemia -occurs in nearly 12 percent of hospitalized patients -60 to 65 percent of intensive care unit patients Signs and symptoms -generalized weakness -anorexia -positive Trousseau and Chvostek -Apathy -delirium -coma -generalized convulsions ECG changes -ventricular arrhythmias -widening of QRS complex -Peaked T waves -prolonged PR interval Associations 1. hypokaelaemia : diuretic therapy and diarrhoea 2. Inhibtion of PTH : hypocalaemia

Neurolytic lumbar sympathetic block: Confirm needle placement by: A. Injection local anaesthetic to check effect just prior to alcohol injection B. Nerve stimulator C. Injection gives sensation of warmth in affected area D. Use an image intensifier

ANSWER D Needle position MUST be confirmed radiographically prior to injection" - Miller 5th 2364

The most appropriate investigation to diagnose Type A aortic dissections in potentially unstable patients is A. angiography B. CAT scan C. magnetic resonance imaging (MRI) D. transoesophageal echocardiography E. transthoracic echocardiography

ANSWER D Patient is potentially unstable.

A child with 10% dehydration is likely to have A. Bradycardia B. Mildly decreased tissue turgur C. Moist mucus membranes D. Deep rapid breathing E. Mood changes

ANSWER D Severe dehydration. See RCH http://www.rch.org.au/clinicalguide/guideline_index/Dehydration/

Patient over-warfarinised and is for surgery. Prothrombinex 50U/kg may NOT reverse an INR of 5.5 because it contains: A. Citrate B. Heparin C. Anti-thrombin III D. Not enough Factor VII E. Not enough Factor X

ANSWER D Warfarin inhibits the synthesis of 2, 7, 9, 10 Prothrombin X contains 2, 9 ,10, it has little to no Factor 7

Type I allergic (urticaria, anaphylaxis) reactions to latex are due to the: (a) chemical addatives (b) corn starch (c) latex polysacharides (d) latex protein

ANSWER D http://www.allergy.org.au/aer/infobulletins/pdf/Latex_Allergy.pdf

The action of which drug is unchanged in a recipient following cardiac transplantation A. Adenosine B. Adrenaline C. Atropine D. Digoxin E. Isoprenaline

ANSWER E

To achieve maximum anaesthesia with minimal risk of trauma to veins, the tip of a needle used for a medial peribulbar injection should be advanced no further past the equator of the globe than A. 5 mm B. 10 mm C. 15 mm D. 20 mm E. 25 mm

ANSWER B 10mm past equator as CEACCP says should not go past posterior border of globe.

Blood donors most commonly implicated in TRALI (Transfusion Related Acute Lung Injury) are A. diabetics B. donors previously resident in the United Kingdom C. indigenous Australians D. men under 50 years of age E. multiparous females

ANSWER E Transfusion-related acute lung injury TRALI is a form of non-cardiogenic pulmonary oedema (ARDS) that occurs following transfusion of blood or blood products. TRALI is most commonly caused by a reaction to leukocyte antibodies present in the plasma component of blood products. These antibodies can activate granulocytes that cause plasma leakage into the lungs, resulting in acute pulmonary edema. According to the Food and Drug Administration, TRALI is a leading cause of transfusion-related deaths in both male and female patients. Multiparous women: The implicated components in TRALI reactions are usually associated with female donors who have had one or more pregnancies or any donor who has been previously transfused. Women are more likely than men to have antibodies to leukocytes due to exposure to fetal antigens during pregnancy. There is currently no screening test for the prevention of TRALI and there is no single intervention that can eliminate the risk of TRALI. So: * Donors with potentially causative antibodies who have been implicated in TRALI reactions are permanently deferred. * Donor leucocyte antibodies are produced mainly as a result of pregnancy. Consequently, transfusing blood from male donors only greatly reduces the risk. In future, all FFP will be sourced from male donors and this should virtually eliminate the risk of TRALI. Presentation TRALI is similar to adult respiratory distress syndrome (ARDS) in its clinical presentation. It presents with: * respiratory distress (dyspnoea, cyanosis), * tachycardia, * fever, and * hypotension. Over several hours the CXR shows 'white-out' with diffuse alveolar and interstitial infiltrates. Symptoms usually arise within 1-6 hours of commencement of transfusion of a plasmacontaining product. Approximately 80% of patients with TRALI improve rapidly (clinically and radiologically) over 48 hours provided there is prompt and vigorous respiratory support. Many patients will require intubation and respiratory support." Diagnostic features * Onset within 1-6 hours of transfusion; * Acute respiratory distress; * Acute bilateral pulmonary edema (noncardiogenic); * Severe hypoxemia; * Hypotension (rarely hypertension); * Fever; * Clinical spectrum: from mild to severe; and * Mortality rate: 6% to 10%. Differential diagnosis Besides TRALI, other transfusion-related complications need to be evaluated in the differential diagnosis, including * circulatory overload (TRACO), * bacterial contamination, and * acute hemolytic transfusion reactions. At variance with circulatory overload that is associated with increased central venous pressure, TRALI is usually not associated with cardiac dysfunction (noncardiogenic pulmonary edema).

Acute visual loss after non-ocular surgery is most commonly caused by A. ischaemic optic neuropathy B. prolonged direct compression of the globe C. cortical blindness D. retinal artery occlusion E. electrolyte imbalance

ANSWER A

"Allodynia" is: A. Pain caused by stimuli that are usually not painful B. The 'burning' sensation of causalgia C. Red flare with nerve damage D. Due to reflex sympathetic dystrophy E. Not associated with nerve damage

ANSWER A Allodynia is a pain due to a stimulus which does not normally provoke pain and can be either thermal or mechanical

MR36 ANZCA version [2002-Aug] Q106, [2003-Apr] Q2, [2004-Aug] Q14, [2005-Apr] Q5 The most likely cause of superior vena cava syndrome is A. bronchogenic carcinoma B. mesothelioma C. thymoma D. teratoma E. apical pulmonary bullae

ANSWER A Bronchogenic carcinoma is BEST answer: Lung cancer is now the underlying process in approximately 85% of the patients with SVC syndrome 1. SVC Obstruction and Collateral a) Obstruction below azygous vein · Azygous - hemiazygous, lumbar veins to IVC b) Obstruction above azygous vein · Venous collateral in neck to azygous to SVC c) Obstruction includes azygous vein · Internal mammary, paraspinous, esophageal and subcutaneous vein to IVC d) Cerebral decompression through a single jugular vein via midline intracranial venous sinuses 2. Pathogenesis a) Extrinsic compression of SVC · Gradual SVC obstruction b) Invasion of SVC · Obstruction develops rapidly c) Thrombosis of SVC · Acute obstruction d) Venous hypertension and lymphatic obstruction - all empty into the subclavian veins 3. Causes a) Benign 10% · Inflammatory - histoplasmosis, idiopathic fibrosing mediastinitis · Iatrogenic - pacemaker electrode, hyperalimentation or other CV line b) Malignant 90% · Bronchogenic, epidermoid 65-80% · Small cell 12-30% · Lymphoma 12-20% 4. Symptoms and Signs · Swelling face, neck, arms · Shortness of breath, orthopnea, cough and chest pain suggest upper airway obstruction · Hoarseness, stridor, tongue swelling, nasal congestion · Headaches, syncope and lethargy are caused by cerebral edema from venous hypertension · Symptoms worse lying down, bending forward · Symptoms of cerebral or laryngeal edema is associated witha reduced life expectancy of about 6 weeks, demanding urgent intervention · Caval obstruction may be the life-limiting problem of patients with underlying malignancy 5. Diagnosis a) Chest x-ray · Right hilar mass - bronchogenic carcinoma · Anterior mediastinal mass - lymphoma · Calcification - histoplasmosis b) Simultaneous bilateral arm venogram · Defines obstruction and collateral circulation · Identifies thrombus c) Computerized axial tomography · Assessment of mediastinum · Determine patency of jugular veins · Directed needle biopsy 6. Radiation Therapy · Since most cases due to malignancy, nearly all patients receive radiation · 80-90% relieved of SVC Syndrome · 50% of patients relapse · Relapse occurs in benign disease as well; although collaterals develop, thrombosis will continue to propogate and occlude these collaterals over time 7. Medical Therapy · Chemotherapy for lymphomas and small cell carcinoma · Diuretics and corticosteroids reduce cerebral edema · Anticoagulants in selected cases to prevent clot propagation · Thrombolytic therapy for selected acute thrombosis 8. Surgery · Severe SVC Syndrome associated with thrombosis of caval tributaries and inadequate collateral circulation · SVC bypass with composite autogenous vein grafts or PTFE 6-12 months after onset in benign causes or for palliation in malignant causes with severe or acute onset SVC syndrome

When hearing loss occurs following spinal anaesthesia, it is usually in which of the following frequency ranges: A. 125 - 1000 Hz B. 1500 - 3000 Hz C. 3500 - 5500 Hz D. 6000 - 10000Hz E. > 11000Hz

ANSWER A Hearing loss after spinal anesthesia at 125-1000Hz A decrease in CSF pressure may lead to an endolymphatic hydrops. No relation to age, weight, height, blood pressure. No association with post-dural puncture headache.

Paediatric ALS - 20 kg, VF has had 2 shocks only. Next step A. Adrenaline B. Amiodarone C. Shock 50J D. Shock 100J

ANSWER A Infant/Child ALS guidelines from UK Resus Council Etieology : respiratory or circulatory failire more common than arrythmias, leading to secondary cardiopulmonary arrest Changes in 2010 -adrenaline is given after 3rd shock for shockable rhythms and then every alternate cycle (3-5min of CPR) -if non shockable, adrenaline should be given ASAP -amiodarone is given after 3rd shock for shockable rhythms, repeat after 5th shock - Compression to Vent Ratio

Young child with WPW undergoes general anaesthesia. Intra operatively developed tachycardia. HR 220, BP 80/40. Best drug to cardiovert A. Adenosine B. Amiodarone C. Verapramil D. Sotolol E. Esmolol

ANSWER A Unstable arrhythmias should be shocked. If stable and SVT (regular narrow complex) give adenosine. If irregular (WPW with AF) treat with Fecainide and shock.

White cylinder with grey shoulder? A. CO2 B. Air C. O2 D. N2O E. N2

ANSWER A www.anaesthesia.med.usyd.edu.au/resources/lectures/gas_supplies_clt/gas_supplies.html Specific gases are assigned the following colours: • oxygen (white), nitrous (blue) • nitrogen (black) • acetylene (maroon), medical ethylene (violet), medical cyclopropane (orange) • carbon dioxide (grey), helium (brown) and argon (dark green) • medical breathing gas mixtures containing oxygen and an inert gas must be marked with alternating white and the second gas's colour on the shoulder e.g. black + white for air, brown + white for Heliox, etc. Wikipedia "CO2 cylinders have a grey shoulder"

Your registrar gives a Duchenne patient 1mg/kg of suxamethonium. What are you most worried about? A: hyperkalaemia B: rhabomyolysis C: MH

ANSWER A and B o Sux: Hyperkalaemia, rhabdo o NDMR: safe but reduced dose, prolonged block

Young woman with subarachnoid haemorrhage, hyponatraemia and increased urinary sodium (did not specify if high sodium concentration or total amount lost). What is likely cause? A. cerebral salt wasting syndrome B. SIADH C. HHH therapy D. Excess NS administration E. diabetes insipidus

ANSWER A or B

34. NEW. Patient with Marfan's and 2 hours of severe chest pain, mild hypertension and ECG showing ischaemia. The next best step is urgent: A: CT B: TOE C: ? D: Angiography and PCI E: Thrombolysis

ANSWER A? B?

Hyperparathyroidism and increased Ca+ A. Long QT B. Polydipsia and polyuria C. Short PR D. Increased GFR E.

ANSWER B

PL06 In 5 mls of 1% lignocaine with 1:200,000 adrenaline, the amount of adrenaline is: A. 10 mcg B. 25 mcg C. 50 mcg D. 100 mcg

ANSWER B

Respiratory function in quadriplegics is improved by A. abdominal distension B. an increase in chest wall spasticity C. interscalene nerve block D. the upright position E. unilateral compliance reduction

ANSWER B

Which of the following is NOT seen in haemophilia: A. Normal bleeding time B. Increased thrombin time C. Haemarthrosis D. Abnormal thromboplastin generation time

ANSWER B

Which volatile has the minimum effect on ICP at 1 MAC A. Isoflurane B. Sevoflurane C. Desflurane D. Enflurane E. Halothane

ANSWER B

In a penetrating chest injury what part of the heart is most likely to be injured A. Left ventricle B. Right ventricle C. Right coronary artery D. Right atrium E. Sinus node

ANSWER B RA 17% LV 40% RV 45% -> Right ventricle. http://www.medicine.mcgill.ca/mjm/issues/v01n01/cardiac.html

Early features of pulmonary hypertension include each of the following EXCEPT A. a narrowly split second heart sound (S2) B. an early diastolic murmur C. decreased oxygen saturation D. dyspnoea on exertion E. hyperlucent lung periphery on chest Xray

ANSWER B A. TRUE : a narrowly split second heart sound (S2) B. FALSE :an early diastolic murmur - this is a late sign of PHT C. TRUE decreased oxygen saturation - d/t V/Q mismatch, R>L via PFO, and decr CO w decr PvO2 D.TRUE : dyspnoea on exertion - cardinal sign E. TRUE : hyperlucent lung periphery on chest Xray - classic finding thus true

A 6 month old baby is booked for an elective right inguinal hernia repair. An apropriate fasting time is A. 2 hours breast milk B. 4 hours formula milk C. 5 hours breast and formula milk D. 6 hours solids E. 8 hours solids, 4 hours all fluids

ANSWER B As per RCH guidelines 2 hours clear fluids 3 hours for breast milk 4 hours for formula/cows milk 6 hours for solids But infants less than 6 months 4 hours for solids and forumula milk

What raises intra-ocular pressure (IOP)? A. metabolic acidosis B. respiratory acidosis C. miosis D. reverse trendelenberg (head up) E. carbonic anhydrase inhibitor

ANSWER B Increases ICP -Respiratory acidosis -MAP -Hypoxaemia -Straining -Blinking -Mydriasis Decreases ICP -metabolic acidosis -Miosis

Handling of hydatid cysts can lead to: (a) DIC (b) anaphylaxis (c) bradycardia (d) haemolysis (e) pulmonary hypertension

ANSWER B Rupture of the cysts during excision releases highly antigenic 'scolices' necessitating treatment with adrenaline and steroids.

Which patients do not get pulmonary hypertension a. ASD b. Chronic thromboembolism c. Tetralogy d. MR e. MS

ANSWER C

Autologous transfusion results in less A. Cost B. Blood waste C. Incompatible transfusion D. Unrequired transfusion E. Storage lesions

ANSWER C "Preoperative autologous blood collection reduces the risk of allogeneic red cell transfusion but increases the risk of receiving any red cell transfusion (allogeneic and autologous)." http://www.transfusion.com.au/transfusion_practice/blood_conservation/autologous_collection_requirements

A patient having a craniotomy has the CVP/arterial transducers at the level of the right atrium. The head is 13cm above the level of the heart. If the MAP is 80mmHg and the CVP is 5mmHg what is the cerebral perfusion pressure in mmHg A. 60 B. 62 C. 65 D. 70 E. 75

ANSWER C 1mmHg is 1.36 cm H2O. Head is 13 cm above heart (10mmHg) CPP=80-10-5 (CVP)= 65mmHg pressure at head.

If a patient experiences parasthesia in the little finger during supraclavicular brachial plexus block, the needle is in proximity to the A. posterior cord B. middle trunk C. Ulnar nerve D. lower trunk E. medial cord

ANSWER D

Which of the following causes the most heat loss in a neonate? A: conduction B: convection C: evaporation D: radiation E: vasodilation

ANSWER D

Which one of the following is most likely to be associated with a high mixed venous oxygen saturation (SvO2)? A. acute myocardial infarction B. acute pulmonary embolism C. cardiac tamponade D. sepsis E. severe liver disease

ANSWER D

Ketamine for acute pain relief A. an appropriate dose is 0.5 -1 mg/kg B. Midazolam does not help in unpleasant dreams / delirium C. Morphine is contraindicated D. Hallucinations are common E. Subcut is better than IV

ANSWER D A. FALSE * Usual dose is 0.1 - 0.3mg/kg/hour (or as an initial bolus) B. FALSE * Midazolam is useful C. FALSE * Need an opioid for best effect D. TRUE * Don't know about it being common, but... E. FALSE * S/c is not better than; but can be used instead of IV. "however SC infusion is also used, especially in palliative care, with a bioavailability (similar to IM) of approximately 90% (Clements et al, 1982)." From APMSE

Long-standing T6 paraplegia. Which is INCORRECT? A. constipation B. Poikilothermia C. Labile BP D. Reduced reflexes E. Flaccid paralysis

ANSWER E

Tracheo-oesophageal Fistula (TOF) A. is associated with cardiac anomalies in approximately 60% of cases B. is associated with oesophageal atresia in approximately 20% of cases C. is more common in males than females D. is usually left sided E. does not usually require contrast studies for diagnosis

ANSWER E A. Cardiac in 20% B. Atresia in 80-90% C. 1:1 D. right

AM24 [Aug96] Malignant hyperthermia: A. Autosomal recessive inheritance B. Defect located on chromosome 15 C. Calcium concentration increased in normal muscle cell D. ?

Inheritance of malignant hyperthermia is autosomal dominant -variable penetration and expression -numerous genetic mutations responsible for this disorder -not all of them directly affect the ryanodine receptor -chromosome 19.

A man on PCA controlled with 2 mg morphine bolus is having a lot of pruritus. You decide to switch him to fentanyl. Which dose is the most appropriate bolus to be equi-analgesic with morphine 2mg: A. 10mcg B. 20mcg C. 40mcg D. 60mcg E. 80mcg

ANSWER C

A patient with bitemporal hemianopia has a lesion in the: A. Optic nerve B. Optic tract C. Optic chiasma D. Optic radiation

ANSWER C

During elective major vascular surgery the best way to reduce the risk of acute renal failure is to maintain a normal A. central venous pressure B. mean arterial blood pressure C. renal blood flow D. systemic vascular resistance E. urine output

ANSWER C

In a group of subjects, the proportion vomiting is 80%. With treatment, this can be reduced to 60%. The number needed to treat (NNT) with this treatment is A. 3 B. 4 C. 5 D. 6 E. 7

ANSWER C

What is 1 MET uptake of oxygen DUKE'S ? A. 1-2 ml O2/kg/min B. 2-3 ml O2/kg/min C. 3-4 ml O2/kg/min D. 5-6 ml O2/kg/min E. 7-8mls O2/kg/min

ANSWER C

Apnoeic oxygenation in obese patient can be increased by A. Sniffing position B. Prone C. Supine D. Lateral E. Head up

ANSWER D? Apnoeic oxygenation requires a patent airway to allow diffusion of O2 down a concentration gradient to the alveoli.

The commonest post operative complication in a patient with a # NOF is A. UTI B. Pneumonia C. Delirium D. Myocardial infarction

ANSWER C Up to 50% have delerium. http://www.anzca.edu.au/resources/college-publications/pdfs/books-and-publications/Australasian%20Anaesthesia/australasian-anaesthesia-2007/Semple.pdf Recent article published by Weinberg, data from the Northern. Most common complication being anaemia requiring transfusion, followed closely by delerium (keep in mind this is a retrospective study and N=180). http://www.ncbi.nlm.nih.gov/pmc/articles/PMC3746209/

A Full Size C oxygen cylinder has pressure downregulated from? A. 16,000 kPa to 400 kPa B. 16,000 kPa to 240 kPa C. 14,000 kPa to 400kPa C. 11,000 kPa to 400 kPa D. 11,000 kPa to 240 kPa

ANSWER C http://www.anaesthesia.med.usyd.edu.au/resources/lectures/gas_supplies_clt/gas_supplies.html

In a patient with porphyria, the drug most likely to cause an acute episode is A. morphine B. propofol C. propanidid D. phenytoin E. atropine

ANSWER D

Lumbarsacral nerve does not supply: A. Subcostal nerve B. Ilioinguinal n C. Iliohypogastric n D. Femoral n E. Genitofemoral n (?)

ANSWER A

Trauma patient best indicator of good resuscitation A. Lactate level B. Heart rate C. Blood pressure D Acidosis (?)

ANSWER A

Trauma pregnant patient (?32wks) BP 70/40, P 50, intubated in emergency department, next management step: A)L tilt pelvis B)IV fluid bolus C)Arrange urgent caesarean section D)Vasopressor options (?Adrenaline, Metaraminol) A. Basics first, then fluid, then pressor.

ANSWER A

What antibiotics are required for bacterial endocarditis prophylaxis in a woman with MV prolapse for cholecystectomy. A. None B. gentamicin C. ampicillin and gentamicin D. ampicillin E. cephazolin

ANSWER A

Which drugs below does not need dose adjustment in renal failure patient A. Buprenorphine B. Morphine C. Tramadol D. Clexane E. Fluconazole

ANSWER A

The cause of early mortality (early - within 30 minutes) in a pregnant women with amniotic fluid embolism is A. Bronchospasm B. Hypovolaemia C. Malignant arrhythmia D. Pulmonary hypertension E. Pulmonary oedema

ANSWER D

Numb tongue and impaired taste sensation post LMA anaesthesia. A. Facial Nerve B. Mandibular division of CNV C. Lingual Nerve

ANSWER C

Systemic Vascular resistance index (SVRI) is equal to: A. SVR x BSA B. SVR / BSA C. SVR -BSA D. BSA / SVR E. SVR^2 / BSA

ANSWER A

AZ70 ANZCA version [2003-Apr] Q102, [2004-Apr] Q57, [2004-Aug] Q64 Acute cannabis use just prior to surgery is most likely to A. cause intra-operative bradycardia B. decrease anaesthetic requirements C. increase the incidence of peri-operative nausea D. increase the risk of infra-operative awareness E. reduce the reliability of a BIS (bispectral index) monitor

ANSWER B acute cannabis use have decreased anaesthetic requirement(doesn't stimulate the sym system) Amphetamines & Cocaine (stimulate the SYM system) - increase req on acute use only

AM40 ANZCA version [2001-Apr] Q125 In testing for malignant hyperthermia susceptibility 1. an increase in contracture to either halothane or caffeine (during in vitro testing) is considered a positive result. 2. defects in the RYR1 gene account for a major proportion of cases 3. the false positive rate for in vitro testing is approximately six percent 4. genetic testing can now be used to replace in vitro contracture testing

ANSWER 1 2 3 1. by the American protocol only, European needs both. 2. RYR1 in 85% of families 3. specificity 94% - Blue book 2005 4. only if the defect has been identified in the proband

A 40 year old man, 18 hours after a gastrectomy, suddenly becomes hypotensive with a blood pressure of 80/60, tachycardia of 110/min, CVP +2 cmsH2O and a temperature of 39C. The likely diagnosis is: A. Ruptured anastomosis B. Haemorrhage C. Acute gastric dilatation D. Pulmonary embolism E. Septicaemia

ANSWER A

Postdural puncture headache: which of the following have (NOT) been reported to help? A. Dopamine B. Saline C. Blood D. Binders E. IV caffeine

ANSWER A

Which of the following is NOT a feature of long-standing paraplegia above T6? A. Flaccidity of the leg muscles B. Poikilothermia C. Mass autonomic reflex D. Hyperkalaemia after Suxamethonium administration E. Labile blood pressure

ANSWER A

Rapid infusion of mannitol IV initially causes: A. Raised ICP B. Reduced CBF C. Reduced K D. Reduced Na E. ?

ANSWER A The physical bolus of Mannitol causes an initially transient increase, then decreases as interstitial water is drawn out.

Factors in spread of (thoracic) epidural EXCEPT A. Age B. Height C. Weight D. Volume of LA E. Speed of injection

ANSWER A and E

Lowering intra-ocular pressure by applying pressure to the globe (e.g. Honan balloon) is typically contraindicated in a patient having A. a revision corneal graft B. a revision trabeculectomy C. an extra-capsular lens extraction D. a redo vitrectomy E. repeat retinal cryotherapy

ANSWER B

Patient ingested 500mg/kg aspirin. In ICU, the most effective method to remove aspirin A. IV fluid B. Haemodialysis C. Sodium bicarbonate infusion D. Frusemide

ANSWER B

Pneumoperitoneum for laparoscopy is commonly associated with an INCREASE in each of the following EXCEPT A. arterial pressure B. inotropic state C. secretion of vasopressin D. systemic vascular resistance E. venous resistance

ANSWER B

Which of the following contributes the LEAST to motor neuropathy following the lithotomy position? A. age B. BMI (body mass index) >25 C. diabetes D. peripheral vascular disease E. smoking

ANSWER B * Surgical Factors * Improper lithotomy position * Extreme high lithotomy position * Prolonged maintenance of lithotomy (>2 hours) * Patient Factors * Hypotension * Thin body habitus * Old age * History of vascular diseasae * Diabetes * Smoking

The treatment LEAST likely to be useful for torsades de pointes is A. defibrillation B. procainamide C. magnesium D. electrical pacing E. isoprenaline

ANSWER B Torsade de pointes -uncommon and distinctive form of polymorphic ventricular tachycardia -characterised by a gradual change in amplitude and twisting of the QRS complexes around and isoelectric line, -150-250 bpm -associated with prolong QT, usually greater than 600msec Pathophysiology -abnormality underlying both acquired and congenital long QT syndromes is the ionic current flow during repolarization : affecting the QT interval -slow inactivation of the Ca++ and Na+ during phase 3 (repolarization) leading to prolonged QT and leaves a 'window' where single or repetitive depolarizations can occur, ei early afterdepolarization (EAD) Epidemiology -unknown prevalence -QTc is longer in Caucasians than blacks -Brugada syndrome is more frequent in asians -Torsades is 2-3 more common in women : longer QTs and more sensitive to QT prolonging drugs Risk factors -Congenital long QT syndrome -Female gender -Acquired long QT syndrome (causes of which include medications and electrolyte disorders such as hypokalemia and hypomagnesemia) -Bradycardia -Baseline electrocardiographic abnormalities -Renal or liver failure Etiology Congenital - Jervell and Lange-Nielsen syndrome -Romano-Ward syndrome -Brugada syndrome is characterized by a coved ST segment in the right precordial leads. The syndrome may cause sudden death due to polymorphic VT resembling torsade -Takotsubo cardiomyopathy (stress-induced cardiomyopathy) causes a predisposition to torsade Conditions assocaited - Electrolyte abnormalities - Hypokalemia, hypomagnesemia, hypocalcemia - Endocrine disorders - Hypothyroidism, hyperparathyroidism, pheochromocytoma, hyperaldosteronism -Cardiac conditions - Myocardial ischemia, myocardial infarction, myocarditis, bradyarrhythmia, complete atrioventricular (AV) block, takotsubo cardiomyopathy[6] -Intracranial disorders - Subarachnoid hemorrhage, thalamic hematoma, cerebrovascular accident, encephalitis, head injury -Nutritional disorders - Anorexia nervosa, starvation, liquid protein diets, gastroplasty and ileojejunal bypass, celiac disease Drug Causes -antiarrhythmic drugs : Class 1A (quinidine, procainamide, dispyramide) Class 1C (encainide, flecainide) Class 3 (sotalol, amiodarone) -Antihistamines : astemizole and terfenadine -Antibiotics - Erythromycin, clarithromycin, azithromycin, levofloxacin, moxifloxacin, gatifloxacin, trimethoprim-sulfamethoxazole, clindamycin, pentamidine, chloroquine - Antifungals - Ketoconazole, itraconazole -Antivirals - Amantadine -Antipsychotics - Haloperidol, phenothiazines, thioridazine, trifluoperazine, sertindole[9] , zimeldine, ziprasidone[7] -Tricyclic and tetracyclic antidepressants -Antihistamines (histamine1-receptor antagonists) - Terfenadine, astemizole, diphenhydramine, hydroxyzine -Cholinergic antagonists - Cisapride, organophosphates (pesticides) -Diuretics - Indapamide, hydrochlorothiazide, furosemide -Antihypertensives - Bepridil, lidoflazine, prenylamine, ketanserin -Lithium -Anticonvulsants - phenytoin, carbamazepine (possible) -Oral hypoglycemic -Citrate (massive blood transfusions) - Cocaine -Vasopressin (possible) - Fluoxetine (possible)%%Acute Treatment Short Term -same for acquired and congenital long QT syndrome except Beta1-adrenergic stimulation may be tried in acquired but contraindicated in congential 1. Remove offending agent 2. DC cardioversion if unstable 3. Predisposing conditions such as electrolyte disturbances corrected Pharmcological *magnesium is the drug of choice -1-2g IV over 30-60 seconds -repeated 5-15 minutes -OR continuous infusion at 3-10mg/min *Lidocaine has no effect in torsades, initial benefit but torsade will always reoccur *Isoprenaline can be used, accelerated AV conduction and decrease QT but increasing the heart rate (contraindicated in congenital) -Lignocaine and phenytoin provide inconsistent benefit. Amiodarone should be avoided because it can precipitate torsardes. Temporary transvenous pacing -same as isoprenaline -maintain HR 90-110 -atrial pacing is preferred to maintain a narrow QRS and short QT -however in AV block, ventricular pacing can be used -overdrive to 140bpm may be required Longterm treatment -Beta-adrenergic antagonists at maximally tolerated doses are used as a first-line long-term therapy in congenital long QT syndrome. -Propranolol -esmolol or nadolol also can be used. -Beta-blockers should be avoided in those congenital cases in which bradycardia is a prominent feature. Insertion of AICD -Permanent pacing benefits patients who remain symptomatic despite receiving the maximally tolerated dose of beta-blockers and can be used adjunctively with beta-blockers. It decreases the QT interval by enhancing the repolarizing potassium currents and suppressing EADs. High left thoracic sympathectomy, another antiadrenergic therapy, is effective in patients who remain refractory to beta-blockade and pacing. Accidental ablation of ocular efferent sympathetic nerves may result in Horner syndrome. Implantable cardioverter-defibrillators (ICDs) are useful in instances when torsade recurs despite treatment with beta-blockers, pacing, and possibly left thoracic sympathectomy. Beta-blockers should be used along with ICDs because shock can further precipitate torsade by adrenergic stimulation. In the United States, an ICD for refractory cases may often precede sympathectomy. Long-term treatment in acquired long QT syndrome usually is not required because the QT interval returns to normal once the inciting factor or predisposing condition has been corrected. Pacemaker implantation is effective in cases that are associated with heart block or bradycardia. ICDs are indicated in cases that cannot be managed by avoidance of the offending agent. The boundary between acquired and congenital may not always be clear. Additive factors are often present, and individuals may show increased susceptibility to QT effects.

Ageing (adult) causes: A: Decreased FRC B: Decreased Cardiac output C: Diastolic dysfunction D: Increased creatinine

ANSWER C

Aneursym clipping, Best monitor of depth of block during this is ? A. TOFR B. TOFC B. DBS C. PTC

ANSWER C

The nerve providing sensory supply to the airway muscle below (inferor) to the vocal cords is the A phrenic nerve B posterior thyroid nerve C recurrent laryngeal nerve D superior laryngeal nerve E tracheal nerve

ANSWER C Above the cord : SLN Below the cords : RLN

How quickly does the CO2 rise in the apnoeic patient ? A. 1 mmHg per min B. 2 mmHg per min C. 3 mmHg per min D. 4 mmHg per min E. 5 or ?8 mmHg per min

ANSWER C From Miller, 1st minute Co2 increases by 6 mm Hg all subsequent minutes 3 -4 mm Hg

What is the distance from the lips to the carina in a 70kg adult male in cm A. 21 B. 23 C. 25 D. 27 E. 29

ANSWER C Front teeth to carina (FTC) distance should be similar if not the same as the lip to carina distance. The distance between teeth and vocal cords is 12 to 15 cms and the distance between vocal cords and carina is 10 to 15 cms. http://www.ncbi.nlm.nih.gov/pmc/articles/PMC3237149 Paedatrics: Morgan formula FTC=0.1 x Height (cm) +5 http://www.ncbi.nlm.nih.gov/pubmed/18497600

MR08 ANZCA version [2002-Mar] Q8, [2003-Apr] Q46 (Similar reported question in [1985] [1987] [1988] [Sep90] [Mar91] [Aug91] [Mar06]) The development of a pleural effusion would be an unusual complication of A. streptococcal pneumonia B. staphylococcal pneumonia C. mycoplasma pneumonia D. pneumococcal pneumonia E. none of the above

ANSWER C In community-acquired pneumonia with pleural infection, the most common organisms are Streptococcus pneumoniae and the Streptococcus anginosus/milleri group, with Staphylococcus species accounting for 10% or less of infections.

Ventricular fibrillation (VF) following caudal anaesthesia in 20kg six year old child. The recommended dose of of Intralipid 20% is: A. 10mls B. 20mls C. 30mls D. 40mls E. 50mls

ANSWER C Intralipid 20% Bolus 1.5ml/kg over 1 minute Infusion 0.25 ml/kg/min Repeat bolus every 5 minutes X2 There after 0.5mk/kg bolus Stop infusion 10 minutes after ROSC Do not exceed maximum cumulative dose of 12ml/kg

The toxic level of lignocaine is: A. 1 mcg/ml B. 2 mcg percent C. 5 mcg/ml D. 3 mcg percent E. 10 mcg/ml F. 10 mcg percent

ANSWER C Local Anaesthetic Toxic plasma concentration (mcg/ml) Lignocaine >5 Prilocaine >5 Bupivacaine >1.5 Ropivacaine >4 Lignocaine: 5-10: excitatory 10-15: seizures, 15-25 coma >25 cvs collapse.

Which patients do not get pulmonary hypertension a. ASD b. Chronic thromboembolism c. Tetralogy d. MR e. MS

ANSWER C Must be C as you have some degree of RVOT obstruction thus pulmonary blood flow is decreased, and your mitral valve is competent so there is no increase in LA pressure.

Indication for percutaneous closure of ASD A. Ostium primum < 3cm B. Ostium primum > 3cm C. Ostium secundum < 3 cm D. Ostium secundum > 3cm E. Sinus venosus ASD

ANSWER C Of these, only an ostium secundum ASD is suitable for percutaneous closure.

A patient is requiring 70 mg per day of morphine by continuous subcutaneous infusion, for the treatment of cancer pain. You are asked to change the patient to oral morphine. An appropriate initial order for slow release oral morphine would be A. 35 mg twice a day B. 70 mg twice a day C. 100 mg twice a day D. 200 mg twice a day E. 300 mg once a day

ANSWER C Parental or oral morphine conversion is 3:1

Aneurysm sugery. Propofol/remifentanil/NMDR. DOA monitoring (Entropy). MAP 70 , HR 70/min, State entropy 50, Response entropy 70. What do you do? A. Fluid bolus B. Metaraminol C. Check TOF D. Nothing E. Increase TCI

ANSWER C Response entropy is a product of EMG & EEG. This implies that there is muscle activity & I would check TOF to see if more relaxant is required

Risk factors for post-operative delirium include all the following EXCEPT A. age greater than 70 years B. history of alcohol abuse C. perioperative morphine administration D. electrolyte disturbance E. benzodiazepine administration

ANSWER C Risk factors In all likelihood, patient predisposition, type of surgery and postoperative factors may be more important to the development of delirium than the choice of anaesthesia. A painstaking prospective study of the risk factors for postoperative delirium has been carried out. Independent predictors of delirium were: 1. Age appears to place patients at greatest risk. Usually, elderly patients (more than 65 years) are on polypharmacy. However, they have decreased ability to metabolize drugs. Furthermore, visual and hearing impairments predispose elderly to disorientation. Postoperative hypoxia is more common in the elderly given the higher incidence of cardiovascular, respiratory and cerebrovascular diseases in this group. 2. Existence of underlying brain disease (cerebrovascular accident, dementia); psychiatric illness or other medical conditions (congestive heart failure, liver disease, renal failure). 3. History of taking medicines causing altered mental status (e.g., benzodiazepines, antidepressants). 4. Patients with preexisting central nervous system disorders (dementia, Parkinson's disease) have higher rates of postoperative delirium. 5. Type of surgery: a) Procedures, longer in duration place patients at increased risk of intraoperative hypoxia. b) Cardiac surgery can result in hypoperfusion and microemboli formation, causing cerebral ischaemia. c) Orthopaedic procedures (repair of femoral neck fracture) chances of fat embolism, may cause delirium. d) Cataract surgery is often associated with delirium due to age, loss of vision and anticholinergic side-effects of the used ophthalmic drugs. 6. Metabolic insults include dehydration, hyponatremia, hyperglycemia, hypoglycemia, acid - base disorders, hepatic disease, renal disease and endocrine disease can cause delirium. 7. Infections like pneumonia, urinary tract infections, intra-abdominal infections and wound infections all can cause confusion in susceptible patients. 8. The sensory overload in the ICU can lead to sleep deprivation, which is a risk factor for developing delirium. 9. Postoperative pain, either uncontrolled or unaddressed has been shown to increase delirium rates. 10. Abuse of alcohol/benzodiazepines, can become delirious secondary to withdrawal symptoms. 11. Type of anaesthesia - sometimes it is stated that delirium is less common after regional as opposed to general anaesthesia. In a large randomized controlled-study of patients undergoing elective total knee replacement, Williams - Russo et al found no statistical difference between the incidence of postoperative delirium in patients following general anaesthesia and that following epidural anaesthesia.

What is the major cause of death in a patient with perforation of the pharynx, oesophagus or trachea? A: failure to intubate B: failure to ventilation C: sepsis D. pneumothorax E. cardiogenic shock

ANSWER C Risk factors include 1. Children and women (smaller airways) 2. Difficult intubation 3. Use of stylet or bougie 4. malpositioning of the tube tip, tube repositioning without cuff deflation 5. inadequate tube size, 6. vigorous coughing Mortality is due to innoculation of mediastinum leading to mediastinitis and sepsis

When stimulating the ulnar nerve with a nerve stimulator, which muscle do you see twitch? A. opponens abducens B. abductor pollicis brevis C. adductor pollicis brevis D. extensor pollicis E. flexor pollicis brevis

ANSWER C Ulnar nerve supplies the intrinsic muscles of the hand, except LOAF

A 5kg baby loses 65mls of blood at operation. Which of the following is correct? A. Transfuse as blood loss >20% B. Don't transfuse as loss is <10% C. Transfuse if further blood loss anticipated D. >25% blood volume lost E. Definitely do not give blood

ANSWER C Weight = 5kg Blood volume = 80 mls/kg = 80 x 5 = 400mls Blood loss = 65/400 = 16%

60 y.o with acute aortic regurgitation. In pulmonary oedema. BP 160/90. HR was about 90. What is the best treatment pre-op: A. Intra aortic balloon pump B. Dobutamine infusion C. Sodium nitroprusside infusion D. Beta blocker E. Dopamine infusion

ANSWER C the systolic blood pressure remains above 100 mmHg with good mentation and renal function, intravenous sodium nitroprusside should be added. The initial dose is 0.25 to 0.5 microg/kg per minute. Nitroprusside should not be used without beta blockade since vasodilation alone induces reflex activation of the sympathetic nervous system leading to enhanced ventricular contraction and increased aortic shear stress. Aim should be to augment forward flow as a temporising treatment -patient needs a new AV

A multi-trauma patient opens his eyes and withdraws to painful stimuli, but does not respond to voice. He is moaning but makes no comprehensible sounds. His Glasgow Coma Score is A. 5 B. 6 C. 7 D. 8 E. 9

ANSWER D

Factors which contribute to the increased risk of aspiration pneumonitis during pregnancy include all the following EXCEPT A. a tendency for the stomach to be pushed up against the left diaphragm B. decreased secretion of the hormone motilin C. increased acidity of gastric secretion D. increased gastrin production E. increased volume of gastric secretion

ANSWER D

Fat: blood coefficient- N2O, Desflurane, Sevoflurane, Isoflurane A. N2O ~ D > S > I B. N2O > D > S > I C. D > N2O > S > I D. N2O > D > S ~ I E . D > N2O > I > S

ANSWER D

Immediate treatment of oculocardiac reflex is: A. IV atropine B. Deepen anaesthesia C. Beta-blocker D. Stopping muscle traction E. Retrobulbar block

ANSWER D

In infants with congenital pyloric stenosis A. dehydration is associated with early hyponatremia B. plasma chloride levels seldom fall below 85 mmol.1-I C. renal conservation of hydrogen and potassium ions occurs D. the urine is initially alkaline, then may become acidic E. vomiting causes a loss of potassium ions

ANSWER D

Prolonged Trendelenburg (head-down) positioning causes: A. no change in intracranial pressure B. no change in intraocular pressure C. no change in pulmonary venous pressure D. increased myocardial work E. increased pulmonary compliance

ANSWER D

Thermoneutral zone in 1 month old infant ? A. 26 - 28 degrees Celcius B. 28 - 30 degrees Celcius C. 30 - 32 degrees Celcius D. 32 - 34 degrees Celcius E. 34-46 degrees celcius

ANSWER D

When investigating a patient following severe hypotension during anaesthesia, the most specific test to determine if there was no immmunological basis for the hypotension is A. plasma histamine level B. total IgE concentration C. complement activation D. mast cell typtase E. specific IgE levels

ANSWER D

Which local anaesthetic vasoconstricts skin blood vessels: A. Mepivacaine B. Bupivacaine C. Lignocaine D. Ropivacaine E. Prilocaine

ANSWER D

In patients with severe anaphylactoid reaction, A. previous exposure to the triggering drug is necessary B. pulmonary oedema is a common clinical finding C. bronchospasm is the most dangerous feature D. an aura may precede the acute reaction (if patient awake)

ANSWER D 1. FALSE : don't need prior exposure 2. FALSE : 3. FALSE : circulatory collapse is most dangerous feature 4. TRUE

Alcoholic patient undergoes unremarkable anaesthesia for explorative laparotomy for investigation of abdominal pain. No pathology is found. However, in recovery the following electrolyte disturbances found: Na 140 K 5.0 Cl 115 HCO 18 What is the most likely cause A. Acute renal failure B. Lactatic acidosis C. Methanol ingestion D. Chloride E DKA

ANSWER D ?Hyperchloraemic acidosis?

A patient has Haemoglobin 8G% and a reticulocyte count of 10%. This picture is compatible with: A. Bone marrow suppression B. Leukaemia C. Aplastic anaemia D. Hereditary spherocytosis E. Untreated pernicious anaemia

ANSWER D Hereditary spherocytosis is associated with an increased reticulocyte count. None of the others are.

Vitamin K dependent factors include: A. V B. VI C. VIII D. X

ANSWER D Vit K dependent clotting factors are II, VII, IX and X.

Epidural block to T2 causes all of the following EXCEPT: A. Bradycardia B. Vasodilatation C. Reduced circulating catecholamines D. Dyspnoea E. Elevated PaCO2

ANSWER E

Established Legionnaires disease is best treated with A. Aminoglycosides B. Cephalosporins C. Chloramphenicol D. Crystalline penicillin E. Erythromycin

ANSWER E

In body protected OR with a Line Isolation Monitor reading 0 mA. If you touch one active wire what will happen. a. nothing, because no connection to earth is completed b. you get shocked c. nothing because the floor is insulated d. nothing because your shoes are nonconductive e. RCD trips

ANSWER E

Patient with placenta acreta. Surgical management MOST likely to save her life A B lynch suture around the uterus for external tamponade B Rusch balloon in the uterus for internal tamponade C ligation of the internal iliac arteries D ligation of the uterine arteries E subtotal or total hysterectomy

ANSWER E

Side-effects of protamine include all of the following except: A. Anaphylaxis B. platelet dysfunction C. Pulmonary hypertension D. Pulmonary oedema E. Systemic hypertension

ANSWER E

The double stranded hepatitis B virus can survive outside the body for A. less than 4 hours B. six to twelve hours C. one to two days D. two to seven days E. more than seven days

ANSWER E

The organ most UNLIKELY to demonstrate an increase in blood flow in response to decreased capillary partial pressure of oxygen? A. Liver B. Skeletal muscle C. Heart D. Kidneys E. Lung

ANSWER E

Features of the transurethral resection of the prostate (TURP) syndrome include a11 of the following EXCEPT A. agitation B. angina C. bradycardia D. nausea E. tinnitus

ANSWER E 1. TURP syndrome * CVS - hypertension and bradycardia : volume overload a balance between absorption and loss +/- increased ADH (from stress and glycine) * Neurological : N, weakness, confusion, agitation, seizures and coma, from : o Dilutional hyponatraemia and hypoproteinaemia à cerebral oedema (controversy here because osmolality is normal) o Glycine toxicity - direct and from ammonia 2. Glycine Toxicity - direct effects on eye and heart (depressant - reversed by arginine) * Transient blindness from glycine toxicity (? An inhibitory neurotransmitter), last 12-24 hrs * Increased ammonia - biotransformation of glycine -> 500 mmol/L à N+V and coma (awakens when < 150 mmol/L), more likely if arginine deficient (as normally ammonia -> urea via ornithine cycle of which arginine important intermediary) * Transient hyperkalaemia has been reported (more than with mannitol) 3. Bladder perforation (1%) - traumatic or overdistension or rarely explosion of hydrogen (cautery - plus need O2 to enter system) * Extra-peritoneal - peri-umbilical pain * Intra-peritoneal - symptoms faster - generalised abdo and shoulder tip pain, N+V, abdo rigidity, pallor, sweating and hypotension * Management - cystourethography will diagnose and Rx : supra-pubic cystostomy (for intra-peritoneal rupture at least ? conservative Mx with IDC for extra) 4. Bacteraemia/Septicaemia - in 6-7% : chills, fever and tachycardia ; rarely septic shock (high mortality) - prostate often has bugs, +/- pre-op IDC * pre-op antibiotics - do not sterilise prostate as difficult penetration, but is recommended to decrease bacteraemia 5. Hypothermia - should use warmed fluids (does not increase bleeding due to v/d) 6. Bleeding and coagulopathy - commonly bleed postop * Very vascular and blood loss difficult to estimate as mixes with irrigation fluid * Dilutional thrombocytopaenia * Abnormal bleeding in < 1%, possibly either : o Local fibrinolysis due to release of plasminogen activator and urokinase from mucosa of urinary tract o Systemic absorption of resected prostate rich in thromboplastin (with assoc. low platelet count and low fibrinogen) and can lead to secondary fibrinolysis (giving high levels of FDP) If suspect 1o fibrinolysis could give aminocaproic acid 5g in first hr then 1g/hr, (but beware DIC : Amicar contra-indicated, instead give platelets, FFP and cryoprecipitate, pc as needed - HEPARIN is controversial) 7. Hypotension Several causes : * Blood loss -> hypovolaemia * CCF - due to volume overload * Anaemia and myocardial ischaemia or infarction * Hyponatraemia (? mechanism)

Recognised weaknesses of systematic reviews include all of the following EXCEPT A. publication bias B. duplicate publication C. study heterogeneity D. inclusion of outdated studies E. systematic review author bias

ANSWER E A, B, C and D are word for word out of Myles and Gin 115-116.

A patient with a head injury who has eye opening to speech, no verbal responses and withdrawal motor responses would have a Glasgow Coma Scale score of A. 4 B. 5 C. 6 D. 7 E. 8

ANSWER E E=3/4 V=1/5 M=4/6

The strongest indication for the use of transoesophageal echocardiography during major non-cardiac surgery is A. recent myocardial infarction (less than six weeks old) B. acute significant intra-operative ST segment depression on ECG C. massive intraoperative blood loss D. a history of severe valvular heart disease E. unexplained significant intraoperative hypotension

ANSWER E Practice Guidelines for TOE from ASA 2010 1. Known heart disease which would cause hemodynamic, pulmonary or neurological compromise eg. ASD, AS, HOCM, MI 2. Unexplained hypotension 3. Unexplained hypoxemia 4. Where life threatening hypotension is anticipated

Relative contra-indications to mediastinoscopy include A. cervical spondylosis B. emphysema C. mediastinal lymphadenopathy D. poor left ventricular function E. superior vena cava syndrome

ANSWER E Previous mediastinoscopy is a relatively strong contraindication to a repeat procedure because scar tissue eliminates the plane of dissection. Relative contraindications -Superior vena cava (SVC) syndrome increases the risk of bleeding from distended veins -severe tracheal deviation -cerebrovascular disease, -severe cervical spine disease with limited neck extension -stroke -previous chest radiotherapy -thoracic aortic aneurysm.

The most effective method of deep venous thrombosis (DVT) prophylaxis for a fifty-year-old woman presenting for anterior resection for cancer of the colon would be A. electrical calf stimulation B. Dextran 70 infusion C. graduated compression stockings D. intermittent pneumatic leg compression E. low dose heparin (5000 units bd)

ANSWER E Refer to NHMRC 2009 Guide to prevention of VTE - Clinical Practice Guide LMH > H > mechanical

MRI notes

Overview The 3 particular aspects of MRI affecting anaesthesia are: * need for the subject to lie still during the scan * the effects of the magnetic field (eg need for special monitoring modifications) * location outside OT The 2 indications for anaesthetic involvement are: * to provide immobility (eg children, uncooperative adults) * very sick patients requiring intensive monitoring About 3% of adults cannot tolerate MRI without anesthesia Duration A typical scan sequence lasts 15 to 25 minutes, although complex procedures may up to two hours. Generic list of hazards * from magnetic field * from precautions for field * hard to see or get to patient in tunnel * from remote location * from associated conditions * hypothermia in small children Issues with magnetic field Basic physics * most are 0.5-1.5 Tesla (T). POWH, Randwick, has a 3T MRI Scanner o 1 gauss= Earth's Magnetic Field o 1T = 10,000 gauss o gauss = 0.1milliT ie 5 gauss = 0.5mT * magnetic field decreases with distance problems to do with magnetic field * Hazards from magnetic field * Precautions to avoid hazards * Interference with magnetic field/signals from other equipment Hazards from magnetic field movement of ferromagnetic objects * projectiles (eg needles, laryngoscope, scissors) * movement of implants (eg aneurysm clips leading to intracranial haemorrhage; implanted heart valves -usually OK; cardiac stents areoften ferrous - wait 8/52 for them to be stuck down; endoluminal AAA - some kits have ferrous material in iliac sleeves/"legs"; foreign body in eye can lead to vitreoretinal haemorrhage) hearing * rapidly changing magnetic field leads to VERY loud 65-95dB knocking noises burns * magnetic field leads to eddy currents in electrical devices eg ecg electrodes - need carbon fibre ones * burns have been reported with: o standard ECG o standard SpO2 - (In one case, so so severe that patient lost a finger) o temperature monitoring * large metallic prosthesis will get localised heating but clinical impression is that thermal fx are mild and tolerable * electrical equipment tends to malfunction near magnetic field ECG * distorted so much it is usually useless * magnetic storage media will be erased (eg Credit cards, floppy discs) All staff, patients or relatives have to fill out questionaire to detect potential problems issues due to precautions due to magnetic field * see also positioning * long tubing o outside scan room set at 0.5mT boundary o gas analysis response time inc by 5-10sec * can't use invasive pressure monitoring * unfamiliar equipment * can't resuscitate patient while in the scanner interference with the test * magnetic field + quality of images can be messed with by radiofrequency waves from: o large nearby ferromagnetic objects o even small amounts of ferrous materials within 3D space to be scanned * so decreasing quality of images may be non-diagnostic * scan room shielded from outside electrical equipment * electrical equipment inside scan room needs to similarly shielded associated conditions -syndromic kids - watch for airways & CVS changes remote location * generic probs o of consideration to anesth needs in design (eg pipeline gases, suction, waste gas exhaust capabilities) o lack of trained personnel to assist in emergency IV contrast media for MRI * Paramagnetic * May not contain iodine * Gadolinium is the common ingredient * High therapeutic ratio * Side effects: o Headache o Nausea & vomiting o Local burning o Weals 2.5% * Anaphylaxis 1/100 000 Common agents * Gadopentate o Magnevist o 0.2-0.4ml/kg * Gadodiamine o Omniscan o 0.2ml/kg * gadoversetamide o Optimark Positioning issues and monitoring * equipment positioning * ear protection Two approaches to anaes equipment and magnetic field * Use specialised MRI compatible equipment within scan room * Use conventional equipment located outside field in control room Standardising on one option keeps anaesthetist and equipment together ie you either do one or the other Factors in decision * funds * space * expected frequency of GAs * individual preference Conventional outside approach advantages: * allows faulty monitors to be changed * cheaper * less crowding in scan room * can apply simply rule: Nothing enters scan room except patient and trolley typical set up * induction area adjacent to but outside scan room 0.5mT boundary * compact conventional anaes machine/monitoring in induction room * non magnetic tipping trolley * compact eg wall mounted conventional anaes machine in ctl room with 10m coaxial/bain syestem * MRI-compatible saturation probe fibreoptic pt probe shielded cable * extended gas sampling line (slows response time by 5-10 sec) * MRI compatible ECG - carbon fibre leads and electrodes * NIBP machine with extended hose, non-metallic connectors, range of cuffs * ECG: carbon fibre leads and electrodes Airway 3 issues/points: * decreased access * options available * practical issues Access limited so low threshold for intubation relative categories/options:3 -<2/12 will often sleep through short scan if fed wrapped up placed on side in scanner -ETT indications *suspect increased ICP *suspect Space occupying lesion head * kids <15kg *aspiration risk * LMA /sv ok if older kids or adults without above risks sedation * adults where anxiety main issue * back pain where ++ analgesia required to tolerate lying still * some centres have protocol for kids - questionable risk:benefit equipment/practical issues * RAE good cos keeps connectors/filters out of head ring * tape pilot balloon to something eg filter * cuffed tube handy - see below Ventilation issues with long tubing * if long tubing, use circuit that decreased dead space eg **bain **T piece *these systems tend to have expir resistance that leads to some PEEP, especially at higher FGF * PPV tidal volumes [=Vt] tend to be lower due to **expans'n of tubing **"compression losses " within tubing * cos Vt's tend to be lower to start with, leak around an uncuffed ETT not as well tolerated - consider going a size up or using cuffed ETT * simple Pressure generating ventilators may not cut the mustard * P's measured at anaesthetic machine end don't accurately reflect P's at pt end of tubing If patient arrests in scan room * Don't do ACLS in scan room * Don't allow arrest team into scan room * BLS in scan room: o external cardiac compression o non metallic bag/mask ventilation * transfer out using non metallic trolley Contraindications Absolute * Cardiac pacmakers * ICD * Metal in the orbit * Metal in the CNS (some modern aneurysm clips are not ferromagnetic) * ferromagnetic stapedial implants Relative * Other electronic implants o spinal stimulators o insulin pumps o lead wires * prosthetic heart valves * cochlear implants * Metal in other parts of body (joint replacements and secure plates are OK) * Tattoos

Uterine tone decreased / increased by: A. Serotonin B. Prostaglandin E2 C. Ketamine D. Verapamil E. Magnesium F. Beta-2 agonists G. GTN

Uterine tone increased with * Prostaglandin E2 * Ketamine * Serotonin Uterine tone decreased with * verapramil * magnesium * Beta 2 agonist * GTN

Flow with the O2 flush button pressed and volatile agent turned on will give you: A: 20-30l/min O2 B: 30-70l/min O2 C: volatile agent + 30l/m O2 D: volatile agent + 40l/m O2 E: volatile agent + 50l/min O2

ANSWER B

Which drug is an example of a specific PDE III inhibitor: a. Aminophyline b. Sildenafil c. Milrinone d. Dipyridamole e. ?

ANSWER C

Which is a specific PDE inhibitor? A. Theophylline B. Dipyridimole C. Milrinone

ANSWER C

ANZCA July 2007 [6] The most frequently reported clinical sign in malignant hyperpyrexia is A. arrhythmia B. cyanosis C. sweating D. tachycardia E. rigidity

ANSWER D

The following capnograph tracing is most likely to represent A. partial obstruction of sampling tube B. sticking ventilator bellows C. incomplete neuromuscular blockade D. air entrainment into sampling tube E. partial obstruction of endotracheal tube

//d3cgb598vs7bfg.cloudfront.net/images/upload-flashcards/front/1/5/46351363_m.jpg%%ANSWER D Low plateau is caused by entrained air during respiration Late peak results at end inspiration where there is a decrease in circuit pressure and briefly stop air entrainment. Usually due to breach in the sampling tube of side-stream capnometer

This ECG (Lead V5) most likely represents A. Wolff-Parkinson-White (WPW) syndrome B. atrial fibrillation C. complete heart block D. atrio-ventricular junctional rhythm E. sinoatrial block

//d3cgb598vs7bfg.cloudfront.net/images/upload-flashcards/front/8/6/47068222_m.jpg%%ANSWER D * Regular rate -> so not AF B * Rate 75/min -> too fast for complete heart block - so not C * No delta wave (so not WPW (option A)) * Narrow QRS -> Supraventricular rhythm PLUS (no P wave before QRS) ->AV junctional rhythm (D)

Hepatocellular damage indicated by raised: A. AST B. Alkaline phosphatase C. Alanine transaminase D. CK

ANSWER A and C

A multi-trauma patient opens his eyes and withdraws to painful stimuli, but does not respond to voice. He is moaning but makes no comprehensible sounds. His Glasgow Coma Score is A. 5 B. 6 C. 7 D. 8 E. 9

ANSWER D E=2/4 V=2/5 M=4/6

Subarachnoid haemorrhage patient. What percentage rebleed in the first 24hours A. <5% B. 5-10% C. 10-15% D. 15-20% E. >20

%ANSWER A

Dystrophia myotonica

* AKA Myotonia Dystrophica. A form of myotonic dystrophy. Not to be confused with muscular dystrophy. * Most common muscular dystrophy among Caucasians * AD with variable penetrance * Chromosome 19 locus q 12.3 * Defect is in Na conductance (via serine/threonin protein kinase) resulting in increased intra-cellular Na * incidence of 1 in 8,000. * A multisystem disorder, * onset 10 - 30 yrs (a congenital form exists) * distal muscles more involved than proximal muscles * Plasma CK normal or sl elevated. (M&M, Harrisons) * Progression: o myotonia early (persistent contraction after voluntary or mechanical stimulation) then o atrophy of face and neck, also o frontal balding and cataracts, o usually low intelligence, o Also possible dysarthria, dysphagia, and mild ophthalmoplegia. antagonist m. spasms when relaxes Drug effects * Prolonged respiratory depression with any resp depressant drug * Treatment: Na channel blockers: quinine, phenytoin and procainamide System effects * CVS : o conduction blocks (ECG : increased PR, ST and A flutter o Cardiomyopathy (<10%)-note that severe skeletal muscle disease is not necessary for a sever cardiomyopathy (see AIC 2001: Anaesthesia and myotonia-an Australian experience.) * Resp : o reduced VC, o exp. Effort/Pressure; o aspiration risk (common) * GIT : o Smooth mm is involved leading to dysmotility in stomach/oesophagus, colon, uterus * Endocrine : o Insulin resistance, o adrenal atrophy, o infertility in women, o testicular atrophy, primary testicular abnormalities * Early cataracts Diagnosis * Clincial * +ve FHx * EMG/Muscle bx * Genetic testing for mild cases useful Congenital form occurs in children born to mothers with myotonic dystrophy; some patients present with profound hypotonia at birth, with facial diplegia, feeding, respiratory difficulties, and skeletal deformities (such as clubfeet). Later, during childhood, delayed developmental progression is noted

Most common cause of maternal cardiac arrest A. Pulmonary embolism B. Amniotic fluid embolism C. Haemorrhage D. Preeclampsia E. Cardiomyopathy

ANSWER B CEMACH

The maximum osmolality to go up to when managing raised ICP when using osmotic diuretics: A. 300 B. 320 C. 340 D. 360 E. 280

ANSWER B

Drug LEAST likely to cause hypoxia in ARDS a. Noradrenaline b. Milrinone c. Isoprenaline d. Isoflurane e. SNP

ANSWER A

Anaemia in Chronic Renal Failure:- A. Hypochromic microcytic B. Normochromic microcytic C. Normochromic normocytic D. Hypochromic normocytic E. Hypochromic macrocytic

ANSWER C

Indicator in sodalime? A. Ethyl violet B. Potassium permangenate C. Blue dye D. Ethylene blue E. Magic marker

ANSWER A

Features most suspicious for myocardial ischaemia a. ST depression 2mm during fem pop bypass in 60 yo man under spinal b. T wave inversion in fem pop bypass in 60yo under spinal c. 0.7mm ST elevation in fem pop bypass in 60 yo man under spinal d. SAH in young man e. 32 yo woman during LSCS

ANSWER A

HOCM, VF arrest on induction, what's the first priority in management? A. defibrillate B. amiodarone C. Intubate and ventilate D. Pre-cordial thump E. adrenaline

ANSWER A

How do you calculate the inspiratory time constant for lungs A. resistance multiplied by compliance B. resistance divided by compliance C. compliance divided by resistance D. resistance minus compliance E. resistance plus compliance

ANSWER A

PP96 [Jul07] Newborn with diaphragmatic hernia. Initially sats 95% on RA. Now beginning to develop respiratory distress. Next appropriate step: A. Awake intubation B. Trial NO prior to intubation and ventilation C. Rapid sequence induction D. Mask ventilation throughout case with CPAP E. Gas down with CPAP

A. Difficult in newborn B. Contraindication to nitrous oxide as it will cause distention and worsen hypoemia C. RSI sounds good D. No CPAP E. NO CPAP

A 30-year-old man presents to the Emergency Department following a high speed motor vehicle accident. He has a Glasgow Coma Score of 7 and arrives with a cervical collar in situ and an 18 gauge intravenous cannula in his right hand. You first priority in managing this patient would be to A. insert a large bore intravenous cannula B. perform a CAT scan of the brain C. perform a cervical spine X-ray D. perform a plain chest and pelvic X-ray E. secure the airway with an endotracheal tube

ANSWER A

Cephalothin doesn't cover: A. Proteus B. E coli C. Staph D. Strep E. Pseudomonas

ANSWER E

Dural Sac ends at what level in a neonate? A. L1 B. L3 C. L5 D. S1 E. S3

ANSWER E NYSORA "The dural sac in neonates and infants also terminates in a more caudad location compared to adults, usually at about the level of S3 compared to the adult level of S1"

The MAC (Minimum Alveolar Concentration) of desflurane in infants less than one year of age is A. less than 6% B. 6 to 7% C. 7 to 8% D. 8 to 9% E. greater than 9

%ANSWER E Oxford Paediatric Anaesthesia says infant = 9.4%, neonate = 9.1%

18yo patient in a psych unit, being treated for frequent vomiting. Collapses and found unconscious. ABG's: pH 7.22 CO2 40 PO2 100 HCO3 16 Na 138 K 4.0 Cl 105 Diagnosis is: A. anorexia nervosa B. diabetic ketoacidosis C RTA D. Normal saline administration E) Gastric outlet obstruction

ANSWER B the gas definitely showed a metabolic acidosis with AG 21; pO2 was safe. High anion gap metabolic acidosis with normal chloride - only one that fits is DKA.

A three-week-old infant presents with vomiting for 3 days and a diagnosis of pyloric stenosis. The most correct statement concerning resuscitation is that: A. alkalosis should be treated with dilute hydrochloric acid B. 5% albumin is an appropriate fluid for initial treatment of shock C. fluid deficit should be completely replaced with 5% dextrose D. hyponatraemia should be treated with 2N (twice normal) saline E. potassium replacement should begin immediately

ANSWER B • A. False • B. True - well documented benefits of 5% albumin for resuscitation in children - outcome benefit in surgical patients with 40ml/kg. • C. False - poor choice, min time in intravascular volume, inc. Intracellular fluid • D. False - slow rehydration/optimization better or risk hyperchloraemic metabolic acidosis and Central Pontine Myelinolysis. • E. False - slow correction after fluid resuscitated first

The prevalence of latex positive latex skin prick in atopic individuals without increased exposure to latex is about: (a) 0.36 % (b) 1.8 % (c) 9 % (d) 50

%ANSWER C No risks : 0.4% Increased exposure : 7% Atopic : 9% Increased exposure and atopic : 37% ANZCA/ASA Welfare of Anaesthetists SIG http://www.anzca.edu.au/ceqa/sig_general/welfare/rd18.htm

Complex regional pain syndrome. What proportion of patients have motor involvement ? A. 0 % B. 25 % C. 50 % D. 75 % E. 95

%ANSWER D Motor / trophic changes - motor dysfunction 57-98% - weakness 75-95% - limited range of movement 80-88% - incoordination 47% - tremor 48% - spasm 13% - dystonia 14% - myoclonus 4-20%

Sepsis from Yersinia infection from blood transfusion, mortality? A. <5% B. 20% C. 40% D. 60% E. 80

%ANSWER D Yersinia Contamination -uncommon -rapid onset, usually immediate but must be considered for any hypotension post 24hours after infusion -high mortality (50%) -Platelets >> PRBC = FFP=Cryo -Caused by bacterimic donor at time of donation Treatment -early recognition -stop -broad antibiotics -steriods -ICU Prevention -visual inspection : cloudy/clots -tranfusion rate <4 hours

Retrolental fibroplasia: A. Is prevented by control of FIO2 B. Is a common cause of myopia C. Causes retinal detachment leading to blindness D. May recover if mild E. Occurs more commonly in premature babies F. Is more common in premature infants

ANSWER C A. Is prevented by control of FIO2 - Not prevented, but high FiO2 increases risk B. Is a common cause of myopia - No C. Causes retinal detachment leading to blindness - Yes, it can do, due to fibrous scar tissue; the key disease element is fibrovascular proliferation. D. May recover if mild - Well, it may not progress to retinal detahcment, but recover?? E. Occurs more commonly in premature babies - It ONLY occurs in premature babies F. Is more common in premature infants - as (E)

A 25 kilogram child has a haemoglobin of 65 g.l-1. The child is normovolaemic and there is no on-going blood loss. The volume of packed cells required to increase the haemoglobin level to 90 g.l-1 is A. 100 ml B. 150 ml C. 250 ml D. 350 ml E. 450 ml

ANSWER C Frank Shan 4 x 2.5 x25 = 250ml

Pre eclamptic patient post LSCS continue on Mg infusion in ICU. Found to be in respiratory depressed. Next management A. Calcium gluconate B. IV fluid C. Frusemide D. Naloxone E. Bolus more magnesium

ANSWER C From RWH Request magnesium level and review management if: * respiratory rate < 12 breaths/minute * urine output < 100mLs in 4 hours * loss of patellar reflexes * further seizures occur. Response to magnesium toxicity The following clinical signs of magnesium toxicity must be reviewed by a consultant obstetrician/anaesthetist: * urine output <100mL in 4 hours * absent patellar reflexes * respiratory depression. The antidote for magnesium toxicity is: calcium gluconate (10mL of 10% solution over 10 minutes) by slow intravenous injection. The patient requires ECG monitoring during and after administration because of the potential for cardiac arrhythmias. Resuscitation and ventilator support should be available during and after dose administration of both magnesium sulphate and calcium gluconate.

2 month old systolic murmur heard at apex no change with posture, now on 5th centile for weight after being on 30th at birth, mother states has difficulty feeding. Peripheral pulses reduced femoral more than upper body. Most likely cause: A. HOCM B. VSD C. PDA D. venous hum E. ASD

ANSWER C PDA - collapsing pulse with sharp upstroke from ejection of large volume of blood into empty aorta during systole, low diastolic pressure due to rapid decompression of aorta, hyperkinetic apex, single S2 if large or even reversed splitting of S2, continous loud "machinary murmur at 1st LIC space. sometimes associated with flow murmur through left heart eg. mitral mid diastolic murmur.

When compared to a non-obese patient, in an obese patient (BMI >35) when fasted for an elective procedure the gastric secretion will have: A: more volume, higher pH B: more volume, lower pH C: same volume, same pH D: less volume, lower pH E: less volume, higher pH

ANSWER C Same volume same pH See A & A December 2001 vol. 93 no. 6 1621-1622 Initially found to have increased volume and decreased pH. However, more recent studies have failed to demonstrate this finding. Possible confounder is Innovar (premed of droperidol and fentanyl) http://www.anesthesia-analgesia.org/content/93/6/1621.full

Globe perforation with eye blocks is most likely with: A. Axial length <25mm B. Medial canthus peribulbar injection C. Inferotemperal peripulbar injection D. Age < 40 years E. Sub-Tenons

ANSWER C Unintentional globe perforation and rupture is the most devastating complication of eye blocks -poor prognosis, especially when the diagnosis is delayed -incidence 1 in 350 and 7 in 50,000 cases Main risk factors include -inadequate experience of the physician -highly myopic eye (axia length < 26mm) -injection temporal or inferior margin (peri and retrobulbar)

Radiation from X-rays A. has a non-cumulative effect on the bone marrow B. affects the thyroid gland more in patients over 16 years old C. has a recommended maximum whole body dose for the general population of 1,000 µSv.year-1 D. should be limited to less than 100 µSv.year-1 in pregnancy E. has a recommended maximum whole body dose for occupational exposure of 500,000 µSv.year-1

ANSWER C * A. False. Cumulative. * B. False. Affects patients under 16y more. * C. TRUE. * D. False. Should be limited to less than 1mSv per year (same as general population). * E. False. 50mSv per year = 50,000 microSv per year.

PZ26 ANZCA version [1988] [Mar91] [2002-Aug] Q43, [2003-Aug] Q76, [2004-Apr] Q46, [Mar06], [Jul06] Q9 The antibiotic LEAST likely to be effective for the management of anaerobic peritonitis is A. carbenicillin B. cefoxitin C. cephalothin D. chloramphenicol E. clindamycin

ANSWER C - Cephalothin Bacteroides fragilis and Clostridium perfringens are commonly implicated in anaerobic infections. Both these bugs are sensitive to cefoxitin, clindamycin and chloramphenicol and demonstrate some sensitivity to carbenicillin. (B. fragilis is resistant to cephalothin and C. perfringens has variable sensitivity.)

A 60yo man with anterior mediastinal mass, during induction for mediastinoscopy....lose cardiac output, decreased saturations, drop in ETCO2. Management: A. Adrenaline B. CPR C. CPB D. Place prone E.

ANSWER D

All of the following may be useful in the treatment of ventricular fibrillation due to bupivacaine cardiotoxicity EXCEPT A. adrenaline B. diazepam C. intralipid D. propofol E. vasopressin

ANSWER D

Young asthmatic male in emergency department. Resp rate 26/min, pCO2 27, SAO2 92%, struggling, talking in sentences. Given nebulised salbutamol, and ipratropium, 200mg IV hydrocortisone. After 30 minutes - no improvement. Further management: (NEXT step) A. IV salbutamol B. IV aminophylline C. IV magnesium D. Intubate and ventilate E. helium/oxygen mixture

ANSWER D A Cochrane meta-analysis of 7 studies concluded that IV magnesium sulfate improves pulmonary function and reduces hospital admissions, particularly for patients with the most severe exacerbations of asthma.

A patient with bipolar disorder presents for elective laparoscopic cholecystectomy. She takes lithium and has therapeutic blood levels. The lithium A. may increase her requirement for volatile anaesthetic agents B. may prolong the duration off depolarising muscle relaxants C. may prolong the duration of non-depolarising muscle relaxants D. may prolong the duration of both depolarising and non-depolarising muscle relaxants E. should be ceased 2 weeks pre-operatively

ANSWER D A. FALSE: reduced MAC D. TRUE: most correct answer E. FALSE: may precipitate rebound depression and should not be stopped perioperatively

AZ30 [Mar91] [Aug91] [Apr97] In a lightly anaesthetised patient who is apnoeic, the rise in pCO2 per minute is: A. 0.5 mmHg B. 1 mmHg C. 2 mmHg D. 3 mmHg E. 5 mmHg

ANSWER D D is best answer Rise in PaCO2 is 3-6mmHg/min (Nunn p238) Higher rate of rise early on then 3mmHg/min

A 30-year-old man presents to the emergency department following a high speed motor vehicle accident. His blood pressure is 70/50 mmHg with a strongly positive FAST (focused abdominal sonography for trauma). His chest X-¬ray shows a widened mediastinum. The most appropriate method to assess the widened mediastinum in this patient is A. aortogram B. CT angiogram of the chest C. repeat chest X-ray D. intraoperative TOE (transoesophageal echocardiography) E. transthoracic echocardiography

ANSWER D Grade 3-4 haemorrhagic shock with positive FAST : needs to go to theatre ASAP. The widen mediastinum can be evaluated intraoperatively with TOE

A morbidly obese 140kg, 40-year-old male is scheduled for cholecystectomy. He has no history of cardiac disease. His ideal body weight is 70kg. Compared to his resting cardiac output at ideal body weight, his resting cardiac output at his weight of 140 kg would be A. decreased by 20% or more B. decreased by 10% C. unchanged D. increased by 10% E. increased by 20% or more

ANSWER D Miller 6th edition says CO increases by 0.01L/min for each kg of adipose tissue. If this is true, a 10% increase in CO represents 50kg extra adipose tissue, whilst a 20% increase in CO represents 100kg extra adipose tissue (ie, more than 140kg!). Furthermore, not all the extra weight in the 140kg person is adipose tissue, some will be due to increased plasma, red cells and interstitial fluid.

PL11b ANZCA Version [Jul06] Q109, [Apr07] The local anaesthetic LEAST likely to cause cardiac toxicity after inadvertent intravenous injection is A. bupivacaine B. etidocaine C. levobupivacaine D. lignocaine E. ropivacaine

ANSWER D bupivacaine >tetracaine = etidocaine > lignocaine = chloroprocaine > mepivacaine = prilocaine >>>procaine

Acute visual loss following surgery in the prone position is most commonly caused by A. trauma to the cornea B. compression of the globe C. ischaemia of the visual cortex D. retinal detachment E. ischaemia of the optic nerve

ANSWER E Ischemic optic neuropathy is the most common diagnosis in postoperative visual loss. Ischemic optic neuropathy is divided into anterior and posterior, depending upon the location of the lesion on the optic nerve. Anterior ischemic optic neuropathy -53% cardiopulmonary bypass procedures -12% prone Posterior ischemic optic neuropathy -48% neck, nose or sinus operations -16% prone -11% cardiopulmonary bypass procedures

The most correct statement regarding intra-operative glucose management in children is that A. 5% dextrose should be used B. there is a higher incidence of hypoglycaemia in children following overnight fasting than in children fasting during the daytime C. blood glucose monitoring is mandatory D. hyperglycaemia is harmless E. most children do not require glucose intra-operatively

ANSWER E * A: False - 5% dextrose should NOT be used due to risk of hyperglycaemia and hyponatraemia * B: False - due to diurnal variation in cortisol, daytime fast result in higher incidence of hypoglycaemia * C: False - NOT mandatory * D: False - hyperglycaemia results in diuresis, dehydration, electrolyte disturbances, increase risk of hypoxic brain injury etc * E: True - can use either low glucose solutions or glucose free solutions for most kids.

Called to emergency department to review a 20 year old male punched in throat at a party. Some haemoptysis / hoarse / soft voice. Next step in management: A. CT to rule out thyroid cartilage fracture B. XR to rule out fractured hyoid C. Rapid sequence induction / laryngoscopy / intubation D. Awake fibreoptic intubation E. Nasoendoscopy by ENT in emergency department

ANSWER E Initial treatment 1. history, examination, XR 2. nasoendoscopty 3. CT scan Laryngeal trauma 1. impending airway => surgical tracheostomy 2. patent airway => AFO 3. AFO fails => surgical tracheostomy

During lumbar plexus block placement, which of the following indicates inappropriate needle placement? A: hip flexion B: hip adduction C: knee extension D: knee flexion E: lumbar extension

ANSWER E Method 1. Mark out midline over spinus processes 2. Mark out perpendicular line, line connect iliac spines 3. Needle insertion 4cm from midline along interiliac spine, aim perpendicular, parallel to spine 4. 6-8 cm depth 5. aim is for twitch of quadraceps (femoral nn), but twitch of any of the lumbar nerves possible 6. Nerve stimulation from 1mA to 0.5mA 7. aspirate and inject 20-25ml of solution (0.5-0.75% ropivacaine)

The strongest indication for the use of transoesophageal echocardiography during major non-cardiac surgery is A. recent myocardial infarction (less than six weeks old) B. acute significant intra-operative ST segment depression on ECG C. massive intraoperative blood loss D. a history of severe valvular heart disease E. unexplained significant intraoperative hypotension

ANSWER E TOE here is indicated as it will assist diagnosis, e.g. PE, AMI, CCF, underfilling

The commonest congenital cardiac defect is A. coarctation of the aorta B. patent ductus arteriosus C. pulmonic stenosis D. transposition of the great vessels E. ventricular septal defect

ANSWER E VSD 32% PDA 12% ASD 6% TGA 5%

Drug LEAST likely to cause hypoxia in ARDS a. Noradrenaline b. Milrinone c. Isoprenaline d. Isoflurane e. SNP

Answer: A Wiki: "All the others would bugger up your HPV."

AZ61 ANZCA version [2001-Apr] Q116 (Similar question reported in [Jul00]) Methods to potentiate electroconvulsive therapeutic seizures include 1. hyperoxygenation 2. administration of adenosine antagonists 3. ensuring good electrode scalp coupling 4. unilateral electrode placement

1, 2, 3 true, 4 false Increase seizure duration: *Hyperoxia *hypocapnia *ketamine *alfentanil/remi (by reducing anaes agent dose) *caffeine *ensure good coupling Decrease seizure duration: Opposite of above + *methohexital/thio/diazepam/propofol/esmolol

AZ59 ANZCA version [2001-Apr] Q111 (Similar question reported in [Mar00]) The lower oesophageal sphincter 1. has both longitudinal and circumferential fibres 2. is supplied by both sympathetic and parasympathetic nerves 3. exerts a pressure in excess of 30 cmH2O 4. does not usually extend above the diaphragm in normal individuals

1, 2, 3 true, 4 false,

PL04 Adrenaline is an advantage in increasing the duration of action of anaesthesia when used with: A. Bupivacaine B. Etidocaine C. Prilocaine D. Lignocaine E. Amethocaine

ANSWER ?? From Miller's, 6th edition, pp. 586,7: Epinephrine will prolong the duration of infiltration anesthesia by all local anesthetics, although this effect is most pronounced when epinephrine is added to lidocaine

The American Heart Association guidelines for pre-operative cardiac risk assessment define poor functional capacity as being only able to exercise at a level of less than 4 METS. An exercise capacity of 4 METS would correspond to: A. Light housework such as dishwashing B. heavy work around the house such as moving heavy objects C. jogging for 2 kms D. walking briskly on level ground (6kph) E. walking slowly on level ground (3kph)

ANSWER A

The MAC (minimum alveolar concentration) and blood:gas partition coefficient of Xenon are A. 71% and 0.14 respectively B. 45% and 0.24 respectively C. 101% and 0.24 respectively D. 45% and 0.42 respectively E. 71% and 0.47 respectively

ANSWER A

Timing of worst coagulopathy after liver transplant a. 1-2 days b. 3-4 days c. 5-6 days etc

ANSWER A

Patient with severe Rheumatoid arthritis. Has C1/C2 instability. Most likely C-spine Xr finding would be A. Anterior Atlantoodental interval >9 B. Increased sagittal diameter C. Posterior atlantodental interval >14 D. Midpart of C1 over C2 E. Tear drop sign of C2

ANSWER A A 9mm gap between the anterior arch of C1 and the odontoid peg OHA: Anterior AAS 80% C1 forward on C2 due to destruction of transverse ligament.. Sign with >3mm gap between peg and arch of C2. Worse with flexion on flexion/extension lateral neck XR.

Most common cause of mortality post transfusion? A. TRALI B. Contamination/infection C. Mismatched blood D. GvHD E. Anaphylaxis

ANSWER A CEACCP 2006

What is the half life of clopidogrel? A. 6 hours B. 14 hours C. 24 hours D. 7 days E. 10 days

ANSWER A Stupid question: Stoelting elimination t1/2 =7hours. Recovery of pl function by 7 days after drug discontinuation. Br J Clin Pharmacol. 2001 September; 52(3): 333-336

37. A pregnant lady is undergoing neuroradiological coiling of a cerebral aneurysm. At what gestational age should you monitor foetal heart rate to ensure adequate uteroplacental blood flow A. 20 weeks B. 24 weeks C. 28 weeks D. 30 weeks E. 32 weeks

ANSWER B

AM17a [Sep90] Dystrophia myotonia associated with:- A. High arched palate B. Risk of aspiration C. Thyroid disease D. Optic atrophy E. None of the above

ANSWER B

An 18 yo with Fontan circulation undergoing exploratory laparotomy. On ICU ventilation, saturation is 70%. Which ventilator parameter would you INCREASE to improve his saturation? A. Bilevel pressure B. Expiratory time C. Inspiratory time D. Peak inspiratory pressure E. PEEP

ANSWER B

AM47 ANZCA version [2004-Aug] Q139, [2005-Apr] Q31 In patient with with myasthenia gravis presenting for abdominal hysterectomy A. anticholinesterase therapy will have an effect on her plasma cholinesterase B. epidural analgesia is contraindicated C. preoperative sedation should be used D. there is an increased sensitivity to suxamethonium E. volatile agents should not be used

ANSWER A

Clinically the most significant murmur in pregnancy is? A. MS B. AS C. MR D. AR E. VSD

ANSWER A

POISE trial showed A. Increase CVA B. Anaphylaxis C. renal failure D. Increased AMI

ANSWER A

The nerve supplying area of skin between greater trochanter and iliac crest: A. subcostal nerve B. ilioinguinal nerve C. genitofemoral nerve D. femoral nerve E. lat cutaneous femoral nerve

ANSWER A

Time to reach peak plasma concentration after injection of 2% lignocaine with adrenaline into epidural space A. 20 min B. 30 min C. 40 min D. 50 min E. 60 min

ANSWER A

You see a patient in your clinic for a total knee replacement. He is 65 and has atrial fibrillation for which he takes dabigatran. He is otherwise well. A spinal anaesthetic is planned. What is the correct advice regarding his medication? a. he should stop his dabigatran 7 days prior b. he should stop his dabigatran 3 days prior c. he should stop his dabigatran 3 days prior and have bridging enoxaparan d. he should stop his dabigatran the day before and have an INR on the day of surgery e. he should continue to take his dabigatran until the morning of surgery

ANSWER A

Which of the following alternatives is correct regarding the range of values that odds ratios (OR) and relative risks (RR) can take? A. OR (0 to positive infinity); RR (0 to positive infinity) B. OR (negative infinity to positive infinity); RR (negative infinity to positive infinity) C. OR (0 to 1); RR (0 to 1) D. OR (0 to positive infinity); RR (negative infinity to positive infinity) E. OR (0 to positive infinity); RR (negative 1 to positive 1)

ANSWER A * Both relative risk and odds ratio are used to measure effect size * Both have a range of 0 to positive infinity with value of 1 meaning both groups have same probability * Relative risk = pexposed / pcontrol o Used in randomised trials o More intuitive * Odds ratio = p1(1-p2) / p2(1-p1) o Used in case-control studies and retrospective studies Easy to remember - no of times something occurs can never be negative! ie can't have negative MI's in a trial! By the by, odds ratio is an inferior tool cf risk ratios. They are used when the denominator is not known. This means that the experimenter has looked for two groups (case-control) to compare, rather than sampling a COHORT of the population. Risk ratios should always be used where possible; odds ratios overestimate the "risk" ie OR of 3 is actually less than three times more likely to happen.

The administration of blood contaminated with Yersinia Enterocolitica will typically produce symptoms of infection A. during the transfusion B. within 6 hours C. within 12 hours D. within 24 hours E. within a week

ANSWER A 'Yersinia enterocolitica' is the most common organism found in RBCs, whereas Staphylococcus aureus, Klebsiella pneumoniae, Serratia marcescens, and Staphylococcus epidermidis are most common in platelets. Bacterial infection is probably underdiagnosed, and the possibility of bacterial infection (and antibiotic coverage) should be considered in any patient in whom fever develops within 6 hours of transfusion. The onset of clinical symptoms typically occurs acutely during transfusion, with a mortality rate of 60% and a median time to death of only 25 hrs

Long stem about an old #NOF patient with aortic stenosis. What is a sign/ investigation/ symptom that shows the most severity? (ie Which one of these would indicate that the lesion was severe?) A: Thrill in Aortic area B: Murmur in lower left sternal edge C: Murmur radiating to carotids D: History of ischaemic heart disease or coronary artery disease E: history of angina/ syncope

ANSWER A A - True - "A precordial thrill may be felt, especially on leaning forward in expiration. Its presence is reasonably specific for severe aortic stenosis"

In a normal pregnant woman laboratory tests would show: A. an arterial pH of 7.4 B. an increase in functional residual capacity (FRC) C. decreased oxygen consumption D. an arterial base excess of +5mmol.l-1 E. a PaCO2 of 50 mmHg

ANSWER A A : TRUE - respiratory acidosis (progesterone/estrogen) is balanced by compensatory excretion of bicarbonate (metabolic acidosis) resulting in normal pH B: FALSE - reduced C : FALSE - O2 consumption is proportional to CO, which increases 20% at term, 40-50% during labor D : FALSE - +3mmol/L E : FALSE - PCO2 30mmHg%%pH 7.41-7.46 (N) pO2 105 (inc) pCO2 27-32 (dec) HCO2 19-24 (dec)

Characteristic cardio-pulmonary effects of pulmonary thrombo-embolism include A. hypoxaemia due to excess perfusion of lung units with a low V/Q ratio B. hypercarbia due to an increase in physiological dead-space C. reverse splitting of the second heart sound D. an increase in compliance of the left ventricle E. an increase in coronary blood flow to the right ventricle during systole

ANSWER A A. TRUE : Impaired gas exchange due to PE cannot be explained solely on the basis of mechanical obstruction of the vascular bed and alterations in the ventilation to perfusion ratio. Gas exchange abnormalities are also related to the release of inflammatory mediators, resulting in surfactant dysfunction, atelectasis, and functional intrapulmonary shunting B. FALSE : Arterial blood gases usually reveal acute respiratory alkalosis due to hyperventilation. The arterial PO2 and the alveolar-arterial oxygen difference (A-a-DO2) are usually abnormal in patients with PE compared with healthy, age-matched controls. However, arterial blood gases are not diagnostic: among patients who were evaluated in the PIOPED I study, neither the PO2 nor the A-a-DO2 differentiated between those with and those without pulmonary emboli. Profound hypoxia with a normal chest radiograph in the absence of preexisting lung disease is highly suspicious for PE. C. FALSE : P2 before A2 in delayed LV ejection eg severe AS or coarctation or ↑ LV load eg large PDA. P2 is LOUD in pulm hypertension and is not typically described as being earlier (enough to reverse the splitting). D. FALSE : ↑ PVR → ↑ RV size → deviates septum → ↓ LV compliance (TeOh p281) E. FALSE : TeOH p351 - acute pulmonary hypertension increases right ventricular afterload and wall tension, with coronary ischaemia PE physiology (Nunn p551) * ↑ PVR 1. Physical occlusion 2. Platelet activation in thrombus → 5HT and TXA2 release → vasoconstriction → ↑ PVR * Respiratory lesion o ↑ alveolar deadspace o ↑ A-a gradient + Normal to low PaCO2 (in SV) because ↑ RR ? due to J-receptor stimulation + hypoxia * ↓ PaO2 o Deranged V/Q relationships o ↓ CO → low mixed venous O2 * Bronchospasm due to 5HT release from platelets * ↓ pulmonary compliance (still unknown mechanism)

34. (NEW) Best way to assess fluid resuscitation after burns is a. Urine output b. mixed venous sats c. blood pressure d. cvp e. capillary refill

ANSWER A Blue Book 2005 - "End points of resuscitation The optimal end points for burns resuscitation continue to generate much debate. Despite the administration of fluid therapy according to prescribed guidelines, problems frequently noted at the end of the burn resuscitation are generalized oedema, decreased efficiency of pulmonary gas exchange, hypoalbuminaemia and intermittent episodes of hypotension and oliguria. Some problems may indicate over resuscitation, whereas others are suggestive of ongoing hypovolaemia. Clinical examination, together with assessment of end organ perfusion (urine output 1⁄2 to 1 ml/kg/hr; intact sensorium), is the minimum assessment possible to guide burn resuscitation. Pulmonary artery catheters and other more invasive forms of monitoring of haemodynamic parameters have not been shown to improve outcome in surgical, medical or burns patients. Both subcutaneous and splanchnic oxygenation are sensitive indicators of evolving haemorrhagic shock, and have been used in burn care to monitor tissue oxygenation indices during burn shock and resuscitation.15 Recently, Rivers et al investigated the use of central venous oxygen saturation (ScvO2) as part of a package to guide therapy for severe sepsis, and showed an improvement in outcome when it was used in a single centre.16 However, the role of ScvO2 to guide resuscitation of burn shock is not established. A single centre Australasian study is planned.

von Willebrands type 1: A. Variable increase in bleeding time B. Increased aPTT C. Increased PT D. Haemarthroses E. Delayed bleeding after trauma

ANSWER A PT normal APTT normal (25% riased) BT raised

At an altitude of 14,000 feet (4,200m), ambient pressure is 450mmHg. Breathing air, a normal man has an alveolar pO2 of: A. 40 mmHg B. 50 mmHg C. 55 mmHg D. 60 mmHg E. 80 mmHg

ANSWER A PiO2 = FiO2 × (Pb - 47) PiO2 = 0.21 × (450-47) = 84 PAO2 = PiO2 - (PaCO2 / R) PAO2 = 84 - (35 / 0.8) = 40mmHg (Assuming PaCO2 lower because of hyperventilation)

A woman is being treated for pre-eclampsi. She is given 10 grams Magnesium sulphate in 1 h, instead of 1 gr per 1 h, Mg level 5-6, and patient is hyporeflexia. The best treatment is: a. calcium b. IV fluid c. Furosemide d. ?

ANSWER A Was there and option of waiting to repeat the level a few hours later?

Proprioception is involved in lesions of: A. Posterior columns B. Tabes dorsalis C. Cerebellum D. Anterior horn cells E. Spinothalamic tract

ANSWER A B C Position sensation travels in the DORSAL COLUMN-MEDIAL LEMNISCUS system Ganong 9th pg 597 Tabes dorsalis, a demyelination of DORSAL COLUMNS causes alterations in proprioception. - Neurological Skills Harrison. Sensory tracts to cerebellum include DORSAL and ventral cerebellar tracts including signals regarding joint position, Guyton 9th edition pg 717.

AM36 ANZCA version [2001-Aug] Q134 Therapies that have been reported to be useful in the management of Neuroleptic Malignant Syndrome include 1. dantrolene 2. bromocriptine 3. levodopa 4. amantadine

ANSWER ALL

Acromegaly due to excess of growth hormone. Why is it difficult to do a direct laryngoscopy? A: Distorted facial anatomy B: Macroglossia C: Glottic stenosis D: Prognathe mandible E: Arthritis of the neck

ANSWER B

CT reprint showing large MNG. Uppermost concerns to anaesthetist is a. Involvement of the Right carotid artery b. Tracheal deviation to the left c. Tracheal deviation to the right d. Malignant involvement of the paratracheal nodes e. compression of upper lobe of rt.lung

ANSWER B

Clinical features supporting the diagnosis of cardiac tamponade include all of the following EXCEPT A. equal diastolic pressures of all heart chambers B. ST segment abnormalities C. increased venous pressure and cardiac output with fluid loading D. increased patient comfort in the sitting position E. impalpable apex beat and soft heart sounds

ANSWER B

Common complications of spinal cord injury include each of the following EXCEPT A. deep vein thrombosis B. diarrhoea C. heterotopic calcification D. pressure sores E. urinary tract infection

ANSWER B

Complications of mediastinoscopy include all of the following EXCEPT A. air embolism B. cardiac laceration C. pneumothorax D. recurrent laryngeal nerve palsy E. tracheal compression

ANSWER B

Doing an awake CEA. Patient becomes confused & combative after carotid clamped and opened. Priority is... a) tell surgeon to release clamp b) tell surgeon to place shunt c) induce GA d) give midazolam

ANSWER B

Immunologically mediated heparin-induced thrombocytopaenia is characterised by A. onset within a few days of first starting heparin therapy B. intravascular thromboses C. platelet count rarely reduced below 100 x 109.1-I D. continuation of thrombocytopaenia after cessation of heparin E. presence of non-specific (heparin-independent) platelet antibodies

ANSWER B

Post delivery neonate did not breath post stimulation by midwife, not vigorous, heart rate drop from 140 to 90bpm. Next step of action A. 100% oxygen B. Positive pressure ventilation C. Intubation D. CPR E. Adrenaline

ANSWER B

Post intubation, you manual ventilate and noted patient high airway pressure. What would you do next A. Open the APL valve B. Auscultate the lung C. Switch to ventilator D. Administer magnesium E. Administer ketamine

ANSWER B

Prolonged PT can be seen in all except, A. Vitamin K deficiency B. Critical aortic stenosis C. Liver disease D. Coumarin therapy E. DIC

ANSWER B

Pulsus paradoxus is: A. Reduced BP on inspiration unlike normal (ie normally increased on insp) B. Reduced BP on inspiration exaggerated from normal C. Reduced BP on expiration unlike normal D. Reduced BP on expiration exaggerated from normal E. ?

ANSWER B

RH26b ANZCA version [2004-Apr] Q126, [Jul07] To achieve maximum anaesthesia with minimal risk of trauma to veins, the tip of a needle used for a medial peribulbar injection should be advanced no further past the equator of the globe than A. 5 mm B. 10 mm C. 15 mm D. 20 mm E. 25 mm

ANSWER B

Regarding the normal term infant A. foetal haemoglobin (HbF) comprises approximately one-third the total haemoglobin at birth and falls to negligible levels by 3 months of age B. foetal haemoglobin (HbF) comprises approximately 70% of the total haemoglobin at birth and falls to negligible levels by 6 months of age C. haemoglobin level below 90g.l-1 at 9-12 weeks (physiological anaemia) is common and does NOT require investigation D. normal haemoglobin at birth should be greater than 200g.l-1 unless there has been delay in umbilical cord clamping E. total blood volume is approximately 70 ml.kg-1 body weight

ANSWER B

Respiratory function in quadriplegics is improved by A. abdominal distension B. an increase in chest wall spasticity C. interscalene nerve block D. the upright position E. unilateral compliance reduction

ANSWER B

SF29d ANZCA version [2004-Aug] Q124, [2005-Apr] Q37 The target serum magnesium level in a patient with pre-eclampsia receiving a magnesium infusion is A. 1-2 mmol/l B. 2-3.5 mmol/l C. 3-6 mmol/l D. 4-6 mmol/l E. 5-8 mmol/l

ANSWER B

The anterior branch of the femoral nerve supplies everything but: A: pectinius B: rectus femoris C: Medial thigh D: anterior thigh E: sartorius

ANSWER B

What is NOT useful in the treatment of Torsades? A. Isoprenaline B. Procainamide C. DCCV D. Electrical pacing (Amiodarone was not an option)

ANSWER B

What is the best indicator of pending respiratory depression when using a morphine PCA A. Respiratory rate B. Sedation score C. Constricted pupils D. E. Reduced saturations

ANSWER B

What is the most common way to measure end tidal gas concentrations on our anaesthetic machines? A: mass spectometry B: Raman scattering C: ultrasonic D: infrared E: piezoelectric

ANSWER B

When ketamine is used for management of acute post-operative pain A. analgesia is usually accompanied by hallucinations B. a starting dose of 0.05 to 0.1 mg.kg-1.hour-1 is appropriate C. benzodiazepines are ineffective in decreasing dysphoric reactions D. morphine is contraindicated as respiratory depressant effects are additive E. the intravenous route is recommended because absorption is more reliable than via the subcutaneous route

ANSWER B

Which is NOT a disadvantage of drawover vaporiser versus plenum vaporiser: A. Temperature compensation B. Cannot use sevoflurane C. Small volume reservoir D. Flow compensation E. ?

ANSWER B

Which is NOT a disadvantage of drawover vaporiser versus plenum vaporiser? A. Temperature compensation B. Cannot use sevoflurane C. Small volume reservoir D. Flow compensation E.

ANSWER B

PN47 ANZCA version [2006-Mar] Q129, [Jul06] Q48 Correct statements concerning naloxone include each of the following EXCEPT A. appropriate titration of naloxone will allow reversal of opioid induced respiratory depression B. naloxone is a partial agonist C. naloxone is most effective at blocking mu receptors D. serious side effects such as arrhythmias and pulmonary oedema are rare E. the elimination half-life of naloxone is approximately 60 minutes

ANSWER B * A True, respiratory depression is mu effect * B False (so correct answer) naloxone is competitive antagonist * C True but also some action at delta and kappa * D True rare, but reported * E True 1.2 hours

PZ86c ANZCA version [2005-Apr] Q109 Correct statements regarding complementary and alternative medications include A. ginger is a superior antiemetic to metoclopramide B. ginseng and ephedra may produce cardiovascular complications during general anaesthesia C. ginseng and ephedra may potentiate the analgesic properties, but not the bleeding side-effects, of the non-steroidal anti-inflammatory drugs (NSAIDS) D. it is generally recommended that patients can continue these medications perioperatively E. St. John's Wort is associated with acute tubular necrosis perioperatively

ANSWER B * A. ginger is a superior antiemetic to metoclopramide - false: o "Ernst and Pittler performed a systematic review of the evidence from six double-blind, randomised, placebocontrolled studies of the efficacy of ginger in preventing nausea and vomiting [87]. Of the three studies on PONV, two studies suggested that ginger is equally effective as metoclopramide, and is superior to placebo. However, the pooled absolute risk decrease for the incidence of PONV indicated that there was no significant difference between ginger and the placebo groups. A recent doubleblind, randomised, placebo-controlled trial, which was not included in Ernst's systematic review, concluded that ginger was not effective in decreasing the incidence of PONV after day-case gynaecological laparoscopy" (Anaesthesia, 2002, 57, pages 889-899) * B. ginseng and ephedra may produce cardiovascular complications during general anaesthesia - true: o "Furthermore, because of ginseng's association with hypertension and the deleterious outcomes linked to chronic hypertension, the anesthesiologist should be aware of whether and for how long patients may have been taking this herbal product. Many anesthetic agents can cause generalized vasodilation, and so hemodynamic lability may be observed." (Current Opinion in Anaesthesiology 2007, 20:294-299) o "Ephedra contains alkaloids such as ephedrine, pseudoephedrine, methylephedrine and norpseudo-ephedrine, obtained from the roots and branches of a shrub native to central Asia [79]. Ephedrine is the predominant active component and is a non-catecholamine sympathomimetic agent that acts directly and indirectly at alpha- and betaadrenergic receptors, causing increased blood pressure and heart rate, relaxation of bronchial and gastrointestinal smooth muscle, central nervous system stimulation and mydriasis." (Anaesthesia, 2002, 57, pages 889-899) * C. ginseng and ephedra may potentiate the analgesic properties, but not the bleeding side-effects, of the non-steroidal anti-inflammatory drugs (NSAIDS) - false * D. it is generally recommended that patients can continue these medications perioperatively - false: o "Currently, the American Society of Anaesthesiologists recommends that patients cease herbal medicines at least 2 weeks before surgery" (Anaesthesia, 2002, 57, pages 889-899) * E. St. John's Wort is associated with acute tubular necrosis perioperatively - false

The commonest cause of maternal convulsions in the immediate post-partum period is A. amniotic fluid embolism B. eclampsia C. epilepsy D. local anaesthetic toxicity E. water intoxication due to syntocinon infusion

ANSWER B "The most common cause of postpartum seizures is eclampsia." p.208 Handbook of Obstetric and Gynecologic Emergencies,3e Benrubi 2005, Differential diagnoses for postpartum seizure following epidural analgesia: * Eclampsia * Epilepsy * Drug/alcohol withdrawl * Meningoencephalitis * Subarachnoid haemorrhage * Space occupying lesion * Cortical vein thrombosis * Metabolic disturbances * Migraine * Idiopathic * Post dural puncture headache * Pneumocephalus

Which does disease process does not results in total body depletion of sodium occurs in: A. Addison's Disease B. nephrotic syndrome C. Diabetic ketoacidosis D. Chronic diuretic use/ frusemide

ANSWER B Although nephrotic syndrome causes of hyponatraemia -they cause hyponatraemia due to water excess with smaller sodium excess, ie total body sodium is not depleted.

AZ39 [Aug93] [Mar94] The major anabolic hormone in response to surgery is: A. Thyroxine B. Human growth hormone C. Cortisol D. Insulin E. Glucagon

ANSWER B Catecholamines, corticosteroids, GH, glucagon all increased. GH only anabolic hormone. TSH unchanged. Thyroxine may be increased by symathetic stimulation, but catabolic effects.

Lap chole on citalopram. What is NOT relatively contraindicated? A. Omeprazole B. Clonidine C. Pethidine D. Tramadol E. ?Midazolam

ANSWER B Citalopram shares CYP3A4 metabolism with midazolam Citalopram shares CYPC19 metabolism with omeprazole Tramadol and pethidine with citalopram may be relatively contraindicated due to serotonin syndrome concerns

SF35 [Mar93] [Apr96] Recognised effects of aspirin 100mg taken daily until term by patients with at risk of developing pregnancy-induced hypertension include: A. Increased platelet aggregation B. Decreased thromboxane A2 synthesis C. Decreased prostacyclin synthesis D. Increased blood loss at delivery E. Premature closure of the foetal ductus arteriosus

ANSWER B Compared to women with normal pregnancies, women with preeclampsia have a relative excess of thromboxane A2 (a platelet-derived vasoconstrictor and platelet aggregation promoter) compared to prostacyclin (an endothelial cell-derived vasodilator and platelet aggregation inhibitor).9-11 The correction of the thromboxane A2 to prostacyclin ratio caused by aspirin may help prevent the development of preeclampsia and its complications.

A 35-year-old woman with a history of palpitations has the following ECG at rest: * 1st ECG pre-op with possible delta wave and big R wave in V1 She presents for laparoscopic appendicectomy for suspected acute appendicitis. Prior to induction she feels faint and you feel a very rapid irregular brachial pulse. A portion of her 12 lead ECG now is shown below * 2nd ECG 12 lead at time of event HR>240 bpm ?irreg AF/SVT/VT wide-complex VT Your treatment of this arrhythmia should be A. carotid sinus massage B. DC cardioversion C. IV lignocaine D. IV verapamil E. IV adenosine

ANSWER B Haemodynamically unstable : SHOCK * orthodromic AVRT (narrow QRS)- want to block conduction thru AV node. -Try vagal manouvre -then adenosine -then verapamil * antidromic AVRT (wide QRS) - want to block conduction thru accessory pathway. -procainamide 10mg/kg -blocking AV conduction (ie vagal/adenosine) not effective -verapamil contraindicated as won't stop the AVRT but has v/d effect so may worsen haemodynamics. * AF - again want to block conduction via accessory pathway. -procainamide -verapamil &digoxin contraindicated as they block AV node and may cause preferential conduction down accessory pathway, causing faster ventricular rate

Labour epidurals increase maternal and foetal temperature. This results in neonatal A. Increased sepsis B. Increased investigations for sepsis C. increased non shivering thermogenesis D. Increased need for resuscitation E. Cerebral palsy

ANSWER B Labour epidural analgesia is associated with an increase in maternal core body temperature, but also with an increased neonatal temperature and fetal heart rate. Several studies have shown that labour epidural analgesia is associated with increased neonatal neonatal sepsis evaluations, but no increase in neonatal sepsis.

Half-life of mast cell tryptase? A. 1 hour B. 2 hours C. 24 hours D. 36 hours E. 72 hours

ANSWER B NICE Guidelines Anaphylaxis "Very low-quality evidence from six observational studies including 147 patients showed that the half-life of tryptase ranged from 30 minutes to 300 minutes (median 90 minutes)" Mayo Medical Laboratories After anaphylaxis, mast cell granules release tryptase; measurable amounts are found in blood, generally within 30 to 60 minutes. The levels decline under first-order kinetics with half-life of approximately 2 hours.

A size C oxygen cylinder that reads 5000kpa contains approximately how many litres of oxygen A. 100 B. 150 C. 200 D. 350 E. 600

ANSWER B x/5000 = 490/16300 = 150 http://www.boc-healthcare.com.au/internet.lh.lh.aus/en/images/HCD130_Cylinder%20identification%20chart350_72667.pdf

Mechanism of action of antiepileptics in chronic pain, which is false? a. Phenytoin workes at Na channels b. Gabapentin increases gaba in cns c. Carbamazepine works at Na channels d. Valproate increases GABA in the CNS e. lamotrogine acts at Ca channel

ANSWER B and E

A patient had an abdomino-perineal resection for carcinoma of the rectum. The postoperative course was uneventful for 5 days. On the 6th day, the abdomen became distended, the patient began vomiting and bowel function ceased. On examination, the patient's temperature was 36C, the BP 90/50 with tachycardia. He was in pain. Diagnosis is: A. Secondary haemorrhage B. Septicaemia C. Large bowel obstruction D. Small bowel obstruction E. Any of the above

ANSWER B,C,D

2 yo 15kg child following seizure on surgical ward. Admitted with appendicitis and perforation. 60ml/hr of ½ N. Saline 5%dextrose Na+ 119 K+ 4.5 HCO3- 19 Cl- 90 Best treatment would be A. Desmopressin B. Frusemide C. 3% normal saline D. Normal Saline E. Fluid restrict

ANSWER C

A 20kg child suffered 15% full thickness burns 6 hours ago. Optimum crystalloid resuscitation for the first hour is A. 160ml B. 260ml C. 360ml D. 460ml E. 660ml

ANSWER C

A patient is having surgery on the distal phalanx of his fifth digit. Which of the following regional techniques is the LEAST appropriate? A. Infraclavicular B. Digital C. Interscalene D. Axillary

ANSWER C

A pediatric patient is having a laparotomy for resection of a neuroblastoma. What is the best way to avoid hypothermia? A. Set the room temperature at 29 degrees Celsius B. Warm IV fluids C. Forced air warmer D. Humidified inhaled air

ANSWER C

A woman has an epidural placed for forceps delivery after a prolonged second stage of labour. The next day she has a right foot drop, and numbness over the anterior part of her lower leg and the dorsal surface of her right foot. The most likely cause is A. right common peroneal nerve lesion from the use of stirrups in the lithotomy position B. right L5 nerve root lesion from the epidural placement C. right lumbar plexus lesion from compression by the fetal head D. L5 nerve root lesion from a disc protrusion E. transient neurological symptoms (TNS) syndrome

ANSWER C

An 85y.o for open AAA repair. Refuses blood because of risk of vCJD. You tell him you won't anaesthetise him as the risk is too high. This is an example of: A: Autonomy B: Beneficence C: Malevolence D: Coercion E: Paternalism

ANSWER C

An infant is anaesthetised and ventilated using an endotracheal tube and circle breathing system with CO2 absorber. The item which causes the most resistance to breathing is the A. airway pressure limiting (APL) valve B. circuit hosing C. endotracheal tube D. heat and moisture exchange filter E. inspiratory and expiratory valves

ANSWER C

Baby with Tracheo-oesophageal fistula found by bubbling saliva and nasogastric tube coiling on Xray. BEST immediate management? A. Bag and mask ventilate B. Intubate and ventilate C. position head up, insert suction catheter in oesophagus D. Place prone, head down to allow contents to drain E. Insert gastrostomy

ANSWER C

Carcinoid syndrome - finding on examining heart: A. Fine inspiratory crepitations B. Systolic murmur at apex C. Systolic murmur at left sternal edge D. Murmur at apex with opening snap E. Pericardial rub

ANSWER C

Characteristics of remifentanil include all of the following EXCEPT A. a weakly active metabolite B. high potency C. metabolism by pseudocholinesterase D. muscle rigidity following rapid bolusing E. very short context-sensitive half-life

ANSWER C

Desflurane A. has a boiling point of 29.2º C at 1 atmosphere B. has a blood:gas partition coefficient higher than isoflurane C. has been reported as a triggering agent for malignant hyperthermia D. is associated with a low incidence of reflex airway responses when used for gaseous induction E. increases cerebral blood flow at 1 MAC

ANSWER C

Finding on haemophilia A patient A. Female haemarthrosis B. Male haemarthrosis C. Normal PT, abnormal APTT D. Abnormal PT, normal APTT

ANSWER C

Immunological evidence of immunity to hepatitis B is the presence of A. hepatitis B core antibodies B. hepatitis B core antigen C. hepatitis B surface antibodies D. hepatitis B surface antigen E. any of the above

ANSWER C

Loading dose of IV paracetamol in x kg child? A. (homeopathic) B. (closer) C. 15mg/kg D. 20mg/kg E. 30mg/kg

ANSWER C

MR27 Compliance is increased in: A. Pneumonia B. CCF C. Emphysema D. Obesity

ANSWER C

Middle-aged male with severe MS having general anaesthesia for repair of fractured ulna / radius. 10 minutes into the case you notice a tachyarrythmia with his HR 130 and BP 70. He is normally in sinus. What do you do? A. Adenosine B.Amiodarone C. Shock D. Volume E. Metaraminol

ANSWER C

Patients with sickle-cell trait A. are usually anaemic B. never sickle C. sickle at very low haemoglobin saturations D. sickle with hypercarbia E. sickle with hyperthermia

ANSWER C

Regarding the anatomy of the larynx A. the cricothyroid muscles supplied by the recurrent laryngeal nerve B. the superior laryngeal artery accompanies the recurrent laryngeal nerve C. the vagus nerve supplies motor fibres to all the intrinsic muscles of the larynx D. the posterior cricoarytenoid muscles are adductors of the cords E. the cricothyroid muscle tenses the cords with the cricoarytenoid muscles

ANSWER C

Remifentanil is A. metabolised by non-specific esterases in the liver B. metabolised by plasma cholinesterase C. rapidly metabolised with a context sensitive half life of around 3 minutes regardless of infusion duration D. rapidly metabolised with a context sensitive half life of around 3 minutes after a one hour infusion and around 7 minutes after a 6 hour infusion E. metabolised by non-specific esterases in the kidney and liver

ANSWER C

TMP-Jul10-018 The STRONGEST stimulus for ADH secretion: A. High serum osmolality B. Low serum osmolality C. Hypovolaemia D. High serum Na E.

ANSWER C

TMP-Jul10-045 How quickly does the CO2 rise in the apnoeic patient ? A. 1 mmHg per min B. 2 mmHg per min C. 3 mmHg per min D. 4 mmHg per min E. 5 or ?8 mmHg per min

ANSWER C

The QT interval may be prolonged by each of the following EXCEPT A. high intra-thoracic pressure B. hypothermia C. magnesium sulphate D. suxamethonium E. volatile anaesthetic agents

ANSWER C

Hyperparathyroidism due to chronic renal failure is contributed to by all of the following EXCEPT: A. Hypocalcaemia B. Decreased renal calcium absorption due to acidosis C. increased renal calcium absorption due to increased 1,25 D3 D. Increased serum phosphate E. Decreased levels of 1,25 dihydroxycholecalciferol

ANSWER C 1-25 Dihydroxycholecalciferol acts on the GI tract to cause increased gastric absorption of Calcium. The kidney is involved as part of the formation of 1-25 dihydroxycholecalciferol, but it does not in turn act on the kidney. Secondary Hyperparathyroidism -excessive secretion of PTH in response to hypocalcaemia -associated with hypertrophy of PT glands -especially seen in CRF Presents -excessive bone resorption (seen earliest in the radial aspect of the middle phalanx of the second digit) -soft tissue calcification kidneys, heart, lung and skin Treatment (medical) -dietary phosphate restriction -Ca -Vit D -if this failes them long-term dialysis

An inpatient becomes hyponatraemic 48 hours post op and has a seizure. The most appropriate treatment is A. Fluid restriction B. Normal saline ?ml/hr C. Hypertonic saline D. Salt tablets

ANSWER C 3% N/S 1ml/kg raise Na by 1mmol/l. Plan to increase Na to >125mmo/l

With regard to accidental electrocution, which of the following statements is true? A. all electrical equipment in the operating room should be earthed B. risk of ventricular fibrillation increases with increasing current frequency C. risk of ventricular fibrillation is greater with alternating current (cf. direct) D. risk of electrocution is reduced by earthing the patient E. use of battery operated equipment avoids the risk of ventricular fibrillation

ANSWER C A equipment supplied by an isolated circuit is not earthed B diathermy does not cause electrocution because of its very high frequency C AC is more arrhythmogenic than DC - so this is the answer D Should not earth them E probably not if you connect the equipment up to their pacing wires

Each of the following is effective in the treatment of pain from acute herpes zoster EXCEPT A. acyclovir B. amitriptyline C. carbamazepine D. corticosteroids E. topical aspirin

ANSWER C A. FALSE : Level I evidence to reduce clinical course and acute pain, but no risk reduction in developing trigeminal neurolgia B. FALSE : early use has been shown to reduce risk by half (Level II) C. TRUE : no role D. FALSE : level II evidence, reduction in acute pain within 72 hours E. FALSE : topical aspirin and lignocaine should to reduced acute pain (Level II)

The commonest initial presenting feature in anaphylaxis is A. coughing B. desaturation C. hypotension D. rash E. wheeze

ANSWER C A. coughing - false B. desaturation - false C. hypotension - true: See Table 1, CEACCP 2004; 4:4(111) D. rash - false E. wheeze - maybe

The most appropriate method for improving oxygenation during one lung anaesthesia, after institution of an FiO2 of 1.0, is application of A. 5 cm H2O CPAP to the non-dependent lung B. 10 cm H2O CPAP to the non-dependent lung C. 5 cm H2O PEEP to the dependent lung D. 5 cm H2O CPAP to the non-dependent and 5cm H2O PEEP to the dependent lung E. intermittent re-inflation to the non-dependent lung

ANSWER C Another point of view Note - In Miller's, chapter on thoracics (by Slinger) the steps are * Fio2 of 1.0 * Check position of DLT (using FOB) * ensure adequate CO, reduce volatile to <1MAC * Apply recruitment maneouvre * Apply 5cm H2O PEEP to dependent lung * Apply 1-2cm H2O CPAP to non-dependent lung after recruitment maneouvre * Intermittent Reinflation * Partial ventilation techniques including oxygen insufflation, HFJV, mechanical obstruction of blood flow to non-dependent lung

What percentage of primiparous women experience a headache in the first week following delivery? a. 3-5% b. 5-15% c. 15-40% d. 40-65% e. 65-85%

ANSWER C CEACCP - Postpartum headache: diagnosis and management 2011 "Postpartum headache is described as a complaint of headache and neck or shoulder pain in the first 6 weeks after delivery.1 It is one of the most common symptoms with up to 39% of parturients experiencing headache in the first postpartum week"

75 year old with non-valvular AF usually on warfarin has their warfarin stopped for one week. What is their daily risk of stroke? A: 1% B: 0.1% C: 0.01% D: 4% E: 10%

ANSWER C CHA2DS2Vasc score

Each of the following are causes of right axis deviation on the ECG (electrocardiogram) EXCEPT A. chronic lung disease B. lateral myocardial infarction C. left anterior hemiblock D. left posterior hemiblock E. right ventricular hypertrophy

ANSWER C Conditions associated with right axis deviation * Right ventricular hypertrophy * Left posterior hemiblock * Lateral myocardial infarction * Acute right heart strain

How much air is the maximum to that should be used to inflate a 5 LMA classic cuff A. 15 B. 20 C. 25 D. 40 E. 45

ANSWER C For Classic LMA Size: Maximum cuff inflation volume (ml) 1 : 4 1.5 : 7 2 : 10 2.5: 14 3 : 20 4 : 30 5 : 40 6 : 50 http://www.lmana.com/pwpcontrol.php?pwpID=6551 See LMA quick reference guide for maximum cuff volumes for ProSeal, Supreme, Fastrack, Flexiable http://www.lmana.com/files/prosealquick-reference-card.pdf?PHPSESSID=962c2c927d364072381a042917272486

Oxygen cannot be measured by: A. fuel cell B. mass spectrometry C. infra-red spectrophotometry D. Paramagnetic analysis E. Raman scattering

ANSWER C Gases that have 2 or more different atoms will absorb infra-red radiation. Infrared spectrophotometry can't be used for O2, He, Xe, Argon

In trauma patients, the main mechanism by which hypothermia exacerbates bleeding is by A. altering blood viscosity B. causing disseminated intravascular coagulation C. inhibiting clotting factors D. potentiating anticoagulants used for DVT (deep venous thrombosis) prophylaxis E. reducing platelet function and number

ANSWER C Hypothermia impairs coagulation by two main mechanisms: * impaired platelet function, and * impaired clotting factor enzyme activity. Note: Fibrinolytic activity is not affected by mild hypothermia, BUT is increased with HYPERthermia. Fibrinolysis doe NOT contribute to increased bleeding with hypothermia.) Hypothermia inhibits function but does not reduce the number.

Please use the Parkland formula to calculate the appropriate fluid requirements for a patient who is 50 kg with a 40% full thickness BSA burn, which occurred 4 hours ago. A. Ringer's Lactate 500cc/hr x 8 hours then 250cc/hr x 16 hours B. Ringer's Lactate 250cc/hr x 4 hours then 125cc/hr x 16 hours C. Ringer's Lactate 1000cc/hr x 4 hours then 250cc/hr x 16 hours D. Ringer's Lactate 250cc/hr x 16 hours then 125cc/hr x 8 hours

ANSWER C Mls of fluid in 1st 24 hours (time taken from time of burn) = % burn x wt in kgs x 4 = 40 x 50 x 4 = 8,000 mls * Half of this is given in the first 8 hours after a burn. As 4 hours has passed, then give this half in the next 4 hours. Thus, 1,000mls/hr for 4 hours * Second half given over next 16 hours. So give (4000/16) = 250mls/hr for next 16 hours. * Note that fluids need to be adjusted based on urine output (aim for 1ml/kg/hour) and frequent clinical assessment.

A placebo should be used in a clinical trial when A. observer bias is possible B. a type one error is probable C. an acceptable standard treatment is not known to exist D. a placebo effect is anticipated E. human patients are used as subjects

ANSWER C Placebo use and risk Article II. 3 of the current Declaration states that "in any medical study, every patient - including those of a control group, if any - should be assured of the best proven diagnostic and therapeutic method. This does not exclude the use of inert placebo in studies where no proven diagnostic or therapeutic method exists. Article 19 of the Proposed Revision states that "when the outcome measures are neither death nor disability, placebo or other no-treatment controls may be justified on the basis of their efficiency".6 Helsinki declaration.

AM28c [Mar06] [Jul06] [Apr07] Q136 In a patient with myasthenia gravis undergoing a laparotomy for large bowel obstruction, the need for post-operative ventilation is significantly increased by a A. High daily doses of pyridostigmine B. Long history of the disease C. Previous exaggerated response to NMDBs D. Previous crisis requiring ventilation E. Bulbar palsy

ANSWER C Preoperative predictors of postoperative need for ventilation: * duration of disease of greater than 6 years * history of coexisting chronic resp disease * dose requirements of pyridostigmine > 750 mg/day less than 48 H prior to surgery * preoperative VC < 2.9L

From the following list, choose the factor that does NOT decrease the measurable response to platelet transfusion: A. Fever B. ABO incompatibility C. Rh incompatibility D. Recipient HLA antibodies E. Splenomegaly

ANSWER C Rh incompatibility does not affect how well a patient responds to a platelet transfusion, because there are no Rh antigens on platelets. This does not mean, however, that Rh incompatibility may not be an issue, as D-negative recipients can, in uncommon situations, develop anti-D when given D-positive platelet transfusions, as a result of a small number of red cells present in the platelet product. There are ABO antigens on platelets, so ABO incompatibility can decrease the response somewhat. All of the other factors have also been shown to decrease a patient's post-platelet transfusion response to at least some extent.

Aneurysm clipping. Best monitor of depth of block during this is: A. TOFR B. TOFC C. DBS D. PTC

ANSWER D

Best indicator of return function of laryngeal muscle A. Sustained head lift 5 sec B. Sustained leg lift 5 sec C. TOF 0.9 D. DBS no fade E. Tetanus 50Hz

ANSWER D

In a rotameter the A. bobbin spins inside a tube that has parallel sides B. flow is laminar at high flow rates C. height of the bobbin is proportional to the pressure drop across the bobbin D. pressure drop across the bobbin is constant at varying flows E. resistance increases with increasing gas flow

ANSWER D

The muscles of the larynx which separate the vocal cords are the A. inter-arytenoids B. lateral crico-arytenoids C. oblique arytenoids D. posterior crico-arytenoids E. thyro-arytenoids

ANSWER D

A 60kg female is given 50 mg of rocuronium, she is unable to be intubated, what dose of sugamadex is required to reverse the rocuronium A. 240 B. 480 C. 800 D. 960 E. 1920

ANSWER D 16mg/kg if large dose and reversal within 3min

Which one of the following is most likely to be associated with a high mixed venous oxygen saturation (SvO2)? A. acute myocardial infarction B. acute pulmonary embolism C. cardiac tamponade D. sepsis E. severe liver disease

ANSWER D Other causes for high mixed venous sat sepsis A-V fistulae cirrhosis left-to-right cardiac shunts cyanide poisoning hypothermia unintentional PA catheter wedging

Features of eaton lambert include all EXCEPT... a) Associated with SCLC b) improvement with exercise c) defect in ACh release from motor end plate d) dry mouth e) Fatigue with exercise

ANSWER E

Hyperosmolar non-ketotic coma is associated with A. a normal coagulation profile B. a normal serum sodium concentration C. a normal urinary potassium concentration D. acidosis E. known diabetic patients

ANSWER E

Hypertension- severe- in pregnancy (or was it PET?) What NOT to use? A: Hydrallazine B: Nifedipine C: Labetalol D: Metoprolol E: SNP

ANSWER E

In acute coronary syndromes with ST elevation on ECG A. aspirin should be administered only after reperfusion strategies have commenced B. patients can wait up to 24 hours from onset of ischaemic symptoms before implementation of a reperfusion strategy C. patients should have elevated cardiac enzymes before proceeding to reperfusion strategies D. patients undergoing reperfusion strategies should have aspirin or clopidogrel but NOT both E. percutaneous coronary intervention is preferable to fibrinolytic therapy

ANSWER E

The absorption of fluid into the circulation during transurethral prostatectomy (TURP) is NOT related to A. prostate size B. height of the irrigation fluid bag C. duration of surgery D. surgical technique E. type of irrigation fluid

ANSWER E

Which ONE of the following do you NOT expect to see in a fit young ASA 1 patient when the abdomen is inflated with CO2? A. Atelectasis B. Increase airway pressure C. VQ mismatch D. Hypertension E. Hypoxia

ANSWER E

You are commencing general anaesthesia for a 2-year-old child to allow biopsy of an anterior mediastinal mass, A pre-operative CT scan demonstrated compression of the lower trachea and the carina by Ihe mass. During inhalational induction, the child desaturates to 70% due to airway compression by the mass. You should A. apply continuous positive airway pressure (CPAP) via facemask B. arrange urgent median sternotomy C. intubate the patient and allow spontaneous ventilation D. intubate the patient and provide positive pressure ventilation E. place the patient in the prone position

ANSWER E

Ehlers-Danlos syndrome. Most important to specifically do all EXCEPT: A. Avoid hyperextension of the neck B. Damage to the teeth C. Avoid joint hypermobility D. Gastro oesophageal reflex E. Strict temperature regulation

ANSWER E The Ehlers-Danlos family of disorders is a group of related conditions that share a common decrease in the tensile strength and integrity of the skin, joints, and other connective tissues. Patients can often perform "amazing, almost unnatural, contortions" and worked in circuses. All forms of Ehlers-Danlos syndrome share the following primary features to varying degrees: * Skin hyperextensibility * Joint hypermobility and excessive dislocations * Tissue fragility * Poor wound healing, leading to wide thin scars ("cigarette paper scars") * Easy bruising

Carbon monoxide production by soda lime degradation of volatile anaesthetic agents, is LEAST likely to occur with A. the first case of the day and the use of isoflurane B. the first case of the day and the use of sevoflurane C. the first case of the day and the use of desflurane D. fresh soda lime and the use of desflurane E. fresh soda lime and the use of sevoflurane

ANSWER E The anesthetic used: for a given minimum alveolar anesthetic concentration (MAC)-multiple, the magnitude of CO production (greatest to least) is Desflurane > enflurane > isoflurane >> halothane = sevoflurane. (Ref: Anesth Analg 1995; 80:1187-93) In summary regarding CO production: * Baralyme > Sodalime; * Dry > fresh * D>E>I>S=H

Epidermolysis bullosa may be associated with A. oesophageal stricture B. anaemia C. amyloidosis D. porphyria E. all of the above

ANSWER E The disease is classified according to three types: * Simplex * Junctional * Dystrophic Incidence Dystrophic EB occurs in about 1 in every 300,000 births (See reference) EB Dystrophica * Pseudosyndactlyly * Constriction of the mouth (microstomia) * Oesophageal strictures * Teeth are often poor * Malnutrition * Anaemia * Electrolyte derangements * Hypoalbuminaemia Diseases associated with EB * Porphyria * Amyloidosis * Multiple myeloma * Diabetes mellitus * Hypercoagulable states

Name the most common complication of thoracic paravertebral block A. Pneumothroax B. IV injection C. Subarachnoid injection D. Epidural injection E. Failure

ANSWER E The overall incidence of complications is 2.6-5%. Failure rate is 6.5-10% Hypotension 4.6% Vacular puncture 3.8% Pleural puncture 1.1% Pneumonthorax 0.5%

An HIV-positive mother gives birth to a baby. What is the percentage chance of the baby being HIV antibody positive? A. 5-10% B. 20-30% C. 50% D. 70-85% E. 95-99%

ANSWER E The question asks about the baby being HIV ANTIBODY positive (NOT HIV positive) Almost all babies will have antibodies to HIV from the mother and will test positive for 6-18months. During pregnancy, the mother's antibodies (part of the immune system that fights germs) are passed on to her baby. So all babies of women with HIV will test positive for HIV antibodies at first. This doesn't mean the baby is infected, though. Babies keep the mother's antibodies until they can make their own, which happens between 6 and 18 months of age. An HIV positive woman can transmit the virus to her baby during pregnancy, labour and delivery, and through breastfeeding. If she takes no preventive drugs and breastfeeds then the chance of her baby becoming infected is around 20-45%. Risk is decreased to 2-3% if mother is compliant with antivirals, viral load low, LUSCS, avoid breast milk.

Following major surgery, there is an increased risk of thrombosis, associated with a decrease in A. fibrinogen B. factor VIII coagulant6 C. factor VIII; Ag (von Willebrand related antigen) D. interleukin 6 E. protein C

ANSWER E There is an increased risk of thrombosis with: * Protein C deficiency * Protein S deficiency * Antithrombin III deficiency * Lupus anticoagulant and/or anticardiolipin antibody * Factor V Leiden mutation * Thrombocytosis * Polycythemia

PN42b ANZCA Version [2006-Mar] Q125, [Jul06] Q69 When instructing ward staff on monitoring for respiratory depression in a patient using PCA (patient controlled analgesia) you would advise that early respiratory depression is best detected by monitoring A. frequency of boluses on PCA machine B. pulse oximetry C. pupil size D. respiratory rate E. sedation scores

ANSWER E These studies confirm that assessment of sedation is a more reliable way of detecting opioid induced respiratory depression, although monitoring respiratory rate is still important"

A patient with a central dislocation of the hip following a motor car accident is noted to be shocked on admission, one hour after the accident. The most likely cause is A. Ruptured bladder B. Fat embolism C. Ruptured urethra D. Neurogenic shock E. None of the above

ANSWER E Trauma haemorrhage 750mls in tibia or humerus 1500mls in femur >2000mls retroperitoneal haematoma in pelvic fractures

Maximum dose (with low risk of toxicity) of lignocaine (with adrenaline 1:100000) for liposuction with tumescence technique: A. 3 mg/kg B. 7 mg/kg C. 15 mg/kg D. 25 mg/kg E. 35 mg/kg

ANSWER E Tumescent liposuction - Klein noted that the correct maximum safe dose of lidocaine was never investigated but rather extrapolated from procaine. He showed that infusion of lidocaine, by using the tumescent formula of 0.1% lidocaine with 1:1,000,000 epinephrine, into the subcutaneous tissues of a concentration of 35 mg/kg was safe. The maximum plasma level that was reached at 11-15 hours postoperatively was 0.8-2.7 mcg/mL, well below the toxic level of 5 mcg/mL. Tumescent anesthetic produces a delay in achieving the peak serum lidocaine level and does not produce as high a level compared with conventional local anesthetic.

Young infant with Failure to Thrive. Born on the 20th percentile now is on the 5th percentile. Found to have a apical systolic murmur, tachynpnea with strong femoral pulse. The most likely diagnosis is A. Coarctation B. HOCM C. PDA D. AS E. VSD

ANSWER E VSD (not coarctation since femoral pulses felt, midsystolic murmur over precordium and back secondary to collateral vessels and coarctation). VSD gives lower left parasternal, pansystolic murmur though. PDA gives bounding, collapsing pulses. (Hyperkinetic apex beat on palpation, 1st left IC space, machinery murmur.

If type and Rh specific blood is given to a patient, how safe is the transfusion (Can't quite remember wording, but similar to what is in Dr Brandis' physiology viva book)? A: ? B: ? C: 97% D: 98.6% E: 99.8%

ANSWER E a simple ABO-Rh type reduces the risk of a transfusion reaction to 99.8%. Screening lowers this risk to 99.94%, and crossmatching lowers it to 99.95%

PC35 ANZCA version [Apr99] [2002-Aug] Q56, [2003-Apr] Q11, [2006-March] Q45, [Jul06] Q30 Side effects of digoxin are increased by A. hyperthyroidism B. hypocalcaemia C. hyperkalaemia D. hypermagnesaemia E. hypothyroidism

ANSWER E hypokalemia, hypomagnesemia, myocardial disease, old age, hypothyroidism, and a variety of other metabolic disturbances (hypoxemia, acid-base abnormalities, hypercalcemia, and hypernatremia

When a new diagnostic test is evaluated in a group of subjects in whom the diagnosis is known, the following results are obtained. Disease known Disease known to be present to be absent Test result positive x y Test result negative 30 60 If a subject from this population tests negative, the probability of NOT having the disease is A. unable to be calculated because "x" is unknown B. unable to be calculated because "y" is unknown C. unable to be calculated because "x" and "y" are unknown D. 0.33 E. 0.67

ANSWER E refers to NPV which = TN/(TN + FN) = 0.67 thus E correct. how to calculate the sensitivity, specificity, PPV, NPV, FPR, FNR. Sensitivity = TP / TP + FN = TP rate Specificity = TN / TN + FP = TN rate PPV = TP / TP + FP = probability of true positive given positive test result NPV = TN / TN + FN = probability of true negative given negative test result False positive rate = FP / TN + FP = 1 - spec False negative rate = FN / TP + FN = 1 - sens

Which nerve does not run in contact with the mandible? A. Nerve to mylohyoid B. Auriculotemporal nerve C. Lingual nerve D. Inferior alveolar nerve E. Buccal nerve

ANSWER E * All are branches of the mandibular division of V. * Lingual related to medial pterygoid and mandible. * Inf alveolar goes through. * Mylohyoid n comes off inf alveolar and runs along mandible. * Auriculotemporal runs between mandible and sphenomandibular ligament. * Buccal does not run right next to mandible

Left bundle branch block (LBBB) on the ECG is A. commonly a normal variant B. associated with a delay in atrio-ventricular conduction C. demonstrated by a notched R wave in chest lead V1 D. associated with right axis deviation E. a relative contra-indication to flotation of a pulmonary artery catheter

ANSWER E A False - due to IHD or LVH (RBBB can be normal variant) B ?False - bundle branch blocks are after the AV node C False - LBBB has notched R in I,III,VL,VF,V5 (Hampton ECG in Practice p272) D False - but RBBB associated with right axis deviation E True - 5% of IHD patients get RBBB on PAC insertion ∴ relative contraindication

Main heat loss in anaesthetic for neonate A. vasodilatation B. radiation C. convection D. conduction E. evaporative

ANSWER E Evaporation if newborn. Radiation if few days old.

A fifty-five-year-old man on antihypertensive medication, including an ACE inhibitor, has a total knee replacement. Red cell transfusion is begun in recovery through a leukocyte reduction filter after brisk bleeding into his drains. A recognised complication of the use of this filter in this situation is A. air embolism B. clotting factor depletion C. haemolysis D. increased risk of postoperative infection E. severe hypotension

ANSWER E. severe hypotension - true: "Profound hypotension has been reported in patients taking angiotension-converting-enzyme (ACE) inhibitors and receiving pretransfusion leukocytereduced blood products—platelets in particular (96). Presumably, ACE inhibitors decrease bradykinin degradation thereby prolonging its intravascular half-life."

Drugs that may be used for the management of heart failure, secondary to dilated cardiomyopathy in pregnancy, include each of the following EXCEPT A. ACE (angiotensin-converting enzyme) inhibitor B. beta-blockers C. digoxin D. loop diuretics E. nitrates

ASNWER A * A. ACE (angiotensin-converting enzyme) inhibitor - true: Category D * B. beta-blockers - false: Oxyprenolol routinely used for pregnancy induced hypertension. Category C * C. digoxin - false: Category A * D. loop diuretics - false: Category C * E. nitrates - false: Cat B2

When used for treatment of neuropathic pain, the dose of gabapentin should be modified if the patient A. has impaired hepatic function B. has impaired renal function C. is also receiving amitriptyline D. is also receiving a proton-pump inhibitor E. is also receiving fentanyl transdermally

ASWERB renal impairment will decrease clearance, and dose should be reduced

Unequal consolidation on CXR can be caused by all except: A. Pleural effusion B. Pulmonary infarction C. Pulmonary haemorrhage D. APO E. Pneumonia

ANSWER D

Latex allergy may present with: A. Bronchospasm B. Urticaria C. Pruritis D. Pulmonary oedema E. All of the above

ANSWER E

The prevalence of latex positive latex skin prick in individuals without increased exposure to latex is about: (a) 0.016 % (b) 0.08 % (c) 0.4 % (d) 2 % (e) 10

%ANSWER C ANZCA/ASA Welfare of Anaesthetists SIG http://www.anzca.edu.au/ceqa/sig_general/welfare/rd18.htm

Methylene blue given intravenously has the effect: A. Pulse oximetry goes down B. Blood Gas Pa02 decrease C. Hypotension D. Metabolic acidosis E. Increased heart rate

ANSWER A

Most likely change on CTG with anaesthesia for non-obstetric surgery at 32 wks A. Loss of beat to beat variability B. No change C. Late decels D. Variable Deccels E. uterine contractions

ANSWER A

Negative pressure leak test in a Boyles type machine. This means a. Vaporiser leak b. Circuit leak c. Brain leaking out of my ears by now d. Leak in non return valve

ANSWER A

Neonate if febrile with rash and periodic breathing. which is likely ABG? a) metab acidosis resp acidosis b) metab acidosis compensated c) resp acidosis d) alkalosis e) alkalosis

ANSWER A

Normal maternal blood gases: A. pH 7.4 B. Bicarbonate 31mmol/l C. pCO2 50mmHg D. Metabolic alkalosis E. None of the above

ANSWER A

Patient with IV in right arm, has mediastinal mass and SVC compression undergoing mediastinal biopsy, suddenly uncontrolled surgical bleeding in mediastinum. Next step in management prior to thoractomy: A. insert femoral cannulae and place on bypass B. insert IV in left arm C. insert IV into foot D. insert jugular CVC E. trendelenburg

ANSWER C

In what proportion of people is the AV node supplied by the R coronary artery? a. 5 b. 15 c. 40 d. 40 e. 85

ANSWER E Anatomy for Anaesthetist: "The atrioventricular node is perfused by the right coronary artery in 80% of subjects"

In a 140kg obese patient, compared to a 70 kg person A. cardiac output >20% lower B. cardiac output 10% lower C. cardiac output no different D. cardiac output 10% higher E. cardiac output >20% higher

ANSWER D

In a clinical trial, 3 out of 10 patients develop a complication in the control group, and 1 of 10 patients develops the complication in the treated group. To assess whether this is a statistically significant difference the most appropriate statistical test to use would be the A. Chi-square Test B. Chi-square Test with Yates correction C. Student's t-test D. Fisher's Exact Test E. Mann-Whitney Test

ANSWER D

In pregnancy the dural sac ends at: A. T12 B. L2 C. L4 D. S2 E. S4

ANSWER D

In pregnancy the dural sac ends at: A. T12 B. L2 C. L4 D. S2 E. S4

ANSWER D

In relation to obstetric haemorrhage A. amniotic fluid embolism is unlikely to present as unexplained haemorrhage B. cell salvage is too dangerous to be recommended because of potential contaminants C. coagulopathy after severe placental abruption is ONLY likely if fetal death in-utero occurs D. intravenous magnesium may facilitate replacement of an inverted uterus E. placenta percreta can be excluded by ultrasound examination

ANSWER D

Which is NOT an advantages of RA vs. GA for TURP? A. Less post-operative confusion B. Fewer pulmonary complications C. Lower DVT risk D. Less blood loss E. Earlier recognition of electrolyte changes

ANSWER B

Which of the following is the best guide to fluid replacement in a burns patient? A. Haemoglobin and haematocrit B. Urine output C. Urine osmolality D. Plasma volume E. Specific gravity

ANSWER B

Which of the following statements about "bias" in scientific studies is FALSE? A. bias is a systematic deviation from the truth B. observer bias can be eliminated by blinding C. randomisation is one of the most important ways to reduce bias D. the Hawthorne effect may be reduced by masking the actual intent of a study E. triple-blinding refers to the blinding of patient, observer and investigator

ANSWER B

A hypothetical volatile anaesthetic agent with a saturated vapour pressure of 380 mmHg at 20C is placed in a copper kettle vaporiser. The flow meters are set at oxygen 2 l/min & nitrous oxide 2 l/min. Flow into kettle 0.5 l/min. At 20C the concentration of the volatile anaesthetic agent delivered in the gas is: A. 1% B. 2% C. 5% D. 10% E. 11

%ANSWER C Cooper Kettle - 2 flows 1. Vopor flow = 100% saturated -concentration of volatile = SVP/ATM x100 = 50% -if flow 0.5L flow in, therefore 0.5L of volatile is added -meaning 1L volume exits with 50% concentration of anaesthetic vapor 2. Dilutant flow -4L in total (2L O2 and 2L N2O) -therefore 0.5/5 =10%

AZ64 ANZCA version [2002-Aug] Q90, [2003-Aug] Q50, [2005-Apr] Q23, [2005-Sep] Q13 The Mallampati classification of the airway using the view of the faucial pillars, the soft palate and the uvula obtained with the tongue protruding, is A. validated in both children and adults B. associated with a high false positive rate for difficult laryngoscopy C. not applicable to edentulous patients D. useful for choosing the size of endotracheal tube E. associated with a positive predictive value of approximately 90

%ANSWER C For predicting difficult tracheal intubation, the original Mallampati test had very poor accuracy. Four of the five studies had sensitivities <50%. Small increases in sensitivity led to large sacrifices in specificity. From Cattano's figures rounded out: true -ve (MP<3, easy tube) 1700. false +ve ( MP>=3, easy tube) 180 false -ve (MP<3, hard tube) 30. true +ve (MP>=3, hard tube) 15 Sensitivity (good sens has low false neg) = TP/(TP+FN) = 15/(15+30) = 30% Specificity (good spec has low false pos) = TN/(TN+FP) = 1700/(1700+180) = 90% PPV = TP/(TP+FP) = 15/(15+180) = <10% NPV = TN/(TN+FN) = 1700/(1700+30) =>98%

PR04 [1985] [Mar95] [Apr97] [Jul97] [Apr98] [Jul98] [2002-Aug] Q11, [2003-Apr] Q39, [Jul05] [Mar06] The percentage of the population which is heterozygous as regards pseudocholinesterase, thus having a dibucaine number between 30 and 80 is A. 0.04% B. 0.4% C. 4% D. 14% E. 40

%ANSWER C Heterozygous 4% Homozygous 1:2500 = 0.04% * "All possible combinations of heterozygotes exist - they constitute 3.8% of the population and remain apnoeic for approximately 10 minutes after receiving suxamethonium." Sasada & Smith, 3d ed., p.368. * "U: Homozygote Usual (normal), frequency = 96%, UA: Heterozygote Usual/Atypical, frequency = 3%, A: Homozygote Atypical, frequency 1 in 3,000, US : Heterozygote Usual/Silent, frequency = 0.7%, S: Homozygote Silent, frequency 1 in 40,000, AS: Heterozygote Atypical/Silent, frequency 1 in 8,000 (0.0125%). Others rare." National reference laboratory : Pseudocholinesterase, Dibucaine Inhibition. [1] * "In most Caucasian populations the frequency of occurence of heterozygotes for the normal and atypical genes is about 1 in 25, and the frequency of homozygotes for the atypical gene is about 1 in 2500. The atypical gene is rare in orientals and African blacks." Pantuck EJ: Plasma cholinesterase: Gene and variations. Anesth Analg 77:380-386, 1993 * "In addition, heterozygotes with one normal and one abnormal gene (5% of the population) with dibucaine number of 40-60 may show prolonged paralysis of about 10-20 min." Yentis, 3rd ed., p.117.

In modern anaesthetic practice, mortality from myocardial infarction after non cardiac surgery is A. 2.5-5% B. 5-10% C. 10-15% D. 20-30% E. approximately 50%

%ANSWER C Mortality from myocardial infarction (MI) after noncardiac surgery is believed to be 10 to 15%,1 similar to that in nonsurgical patients.

Hypothetical anaesthetic agent. Flows 2 L oxygen and 2L nitrous. Copper kettle vaporiser flow 0.5 L. Volatile with SVP 380 mmHg. Inspired volatile concentration (no numbers were exactly correct) a. 2% b. 5% c. 10% d. 11% 3. 15

%ANSWER C Reading the explanation in Morgan and Mikhail, if 0.5L of gas enters the vapouriser, 1L exits, with a volatile concentration of 50%. This is diluted by 4L of FGF, so you have a total of 5L of gas. 500mL in 5L is 10%. Doesn't make sense? A copper kettle has a dedicated flow meter which is dialed up separately. To make your 0.5L of gas flow fully saturated with vapour at an SVP of 380mmHg, you need to double it (ie 380mmHg of vapour in 380mmHg of gas). This is mixed with the FGF which is separate; thus giving 500mL of vapour, 500mL of gas which went into the copper kettle, 2L of oxygen and 2L of nitrous.

The incidence of vertebral artery injury in patients with cervical fractures or dislocations is A. 1% B. 5% C. 15% D. 40% E. 70

%ANSWER D 40% The association between cervical spine fractures and vertebral artery injury is well documented, although the true incidence of the condition and its significance remains unclear. Imaging of the vertebral artery is not routine in cervical spine trauma, so those case reports in the literature tend to report only those patients presenting with neurologic deficits, as with this patient. Four studies have prospectively evaluated cervical spine injuries using conventional or magnetic resonance angiography. The incidence of vertebral artery injury with cervical spine fracture ranged between 24% and 46%. However only about one quarter of these patient will have neurological deficits.

The prevalence of positive latex skin prick in individuals with increased exposure to latex is about: (a) 0.056 % (b) 0.28 % (c) 1.4 % (d) 7 % (e) 35

%ANSWER D ANZCA/ASA Welfare of Anaesthetists SIG http://www.anzca.edu.au/ceqa/sig_general/welfare/rd18.htm

Area burnt in adult male - upper half of upper limb, anterior abdo, whole left leg: A. 9% B. 16% C. 23% D. 32% E. 40

%ANSWER D Arm = 9 say 5 %, Abdo 9% (18% depending if chest too?) Leg 18%= 32%

A hypothetical volatile anaesthetic agent with a saturated vapour pressure of 380 mmHg at 20C is placed in a copper kettle vaporiser. The flow meters are set at oxygen 2 l/min & nitrous oxide 2 l/min. Flow into kettle 0.5 l/min. At 20C the concentration of the volatile anaesthetic agent delivered in the gas is: A. 1% B. 2% C. 5% D. 10% E. 11

%ANSWER D Cooper Kettle Vaporizer :dial up 2 separate flows 1. Vapor inlet flow : where vapor is added, where the concentration vapor =SVP/ATM x 100 2. Diluting fresh gas flow : mixes and dilutes. Conc vapor = 380/760 x 100 = 50% As Inlet flow =0.5L, therefore Outlet flow must be 1L Diluting flow is 4L, therefore total flow is 5L Therefore Volatile Concentration = 0.5 / 5 = 10% This is different to a variable pass vaporizer, where FGF is diverted to vaporizer. Conc vapor = 380/760 x 100 = 50% As Inlet flow =0.5L, therefore Outlet flow must be 1L FGF is 4L, but 0.5L used for vaporizer, therefore total flow is 4.5L Volatile Concentration = 0.5 / 4.5 =11%

MR43 ANZCA version [2001-Apr] Q99 Unilateral diaphragmatic paralysis causes 1. no change in total lung capacity 2. a decrease in forced vital capacity 3. no change in maximum voluntary ventilation 4. a decrease in inspiratory reserve volume

A - FALSE B - TRUE C - FALSE D - TRUE Lung function studies can be used to assess the impact a paralysed hemidiaphragm has on the mechanics of ventilation. A mild restrictive pattern is evident with total lung capacity generally reduced to about 85 % of predicted, and vital capacity to about 75 % of predicted; a fall in forced vital capacity of greater than 20 % on lying supine suggests diaphragmatic paralysis. Maximal inspiratory pressures are only mildly reduced in unilateral disease. Functional residual capacity and the forced expiratory volume in one second are usually normal, as the mechanics of expiration and elastic recoil of the lung are unaffected. It must be remembered that there is great reserve in the lungs, and it can take a considerable degree of weakness of the diaphragm(s) before there is a fall in the forced vital capacity. This reserve may differ between individuals and may explain why some patients become symptomatic and others don't

AM25a ANZCA version [2001-Aug] Q54, [2003-Apr] Q60, [2005-Sep] Q84, [Mar06] Q62 (Similar question reported in [Aug96]) In an acute malignant hyperthermia episode A. the serum creatinine kinase level peaks within one hour B. the peak serum creatinine kinase level is a good indicator of the amount of muscle involved C. elevated creatinine kinase levels contribute to acute renal failure D. the serum myoglobin level does NOT peak for at LEAST 24 hours E. muscle rigidity occurs in 75% of cases

A - FALSE * creatine kinase peaks around 24 Hrs? B - FALSE * peak serum CK is not a reliable indicator of MH vs non-MH, as CK levels of MH patients often within levels expected from surgery itself..."patients who have had an acute MH episode during a surgical procedure may have peak CK values within the range of CK values expected from the procedure itself"...Creatine kinase alterations after acute malignant hyperthermia episodes and common surgical procedures o Serum myoglobin (by radioimmunoassay) and creatine kinase were measured for up to 7 days in 30 patients following surgical procedures, including total hip replacement and bilateral subcostal abdominal incisions. Serum myoglobin reached a maximum of 1390 µg litre-1 (median 345 µg litre-1 for major surgery patients) on the first postoperative day but levels were still elevated by day 7 in some patients. Creatine kinase reached a maximum of 1339 i.u. litre-1 at day 2 (median 422 i.u. litre-1 for major surgery patients), generally peaking 1 day after myoglobin in individual patients. These values may have significance when investigating a suspicion of coincident perioperative events such as myocardial infarction or malignant hyperthermia...Serum myoglobin and creatine kinase following surgery C - FALSE * Myoglobin D - FALSE * myoglobin peaks ~ 6hrs later E - TRUE * rigidity 80%

Tracheo-oesophageal fistula: A. Diagnosed reliably by failure to pass a soft tube into the stomach B. Should all be intubated soon after birth to reduce aspiration C. Intercostal blocks are contraindicated for postop pain relief after repair because of local anaesthetic toxicity risk D. Associated with renal abnormalities

A False because in H fistulas you still can pass the nasogastric tube B False because aspiration can still occur following intubation - still have a fistula. Secondly intubation is indicated for respiratory failure (spontaneous breathing is the goal. IPPV can often leads to preferential insufflation of the gut due to lower resistance that leads to respiratory embarrassment), surgery or post surgical recovery for example. C False? I cannot see any contraindication if dosage is in the non toxic range. Also re above "pneumothorax would be a disaster"; if they had a TOF they would often approach with a R thoracotomy and usually an ICC is placed after a thoracotomy? D True. TOFS can be associated with renal abnormalities as in part of the VACTERAL syndrome - vertebral, anal, cardiac, TOF (or TE), renal AND limb anomalies

A two year old child sustains a simple fracture of the lower end of the forearm one hour after having a full meal. The most appropriate approach is to A. postpone surgical reduction for 12 hours and treat as an elective case B. allow immediate reduction using a rapid sequence induction, cricoid pressure and intubation C. allow immediate reduction using a regional technique D. allow immediate reduction, after gastric emptying with a tube followed by rapid sequence induction, cricoid pressure and intubation E. wait 4 hours and treat as elective

A is best answer? (C,D,E are more incorrect) ?B - never heard of a four/five hours fast, but it's close to six. This is an area fought with most contention. My own approach is at least six hour fast and low threshold of RSI with any of the following: - same day injury - meal close to time of injury - unwell looking and distressed child - pain and opioid - afterhour with little help around Face mask/LMA if next day, pt fasted, hungry, etc. Main thing is ensure adequate depth of anaesthesia prior to any manipulation (airway or bones).

MR34 ANZCA version [2003-Apr] Q76, [2003-Aug] Q88 Asbestos exposure is typically associated with each of the following EXCEPT A. pleural effusions B. mesothelioma of the pleura C. laryngeal cancer D. hilar lymphadenopathy E. gastrointestinal cancer

ALL are TRUE A. pleural effusions Association Benign exudative pleural effusions usually occur within 15 years of first exposure to asbestos [6]. They may resolve spontaneously, but leave visible blunting of the costophrenic angle or thickening of the visceral pleura. Exclusion of associated malignancy may require thoracoscopic evaluation. Uptodate B. mesothelioma of the pleura Typical Association C. laryngeal cancer Association + smoking D. hilar lymphadenopathy Secondary association. Hilar and mediastinal lymphadenopathy are not seen with asbestosis and should suggest the presence of another process - Uptodate However mesothelioma is associated with hilar lymphadenopathy though this is not a common feature. E. gastrointestinal cancer Association Oesophageal Cancer.... Asbestos exposure increases the incidence of other neoplasms as well. Asbestos is the only known risk factor for malignant mesothelioma. Other malignancies that have been linked to asbestos include cancers of the larynx, oropharynx, kidney, esophagus, and biliary system.

MR24 [1988] Legionnaire's Disease: A. Diarrhoea B. Conjunctivitis C. Pneumonia D. Liver disease E. Encephalopathy

ALL have been reported to occur. Legionella are gram negative bacilli. They are intracellular organisms. Legionella bacteria cause 2 distinct clinical conditions: 1. Legionnaires disease, a severe multisystem disease which includes pneumonia 2. Pontiac fever, a self-limiting flu-like illness without pneumonia The natural environment for legionella is water, and they multiple at temperatures between 25 and 42C, with 35C being their optimal temperature meaning that hot water systems, whirlpools etc are ideal breeding grounds. Many outbreaks have also been associated with air-conditioning systems or with travel, especially cruise ships. Symptoms of Legionnaire disease include * Fever * Dry cough * Dyspneoa * Nausea, vomiting and abdominal pain * Confusion Relative bradycardia may occur in up to 66% of patients. The vital signs may reveal high fever' and tachypnoea. Other pulmonary manifestations include dyspnea, pleuritic chest pain, and hemoptysis, which may be present in as many as one third of cases. Pericarditis and endocarditis may be present. Legionnaire disease is also associated with * Hyponatraemia (SIADH is associated) * Deranged LFTs (very common, may help distringuish Legionnaires disease from other pneumonias) * Thrombocytopeaenia *pleural effusion (up to50%) Diagnosis may be by * Urinary antigen * Sputum culture * Serology Most patients have abnormal CXR at presentation, most commonly patchy basal consolidation. Up to 30% have pleural effusions. Treatment is with antibiotics with a high intracellular concenration, such as azithromycin, clarithromycin, fluroquinolone or rifampacin. Note that Legionnaires disease is a notifiable disease.

A 50 year old patient is admitted with acute chest pain. An ECG shows tall R waves in lead V1. The most likely diagnosis is A. occlusion of the anterior descending coronary artery B. a lateral myocardial infarction C. pulmonary infarction D. posterior myocardial infarction E. occlusion of the circumflex artery

ANSWE D Tall R in V1 is defined as R/S ratio equal to or greater than 1. This is normal in ONLY 1% of presentations. Causes include 1. RBBB 2. left ventricular ectopy 3. Posterior MI (tall R V1-3, ST depression V2/3, TWI V1-3. Tall T V2) 4. Right ventricular hypertrophy 5. Right ventricular strain, dilation eg PE 6. WPW syndrome with left sided accessory pathway 7. HOCM 8. Dextrocardia 9. Normal Variant 10. Misplaced electrodes

IA young boy sustains a head injury and CT scan shows diffuse axonal injury. He is intubated and ventilated in the ICU and is going to theatre due to rising ICP's (20 to 30) despite mannitol and cooling. The best anaesthetic to give is: A. propofol and fentanyl B. sevoflurane and remifentanil C. isoflurane and remifentanil D. option including nitrous oxide E. ?

ANSWER

AZ42 ANZCA version [2001-Aug] Q1 (Similar question reported in [Aug93] [Apr96]) Carotid endarterectomy for symptomatic carotid disease A. has a peri-operative stroke rate of 8 to 10% B. should be performed under regional anaesthesia in patients with ischaemic heart disease to reduce the incidence of myocardial infarction C. may be performed under induced hypotension to reduce intraoperative blood loss D. when performed under deep cervical plexus block is often associated with a phrenic nerve palsy E. should be performed under regional anaesthesia in patients with a recent transient ischaemic attack to reduce the incidence of peri-operative stroke

ANSWER A. FALSE : 6.5% if symptomatic, 2.3% if assymptomatic B. FALSE : no evidence C. FALSE : BP should be normal to high to reduce cerebral ischemia D. TRUE E. FALSE : no evidence that regional reduces ischemic events (GALA trial no difference between primary outcomes for GA vs LA)

AZ23 ANZCA version [2001-Apr] Q120 (Similar question reported in [1986] [Mar90] [Aug95] [Apr96]) In the management of a 65 year old man who requires thoracotomy for a mediastinal tumour previously treated with large doses of bleomycin 1. a chest x-ray is essential to exclude pulmonary fibrosis 2. oxygen therapy should be given by face mask soon after narcotic premedication 3. impairment of renal function is common 4. surgery should be delayed for 48 hours after lymphangiography

ANSWER Bleomycin * used Testicular Ca,and squamous head and neck tumours, lymphomas * Can be used iv/sc/im, * little myelosuppression * Cleared renaly and you have increased complications if preexisting renal failure * Can be inactivated by bleomycin hydrolase [edit] Complications of bleomycin * allegic reactions common * raynauds, * mucocutaneous reactions eg alopecia, hyperpigmentation common * Toxicity dose usually more than a total of 300-400 units . [edit] pulmonary fibrosis * 5-10 % * usually lower lobes * Risks factors include o total bleomycin dose; o renal failure (it is renally excreted); o concurrent radiotherapy or other chemo; o age > 70. o possibly excessive crystalloid admin o possibly high FIO2

The patient most likely to sustain blunt cardiac trauma (cardiac contusion) is a A. back seat passenger in a motor vehicle which crashes at a speed of 60 km.hr-1 B. motor bike rider who falls from his bike on a bend travelling at 50 km.hr-1 C. painter who falls 10 metres on to grass D. seventy-year-old patient who had cardiopulmonary rescusitation (CPR) performed by a passerby in the street E. water skier who falls from his skis travelling at a speed of 40 km.hr-1

ANSWER By far, the most important cause of significant blunt chest trauma is motor vehicle accidents (MVAs). MVAs account for 70-80% of such injuries. As a result, preventive strategies to reduce MVAs have been instituted in the form of speed limit restriction and the use of restraints. Pedestrians struck by vehicles, falls, and acts of violence are other causative mechanisms. Blast injuries can also result in significant blunt thoracic trauma.

AM41 ANZCA version [2004-Aug] Q15, [Mar06] Q11, [Jul07] The most frequently reported clinical sign in malignant hyperpyrexia is A. arrhythmia B. cyanosis C. sweating D. tachycardia E. rigidity

ANSWER Diagnosis of MH (Table 2 from Ref) 1. Unexplained, unexpected increase in end-tidal carbon dioxide (most sensitive indicator of potential MH) 2. Unexplained, unexpected tachycardia and masseter muscle spasm usually follow the carbon dioxide increase 3. Mixed respiratory and metabolic acidosis 4. Skeletal muscle rigidity 5. Temperature elevation (often a late sign) 6. Laboratory abnormalities: blood—coagulation profile, electrolytes, arterial blood gas, creatine kinase; urine—myoglobin

SF28 ANZCA version [Mar91] [2001-Aug] Q133, [2002-Mar] Q123 Partial lateral tilt during a caesarean delivery at term 1. on average results in improved fetal oxygenation 2. on average results in improved maternal oxygenation 3. is more effective in combating supine hypotension when it is to the left than to the right 4. is effective in combating supine hypotension in all patients when it is 15 degrees in magnitude

ANSWER 1 and 1. True. Late decelerations as a sign of foetal hypoxia are 5x more common in the supine position. 2. False. My extremely detailed review article doesn't mention it anyway. No clear reason exists for maternal hypoxia. 3. True. Left tilt is generally better than right. There is an occasional case where right tipping is better, thought to be due to a left-leaning uterus. 4. Untrue. In some tilt of up to 40 degrees is needed and manual uterine displacement is sometimes used.

AM35 ANZCA version [2001-Aug] Q101, [2002-Mar] Q106 Myotonia dystrophica 1. is associated with cataracts 2. is strongly associated with malignant hyperpyrexia 3. can result in increased sensitivity to respiratory depressants 4. has its inheritance best described as autosomal recessive

ANSWER 1 and 2 Myotonic dystrophy is a dominantly inherited disease characterized by myotonia, progressive myopathy, insulin resistance, defects in cardiac conduction, neuropsychiatric impairment, cataracts, testicular atrophy, and frontal balding in males.[935] Patients with myotonic dystrophy have increased mortality from respiratory complications secondary to aspiration as a result of their muscle weakness, as well as cardiac dysrhythmias. MH association just is not true. This may be referring to Myotonia Congenita which may be associated with susceptibility to MH

If two methods of measuring a physiological parameter have a Pearson correlation co-efficient of 0.99 one could conclude that 1. there is a strong linear association between the two methods of measurement 2. there is good agreement between the two methods of measurement 3. increases in the value of one measurement will usually be associated with increases in the value of the other measurement 4. the two measurements probably use a similar methodology

ANSWER 1 and 3

SF43 ANZCA version [2002-Mar] Q110, [2002-Aug] Q138 (Similar question reported in [Aug96] [Apr97] [Jul97]) A pregnant 30-year-old woman is given nitrous oxide and oxygen for late first stage and second stage analgesia. Induction of analgesia is rapid because 1. functional residual capacity (FRC) is reduced 2. the cardiac output is increased in pregnancy 3. alveolar wash-in of agent is rapid due to hyperventilation 4. hyperventilation increases uterine blood flow

ANSWER 1 and 3 FRC is reduced and hyperventilation causes rapid wash-in of agent. Increased cardiac output would slow onset of analgesia due to N2O and uterine blood flow is irrelevant as N2O exerts a CNS effect to cause analgesia.

AM33 ANZCA version [2001-Apr] Q130 (Similar question reported in [Mar00] [Jul00]) Neuroleptic Malignant Syndrome may be precipitated or exacerbated by 1. metoclopramide 2. droperidol 3. thioxanthenes 4. bromocriptine

ANSWER 2 Neuroleptic malignant syndrome occurs in a small percentage of patients (<1%) taking potent dopamine ANTagonists (eg neuroleptics such as haloperidol Also atypical antipsychotic agents, including clozapine (Clozaril), risperidone (Risperdal), olanzapine (Zyprexa), and quetiapine (Seroquel) The earliest symptom is usually marked muscle rigidity due to excessive extra-pyramidal activity resulting from excess dopamine receptor antagonism. The syndrome is characterized by the following features: * Autonomic dysfunction * Elevations in CK and white blood cell count * Fever * Hyperthermia * Mental status changes * Severe muscle rigidity * Tremor. The 3 major signs supporting the diagnosis of NMS are: * Hyperthermia * Muscle rigidity (usually lead-pipe rigidity) * Elevated creatine phosphokinase (>1,000 IU) - indicating myonecrosis Management * Early recognition and cessation of neuroleptic drugs * Exclusion of infection as a cause for fever (eg blood cultures, antibiotics) * Supportive measures (eg use of physical cooling methods, cardiorespiratory support and monitoring, diuresis to avoid renal failure) * Use of specific drugs may be useful, for example: o dantrolene - a direct skeletal muscle relaxant o bromocriptine - a dopamine agonist o diazepam - central skeletal muscle relaxant and sedative.

AM37 ANZCA version [2002-Mar] Q133 The caffeine-halothane contracture test for malignant hyperthermia 1. may be performed up to 24 hours following collection of the specimen 2. has a 10 - 20% false positive rate 3. is indicated in patients with neurolept malignant syndrome 4. requires that the muscle specimen has NOT been directly infiltrated with local anaesthetic

ANSWER 2 4 1. False. The standard is 5 hours 2. True. Specificity (i.e. true negative rate) 78-94% (Miller 5th edn p 1049) or 85-90% therefore false positive rate 6-22% or 10-15% 3. False. 4. True. "The test must be performed on a biopsy of approximately 2 g of muscle from the vastus lateralis or medialis within 5 hours of harvesting. The patient is anaesthetized with general anaesthesia or with a femoral nerve block or one of its variants. In all cases the anaesthetic drugs used must be safe for MH-susceptible patients. Direct muscle infiltration with local anaesthetic is contraindicated because it could affect tissue viability.

AZ21 ANZCA version [2002-Mar] Q120, [2002-Aug] Q146 (Similar question reported in [1987] [1988] [Aug92] [Apr96] [Aug96] [Apr97] [Jul97] [Jul98] [Apr99] [Aug99] [Jul00]) In an adult with advanced ankylosing spondylitis 1. surgery near the elbow could be done most safely under successful interscalene brachial plexus block 2. there is decreased lung compliance 3. maintenance of the airway during general anaesthesia using a face mask is frequently difficult 4. cardiovascular assessment should be thorough because of the known association with aortic insufficiency

ANSWER 2 and 4 Ankylosing spondylitis is a type of seronegative arthropathy. Implicatications for anaesthesia: * Pulmonary fibrosis hence potential hypoxia * Aortic root pathology particularly AR 1 - incorrect. Interscalene blocks can cause loss of diaphragmatic tone, hence would be contraindicated in patients with respiratory complications. 2 - correct. Pulmonary fibrosis leads to decreased lung compliance 3 - incorrect. Ank. spond does not lead to difficulty with face mask application 4 - correct. Watch for AR.

AZ37 [Mar93] [Aug93] Lightly anaesthetized patient in the lithotomy position (?spontaneously breathing): 1. Marked reduction in vital capacity 2. Increased FRC 3. Normal alveolar ventilation 4. Residual volume & closing volume equal 5. Decreased A-P diameter of thorax

ANSWER 4 1. FALSE : no change to VC 2. FALSE : decrease in FRC 3. FALSE : decreased alveolar ventilation 4. TRUE : CV approaches FRC 5. FALSE : there is decreased crossectional area, but no change anterioly or caudad

What is the main lung function derangement in pregnancy? A. Decreased tidal volume B. Decreased VC C. Decreased FRC D. Decreased airway resistance E. ?

ANSWER ?? Increased Respiratory Rate 15% Decreased FRC 20% (decreased ERV & RV) Increased Tidal Volume 30-40% Increased Minute Volume 50% Increased Alveolar Ventilation 70% decreased ariway resistance

Allergic question, which is true A. Collect tryptase 8hours B. RAST test most sensitive/ specific C. Absent of trytase exclude anaphylactic D. Skin and intradermal test- sensitivity, specificity

ANSWER ??? Skin prick is easy to perform, safe and provides the best sensitivity/specificy combo. However it requires a skilled proceduralist Interdermal has higher sensitivity but higher anaphylaxis rate. RAST is 100% specific but only 75% senstive. Used to determine if pt has IgE antibodies to particular agents. It does not diagnoses anaphylaxis (as this requires 2 IgE to crosslink)

AZ41 [Aug93] [Mar94] Scleroderma: A. Nasal intubation is contraindicated B. Sensitive to non-depolarising muscle relaxants C. Pulmonary involvement is only associated with severe systemic disease D. ? E. Gaseous induction is contra-indicated (Related Q MZ32)

ANSWER ??D Nasal intubation is RELATIVELY contraindicated because of the risk of bleeding from telangiectasia but may be required because of difficult upper airway/neck anatomy. Gas induction is similarly RELATIVELY contraindicated because of the risk of aspiration Introduction * Onset 20-40 years * Characteristics of autoimmune and collagen disease * Do NOT give corticosteroids * Can progress to CREST syndrome Signs and symptoms * Skin and musculoskeletal o Thickening o Increased CK from myopathy o Arthritis * Neuro o Perpipheral or Cranial neuropathy o Trigeminal neuralgia * CVS o Sclerosis coronary arteries o Replacement of cardiac muscle with fibrosis o Arrhythmias o CCF o pulmonary HT o Pericarditis o Oral or nasal telangectasias o Raynauds * Lungs o Diffuse interstitial fibrosis so decreased IC and increased RV o Decreased lung compliance due to fibrosis o Decreased diffusion capacity * Renal o Decreased RBF o Accelerated systemic HT and renal failure most common cause of death * GIT o Xerostomia (dry oral mucosa) o Dysphagia common o Decreased LOS tone so increased risk of reflux Anaesthesia * Decreased mouth opening due to tight skin * Telangectasias may bleed profusely * IV access difficult * Can be difficult getting a pulse oximeter reading from the fingers * Vasomotor instability - minimised by keeping patient warm * Increased risk of aspiration/reflux * Decreased pulmonary compliance so may need high airway pressures * Supplemental O2 needed because of poor diffusion capacity

PL26 ANZCA Version Jul06 Q111 In severe bupivacaine toxicity, drugs likely to improve the cardiac conduction abnormalities include A. clonidine B. lignocaine C. ketamine D. metoprolol E. propofol

ANSWER ?A Outdated question Management * Oxygenation/ventilation and good CPR to prevent acidosis. * Defibrillation, adrenaline or vasopressin (vasopressin better). * Best option would be 1ml/kg intralipid 20% x 3 doses q3min + infusion of 0.25ml/kg/hr as per A&IC. * Cardiopulmonary bypass or intraaortic ballon pump. * Amiodarone forms part of the current ACLS guidelines and has a non-significant trend to improved survival in piggies. * Lignocaine has been reported but I can't find a reference and Miller says not to. * Propofol is useful in early toxicity as may prevent seizures, but should not be used for treatment of arrhythmia or in severe cv collapse because the amount of lipid in propofol is low and the total propofol dose required would cause severe CVS impairment. Better to give Intralipid as that is the therapeutically effective agent.

AZ60 [Mar00] [Jul00] Young male with a ruptured right main bronchus and fractured 1st & 2nd ribs with blood from external auditory meatus. (?Past history of narcotic addiction.) Components of induction include: (OR: Appropriate anaesthetic: ) A. Awake nasal intubation after topicalisation B. Awake fibreoptic orally using topical lignocaine (? & DLT) C. ? D. Left double lumen tube & use scope to check position (Similar to AT19) (see also IC15b)

ANSWER ?B OPTIONS - Awake surgical trache if no laryngeal damage - AFOI if tolerated, with DLT - still is risk of obstruction - Inhalational induction, keep spont breathing, DLT (risk of obst esp with relaxant) In all cases have ENT scrubbed in theatre ready to do emergency cricothyroidotomy/trache Also obviously do all this in OR NOT in ED Locate cricothyoid membrane before induction/AFOI so can do emergency cricothyoidotomy

Redo CABG following median sternotomy surgeon states he has accidentally cut a vein graft, immediately followed by ST elevation on ECG and VF, next action. A) External defibrillation 200J (Biphasic) B) Heparin IV then femoral cutdowns for bypass C) Hand ventilate with 100% oxygen D) GTN infusion E) metaraminol

ANSWER ?B Part of the heart is completely ischaemic. In this specific instance, crashing onto bypass is the most appropriate intervention.

PN46 ANZCA version [2006-Mar] Q128, [Jul06] Q83 (NB. PN43b = the same question) Concerning opioids, A. fentanyl is the agent of choice for patient controlled analgesia (PCA) in the opioid addicted patient presenting for surgery B. morphine in therapeutic dosage is a common cause of postoperative confusion C. pethidine is suitable for subcutaneous injection D. sufentanil has a higher affinity for the mu receptor than morphine E. the patient's age is the best clinical indicator of opioid requirement in the perioperative period

ANSWER ?C A. FALSE B. FALSE: Morphine causes confusion in 14% of patients C. TRUE: MIMs states pethidine can be given SC, most correct answer D. TRUE: Sufentanil has a high affinity for the mu receptor, higher than any other opioid E. TRUE: from latest pain book. But it is also dependent on the type of surgery

04 ANZCA Version[Jul07] Which of the following statements regarding infection control is FALSE? A. devices to be used in the upper airway that may cause bleeding must remain sterile until used B. provided there is an adequate filter between the patient and the breathing circuit, the circuit can be re-used for subsequent patients on an operating list C. when performing central neural blockade, the anaesthetist must adopt a full aseptic technique D. when performing central venous cannulation, the anaesthetist must adopt a full aseptic technique E. when performing vascular cannulation, the anaesthetist must wash hands and should wear gloves

ANSWER A

12 year old child with hip dislocation at 4pm. Ate 1 hour after injury. Now 11 pm. Best anaesthetic: A. RSI with ETT B. delay until next day then treat elective C. inhalational induction and continue with face mask D. Reduce immediately with iv sedation E. inhalational induction and continue with face mask

ANSWER A

18/12 old undergoing routine SV GA under LMA. Sudden onset SVT with HR 220 BP 84/60 ETCO2 32 SpO2 98.Management: A. Adenosine 100mcg/kg B. DCR 2J/kg C. DCR 4J/kg D. Amiodarone 5mg/kg E. CPR

ANSWER A

32 y/o male. Weakness distal and prox muscles, infection 10 days ago, no sensory involvement, temp 37.8, facial weakness. Cause: A. Guillian Barre B. Myasthenia Gravis D. Poliomyelitis E. ?Acute encephalitis F. ?Polymyositis

ANSWER A

32 year old with early acute liver failure (not paracetamol related). Management includes A. prophylactic antibiotics B. N-acetyl cysteine as general liver protection C. avoid early intubation so can assess for encephalopathy D. INR> 3 means should be considered for transplant?? E. avoid saline as resuscitation fluid

ANSWER A

5yo 35kg child having repair of leg laceration. gas induction with sevo N2O and oxygen. Can't get in drip. Put in LMA and immediately get stridor and airway obstruction and desaturate to 90%. Next step after increase FiO2 to 100% is: A Remove LMA and deepen with sevoflurane B Leave LMA and deepen with sevoflurane C Intralingual Suxamethonium D IM Atropine E IM Suxamethonium

ANSWER A

60 year old vascular patient. ECG given. Showed large positive R waves in lateral leads, large negative S waves in anterior leads. ST depression laterally ie LVH with strain; bicuspid p waves A. LVH with strain B. Enlarged RA C. Lateral ischaemia D. LBBB E. L posterior hemiblock

ANSWER A

A 12 year old child with hip dislocation at 4pm. Ate something 1 hour after injury. Now 11 pm. Best anaesthetic: A. RSI with ETT B. delay until next day then treat elective C. inhalational induction and continue with face mask D. Reduce immediately with iv sedation E. inhalational induction and continue with face mask

ANSWER A

A 16-year-old arrives in your Emergency Department having fallen from, and been trampled by, a horse. The ambulance officers report that this initial GCS (Glasgow Coma Scale) was 15 and is now 11. His pulse is 120 and blood pressure 80/60 mmHg. SpO2 is 97% when breathing on a Hudson mask. On auscultation his chest is clear. He has facial bruising and a tender abdomen. Your first priority now is to A. administer an IV (intravenous) fluid bolus B. arrange an urgent CT scan of the head C. administer mannitol D. perform a FAST (focussed assessment with sonography for trauma) examination E. secure the airway by intubation

ANSWER A

A 40 year old female loses 15% of her blood volume as a result of an accidental arterial laceration during a hysterectomy. The most appropriate immediate therapy is: A. Crystalloids B. Colloids C. Crystalloids and packed red cells D. FFP and packed red cells E. Whole blood

ANSWER A

A 59-year-old male is admitted with major trauma following an automobile accident. His blood type is O-negative, but you are out of O-negative blood. Which of the following blood products would be UNACCEPTABLE to transfuse? A. AB positive red cells B. A negative platelets C. O positive red cells D. AB positive FFP E. B positive FFP

ANSWER A

A 78 year old man with past difficult intubation for arm surgery. Supraclavicular block with 25 mls 0.5% bupivacaine. Shortly after begins convulsing. INITIAL management? A. Midazolam 5mg B. Intralipid 20% 1.5 ml/kg C. Thiopentone 150mg D. Suxamethonium 50mg E. Propofol 50mg

ANSWER A

A Swan-Ganz catheter can be unreliable for measuring pulmonary artery systolic and diastolic pressures because the length and compliance of the tubing affects the measuring system by A. reducing its resonant frequency B. reducing its frequency response C. reducing its damping coefficient D. inducing a zero error E. inducing baseline drift

ANSWER A

A diagnosis of pulmonary embolism is most strongly suggested by A. intraluminal filling defects or vascular cutoffs on angiography B. PaO2 less than 85 mmHg and an abnormal lung perfusion scan C. PaO2 less than 85 mmHg and an elevated PaCO2 D. right ventricular hypertrophy with right ventricular strain and right axis deviation on electrocardiography E. "unmatched" ventilation-perfusion defects

ANSWER A

A middle-aged woman, slightly jaundiced and with a spleen just palpable, has a Hb of 7G/dl. The blood film report reads as follows: "Erythrocytes show anisocytosis and range in size from microcytes to oval macrocytes, many showing poikilocytosis. They are normochromic. The platelets are diminished in number. There is a leucopenia and a slight neutropenia. The most likely diagnosis is: A. Pernicious anaemia B. Chronic blood loss C. Haemolytic anaemia D. Hepatitis A E. Carcinomatosis

ANSWER A

A naked neonate in an incubator (room temp 20C) loses heat mainly by: A. Radiation B. Conduction C. Convection D. Evaporation from skin E. Evaporation from airway

ANSWER A

A terrorist attack has taken place involving the nerve gas "VX". Some victims have arrived in the emergency department. The most appropriate management of this situation is to: A. Strip them off and hose them down B. Strip them off, scrub them with a brush, and hose them down C. Leave their clothes on and hose them down D. Leave their clothes on, scrub them with a brush, and hose them down E. Take them to the resuscitation area and put in an IV

ANSWER A

AZ77 ANZCA Version [Jul06] Q149 With regard to peri-operative cardiac risk reduction there is evidence that all of the following reduce perioperative cardiac morbidity EXCEPT A. intra-operative use of nitrous oxide B. peri-operative use of alpha-two agonists C. perioperative use of diltiazem D. peri-operative use of statins E. tight peri-operative glycaemic control (blood sugar between 3.5 and 6mmol.l-1)

ANSWER A

All of the following are major complications of massive transfusion, except: A. Hypokalemia B. Hypothermia C. Hypomagnesemia D. Hypocalcemia

ANSWER A

Amniotic fluid embolism. Cause of death in first half hour ? A. Pulmonary hypertension B. Malignant arrhythmia C. Pulmonary oedema D. Hypovolaemic shock E.

ANSWER A

An epidural in a healthy individual causes all EXCEPT A. Raised Co2 B. Bradycardia C. Vasodilation D. Dyspnea E. Tachycardia

ANSWER A

Atrial septal defect, where is the murmur heard the loadest? A. PV B. MV C. ASD D. AV E. TV

ANSWER A

Best position for IABP is 1-2 cm: A. Distal to Left SCA B. Proximal to Left SCA C. Distal to artery of Adamkiewicz D. Distal to renal artery E. Proximal to renal artery

ANSWER A

Best way to prevent hypothermia in patient undergoing a general anaesthetic A. Prewarming of patient B. Warming blanket C. Aluminium heat blackets D. Warm IV fluids E. Avoid muscle relaxants

ANSWER A

Blood flow across which of the following is used to estimate pulmonary artery pressures during echocardiography? A. Tricuspid valve B. Aortic Valve C. Pulmonary valve D. Mitral Valve E. Aortic Root

ANSWER A

Causes of postoperative hypoxaemia: A. Diminished tidal volume B. Posture C. Inspired oxygen concentration D. Hypercarbia

ANSWER A

Central anticholinergic syndrome, which is NOT true: A. Will improve with neostigmine B. Peripheral anticholinergic symptoms C. Caused by Anti-Parkinson drugs D. CNS depression E. Associated with agitation, delirium, and ???

ANSWER A

Child with 70% burns: A. Myoglobinuria & haemoglobinuria B. Anaemia, thrombocytopaenia & coagulopathy C. Anaemia, thrombocytosis & hypercoagulable D. Infrequently see hypertension late in the hospital stay

ANSWER A

Compared to unfractionated heparin, low molecular weight heparins (LMWH) A. have a higher bioavailability B. are more easily reversed with protamine C. have more effect on platelet function D. have a shorter biological half life

ANSWER A

Diastolic dysfunction is NOT caused by: A Adrenaline B Aortic stenosis C Hypertension D myocardial fibrosis E ?

ANSWER A

During cardiac catheterisation (?) patient become BP 80/60, HR 110, CVP 16. What is the next most important investigation A. Echocardiogram B. CXR C. Electrocardiogram D. CT chest E. Thallium scan

ANSWER A

During scoliosis surgery with monitoring of somatosensory evoked potentials, which tract are they mainly monitoring? A: Dorsal column B: Spinothalamic tract C: Lateral Corticospinal tract D: Cerebrospinal tract E: Anterior horn cells

ANSWER A

ECG- which does NOT have abnormal Q waves: A: Digoxin toxicity B: Anterior myocardial infartion C: Previous AMI D: LBBB E: Wolff-Parkinson-White syndrome

ANSWER A

Elimination Half life of tirofiban A. 2hrs B. 8hrs C. 12hrs D. 24hrs E. 15 minutes

ANSWER A

Emergency caesarean section for foetal distress (and foetal acidosis on scalp probe?). what is best option to raise gastric pH preop: A)Oral Na Citrate B)Ranitidine IV C)Ranitidine oral D)Omeprazole IV E)Omeprazole oral F)Metoclopramide 20 IV

ANSWER A

In hyperparathyroidism, the following findings are present: A. Increased serum and urinary calcium B. Urinary phosphate excretion is usually normal C. Serum phosphate is usually low or normal D. Increased serum but decreased urinary calcium E. None of the above

ANSWER A

In the management of torsades de pointes (polymorphic ventricular tachycardia), all the following drugs may be useful EXCEPT A. amiodarone B. isoprenaline C. [[lignocaine] D. magnesium E. phenytoin

ANSWER A

In the management of torsades de pointes (polymorphic ventricular tachycardia), all the following drugs may be useful EXCEPT A. amiodarone B. isoprenaline C. lignocaine D. magnesium E. phenytoin

ANSWER A

Indicates autonomic neuropathy except A. Sinus arrthymias B. Gastric reflux C. Postural hypotension D. Priaprism E. Constipation

ANSWER A

Iron deficiency anaemia: A. Low ferritin, low serum iron B. Low ferritin, low TIBC C. Elevated ferritin, low marrow iron D. Elevated ferritin, low TIBC E. Elevated ferritin, high serum iron

ANSWER A

Jehovah's witness patient refusing blood products. The ethical principle you are honouring if you continue with elective hip operation A. Autonomy B. Nonmaleficience C. Justice D. Paternalism

ANSWER A

Lower oesophageal sphincter tone is increased by A. suxamethonium B. pancuronium C. metoclopramide D. prochlorperazine E. ergometrine

ANSWER A

MR37 ANZCA version [2005-Apr] Q20 The most useful clinical finding to exclude the presence of airflow limitation in a 60 year old is A. a history of never smoking B. absence of hyperresonance to chest percussion C. normal cardiac dullness to percussion D. absence of pulsus paradoxus E. non-use of accessory muscles at rest

ANSWER A

Major cause of death following difficult intubation with perforated oesophagus? A. Sepsis B. Failure to intubate C. Failure to ventilate

ANSWER A

Most safe side to insert subtenon block A. Inferonasal B. Inferotemporal C. Medial D. Superonasal E. Superotemporal

ANSWER A

PL03 ANZCA version [1986] [1988] [Mar93] [2004-Aug] Q40, [2005-Apr] Q74 The difference in duration of action of lignocaine and bupivacaine is most marked with: A. Infiltration anaesthesia B. Carbonated solutions C. Peripheral nerve block D. Extradural anaesthesia E. Solutions containing adrenaline 1 in 200,000

ANSWER A

PS9 safe provision of anaesthesia for Colonoscopy: A. Medical Practitioner to providing sedation with a skilled assistant who is not assisting the proceduralist. B. Medical practitioner alone C. Specialist Anaesthetist D. Skilled nurse with airway experience E. Two skiled nurses with airway experience

ANSWER A

Paediatric ALS - 20 kg, VF has had 2 shocks only. Next step a. Adrenaline b. Amiodarone c. Shock 50J d. Shock 100J

ANSWER A

Patient with Marfan's and 2 hours of severe chest pain, mild hypertension and ECG showing ischaemia. The next best step is urgent: A: CT B: TOE C: ? D: Angiography and PCI E: Thrombolysis

ANSWER A

Patient with history of COAD and suspected pneumonia. Clinical findings supporting right-sided pneumonia on examination: A. R Dull percussion note & increased vocal resonance B. R Dull percussion note & decreased vocal resonance C. R Decreased air entry D. Tracheal deviation to left E. Tracheal deviation to right

ANSWER A

Patient with known severe aortic regurgitation. Auscultation reveals loud mid diastrolic murmur in aortic area. You also hear a quiet mid diastolic murmur in the apex. This is a. Functional mitral stenosis b. Mitral valve incompetence c. Impaired LV function d. Papillary muscle dysfunction

ANSWER A

Penetrating injury to chest. What part of the heart most likely injured? A. RV B. LV C. RA D. LA E. Right coronary

ANSWER A

Perform a brachial plexus block however the medial forearm is NOT numb. Which nerve has been missed? A. Medial antebrachial cutaneous nerve B. Medial brachial cutanous nerve C. Intercostal brachial nerve D. Axillary superior lateral cutaneous nerve

ANSWER A

Performing a caudal block in a child and add clonidine to prolong duration of block. What significant complication is increased? A. Sedation B. Urinary retention C. Bradycardia D. PONV E. Respiratory depression

ANSWER A

Perioperative ulnar neuropathy A. is more common in diabetics B. is more common in women C. is often associated with contralateral clinical neuropathy D. is usually found to be the result of excess external pressure E. usually presents within 24 hours

ANSWER A

Post CEA on ward, patient seizes. BP has been hard to control. What to do to prevent further seizures? A: Add another antihypertensive B: Start antiplatelet drugs C: Start anticonvulsants D: Do angio and stent E: Nimodipine

ANSWER A

Post Carotid Endarterectomy on ward, patient seizes. BP has been hard to control. What to do to prevent further seizures? A: Add another antihypertensive B: Start antiplatelet drugs C: Start anticonvulsants D: Do angio and stent E: Nimodipine

ANSWER A

Post cervical spine op, there is bulging noted under the incision site. Patient desaturated, combative, keep pulling off the oxygen facemask. Next course of action A. Rapid sequence induction B. Gas induction C. Needle aspiration of the bulge at the neck D. Sedation with precidex E. Olanzepine

ANSWER A

Post local anaesthetic block in difficult intubate patient- patient seizure. What would you give? A. Midazolam 5mg B. thiopentone C. propofol D. Suxamethonium

ANSWER A

Post partum foot drop is most frequently caused by A. compression of the lumbosacral trunk by the foetal head or forceps B. damage to the common peroneal nerve from lithotomy position C. damage to the conus medullaris by misplaced spinal anaesthesia D. L4 Nerve root damage from epidural analgesia E. the excessive lumbar lordosis of pregnancy stretching nerve roots

ANSWER A

Pre-eclamptic woman, BP 170/110, headache, proteinuria 1.2g. Which of the following NOT to use for control of her hypertension: A. Magnesium B. SNP C. GTN D. Hydralazine E. Metoprolol

ANSWER A

Pregnant patient seatbelt, driver- involved in car accident. Suddenly developed severe central chest pain, HR 110, BP 154/80, RR 26, Sat 100%. The most likely cause? A. Sternal fracture B. Aortic dissection C. Pneumothorax D. Rib fracture E. Myocardial infarction

ANSWER A

Pregnant woman presents with narrow complex tachycardia HR 190, stable BP 100/60. No response to vagal manoevures. Management? A. Adenosine 6mg B. DCR C. Amiodarone D. Atenolol E. ?

ANSWER A

Propofol for induction and maintenance of anaesthesia in healthy children, compared to healthy adults, has A. a larger central volume of distribution B. a lower total body clearance C. the same context sensitive half-time D. a more rapid awakening time E. a lower infusion rate requirement

ANSWER A

Reverse splitting of the second heart sound occurs with: A. LBBB B. Pulmonary hypertension C. Acute pulmonary embolus D. ASD E. Severe MR

ANSWER A

Risk factor for PPH? A. Prolonged labour B. Age <20 yrs old C. Primiparity D. FV Leiden Deficiency (yes it said deficiency!) E. Oligohydramnios

ANSWER A

SF84 [Apr07] Q112 Analgesic requirements during labour are reduced by each of the following except A. Acupressure B. Acupuncture C. Hypnosis D. One to one support by midwife E. TENS

ANSWER A

SG2. Called to ward for Postoperative thyroidectomy bleeding in ward. SpO2 92% on 6L, tachycardic and ?hypertensive and neck haematoma. What is the least appropriate management: a. call and arrange CT scan of his neck b. call OT and arrange urgent surgery c. release staples d. increase oxygen supply

ANSWER A

Severe asthma attack. Given continuous nebs & IV hydrocortisone but not responding. PaCO2 low. SpO2 low. Next appropriate treatment? A. IV Magnesium B. IV Aminophylline C. Heliox D. IV salbutamol infusion E. Intubate/ventilate

ANSWER A

Suxamethonium dosage higher in neonates compare to adult because A. Increased volume of distribution B. Increased pseudocholinesterase activity C. More receptors D. Higher cardiac output E. Decreased sensitivity of nicotinic ACH receptors to suxamethonium

ANSWER A

The cause of hypoxia in one lung ventilation A. Blood flow through non ventilated lung B. Impairment of hypoxic pulmonary vasoconstriction C. Ventilation perfusion mismatched (?)

ANSWER A

The first priority in managing a witnessed ventricular fibrillation cardiac arrest is A. defibrillation times 3 B. endotracheal intubation C. establishment of intravenous access D. external cardiac massage E. mask ventilation with 100% oxygen

ANSWER A

The intraoperative hypothermia for aneurysm surgery trial (IHAST) showed that cooling to a target temperature of 33°C A. did NOT improve neurological outcome in WFNS (World Federation of Neurosurgical Surgeons) in grade I-III patients B. did NOT improve neurological outcome in WFNS grade IV-V patients C. improved neurological outcome in WFNS grade I-III D. improved neurological outcome in WFNS grade III E. improved neurological outcome in WFNS grade IV-V

ANSWER A

The most common site of injury to the airway during anaesthesia is A. larynx B. oesophagus C. pharynx D. temporomandibular joint E. tongue

ANSWER A

The most correct statement regarding a 70 kg male who has sustained third degree burns to all of his right arm and second degree burns to the anterior aspect of his trunk is that A. he has burns to 21% of his body B. he will require 2 to 4 ml of Hartmann's solution per kilogram per percent burn in the first 8 hours after admission C. a normal blood gas and chest X-ray on admission will be useful in excluding inhalational injury D. prophylactic antibiotics and steroids are of proven benefit if he has suffered an inhalational injury E. alkalinizing the urine will prevent renal damage if the burns were electrical

ANSWER A

The most effective method for cerebral protection in aortic arch aneurysm repair A. Systemic hypothermia 20degrees B. Antegrade perfusion to carotid arteries C. Retrograde perfusion to jugular veins D. Thiopentone E. Steroid (?)

ANSWER A

The oculocardiac reflex is enhanced by: A. Traction on the medial rectus muscle B. Adrenaline injected into the lacrimal sac C. Muscle relaxation D. More common over twelve years of age

ANSWER A

The power of a statistical test is highest when: A. The expected difference between the 2 groups being compared is large B. The beta error is less than 5% C. The study groups are cohort & case controlled D. Depends on the alpha error & the number of subjects in each study group

ANSWER A

The test to diagnose pulmonary embolism A. CT pulmonary angiogram B. Echocardiogram C. Electrocardiogram D. Ventilation-perfusion scan

ANSWER A

The time constant of the lung is calculated by A. Compliance x resistance B. Compliance plus resistance C. Compliance /resistance D. Resistance/compliance E. Compliance x Compliance

ANSWER A

The use of large quantities of isotonic non-electrolyte solution for irrigation during prolonged transurethral resection of the prostate often results in A. hyponatraemia B. haemolysis C. cell shrinkage D. hyperkalaemia E. Hypotension

ANSWER A

To operate on the anterior 2/3rds of the ear you would need to block: A. Mandibular n B. Maxillary n C. Vagus n D. Greater auricular n

ANSWER A

Twenty-four hours following a vaginal hysterectomy, a 48-year-old obese female complains of severe pain that radiates down both buttocks and thighs. She had received a spinal anaesthetic with hyperbaric lignocaine. A likely explanation for these complaints includes all of the following EXCEPT A. a spinal abscess B. a spinal haematoma C. lumbar disc herniation D. transient neurological symptoms syndrome E. trauma due to improper positioning

ANSWER A

Twenty-four hours following a vaginal hysterectomy, a 48-year-old obese female complains of severe pain that radiates down both buttocks and thighs. She had received a spinal anaesthetic with hyperbaric lignocaine. The most likely explanation for these complaints is A. transient neurological symptoms syndrome B. lumbar disc herniation C. a spinal abscess D. trauma due to improper positioning E. a spinal haematoma

ANSWER A

What drug should NOT be used for tocolysis in 32/40 female? A. Indomethacin B. Magnesium C. Nifedipine D. Salbutamol

ANSWER A

What gestation to monitor uteroplacental flow A. 20 weeks B. 24 weeks C. 28 weeks D. 32 weeks E. 36 weeks

ANSWER A

What is the half life of clopidogrel? a. 6 hours b. 14 hours c. 24 hours d. 7 days

ANSWER A

What is the ratio of breaths to compressions in neonatal resuscitation A. 1:3 B. 1:5 C. 2:15 D. 2:30

ANSWER A

What makes tramadol less effective? A: ondansetron B: prochlorperazine C: metoclopramide D: paracetamol E. droperidol

ANSWER A

When optimising patients for surgery using goal-directed therapy, which of the following parameters is LEAST useful? A. blood pressure B. cardiac index C. oxygen saturation of blood aspirated from a central venous catheter D. oxygen saturation of blood aspirated from the distal port of a pulmonary artery catheter E. stroke volume variability

ANSWER A

When optimising patients for surgery using goal-directed therapy, which of the following parameters is LEAST useful? A. blood pressure B. cardiac index C. oxygen saturation of blood aspirated from a central venous catheter D. oxygen saturation of blood aspirated from the distal port of a pulmonary artery catheter E. stroke volume variability

ANSWER A

When using a T piece for a small child, which is not an advantage? A. Can use low gas flows B. Feel compliance C. Assess tidal volume D. Can rapidly change levels of CPAP E. low resistance

ANSWER A

When using a T piece for a small child, which is not an advantage? A. Can use low gas flows B. Feel compliance C. Assess tidal volume D. Can rapidly change levels of CPAP E. low resistance

ANSWER A

Where should the tip of an IABP lie A. 2cm distal to the left subclavian@ B. 2 cm proximal to the left subclavian C. 2cm proximal to the renal artery D. 2 cm distal to the renal artery

ANSWER A

Which is the first sign of deterioration in a patient with head injury? Change in: A. Level of consciousness B. Pupil size C. Pulse rate D. Respiratory rate

ANSWER A

Which new change on ECG is most likely to indicate acute ischaemia? A. 2mm horizontal ST depression in 60yr old male undergoing TURP with spinal B. 2mm ST depression in previously fit 20yr old female GA caesarean C. 0.7mm ST depression in a 60 yr old male undergoing TURP with spinal D. 2mm downsloping ST depression in 30yr old male craniotomy for SAH E. T-wave inversion in 60yr old male TURP under spinal

ANSWER A

Which of the following are feature of Conn's syndrome? A. Normoglycaemia, hypernatremia , hypokalemia B. Hypoglycaemia, hypernatremia, hypokalemia C. Hyperglycaemia, hyponatremia, hyperkalemia D. Normoglycaemia, hyponatremia, hyperkalemia E. Hypoglycaemia, hyponatremia, hyperkalemia

ANSWER A

Which of the following is NOT a feature of long-standing paraplegia above T6? A. Flaccidity of the leg muscles B. Poikilothermia C. Mass autonomic reflex D. Hyperkalaemia after Suxamethonium administration E. Labile blood pressure

ANSWER A

While performing a popliteal block, how do you know that you are stimulating the common peroneal nerve? A. dorsiflexion B. plantarflexion C. inversion D. ?

ANSWER A

You are performing a bronchoscopy, but are unsure of your location. Then you see trifurcation of bronchi. Most likely location is: A. Right upper lobe B. RML C. RLL D. LUL E. Lingula <br

ANSWER A

You see a patient in your clinic for a total knee replacement. He is 65 and has atrial fibrillation for which he takes dabigatran. He is otherwise well. A spinal anaesthetic is planned. What is the correct advice regarding his medication? a. he should stop his dabigatran 7 days prior b. he should stop his dabigatran 3 days prior c. he should stop his dabigatran 3 days prior and have bridging enoxaparan d. he should stop his dabigatran the day before and have an INR on the day of surgery e. he should continue to take his dabigatran until the morning of surgery

ANSWER A

Your registrar gives a Duchenne patient 1mg/kg of suxamethonium. What are you most worried about? A: hyperkalaemia B: rhabomyolysis C: MH

ANSWER A

young man in trauma, had been drinking,alcohol level >300. Multiple fractures. Initial lactate 10 then post fluid resus lactate 5. a. 2nd lactate more important than first for prognosis b. initial lactate high due to alcohol c. ? d. The initial lactate result carries a mortality exceeding 20% e. ?

ANSWER A

Which can deliver minute ventilation of greater than 5L/min using a 14 G cannula used for needle cricothyroidotomy A. jet ventilation using pressure 400KPA B. oxygen flush button on anaesthetic machine C. oxygen tubing on oxygen port on anaesthetic machine at 12L/min D. E. none of the above

ANSWER A The purpose of the study was to measure gas flow rates using different methods of transtracheal ventilation. Wall oxygen flow (WOF) at 10 and 15L/min, and a self-inflating ventilation bag (SIVB) were used to deliver gas flow through three transtracheal catheters: 13, 14, and 16 gauge (5 trials each). WOF mean gas flow rates (L/min) through the 16G, 14G, 13G catheters, respectively were: 15.7, 15.7, 16.8 at 15L/min, 10.5, 10.5, 10.3 at 10 L/min, and 5.7, 7.5, 7.7 via SIVB. SIVB gas flow was not continuous since it required the bag to reinflate, which reduces its calculated flow rate. A 500 cc tidal volume can be delivered within 3 seconds (WOF) and 5 seconds (SIVB). Catheter size did not substantially affect gas flow rates (Poiseuille's law not applicable). Transtracheal ventilation is best done by using WOF, but if a device to perform this is not available, then an SIVB may still be sufficient.

Day 4 epidural. On 40 mg SC enoxeparin daily postoperatively (8 pm). When is the most appropriate time to remove the epidural? A. Day 5 at 12 midday B. Day 5 at 6 am C. Day 5 at 6 pm D. Day 6 at ?

ANSWER A American Society of Regional Anaesthesia (ASRA) guidelines of neuraxial anaesthesia and anticoagulation "An indwelling epidural catheter should be removed 10-12 hours after the last dose of LMWH"

Acute visual loss after non-ocular surgery is most commonly caused by A. ischaemic optic neuropathy B. prolonged direct compression of the globe C. cortical blindness D. retinal artery occlusion E. electrolyte imbalance

ANSWER A Ischemic optic neuropathy is the most common diagnosis in postoperative visual loss. Ischemic optic neuropathy is divided into anterior and posterior, depending upon the location of the lesion on the optic nerve. Anterior ischemic optic neuropathy -53% cardiopulmonary bypass procedures -12% prone Posterior ischemic optic neuropathy -48% neck, nose or sinus operations -16% prone -11% cardiopulmonary bypass procedures

The most appropriate perioperative management of a patient with a history of oral oedema on kiwi fruit exposure is: (a) increased vigilance for latex allergy (b) perioperative steroids (c) preoperative RAST testing for latex allergy (d) preoperative skin testing for latex allergy (e) treatment in a latex free environment

ANSWER A Laws P (2008) The clinical implications of latex-fruit allergy Anaesthesia 63 (2), 211-212. doi:10.1111/j.1365-2044.2007.05425

The nerve supplying area of skin between greater trochanter and iliac crest: A. subcostal nerve B. ilioinguinal nerve C. genitofemoral nerve D. femoral nerve E. lat cutaneous femoral nerve.

ANSWER A Subcostal nerve The 12th thoracic (subcostal) nerve runs along the lower border of the 12th rib below the subcostal vessels, passes behind the lateral arcuate ligament to run in front of quadratus lumborum behind the kidney and colon. The nerve then passes between transversus abdominis and internal oblique and then has a course and distribution which are similar to the lower intercostal nerves. However, there is one point of difference: the lateral cutaneous branch of the 12th nerve descends without branching to supply the skin over the lateral aspect of the buttock * Lateral cutaneous br of subcostal (T12) and iliohypogastric (L1) supply skin over upper lateral buttocks ie between iliac crest and great troc. * Superior gluteal n (L4,5 S1) is a muscular branch supplying gluteus medius & minimus (of interest is that superior glurteal n and assoc superior gluteal vessels are the only structures to pass through upper compartment of GSF) * Lateral cutaneous nerve of thigh - ant br supplys ant lat skin of thigh down to knee, post br supplys skin of lat thigh from greater troc. to mid-thigh

PZ121 ANZCA Version [2006-Mar] Q114, [Jul06] Q68 In elderly patients each of the following statements is true EXCEPT A. antagonism of neuromuscular blockade with aniticholinesterases is less likely to be effective than in the younger patient B. atropine produces a lesser heart rate response than in younger patients C. ephedrine is less likely to be effective (at raising blood pressure) than in the younger patient D. MAC of all inhalational agents is reduced by 20 to 40% E. time of onset of neuromuscular blockade is prolonged due to a reduction in cardiac output

ANSWER A * A - Antagonism of neuromuscular blockade with anticholinesterase drugs tends to be similar to younger adults FALSE, hence is THE CORRECT ANSWER * B - This relatively poor response to b-stimulation also results in the reduced heart rate response to atropine TRUE * C - Ageing is associated with reduced b-receptor sensitivity, which results in a reduction in response to exogenous b-agonists TRUE * D - The MAC value of all inhalational anaesthetic agents is reduced by 20-40% from young adult values TRUE * E - regarding NDMR, time of onset ... prolonged because of a reduction in cardiac output TRUE

PZ114 ANZCA version [2004-Aug] Q121, [2005-Jul] Q137 [2006-Mar] Q91 Regarding glycoprotein IIb/IIIa antagonists, which of the following statements is INCORRECT? A. platelet dysfunction persists for over 48 hours after cessation of drug B. their effects may be monitored by use of a platelet turbidometry aggregometer C. they are known to cause severe thrombocytopaenia in some patients D. they are used to treat acute coronary syndromes E. they block fibrinogen binding to platelet glycoprotein IIb/IIIa receptors

ANSWER A * A - False (THUS THE CORRECT ANSWER). With regard to abciximab: "ReoPro remains in the circulation for 15 days or more in a platelet bound state, although platelet function recovers over the course of 48 hours." AND "As patients receiving abciximab experience platelet dysfunction for 12-24 h after termination of the infusion, it would seem prudent that surgery should be delayed for at least 12-24 h after abciximab if the patient's cardiac status permits. Surgery should be delayed for 4-6 h to reduce the risk for increased bleeding in patients who have received eptifibatide or tirofiban." Anesthesiology 2002; 96:1237-49 (P. Kam) BUT It has a plasma half-life of 20 minutes but remains bound in the circulation for up to 15 days with some residual activity." Peck, Hill & Williams, 2nd. ed., p.339 AND Table 27.6 in Stoelting 4th. ed., p.518 showing that abciximab has to be stopped 72 hours before surgery. * B - True. "The pharmacological effects of GP IIb/IIIa antagonists can be assessed in a number of different ways. The most widely used method is turbidometric aggregometry..." Scarborough et al., GP IIB/IIIa Antagonists. Circulation 1999;100;437-444. * C - True. "In 0.1-0.5% of patients, severe thrombocytopenia occurs after intravenous administration of glycoprotein IIb/IIIa inhibitors in the nonsurgical setting. Abciximab causes thrombocytopenia in 0.4-1.6% patients, and this can occur after a single dose. In trials with other glycoprotein IIb/IIIa antagonists, the incidence of thrombocytopenia is generally lower than 1%. Thrombocytopenia usually occurs within hours of exposure and resolves on discontinuation of the drug, rarely requiring platelet transfusion unless accompanied by active bleeding." Anesthesiology 2002; 96:1237-49 (P. Kam) * D - True. "The efficacy of the glycoprotein IIb/IIIa antagonists in the treatment of acute coronary syndromes (unstable angina and non-Q wave MI), MI, and percutaneous coronary interventions has been demonstrated in several multicenter trials." Anesthesiology 2002; 96:1237-49 (P. Kam) * E - True. "Eptifibatide (a smaller molecule compared with abciximab) binds to the glycoprotein IIb/IIIa receptor between the IIb and IIIa arms of the extracellular parts of the receptor and prevents binding of fibrinogen." AND "Tirofiban occupies a binding pocket on the glycoprotein IIb/IIIa receptor and competitively inhibits platelet aggregation mediated by fibrinogen and von Willebrand factor." Anesthesiology 2002; 96:1237-49 (P. Kam)

Correct statements regarding fondaparinux include each of the following EXCEPT A. it has a structure unrelated to heparin B. it is administered once daily C. it is a synthetic, selective Factor Xa inhibitor D. it is recommended for DVT prophylaxis in major orthopaedic surgery E. the dosage does NOT need to be adjusted for age and sex

ANSWER A * A - derived from the factor Xa-binding moeity of unfractionated heparain (FALSE, hence ANSWER) * B - single daily dose (TRUE) * C - "It is a synthetic and specific inhibitor of activated Factor X (Xa)" [1] (TRUE) * D - only licensed for DVT prophylaxis in orthopaedic surgery (TRUE) * E - implies standard dose for all (TRUE)

Patients taking a selective serotonin reuptake inhibitor (SSRI e.g. fluoxetine): A. should NOT be given high doses of tramadol B. can have their SSRI medication ceased without risk of withdrawal symptoms C. will have potentiation of the effect of direct acting adrenergic agonists D. are less sensitive to benzodiazepines than the general population E. will have the majority of active drug cleared from the body within 36 hours following discontinuation

ANSWER A * A : True : Seizures and serotonin syndrome have been reported in patients using tramadol. Some medications, including fluoxetine, are known to reduce the seizure threshold. The risk of seizures and serotonin syndrome may be enhanced when fluoxetine and tramadol therapy are combined (Prod Info Ultram(R), 2001). * B : False : Numerous case reports and several controlled studies document a withdrawal syndrome following discontinuation of selective serotonin reuptake inhibitors (SSRIs). * C : False: MAOI's potentiate effect of direct acting adrenergic agonsists * D : False * E : False : Elimination Half-Life (Fluoxetine) : 4 to 6 days, chronic administration

Essential diagnostic criteria on ECG for LBBB A. Loss of septal Q's in V5 and V6 B. RSR in V1 C. Large slurred S in V6 D. T-waves opposite to direction of QRS E. QRS duration minimum 0.2 s

ANSWER A * A TRUE * B FALSE o This represents RBBB * B FALSE o Also RBBB * D FALSE o An associated finding, not essential * E FALSE Diagnostic criteria for LBBB: * QRS duration of 120 ms * Dominant S wave in V1 * Broad monophasic R wave in lateral leads (I, aVL, V5-V6) * Absence of Q waves in lateral leads (I, V5-V6; small Q waves are still allowed in aVL) * Prolonged R wave peak time > 60ms in left precordial leads (V5-6) * About » o Authors o Affiliates o Awards o Archives o Analytics o Legal Disclaimer * Blog * Exams » o ACEM Primary o ACEM Fellowship o CICM Fellowship o Utopian College o ACTM Fellowship o Conference * Education » o Clinical Cases o Toxicology o History o Clinical Examination o Investigations [tests] o Procedures o Lecture Notes o Medicine 2.0 o Oslerisms * Resources » o Stuff We Read o Image DATABASE o Emergency Medicine Blogs o Podcasts o ANZ MedBloggers o International Emergency Medicine o Eponymous Fractures o Glossary of terms o Paediatrics o Medical Humour * ECG Library » o ECG Basics o ECG Clinical Cases o Cardiology Eponyms o A-Z by Diagnosis o ECG References o ECG Exam Template * Books » o On Call Principles o Emergency Medicine o Toxicology Handbook o Critical Care Drug Manual o EMA Journal o Reviews * November 1, 2011 LITFL » ECG Library » ECG Basics » Left Bundle Branch Block Left Bundle Branch Block LBBB@ 590x246 Left Bundle Branch Block Background * Normally the septum is activated from left to right, producing small Q waves in the lateral leads. * In LBBB, the normal direction of septal depolarisation is reversed (becomes right to left), as the impulse spreads first to the RV via the right bundle branch and then to the LV via the septum. * This sequence of activation extends the QRS duration to > 120 ms and eliminates the normal septal Q waves in the lateral leads. * The overall direction of depolarisation (from right to left) produces tall R waves in the lateral leads (I, V5-6) and deep S waves in the right precordial leads (V1-3), and usually leads to left axis deviation. * As the ventricles are activated sequentially (right, then left) rather than simultaneously, this produces a broad or notched ('M'-shaped) R wave in the lateral leads. LBBB M and W Left Bundle Branch Block Dominant S wave in V1 with broad, notched ('M'-shaped) R wave in V6 Diagnostic Criteria * QRS duration of 120 ms * Dominant S wave in V1 * Broad monophasic R wave in lateral leads (I, aVL, V5-V6) * Absence of Q waves in lateral leads (I, V5-V6; small Q waves are still allowed in aVL) * Prolonged R wave peak time > 60ms in left precordial leads (V5-6) Associated Features * Appropriate discordance: the ST segments and T waves always go in the opposite direction to the main vector of the QRS complex * Poor R wave progression in the chest leads * Left axis deviation QRS Morphology in the Lateral Leads The R wave in the lateral leads may be either: * 'M'-shaped * Notched * Monophasic * RS complex Causes * Aortic stenosis * Ischaemic heart disease * Hypertension * Dilated cardiomyopathy * Anterior MI * Primary degenerative disease (fibrosis) of the conducting system (Lenegre disease) * Hyperkalaemia * Digoxin toxicity Diagnostic Criteria RBBB 1. Broad QRS > 120 ms 2. RSR' pattern in V1-3 ('M-shaped' QRS complex) 3. Wide, slurred S wave in the lateral leads (I, aVL, V5-6 Associated Features * ST depression and T wave inversion in the right precordial leads (V1-3) Variations * Sometimes rather than an RSR' pattern in V1, there may be a broad monophasic R wave or a qR complex. Causes of RBBB # Right ventricular hypertrophy / cor pulmonale # Pulmonary embolus # Ischaemic heart disease # Rheumatic heart disease # Myocarditis or cardiomyopathy # Degenerative disease of the conduction system # Congenital heart disease (e.g. atrial septal defect)

Transfusion related acute lung injury (TRALI) A. can be caused by all homologous blood components, but particularly FFP (fresh frozen plasma) B. is associated with significantly elevated pulmonary artery pressure C. is the commonest cause of morbidity associated with blood transfusion D. should be treated with high dose steroids E. typically presents 24 hours following transfusion

ANSWER A * A- True(answer) -SHOT report describes 13 reactions as follows: 6 to FFP, 4 to platelets, 2 to packed cells and 1 to whole blood. The preponderance of reactions with FFP and platelets is thought to result from their 'high plasma component' * B- ? False patients at autopsy have significant pulmonary capillary dilation ie a disease of pulmonary plasma leakage see Kam article --Kevin07 03:31, 10 Dec 2008 (EST) * C- commonest cause of major morbidity * D- False * E- False 6 hours

In providing general anaesthesia for patients with congenital Long QT syndromes, which of the following statements is correct? A. a priming dose of a non-depolarising muscle relaxant protects against succinylcholine induced arrhythmias B. beta-blocking drugs are relatively contra-indicated C. low dose droperidol is safe D. pre-induction infusion of magnesium sulphate should NOT be used E. thiopentone leaves the QT interval unchanged

ANSWER A * A. a priming dose of a non-depolarising muscle relaxant protects against succinylcholine induced arrhythmias - true: o "The ideal muscle relaxant should avoid bradycardia, vagal stimulation, and potassium shifts. It should have little or no histamine release and be short acting to avoid the use of reversal agents. Accordingly, succinylcholine, because of its autonomic effects and potassium release, is far from ideal. It prolongs the QT interval in patients with c-LQTS unless pretreatment with a priming dose of tubocurarine is used." (Anesthesiology. 2005 Jan;102(1):204-10.) * B. beta-blocking drugs are relatively contra-indicated - false: o "The mainstay of treatment of congenital LQTS since 1975 has been ß-block. Schwarz reported a decrease in mortality from 71% in untreated historical controls to 6% in those treated" (Br J Anaesth 2003; 90: 349-66) * C. low dose droperidol is safe - probably false: o "Although the data that led to the 2001 black box warning resulted from case reports and surveillance data rather than scientific publications in medical journals, it seems prudent to avoid using droperidol in the perioperative setting in patients with c-LQTS." (Anesthesiology. 2005 Jan;102(1):204-10.) * D. pre-induction infusion of magnesium sulphate should NOT be used - false: o "Magnesium sulfate is the treatment of choice for prevention of recurrence in patients with drug-induced TdP even with normal magnesium concentrations." (Anesthesiology. 2005 Jan;102(1):204-10.) o "It has been suggested that pretreatment with magnesium (30 mg/kg) may be beneficial, even in patients with normal serum magnesium concentration. Magnesium is very unlikely to cause toxicity, and may block the inward sodium and potassium currents implicated in generating early after-depolarizations" (Anesthesiology. 2005 Jan;102(1):204-10.) * E. thiopentone leaves the QT interval unchanged - false o "Thiopental prolongs the QT interval in healthy patients. 45 Despite this effect, thiopental has been used safely in patients with c-LQTS" (Anesthesiology. 2005 Jan;102(1):204-10.) Long QT syndrome -congential disorder characterized prolongation of QT invertal on ECG -Propensity to ventricular tachyarrythmias which may leave to syncope, cardiac arrest and sudden death 30% of patients present with unexplained syncope or aborted sudden death -60% are symptomatic at time of diagnosis. -prevalence may be as high as 1 per 1100-3000. Diagnosis -QTc >440ms -QTc = QT/ sq root of RR interval Management -Beta blockade -anti-bradycardia pacing -implantation of cardioverter defibrillators -left cardiac sympathetic denervation are used in the treatment of these patients. -However, these treatment modalities do not guarantee the prevention of sudden cardiac death. Anaesthesia management Anaesthesia in patients carries a very high risk of malignant ventricular arrythmias, however despite beta blockade patients remain at risk. Patients should be on maintenance beta blocker therapy and this much be continued throughout the perioperative period, including IV replacement if enteral intake is not possible. Electrolytes must be normal (especially K+, Mg++ and Ca++) Intraoperative management should focus on prevention of excessive sympathetic activity and avoidance of factors that can prolong the QT interval. * Have pacing apparatus, defibrillator and drugs available. * Consider induction and maintenance with propofol (which may shorten the QTc interval) * Response to intubation must be adequately blunted * Avoid high intra-thoracic pressures and PEEP which simulates a Valsalva and can prolong the QT. * Extubate deep * Avoid reversal (see below) * Manage Torsades de pointes Drugs which prolong QTc Note that not all of these have been associated with Torsades de pointes. * Volatile anaesthetic agents all prolong the QT interval * Thiopentone * Atropine, Glycopyrrolate * Class Ia antiarrythmics (Disopyramide, Quinidine, Procainamide) * Class Ic antiarrythmics * Class III antiarrythmics * Butyrophenones (Droperidol, haloperidol) * Phennothiazine and atypical anti-psychotics * SSRIs * Macrolide antibiotics * Cisapride

The respiratory pattern of a patient sustaining an acute C5 spinal cord injury is characterised by A. a rapid respiratory rate B. arterial hypoxaemia C. chest wall immobility D. preservation of the cough E. preservation of the inspiratory force

ANSWER A * A. a rapid respiratory rate - probably most true: For those thinking the answer is B, B is controversial, and if you get B, you're definitely going to have A. Also if intercostal and abdominal muscles are affected, TV will decrease and so RR must increase. * B. arterial hypoxaemia - false although some would disagree * C. chest wall immobility - false: flaccid paralysis initially so chest will move passively; i.e. chest wall compliance decreases * D. preservation of the cough - false: abdominal muscles required to cough * E. preservation of the inspiratory force - false: inspiratory is active. Some muscles will be lost.

In beta-thalassaemia major A. blood transfusion prevents growth retardation in children. B. the red blood cells are hypochromic and macrocytic. C. the average age at death is 25-30 yrs. D. liver failure is the most common cause of death E. iron supplementation improves peri-operative haemoglobin levels.

ANSWER A * A. blood transfusion prevents growth retardation in children. - true: "The clinical benefits of hypertransfusion during the first decade of life are dramatic, with reductions in hepatosplenomegaly, partial correction of abnormal skeletal development, and at least short-term improvements in cardiac dilatation and systolic function." (Uptodate accessed 28/7/10) B. the red blood cells are hypochromic and macrocytic. FALSE : hypochromic, microcytic C. the average age at death is 25-30 yrs. D. liver failure is the most common cause of death : FALSE -leading cause of death is iron overload followed by sepsis E. iron supplementation improves peri-operative haemoglobin levels. FALSE Iron supplimentation does not improve haemoglobin levels as the problem is with globin synthesis not haem and the patient usually has too much iron from multiple transfusions. Thalassemia -inherited blood disorder characterised by reduced or absent synthesis of the alpha/beta chains of hemoglobin -severity depends on the degree of impaired globin synthesis Prevalence 1. Mediterranean (mainly b) 2. African (a and b) 3. Asian (mainly a) Diagnosis is conformed by haemoglobin electrophoresis and globin chain analysis Presenation -B-thal major : made at childhood with severe anaemia accompanied by massive ineffective erthropoeisis : splenomegaly, profound microcytosis, elevated elvels of HbF and HbA2. Treatment 1. Long-life blood transfusion to mainly Hct 25-30% which suppresses erythropoisis -results in iron overload and endocrine dysfunction a. glucose intolerance b. thyroid dysfunction c. delayed onset of puberty -also cardiac, liver and pancreas dysfuction 2. Splenectomy Prognosis -poor, death before age 20 -complications due to iron overload

In patients with advanced liver disease who receive muscle relaxants A. decreased plasma clearance of vecuronium and rocuronium leading to prolonged blockade is a consistent feature B. duration of action of mivacurium is NOT affected C. initial doses should be reduced as volume of distribution is decreased D. plasma clearance of atracurium and cisatracurium is increased E. when the desired level of block is achieved with the initial dose, subsequent recovery is comparable to patients without liver disease

ANSWER A * A. decreased plasma clearance of vecuronium and rocuronium leading to prolonged blockade is a consistent feature - best answer: o " Vecuronium and rocuronium, both steroid-based NMBs, have a prolonged elimination phase in severe liver disease." (doi:10.1093/bjaceaccp/mkp040 Continuing Education in Anaesthesia, Critical Care & Pain | Volume 10 Number 1 2010) * B. duration of action of mivacurium is NOT affected - false: o "The metabolism of succinylcholine may be slowed because of reduced pseudocholinesterase concentrations, but in practice this gives few problems." o "the duration of action of mivacurium is prolonged in those patients in whom liver disease was associated with decreased plasma cholinesterase activity" (Stoelting p.217 3rd Ed) * C. initial doses should be reduced as volume of distribution is decreased - false: o "There is an apparent resistance to non- depolarizing neuromuscular blockers (NMBs) in patients with liver disease, which may be due to an increased volume of distribution or to altered protein binding." * D. plasma clearance of atracurium and cisatracurium is increased - false: Why should clearance be increased? If anything will be decreased, but Hoffman elimination buffers this. o "Atracurium and cisatracurium are suitable NMBs as they do not rely on hepatic excretion. After prolonged administration, concentrations of laudanosine (a metabolite of both atracurium and cisatracurium with potential to cause seizures) are lower with cisatracurium than atracurium due to the higher potency of cisatracurium, although this is unlikely to be of clinical significance." * E. when the desired level of block is achieved with the initial dose, subsequent recovery is comparable to patients without liver disease

AZ67 ANZCA version [2003-Apr] Q90, [Jul06] Q47, [Apr07] The primary duty of an expert witness called by the defence is to the A. Court B. Defence Counsel C. defendent doctor D. medical profession E. patient taking legal action

ANSWER A * Be truthful * Be impartial * Must NOT have financial or other interest in the outcome of the case * Must NOT adopt a position of advocacy * Must NOT favour plaintiff or defendant o The ultimate test for accuracy and impartiality is a willingness to prepare testimony that could be presented unchanged for use either by the plaintiff or the defendant

The following statements regarding prolonged QT syndrome are true EXCEPT A. all patients with the syndrome have a prolonged QTc (QT interval corrected for heart rate) B. a QTc greater than 500 msec is considered prolonged C. exercise testing may provoke a prolongation of the QTc D. T and U wave abnormalities are common E. T wave alternans is pathognomonic of the syndrome

ANSWER A # A - False (therefore answer) --> "6% of patients with symptomatic LQTS have a normal QTc interval." # B - True --> "QTc interval of >440 ms is considered prolonged", therefore > 500 msec is definitely prolonged # C - True --> "Exercise testing of patients with LQTS may provoke prolongation of the QTc" # D - True --> "T wave and U wave abnormalities are common in LQTS" # E - True --> "A pathognomonic feature of LQTS is so-called T wave alternans, where there is beat-to-beat variation in T wave amplitude."

Regarding non-obstetric abdominal laparoscopic surgery during the second trimester of pregnancy A. carbon dioxide pneumoperitoneum induces foetal acidosis B. fetal heart rate is depressed if maternal intra-abdominal pressure reaches 12 mmHg C. mechanical ventilation during general anaesthesia should be used to maintain a maternal arterial PaCO2 of 40 mmHg D. premature labour is a common complication unless prophylactic tocolytics are used E. the risk of miscarriage or premature labour is NOT increased

ANSWER A # A - TRUE - "a trend toward increasing fetal acidosis during a 90- to 120-minute exposure to a CO2 pneumoperitoneum" # B - FALSE intraabdo pressures were kept 12-15 in most cases, and no fetal bradycardia # C - False - Should aim lower (eg 32mmHg) (Yao and Artusio) # D - ?? - Probably not best answer; seems quite common if the patient has appendicitis (Y and ) but prophylactic tocolytics are debated # E - False - Clearly increased risk of spontaneous abortion in 1st and 2nd trimester

Factors which do NOT contribute to the increased risk of aspiration pneumonitis during pregnancy include A. increased gastrin production B. a tendency for the stomach to be pushed up against the left diaphragm C. increased acidity of gastric secretion D. increased volume of gastric secretion E. decreased secretion of the hormone motilin

ANSWER A # A. increased gastrin production - true; "We confirmed that pregnant women have much greater and more acidic gastric contents than the nonpregnant patients preoperatively, and it is not because of serum gastrin concentration" (J Clin Anesth. 2005 Sep;17(6):451-5.) [this was after the question was posed obviously] # B. a tendency for the stomach to be pushed up against the left diaphragm - maybe false; the gravid uterus causing raised intraabdominal pressure is traditionally a factor contributing to aspiration risk # C. increased acidity of gastric secretion False, see A # D. increased volume of gastric secretion False, see A # E. decreased secretion of the hormone motilin Apparently this is false; * "Plasma motilin concentrations were measured in 37 women during the second and third trimester of pregnancy and one week after delivery. The mean plasma motilin concentrations, both fasting and after a glucose load and a mixed meal, were significantly (p less than 0.001) reduced during pregnancy, returning to the normal range one week post partum. Pregnancy appears to have a profound inhibitory effect on plasma motilin, and this may in part be responsible for the gastrointestinal hypomotility associated with pregnancy

Most likely change on CTG with anaesthesia for non-obstetric surgery at 32 wks: A. Loss of beat to beat variability B. No change C. Late decelerations D. Variable decelerationss E. Uterine contractions

ANSWER A # Normal CTG under GA = Loss of beat to beat variability, no decelerations. # Normal CTG under neuraxial block without sedation = No change

Obstructive sleep disorder in children A. is associated with pulmonary hypertension and dysfunction of left and right ventricles B. has obesity as a major risk factor C. is rarely seen in children less than 8 years old D. is four times more prevalent in boys than girls E. does NOT usually require tonsillectomy for its management

ANSWER A #A. The mechanism appears to be: * Hypoxaemia and Hypercapnoea secondary to OSA lead to development of pulmonary hyperetension * PHT results in right ventricular dysfunction, remodelling and hypertrophy * Progressive tricuspid regurgitation and movement of the intra-ventricular septum lead to left ventricular outflow tract obstruction * LVOT obstruction results in left ventricular dysfunction and pulmonary oedema, which only exacerbates the hypoxaemia # B. has obesity as a major risk factor - partly true: Definitely a risk factor in the fat kiddies, but is it a major risk factor? * "Obesity, a typical feature of adults with OSA, is uncommon in children but if observed, may be associated with reduced daytime activity. More typically these children have poor weight gain and small stature." (Anaesthesia 1998; 53:571-579) # C. is rarely seen in children less than 8 years old - false: "Children usually present between 3 and 7 years of age" (Anaesthesia 1998; 53:571-579) # D. is four times more prevalent in boys than girls - false: "In contrast with adult OSA, in children there is an equal prevalence of affected boys and girls" (Anaesthesia 1998; 53:571-579) # E. does NOT usually require tonsillectomy for its management - false: "In most cases the definitive treatment of children with OSA syndrome is surgical by adenotonsillectomy" (Anaesthesia 1998; 53:571-579)

Features of perfluorocarbon emulsions when used as blood substitutes include A. a dose dependent half-life B. a sigmoidal oxygen dissociation curve C. liver dysfunction, which is a common side-effect D. metabolism by the liver E. prolongation of the the bleeding time

ANSWER A * A. a dose dependent half-life - true o "After intravenous (i.v.) administration, perflubron (perfluoro-octyl bromide) is absorbed by the reticulo-endothelial (RES) system. This absorption determines its intravascular half-time, which is dose-dependent and is in the order of several hours." (Best Practice & Research Clinical Anaesthesiology Vol. 22, No. 1, pp. 63-80, 2008 doi:10.1016/j.bpa.2007.10.003) * B. a sigmoidal oxygen dissociation curve - false o " Unlike Hb solutions, the relationship between PaO 2 and PFC-transported O 2 is linear" * C. liver dysfunction, which is a common side-effect - false o "The side effects of perflubron are usually mild and without serious clinical consequences." * D. metabolism by the liver - false o See A. "In the RES, perflubron droplets are broken down then excreted back in to the bloodstream and transported to the lungs where they are exhaled, with- out any known metabolism." * E. prolongation of the the bleeding time - false o "Nevertheless, perflubron has no effect on coagulation tests, bleeding time and platelet aggregation."

Transfusion related acute lung injury (TRALI) A. can be caused by all homologous blood components, but particularly FFP (fresh frozen plasma) B. is associated with significantly elevated pulmonary artery pressure C. is the commonest cause of morbidity associated with blood transfusion D. should be treated with high dose steroids E. typically presents 24 hours following transfusion

ANSWER A -SHOT report describes 13 reactions as follows: 6 to FFP, 4 to platelets, 2 to packed cells and 1 to whole blood. The preponderance of reactions with FFP and platelets is thought to result from their 'high plasma component' TRALI is most commonly caused by a reaction to leukocyte antibodies present in the plasma component of blood products. These antibodies can activate granulocytes that cause plasma leakage into the lungs, resulting in acute pulmonary edema.

AZ81 ANZCA version [2005-Sep] Q129 The adverse outcomes resulting in the most frequent medicolegal claims against anaesthetists are A. dental damages from airway instrumentation B. eye injuries C. non-obstetric epidural complications D. obstetric epidural complications E. peripheral nerve injuries

ANSWER A 1. Cass NM. Medicolegal claims against anaesthetists: a 20 year study. Anaesth Intensive Care. 2004 Feb;32(1):47-58. PMID 15058121 2. Aders A, Aders H. Anaesthetic adverse incident reports: an Australian study of 1,231 outcomes. Anaesth Intensive Care. 2005 Jun;33(3):336-44. PMID 15973916

You are performing an awake nasal fibreoptic intubation and wish to topicalise the airway. Which nerves do you need to anaesthetise from proximal to distal.. A. Trigeminal, then glosspharyngeal, then vagal (different combinations of the nerves given in different orders) B. Facial, trigeminal, vagal C. Facial, trigeminal, glossopharyngeal D. Trigeminal, vagal, glossopharyngeal E. ?

ANSWER A 3 Sensory nerves 1. Terminal branches of the Opthalmic and Maxillary divisions of Trigeminal : Nasal cavity and turbinates 2. Glossopharyngeal (IX) : oropharynx and posterior 3rd of the tongue 3. Vagus SLN AND RLN : epiglottis and distal airway

Essential diagnostic criteria on ECG for LBBB A. Loss of septal Q's in V5 and V6 B. RSR in V1 C. Large slurred S in V6 D. T-waves opposite to direction of QRS E. QRS duration minimum 0.2 s

ANSWER A A Diagnostic criteria for LBBB: • 1) Total QRS duration >0.12 s. • 2) No secondary R wave in V1 to indicate RBBB. • 3) No septal q wave in V5, V6 or in leads further to the left (lead I and aVL in horizontal hearts).

Amniotic fluid embolism A. can be associated with a mortality rate of 80% B. has an incidence of 1 in 2000 pregnancies C. is an uncommon cause of peripartum death D. is associated with a small chance of complications in survivors E. only presents during labour or caesarean section

ANSWER A A - True - Mortality approaches 80%, 90% have cardiac arrest. However, probably due improved vigilance, medical care and inclusion of less severe cases, mortality is now reported as 20-40% B - False - 1:15,000 to 1:50,000 Live Births C - False - 3rd most common in UK and most common cause of direct maternal deaths in Australia, check out http://www.npsu.unsw.edu.au/NPSUweb.nsf/page/md2 ............... or more recent [1] (http://www.npsu.unsw.edu.au/NPSUweb.nsf/page/md3) D - False - neonate 20% mortality, 50% with neurological deficit E - False - Usually occurs during labor but has occurred during abortion, abdominal trauma, and amnioinfusion.

An INCORRECT statement regarding patent foramen ovale (PFO) in adults is that: A. it is found in 5% of patients at postmortem examination B. it can cause hypoxaemia C. it is best seen using transoesphageal echocardiography D. an asymptomatic PFO requires no intervention E. the sitting position should be avoided in patients with a PFO

ANSWER A A : False - 30% have a probe PFO on autopsy, and in vivo, 20% can detect on sensitive tests e.g colour Doppler B : True - minor R-> L shunt occurs commonly with a large ASD -> mild decrease in SpO2 to 93-95%, and any increase in right sided pressure (e.g Valsalva, PE, volume overload) -> shunt C : True - "Transesophageal contrast echocardiography provides superior visualization of the atrial septum and therefore is preferred to transthoracic contrast echocardiography for detecting patent foramen ovale." EMedicine D : True - "Most patients with a patent foramen ovale as an isolated finding receive no special treatment" EMedicine E : True a relative C/I as risk of air embolism is high -> risk of paradoxical embolism with a PFO,

Which of the following is likely to be caused by water intoxication during transurethral resection of the prostate? A. Hypertension B. Nausea C. Confusion D. Headache E. Convulsions

ANSWER A A : May occur but hypotension is more common: Volume overload -> HT and bradycardia but also CCF and pulmonary oedema depending obviously on volume absorbed and the heart's ability to cope with the volume load (also if Na falls rapidly < 120 , then a negative inotrope) B : True C : True dilutional hyponatraenia -> cereberal oedema D : True E : True

A. in the presence of oliguria is a good indicator of renal function B. will increase more than specific gravity with an osmotic dieresis due to glucose C. is measured by elevation of the freezing point D. is expressed in milliosmol.l-1 E. in severely ill patients gives an indication of the effectiveness of frusemide

ANSWER A A : True ATN leads to tubular dysfunction and inability to conc. urine B : False disproportionate increase in specific gravity compared to osmolality if larger molecules (e.g glucose), are present (i.e in glucosuria in uncontrolled DM, and the administration of radiocontrast media) (Up to Date) C : False depression D : False mOsmol/kg E : False

AM15a [Mar90] Myasthenia gravis: which is NOT true? A. Increased incidence in males compared to females B. Increased response to muscle relaxants C. Aggravated by pregnancy D. .....?...IgM......? E. Neonates may be born with muscle weakness

ANSWER A A = false, more common in females cf myasthenic syndrome(pg 527 Stoelting and Dierdorf) B = true (pg 527 Stoelting and Dierdorf) C = true (muscle weakness may be precipitated or aggravated by; infections, electrolyte abnormalities, pregnancy, emotional stress and surgery)(pg 524 S&D) D ? E = true

In congenital diaphragmatic hernia A. there is hyperplasia of pulmonary arterioles in the hypoplastic lung B. right-sided lesions are more pathologically significant C. vasodilator drugs are contraindicated D. right-sided lesions through the foramen of Bochdalek are the most common E. intrapulmonary shunts are the major cause of cyanosis

ANSWER A A TRUE B. FALSE. Left sides lesions are larger and associated with more pulmonary hypoplasia C. FALSE D. FALSE: 90% are left posteriorlateral E. FALSE extrapulmonary Right to Left shunting is more common due to the pulmonary hypertension, and consequent shunting via a patent DA.

Residual Current Devices (also known as Safety Switches) A. detect differences in current between the active and neutral wires of a circuit B. isolate the patient from earth C. monitor the isolation of the power line D. must be fitted to all Cardiac Protection areas E. must be fitted to all operating theatres

ANSWER A A is BEST answer B False - Need to use RCD or line isolation C False - RCD or line isolation D False E False - RCD or LIM (LIM for things that are unsafe to turn off - like ventilators...)

AM30 ANZCA version [2003-Aug] Q92 Which of the following myopathies is associated with an increased risk of malignant hyperpyrexia? A. central core disease B. centronuclear myopathy C. fingerprint body myopathy D. multicore myopathy E. nemaline myopathy

ANSWER A A whole series of articles appears in A&A October 2009 which states that the following diseases could be MHS * Central Core Disease * Multiminicore Diease and Nemaline rod disease both having forms associated with RYR1 Mutation are considered high risk, despite the lack of case reports. (Apparently no one had used volatiles in patients know to have MmD!) * Brody Myopathy * King-Denborough Syndrome * Hypokalaemic Periodic Paralysis is possible * Burkitt's Lymphoma * Osteogenesis imperfecta * Myotonia congenita * NMS * Myelomeningocele

A widened mediastinum after a chest injury is suggestive of a ruptured aorta especially if associated with A. displacement of the mid-oesophagus to the right B. right haemothorax C. elevated left main bronchus D. fractured left third rib E. left pneumothorax

ANSWER A A widened mediastinum ALWAYS arouses suspicion of a ruptured aorta. In one series, mediastinal width greater than 8cm at the level of the aortic arch was present in all 10 patients with ruptured thoracic aorta. Diagnosis is enhanced by ONE or more of the following radiological features: * Left haemothorax * Depressed left main bronchus * Blurred outline of the arch or descending aorta * Fractured first rib or left apical haematoma * Displacement of the mid-oesophagus to the right (easily detectable with a nasogastric tube in situ)

Regarding Epidural Abcess - which is WRONG A. Diagnosis is DEPENDENT on triad of back pain, fever, and paralysis B. Occurs at a rate of 1:1000-3000 (OR 1:2000 - 1:5000) C. Worse outcomes if advanced age D. Usually gram positive cocci E. Expectant management may be appropriate

ANSWER A A. Diagnosis is DEPENDENT on triad of back pain, fever, and paralysis - false and the ANSWER to CHOOSE: "The early signs and symptoms may be vague, the 'classic' triad of back pain, fever and variable neurological deficit occurred in only 13% of patients by the time of diagnosis, and contributed to diagnostic delay in 75%. B. Occurs at a rate of 1:1000-3000 (OR 1:2000 - 1:5000) - true: "Estimating the true incidence of a rare complication from such disparate reports is not easy, but there is some suggestion that it might be of the order of 1 in 1000 in surgical, and 1 in 2000 in obstetric, patients. C. Worse outcomes if advanced age - true: With every decade increase in age, the likelihood of poor outcome doubled, presumably due to declining health and, possibly, reduced 'plasticity' of the spinal cord. D. Usually gram positive cocci - true: "In the 'developed' world the organisms most frequently encountered are Staphylococcus aureus (57-93% of cases), Streptococci (18%) and a variety of Gram-negative bacilli (13%). E. Expectant management may be appropriate - true: "It might be assumed that every patient with an epidural abscess should undergo surgery, but 11% of those identified in a major review did not, and another report identified 38 such individuals in case series and reports published between 1970 and 1990... The neurological deficit was unchanged or improved in all these patients except two, who died from sepsis syndrome, suggesting that the results of medical and surgical treatment are equivalent

MR47 [Mar06] [Apr07] Unequal consolidation on a CXR can be caused by all except: A. pulmonary oedema B. pneumonia C. pulmonary haemorrhage D. pulmonary infarction E. pulmonary effusion

ANSWER A A. FALSE : strictly is not a form of consolidation The remainder are causes of unilateral consolidation.

Regarding sampling blood for elevated histamine and mast cell tryptase (β−tryptase) levels to confirm a possible anaphylactic reaction, which of the following statements is INCORRECT? A. a sample for histamine can be taken up to two hours after the event B. a sample for tryptase can be taken up to six hours after the event C. a sample for tryptase can be taken up to two days after fatal anaphylaxis D. tryptase levels can be elevated after amniotic fluid embolism E. tryptase levels can be normal after clinical anaphylaxis

ANSWER A A. Histmine peaks at 5-15 minutes and returns to baseline in 30 minutes Causes of a raised tryptase: * Anaphylaxis * Asthma/Allergic rhinitis * Amniotic Fluid Embolism * SIDS * Systemic mastocytosis * Hyper-eosinophic syndrome

Femoral artery cannulation for arterial pressure monitoring A. is associated with more major complications than radial artery cannulation B. is frequently unreliable in the immediate post-bypass period C. should be performed with an approach from above the inguinal ligament D. should be performed with a large bore cannula to avoid erosion of the vessel wall E. should not be used for over 24 hours because of the risk of infection

ANSWER A A. Is associated with more major complications than radial artery cannulation - true Femoral has more MAJOR complications -permanent ischaemic damage, fem 0.18% cf rad 0.09% -sepsis fem 0.44% cf rad 0.13% -pseudoaneurysm fem 0.3% cf 0.09% rad) Complications of direct arterial pressure monitoring (Miller) 1. Distal ischemia, pseudoaneurysm, arteriovenous fistula 2. Hemorrhage 3. Arterial embolization 4. Infection 5. Peripheral neuropathy 6. Misinterpretation of data 7. Misuse of equipment B. Is frequently unreliable in the immediate post bypass period - false C. Should be performed with an approach from above the inguinal ligament - false: o It is the external iliac above the ligament D. Should be performed with a large bore cannula to avoid erosion of the vessel wall - depends on the def'n of large bore, likely false o Definitely won't be sticking a 16G cannula in E. Should not be used for over 24 hrs because of the risk of infection - false o Happens all the time in ICU

When investigating a patient following anaphylaxis during anaesthesia, the test most widely applicable and least likely to produce false positive results, in identifying the responsible drug or drugs is: A. skin prick testing B. I125 radiommunoassay (RIA) to specific circulating IgE C. RIA inhibition D. intradermal skin testing E. assessment of basophil degranulation and histamine release from leukocytes

ANSWER A A. TRUE Skin prick testing remains the first choice for detection of reactivity. It has advantages of high sensitivity and specificity, rapid results, flexibility, low cost, good tolerability. Dermis is pricked with sterile lancets with allergen extract and negative controls - observe for >3mm wheal and flare as mast cells in dermis release histamine (specific IgE to allergen) The presence of IgE antibody does not prove that the patient is clinically reactive, this needs to be correlated with history. There is small risk for precipitating anaphylaxis B. FALSE : RAST, although used, it is not most widely used due to high cost and long turn around time. Radioallergosorbent Test In vitro test, where radiolabelled antigens react with specific IgE antibodies. It is used to confirm circulating IgE antigen from Skin Prick Tests. But does not prove IgE cross-linking (Type 1 anaphylaxis) C. FALSE D. FALSE : similar sensitivity and specificity to Skin Testing, but carries higher risk of precipitating anaphylaxis. E. FALSE : Leucocyte histamine release is available only in specialised labs and has 20% false -ves. Histamine levels need to be taken within 10-30 minutes and only indicate degranulation, not identify drug

PDPH. a. IV caffeine treatment used to relieve symptoms. b. Is usually frontal headache c. Bed rest for 24 hrs is beneficial d. no use if blood patch done after 48 hrs. e. usually manifests within first 4 hrs.

ANSWER A A. TRUE B. FALSE : occipital to frontal C. FALSE : no benefit in preventing or treating PDPH, but symptomatic relief D. FALSE E. FALSE : usually 24-36 hours

The hyperglycaemic hyperosmolar non-ketotic syndrome A. may be associated with a high serum sodium B. only occurs in elderly patients C. is commonly complicated by thromboembolism D. is characterized by only a moderate degree of hyperglycaemia E. is NOT characterized by dehydration

ANSWER A A. TRUE : -mild cases : less dehydration, dilutional hyponatraemia -as dehydration worsens hypernatraemia compounds the hyperosmolality B .FALSE : middle aged to elderly C. FALSE : potential complication but not common D. FALSE E. FALSE Complications: * Ischemia or infarction to any organ, including heart and brain * Thromboembolism * ARDS/DIC or multiorgan dysfunction syndrome * Cerebral edema (rare) Prognosis: * The overall mortality rate is between 10-20% and is dependent on coexisting conditions and complications."

AZ78 ANZCA Version [2006-Mar] Q124 [Jul07] A 25-year-old 80 kg male with no other health problems is undergoing ECT (electroconvulsive shock therapy) for severe depression. Anaesthesia for his first 2 treatments consisted of thiopentone 350 mg and suxamethonium 50 mg. The treating psychiatrist is concerned at the limited duration of seizure activity with treatment despite maximal seizure stimulus. An acceptable seizure duration would be best be achieved by: A. adjunctive use of remifentanil to reduce the dose of induction agent B. clonidine premedication C. hypoventilating the patient to reduce seizure threshold D. pretreatment with lignocaine to reduce seizure threshold E. using propofol instead of thiopentone for induction of anaesthesia

ANSWER A A. TRUE : Anaesthesia Secrets' p513: Remifentanil 1 mcg/kg (makes it) possible to reduce the doses of methohexitone and propofol required to induce unconsciousness by 33% thus resulting in prolongation of ECT induced seizure duration. Adjunctive use of potent rapid and short acting opioid analgesics has become an effective way of extending siezure times in patients with short seizure duration.

Drugs that may be used for the management of heart failure, secondary to dilated cardiomyopathy in pregnancy, include each of the following EXCEPT A. ACE (angiotensin-converting enzyme) inhibitor B. beta-blockers C. digoxin D. loop diuretics E. nitrates

ANSWER A A. TRUE : Category D B. FALSE : labetolol is routinely used metoprolol and esmolol is preferred as it is beta selective -reserved for later pregnancy -caused IUGR, bradycardias and hypoglycaemia C. FALSE : Cat A D. FALSE : Cat C E. FALSE : Cat B2

A low ejection fraction is most likely to be seen in a patient with A. poor left ventricular (LV) systolic function B. poor LV systolic function and severe mitral regurgitation C. poor LV diastolic function D. septic shock E. a post myocardial infarct ventricular septal defect

ANSWER A A. TRUE : EF = (EDV-ESV) / EDV, as EDV will be normal, but ESV will be high, EF is low B. FALSE : MR will decrease ESV as blood will flow retrograde to atria, therefore EF will be high C. FALSE : EDV high with ESD normal, therefore EF will be high D. FALSE : high output state E. FALSE : same as B

AZ51 ANZCA version [2001-Apr] Q33, [2003-Apr] Q21, [2003-Aug] Q46 (A-type with only 4 options) [2006-Mar] Q110-similar question but with 5 options During anaesthesia for Magnetic Resonance Imaging, A. any ferromagnetic items outside the 30 Gauss line will not be strongly attracted to the magnetic core B. any battery-powered monitor of non-ferromagnetic construction can be safely placed close to the magnetic core without risk of attraction to the magnet C. the most commonly used MRI contrast medium, gadopentetate dimeglumide (Magnevist) induces a similar incidence of side effects as does the iodinated contrast media used in radiology D. it is essential to limit the use of halogenated agents as halogen atoms exhibit the property of nuclear magnetic resonance and may interfere with image quality E. permanent pacemakers can only be permitted if they have been completely disabled prior to the MRI (2006 March paper)

ANSWER A A. TRUE : Ferromagnetic items must be kept outside the 30-50 Gauss lines5 or, ideally, excluded from the MRI suite. B. FALSE : most batteries are ferromagnetic in construct, even though the monitor maybe non ferromagnetic C. FALSE : The most common agent, dimeglumine gadopentetate (Magnevist, Schering Pty Ltd), has a high therapeutic index compared to iodinated agents. D. FALSE E. FALSE : The risks of scanning patients with permanent pacemakers or ICDs are related to possible movement of the device, programming changes, asynchronous pacing, activation of tachyarrhythmia therapies, inhibition of pacing output, and induced currents in lead wires leading to heating and/or cardiac stimulation. Despite reports that some patients with pacemakers or ICDs have been scanned safely, the presence of a pacemaker or ICD is still considered a strong relative contraindication to routine MR examination.

A thoracic epidural inserted for pain relief: A. Allows earlier return of bowel function B. Prevents wasting of total body protein C. Does NOT reduce the incidence of MI D. Epidural opioids alone provide better analgesia than systemic opioids alone E. Addition of adrenaline significantly reduces local anesthetic dose requirement

ANSWER A A. TRUE : Level 1 evidence for thoracic but not for lumbar B. FALSE : Level 1 evidence that thoracic epidurals AND nutritional support C. FALSE : High thoracic epidural analgesia used for coronary artery bypass graft surgery reduces postoperative pain, risk of dysrhythmias, pulmonary complications and time to extubation when compared with IV opioid analgesia D. FALSE : For all types of surgery, epidural analgesia provides better postoperative pain relief compared with parenteral (including PCA) opioid administration (S) (Level I [Cochrane review]); EXCEPT epidural analgesia using a hydrophilic opioid only (N) (Level I). E. FALSE : the addition of adrenaline improves analgesia, prolongs motor and sensory block, but has no documented effect in reducing local anaesthetic dose requirement

With regard to peri-operative cardiac risk reduction there is evidence that all of the following reduce perioperative cardiac morbidity EXCEPT A. intra-operative use of nitrous oxide B. peri-operative use of alpha-two agonists C. perioperative use of diltiazem D. peri-operative use of statins E. tight peri-operative glycaemic control (blood sugar between 3.5 and 6mmol.l-1)

ANSWER A A. TRUE : N2O has limited potency, used as adjunct with volatile agents or TIVA. -N2O interferes with vitamin B12 and folate metabolism -impairs production of methionine (from homocysteine), -megaloblastic anaemia, immunosuppression, myocardial ischaemia, nerve and spinal cord toxicity, carcinogenicity, teratogenicity (esp. with occupational exposure); as well as postoperative nausea and vomiting, and increased risk of hypoxia. -ENIGMA trialpatients receiving N2O had a significant increased risk of postoperative wound infection, severe vomiting, pneumothorax or atelectasis, and pneumonia. Additionally nitrous oxide patients recorded lower QoR scores, and for ICU patients (but not others), a greater length of stay. There was a nonsignificant reduction in AMI, mortality and awareness. B. FALSE : large meta analysis, highlighted 31 RCT's, poor as only 6 studies clearly reported blinding and randomization, showed a reduction in mortality, non fatal MI, but also hypotension and bradycardia C. FALSE : meta analysis of 11 studies shows a reduction in myocardial ischemia and SVT. And a tread towards reduction in mortality. Mainly observed with diltiazem. Dihydropiridone and verapramil has no effect. D. FALSE : Plaque stablization, oxidative stress reduction and decreased vascular inflammation. E. FALSE : shown to reduce wound infection

A 33-year-old chronically spinally injured patient becomes hypertensive and sweaty during general anaesthesia for urinary sphincterotomy. His level of spinal cord injury is T4 and it is complete. You consider the diagnosis of autonomic hyperreflexia. Autonomic hyperreflexia A. could have been prevented by performing subarachnoid anaesthesia B. is unlikely with a T4 lesion C. should be treated by administration of a beta-blocker D. should be treated by administration of an opioid analgesia E. will resolve once the surgical stimulus ceases

ANSWER A A. TRUE : Spinal anaesthesia.can reliably prevent autonomic dysreflexia and spasm B. FALSE : Increased sensitivity of sympathetic reflexes in patients with spinal cord injury above T5/6. Cutaneous or visceral stimuli below the level of the lsion may result in mass discharge of sympathetic nerves, causing sweating, vasoconstriction and hypertension, with high levels of circulating catecholamines. Baroreceptor stimulation results in compensatory bradycardia. Distension of hollow viscera, especially the bladder, is a potent stimulus. It may occur during abdominal surgery and labour. Onset of susceptibility is usually within a few weeks of injury. C. FALSE : Where autonomic dysreflexia occurs during surgery, management should begin with removal of the precipitating stimulus, if possible. Most episodes appear to be brief and self-limiting and in 21 of our 43 cases no specific treatment was given. Dysreflexia occurring under general anaesthesia is best treated with increasing anaesthetic depth in the first instance. Where drug therapy is required, nifedipine and labetalol are used most commonly, though propranolol, midazolam, spinal anaesthesia and transdermal glyceryl trinitrate patches were all used on survey patients. D. FALSE : Nifedipine (10mg SL) or GTN (SL or TD) or alpha-blockers are all used in first-line therapy" E. FALSE Window -3 weeks to 12 years Stop stimulation Deepen Anaesthesia Nifed/GTN/labetolol consider phentalamine

When considering perioperative hypothermia A. the brain and major abdominal organs generate more metabolic heat than resting muscle B. the pattern of intra-operative hypothermia has 2 phases C. induction of general anaesthesia leads to a core temperature drop of < 0.5oC within the first hour D. the extent of redistribution hypothermia is primarily related to patient initial body heat content E. conduction contributes most to heat loss during the linear phase of hypothermia

ANSWER A A. TRUE : The brain and major organs in the trunk are the most metabolically active tissues and generate more metabolic heat than does muscle at rest B. FALSE : 3 phases C. FALSE : A quantitative study of systemic heat balance and regional body heat distribution in volunteers indicated that after 1 h of anesthesia, core temperature decreased 1.6°C, with redistribution contributing 81% to the decrease. D. FALSE : the degree of effect of redistribution is related to the initial difference between core and peripheral temperatures E. FALSE : radiation is 60% heat loss linear phase

Regarding venous air embolism (VAE) during posterior fossa neurosurgery A. attempts to aspirate air from the caval-atrial junction are of little value B. children are at greater risk than adults C. Doppler ultrasonography is the most specific monitor available D. end-tidal nitrogen monitoring is the most sensitive monitor available E. positive end-expiratory pressure (PEEP) should be used in patients with intra-cardiac shunts

ANSWER A A. TRUE : air has been successfuly aspirated form the right ventricle via central lines B. FALSE : children are at no greater risk, however the effect will be much worse given their smaller size C. FALSE : TOE is most sensitive and ETN2 is most specific D. FALSE : ETN2 most specific E. FALSE : PEEP will increase the right sided pressures, increasing the risk of R>L shunt and therefore PAE

At the wrist joint A. The median nerve lies between the tendons of palmaris longus and flexor carpi radialis B. The median nerve lies medial to the tendon of flexor carpi ulnaris C. The radial artery is usually palpable just medial to the tendon of flexor carpi radialis D. The ulnar artery and nerve enter the hand by passing deep to the flexor retinaculum

ANSWER A A. TRUE : at the wrist, the median nerve lies between the tendons of flexor carpi radialis (lateral) and palmaris longus (median) + flexor digitialis superficialis B. FALSE : very well lateral C. FALSE : The radial artery lies medial to BRACHIORADIALIS but lateral to FCR. D. FALSE Contents of the flexor retinaculum 1. 9 flexor tendons * flexor digitorum profundus (four tendons) * flexor digitorum superficialis (four tendons) * flexor pollicis longus (one tendon) 2. 1 nerve : median nerve Structures which pass superficially to flexor retinaculum 1. Palmaris longus 2. Ulnar artery 3. Ulnar nerve 4. Palmar cutaneous branch of the ulnar nerve 5. Palmar cutaneous branch of the median nerve

Sickle cell disease is: A. Homozygous recessive B. Confined to black Africans C. Autosomal dominant D. NOT contraindication to Bier's block E. Associated with haemoglobin A

ANSWER A A. TRUE : autsomal recessive B. FALSE : common in africans C. FALSE : autosomal recessive D. FALSE : IBRA and tourniquests are contraindicated E. FALSE : HbS or HbC

Regarding a 75-year-old female patient with moderate aortic stenosis presenting for an elective hip replacement, A. atrial systole has an increased contribution to stroke volume (compared to a patient with no aortic stenosis) B. beta-blockers are poorly tolerated in this degree of aortic stenosis C. hypotension is better tolerated than hypertension D. rheumatic heart disease is the commonest aetiology in this age group in Western Society E. single shot spinal anaesthesia is the preferred method of anaesthesia

ANSWER A A. TRUE : hypertrophied ventricle is stiff, causing diastolic dysfunction and reduced compliance. . Consequently, left ventricular filling becomes dependent on atrial contraction with atrial systole contributing 40%, instead of the usual 20%, of left ventricular end-diastolic volume. The left atrium hypertrophies to maintain left ventricular filling. Preservation of sinus rhythm becomes vital for the maintenance of cardiac output. B. FALSE : LV is dependent on preload and filling pressures, a normal to high HR is preferred. Tachy and bradycardia should be avoided C. FALSE : Avoidance of systemic hypotension is essential. Hypotension leads to myocardial ischaemia and a downward spiral of reduced contractility causing further falls in blood pressure and coronary perfusion. D. FALSE : Degenerative calcific aortic stenosis: This is now the most common form of aortic stenosis in the UK and tends to occur over the age of 70 yr. E. FALSE : Anaesthetic techniques that reduce systemic vascular resistance (e.g. regional neuroaxial techniques) must be used with extreme caution although successful cases of carefully titrated epidural and spinal blocks using catheters have been reported. Slow titration of epidural anaesthesia may be done with caution.

The Standard Error of the Mean is best described as: A. The Standard Deviation of sample means B. An indication of the likelihood of making a type II error C. Inversely proportional to statistical power D. Directly proportional to sample size E. Dependent for its validity on a normal distribution in the population

ANSWER A A. TRUE The Standard deviation of sample means - Standard error is also known as the standard error of the mean. If one takes a number of samples from a population, we will have a mean for each sample. The SD of the sample means is the standard error Standard deviation is a measure of spread about a central value. B. FALSE An indication of the likelihood of making a type II error - standard error is used to calculate confidence intervals, and so is a measure of precision Beta Error/Type II error occurs when one accepts the H0 incorrectly and the probability of this occurring is termed beta. Alpha Error/Type I Error : occurs when one incorrectly rejects the null hypothesis. The beta and alpha are normally pre-determined before a trial to calulate the sample size required to obtain a statistically significant outcome c.f. the confidence interval which is a measure of the precision of the sampling in relation to the population. C. Inversely proportional to statistical power - false D. Directly proportional to sample size - false: SE=SD/SQRT(n) E. FALSE Dependent for its validity on a normal distribution in the population

During scoliosis surgery with monitoring of somatosensory evoked potentials, which tract are they mainly monitoring? A: Dorsal column B: Spinothalamic tract C: Lateral Corticospinal tract D: Cerebrospinal tract E: Anterior horn cells

ANSWER A A. dorsal column r -white matter in the dorsomedial side of the spinal cord -fasciculus gracilis and fasciculus cuneatus -ascending pathway -fine touch and proprioception B Spinothalamic -sensory for pain, temperature, itch and crude touch -two main parts 1. lateral spinothalmic: pain and temp 2. anterior spinothalamic: crude touch C. lateral corticospinal tract controls movement of ipsilateral limbs -Control of more central axial and girdle muscles D. Cerebrospinal tract ??

Prostacylin: A. Equally efficacious with inhaled NO B. Half-life of 2 to 3 mins C. Platelet (?aggregation/?inhibition) causes it to be contraindicated D. Metabolised in the liver to arachidonic acid E. Improves oxygen saturation & decreases PAP given IV

ANSWER A A: T B: T ?Maybe false - Uptodate quotes a half-life of 6 minutes C: F D: F E: F PGI2 is the main arachadonic acid product in vascular endothelium and smooth muscle. It inhibits platelet function, causes vasodilation and releases NO. It has a short half life of 2-3 minutes. For the treatment of pulmonary hypertension, all studies agree that when given via the inhaled route, it is as effective as NO. It has less efficacy when given IV as it causes systemic vasodilation, reducing pulmonary blood flow and resulting in V/Q mismatch with no improvement in SaO2. Given via the inhaled route, it reduces mean pulmonary arterial pressures and improves SaO2. It is metabolised by spontaneous hydrolysis in the plasma, these products then undergoing further metabolism in the liver. It is not re-formed into arachadonic acid. No important effects on coagulation are reported when it is inhaled.

Thalassaemia (beta-trait) -you would expect: A. Transferrin levels are normal (or ?increased) B. HbF shifts curve to the right C. Hepatosplenomegaly is a common presentation D. Anaemia worse with infection E. Very large amounts of HbF

ANSWER A A: TRUE * In thalassaemia triat serum ferritin and iron stores are normal...Kumar and Clarke B: FALSE * Left (? due to increased HbA2) C: FALSE * Common in thalassaemia major - represents extra-medullary hamaetopoiesis (along with bone expansion)...Kumar and Clarke D: ? * No ref in Harrison's. I think this is a distractor relating to sickle cell anaemia * Beta-thalassaemia trait (ie heterozygous) may be asymptomatic or mild anaemia worsened in infections or pregnancy. E: FALSE * Hallmark of thalassaemia major

Correct statements regarding tricyclic antidepressant drugs used in the treatment of chronic pain include each of the following EXCEPT A. are more effective if they have predominantly noradrenergic effects B. block alpha-adrenergic and NMDA receptors C. block neuronal reuptake of serotonin and noradrenaline D. enhance descending inhibitory actions on the spinal cord E. should be used with caution in patients with abnormalities of cardiac conduction

ANSWER A A: false, and therefore correct answer "It is thought by many that mixed reuptake inhibitors such as amitriptyline are more effective than selective agents, emphasising the importance of both serotonergic and noradrenergic pathways in pain perception." CCEACP article B: true "However, their efficacy is generally thought to be related to central blockade of central nervous system (CNS) monoamine uptake, specifically serotonin and/or norepinephrine, in addition to other neurotransmitters. They may alter nociceptive processing by prolonging synaptic activity of these monoamines, thereby enhancing descending inhibitory action in the spinal cord in addition to monoaminergic effects elsewhere in the CNS. The drugs also, to varying degrees, block a number of other receptor types involved in pain processing including a-adrenergic, H1-histaminergic and N-methyl-D-aspartate (NMDA) receptors." CCEACP article C: true (see above) D: TRUE (see B above) E: TRUE Side-effects (which commonly limit their use) include sedation and anticholinergic effects, particularly dry mouth. Constipation and urinary retention are less common but well documented. The drugs have a number of effects on the heart including slowing of atrioventricular and intraventricular conduction. Cardiac side-effects are important as they may preclude the use of these drugs in patients with cardiac conduction disturbances or recent infarction.

Absolute contraindication to sitting position for posterior fossa craniotomy for meningioma A: Prescence of patent ventriculo-atrial drain/shunt B: 0.5mm PFO C: Oesophageal stricture so transoesophageal echo placement is out D: History of TIA E: Aortic stenosis

ANSWER A ABSOLUTE CONTRAINDICATIONS * Patent ventriculo-atrial shunt * Severe cardiovascular disease * Large patent foramen ovale or other pulmonary-systemic shunt * Cerebral ischaemia when upright and awake * Anaesthesia or surgical team not familiar with the position

A 29 year old female undergoes craniotomy for posterior fossa tumour. Which of the following is an absolute contraindication to the sitting position A. Patent ventriculo-atrial shunt B. Previous back surgery C. Pacemaker D. Small patent foramen ovale E. Oesophageal stricture contraindicated for transoesophageal echocardiogram

ANSWER A ABSOLUTE CONTRAINDICATIONS • Patent ventriculo-atrial shunt • Severe cardiovascular disease • Large patent foramen ovale or other pulmonary-systemic shunt • Cerebral ischaemia when upright and awake • Anaesthesia or surgical team not familiar with the position

A patient in recovery post op total hip replacement develops crushing central chest pain, ECG shows ST segment elevation. The most appropriate action is to give A. Aspirin B. IV GTN C. IV heparin D. Calcium channel blocker E. Beta blocker

ANSWER A AHA guidelines Management of patients with ST-segment-elevation myocardial infarction. Adjuvant therapy in association with reperfusion All patients undergoing reperfusion therapy for STEMI (PCI or fibrinolysis) should be given aspirin and clopidogrel unless these are contraindicated. http://circ.ahajournals.org/content/early/2012/12/17/CIR.0b013e3182742cf6.full.pdf+html

Analgesic requirements during labour are reduced by each of the following except A. Acupressure B. Acupuncture C. Hypnosis D. One to one support by midwife E. TENS

ANSWER A AMPSE : "complementary and other methods of pain relief in labour" Midwife/support person one on one- reduces analgesic use (level 1) hypnosis- decreased requirement for phar,acological analgesia (level 1) acupunture- decreased need for analgesics (level 1) TENS- "evidence of a weak opioid sparing effect" (level 1) no mention of acupressure 'Analgesia in labour: non-regional techniques' Caroline Fortescue, Michael YK Wee BJA CEA CCP Volume 5 Number 1 2005 p9-13 * Acupuncture (Acupressure, laser acupuncture) - One RCT of 100 women in Sweden comparing acupuncture with no acupuncture suggested former group needed less analgesia, including epidurals * Hypnosis - Cochrane review of three RCTs: one reported less anaesthesia and another less narcotic use, but overall meta-analysis showed no difference in the need for pain relief * Continuous support - Cochrane review of 15 RCTs involving 12 791 women. Those with continuous support, as opposed to conventional care, were less likely to have intrapartum analgesia, operative birth or be dissatisfied with their experiences * TENS - Systematic review of eight RCTs failed to demonstrate analgesic effect

Pregnant woman presents with narrow complex tachycardia HR 190, stable BP 100/60. No response to vagal manoevures. Management? A. adenosine 6mg B. DCR C. amiodarone D. Atenolol E. ?

ANSWER A ARC Guideline 11.11 - Managing Acute Dysrhythmias says that in a patient with no "adverse features" start with vagal manoeuvres. If that does not work next step is adenosine 6mg, then 12mg if required. Next step is Ca2+-channel blocker (verapamil or diltiazem). The fact that she is pregnant is not irrelevant from the treatment point of view, but main priority is the life of the mother. Consideration of effects of drugs on fetus is important, but there is no point witholding the correct treatment because of potential effects on fetus if the mother dies as a result of witholding the drug anyway.

SF44 ANZCA version [2003-Aug] Q48 (Similar question reported in [Aug96] [Apr97] [Jul97]) A 35 year old primipara at 38 weeks gestation is admitted to hospital with abruptio placentae. The fetal heart rate is 100 beats per minute, and the cervix is unfavourable. Caesarean section is planned. The anaesthetic of choice is A. thiopentone, succinylcholine, isoflurane and oxygen B. spinal anaesthesia C. thiopentone, nitrous oxide, isoflurane and oxygen D. thiopentone, succinylcholine, nitrous oxide and oxygen E. epidural anaesthesia

ANSWER A Abruption associated with risk of DIC. GA: Thio, sux, iso, oxygen. FUrthermore A heart rate of 100 is nonreassuring, and placental abruption associated with siginificant bleeding, and coagulopathy, agree GA, thio sux iso oxygen ( avoid N20 as increase risk of gas embolus/ amniotic fluid embolus and fetus is compromised and will need supplemental oxygen) The indications for GA are * Speed * Coagulopathy * Anticipated bleeding - such as anterior pl. praevia in multip * Sepsis (relative) * Tethered cord * Failed regional * Patient refuses regional

Which of the following is not an absolute contra-indication for MRI? A. cochlear implant B. heart valve prosthesis C. ICD D. pacemaker E. intracranial clips

ANSWER A Absolute Contraindications * electronically, magnetically, and mechanically activated implants: * cardiac pacemakers * ferromagnetic or electronically operated stapedial implants * hemostatic clips (CNS) * metallic splinters in the orbit Relative Contraindications * electronically, magnetically, and mechanically activated implants: * other pacemakers, e.g., - for the carotid sinus; - insulin pumps and nerve stimulators; - lead wires or similar wires * non-ferromagnetic stapedial implants * cochlear implants * prosthetic heart valves (in high fields, if dehiscence is suspected) * hemostatic clips (body) * makeup and tattoos * congestive heart failure * pregnancy (claustrophobia)

Rate of phenytoin administration: a. 50 mg/min b. 70 mg/min c. 100 mg/min d. Over 5 minutes e. As fast as possible

ANSWER A Admin by slow IVI (max 50 mg/min) into large vein via catheter

Diastolic dysfunction Not caused by A. Adrenaline B. Myocardial fibrosis C. Aortic stenosis D. Hypertension

ANSWER A Adrenaline increases the efflux of Ca, therefore aiding relaxation (lusitropy)

Advantages of bronchial blockers over double lumen tubes: A. Able to achieve lobar isolation B. Lower cuff pressure C. Quicker deflation of isolated lung D. Pneumonectomy E. Lower incidence of malposition

ANSWER A Advantages -do not need to change tube ; post op ventilation -use in kids -isolate lobes -no cuff damage -easier to fit in sick patient Disadvantage -smaller lumen -harder to switch bakc to OLV - 2LV -easily dislodged

Post liver resection, the time when prothrombin time derangement is maximal is: A. day 1-2 B. day 3-4 C. day 4-5 D. day 6-7 E. unpredictable time after surgery

ANSWER A After major liver resection 1. Factor depletion secondary to consumption and the half life dependent decay a. Factor II (72 hour half life) b. Factor VII (8 hour half life) c. Factor IX (24 hour half life) d. Factor X (39 hour half life) 2. transient synthetic insufficiency of the remnant liver 3. Further compounded by massive transfusion : dilutional coagulopathy As a result, prolongation of the PT is a common finding after major hepatic resection.

70 year old man having lung resection for SCC of left lung FEV1 2.3L (? % predicted), FVC 3.5L (? % predicted). Do you... A. Accept for lobectomy or pneumonectomy B. Decline pneumonectomy, proceed to lobectomy C. Cardiopulmonary exercise testing D. Differential V/Q scan E. Decline both pneumonectomy and lobectomy

ANSWER A Algorithms from the British Thoracic Society and American College of Chest Physicians. -initial screening tool is preoperative FEV1 -FEV1 >2L for pneumonectomy -FEV>1>1.5L for lobectomy If these threshold lung volumes are not present -full respiratory testing to calculate ppoFEV1 and DCLO -both need to be >40% and O2 sat>90% on RA If both ppoFEV1 and DLCO are <40%, then CPX testing -VO2 max <15 ml/kg/min delineates between high and medium risk patients

Best immediate treatment of severe post-partum haemorrhage after delivery of a complete placenta: A. IV Ergometrine B. Blood transfusion C. Evacuation of uterus without blood transfusion D. Bimanual compression of the uterus E. Aortic compression

ANSWER A All answers are correct, but the 'best immediate treatment'

All of the following may be associated with ulcerative colitis EXCEPT A. cirrhosis B. iritis C. psoriasis D. arthritis E. sclerosing cholangitis

ANSWER A All of the following may be associated with ulcerative colitis EXCEPT A. cirrhosis - probably false and the best answer according to Harrison's but maybe not the exam answer: "Hepatic steatosis is detectable in about half of the abnormal liver biopsies from patients with CD and UC; patients usually present with hepatomegaly. Fatty liver usually results from a combination of chronic debilitating illness, malnutrition, and glucocorticoid therapy." (Harrison's Ch 289 Extraintestinal manifestations section) Cirrhosis may occur secondary to sclerosing cholangitis though... B. iritis - true: "The incidence of ocular complications in IBD patients is 1-10%. The most common are conjunctivitis, anterior uveitis/iritis, and episcleritis." C. psoriasis - true but classically taught to be false as Crohn's has a stronger association; probably the exam answer: "Psoriasis affects 5-10% of patients with IBD and is unrelated to bowel activity." (Harrison's) ""We suggest that future studies on comorbidities in psoriasis should also focus on ulcerative colitis and not only Crohn's disease," say Cohen et al." [1] D. arthritis - true: "Peripheral arthritis develops in 15-20% of IBD patients, is more common in CD, and worsens with exacerbations of bowel activity. It is asymmetric, polyarticular, and migratory and most often affects large joints of the upper and lower extremities. Treatment is directed at reducing bowel inflammation. In severe UC, colectomy frequently cures the arthritis." E. sclerosing cholangitis - definitely: "1-5% of patients with IBD have PSC, but 50-75% of patients with PSC have IBD."

Analgesic requirements during labour are reduced by each of the following except A. Acupressure B. Acupuncture C. Hypnosis D. One to one support by midwife E. TENS

ANSWER A Analgesic requirements during labour are reduced by each of the following except: * A. Acupressure - false: "Published in the Cochrane Library collection of evidence-based literature was a systematic review of complementary and alternative therapies for pain management in labor. This meta-analysis of 14 studies evaluated the efficacy of acupuncture, acupressure, audioanalgesia, aromatherapy, hypnosis, massage, and relaxation for labor analgesia. Only two therapies were found to be beneficial, acupuncture and hypnosis." (Macarthur, Gerard W. Ostheimer "What's New in Obstetric Anesthesia" Lecture, in Anesthesiology 2008; 108:777-85) * B. Acupuncture - true: "Acupuncture decreases the need for analgesics" (ANZCA and FPM, Acute Pain Management: Scientific Evidence, 2E, 2005, p229) * C. Hypnosis - true: "Hypnosis used in labour also leads to a decreased requirement for pharmacological analgesia" (ANZCA and FPM, Acute Pain Management: Scientific Evidence, 2E, 2005, p229) * D. One to one support by midwife - true: "Continuous or one-to-one support by a midwife or trained layperson during labour reduces analgesic use" (ANZCA and FPM, Acute Pain Management: Scientific Evidence, 2E, 2005, p229) * E. TENS - true but possibly dodgy evidence: "Randomised controlled trials provide no compelling evidence for TENS having any analgesic effect during labour. Weak positive effects in secondary (analgesic sparing) and tertiary (choosing TENS for future labours) outcomes may be due to inadequate blinding causing overestimation of treatment effects" (Carroll et al, Transcutaneous electrical nerve stimulation in labour pain: a systematic review, BJOG February 1997, Vol. 104, pp. 169-175)

Serotonin syndrome may be appropriately managed with each of the following EXCEPT A. bromocryptine B. chlorpromazine C. cyproheptadine D. diazepam E. non-depolarizing neuromuscular blockers

ANSWER A As per A&IC article "Serotonin Syndrome and the Anaesthetist" April 2005 (local rag with perfect timing for Jul05 exam), drugs useful inclde: * cryproheptidine (5HT2 antagonist) * chlorpromazine (70% as potent as cryproheptidine)- antipsychotic. * diazepine (to treat neuromuscular abnormalities) * neuromuscular paralysis ie NDMR (aims to counter myoclonus to prevent rhavdomyolysis) Bromocriptine is a dopamine receptor agonist, so should not have a role in serotonin syndrome

Correct statements regarding fondaparinux include each of the following EXCEPT A. it has a structure unrelated to heparin B. it is administered once daily C. it is a synthetic, selective Factor Xa inhibitor D. it is recommended for DVT prophylaxis in major orthopaedic surgery E. the dosage does NOT need to be adjusted for age and sex

ANSWER A As per Yentis pg 214 * A - derived from the factor Xa-binding moeity of unfractionated heparain (FALSE, hence ANSWER) * B - single daily dose (TRUE) * C - "It is a synthetic and specific inhibitor of activated Factor X (Xa)" [1] (TRUE) * D - only licensed for DVT prophylaxis in orthopaedic surgery (TRUE) * E - implies standard dose for all (TRUE)

MR39c ANZCA version [2005-Sep] Q150, [Mar06] The most correct statement concerning the respiratory changes of morbid obesity is that A. alveolar hypoventilation is characteristic of Pickwickian obesity B. the functional residual capacity (FRC) is unchanged C. the residual volume (RV) is decreased D. the weight of the torso is responsible for increased chest wall compliance E. the work of breathing is unchanged

ANSWER A At its extreme, obesity hypoventilation syndrome culminates in the pickwickian syndrome, which is characterized by obesity, daytime hypersomnolence, arterial hypoxemia, polycythemia, hypercarbia, respiratory acidosis, pulmonary hypertension, and right ventricular failure.

MZ-25 Post op patient (surgery 3/7 ago). Patient dyspnoeic. V/Q scan organized which shows non segmental matched perfusion/ventilation defects. This is consistent with a. Atelectasis b. COPD (multiple, segmental, peripheral, bilateral, matched) c. Pulmonary embolus (mismatched) d. Pneumonia (reverse mismatch) e. Pulmonary infarction (mismatched)

ANSWER A Atelectasis : both compression of pulmonary vessels and alveoli (matched VQ defect plus non segmental)

Which of these statements regarding atrial fibrillation is INCORRECT A. has a risk of atrial thrombus and embolic stoke which decreases with time B. of less than 24 hours duration will usually revert to sinus rhythm spontaneously C. is contributed to by alcohol excess, pericarditis and chronic lung disease D. of long duration has lower success rates for both pharmacological and electrical conversion

ANSWER A Atrial Fibrillation 1. Incidence of AF doubles each decade 2. It is an independant RF for death, most commonly due to heart failure 3. Spontaneous conversion to SR within 24 hours in 2/3 of patients -likelihood decreases after 24 hours -after one week, spontaneous reversion is rare Risk factors -age -valvular heart disease -CCF -hypertension -diabetes -LVH -IHD -COPD -alcohol **'lone AF in 30% of cases

To operate on the anterior 2/3rds of the ear you would need to block: A. Mandibular n B. Maxillary n C. Vagus n D. Greater auricular n

ANSWER A Auriculotemporal is the first branch of the posterior trunk of V3 after it passes through the foramen ovale.

What is the area burnt in man if Half of left upper arm, all of left leg and anterior abdomen (repeat Q) A. 27% B. 32% C. 42% D. ? E. ?

ANSWER A Based on the 'rule of nines', the estimated burn surface area would be as follows: - half of upper arm = approx 2% - all of left leg = 18% - anterior abdomen = approx 9% depending on what proportion is burnt; whole thorax/abdomen on either anterior or posterior is 18% - i.e. 36% for front and back -> TOTAL = 29%

28. (NEW) Utility of BNP (brain naturietic peptide) is for a. Dyspnoea after pneumonectomy b. Loss of consciousness after .. c. Confusion after CABG

ANSWER A Both BNP and NT-proBNP levels in the blood are used for screening, diagnosis of acute congestive heart failure (CHF) and may be useful to establish prognosis in heart failure and in Anesthesiology, preoperative BNP independently predicts in-hospital ventricular dysfunction, hospital length of stay (HLOS) and mortality up to 5 years after primary CABG surgery. So hard to know without the real question. Other things I've read suggest that BNP can help differentiate dyspnoea due to cardiac failure from other causes. After pneumonectomy, there is a sudden increase in PVR, so perhaps this is indicative of whether the right heart is coping. I think the key is dyspnoea; when the real answers appear (in the next exam no doubt) its utility is to suggest whether the dyspnoea is related to cardiac failure or something else.

Hyponatraemia and hypovolaemia all except A. SIADH B. Pancreatitis C. Nephritis D. Renal tubular acidosis E. Addisons

ANSWER A CAUSES OF HYPONATRAEMIA Hypovolaemic -renal losses - diuretic, mineralocorticoid deficency, cerebral salt waste, renal tubular acidosis -GIT : vomiting dirrhoea -Skin : burns, sweat -Pancreatitis

Advantages of off-pump CABG over on-pump CABG A. decreased transfusion rate B. decreased mortality C. decreased cost D. increased graft patency E. less cognitive impairment F. less stroke

ANSWER A CEACCP Hett 2006 Benefits of OPCABG -mortality reduced from 2.9% -> 2.3% and complication rate 12% -> 8% (conflicting results from other studies) -Most studies have shown a reduction in the need for transfusion and other blood products (effect of extracorporeal circulation and hypothermia) -Incidence of stroke is less (no manipulation of aorta resulting in macro and micro embolii) -reduced rise in inflammatory markers No difference -mortality and morbidity is unchange -incidence of AF is similar -short term patency rate are comparable. There is no info on long term patency -no difference in neurological dysfunction

When optimising patients for surgery using goal-directed therapy, which of the following parameters is LEAST useful? A. blood pressure B. cardiac index C. oxygen saturation of blood aspirated from a central venous catheter D. oxygen saturation of blood aspirated from the distal port of a pulmonary artery catheter E. stroke volume variability

ANSWER A CEACCP Volume 2 Number 6 2002 Pre-operative care of the high-risk surgical patient "In order to target therapy, markers of tissue oxygen delivery are required. Vital signs such as heart rate and blood pressure are known to be unreliable, whereas assessment of cardiac index (CI), and subsequent calculation of global O2 delivery index (DO2I) and O2 consumption index (VO2I), provides meaningful physiological end points for resuscitation" Which goals should we target? Apparently adequate tissue oxygen delivery is the key physiological variable. We're going with answer A ANZCA version: blood pressure. Goal directed therapy • insert IABP and CVP • fill to CVP 8 - 12mmHg • if MAP remains < 65mmHg commence Noradrenaline • if high dose NA required, insert non-invasive CO monitor (PiCCO) and commence inotrope (Dobutamine) • guide management using mixed or central venous oxygen (aim > 70%) • consider: - levosimendan - milrinone - vasopressin (0.01 - 0.05U/min) - Activated Protein C

PL31 Aug10 Time to reach peak plasma concentration after injection of 2% lignocaine with adrenaline into epidural space A. 20 min B. 30 min C. 40 min D. 50 min E. 60 min

ANSWER A CJA. Between 20-30mins. Only took 2 samples though. Could have missed peak. Highest 4.4mcg/ml. http://www.springerlink.com/content/12727258463u7q30/ Look at Fig 1 in Anaes & Anal: 1979 58(5); 360-363

Pneumonia in children A. if bacterial is commonly associated with a pleural effusion B. is often caused by Mycoplasma Pneumoniae in infants C. is rarely caused by Chlamydia Trachomatis in neonates D. is rarely fatal in developed countries E. often presents as lethargy without fever or cough

ANSWER A Causative bugs in paediatric pneumonia *90% viral (mostly RSV) *10% bacterial - bug depends on age of child: -Pneumococcal - older children - 90% of bacterial paediatric pneumonia -Haemophilus influenzae - all ages -Staph - younger than one, and usually sick/debilitated infants -Mycoplasma - ages 5 - 15 -Pneumocystis - immunocompromised Neonatal pneumonia * may be contracted in-utero, or aquired during birth * usually bacterial - GBS, gram neg bacilli (e coli, klebsiella, pseudomonas), listeria, staph, rarer bugs are anaerobic bacilli and chlamydia Mycoplasma causes an 'atypical pneumonia' with influenza-like respiratory illness of gradual onset with headache, malaise, fever, cough Pneumococcal pneumonia is the most common typical bacterial pneumonia in children of all ages. In one retrospective review of 254 children and young adults (age <1 month to 26 years) with pneumococcal pneumonia, the most common signs and symptoms and their approximate frequencies are listed below: • Fever: 90 percent, • Cough: 70 percent; • Productive cough: 10 percent • Tachypnea: 50 percent

Pneumonia in children A. if staphylococcal, is commonly associated with a pleural effusion B. is commonly caused by mycoplasma pneumoniae in young children C. is most commonly caused by streptococcus pneumoniae D. is rarely caused by chlamydia trachomatis in neonates E. often presents as lethargy without fever or cough

ANSWER A Causative bugs in paediatric pneumonia *90% viral (mostly RSV) *10% bacterial - bug depends on age of child: -Pneumococcal - older children - 90% of bacterial paediatric pneumonia -Haemophilus influenzae - all ages -Staph - younger than one, and usually sick/debilitated infants -Mycoplasma - ages 5 - 15 -Pneumocystis - immunocompromised Neonatal pneumonia * may be contracted in-utero, or aquired during birth * usually bacterial - GBS, gram neg bacilli (e coli, klebsiella, pseudomonas), listeria, staph, rarer bugs are anaerobic bacilli and chlamydia Mycoplasma causes an 'atypical pneumonia' with influenza-like respiratory illness of gradual onset with headache, malaise, fever, cough Pneumococcal pneumonia is the most common typical bacterial pneumonia in children of all ages. In one retrospective review of 254 children and young adults (age <1 month to 26 years) with pneumococcal pneumonia, the most common signs and symptoms and their approximate frequencies are listed below: • Fever: 90 percent, • Cough: 70 percent; • Productive cough: 10 percent • Tachypnea: 50 percent

What is the anatomic location of the celiac plexus? A. anterior to the body of L1 B. lateral to adrenals and kidneys C. surrounding the IVC D. posterior to the crus of the diaphragm

ANSWER A Celiac Plexus -largest sympathetic plexus -main junction for autonomic nerves supplying the upper abdominal organs : * liver * gall bladder * spleen * stomach * pancreas * kidneys * small bowel * 2/3 of the large bowel Relations -surrounds the root of the coeliac artery at the level of L1 -medial to the adrenals and kidney -posterior to the IVC on the right and aorta on the left -anterior to the crura of the diaphragm Sympathetic supply: * Greater splanchnic nerve (T5/6 to T9/10) * Lesser splanchnic nerve (T10/11) * Least splanchnic nerve (T11/12)

Central anticholinergic syndrome, which is NOT true: A. Will improve with neostigmine B. Peripheral anticholinergic symptoms C. Caused by Anti-Parkinson drugs D. CNS depression E. Associated with agitation, delirium, and ???

ANSWER A Central anticholinergic syndrome Historically, anticholinergic syndrome was a commonly encountered sequel to anaesthesia. Nowadays, less anticholinergic medications are used. Symptoms range from cerebral irritation with delirium and agitation to CNS depression with stupor and coma. These accompany peripheral anticholinergic effects that is tachycardia, blurred vision, dry mouth and urinary retention. The symptoms are rapidly reversed by physostigmine (an acetylcholinesterase inhibitor), but may recur when its effect terminates. Anti-Parkinsonian, antidepressant and antihistamine drugs can cause central anticholinergic syndrome. Emedicine Physostigmine is the only reversible acetylcholinesterase inhibitor capable of directly antagonizing the CNS manifestations of anticholinergic toxicity; it is an uncharged tertiary amine that efficiently crosses the blood brain barrier

Post CEA on ward, patient seizes. BP has been hard to control. What to do to prevent further seizures? A: Add another antihypertensive B: Start antiplatelet drugs C: Start anticonvulsants D: Do angio and stent E: Nimodipine

ANSWER A Cerebral hyperperfusion syndrome (most CEA neurological complications are embolic/ischemic in nature) -however small subset due to CHS -occurs in 0.75%-3% post CEA Characterised (trial) 1. ipsilateral headache (severe, pounding) 2. focal seizure actiivity 3. focal neurological deficit (hemiplegia, neglect,hemianopioa, aphasia) -possible ipsilateral intracerebral haemorrhage or edema -associated hypertension Risk factors 1. Hypertension 2. High grade stenosis with poor collateral flow 3. Decreased CVR 4. Increased peak flow velocity 5. Contralateral carotid occlusion 6. Recent contralateral CEA (<3 months) 7. Intraoperative distal carotid pressure of <40mmHg 8. Intraoperative ischaemia Detection -high flow through Trans-Cranial Doppler of MCA Prevention -early identification and control of blood pressure -aggressive BP control in HDU settling -labetolol and clonidine prefered as they not increase CBF -avoid vasodilators such as ACEI, Ca CB, nitroprusside or GTN.

Emergency caesarean section for foetal distress (and foetal acidosis on scalp probe?). what is best option to raise gastric pH preop: A. Oral Na Citrate B. Ranitidine IV C .Ranitidine oral D. Omeprazole IV E. Omeprazole oral

ANSWER A Cisapride works, but withdrawn b/c prolonged QT. Na Citrate immediate. (?urgent) Ranitidine 90-120mins Omeprazole? Time. Metocloparmide pro-kinetic....

AZ79 ANZCA version [2003-Apr] Q100 The first step in developing a clinical audit should be to: A. define the question to be asked B. develop the relevant clinical protocol C. draw up the audit form D. find the relevant standards E. obtain ethics committee approval

ANSWER A Clinical audit - the process The clinical audit process seeks to identify areas for service improvement, develop & carry out action plans to rectify or improve service provision and then to re-audit to ensure that these changes have an effect.-->Clinical audit can be described as a cycle or a spiral, see figure. Within the cycle there are stages that follow the systematic process of: establishing best practice; measuring against criteria; taking action to improve care; and monitoring to sustain improvement. As the process continues, each cycle aspires to a higher level of quality. * Stage 1: Identify the problem or issue * Stage 2: Define criteria & standards * Stage 3: Data collection * Stage 4: Compare performance with criteria and standards * Stage 5: Implementing change

Pain in 3 year old, best objective tool a. FLACC (crying limbs activity consolability) b. Wong baker Faces pain c. Self reporting d. Mum reporting e. Nurse reporting

ANSWER A Communication aids and behavioural scales such as the modified Faces, Legs, Activity, Cry and Consolability (FLACC) scale Key Messages for patients with special needs 1. Regular assessment of pain leads to improved acute pain management (U) (Level III‐3). 2. There is good correlation between the visual analogue and numerical rating scales (U) (Level III‐2). *Self‐reporting of pain should be used whenever appropriate as pain is by definition a subjective experience (U). * Scoring should incorporate different components of pain including the functional capacity of the patient. In the postoperative patient this should include static (rest) and dynamic (eg pain on sitting, coughing) pain (U). * Uncontrolled or unexpected pain requires a reassessment of the diagnosis and consideration of alternative causes for the pain (eg new surgical/ medical diagnosis, neuropathic pain) (U). * The pain measurement tool chosen should be appropriate to the individual patient; developmental, cognitive, emotional, language and cultural factors should be considered (U).

Major complications of mediastinoscopy include all of the following EXCEPT A. compression of the great vessels B. air embolism C. pneumothorax D. major haemorrhage E. phrenic nerve damage

ANSWER A Complications of mediastinoscopy * Death * MAJOR complications Major haemorrhage Tracheobronchial laceration Oesophageal perforation Recurrent nerve paralysis Phrenic nerve paralysis Thoracic duct injury Cerebrovascular accident Mediastinitis Venous air embolism Tumour implantation * MINOR complications Pneumothorax Superficial wound infection Recurrent nerve paresis Minor bleeding Autonomic reflex braydcardia Mediastinal lymph node necrosis Compression of the great vessels occurs and warrents invasive blood pressure monitoring, however it is rarely a complications as it is transient.

Major complications of mediastinoscopy include all of the following EXCEPT A. compression of the great vessels B. air embolism C. pneumothorax D. major haemorrhage E. phrenic nerve damage

ANSWER A Compression of major vessels is noted to be a reason for invasive blood pressure monitoring in the CEACCP article but is not listed in the table of complications, perhaps because it is a transient phenomenon rather than a complication per se. Complications as follows : DEATH MAJOR complications -Major haemorrhage -Tracheobronchial laceration -Oesophageal perforation -Recurrent nerve paralysis -Phrenic nerve paralysis -Thoracic duct injury -Cerebrovascular accident -Mediastinitis -Venous air embolism -Tumour implantation MINOR complications -Pneumothorax -Superficial wound infection -Recurrent nerve paresis -Minor bleeding -Autonomic reflex braydcardia -Mediastinal lymph node necrosis

What is a contraindication to an IABP? A. Aortic regurgitation B. Aortic stenosis C. Mitral regurgitation D. Mitral stenosis E. Tricuspid stenosis

ANSWER A Contraindication to IABP AR Aortic dissection Major coagulopathy Severe bilateral PVD Bilateral fem-pop bypass graft Sepsis http://ceaccp.oxfordjournals.org/content/9/1/24.full

When assessing a patient with unstable angina, the clinical feature most predictive of an adverse short-term outcome (death or myocardial infarction within 6 months) is A. increased angina frequency or severity B. angina provoked at a lower threshold of exercise C. new onset angina on effort D. an angina episode lasting more than 10 minutes E. age less than 65 years

ANSWER A Coronary syndrome I no Sx with ordinary activity II slight limitation of ordinary activity III marked limitation of ordinary activity IV inabilty to carry on any physical activity without discomfort- anginal symptoms may be present at rest. high risk of death/ MI are History -acceleration tempo of ischaemic symptoms in preceding 24 hours Character of pain -prolonged ongoing rest pain (>20 mins) Clinical findings -Pulm Oedema most likely due to ischaemia -New or worsening MR murmur -Hypotension, bradycardia or tachycardia -Age > 75 years -ECG Angina at rest with transient ST changes > 0.05mV, -New or presumed new BBB -Sustained VT Cardiac markers -elevated TnT or TnI >0.1 ng/ml

PP90 [Jul06] [Apr07] Q8 Child having multiple lower limb osteotomies. He has frequent painful muscle spasm and cerebral palsy. Best analgesia: A. continuous epidural infusion with bupivacaine and clonidine B. regular NSAID plus tramadol C. morphine PCA D. paracetamol and oxycodone E. spinal morphine

ANSWER A Current practice at RCH

AZ49 ANZCA version [2002-Aug] Q66, [2003-Apr] Q71 (Similar question reported in [Aug96] [Apr98] [Jul98]) A patient taking cyclosporine after renal transplantation is UNLIKLELY to have A. Macrocytic anaemia B. Interstitial renal fibrosis C. Hypertension D. Peripheral neuropathy E. Suppresion of humoral immunity

ANSWER A Cyclosporin -DMARDS (disease modifying anti-rheumatic drugs) -immunosuppressive -used to treat RA, SLE, polymyositis, rejection of transplant Genitourinary -renal dysfunction (common 30%) -glomerular capilliary thrombosis Cardiovascular -hypertension (50%) -IHD Skin -Acne -Hirsutism CNS -tremor (common 20%) -convulsion -headache -somnolence GIT -gum hyperplasia (common 15%) -diarrhoea -nausea vomiting (common 10%) -hepatotoxity (common 50% of patients have elevated bilirubin, tramaminases, alk phos) -gynecomastia CNS -distal plaresthesia -Flushing Blood -leukopenia -lymphoma (infectious complications include : sepsis, abscess, systemic fungal infection CMV) Resp -sinusitis

Ventricular fibrillation in children A. if resistant to defibrillation, should be treated with amiodarone 5 mg.kg-1 B. is not associated with tricyclic antidepressant overdose C. is often associated with respiratory arrest D. is the commonest arrhythmia associated with cardiac arrest E. should be immediately defibrillated with a 5 J.kg-1 shock

ANSWER A DC shock is 2J/k then 4J/kg thereafter. The first line antiarrhythmic for shock-refractory VF/pulselessVT is amiodarone, 5mg/kg * The most common arrest scenario in children is bradycardia proceeding to asystole - a response to severe hypoxia and acidosis. * VF is relatively uncommon, but may complicate hypothermia, TCA poisoning, and those children with pre-existing cardiac disease

Which statement regardingDesmopressin (DDAVP) is INCORRECT A. has a half life of 5-20 minutes B. is a synthetic analogue of the natural hormone arginine vasopressin C. has a longer duration of action than antidiuretic hormone D. causes an increase in Factor VIII, von Willebrand Factor and tissue plasminogen activator E. is contraindicated in Type IIB von Willebrand's disease since platelet aggregation may be induced

ANSWER A DDAVP half-life is 1-3 hours, ADH (and IV vasopressin) 5-20 mins ↑ Factor VIII and vWF, tPA effect IIb associated with worsening with DDAVP

Bleeding patient. What is relative contraindication to Prothrombinex? A. History of HITS B. Von Willebrands C. Haemophilia B D. Warfarin overdose E. Renal failure

ANSWER A DESCRIPTION Each pack contains one vial of lyophilised high-purity human factor IX (500 IU), factor II (500 IU), factor X (500 IU), antithrombin (25 IU) and heparin (200 IU), one ampoule of Water for Injection BP and a filter needle. INDICATIONS Prothrombinex is indicated only as directed by the Consultant Haematologist. For the prophylaxis and treatment of bleeding in patients with single or multiple congenital deficiencies of factor II or X and in patients with single or multiple acquired prothrombin complex factor deficiency requiring partial or complete reversal (eg. Reversal of Warfarin anticoagulant therapy) Note: Prothrombinex should NOT be used for prophylaxis or treatment of haemorrhage in patients with Haemophilia B. CONTRAINDICATIONS Prothrombinex should NOT be used in patients showing clinical or laboratory signs of thrombosis or DIC. Prothrombinex should NOT be used for prophylaxis or treatment of haemorrhage in patients with Haemophilia B. DOSE AND ADMINISTRATION Dosage and administration should be discussed with the Consultant Haematologist. Dosage varies from 20-30 IU/kg for minor haemorrhage up to 50 IU/kg for moderate to severe haemorrhage. Exact loading and maintenance doses and dosing intervals should be based on the patient's clinical condition, response to therapy and relevant laboratory tests Prothrombinex is supplied as a freeze-dried powder with a 20mL ampoule of water for injection (WFI) and one filter needle. Before reconstitution allow the product to reach room temperature. Using an aseptic technique draw up 20mL of WFI and inject into the vial of Prothrombinex. Dissolve the Prothrombinex by gentle agitation. Do not shake. A clear or slightly opalescent solution is obtained within 10 minutes. If clots or a gel form return the product to the Blood Bank. Infuse slowly intravenously approximately 3mL/minute.

Left ventricular diastolic dysfunction A. can be diagnosed with transoesophageal echocardiography. B. is improved by propofol anaesthesia. C. is suggested by an early to late (atrial) mitral inflow velocity ratio (E/A) of greater than one D. following an old myocardial infarction is mainly due to delayed ventricular relaxation

ANSWER A Diastolic dysfunction -very common -2 components a. poor relaxation b. decreased compliance Diagnsosis 1. Clinical features of heart failure 2. Normal EF >50% 3. Increased left diastolic filling pressure (this is invasive, therefore non-invasive surrogates are used) a. Isovolumic Relaxation Time : time from AV closure to MV opening -IRT >110ms (60-80ms is normal) b. Transmitral inflow : E/A ratio -E is the early and A (atrial) is late ventricular filling velocity -E wave related to LV 'suction' and LA pressure -E wave acceleration will be higher with high LA pressure; and lowered with impaired relaxation -as DD progresses, the atrial component becomes more important in maintaining preload c. Pulmonary Venous Inflow : TOE Categorising DD i. Mild dysfunction (E/A <1, IRT>100ms, normal E wave deceleration, atrial reversal of pulmonary venous inflow ii. Mild-Mod (same E/A and ITR but atrial reversal >35ms) iii. Moderate ('pseudonormalization' with normal E/A iv. Severe ('restrictive' large E small A, short IRT, reduced pukmonary systolic wave) Management The following agents may be of value: * calcium channel blockers (but beware of some with associated ischaemic heart disease) * beta blockers * ACE inhibitors and AT-II receptor antagonists.

which does NOT have abnormal Q waves: A: Digoxin toxicity B: Anterior myocardial infartion C: Previous AMI D: LBBB E: Wolff-Parkinson-White syndrome

ANSWER A Digoxin does not cause Q waves

TMP-138 [Apr08] Q100 A 63-year-old, 70 kg man is in the recovery room following a lobectomy for small cell carcinoma of the lung. You are asked to review him because he is complaining of difficulty breathing. He is unable to lift his arms or flex his hips. He has a weak but sustained grip. Cisatracurium 8 mg was given 90 minutes previously. Neostigmine 2.5 mg and atropine 1.2 mg were given at the conclusion of the surgery. The likely diagnosis is: A. Eaton-Lambert syndrome B. limb-girdle muscular dystrophy C. myasthenia gravis D. myotonic dystrophy E. steroid myopathy

ANSWER A Eaton-Lambert Syndrome -rare condition -weakness results from pre-synaptic abnormality of ACh release at NMJ -associated with small cell carcinoma -antibodies against voltage gated Ca++ channels on presynaptic motor terminal Features -proximal muscle weakness better with repitition -decreased tendon reflexes Treatment -steriods -plasma exchange -IV immunoglobins

During a transoesophageal echocardiography (TOE) examination the end-diastolic area is found to be 15 cm2 and the end-systolic area 10 cm2 . Based on these measurements one could calculate this patient's A. area ejection fraction to be 33% B. area ejection fraction to be 67% C. ejection fraction to be 50% D. ejection fraction to be 67 % E. fractional area change to be 50%

ANSWER A Ejection fraction = (end-diastolic volume - end-systolic volume ) / end-diastolic volume] x 100 (%) Ejection fraction area = (EDV area - ESV area) / EDV area x 100 (%)

Following severe allergic reactions, serum tryptase is measured to: (a) confirm that an anaphylactic or anaphylactoid reaction occurred (b) diferentiate between anaphylactic and anaphylactoid reactions (c) identify the allergen

ANSWER A Elevated serum tryptase concentrations do not differentiate an anaphylactic from an anaphylactoid reaction, but does confirm one occured

What is the commonest symptomatic cardiac condition in pregnancy A. Mitral stenosis B. Aortic stenosis C. Eisenmengers D. Tetralogy of fallot E. ASD

ANSWER A Especially in the developed world Great review on cardiac conditions in pregnancy http://www.frca.co.uk/Documents/111%20Cardiac%20disease%20in%20pregnancy.pdf http://ceaccp.oxfordjournals.org/content/9/2/44.full

Correct statements regarding chronic fatigue syndrome are that A. cognitive behavioural therapy is a primary treatment of the condition B. it is associated with alcohol abuse (>40g/day) C. it is associated with morbid obesity (BMI>30) D. it is a major depressive disorder E. SSRIs (selective serotonin reuptake inhibitors) are the mainstay of treatment

ANSWER A Exclusion criteria 1. Alcohol abuse 2. Morbid obesity 3. Major depressive disorder Treatment 1. CBT

All of the following may be associated with ulcerative colitis EXCEPT A. cirrhosis B. iritis C. psoriasis D. arthritis E. sclerosing cholangitis

ANSWER A Extraintestinal complications — Ulcerative colitis may be associated with a number of extraintestinal complications. These include: ** Eye involvement with uveitis and episcleritis (picture 7A-B) (see "Skin and eye manifestations of inflammatory bowel disease") ** Skin disorders such as erythema nodosum and pyoderma gangrenosum (picture 8A-C) ** A peripheral arthritis, which primarily involves large joints (with no synovial destruction), and ankylosing spondylitis; furthermore, an undifferentiated spondyloarthropathy or ankylosing spondylitis may be the presenting manifestation of ulcerative colitis (see "Arthritis associated with gastrointestinal disease") ** Sclerosing cholangitis which typically presents with an elevation in the serum alkaline phosphatase concentration ** Lung disease, ranging in severity from asymptomatic decreases in diffusing capacity to disabling bronchiectasis (see "Pulmonary complications of inflammatory bowel disease") ** Venous and arterial thromboembolism [24-28] (table 2) ** Autoimmune hemolytic anemia

In regards to laparotomy and hemicolectomy, the least effective way to minimize post op surgical infection is: A. Aggressive peri-operative fluid management B. Timely administration of prophylactic antibiotics C. Perioperative hyperoxia D. Avoidance of hypothermia E. Avoidance of blood transfusion

ANSWER A Factors that can be optimized in the perioperative period can be divided into: (i) Well-established interventions (supported by good evidence) (a) antibiotic prophylaxis, (b) hand hygiene, (c) aseptic technique during invasive procedures, (d) perioperative thermoregulation. (ii) Less certain interventions (some supporting evidence) (a) face masks and theatre traffic, (b) regional anaesthesia techniques, (c) inspired oxygen, (d) glycaemic control. (iii) Speculative interventions (no supportive evidence as yet) (a) goal-directed fluid management, (b) minimizing blood transfusions, (c) enhanced recovery after surgery (ERAS), (d) avoidance of selected opioids. CEACCP Preventing postoperative infection: the anaesthetist's role

A diagnostic test has a sensitivity of 90% and a specificity of 99% in detecting a certain disease. From this we can conclude that A. the false positive rate of this test is 1% B. the false negative rate of this test is 1% C. the positive predictive value of this test is 90% D. the negative predictive value of this test is 90% E. this test would be a useful screening test for this disease

ANSWER A False Positive Rate = 1 - specificity False Negative Rate = 1 - sensitivity And as a further reminder: PPV = TP / (TP + FP) And NPV = TN / (TN + FN)

The following data on Apgar scores are best analysed by which test? Apgar: <4 4-7 >7 Group A 67 55 22 Group B 58 53 8 A. Fisher's exact test B. Wilcoxon sign rank test C. ANOVA D. Chi-squared

ANSWER A Fisher's Exact test Analysis of contingency tables were sample sizees are small, categorical data,

Cell saver. Which does NOT get filtered ? A. Foetal cells B. Free Haemoglobin C. Platelets D. Clotting factors E. Microaggregates of leukocytes

ANSWER A Foetal cells (particularly fetal RBC's) are NOT filtered/removed, and are spun down with maternal RBC's (ie the process does not differentiate between maternal and fetal RBCs) and then reinfused into the mother. However, the risk of alloimmunisation is thought to be no greater than that occurring during a normal vaginal delivery. The risk of amniotic fluid embolism is also thought to be very low - so much so that cell salvage in obstetrics has been endorsed by the Obstetric Anaesthetists Association, AAGBI, NICE and CEMACH in the UK, as well as the ASA in America. Cell salvage filters the following substances: * Free haemoglobin * White blood cells * Plasma * Platelets * Heparin * Clotting factors * Complement

Which of the following is NOT a side effect of cyclosporine A. Alopecia B. Hypertension C. Renal impairment D. Gum hyperplasia

ANSWER A From Wiki ADRs can include gingival hyperplasia, convulsions, peptic ulcers, pancreatitis, fever, vomiting, diarrhea, confusion, hypercholesterolemia, dyspnea, numbness and tingling particularly of the lips, pruritus, high blood pressure, potassium retention possibly leading to hyperkalemia, kidney and liver dysfunction (nephrotoxicity[21] and hepatotoxicity), burning sensations at finger tips and an increased vulnerability to opportunistic fungal and viral infections.In short, it is nephrotoxic, neurotoxic, causes hypertension (due to renal vasoconstriction and increased sodium reabsorption), increases the risk of squamous cell carcinoma and infections. It also causes gingival hypertrophy and hirsutism which is not seen with tacrolimus (another calcineurin inhibitor).

In a trial, 75 patients with an uncommon, newly described complication and 50 matched patients without this complication are selected for comparison of their exposure to a new drug. The results show Complication present Complication absent Exposed to new drug 50 25 NOT exposed 25 25 From this data A. the relative risk of this complication with drug exposure CANNOT be determined B. the odds ratio of this complication with drug exposure CANNOT be determined C. the relative risk of this complication with drug exposure is 2 D. the odds ratio of this complication with drug exposure is 1.33 (recurring) E. none of the above

ANSWER A From these data * A. the relative risk of this complication with drug exposure CANNOT be determined - true: o "Because accurate information concerning all patients at risk in a retrospective case-control study is not available (because sample size is set by the researcher), incidence rate and risk cannot be accurately determined, and the odds ratio is used as the estimate of the risk ratio" (Myles and gin p.74) * B. the odds ratio of this complication with drug exposure CANNOT be determined * C. the relative risk of this complication with drug exposure is 2 * D. the odds ratio of this complication with drug exposure is 1.33 (recurring) * E. none of the above

AM39 ANZCA version [2003-Aug] Q98 Features of Guillain-Barre syndrome include: A. distal loss of sensation B. extensor plantar responses C. facial weakness D. proximal weakness greater than distal weakness E. pseudobulbar palsy

ANSWER A Guillain-Barre syndrome (GBS) is a heterogeneous grouping of immune-mediated processes generally characterized by motor, sensory, and autonomic dysfunction. In its classic form, GBS is an acute inflammatory demyelinating polyneuropathy characterized by progressive symmetric ascending muscle weakness, paralysis, and hyporeflexia with or without sensory or autonomic symptoms; however, variants involving the cranial nerves or pure motor involvement are not uncommon. In severe cases, muscle weakness may lead to respiratory failure, and labile autonomic dysfunction may complicate the use of vasoactive and sedative drugs. Treatment The ONLY two effective treatments are: * plasma exchange therapy * intravenous immune serum globulin Both treatment are very similar in efficacy, and can decrease recovery time by as much as 50%. Corticosteroids are ineffective, and some case reports indicate they may slow recovery if used alone. Outcome 80-85% of cases recover completely within 6-12 months.

All of EXCEPT ONE of the following is seen following stellate ganglion block. Indicate the exception A. mydriasis B. facial anhydrosis C. enopthalmos D. nasal congestion

ANSWER A Horner's syndrome Horner's syndrome is characterised by an interruption of the oculosympathetic nerve pathway somewhere between its origin in the hypothalamus and the eye. The classic clinical findings associated with Horner's syndrome are * ptosis * pupillary miosis * facial anhidrosis. * enophthalmos * increased amplitude of accommodation * heterochromia of the irides (if it occurs before the age of 2 years), paradoxical contralateral eyelid retraction, transient decrease in intraocular pressure and changes in tear viscosity.

Which of the following are feature of Conn's syndrome? A. Normoglycaemia, hypernatremia , hypokalemia B. Hypoglycaemia, hypernatremia, hypokalemia C. Hyperglycaemia, hyponatremia, hyperkalemia D. Normoglycaemia, hyponatremia, hyperkalemia E. Hypoglycaemia, hyponatremia, hyperkalemia

ANSWER A HypoK+ most common, Na+ normal or high, some articles mention hyperglycaemia, but not hypoglycaemia. All have hypertension.

The EARLIEST sign of hypocalcaemia is: A. Tingling of face and hands B. Chvostek's sign C. Carpopedal spasm D. Hyperreflexia E. Clonus

ANSWER A Hypocalaemia -Ca <2.1 mmol/L or ionized Ca <1.1 mmol/L Correction for albumin -50% is bound to albumin -CorrCa = Measured serum Ca + [(4.0 - measured serum albumin) * 0.8] Symptoms * Petechia which appear as on-off spots, then later become confluent, and appear as purpura * Oral, perioral and acral paresthesias, tingling or 'pins and needles' sensation in and around the mouth and lips, and in the extremities of the hands and feet. This is often the earliest symptom of hypocalcaemia. * Carpopedal and generalized tetany * Latent tetany o Trousseau sign of latent tetany (eliciting carpal spasm by inflating the blood pressure cuff and maintaining the cuff pressure above systolic) o Chvostek's sign (tapping of the inferior portion of the zygoma will produce facial spasms) * Tendon reflexes are hyperactive * Life threatening complications o Laryngospasm o Cardiac arrhythmias * ECG changes include: o Intermittent QT prolongation Treatment -Urgent -10ml 10% Ca gluconate over 3 minutes -Vit D 1-5g orally -Consider Ca infusion -Consider acidosis : to increase the freely ionized Ca concentration For every 0.1 increase in pH, ionized calcium decreases by about 0.05 mmol/L.

A 4 year old child booked for minor surgery is seen in pre-admission clinic where a murmur is detecte:D. Which feature will warrant further investigation A. Loudness 4/6 B. Decreases on inspiration C. Vibratory quality D. Ejection systolic murmur E. Louder on supine

ANSWER A I. Signs: Innocent Murmurs A. Precaution: These signs are unreliable in under age 1 year due to higher Incidence of Congenital Heart Disease B. Auscultation (Seven S's - key reassuring findings in innocent murmurs) 1. Sensitive a. Murmur accentuates with position changes, activity - see below 2. Short duration . Not holosystolic 3. Single . Isolated murmur without click, gallup or other extra heart sounds 4. Small . Murmur limited to small, focal distribution without radiation 5. Soft . Low amplitude (e.g. II/VI murmur) 6. Sweet . Non-harsh quality 7. Systolic . Limited to systole (Diastolic Murmurs are typically pathologic) C. Accentuation maneuvers (innocent murmurs become louder in this position) 1. Sitting forward 2. Exercise or increased Heart Rate 3. Fever 4. Anxiety, Restlessness, or crying II. Signs: Pathologic Murmur . See Congenital Heart Disease A. Red flag general exam findings 1. Evidence of Failure to Thrive 2. Lethargy 3. Cyanosis 4. Shortness of Breath 5. Parasternal heave or thrill 6. Murmur radiates to back or neck B. Auscultation (6 Cardinal signs of pathology) 1. Harsh murmur 2. Pansystolic murmur 3. Murmur Grade III or more 4. Murmur at high Left sternal border 5. Early or Midsystolic click or murmur 6. Abnormal S2 Heart Sounds

Hypotension post propofol induction in elderly patient. More pronounced / profound than in younger patient. Reason ? A. Concentric LVH associated with ageing and therefore preload dependent B. Because of increased lean body mass C. Decreased cardiac output with ageing D. Increased sensiticity to all anaesthetic agents, thus relative overdose is common E. Decreased liver blood flow with ageing, decrease drug clearance and increased drug concentration

ANSWER A IV propofol causes hypotension in many patients. An incidence of 25-40% is quoted for typical induction doses. The incidence is highest in: * the elderly * those with pre-existing hypotension (an obvious clinical indication there) * patients at ASA >= III The main reason for the hypotension is venodilatation, and this is more pronounced with propofol than thiopentone A = ? True * The reduction in preload is the main contributor to hypotension according to Fundamentals of Anaesthesia, p576 B = False * Reduced C = False * The statement is true, but this is not the main mechanism of hypotension D = ? False * A true statement in terms of loss of consciousness, but not necesscarily the cause of hypotension E = ? False * For the same reasons as D

Accidentally cannulate carotid artery with 5 lumen 7 Fr CVC preop for a semi urgent CABG. MOST appropriate next response is to: A. Get vascular surgeon to repair it and continue with surgery and heparin B. Leave it in. Do CABG. Pull it out post op. C. Pull it out, compress. Delay surgery for 24hrs D. Pull it out compress. Continue with surgery + heparin. E. Pull it out. Compress. Continue with surgery no heparin.

ANSWER A Inadvertent carotid arterial puncture -incidence 2-8% (probably lower with US) Complications -haematoma -Airway obstruction -pseudoaneurysm -arterio-venous fistula formation -retrograde aortic dissection If the carotid artery was punctured by a small bore needle or cannula (18-gauge or smaller), then serious damage was unlikely to result. Risk of complications dramatically increase with -coagulopathy -dilator =these should be surgically repaired The decision to proceed with surgery should thus be based on the following considerations: 1. The urgency of surgery. 2. The general condition of the patient. 3. The possibility that other vascular damage has occurred 4. The possibility that arterial thrombosis, dissection or embolisation might precipitate a neurological event.

Trauma with # pelvis and femur. Has a splenectomy. Day 2 patient thrombocytopenia, confused and hypoxic. Diagnosis is a. fat embolism syndrome b. pneumcoccal pneumonia c. thromboembolis

ANSWER A Incidence Fat embolism occurs in < 10% of trauma cases, but is present in about 90% of fatal trauma cases. Fat (& marrow) embolism occurs very quickly after the injury (eg see reference below about Mt Erebus crash), but the clinical syndrome typically presents from 24-72hrs post injury Associations * major trauma - this is responsible for 95% of all fat embolism o long bone fractures o pelvic fractures o severe soft tissue injuries including burns * iatrogenic causes o elective major orthopaedic procedures o liposuction o bone marrow harvesting and transplant o glucocorticoids o lipid infusion * rarer causes o pancreatitis o diabetes o osteomyelitis and panniculitis o sickle cell crisis o acute fatty liver Clinical Features * respiratory changes o acute lung injury through to ARDS * neurological abnormalities o acute confusional state / delerium is very common o focal neurological signs can occur o seizures rarely * haematological abnormalities o petechial rash in about 50% o DIC if severe Treatment * supportive * corticosteroids shown to helpful for prophylaxis o they appear unhelpful for the treatement of established fat embolism however * aspirin may be useful o heparin appears not to be

A 78 year old man with past history of a difficult intubation presents for arm surgery. Technique is supraclavicular block using 0.5% bupivacaine. Shortly after, started convulsing. INITIAL management? A. Midazolam 5mg B. Intralipid 20% 1.5 ml/kg C. Thiopentone 150mg D. Suxamethonium E. Propofol 50mg

ANSWER A Initial management should be ABC and terminate seizures. Midazalam is preferred as it has the least cardiovascular depression. -2-5mg IV Propofol 1mg/kg and thiopentone 50mg doses can be used as boluses can be used, however they cause hypotension and can lead to refractory CV collapse ASRA Practice Advisory Guidlines on LA toxicity

Which nerves need to blocked to anaesthetise the hard palate: A. Superior labial nerve and greater palatine nerve B. Greater palatine nerve and nasopalatine nerve C. Inferior orbital nerve and nasopalatine nerve D. Glossopharyngeal nerve and... E. Anterior ethmoidal nerve and...

ANSWER A Innervation of the palate -sensory nerves of the palate are branches of the maxillary nerve which branch from the pterygopalatine ganglion -greater palatine nerve supplies gingiva, mucous membrane and glands of most of hard palate -nasopalatine nerve supplies mucous membrane of anterior part of hard palate -lesser palatine supply soft palate

The most important effect of Lugol's iodine administration prior to thyroid surgery is A. a reduced incidence of thyroid storm B. a reduced incidence of vocal cord palsy C. an increased chance of preservation of the parathyroid glands D. increased pigmentation to assist in gland identification E. reduced vascularity of the gland

ANSWER A Lugol's iodine * 5% iodine and 10% potassium iodide distilled in water * administered preoperatively to reduce the release of thyroid hormone and the vascularity -3 drops twice daily beginning 10 days preop -in thyroid storm it can be given IV 0.5-1gm every 8-12 hours. However, more rapid control of the hyperthyroid state can be achieved with beta blockers, thionamides.

MZ79 [Jul07] In Marfan's Syndrome which is NOT related: A. If develop aortic disease most likely to be aortic stenosis B. At risk iliac aneurysm C. development of mitral valve prolapse is more likely than in general population D. Cardiac myopathy due to medial cystic necrosis/atrophy/ degeneration E. Intracranial aneurysm

ANSWER A MARFANS SYNDROME OVERVIEW • AD inheritance • mutation in fibrillin 1 gene (connective tissue protein) • mean age survival 32years, death almost always due to cardiovascular abnormalities CLINICAL FEATURES Skeletal • Tall stature • Long tubular bones, joint hyper-extensibility • high arched palate Resp • pectus excavatum • kyphoscoliosis • high incidence spontaneous pneumothorax Ocular • lens dislocation, myopia, retinal detachment Cardiac • aortic dissection, aortic regurgitation • MVP • cardiac conduction abnormalities esp BBB ANAESTHETIC IMPLICATIONS • Airway difficulties are uncommon. TMJ dislocation is possible • Avoid sustained increases in systolic blood pressure in view of risk of aortic dissection, consider peri-op B-blockade & invasive monitoring • high index of suspicion for development of pneumothorax • easy joint dislocation

Mode is a statistical term relating to: A. Most common number in a group B. Average value C. Middle value ie. equal number of values on either side of it D. Measure of precision E. 25th Quartile

ANSWER A MODE is the value that occurs most frequently in a data set or a probability distribution. In a discrete probability distribution, it is the value which takes the maximum value, ei most likely to be sampled In a continuous probability distribution, it is the value which attains the maximum value, ei the peak. MEAN is the average value. MEDIAN is the middle value.

18/12 old undergoing routine SV GA under LMA. Sudden onset SVT with HR 220 BP 84/60 ETCO2 32 SpO2 98.Management: A. Adenosine 100mcg B. DCR 2J/kg C. DCR 4J/kg D. Amiodarone 5mg/kg E. CPR

ANSWER A Management of SVT in a child: • Attempt valsalva maneuvers as able by child, ie; forceful cough, cold packs on face and neck, or blow through a straw • Establish IV/IO NS. Do not withhold IO if unable to start IV promptly and child is unstable • If unable to maintain airway, consider endotracheal Intubation • If child is rapidly deteriorating perform immediate synchronized cardioversion at 0.5J/kg. Consider pain management or sedation • If time allows may try Adenosine first at 0.1mg/kg IV or IO, being sure to administer rapidly and follow with a rapid fluid flush. During administration, record a rhythm strip. • Consider Amiodarone.

A patient with chronic pain using morphine PCA after lower limb orthopaedic surgery. Daily usage of IV morphine works out at about 400mg/day. What dose of oral methadone would you start him on to replace the morphine? A. 60mg/day B. 120mg/day C. 400mg/day D. 600mg/day E: 1200mg/day

ANSWER A Methadone replacement depends on dose of oral morphine Daily Dose : Conversion Ratio <100mg 3:1 100 - 300mg 5:1 300 - 600mg 10:1 600mg - 800mg 12:1 800mg - 1000mg 15:1 >1000mg 20:1 400mg IV = 1200mg oral. 20:1 conversion so 60mg of methadone per day = A

75 year old male with normal renal function for an endoluminal aortic repair. What is the best protection to prevent the development of renal dysfunction? A: NaCl B: NAC C: mannitol D: dopamine E: dialysis

ANSWER A Methods to reduce AKI after contrast (?is this adaptable to EVAR) -Witholding nephrotoxic drugs: NSAIDs, aminoglycosides, metformin etc Volume expansion -well established in AKi -evidence indicated normal saline or CSL is more effective than half normal saline -consider bicarbonate solutions -maintain UO>2ml/kg Dialysis or hemofiltration -no evidence for prophylactic dialysis with normal renal function -evidence in CRI -may be confounded by hemofiltration ensures adequate intravascular volume Pharmacological -no approved agents for prevention of AKI -Ascorbic acid 3g nocte before and 2g bd -multicentre RCT, placebo controlled showed reduced incidence -NAC not consistently shown to be effect, meta analysis showed no effect Fenoldopam, dopamine, calcium-channel blockers, atrial natriuretic peptide, and L-arginine have not been shown to be effective in the prevention of contrast-induced AKI. Furosemide, mannitol, and an endothelin receptor antagonist are potentially detrimental

75 year old male with normal renal function for an endoluminal aortic repair. What is the best protection to prevent the development of renal dysfunction? A: NaCl B: NAC C: mannitol D: dopamine E: dialysis

ANSWER A Methods to reduce AKI after contrast (?is this adaptable to EVAR) -Witholding nephrotoxic drugs: NSAIDs, aminoglycosides, metformin etc Volume expansion -well established in AKi -evidence indicated normal saline or CSL is more effective than half normal saline -consider bicarbonate solutions -maintain UO>2ml/kg Dialysis or hemofiltration -no evidence for prophylactic dialysis with normal renal function -evidence in CRI -may be confounded by hemofiltration ensures adequate intravascular volume Pharmacological -no approved agents for prevention of AKI -Ascorbic acid 3g nocte before and 2g bd -multicentre RCT, placebo controlled showed reduced incidence -NAC not consistently shown to be effect, meta analysis showed no effect Fenoldopam, dopamine, calcium-channel blockers, atrial natriuretic peptide, and L-arginine have not been shown to be effective in the prevention of contrast-induced AKI. Furosemide, mannitol, and an endothelin receptor antagonist are potentially detrimental

During surgery for tracheostomy insertion, surgical diathermy is being used at the tracheal incision. You are ventilating with 100% oxygen. As the trachea is opened you notice a blue flame shooting up from the incision. Your first action should be to A. disconnect the breathing circuit from the endotracheal tube B. douse the wound with saline C. insert a tracheostomy tube D. remove the endotracheal tube E. turn off oxygen and ventilate with air

ANSWER A Miller covers this under laser anaesthesia airway fire protocol chapter 67: 1. remove source of flame (Surgeon) 2. stop ventilation (Anaesthetic) 3. disconnect breathing circuit 4. extinguish with bucket of water, flood oropharynx, consider flushing saline down ETT to extinguish intraluminal fire 5. consider removing ETT and ventilate with 100% O2 via face mask (doesn't actually talk about removing the ETT but I got the impression this is when it would be taken out - when no flame, no gas to combust) 6. re-intubate and assess damage and remove debris - direct laryngoscopy, rigid bronchoscopy, +/- gentle lavage and distal FOB 7. If damage, retube and if severe damage may need low tracheostomy 8. Consider steriods and antiobitcs 9. Post event CXR Prevention of airway fires for surgeric tracheostomy insertion : The following anaesthetic and surgical recommendations are suggested. * Anaesthetic 1. Use the minimum concentration of oxygen to maintain arterial saturation. 2. Use other non-oxidizing agents such as helium, air or nitrogen in place of nitrous oxide. 3. Position the ETT near the carina to minimize exposure of the cuff to injury in opening the trachea. 4. A good seal on the cuff is essential to prevent leaking of anaesthetic gases. 5. Fill the cuff with water to act as a fire retardant. 6. Consider using fire resistant ETT (silicone/metallic) often used in laser surgery. * Surgical 1. Minimize use of diathermy particularly once the trachea is opened. 2. Use bipolar diathermy if bleeding occurs once the trachea is opened. 3. Use suction to remove oxygen rapidly from the field. 4. Surround the operating field with moist pack if diathermy has to be used. 5. Always have sterile saline at hand in case of fire.

Newborn with diaphragmatic hernia. Initially sats 95% on RA. Now beginning to develop respiratory distress. Next appropriate step: A. Awake intubation B. Trial NO prior to intubation and ventilation C. Rapid sequence induction D. Mask ventilation throughout case with CPAP E. Gas down with CPAP

ANSWER A Miller, Yao and Artusio, and Morgan Mikhail all say awake intubation Assumption: this occurs immediately post delivery Problems 1. pt has pulmonary hypoplasia and pulmonary HTN with a transitional circulation apnoea, hypoxia and hypercarbia will lead to reversal of shunts If this neonate is desaturating, it needs to be intubated immediately. Waiting for IV access (difficult in a neonate) may not be possible. However, awake intubations due increase risk of ICH (neonates are already at risk of this, this increases the risk further)

The chief concern in the anaesthetic management of a patient with mitral stenosis is to avoid tachycardia because it A. increases the trans-mitral pressure gradient B. aggravates left ventricular ischaemia C. increases myocardial oxygen consumption D. frequently precipitates atrial fibrillation E. reduces the size of the left atrium

ANSWER A Mitral stenosis is characterized by mechanical obstruction to left ventricular diastolic filling secondary to a progressive decrease in mitral valve orifice size. This leads to a relatively fixed CO in combination with elevated LA pressures. Epidemiology Most common cause is rheumatic fever, and primarily affects females. These patients tend to have mixed MR / MS; a minority willl have pure MS. Patients remain asymptomatic for 15-20 years. Symptoms appear at 31 years on average with 50% mortality at 5 years if not corrected. Much less common causes of mitral stenosis include carcinoid syndrome, left atrial myxoma, severe mitral annular calcification, thrombus formation, cor triatriatum, rheumatoid arthritis, systemic lupus erythematosus, and congenital mitral stenosis. Pathophysiology * Underfilled LV with fixed CO due to stenotic valve; the LV has reduced preload reserve. With mild mitral stenosis, left ventricular filling and stroke volume are usually maintained at rest by an increase in left atrial pressure. However, stroke volume will decrease during stress-induced tachycardia (as diastolic filling is decreased) or when effective atrial contraction is lost as with atrial fibrillation. * Elevated left atrial pressure to produce a diastolic pressure gradient across stenosed mitral valve. This eventually results in elevated pulmonary venous pressures, and also pulmonary arterial hypertension. * LA dilatation initially is compensatory and keeps PAP low. However this fails eventually. Complications * atrial fibrillation occurs as a result of LA dilatation and fibrosis of the atrial wall. * Acute pulmonary oedema can occur if LAP rises suddenly (eg AF, fluid overload, autotransfusion with oxytocics). * Pulmonary venous congestion causes airway narrowing (increases work of breathing and airway resistance), transudation of fluid into pulmonary interstitial space (reducing pulmonary compliance) and redistribution of blood from bases to apices (V/Q mismatch). * Pulmonary arterial hypertension occurs 5 - 10 years after the onset of symptoms. Initially PVR elevation is reversible, but chronically elevated PVR causes pulmonary artery medial hypertrophy and fibrosis leading to an irreversible rise in PVR. * RV failure occurs as a result of PAH, when PAP are >70mmHg. * Thromboembolism occurs commonly in MS, correlating with the patient's age and size of the LA appendage. 80% of patients who develop systemic embolic are in atrial fibrillation. Presentation : History * Dyspnoea on exertion / fatigue * PND, Orthopnoea * Haemoptysis (due to rupture of bronchopulmonary varices) * Recurrent bronchitis Presentation : Examination * Mitral facies - malar flush on cheeks (dilated capillaries in combination with peripheral cyanosis) * Peripheral cyanosis - due to poor CO * AF * Signs of RVF (elevated JVP, hepatomegaly, peripheral oedema, ascites). * Tapping apex beat (due to palpable S1 - valve cusps widely apart at onset of systole) * Diastolic thrill * Pulmonary HTN (RV heave, palpable S2) * Loud S1 * Opening snap if in SR (at LSE - high LAP forcing cusps apart, but valve cone is abruptly halted) * Mid-diastolic, low pitched rumbling murmur best heard at apex%%Medical Management * Diuretics - symptom relief in mild disease * DCR * Rate control of AF (digoxin, beta blockers, Ca channel blockers) * Anticoagulation Surgical Managment Indicated when symptoms worsen or when pulmonary hypertension develops. May be balloon valvuloplasty if suitable (in young patients with non-regurgitant valves that are not grossly affected), open mitral valvotomy/repair or replacement. Anaesthetic Management Pre-Operative *Control rate esp if in AF. Digoxin is useful in patients in chronic AF. *Operative risk correlated with the severity of the stenosis of the value Intra-Operative * Firstly, manage anti-coagulation cessation (Only applies to patients on warfarin for AF) * Secondly, antibiotic prophylaxis for bacterial endocarditis as per new guidelines (so rarely needed) * Thirdly, optimise haemodynamics (Applies mostly to patients with MV area is less than 1.5 cm2) * Haemodynamic goals are Slow, full and tight for stenotic lesions. Anaesthetic Technique * No anaesthetic technique has been demonstrated to be safer but GA may be more CVS stable. * Asymptomatic patients do not require additional monitoring. IAL, CVC, PAFC and TOE may all be considered for patients with moderate to severe disease having major surgery. * Careful induction of anaesthesia with small titrated doses of propofol or an opioid dominant technique is suitable. Ketamine should be avoided. Haemodynamic response to intubation should be adequately blunted. * Ensure deep anaesthesia - sevoflurane or opioids can be used safely. Avoid muscle relaxants with significant haemodynamic side effects (eg atracurium, mivacurium, pancuronium). Rhythm and rate * Sinus rhythm is preferred if possible for mild disease(eg preop cardioversion or drug conversion) * In moderate to severe MS atrial contraction are ineffective due to dilatation; rate control is paramount. * ensure rate is controlled (60-80/min is best) whether in sinus rhythm or in AF * avoid tachycardia (May require beta-blockers eg Esmolol if intraop tachycardia); * avoid bradycardia. (Bradycardia causes problems because of the fixed flow rate across the mitral valve. A decrease in heart rate means a decrease in cardiac output) * Cardioversion is an option if AF develops intraop Maintain Preload Margin for error is small. Avoid hypovolaemia and replace losses early and appropriately. If the preload (LA pressure) is increased the risk of pulmonary congestion is increased so excessive volume loading (IV, head down positioning) is also quite deleterious; may cause APO. Decreased preload is poorly tolerated; decreased LAP will reduce the gradient across the stenosed valve. Note that preload can be decreased by high ventilation pressures or the head-up position. (Note that the use of the term 'preload' here refers to LA pressure and not LV diastolic pressure or volume. This is because the LV is not well filled in severe MS and LVEDP will be normal or low so the preload referred to is the pressure upstream of the mitral valve ie LAP). Mantain afterload Minimal margin for errors. Vasodilatation results in reduced LA filling; the SV is fixed and cannot compensate resulting in severe hypotension. A 'pure' alpha vasopressor (eg phenylephrine) may be required. Reduce PVR Avoid hypercapnia, hypoxia, hypothermia and acidosis These are common recommendations in the literature because these factors are said to cause increased pulmonary artery pressure, and could possibly precipitate pulmonary oedema in patients with severe MS. Maintain Contractility To maximise SV from limited filling. Post Operative Optimal haemodynamics should be maintained postoperatively. This means initial ICU management if MS is severe. Mild & moderate MS is usually managed in a ward setting if the patient does not have major co-morbidities and there are no other reasons for ICU admission. Anti-coagulation may need to be recommenced as the situation allows.

A patient develops a nerve injury post-operatively after a long surgery in the lithotomy position. What would you most likely see a loss of? A. dorsi flexion B. plantar flexion C. inversion D. sensation to the lateral aspect of the foot

ANSWER A Most commonly injured lower extremity nerve in patients undergoing surgery in lithotomy position was the common peroneal nerve (L4-S2, represented 78% of nerve injuries in a study which looked retrospectively at 198,461 patients undergoing surgery in the lithotomy position). Etiology: Is thought to be secondary to compression of the nerve between the lateral head of the fibula and the bar holding the legs. When the candy cane stirrups are used, special attention must be paid to avoid compression. Associated factors: patients with a low body mass index, recent cigarette smoking, prolonged duration of surgery A prospective review of 991 patients undergoing surgery in the lithotomy position from 1997 to 1998, showed that there were no motor neuropathies in the lower extremity, although paresthesias in the distribution of the obturator, lateral femoral cutaneous, sciatic, and peroneal nerves were found [Warner MA et al. Anesthesiology 93: 938, 2000] Nerve Injuries in Lithotomy Position * Most Commmon: common peroneal (78%) * Etiology: compression * Risk Factors: low BMI, smoking, prolonged surgery * Manifestation: common peroneal is L4-S2, responsible for foot dorsiflextion and toe extension (thus leads to foot drop)

The nerve which supplies the lobule of the ear is the A. greater auricular B. lesser occipital C. auricular branch of the vagus D. auriculo-temporal E. greater occipital

ANSWER A Multiple nerves supply the ear. Greater Auricular Nerve -cervical plexus -posterior surface of ear and lower third anterior surface Auriculotemporal nerve -branch of the mandibular division of the trigeminal nerve -superior 2/3 of the anterior surface Auricular branch of vagus -sensory supply to the external auditory meatus and lower part of the tympanum Greater occipital nerve -spinal nerve of dorsal primary ramus of C1 and C2 (along with lesser occipital nerve) -emerges from the subocciptial triangle obliquely between inferior oblique and semispinalis capitis muscle -passes through trapezius -innervates the skin along the posterior part of the scalp to vertex Lesser occipital -cutaneous nerve arising form C2 and C3 -follows greater occipital -innervates the scalp in the lateral area posterior to ear

AZ02 [Jul07] Fit and healthy young female for lap. gyne operation, which of the following doesn't require continuous monitoring (See PS 18): A. ECG B. Saturation probe C. Disconnect alarm D. Oxygen analyzer E. Capnography

ANSWER A Must be monitored continuously 1. Ventilation : continuously by direct and indirect means : capnography 2. Oxygenation : oximeter 3. Intermittent non-invasive blood pressure monitor 4. Oxygen analyiser with audible alarm to warm of low oxygen concentrations 5. Breathing system disconnection or ventilator failure alarm 6. Volatile anaesthetic agent concentration monitor if using volatile 7. Must be available 1. Electrocardiogram 2. Continuous intra-arterial blood pressure monitor 3. Temperature probe 4. NMB monitoring 5. BIS 6. CVP 7. spirometry 8. TOE

Thallium scan: A. High negative predictive value B. High positive predictive value C. Not as good as a dobutamine stress echocardiography D. Requires the patient to exercise E. Not effected by caffiene

ANSWER A NPV 95% PPV 30% Probably the same as dobutamine stress echo

In relation to nausea during obstetric regional anaesthesia A. atropine is more effective treatment than vasopressors when there is a high spinal block B. nausea is worse with phenylephrine infusion compared to ephedrine infusion C. phenylephrine increases the emetic effect of decreased preload D. metoclopramide is the treatment of choice E. ondansetron is the treatment of choice

ANSWER A Nausea and vomiting may have been secondary to an absolute, or relative, increase in vagal tone. There is evidence for a vagal mechanism causing nausea during spinal anesthesia. Atropine has been found to be more effective at treating nausea associated with high spinal anesthesia than vasopressors. More recently, glycopyrrolate has been found to reduce nausea during spinal anesthesia for cesarean delivery

The electrical requirement that distinguishes a "cardiac protected area" from a "body protected area" is the A. equipotential earth B. isolation transformer C. line isolation monitor D. maximum leakage current to patient limit of 500 microamperes E. residual current device

ANSWER A Need a class A area for a cardiac protected area. Class Z -Fuse Boxes which limit the maximum current through the active wire. -8 Amp fusewire for lighting circuits -15 Amp for power -Standards limiting the amount of current through any set of power outlets. Class B -As for Class Z Areas plus: -Earth-leakage detection devices must be used, ie Earth Leakage Core Balance Relays (RCD's) or Isolating transformers with Line Isolation Monitors. -The earth wiring must be used and tested regularly for low resistance. -All equipment having patient circuits must be class BF or CF. Equipotential Earth = Class A

Patient has IDDM and stable angina. Present for cataract extration. Surgeon was not willing to do it with topical LA, but you accidently block the WRONG eye. After explanation and apologising to pt, what do you do next? A. cancel surgery and re schedule on a day that is convenient to patient B. cancel surgery, do not rebook case until an incident form has been processed and you are aware of the outcome of the enquiry C. give a GA D. block the other eye (topical) and continue (edit: i think this stem was proceed with eye block to correct side - definitely a block I agree) E. proceed under topical LA (edit: convince the surgeon to do the correct eye with topical anaethesia)

ANSWER A No references, but seems the most logical approach

What statistical test would be best to evaluate the effects of ? 2 drugs in patients at ? 3 different points in time A. ANOVA B. Mantel Hantzel C. Crusckall Wallis D. Students t test

ANSWER A Numerical data but more than 2 groups therefore ANOVA Analysis of variance (ANOVA) is a collection of statistical models used to analyze the differences between group means and their associated procedures (such as "variation" among and between groups). In ANOVA setting, the observed variance in a particular variable is partitioned into components attributable to different sources of variation. In its simplest form, ANOVA provides a statistical test of whether or not the means of several groups are equal, and therefore generalizes t-test to more than two groups. Doing multiple two-sample t-tests would result in an increased chance of committing a type I error. For this reason, ANOVAs are useful in comparing (testing) three or more means (groups or variables) for statistical significance.

Characteristic cardio-pulmonary effects of pulmonary thromboembolism include: A. hypoxaemia due to excess perfusion of lung units with a low V/Q ratio B. hypercarbia due to an increase in physiological dead-space C. reverse splitting of the second heart sound D. an increase in compliance of the left ventricle E. an increase in coronary blood flow to the right ventricle during systole

ANSWER A PE physiology (Nunn p551) * ↑ PVR 1. Physical occlusion 2. Platelet activation in thrombus → 5HT and TXA2 release → vasoconstriction → ↑ PVR * Respiratory lesion o ↑ alveolar deadspace o ↑ A-a gradient + Normal to low PaCO2 (in SV) because ↑ RR ? due to J-receptor stimulation + hypoxia * ↓ PaO2 o Deranged V/Q relationships o ↓ CO → low mixed venous O2 * Bronchospasm due to 5HT release from platelets * ↓ pulmonary compliance (still unknown mechanism)

Based on the information in the 2x2 contingency table, what is the chance of having the disease if the test is positive? Disease Yes No Test Positive 80% 10% Negative 20% 90% A. 0.89 B. 0.8 C. 0.67 D. 0.5 E. <0.3 D. ?

ANSWER A PPV, which = TP/(TP+FP) which in this case = 0.89

Bowel surgery patient. Best method for intraoperative optimization of fluid therapy? A. Arterial pulse pressure contour analysis B. CVP C. PAOP D. UO

ANSWER A Perhaps SVV or oesophageal doppler would be better? Update in Anaesthesia - Enhanced recovery after surgery - current trends in perioperative care "Accurate fluid management and resuscitation requires regular reassessment of physiological parameters and, where available, invasive haemodynamic monitoring. Historically this has been provided by pulmonary artery catheters, but these are increasingly being replaced by targeted stroke volume optimisation with oesophageal Doppler probes. Given the relative simplicity and lack of complications, where available the latter is the recommended method of guiding fluid administration in the operating room"

A 60 year old female is undergoing hysterectomy. Gabapentin reduces postoperative: A. Nausea B. Vomiting C. Sedation D. Pruritus E. Constipation

ANSWER A Perioperative gabapentinoids (gabepentin/pregabalin) reduce postoperative pain and opiods requirements and reduce the incidence of vomiting, pruritus and urinary retention but increase the risk of sedation.

Complications, which usually present early following pneumonectomy, include all of the following EXCEPT A. Broncho-pleural fistula B. Respiratory failure C. Right heart failure D. Right to left shunt E. Supraventricular arrhythmia

ANSWER A Pneumonectomy is the surgical removal of an entire lung -mainly for bronchogenic carcinoma in main stem bronchus -rarely for pulmonary metastases, pulmonary tuberculos, fungal infections and broncheicatasis, traumatic lung injury, congential disease and bronchial obstruction IMMEDIATE Contralateral pneumonthorax Sputum retention Prolonged air leakage Haemorrage Phrenic nerve injury R-L shunt through PFO due to increase in PA pressures Acute respiratory insufficiency EARLY A. Respiratory : -ARDS/post pneumonectomy pulmonary oedma - treatment is supportive, mortality>50% -hemothorax, chylothorax -empyema B. Cardiac -arrthymias AF -AMI -PE -Cardiac herniation LATE A. postpneumonectomy syndrome -extrinsic compression of distal trachea and mainstem bronchus due to shifting of the mediastinum -progressive dyspnea, cough, inspiratroy stridor, recurrent pneumonia after 6 months of surgery -treatment surgical repositioning of mediastinum B. Bronchopleural fistula -1.5-4.5%, mortality of 30-70% -assocaited with empyema C. Esophagopleural fistula

Patient post anterior cervical fusion. Patient in recovery. Confused and combative. Nurse concerned about haematoma Taken to theatre: Most appropriate way of securing airway: A. Gas induction / laryngoscopy / intubate B. Awake tracheostomy C. Awake fibreoptic intubation using minimal sedation D. Thiopentone, suxamethonium, direct laryngoscopy and intubation E. Retrograde intubation

ANSWER A Post ACDF -long procedure -edematous airway -anterior haematoma Safest would be AFO But in this case the patient is confused and combative -inhalation induction

Spinal headache which is INCORRECT A. Occurs within 24 hours B. Neck pain C. Can be associated with intracranial haemorrhage D. Can be treated with IV caffeine E. Frontal headache

ANSWER A Post Dural Puncture Headache Symptoms -30-70% with known ADP will develop PDPH -rare for spinal to cause headache -severe, dull and non-trobbing headache -fronto-occiptial -aggrevated when sits up or strains -may be accompanied by nausea, vomiting, anorexia, visual distrubances (photophobia, blurred vision), audio disturbances (tinnitus) or neck stiffness Assessment / Investigations -the nature of the headache and should include the following: * time of onset * location * severity (? interference with daily activities) * postural element (i.e. Is the headache worse when sitting up / standing?) * visual / audio disturbances * analgesia given and if there has been some relief A full neurological assessment should also be performed. Management -Recognised dural puncture at time of epidural insertion. * If needle puncture, thread catheter into subarachnoid space. * If catheter puncture, leave catheter in CSF. * Ensure that the catheter / filter is clearly labelled as 'subarachnoid'. OR * Alternatively, attempt to re-site the catheter at another space. If this is difficult, there is uncertainty with correct placement or another puncture occurs, contact the anaesthetic consultant in charge. * subarachnoid boluses must be given by the anaesthetic registrar. * The PCEA or continuous infusion techniques should not be utilised as there is an increased risk for total spinal block. * The woman must be informed of the ADP and a management plan discussed with her. This management plan must be documented in the progress notes.%%For women where a PDPH has been identified, the following steps should be taken: 0-24 hours following dural puncture Conservative management is best as PDHP is self-limiting process 75% will resolve in first week 90% in 6 weeks 1. Encourage fluid intake Ensures that the rate of CSF production is adequate. It is thought that improvements in the ratio of CSF production to CSF leak will improve the outcome. Dehydration can result in a decrease in CSF production. Conversely, in the presence of adequate hydration, there is no evidence to suggest that overhydration will increase the production of CSF any further. 2. Bedrest Lying in the flat supine position is advised to lessen the severity of symptoms. If the headache is mild and does not interfere with daily activities, then bed rest is not necessary. 3. Analgesia Mild PDPH * paracetamol 4-6 hourly and a NSAID such as voltaren (unless contraindicated) Moderate - Severe PDPH * treat as with mild PDPH (above) plus an opioid (e.g. oxycodone and/or tramadol) * SC, IV or PR routes should be considered in the presence of nausea and vomiting. *use of caffeine is reported 500mg of caffeine sodium benzoate in one litre of fluid, infused over an hour -oral caffeine is only slightly better than placebo -vasoconstriction of cerebral vessls attentuates the vascular distention -avoid in pre-eclamptic patients who are have increased sensitivity to vasoconstrictiors 24-48 hours following dural puncture Women with a mild PDPH, which does not interfere with daily activities, then the above management should be continued. Where the PDPH is moderate-severe and is interfering with daily activities, an epidural blood patch should be considered and discussed with the women. An epidural blood patch should not be performed within the first 24 hours following dural puncture. The anaesthetic consultant in charge should be notified when an epidural blood patch is considered.

Mar 2011 Post CEA on ward, patient seizes. BP has been hard to control. What to do to prevent further seizures? A: Add another antihypertensive B: Start antiplatelet drugs C: Start anticonvulsants D: Do angio and stent E: Nimodipine

ANSWER A Post operative hyperperfusion syndrome. Confusion, decr GCS, seizures. BP should be actively managed, since this Sx has high mortality. CEACCP CEA.

Features of Conn's syndrome (primary hyperaldosteronism) include each of the following EXCEPT A. acidosis B. high urinary potassium C. hypertension D. hypokalaemia E. low plasma renin

ANSWER A Primary hyperaldosteronism is one of the more common causes of secondary hypertension. The term Conn's syndrome is used for primary hyperaldosteronism secondary to an adrenal adenoma secreting aldosterone. The syndrome is due to: * bilateral idiopathic adrenal hyperplasia 70 % * unilateral idiopathic adrenal hyperplasia 20 % * aldosterone-secreting adrenal adenoma (benign tumor, < 5%) * rare forms, including disorders of the renin-angiotensin system Aldosterone acts in kidney to increase Na+ reabsorption and increase K+ secretion. -Water follows Na, therefore increase in extracellular fluid volume -hypokalaemia -metabolic alkalosis -Hypomagnesemia -hypoglycaemia Clinical Features -refractory hypertension, hypervolaemia, metabolic alkolosis -hypokaelaemia -muscle weakness or paralysis -nephrogenic diabetes insipidus -impaired glucose tolerance Treatment is adrenalectomy 1. Medical optimisation a. spironoloctone (400mg/day, compeditively inhibits aldosterone) b. correct K+, HCO3 c. Treat hypertension d. exclude LVH, treat as for CCF if present

Which of the following are characteristic of hyperaldosteronism? A. Low NaCl in serum B. Low NaCl in urine C. High K in serum D. High NaCl in urine E. Metabolic acidosis

ANSWER A Primary hyperaldosteronism is one of the more common causes of secondary hypertension. The term Conn's syndrome is used for primary hyperaldosteronism secondary to an adrenal adenoma secreting aldosterone. The syndrome is due to: * bilateral idiopathic adrenal hyperplasia 70 % * unilateral idiopathic adrenal hyperplasia 20 % * aldosterone-secreting adrenal adenoma (benign tumor, < 5%) * rare forms, including disorders of the renin-angiotensin system Aldosterone acts in kidney to increase Na+ reabsorption and increase K+ secretion. Consequently serum K+ is low: hypokalaemia. The result is hypokalaemic metabolic alkalosis due to H+ ion loss Hypomagnesemia and hypoglycaemia may also be present

Bleeding patient. What is relative contraindication to Prothrombinex? A. History of HITS B. Von Willebrands C. Haemophilia B D. Warfarin overdose ?? or - Overdose vit K (not warfarin) E. Renal failure

ANSWER A Prothombinex contains 192 unit of heparin

ECG- which does NOT have abnormal Q waves: A: Digoxin toxicity B: Anterior myocardial infartion C: Previous AMI D: LBBB E: Wolff-Parkinson-White syndrome

ANSWER A Q Wave * A Q wave is any negative deflection that precedes an R wave Normal Q wave Origin of the Q Wave * The Q wave represents the normal left-to-right depolarisation of the interventricular septum * Small 'septal' Q waves are typically seen in the left-sided leads (I, aVL, V5 and V6) Q waves in different leads * Small Q waves are normal in most leads * Deeper Q waves (>2 mm) may be seen in leads III and aVR as a normal variant * Under normal circumstances, Q waves are not seen in the right-sided leads (V1-3) Pathological Q Waves Q waves are considered pathological if: * > 40 ms (1 mm) wide * > 2 mm deep * > 25% of depth of QRS complex * Seen in leads V1-3 Pathological Q waves usually indicate current or prior myocardial infarction. Differential diagnosis: * myocardial infarction * cardiomyopathies — hypertrophic (HOCM), infiltrative * rotation of the heart — extreme clockwise or counter-clockwise rotation * lead placement errors — e.g. upper limb leads placed on lower limbs Loss of normal Q waves The absence of small septal Q waves in leads V5-6 should be considered abnormal. Absent Q waves in V5-6 is most commonly due to LBBB.

A common finding in hereditary prolonged QT syndrome is A. bifid T waves B. inverted P waves C. prolonged QRS complex D. R axis deviation E. short PR interval

ANSWER A QT interval is generally measured in lead II, as the T-wave ending is usually discrete and the QT interval in lead II has a good correlation with the maximal QT measurement from the whole 12-lead ECG. In many LQTS, the QT interval is not prolonged but have increased variability known as QT dispersion. QTD is significantly increased in symptomatic LQTS patients, but may not be signifcantly different to control values in asymptomatic LQTS patients. T wave and U wave abnormalities are common in LQTS. T waves may be larger, prolonged, or have a notched, bifid or biphasic appearance. A pathognomonic feature of LQTS is so-called T wave alternans, where there is beat-tobeat variation in T wave amplitude. Exercise testing of patients with LQTS may provoke prolongation of the QTc. A notched T wave during the recovery phase of exercise is highly suggestive of LQTS. Head up tilt testing may also provoke abnormal QT prolongation and arrhythmias.

A home handyman leaves his electricity turned on whilst fiddling with wires [repairing a power outlet]. He has a RCD. What happens if he touches the neutral and ground wires? A. Nothing will happen B. Receives macroshock C. Protected from macroshock by RCD D. Protected from microshock by domestic fuse E. Receives microshock

ANSWER A RCD protects by constantly monitoring the current flowing in the live and neutral wires supplying a circuit or an individual item of equipment. Under normal circumstances, the current flowing in the two wires is equal. When an earth leakage occurs due to a fault in the circuit or an accident with the equipment, an imbalance occurs and this is detected by the RCD, which automatically cuts off the power before injury or damage can result. What an RCD will not protect: If a user was to touch both active and neutral at the same time, then current flow would be even through both circuits, there is no imbalance so the RCD WON'T TRIP A: Nothing will happen, Neutral and Ground are connected at the fuse box so they are essentially the same wire so no potential difference between them. RCD will not trip in this case as no current is leaking out of system

When do most patients with SAH rebleed? A. 0-24 hours B. 1-2 days C. 4-10 days D. 14-28 days

ANSWER A Rebleeding after aneurysmal subarachnoid hemorrhage. Neurocrit Care. 2011 Sep;15(2):241-6 "Rebleeding after initial aneurysmal subarachnoid hemorrhage (SAH) can have substantial impact on overall patient outcome. While older studies have suggested rebleeding occurs in about 4% of patients during the first day after initial aneurysmal bleed"

A 70kg male who has been taking prenisolone 10mg per day orally for over 12 months, undergoes an uncomplicated laparotomy and bowel resection. The best management of his steroid therapy perioperatively, until oral intake resumes, is to administer A. Hydrocortisone 100mg iv for approximately 24 hours B. Hydrocortisone 200mg per day for approx 72 hours C. Hydrocortisone 400mg per day for 72 hrs D. No steroids E. 20mg nasogastric prednisolone daily

ANSWER A Recognised guidelines (for someone on 10mg or more in last 3 months) would be to give normal oral dose, 25mg hydrocortisone at induction and 25mg 6 hrly for 2-3 days (major surgery) or 1-2 days (moderate surgery).

The passage of an appropriately sized endotracheal tube in a neonate suffering from the idiopathic respiratory distress syndrome (hyaline membrane disease) will A. impair oxygenation by making grunting impossible B. impair oxygenation by increasing airway resistance C. improve oxygenation by reducing dead space D. improve oxygenation by eliminating laryngeal obstruction E. have no effect on oxygenation

ANSWER A Respiratory Distress Syndrome also called Idiopathic Respiratory Distress Syndrome, or Hyaline Membrane Disease. Result of lack of surfactant at birth Risk factors * Prematurity * Diabetic mothers Lack of surfactant implies that alveoli are more prone to collapse as surfactant decreases work of breathing and helps promote alveolar stability. Hence neonates with HMD have requirements for high PEEP when ventilated, as well as higher pressures due to decreases ventilatory compliance. Neonates will classically grunt at the end of expiration in order to apply some self-PEEP and hence keep alveoli open.

The passage of an appropriately sized endotracheal tube in a neonate suffering from the idiopathic respiratory distress syndrome (hyaline membrane disease) will A. impair oxygenation by making grunting impossible B. impair oxygenation by increasing airway resistance C. improve oxygenation by reducing dead space D. improve oxygenation by eliminating laryngeal obstruction E. have no effect on oxygenation

ANSWER A Respiratory Distress Syndrome also called Idiopathic Respiratory Distress Syndrome, or Hyaline Membrane Disease. Result of lack of surfactant at birth Risk factors * Prematurity * Diabetic mothers Lack of surfactant implies that alveoli are more prone to collapse as surfactant decreases work of breathing and helps promote alveolar stability. Hence neonates with HMD have requirements for high PEEP when ventilated, as well as higher pressures due to decreases ventilatory compliance. Neonates will classically grunt at the end of expiration in order to apply some self-PEEP and hence keep alveoli open. Hence the correct answer is A. However it seems illogical to me that one would place an ETT without ventilation

The initial treatment of oculocardiac reflex is: A. Deepen anaesthesia B. Retrobulbar block C. IV lignocaine D. Carotid sinus pressure

ANSWER A Retrobulbar can cause OCR IV lignocaine makes no difference Carotid sinus massage will worsen symptoms Oculocardiac reflex * Mediated by the trigeminal and vagal nerves * Occurs is up to 30-90% of opthalmic surgery (?ref) * 1:2200 experience transient cardiac arrest in strabismus surgery * Exacerbated by: o young age o hypercapnoea o propofol TIVA o hypoxia o light anaesthesia o beta blockers o calcium channel blockers o potent narcotics o type of stimulus-medial rectus most sensitive Management 1. Stop stimulation 2. Gentle manipulation-reflex fatigues with repetition 3. Local infiltration around muscle 4. IV/IM atropine or glycopyrrolate 5. If refractory, ventilate to normocapnoea and deepen anaesthesia. 6. May cause other arrhythmias

Systemic vascular resistance index (SVRI) is calculated from A. systemic vascular resistance multiplied by body surface area B. systemic vascular resistance divided by body surface area C. mean aortic and central venous pressure difference divided by cardiac output D. cardiac index divided by the mean aortic and central venous pressure difference E. none of the above

ANSWER A SVRI = SVR x BSA

According to PS9 for sedation with Propofol for colonscopy, the staff required is/are: a. Medical practitioner other than proceduralist b. Nurse other than proceduralist c. Anaesthetist in addition to the proceduralist d. Assistant e. Proceduralist alone

ANSWER A Scenario 0: Two personnel - sedation by proceduralist *Medical or dental practitioner proceduralist with airway and resuscitation skills, and training in nitrous oxide or low dose oral sedation techniques *Assistant with training in monitoring sedation *Conscious sedation using nitrous oxide alone and/or low dose oral sedation alone in ASA P 1-2 patients *Heavy oral sedation and intramuscular or intravenous sedative/anaesthetic/analgesic agents must not be used Scenario 1: Three personnel - sedation by proceduralist *Medical or dental practitioner proceduralist with airway and resuscitation skills, and training in sedation *Assistant with training in monitoring sedation *Assistant to assist both *Conscious sedation in ASA P 1-2 patients *Propofol, thiopentone and other intravenous anaesthetic agents must not be used Scenario 2: Three personnel - sedation by medical or dental practitioner * Proceduralist * Medical or dental practitioner with airway and resuscitation skills, and training in sedation * Assistant to assist both * Conscious sedation in ASA P 1-2 patients * Propofol, thiopentone and other intravenous anaesthetic agents may only be used by a medical or dental practitioner trained in their use Scenario 3: Four personnel - sedation by medical or dental practitioner *Proceduralist * Medical or dental practitioner with airway and resuscitation skills, and training in sedation * Assistant to assist each* * Conscious sedation in ASA P 1-3 patients # * Propofol, thiopentone and other intravenous anaesthetic agents may only be used by a medical or dental practitioner trained in their use Scenario 4: Three personnel - sedation by anaesthetist * Proceduralist * Anaesthetist * Assistant to assist both * Conscious, deep sedation or general anaesthesia in all patients * All approved anaesthetic drugs may be used Scenario 5: Four personnel - sedation by anaesthetist * Proceduralist * Anaesthetist * Assistant to assist each* * Conscious sedation, deep sedation or general anaesthesia in all patients * All approved anaesthetic drugs may be used

ANZCA Version [Jul07] You are commencing general anaesthesia for a 2-year-old child to allow biopsy of an anterior mediastinal mass, A pre-operative CT scan demonstrated compression of the lower trachea and the carina by Ihe mass. During inhalational induction, the child desaturates to 70% due to airway compression by the mass. You should A. apply continuous positive airway pressure (CPAP) via facemask B. arrange urgent median sternotomy C. intubate the patient and allow spontaneous ventilation D. intubate the patient and provide positive pressure ventilation E. place the patient in the prone position

ANSWER A See review medind.nic.in/iad/t05/i4/iadt05i4p344.pdf Simple maneuvers first, Sats are 86% 1. Chin lift/thrust 2. CPAP 3. Lateral position, semi sitting, prone 4. Intubate/ventilate 5. Median Sternotomy Recommended technique for Anterior Mediastonal Mass for biopsy ??? TBA

A diagnostic test has a sensitivity of 90% and a specificity of 99% in detecting a certain disease. From this we can conclude that A. the false positive rate of this test is 1% B. the false negative rate of this test is 1% C. the positive predictive value of this test is 90% D. the negative predictive value of this test is 90% E. this test would be a useful screening test for this disease

ANSWER A Sensitivity = TP / (TP + FN) Specificity = TN / (TN + FP) Positive Predictive Value = TP / (TP + FP) Negative Predictive Value = TN / (TN + FN) False Positive Rate = 1 - spec False Negative Rate = 1 - sens False positive rate = FP / (TN + FP) = 1 - spec False negative rate = FN / (TP + FN) = 1 - sens

A diagnostic test has a sensitivity of 90% and a specificity of 99% in detecting a certain disease. From this we can conclude that A. the false positive rate of this test is 1% B. the false negative rate of this test is 1% C. the positive predictive value of this test is 90% D. the negative predictive value of this test is 90% E. this test would be a useful screening test for this disease

ANSWER A Sensitivity = TP / (TP + FN) Specificity = TN / (TN + FP) Positive Predictive Value = TP / (TP + FP) Negative Predictive Value = TN / (TN + FN) False Positive Rate = 1 - spec False Negative Rate = 1 - sens False positive rate = FP / (TN + FP) = 1 - spec False negative rate = FN / (TP + FN) = 1 - sens Therefore A is correct!

A diagnostic test has a sensitivity of 90% and a specificity of 99% in detecting a certain disease. From this we can conclude that A. the false positive rate of this test is 1% B. the false negative rate of this test is 1% C. the positive predictive value of this test is 90% D. the negative predictive value of this test is 90% E. this test would be a useful screening test for this disease

ANSWER A Sensitivity = TP / (TP + FN) measure the proportion that test positive with the disease. Specificity = TN / (TN + FP) measure of the proportion that test negative who are healthy Positive Predictive Value = TP / (TP + FP) Probability that the patient has disease in those with a positive test Negative Predictive Value = TN / (TN + FN) Probability of health in those that test negative to test Negative test in health False Positive Rate = 1 - spec False Negative Rate = 1 - sens False positive rate = FP / (TN + FP) = 1 - spec False negative rate = FN / (TP + FN) = 1 - sens

Systemic vascular resistance index (SVRI) is calculated from A. systemic vascular resistance multiplied by body surface area B. systemic vascular resistance divided by body surface area C. mean aortic and central venous pressure difference divided by cardiac output D. cardiac index divided by the mean aortic and central venous pressure difference E. none of the above

ANSWER A Since SVR = ((MAP-CVP)*80)/CO And CI = CO/BSA SVRI is calculated by substituting CI for CO in the equation for SVR, i.e. SVRI = ((MAP-CVP)*80)/CI Therefore SVRI = ((MAP-CVP)*80)/(CO/BSA) SVRI = ((MAP-CVP)*80)*(BSA/CO) SVRI = (((MAP-CVP)*80)/CO)*BSA SVRI = SVR * BSA

During scoliosis surgery with monitoring of somatosensory evoked potentials, which tract are they mainly monitoring? A: Dorsal column B: Spinothalamic tract C: Lateral Corticospinal tract D: Cerebrospinal tract E: Anterior horn cells

ANSWER A Somatosensory evoked potentials provide monitoring for the dorsal columns of the spinal cord. (wiki)

All are true about stellate ganglion block except: A. it is lateral to the carotid. B. it is lateral to longus coli muscle. C. it is medial to ant vertebral artery D. It is anterior to the longus colli muscle E. Located at T1

ANSWER A Stellate Ganglion -formed from the fusion of the inferior cervical ganglion with the first thoracic ganglion -located at the level of C7 Relations Anterior The structures anterior to the ganglion include the skin and subcutaneous tissue, the sternocleidomastoid and the carotid sheath. The dome of the lung lies anterior and inferior to the ganglion. Medial The prevertebral fascia, vertebral body of C7, oesophagus and thoracic duct lie medially. Posterior Structures posterior to the ganglion include the longus colli muscle, anterior scalene muscle, vertebral artery, brachial plexus sheath and neck of the first rib.

AM23 [Apr96] [Jul98] To relieve myotonia: A. Inject procaine into muscle B. Neostigmine C. Isoflurane D. Propofol

ANSWER A Surgical manipulation and diathermy may cause myotonic contractions Relieved by: * Phenytoin * Procainamide gb-quinine and mexiletine *direct injection with ESTER LA * High concentration of volatile anaesthetic (at the expense of myocrdial depression)

Atrial septal defect, where is the murmur heard the loudest? A. PV B. MV C. ASD D. AV E. TV

ANSWER A Talley: Ostium secundum (90%) does not involve the valves therefore not causing a murmur itself. Fixed splitting S2. High flow to RH causes low pitched diastolic tricuspid flow murmur and more often a pulmonary systolic ejection murmur louder on inspiration.

The third modality to be blocked with spinal anaesthesia is: A. Touch B. Pain C. Autonomic efferents D. Proprioception E. Motor

ANSWER A Temperature Pain Touch Proprioception Skeletal muscle tone

A fourteen-year-old girl is scheduled to have a termination of pregnancy. With regard to consent for this procedure, which of the following statements most truly reflects the law in Australasia? A. A fourteen-year-old girl is able to give consent independently of her parents/guardians if she is considered, by her treating doctors, to be of sufficient maturity to understand the issues. B. A fourteen-year-old girl is able to give consent independently of her parents/guardians only if a court deems her sufficiently mature. C. Minors are not able to give consent, independently of parents/guardians, until sixteen years of age. D. Minors are not able to give consent, independently of parents/guardians, until eighteen years of age. E. Only life-saving treatment may be administered to a fourteen-year-old without parental/ guardian consent.

ANSWER A The Gillick-competence test 2.7 A young person under 18 years of age is legally competent to consent to medical treatment if he or she is capable of understanding the nature and consequences of that treatment. The High Court of Australia settled this common law test for determining a young person's competence in a case commonly known as Marion's case. The majority of the Court held that: A minor is capable of giving informed consent when he or she achieves a sufficient understanding and intelligence to enable him or her to understand fully what is proposed.10 The majority in Marion's case followed a decision of the English House of Lords in Gillick v West Norfolk and Wisbech Area Health Authority.11 In Gillick, it was held that a young girl was competent to consent to contraceptive advice and treatment if she had a sufficient understanding and intelligence to enable her to understand fully what was proposed.12 2.8 The test focuses on an assessment of the individual young person's level of maturity and understanding in relation to the treatment proposed. This means that, before the age of 18 (when a young person legally becomes an adult), there is no fixed age at which it is possible to say, in advance of such an assessment, that a young person is automatically capable of consenting to medical treatment generally, or to specific types of medical treatment. Thus, while a young person of, say, 10 years of age may have the necessary intelligence and understanding of what is proposed to consent to the administration of a course of antibiotics to treat a sore throat, he or she may not, at the same time, possess sufficient understanding to consent to the removal of his or her tonsils in the case of acute tonsillitis. In short, before the age of majority, competence to consent operates on a continuum which ranges from the point at which a young person is clearly able to consent to the medical treatment proposed to that at which he or she is clearly unable to do so. 2.9 The test may prove difficult for medical practitioners, with whom the assessment of Gillick-competence initially rests, to apply in practice. The imprecision of the test places a significant onus on that assessment.13 Medical practitioners sued for providing medical treatment to a young person without consent bear the onus of proving that they believed that the young person was of sufficient maturity and understanding to consent to the treatment in question.

A 40-year-old man with Marfan's syndrome is to undergo thoraco-abdominal aortic reconstruction for chronic aortic dissection. An intrathecal catheter is inserted. The purpose for this is to A. allow drainage of cerebrospinal fluid (CSF) B. allow intrathecal administration of metabolic substrates C. allow intrathecal administration of neuroprotective drugs D. cool the spinal cord E. facilitate spinal cord function monitoring

ANSWER A The blood supply to the spinal cord and the kidney is reduced by 84-90% after cross-clamping the descending thoracic aorta at the level of the left subclavian artery. Elective thoracic aortic repair has a 0.4% incidence of permanent cord injury, but emergency repair of an aortic rupture has a 24-40% incidence. The incidence of perioperative spinal cord injury is increased by: * cross-clamp times of greater than 30 mins, as the warm ischaemia time for the cord is 20-30 mins; * high level of thoracic cross-clamping; * emergency surgery (eg acute dissection or rupture when there are no collaterals); * greater longitudinal extent of the aneurysm; * patient age greater than 70 years; and, * no prophylactic measures undertaken Intraoperative methods of spinal cord protection include -swift surgery, with identification and preservation of vital cord supply blood vessels -cross-clamp times of less than 30 minutes -Lumbar CSF drainage to increase the spinal cord perfusion pressure -mild hypothermia (34-35 °C), which decreases the cerebral metabolic rate for oxygen by 20% -Preliminary reports suggest intrathecal papaverine may provide some cord protection, as a result of its arteriolar vasodilating, calcium channel blocking and oxygen free radical scavenging properties -Spinal cord somatosensory evoked potential (SSEP) monitoring has been recommended to detect spinal cord ischaemia , Shunts to improve perfusion distal to the cross-clamp have not influenced the incidence of paraplegia, due to the high resistance to retrograde flow in the ASA above the artery of Adamkiewicz inflow.

AZ22 Lignocaine spray to the cords reduces laryngospasm by: A. Blocking parasympathetic afferent receptors B. Paralysing the smooth muscle C. Blocks the sympathetic response D. ?

ANSWER A The glottic closure reflex is another protective airway reflex.5 It can be elicited by selective stimulation of the superior laryngeal nerve (CNX branch), but has been reported to be triggered by stimulation of any cranial nerve in experimental animals. The efferent arc encompasses the superior as well as the recurrent laryngeal nerve. Exaggeration of the glottic closure reflex, called laryngospasm, may be noted after prolonged or intense stimulation of the glottis and is initiated solely by stimulation of the superior laryngeal nerve.

The recurrent laryngeal nerve supplies: A sensation below the cords B the intrinsic muscles of the pharynx C the cricothyroid muscle D the inferior constrictor muscle E the epiglottis

ANSWER A The recurrent laryngeal nerve is a branch of the vagus nerve (Xth CN) that supplies motor function and sensation to the larynx (voice box) LEFT laryngeal nerve -is longer, -branches from the vagus nerve to loop under the arch of the aorta, posterior to the ligamentum arteriosum before ascending. RIGHT branch loops around the right subclavian artery -giving off several cardiac filaments to the deep part of the cardiac plexus BOTH -As it ascends in the neck it gives off branches, more numerous on the left than on the right side, to the mucous membrane and muscular coat of the oesophagus; branches to the mucous membrane and muscular fibers of the trachea; and some pharyngeal filaments to the superior pharyngeal constrictor muscle. The nerve splits into anterior and posterior rami before supplying muscles in the voice box — it supplies all laryngeal muscles except for the cricothyroid, which is innervated by the external branch of the superior laryngeal nerve. The recurrent laryngeal nerve enters the pharynx, along with the inferior laryngeal artery and inferior laryngeal vein, below the inferior constrictor muscle to innervate the Intrinsic Muscles of the larynx responsible for controlling the movements of the vocal folds.

Patient with Hemophilia A with known high titres of inhibitors to factor 8. What would you give to prevent bleeding in the patient for ot a. FVIIa b. High dose FVIII concentrate c. FFP d. Cryo e. Platelets

ANSWER A The whole reason why novoSeven exists in the first place. A particular therapeutic conundrum is the development of "inhibitor" antibodies against factor VIII due to frequent infusions. These develop as the body recognises the "normal form" factor VIII as foreign, as the body does not have its own "copy". The problem is that in these patients, factor VIII infusions are ineffective. Recently[update] activated factor VII (NovoSeven) has become available as a treatment for haemorrhage in patients with haemophilia and factor inhibitors.

AZ73 ANZCA version [2004-Apr] Q127, [2005-Apr] Q52, [2005-Sep] Q7 [2009-Aug]q76 A fourteen-year-old girl is scheduled to have a termination of pregnancy. With regard to consent for this procedure, which of the following statements most truly reflects the law in Australasia? A. A fourteen-year-old girl is able to give consent independently of her parents/guardians if she is considered, by her treating doctors, to be of sufficient maturity to understand the issues. B. A fourteen-year-old girl is able to give consent independently of her parents/guardians only if a court deems her sufficiently mature. C. Minors are not able to give consent, independently of parents/guardians, until sixteen years of age. D. Minors are not able to give consent, independently of parents/guardians, until eighteen years of age. E. Only life-saving treatment may be administered to a fourteen-year-old without parental/ guardian consent.

ANSWER A This is a test of Gillick competence. The relevant common law discussion is outlined here[1] (http://www.lawlink.nsw.gov.au/lrc.nsf/pages/ip24chp02) at the NSW Law reform commission site. (All following quotes from this site.) Note that the initial Gillick decision was made in regard to contraception: "In Gillick, it was held that a young girl was competent to consent to contraceptive advice and treatment if she had a sufficient understanding and intelligence to enable her to understand fully what was proposed" Privacy: If you are 12 years or over you can see your doctor/health carer without your parents being present and your doctor is not allowed to give out information about you without your permission. Your health carer must keep what you say to them confidential unless: • You told them you were going to seriously harm or kill yourself • You were thinking about injuring/harming someone else • Someone else is physically harming you or sexually abusing you. Contraception and abortion: • If you are 16 years or more you can give your own consent to the doctor for contraception. • If you are 14 years, and the doctor believes you can give informed consent, then they are able to prescribe contraception or perform an abortion without the permission of the parent or guardian. • If you are under 14 years, the health carer must establish that you are able to give informed consent and some health workers may want to ask your parent/s' permission. If this is a problem for you contact a youth health service or try another health worker. If you are capable of making an informed decision, then your doctor must listen to you if you have not given your consent. Medicare: • At fifteen you can obtain your own Medicare card, and your parents will not be told. The Gillick-competence test A young person under 18 years of age is legally competent to consent to medical treatment if he or she is capable of understanding the nature and consequences of that treatment. The High Court of Australia settled this common law test for determining a young person's competence in a case commonly known as Marion's case. The majority of the Court held that: A minor is capable of giving informed consent when he or she achieves a sufficient understanding and intelligence to enable him or her to understand fully what is proposed. The majority in Marion's case followed a decision of the English House of Lords in Gillick v West Norfolk and Wisbech Area Health Authority.11 In Gillick, it was held that a young girl was competent to consent to contraceptive advice and treatment if she had a sufficient understanding and intelligence to enable her to understand fully what was proposed. The test focuses on an assessment of the individual young person's level of maturity and understanding in relation to the treatment proposed. This means that, before the age of 18 (when a young person legally becomes an adult), there is no fixed age at which it is possible to say, in advance of such an assessment, that a young person is automatically capable of consenting to medical treatment generally, or to specific types of medical treatment. Thus, while a young person of, say, 10 years of age may have the necessary intelligence and understanding of what is proposed to consent to the administration of a course of antibiotics to treat a sore throat, he or she may not, at the same time, possess sufficient understanding to consent to the removal of his or her tonsils in the case of acute tonsillitis. In short, before the age of majority, competence to consent operates on a continuum which ranges from the point at which a young person is clearly able to consent to the medical treatment proposed to that at which he or she is clearly unable to do so. The test may prove difficult for medical practitioners, with whom the assessment of Gillick-competence initially rests, to apply in practice. The imprecision of the test places a significant onus on that assessment. Medical practitioners sued for providing medical treatment to a young person without consent bear the onus of proving that they believed that the young person was of sufficient maturity and understanding to consent to the treatment in question.

Best immediate treatment of severe post-partum haemorrhage after delivery of a complete placenta: A. IV Ergometrine B. Blood transfusion C. Evacuation of uterus without blood transfusion D. Bimanual compression of the uterus E. Aortic compression

ANSWER A This is the simplest and least invasive step of the five options outlined. Williams states that the "fundus should always be palpated following placental delivery to make certain that the uterus is well contracted. If it is not firm, vigorous fundal massage is indicated. Most often, 20 U of oxytocin in 1000 mL of lactated Ringer or normal saline proves effective when administered intravenously at approximately 10 mL/min (200 mU of oxytocin per minute) simultaneously with effective uterine massage." (Williams Obstetrics Ch 35). Given that it is standard practice in Australia to administer oxytocin after the 2nd stage of labour (i.e. after the patient is fully dilated and the baby is delivered), the next step is uterine massage and ergometrine iv.

On bypass, for mitral stenosis repair immediately after cardioplegia the following happens: MAP to 25 SvO2 80% CVP1 Next step in management: A) metaraminol B) give volume C) increase pump flows D) adrenaline infusion E)

ANSWER A This scenario is common, it is all about finding the right balance in circulating volume, pump flow and vascular tone. At the time of cardioplegia, you should be running full flow and therefore volume status should be optimised.

The first drug to be used in the management of an intra-operative thyrotoxic crisis is A. esmolol B. hydrocortisone C. magnesium D. potassium iodide E. propyl thiouracil

ANSWER A Thyroid storm Thyroid storm is defined as fulminant thyrotoxicosis, occuring perioperatively. -TH secreted from thyroid gland -TSH secreted from anterior pituitary gland -TRH from hypothalamus Immediate management issues are: * Airway * Breathing * Circulation Issues of note 1. Tachycardia & arrhythmias e.g. AF as a result of acidosis, electrolyte imbalance. Rate related ischaemia can be an issue also. Hence early use of beta blockers is recommended (Allman et al, 2005, p256). Note that this has two effects * Beta blocker related heart rate changes * Beta blocker treatment of hypertension 2. Hypertension complications * Short term -> APO * Long term -> End organ damage 3. Hyperpyrexia * Prolonged hyperpyrexia can lead to seizures. * Associated with muscle damage, myoglobin release and myoglobinuria. Associated ARF. 4. Muscle function * Manifest in several ways * Loss of breathing hence requires ventilation * - Loss of muscle tone - often requires airway support i.e. ETT Timing of drugs * Minutes - beta blockers. Esmolol is drug of choice due to most rapid action * Hours - Antithyroid drugs. PTU prevents peripheral conversion. * Hours - Steroids * Days - Lugols iodine takes several days to work.

The most common coagulopathy in trauma patients is A. clotting factor inhibition and depletion B. disseminated intravascular coagulation C. citrate toxicity D. hypofibrinogenaemia E. thrombocytopaenia

ANSWER A Trauma Associated Coagulopathy The causes of coagulopathy are multifactorial and interrelated, including consumption and dilution of coagulation factors and platelets, dysfunction of platelets and the coagulation system, increased fibrinolysis, compromise of the coagulation system by the infusion of colloid, hypocalcaemia, and disseminated intravascular coagulation like syndrome. * Massive crystalloid infusion induces early coagulopathy * Using more than 6 units packed cells → PT and APTT > 1.5 times normal * > 10-30 units packed cells → platelets < 50 × 109 * Massive transfusion protocols aim to prevent rather than 'catch up' with coagulopathy * Factors in trauma that lead to coagulopathy o Clotting element loss, consumption and dilution + Small normal body stores ∴ easily lost # 10g fibrinogen # 15mls platelets o Hypothermia + ↓ platelet activation and adhesion + Slows metabolic rates of coagulation enzymes o Acidosis + Affects enzyme complex activity on lipid surfaces o DIC + DIC in trauma doubles the mortality rate + Embolisation of brain substance, marrow fat, amniotic fluid and other strong thromboplastins → DIC * Thrombocytopenia may be the first cause of bleeding to treat in trauma, whereas correction of coagulation factor deficiency due to dilution may be undertaken first in elective surgery

What antibiotics are required for bacterial endocarditis prophylaxis in a woman with MV prolapse for cholecystectomy. A. None B. gentamicin C. ampicillin and gentamicin D. ampicillin E. cephazolin

ANSWER A Trickly wording! "Widely accepted indications for antibiotic prophylaxis are contaminated and clean-contaminated surgery and operations involving the insertion of an artificial device or prosthetic material." Australian Prescriber. So this patient would likely get cephazolin, but not for BE prophylaxis. The new guidelines state that the following patients should be given BE prophylaxis; Prosthetic valve or valve repair material pHx of BE Unrepaired cyanotic congenital heart disease Partially repaired cynanotic CHD with defect over the repair Repaired CHD within the 1st 6 months of repair Heart transplant with valvulopathy So the devil is in the detail!

First-line therapy for trigeminal neuralgia is A. carbamazepine B. amitriptyline C. mexilitene D. gabapentin E. baclofen

ANSWER A Trigeminal neuralgia is a neuropathic disorder characterized by episodes of intense pain in the face, originating from the trigeminal nerve. -1 or all 3 branches of trigeminal branches may be affect -Distribution * V2 + V3 commonest → V3 → V2 → V1 rarest * Usually unilateral R>L (about 3% bilateral) -10% bilateral -intense facial pain last from a few seconds to hours Well established link to multiple sclerosis (MS) * MS occurs in 2-4% of patients with Trigeminal neuralgia * Trigeminal neuralgia diagnosed in 1-5% of MS patients Treatment 1. Medical -first line : carbemazepine (NNT=2.6) -second line : baclofen, lamotrigine, phenytoin, gabapentin (NNT=3.6), valproate -low doses of amytriptyline (NNT=2.1) is thought to be effective, [pprer side effect profile and dangerous if overdosed -opiates 2. Surgical -evidence is poor -microvascular decompression resluts in longest pain releif -percutaneous radiofrequency thermorhizotomy → 20% relief at 10 years -glycerol injections

Tryptase 1. Is a neutral protease and 99% is located primarily within the mast cell 2. Half-life of 2 to 3 hours 3. A level > 10ng/ml indicates anaphylaxis 4. Must collect blood in lithium heparin tube Select A if options 1, 2 & 3 are correct Select B if options 1 & 3 are correct Select C if options 2 & 4 are correct Select D if option 4 only is correct Select E if all options are correct

ANSWER A Tryptase -is a tetrameric neutral serine protease -99% in mast cells -Pro-B-typtase is secreted constitutively and used to measure mast cell number -mature-B-typtase reflects mast cell activation -1% in basophils Tryptase half life 120min -rise can be quantified as soon as 30 minutes after onset of symptoms -peak at 120 min, then gradual decay -back to baseline by 2 days Tryptase raised in both but anaphylaxis >> anaphylatoid No raise in tryptase does not exclude an anaphylaxis. Collection immediately during event, 6 hours post. Post mortem blood is also suitable if collected within 4 days of death (provided death followed within 30 minutes of the event) Reference <13 u/ml

47. Features of ventricular tachycardia DO NOT include A. Absence of p waves B. Monophasic waves C. Prominent R wave in V1 D. A-V dissociation E. Fusion beats

ANSWER A Unfortunately, the electrocardiographic differentiation of VT from SVT with aberrancy is not always possible. There are several electrocardiographic features that increase the likelihood of VT: •Absence of typical RBBB or LBBB morphology •Extreme axis deviation ("northwest axis") - QRS is positive in aVR and negative in I + aVF. •Very broad complexes (>160ms) •AV dissociation (P and QRS complexes at different rates) •Capture beats — occur when the sinoatrial node transiently 'captures' the ventricles, in the midst of AV dissociation, to produce a QRS complex of normal duration. •Fusion beats — occur when a sinus and ventricular beat coincides to produce a hybrid complex. •Positive or negative concordance throughout the chest leads, i.e. leads V1-6 show entirely positive (R) or entirely negative (QS) complexes, with no RS complexes seen. •Brugada's sign - The distance from the onset of the QRS complex to the nadir of the S-wave is > 100ms •Josephson's sign - Notching near the nadir of the S-wave •RSR' complexes with a taller left rabbit ear. This is the most specific finding in favour of VT. This is in contrast to RBBB, where the right rabbit ear is taller.

The pain of uterine contraction can be blocked by which of the following? A. Lumbar epidural B. Sacral epidural C. Uterosacral block D. Pudendal block

ANSWER A Uterine Contraction is carried with sympathetic fibres to T11/12 Blocked by : spinal, llumbar/low thoracic epidural, paravertebral, lumbar plexus block, paracerival block Second stage : pain carried by pudendal nerve Blocked by pudendal nerve block, spinal, sacral epidural

The observed fall in cardiac output induced by carbon dioxide pneumoperitoneum during laparoscopic cholecystectomy is primarily a result of A. a fall in venous return to the heart B. reflex bradycardia C. an increase in systemic vascular resistance D. head-up tilt of patient E. increased pulmonary vascular resistance

ANSWER A VR will fall if pressure > 15mmHg SVR will increase over all ranges

A terrorist attack has taken place involving the nerve gas "VX". Some victims have arrived in the emergency department. The most appropriate management of this situation is to: A. Strip them off and hose them down B. Strip them off, scrub them with a brush, and hose them down C. Leave their clothes on and hose them down D. Leave their clothes on, scrub them with a brush, and hose them down E. Take them to the resuscitation area and put in an IV

ANSWER A VX = S-[2-(diisopropylamino)ethyl]-O-ethyl methylphosphonothioate] known nerve-agent exposure or who exhibits definite signs or symptoms of nerve-agent exposure should immediately have the nerve agent antidote drugs atropine, pralidoxime (2-PAM), and diazepam Immediate first aid is to remove pt from the area of danger to a well-ventilated area before removal of clothing and decontamination of the skin

Desflurane vaporiser, heated because of A. High SVP B. High boiling point C. Low SVP D. High MAC E. Low MAC

ANSWER A Vaporizer heated because of the high SVP. Desflurane's BP is so much LOWER than the other volatiles, at 23C, that ambient temperature may cause the liquid to boil, causing significant variation in the vapour concentration. Heated to well above boiling point to ensure reliable concentration of desflurane gas mixture.

Desflurane vaporiser, heated because of A. High SVP B. High boiling point C. Low SVP D. High MAC E. Low MAC

ANSWER A Vaporizer heated because of the high SVP. Desflurane's BP is so much LOWER than the other volatiles, at 23C, that ambient temperature may cause the liquid to boil, causing significant variation in the vapour concentration. Heated to well above boiling point to ensure reliable concentration of desflurane gas mixture.

Appropriate infection control measures when anaesthetising a patient with suspected variant-CJD, the airway equipment should be a.thrown away b.plastic sheath, reuse c.sterilization with ethylene oxide d. sterilization with heat at 134 degrees for 3 minutes e. Autoclave

ANSWER A Variant CJD - spongiform encephalopathy -psychiatric disturbance -failure of muscle conduction -memory impairment Prions are not autoclavable

Patient (?48h post) SAH following bloods: Na 155 Plasma osmolality 350 urine osmolality 250 Management includes: A)DDAVP (?nasally) B)Water restriction

ANSWER A basically hypernatremia with high plasma osmolality N=280 and low urine osmolality (ranges from 50-1400, but average is 500-800), suggests DI ie inadequate ADH secretion - therefore treatment DDVAP

PL11a Which of the following has the greatest cardiotoxic effect? A. Amethocaine B. Procaine C. Dibucaine D. Etidocaine E. Prilocaine

ANSWER A bupivacaine >tetracaine = etidocaine > lignocaine = chloroprocaine > mepivacaine = prilocaine >>>procaine

Type IV allergic reactions (contact dermatitis) to latex are due to the: (a) chemical additives (b) corn starch (c) latex polysacharides (d) latex protein

ANSWER A http://www.allergy.org.au/aer/infobulletins/pdf/Latex_Allergy.pdf

Photograph of an Arndt endobronchial blocker. Orifice labelled 'X'. What goes in 'X'? A. Bronchoscope B. the circuit C. the guidewire D. Nylon... E. Blocker

ANSWER A http://www.anaesthesiauk.com/images/clip_image001.jpg

What is NOT true for PDPH following puncture A: Prophylactic bed rest B: Decreased incidence when catheter is inserted intrathecally C: Decreased incidence in elderly D: Caffeine is not effective E: Epidural blood patch is 80% effective

ANSWER A in Section 9.6.5 Acute Pain Management: Scientific Evidence (2005) and Update (2007) Incidence of headache following dural puncture 0.4-24% * postural in nature * commoner in patients under 50 yrs * commoner in parturients * significanntly less common in males than non-pregnant females (level 1 evidence - 2007 update) * 90% resolve spontaneously within 10 days Incidence may be reduced by using: (level I) * 26 gauge or smaller needle (NNT=13) * use of needle with a non-cutting bevel (NNT=27) No evidence that bed rest is beneficial in preventing PDPH (Level I) * PDPH may causes difficulty mobilising, and headache may then subside with bed rest * Non-opioid and opioid analgesics may provide temoporary relief * Preventive role of fluid therapy unclear (Level I) NO evidence to support the use of: * Sumatriptan (Level II) * ACTH * Epidurally administered saline, dextran, fibrin glue or neuraxial opioids IV and oral caffeine (both level II) are: * effective in treating PDPH * do not reduce blood patch rate Epidural blood patches: * are common practice but further high quality trials are required to determine efficacy (level I) * significant symptomatic relief obtained in 75-95% of patients given a 15 mL blood patch (level IV, three studies) * conflicting evidence regarding use of prophylactic blood patches - one trial showed decreased incidence of PDPH (level III) Autologous epidural blood patches may be contra-indicated in: * leukaemia * coagulopathy * infection, including HIV

33. (NEW) Best way prevent first phase of heat loss after induction a. Prewarming the patient with forced air warming b. Warm blankets c. Warm fluids d. Warm theatre e. Humidified gases

ANSWER A pre-warming eliminates the gradient between core and peripheries thus the redistribution phase does not occur.

Complications of diabetes mellitus in the pregnant patient include each of the following EXCEPT A. increased risk of oligohydramnios B. greater risk of foetal death in the third trimester C. retinopathy and retinal detachment D. potentiation of hypotension when regional anaesthesia is administered to assist delivery E. reduced foetal oxygen delivery

ANSWER A Complications of Diabetes mellitus for baby: 1. increased fetal malformations, persists despite better treatment of T1DM. two- to sixfold increase in major malformations. Mainly neurological (neural tube), cardiac and sacral. 2. supply demand relationship affects: maternal vasculopathy, preeclampsia, hyperglycaemia and DKA causing poor placental perfusion AND the fetus has increased metabolic needs due to hyperinsulinism and macrosomia. 3. stillbirth previously occurred in 10-30% of T1DM, usually after 36weeks, thought to be due to chronic intrauterine hypoxia. 4. fetal umbilical cord blood samples from pregnant women with type 1 diabetics have demonstrated "relative fetal erythremia and lactic acidemia." 5. macrosomic children: birth trauma, obesity when older 6. neonatal hypoglycaemia 7. respiratory distress syndrome 8. polycythaemia and jaundice 9. Ca and Mg metabolic changes

The medical therapy for unstable angina which is most effective in reducing progression to myocardial infarction is A. aspirin B. beta-blockers C. calcium channel blockers D. glyceral trinitrate E. thrombolytic therapy

ANSWER A See ACC/AHA guidelines .The recommendations of medical therapy ('to prevent death and MI') include: Class 1. (level of evidence) * Aspirin (A) * Clopidogrel if ASA not tolerated (A) * ASA and clopidogrel for 9 months post UA/STEMI (B) * Beta-blockers if not contraindicated (B) * Lipid lowering agents (A or B depending on patient) * ACEI for pts with CCF, LV dysfunction (EF <0.4), HTN or DM (A).

Line isolation monitoring protects against microshock A. in no circumstances B. only when all equipment in the region is monitored C. as long as the hazard current is set to 30 milliamps D. as long as the hazard current is set to 10 milliamps E. only if grounded equipment is used.

ANSWER A * Isolating Transformers and Line Isolation Monitors. * These are the more expensive alternative to RCD's and are widely used in operating theatres because they do not disconnect the power when a fault is detected, yet provide safety should such a fault exist. * The first component is a large transformer (the Isolating Transformer) mounted in the wall cavity which converts the earth-referenced mains supply to a "floating" supply. The floating supply provides 240V between two active wires, but because the supply is not earth-referenced, the presence of an earth circuit through the patient or anyone else is perfectly safe and no current will flow. All the circuit to earth does is to reference the floating supply to earth; no current actually flows through the earth connection. * The Line Isolation Monitor continually checks that the floating supply is not earth-referenced, and indicates on a dial how much current could flow to earth if there was an earth connection. If the potential earth current would be more than 5mA an alarm will sound, alerting the anaesthetist to the presence of a loss of the "floating" nature of the supply. It does this by intermittently connecting one of the two active wires to ground through a very large resistance. If the other wire is connected to ground a circuit will be formed and current will flow, and this indicates how much current would flow through the circuit if either of the two active wires are connected to ground. * As with an RCD the device will not alarm under 5mA, so microshock may still occur unnoticed, however macroshock is very unlikely; only current flowing through the patient from between the active wires will no be detected. (Electricity and electrical hazards

AM19 [Mar91] [Aug91] [Mar92] [Aug93] [Aug95] Duchenne muscular dystrophy: A. May cause myoglobinuria (? or myoglobinaemia) post-suxamethonium B. Associated with sudden death postoperatively C. Commonly associated with cardiomyopathy D. Can be present in a seemingly muscular person E. Inheritance is autosomal dominant

ANSWER A B C D DMD: * Incidence: 3 per 100 000 births * Inheritance: X-linked recessive, therefore DMD primarily affects males * Skeletal muscles: Most individuals are wheelchair bound by 8-11 years of age. * Respiratory: Respiratory muscle weakness leading to pulmonary insufficiency is a leading cause of morbidity and mortality in DMD. Respiratory failure inevitably occurs in the second decade of life * Cardiac: DMD-associated cardiomyopathy begins at the same time as clinically evident skeletal muscle dysfunction. By 15 years of age, over 50% of patients have some degree of dilated cardiomyopathy with a reduced ejection fraction (<45%). Even patients with apparently normal preoperative cardiac workup may succumb to adverse perioperative cardiac events.

A woman already in the lateral tilt position for LSCS with epidural in situ, develops profound hypotension. The immediate treatment is: A. IV crystalloids B. IV Ephedrine C. IM Methoxamine D. IV colloid E. Trendelenburg

ANSWER A and B

A Swan-Ganz catheter can be unreliable for measuring pulmonary artery systolic and diastolic pressures because the length and compliance of the tubing affects the measuring system by A. reducing its resonant frequency B. reducing its frequency response C. reducing its damping coefficient D. inducing a zero error E. inducing baseline drift

ANSWER A and B A. TRUE : Resonant frequency is decreased by long and compliant tubing B: TRUE : frequency response is dependent on the resonant frequency, C. FALSE : long compliant tubes have increased damping coefficients D. FALSE : zero error is caused by incorrectly zeroing the system (not dependent on length and compliance) E. FALSE : length and compliance is not related to baseline drift. TWO forms of accuracy *Static eg. MAP *Dynamic eg BP To accurately measure systolic and diastolic pressures with a catheter transducer system, the system needs to have an appropriate dynamic response. It is dependent on *resonant frequency : inherent frequency of any oscillating system at which the system would tend to oscillate is disturbed *damping coefficient : calculated from the ratio of the amplitude of 2 successive peaks -optimal damping is D=0.64 allowing for minimal amplitude distortion, optimal frequency respsonse and minimal phase distortion -critical dampening D=1, however if Fn is high (ei short), the dynamical accuracy will still be good

AM09c ANZCA version [2004-Aug] Q94, [Jul06] Q93, [Apr07] In the Neuroleptic malignant syndrome A. there is a familial incidence B. non-depolarizing muscle relaxants decrease the muscle rigidity C. creatinine kinase levels are elevated following an episode D. there is an association with malignant hyperpyrexia E. hyperthermia does not always occur

ANSWER A and B A. there is a familial incidence - true: o "Familial clusters of NMS suggest a genetic predisposition to the disorder. Genetic studies have shown that the presence of a specific allele of the dopamine D2 receptor gene is over represented in NMS patients. This allele is associated with reduced density and function of dopamine receptors as well as decreased dopaminergic activity and metabolism B. non-depolarizing muscle relaxants decrease the muscle rigidity - true: o "Many of the previous reports documented that succinylcholine may be safely used during ECT in a patient of NMS3, but some do not agree8, as even small dose of succinylcholine can produce side effects. Reports quote that mivacurium (0.15-0.25mg kg-1) and rapacuronium (0.6- 0.8 mg kg-1) may be safe alternatives to succinylcholine C. creatinine kinase levels are elevated following an episode - false: NMS and malignant hyperthermia have clinical features in common, including hyperthermia, rigidity, an elevated creatine kinase concentration and a mortality rate for both NMS and malignant hyperthermia of 10-30%" Note that it is CREATINE KINASE not CREATININE KINASE D. there is an association with malignant hyperpyrexia - false the association between NMS and other potentially fatal syndromes, such as malignant hyperthermia, is unclear E. hyperthermia does not always occur - false: Three major symptoms indicate a high probability of the presence of NMS: hyperthermia, rigidity and an elevated creatine phosphokinase concentration, reflecting rhabdomyolysis. In the absence of these criteria, the diagnosis of NMS should be questioned, since other symptoms of the disorder may be seen in patients taking neuroleptics without having NMS. Elevated temperature (38.5°C) in the absence of other systemic illness is observed in most patients

Regarding post craniotomy pain: A. Local infiltration proven to reduce long-term pain B. Local more painful than discrete nerve blocks C. Local infiltration more efficacious than discrete nerve blocks D. Local infiltration more efficacious than opioid analgesia E. Local infiltration more efficacious with clonidine included

ANSWER A and B Cranial Surgery Key Messages 1. 70% of patients report moderate to severe pain D1 postop -this is contrast to belief that intracranial surgery is not painful 2. Pain is more severe after intratentorial rather than supertentorial approach 3. Non‐craniotomy neurosurgery, for example trans‐sphenoidal surgery, seems to be associated with very limited pain and minimal morphine requirements 4. Paracetamol as sole analgesic agent is ineffective 5. NSAIDs are more effective than paracetamol in reducing PCA opiods requirements but with minimal benefits in regard to pain scores 6. Morphine is more effective than codeine and tramadol for pain relief after craniotomy (MPCA better than PRN Morphine IM) (MPCA better than TPCA) 7. Intraoperative remifentanyl may result in increased pain and increased analgesia post op 8. Local anaesthetic infiltration of the scalp provides early analgesia after craniotomy and reduces incidence of subsequent chronic pain 9. Local infiltration was no different to nerve blocks, IV morphine or IV fentanyl 10. Nerve blocks are less painful to administer than local infiltration. 11. Clonidine does not improve analgesia

AM26 [Jul97] [Apr99] [Aug99] Dystrophia myotonica (type A) A. Thiopentone may cause prolonged respiratory depression B. Non-depolarising relaxants are useful for blocking contractures C. Volatiles & neuromuscular blockers do not block myotonia

ANSWER A and C

Reverse splitting of the second heart sound occurs with: A. LBBB B. Pulmonary hypertension C. Acute pulmonary embolus D. ASD E. Severe MR

ANSWER A and D Physiology of the Second Heart Sound -Audible component of the closure of aortic and pulmonary valves -A2 followed by P2, separated only by 0.02 - 0.08 seconds. -Although contraction of both ventricles occurs at the same time, difference in opening and closure are due to differences in impedance of each vascular bed -as PVR is low, PV opens sooner and closes later -as SVR is high, AV opens later and closes sooner -therefore A2-P2 -this interval is widened by deep inspiration, known as Physiological Splitting. Abnormal Splitting of Second Heart Sound (1) persistently single; -S2 remains single throughout respiratory cycle -most commonly due to inaudibility -in setting of CHD : absence of PV : Fontans (pulmonary atresia, severe PV stensosis, dysplastic PV, transposition of great arteries) (2) persistent (audible expiratory) splitting, with normal respiratory variation; -most common cause if RBBB (3) persistent splitting without respiratory variation (fixed splitting); -A2 and P2 is constantly wide and remains unchanged during respiratory cycle -Hallmark of ASD (4) reversed (paradoxical) splitting. -Reversed sequence of semilunar valve closure P2-A2 -Common causes : LBBB and Right ventricular PPM (activation of the RV before LV) -Other causes : RV ectopic beats, prolonged LV mechanical systole, HOCM, Severe hypertension, PDA

PZ65b ANZCA version [2003-Apr] Q113, [Jul06] Q52, [Apr07] Q113 Aug10 To normalise platelet function prior to surgery, chronic diclofenac therapy should be ceased for at least A. 12 hours B. 1-2 days C. 4 days D. 7 days E. 10 days

ANSWER A or B depending on the formulation * A. would seem appropriate for rapidly absorbed formulations * B. would seem appropriate for slow-release formulations Elimination t1/2 of diclofenac ("Voltaren") is 1.1-1.8 hours therefore stop for 1-2 days preop is adequate. * However, diclofenac is often taken as a 'SR' preparation which delays absorption significantly. It's effects last much longer than the plasma t1/2 in these people. --lovethedrugs 23:33, 17 Jun 2008 (EDT) * The reason diclofenac has such a short half-life is its high hepatic metabolism. Other NSAIDs (except for aspirin) have low rates of hepatic metabolism. * The other important point to make here is that the inhibition of cyclo-oxygenase (COX) by diclofenac is reversible. In contrast the inhibition of COX by aspirin is irreversible so even with its short half-life, you have to wait longer for the effect of aspirin to be reversed (ie needs production of sufficient amounts of new platelets with newly synthetised COX.

What are the ECG changes of hyperkalaemia? A. Prolonged PR interval B. Prolonged QRS C. Peaked T wave D. Shortened PR interval E. Deep S wave

ANSWER A, B, C

It is MOST important to re-program a patient's implanted cardiac pacemaker prior to: A. electroconvulsive therapy (ECT) B. laser therapy to a laryngeal papilloma C. lithotripsy of a renal calculus D. magnetic resonance imaging of the thorax E. percutaneous transhepatic cholangiography

ANSWER A, C, D Although C is best, as you PPM is absolute contraindication for MRI. Situations Probably Requiring Pacemaker Reprogramming *Any rate-responsive device *Special pacing indication (hypertrophic obstructive cardiomyopathy, dilated cardiomyopathy, pediatric patients) *Pacemaker-dependent patients *Major procedure in the chest or abdomen *Rate enhancements present that should be disabled *Lithotripsy *Transurethral resection *Hysteroscopy *Electroconvulsive therapy *Succinylcholine use *Magnetic resonance imaging (generally contraindicated by device manufacturers) In some situations and for certain patients, a pacemaker should be reprogrammed either to avoid potential patient injury or to prevent a pacemaker rhythm that could be confused with pacemaker malfunction.

The features of Pierre Robin sequence include cleft palate, micrognathia and: A. Glossoptosis B. Craniosynostosis C. Macroglossia D. Microstomia

ANSWER A: Glossoptosis Posterior displacement or retraction of the tongue with airway obstruction. Also cleft palate. PRS Chromosomal abn Can be part of underlying disorder or syndrome Stickler Syndrome Velocardiofacial syndrome Fetal alcohol syndrome Treacher Collins Syndrome

Regarding anticholinesterases: A. pyridostigmine has slow onset of effect B. physostigmine does not rely on renal metabolism/excretion C. neostigmine cannot reverse centrally acting cholinergics D. edrophonium is less reliable in reversal?

ANSWER ABC Is this an "EXCEPT" question? A. True. Has a slower onset of action than neostigmine and its duration of action is longer. B. True. "Physostigmine is hydrolysed at its ester linkage and renal excretion is of minor importance" (Stoelting) C. True. It contains "a quaternary ammonium group... poorly lipid soluble and thus do not easily penetrate lipid cell membranes barriers such as... blood brain barrier" (Stoelting) D. False. Edrophonium produces "reversible inhibition of acetylcholinesterase through its electrostatic attachment to the anionic site on the enzyme...further stabilized by hydrogen bonding on the esteratic site on the enzyme... duration of action considered to be brief, reflecting its reversible binding" (Stoelting)

45 y.o for elective laproscoptic choecystectomy. No recent hx cholelithiasis. Drinks 40g of alcohol per week and smokes 40/day. URTI 1/52 ago. Finishing course of augmentin. No respiratory symptoms now. LFTS done - ALP 300, GGT 300, ALT normal, AST normal, albumin normal. What is next step? A. LFTS changes likely due to recent illness. Proceed with case B. Defer case until liver USS done C. Cancel case as patient has early cirrhosis D. Perform hepatitis screen pre-operation E. likely secondary to drugs, so just continue

ANSWER B

64. A patient has a suspected anaphylactic reaqction under GA. What is the best time to perform the tryptase test? A><1 hour B>1 to 3 hours C>3 to 6 hours D>numerous other options

ANSWER B

70 y/o postop in recovery following hip surgery. Develops severe chest pain, ST elevation. Immediate mx: A Beta blocker B Aspirin C GTN infusion D Heparin infusion

ANSWER B

: Normal systolic BP at birth? A. Something less than 70 mmHg B. 70 mmHg C. 85 mmHg D. Something more than 85 mmHg E. 115 mmHg

ANSWER B

A 25-year-old man, involved in a motor vehicle crash, presented with a GCS (Glasgow Coma Score) of 5. He was intubated and ventilated, and CT scan of his head was consistent with diffuse axonal injury. An ICP (intra-cranial pressure) monitor has been placed. The patient's ICP has ranged between 15 and 25 mmHg over the last 2 hours, despite intravenous mannitol and moderate hypocapnia. He has stable haemodynamics with a mean arterial pressure of 95-100 mmHg. He now requires general anaesthesia to manage his orthopaedic injuries. The most appropriate agents for maintenance of anaesthesia would be A. isoflurane / remifentanil B. propofol and fentanyl C. propofol and nitrous oxide D. sevoflurane and nitrous oxide E. sevoflurane and remifentanil

ANSWER B

A 25kg child is having liver resection and is rapidly given 1 unit of blood. Her CVP is 8. The cause most likely to be responsible for any haemodynamic instability she experiences is: A. coagulopathy B. hyperkalaemia C. ABO incompatability D. Hypocalcaemia E. Hypothermia

ANSWER B

A 32 year-old man has a four day history of progressive weakness in the extremities. He has been well apart from an upper respiratory tract infection 10 days ago. His temperature is 37.8OC and respiratory rate 42 breaths.min-1 and shallow. He has symmetric weakness on both sides of his face and the proximal and distal muscles of the extremities. Sensation is intact. His most likely diagnosis is A. acute disseminated encephalomyelitis B. Guillain-Barre syndrome C. myasthenia gravis D. poliomyelitis E. polymyositis

ANSWER B

A 35kg 5 year old girl is having elective surgery for suturing of a superficial leg laceration. After induction with N2O/Sevoflurane/O2 and in absence of any visible veins you have placed an appropriately sized LMA. Following this her SpO2 immediately drop to 90%. What is your initial management? A. Increase inhaled sevoflurane concentration with LMA in situ B. Increase inhaled sevoflurane concentration after removing LMA C. Intralingual suxamethonium D. Intramuscular suxamethonium E. Intramuscular atropine

ANSWER B

A 60 year old man 24 hours post CABG is confused, oliguric, with BP 80/40, pulse 120. The most appropriate and useful investigation is A. electrocardiogram B. echocardiogram C. chest x-ray D. arterial blood gas E. coronary angiogram

ANSWER B

A machine with a soda lime absorber was left on overnight with oxygen running at 6 litres per minute. In the morning a desflurane vaporiser is connected. What toxic substance may be produced? A. Substance A B. Carbon monoxide C. Carbon dioxide D. Calcium hydroxide E. Substance B

ANSWER B

A new test has been developed to diagnose a disease. To determine the SPECIFICITY of this new test it should be administered to A. a mixed series of patients i.e. some known to be suffering from the disease and some known to NOT be suffering from it B. a series of patients known to NOT be suffering from the disease C. a series of patients known to NOT be suffering from the disease and an estimate of the prevalence of the disease in the population obtained D. a series of patients known to be suffering from the disease E. a series of patients known to be suffering from the disease and an estimate of the prevalence of the disease in the population obtained

ANSWER B

A young woman has an MVA when driving home from a nightclub. She was known to have moderate MDMA (ecstasy) consumption. Which of the following complications is least likely to be due to MDMA? A. Hyponatraemia B. Hypotension C. Pneumothorax D. Sweating E. Tachycardia

ANSWER B

AZ35 [Mar92] [Apr97] [Jul97] [Aug99] For a difficult intubation using a gum elastic bougie - to achieve the least resistance to entry of ETT: A. Leave laryngoscope as is & do a 1/4 turn clockwise with the ETT B. Leave laryngoscope as is & do a 1/4 turn anticlockwise C. Take laryngoscope out & do a 1/4 turn clockwise D. Take laryngoscope out & do a 1/4 turn anticlockwise

ANSWER B

AZ52 [Aug96] [Apr98] [Jul98] [Jul00] (type A) In MRI scan, which will NOT cause patient burns: A. Thermistor temperature probe B. Non-metallic face makeup C. Coiled monitor leads on patient chest in magnetic core D. Non-ferromagnetic equipment touching patient E. Pulsating magnetic field

ANSWER B

Acromegaly due to excess of growth hormone. Why hard to do direct laryngoscopy? A. Distorted facial anatomy B. Macroglossia C. Glottic stenosis D. Prognathe mandible E. Arthritis of the neck

ANSWER B

An INCORRECT statement regarding the autonomic nervous system is that A. autonomic dysfunction is a predictor for worse long term survival after myocardial infarction B. heart rate responses are primarily mediated through the sympathetic nervous system C. inhalation anaesthetics all impair autonomic reflex responses D. autonomic dysfunction is a predictor for haemodynamic instability following anaesthetic induction E. low heart rate variability is associated with worse cardiac outcomes following non-cardiac surgery

ANSWER B

An INCORRECT statement regarding the autonomic nervous system is that A. autonomic dysfunction is a predictor for worse long term survival after myocardial infarction B. heart rate responses are primarily mediated through the sympathetic nervous system C. inhalation anaesthetics all impair autonomic reflex responses D. autonomic dysfunction is a predictor for haemodynamic instability following anaesthetic induction E. low heart rate variability is associated with worse cardiac outcomes following non-cardiac surgery

ANSWER B

An indication for NON-operative management of blunt liver trauma in adults is A. absence of peritoneal signs B. a haemodynamically stable patient C. a haemopertitoneum of LESS than 500 ml D. a LOW grade injury on CT scan E. severe chronic obstructive airway disease

ANSWER B

An obese 40-year-old woman is having laparoscopic surgery for endometriosis. She is intubated and ventilated with a mixture of oxygen and air. The ventilator is set to provide a tidal volume of 600ml at 12 breath.min-1 with 5 cm H2O positive end-expiratory pressure (PEEP). The peak inspiratory airway pressure is 35 cm H2O. She was stable on induction and during preparation for surgery but 10 minutes after introduction of the pneumoperitoneum and being placed in the Trendelenburg position, her arterial oxygen saturation (SaO2) falls to 80%. The SaO2 remains unchanged despite ventilation with 100% oxygen. Her blood pressure is 130/80, pulse 100 min-1 and end-tidal carbon dioxide 44 mmHg. The most likely cause of her desaturation is: A. Aspiration B. endobronchial intubation C. gas embolism D. hypoventilation E. pneumothorax

ANSWER B

Anti-retroviral agents for HIV infection include A. nucleoside r-transcriptase inhibitors (NRTIs) which are associated with diarrhoea and intravascular volume depletion B. protease inhibitors which have important effects on the cytochrome P450 system C. non-nucleoside r-transcriptase inhibitors (N-NRTIs) which may cause elevations in liver function tests D. protease inhibitors which can lead to problematic hypoglycaemia

ANSWER B

Antidepressant drugs are useful in the management of all of the following conditions EXCEPT: A. chronic back pain B. chronic headaches C. chronic neuropathic pain after breast surgery D. chronic neuropathic pain after herpes zoster infection E. trigeminal neuralgia

ANSWER B

Antidepressants are not effective/recommended for a. Chronic headache b. Chronic back pain c. Chronic pain post mastectomy d. Chronic pain post acute herpes zoster e. Trigeminal neuralgia

ANSWER B

Arrest in a 10 year old. Has ventricular tachycardia after a near drowning accident. Patient is intubated and is being ventilated with 100% O2 and has IV access. A single DC monophasic shock of 60J has been given. The next step is to give A adrenaline 10mcg/kg and DC shock 60J B adrenaline 10mcg/kg and DC shock 120J C amiodarone 5mg/kg D DC shock 60J E DC shock 120J

ANSWER B

Arterial blood gases in a patient: pH 7.12, PO2 100, PCO2 65, HCO3 20.3, BE -10. These results are consistent with? A. Chronic renal failure B. Malignant hyperthermia C. Diabetic ketoacidosis D. End-stage respiratory failure E. Ethylene glycol toxicity

ANSWER B

Autonomic dysreflexia. Which ONE is true? A: 50% of patients with a level below T6 B: Unlikely if below T10 C: Can be prevented?? D: Can be precipitated by light touch E. ?

ANSWER B

Best agent to decrease both gastric volume and gastric acidity? A. Omeprazole B. Ranitidine C. Metoclopramide D. Cisapride E. Na citrate

ANSWER B

Black bank August 2010 SZ11 Lowering intra-ocular pressure by applying pressure to the globe (e.g. Honan balloon) is typically contraindicated in a patient having A. a revision corneal graft B. a revision trabeculectomy C. an extra-capsular lens extraction D. a redo vitrectomy E. repeat retinal cryotherapy

ANSWER B

CHADS2 score. Which is not a feature? A. Age B. Gender C. Diabetes mellitus D. Stroke E. CCF

ANSWER B

Detrimental post-operative effects of hypothermia following general anaesthesia in the elderly include each of the following EXCEPT A. delayed emergence B. hypercoagulability C. increased body metabolism D. reduced elimination of anaesthetic agents E. shivering

ANSWER B

FALSE statements regarding natural rubber latex allergy include A. sensitivity of skin prick testing is greater than that of specific IgE antibody detection (RAST) B. signs of Type I hypersensitivity are usually immediate C. latex antibodies fall in time in a latex free environment D. the risk factors of frequent exposure and atopy are additive E. theatre latex aerogens are lowest in the morning

ANSWER B

Following a cadaveric renal transplant under general anaesthesia, your patient's plasma K+ increases to 6.0 mmol.l-1 in recovery after being 5.0 mmol.l-1 pre and intra-operatively. This patient requires A. an intravenous infusion of CaCl2 (10 mls over 20 minutes) B. arterial blood gases to ascertain the acid/base status C. potassium exchange resins rectally D. sodium bicarbonate infusion (50- 100 mEq over 5- 10 minutes) E. urgent haemodialysis

ANSWER B

Following anaphylaxis, serum tryptase has a half-life of about: (a) 0.5 hours (b) 2 hours (c) 8 hours (d) 30 hours

ANSWER B

Following suspected occurence of anaphylaxis, serum tryptase levels should be taken as soon as possible and at: (a) 1 day and 7 days after the episode (b) 1 to 2 hours and 5 to 6 hours after the episode (c) 10 minutes and 30 minutes after the episode (d) 4 to 6 hours and 24 to 36 hours after the episode

ANSWER B

Haemodynamic responses to pneumoperitoneum include all the following EXCEPT A. decreased cardiac output B. decreased venous vascular resistance C. increase in pulmonary artery occlusion pressure D. increased systemic vascular resistance E. increased systemic blood pressure

ANSWER B

Haemoglobin in infants: A. 30% fetal Hb at birth, adult levels by 3 months B. 70% fetal Hb at birth, negligible amounts by 6 months C. Hb 90 at 6 months normal doesn't need Ix D. Hb 200 at birth unless delayed cord clamping E. ?

ANSWER B

Histamine release during anaphylaxis does NOT cause A. tachycardia B. decreased myocardial contractility C. coronary vasodilatation D. a shift in the cardiac pacemaker site E. an increased PR interval

ANSWER B

Histamine release in anaphylaxis does NOT cause: A. Tachycardia B. Myocardial depression C. Coronary artery vasodilatation D. Prolonged PR interval E. Decreased impulse conduction

ANSWER B

Hyperosmolar non-ketotic coma A. is relatively common in young diabetics B. occurs in patients who have a history of diabetes in the majority cases C. is more acute in presentation than diabetic ketoacidosis D. is NOT associated with intracellular and intravascular dehydration E. is associated with a decrease in serum sodium concentration

ANSWER B

In a child with NO history of epilepsy the commonest cause of post-operative convulsions is A. hyperthermia B. hyponatraemia C. hypoxia D. local anaesthetic toxicity E. pethidine toxicity

ANSWER B

Interscalene block after injection of 2ml bupivacaine- patient seizure. Most likely injected to A. Dural cuff B. Vertebral arteries C. Internal carotid arteries D. Jugular veins E. Subarachnoid

ANSWER B

Inverted P waves in lead II may be caused by? A. transposed lower limb leads B. junctional rhythm C. hypothermia D. left axis deviation E. inferior myocardial infarction

ANSWER B

Iron deficiency A. decreased serum ferritin, increased serum iron B. decreased serum ferritin, absence of bone marrow iron C. decreased serum ferritin, normal serum iron D. increased serum ferritin, decreased serum iron E. increased serum ferritin, decreased total iron binding capacity

ANSWER B

Labour epidurals increase maternal and foetal temperature. This results in neonatal A. Increased sepsis B. Increased investigations for sepsis C. increased non shivering thermogenesis D. Increased need for resuscitation E. Cerebral palsy

ANSWER B

Lateral approach to popliteal block. A. Passes through semimembranosis B. May be performed supine or prone C. Adequate for ankle surgery D. Less effective in comparison to posterior approach E. Eversion is an endpoint for nerve stimulation

ANSWER B

MR44 ANZCA version [2003-Aug] Q148, [2004-Aug] Q89, [2005-Apr] Q85, [Mar06] Q82, [Jul06] Q70, [Mar10] Correct statements regarding expiratory-inspiratory flow-volume loops include all of the following EXCEPT A. in obstructive disease the expiratory curve has a scooped out or concave appearance B. in restrictive disease expiratory flows are usually decreased in relation to lung volume C. in restrictive disease the expiratory curve has a convex appearance D. the expiratory curve is largely effort independent E. the inspiratory curve is effort dependent

ANSWER B

Main heat loss in anaesthetic for neonate A. vasodilatation B. radiation C. convection D. conduction E. evaporative

ANSWER B

Management of an overdose of paracetamol could include all of the following EXCEPT A. activated charcoal B. alkalinisation of the urine C. gastric lavage D. hepatic transplantation E. N-acetylcysteine

ANSWER B

Maternal cardiac arrest. In making the diagnosis of amniotic fluid embolism, large amount of PMNs surrounding foetal squamous cells are A. Pathonomonic B. Supportive C. Only found at postmortem D. Irrelevant E. Incidental

ANSWER B

Maternal collapse post-delivery. What is NOT consistent with Amniotic fluid embolism? a. Seizure b. Petechial rash c. Hypotension d. Coagulopathy e. Cardiac arrest

ANSWER B

Maximum dose of local infiltration of 0.5% bupivacaine in an x kg child? A. 1mg/kg B. 2.5mg/kg C. 5mg/kg D. 7 mg/kg with adrenaline

ANSWER B

Most common cause of paediatric post anaesthesia cardiac arrest A. Drug error B. Respiratory cause C. Multifactorial D. Cardiac problem (?)

ANSWER B

Most effective treatment for post-sevoflurane agitation following grommets in a 4yo child? A. 1 mg/kg propofol B. 1 mcg/kg fentanyl C. 1 mcg/kg clonidine D. ?dose midazolam E. Sucrose

ANSWER B

Myasthenia gravis, Eaton Lambert Syndrome What happens with exercise? A. MG better, EL worse B. EL better, MG worse C. Both EL and MG get worse D. Both EL and MG get better E. MG worse and EL no difference

ANSWER B

Neonate to drug addicts found by grandmother in the house, brought into ed, mildly jaundice, slight tachycapnic. ABG PH 7.54, PaCO2 46, pO2 74, HCO 13 A. Septicaemic B. Pyloric stenosis C. Opiod overdose D. Meningitis E. Hepatitis

ANSWER B

PZ107 [Mar06] Q149, [Jul06] Q96 In patients with renal impairment, doses of all of the following may require adjustment EXCEPT A. carbamazepine B. gabapentin C. hydromorphone D. morphine E. oxycodone

ANSWER B

Plasma glucose level compared to blood glucose level a)32% higher b)14% higher c)same d)14%lower e)32% lower

ANSWER B

Post-thoracotomy the drain is leaking fluid with protein, fat, lymphocytes etc. What could be the cause? A. Bleeding B. Thoracic duct injury C. sympathectomy D. Pleural fluid E. ?? "something like CHF or pulmonary oedema"

ANSWER B

Preoperative assessment shows a Mallampati (ML) score of III and thyromental distance (TMD) of < 6cm. A grade 3 to 4 on Cormark and Lehane is predicted. Compared to the ML score, the TMD is A less sensitive, less specific B less sensitive, more specific C more sensitive, less specific D more sensitive, more specific E equal sensitivity an specificity

ANSWER B

Preoperative assessment shows a malampati (ML) score of III and thyromental distance (TMD) of < 6cm. A grade 3 to 4 on Cormark and Lehanes is predicted. Compared to the ML score, the TMD is: A less sensitive, less specific B less sensitive, more specific C more sensitive, less specific D more sensitive, more specific E equal sensitivity an specificity

ANSWER B

Preoperative assessment shows a malampati (ML) score of III and thyromental distance (TMD) of < 6cm. A grade 3 to 4 on Cormark and Lehanes is predicted. Compared to the ML score, the TMD is: A less sensitive, less specific B less sensitive, more specific C more sensitive, less specific D more sensitive, more specific E equal sensitivity an specificity

ANSWER B

Preoperative autologous blood donation results in less: A. Cost B. Incompatible transfusion C. Less blood wastage D. Less unnecessary transfusion

ANSWER B

Publication Bias is that A. researchers with a strong track record are more likely to get research published B. studies with positive results are more likely to be published C. studies with negative results are more likely to be published D. studies on important clinical questions are more likely to be published E. the prestige of the journal will affect readers' perception of the quality of the study

ANSWER B

Pulmonary hypertension, which will affect PVR the most a. Isoflurane b. Sevoflurane c. Desflurane d. Propofol e. Remifentanil

ANSWER B

Pulsus paradoxus is: A. Reduced BP on inspiration unlike normal (ie normally increased on insp) B. Reduced BP on inspiration exaggerated from normal C. Reduced BP on expiration unlike normal D. Reduced BP on expiration exaggerated from normal E. ?

ANSWER B

Regarding remifintanil, which is incorrect? a. high potency b. metabolised by pseudocholinesterase c. muscle rigidity in high doses d. weakly active metabolite e. short context sensitive half time

ANSWER B

SF64 ANZCA version [2005-Apr] Q102 In women with congenital heart disease the clinical scenario associated with the greatest maternal mortality is A. aortic stenosis with a valve area less than 0.7 cm2 B. Eisenmenger's syndrome C. left ventricular ejection fraction less than 35% D. Marfan's syndrome with dilated aortic root E. significant aortic coarctation

ANSWER B

Sensation from the lobule of the external ear is mediated mostly by: A. The auriculotemporal nerve B. The great auricular nerve C. The lesser occipital nerve D. The greater occipital nerve E. None of the above

ANSWER B

Septic elderly man. (Given lots of results but essentially mixed venous oxygen sat 65%, lactate 4, MAP low) Management? A. Transfuse B. Fluid bolus C. Noradrenaline D. ? E. ?

ANSWER B

The equivalent shock energy with a biphasic defibrillator, to that of a 360 joule monophasic defibrillator shock is A. 90 joules B. 180 joules C. 360 joules D. 450 joules E. 720 joules

ANSWER B

The most commonly reported cause of awareness during general anaesthesia for a non-obstetric procedure is A. equipment failure B. human error C. lack of premedication D. recreational drug use E. the use of total intravenous anaesthesia

ANSWER B

The nerve roots which usually conduct the pain of uterine contraction & cervical dilatation in the first stage of labour are: A. T8-T12 B. T10-L1 C. T11-L2 D. T12-L4 E. T10-L4

ANSWER B

The plasma half-life of low molecular weight heparin is A. increased in conditions with raised plasma proteins B. 2 to 4 times that of unfractionated heparin C. much less predictable than unfractionated heparin D. dependent upon a saturatable mechanism for clearance E. longer than unfractionated heparin because of a higher affinity for plasma protein

ANSWER B

The presence of small bubbles of air in an invasive arterial pressure monitoring system will always decrease the A. damping coefficient of the system B. resonant frequency of the system C. recorded systolic pressure D. recorded mean pressure E. extinction coefficient of the system

ANSWER B

The safe maximal pressure for endotracheal cuff at the lateral side of the trachea A. 0-10 cm water B. 10-20 cm water C. 20-30 cm water D. 30-40 cm water E. 40-50 cm water

ANSWER B

The spread of an epidural block depends on: A. Speed of injection of the local anaesthetic B. Mass of local anaesthetic C. The direction in which the solution is injected D. The age of the patient E. All of the above

ANSWER B

The tapered connector between the ETT and machine is A. 12-20mm B. 15-22mm C. 20-30mm D. 22-30mm E. 25-35mm

ANSWER B

To improve oxygenation in a patient intubated and ventilated for a laparotomy you adjust the ventilator settings to apply 10 cm H2O of PEEP (positive end-expiratory pressure). The patient's blood pressure falls from 130/80 to 90/50 mmHg. The addition of PEEP may result in a fall in blood pressure because PEEP causes A. decreased myocardial contractility B. decreased venous return C. increased left ventricular afterload D. increased left ventricular compliance E. increased right ventricular afterload

ANSWER B

Visual loss with pupillary reflexes retained. Likely cause ? A. Retinal detachment B. Occipital mass (! I don't think this option was on the 2011 paper??) C. Frontal mass D. Chiasmal mass E. Optic neuritis

ANSWER B

What do C6/7 motor function do A. flex/extension of fingers B. flex /extend wrist C. shoulder ext rotation / abduction D. elbow pronation/supination E. flexion at elbow

ANSWER B

What is the mechanism of central sensitisation? A. Increased intracellular magnesium B. Antagonism of the NMDA receptor C. Glycine is the major neurotransmitter involved D. Recurrent a-delta fibre activation E. Alteration in gene expression

ANSWER B

What is the ratio of MAC awake:MAC of sevoflurance A. 0.2 B. 0.3 C. 0.4 D. 0.5 E. 0.6

ANSWER B

What percentage of patients with SAH are troponin positive? A: <5% B: 15-30% C: 40-60% D: 70-90% E: 100%

ANSWER B

Which of the following can be used to describe the spread of non-parametric data? A. standard deviation B. interquartile range C. confidence interval D. standard error E. variance coefficient

ANSWER B

Which of the following is incorrect, with obstructive sleep apnoea syndrome A. More common in males than females B. upper airway edema rarely occurs C. Can develop pulmonary hypertension, polycythaemia, and cor pulmonale D. Commonly die of a cardiac event E. Can get anatomical pharyngeal narrowing just posterior to the tongue

ANSWER B

Which of the following is not associated with systolic hypertension A. Cushing's syndrome B. hyperparathyroidism C. primary aldosteronism D. congenital adrenal hyperplasia

ANSWER B

Which of the following is the best guide to fluid replacement in a burns patient? A. Haemoglobin and haematocrit B. Urine output C. Urine osmolality D. Plasma volume E. Specific gravity

ANSWER B

Which statement regarding the trigeminal nerve is incorrect? A. is attached to the pons B. is joined to the trigeminal (semilunar) ganglion by a motor root and by a sensory root C. is sensory to the face D. is motor to the muscles of mastication

ANSWER B

Why is codeine not used in paediatrics? A. Poor taste B. High inter-individual pharmacokinetic variability C. Not licensed for children < 10 years old D. Not as effective as adult when given in ?weight adjusted dose? E. ?

ANSWER B

You are called for a labour epidural. The woman is extremely distressed and in the middle of your consent process states "Just take my pain away" . You: A. Place epidural then when calmed return to advise her of risks and complications B. Explain she has to hear all the potential complications and refuse to place epidural without consent C. Take consent from partner D. Perform spinal to relieve pain, then consent her for epidural E. Go away and return when she is more cooperative

ANSWER B

Young female having cholecystectomy. Venous air embolus: A. Mechanical ventilation and PEEP is part of treatment strategy B. Most likely to occur at initial gas insufflation, but can occur at any time C. Inert gas (argon, xenon) is safer D.

ANSWER B

[ANZCA version Apr08 q123] All the following are predictors of difficult intubation EXCEPT: A. Inter-incisor distance <3cm B. Prominent C1 spinous process C. Prominent maxillary canines D. Samsoon classification - Class IV E. TMD <6cm

ANSWER B

xray shown of patient post Left pneumonectomy with heart swung to left side. Management: A. Increase PEEP B. Roll onto right side C. Turn on suction to left pleural catheter D. Lung biopsy E.

ANSWER B

A patient with pulmonary hypertension secondary to lung disease presents for laparotomy. To avoid exacerbating pulmonary hypertension, the anaesthetist could employ A. Permissive hypercapnoea B. ketamine as the primary anaesthetic agent C. volatile anaesthetic agents D. N2O as an adjunct to volatile anaesthetic agents

ANSWER B * Hypercapnoea is bad for pulmonary hypertension * Ketamine is bad for PHT * N2O is bad for PHT * TIVA is acceptable * Volatiles are also acceptable

Changes in the fetal circulation following birth include A. an immediate increase in pulmonary vascular resistance B. functional closure of the foramen ovale at one hour C. physical closure of the foramen ovale at several weeks D. a decrease in systemic vascular resistance E. closure of the ductus arteriosus in the first few hours

ANSWER B A. An immediate decrease in pulmonary vascular resistance occurs with the first breaths. The entry of air (with oxygen) into the lungs with onset of breathing results in removal of hypoxic pulmonary vasoconstriction so PVR falls markedly. B. Foramen ovale functional closure is generally in place at one hour; it occurs as pulmonary venous return increases, causing a rise in left atrial pressure compared to right atrial pressure C. Total physical closure of the foramen ovale (when it occurs) takes up to a year D. An Increase in systemic vascular resistance occurs as the cord is clamped E. The actual closure of the ductus takes place in two stages. In healthy full term newborns, functional ductal closure occurs by 96 h. This functional closure is followed later by anatomical closure via endothelial and fibrous tissue proliferation

Regarding CSF: A. The pH of CSF is the same as arterial blood B. The pH is lower than arterial blood C. The HCO3 concentration of CSF is the same as arterial blood D. The pCO2 of CSF is the same as arterial blood E. In active hyperventilation, greater difference in pCO2 in CSF as compared to ABG

ANSWER B CSF composition -Reference values for CSF are as follows: * Protein 0.15-0.45 g/L Electrolytes: * Osmolality 280-300 mmol/L * Sodium 135-150 mmol/L * Potassium 2.6-3.0 mmol/L * Chloride 115-1 30 mmol/L * Carbon dioxide 20-25 mmol/L * Calcium 1.00-1 .40 * Magnesium 1.2-1.5 mmol/L * Lactate 1.1-2.4 mmol/L * pH 7.28-7.32 * PCO2 44-50 mm Hg * PO2 40-44 mm Hg Other constituents: * Creatinine 50-110 umol/L * Glucose 2.8-4.4 mmol/L * Iron 0.2-0.4 umol/L * Phosphorus 0.4-0.6 umol/L * Urea 3.0-6.5 mmol/L A comparison of the composition of CSF and plasma reveals that: 1. CSF proteins are -1 % that of plasma; 2. CSF calcium levels are -50% that of plasma; 3. CSF glucose levels are -60% that of plasma; 4. CSF chloride and magnesium levels are higher than plasma; 5. CO2 diffuses rapidly, and HCO3 slowly, from the plasma into the CSF.

AZ75 ANZCA version [2005-Apr] Q127, [2005-Sep] Q3 The usual cardiovascular response to ECT (electro-convulsive therapy) is A. not predictable as it is very variable B. transient bradycardia followed by tachycardia and hypertension C. transient bradycardia with hypotension D. transient tachycardia and hypertension E. transient tachycardia followed by bradycardia and hypotension

ANSWER B Initial parasympathetic induced bradycardia lasting 10-15 seconds followed immediately by a more prominent sympathetic response that results in tachycardia and hypertension lasting 5 minutes or longer.

Which LMA has highest seal pressure? A. Classic B. Disposable supreme C. Flexible D. Intubating E. Proseal

ANSWER B LMA Classic - Seal pressure up to 20cm H20 LMA Supreme™ - measured oropharyngeal leak pressures up to 37 cm H2O LMA Flexible™ - oropharyngeal LMA Fastrach - Seal pressures up to 20 cm H2O LMA ProSeal™ - leak pressures up to 32 cm H2O

One hundred vomiting patients receive ondansetron. If 25 patients, who would not have stopped vomiting had they received a placebo, stop vomiting, then the number needed to treat (NNT) for ondansetron to stop vomiting is A. 1.3 B. 4 C. 25 D. 100 E. can't be calculated without information on placebo success rate

ANSWER B [ARR = (100 - 75)/100 = 0.25; NTT = 1/ARR = 4]

When compared with intra-muscular or subcutaneous opioid regimens, patient controlled analgesia (PCA} with opioids A. is equally preferred by patients B. provides better analgesia C. results in less opioid-related adverse effects D. results in lower opioid consumption E. results in shorter hospital stay

ANSWER B * "IV opioid PCA provides better analgesia than conventional (IM or SC) opioid regimes." * "no differences in opioid consumption, duration of hospital stay or opioid-related adverse effects." * "Patient preference for IV PCA was significantly higher ... although there was no difference in patient satisfaction"

Plenum Vaporiser A. ? something with fresh gas flows B. Relies on a constant flow of pressurised gas C. ? can be used out of circle D. Not temperature compensated E. ? Volatile injected into fresh gas flow

ANSWER B * A - ? * B - TRUE - Upstream gas source required to push fresh gas through the vaporizer (opposite to Draw-Over vaporizer) * C - ? FALSE - Don't exactly understand the question/stem. You can use a plenum vaporizer with OR without a circle (e.g. T-piece in paeds) o Think they are referring to VOC or VIC configuration...kingfed * D - FALSE - ARE temperature compensated * E - FALSE - This is refering to the Tec-6 for Desflurane but that is NOT a plenum vaporizer

Why is codeine not used in paediatrics? A. Poor taste B. High inter-individual pharmacokinetic variability C. Not licensed for children < 10 years old D. Not as effective as adult when given in ?weight adjusted dose? E. ?

ANSWER B * A - As far as I am aware it doesn't have a particularly nasty taste, and is used in cough suppressants, so FALSE. * B - TRUE. Variations in CYP2D6 function affect how much codeine is converted to morphine, and therefore how effective it is, but also how "sensitive" patients are to codeine. * C - Painstop was used in children when I was an RMO a few years ago and contains codeine, and as far as I know it is still used, but I don't know the specific licensing info for codeine in paeds. It is not used in our hospital, but mostly because we use oxycodone instead.- You can still buy it OTC. We have some at home. (Because you can't buy oxycodone OTC!) * D - ? False. It's crap in children just like it's crap in adults isn't it!

Thiazide diuretics may cause A. hypernatraemia B. precipitation of acute gout C. hypoglycaemia D. hyperkalaemia E. hepatic failure

ANSWER B * A - False. "Side effects include hypokalaemia, hyponatraemia, hyperuricaemia, hypomagnesaemia, hypochloraemic alkalosis, hyperglycaemia, hypercholesterolaemia, exacerbation of renal and hepatic impairment, impotence, and rarely rashes and thrombocytopaenia." Yentis, 3rd. ed., p.507. * B - True. "Thiazides and uric acid are secreted by the same mechanism within the renal tubules. This competition leads to reduced uric acid excretion and a rise in plasma levels which may precipitate gout."Peck, Hill & Williams, 2nd ed., p.304. * C - False. See quote at A. * D - False. See quote at A. * E - False. "Borderline renal or hepatic function may deteriorate further during treatment with thiazide diuretics, presumably reflecting drug-induced decreases in blood flow to these organs." Stoelting, 4th ed., p. 487-488. AND "Impaired hepatic function. Thiazides should be used with caution in patients with impaired hepatic function or progressive liver disease, since minor alterations of fluid and electrolyte balance may precipitate hepatic coma." Mims Online.

The most correct statement regarding the Child-Pugh score for liver disease is that A. a high-risk score is not possible with normal aminotransferase levels B. a high-risk score is possible without encephalopathy C. a prothrombin time greater than 10 seconds above normal confers extra points to the raw score D. it has not been validated for non-shunt and non-transplant laparotomies E. it was originally developed for patients undergoing hepatic transplantation

ANSWER B * A False - AST/ALT not part of classification * B True - could score 13/15 without encephalopathy (>9 is Class C) * C False - more than 6 secs scores a 3 (see below) * D False - it was initially used in the context of non-shunt & non-transplant laparotomies * E False

Which of the following does NOT occur in response to trauma? A. Rise in urinary nitrogen B. Rise in urinary sodium C. Water retention D. Sodium retention E. Rise in serum sodium

ANSWER B * A. Rise in urinary nitrogen Effect - Catabolic * B. Rise in urinary sodium Effect - Increased vasopressin and water retention * C. Water retention Effect - Increased vasopressin * D. Sodium retention Effect - Increased renin, Angiotensin II, and Aldosterone * E. Rise in serum sodium Occurs with DI from head trauma

Anatomical features of the spinal cord do NOT include A. an anterior median fissure and a posterior median septum B. thirty-two pairs of spinal nerves C. a filum terminale ending at the coccyx D. four to six spinal arteries arising from the posterior inferior cerebellar arteries E. the anterior spinal artery arising from the vertebral arteries

ANSWER B * A. an anterior median fissure and a posterior median septum - true: "The spinal cord presents an anterior median fissure and a shallow posterior median sulcus from which a glial posterior median septum extends about halfway into the substance of the cord." * B. thirty-two pairs of spinal nerves - false: "There are 31 pairs of spinal nervesaeight cervical, 12 thoracic, five lumbar, five sacral and one coccygeal. Each is formed by the fusion of an anterior and posterior spinal root." * C. a filum terminale ending at the coccyx - true: "Below, the spinal cord tapers into the conus medullaris, from which a glistening thread, the filum terminale, continues down to become attached to the coccyx." * D. four to six spinal arteries arising from the posterior inferior cerebellar arteries - unsure so make your own decision: "The posterior spinal arteries comprise one or two vessels on either side derived from the posterior inferior cerebellar arteries. They supply the posterior grey and white columns on either side. These arteries are reinforced serially by spinal branches of the vertebral, deep cervical, intercostal, lumbar, ilio-lumbar and lateral sacral arteries; the lower branches being responsible for the blood supply of the cauda equina." * E. the anterior spinal artery arising from the vertebral arteries - true: "The anterior spinal artery is a midline vessel lying on the anterior median fissure and is formed at the foramen magnum by the union of a branch from each vertebral artery."

In patients with portal hypertension undergoing surgery, laboratory results associated with an increase in postoperative mortality include A. an elevated aPTT (activated partial thromboplastin time) B. a total bilirubin over 25 micromol.l-1 (normal range 3 - 17) C. a serum albumin less than 30 g.l-1 (normal range 35 - 50) D. an ALT (alanine transaminase) of 80 (normal range < 55) E. a serum albumin greater than 55 g.l-1 (normal range 35 - 50)

ANSWER B * A. an elevated aPTT (activated partial thromboplastin time) * B. a total bilirubin over 25 micromol.l-1 (normal range 3 - 17) - false: Child Pugh scoring only rates BR>34. <34 does not increase a Child Pugh score. (See Harrison's table 295-4) * C. a serum albumin less than 30 g.l-1 (normal range 35 - 50) - true: This patient will have a Child Pugh score of >7. * D. an ALT (alanine transaminase) of 80 (normal range < 55) * E. a serum albumin greater than 55 g.l-1 (normal range 35 - 50)

Post-dural puncture headaches (PDPH) in children A. are less effectively treated by a blood patch (compared to adults) B. are rare in children under 10 years of age C. have an incidence of approximately 5% to 12% following spinal anaesthesia D. have an incidence which is independent of needle gauge E. presents with signs which are not posturally related

ANSWER B * A. are less effectively treated by a blood patch (compared to adults) - probably false * B. are rare in children under 10 years of age - controversial in literature but possibly best answer of a bad lot o " Postspinal headache is one of the most common complications of lumbar puncture (LP). Limited evidence suggests that the incidence in children is lower than in adults [22-25]. However, in a prospective study of LP in 112 children and adolescents (age 2 to 16 years), postspinal headache occurred in 18 and 40 percent of children <10 years and ≥10 years, respectively [26]." (Uptodate) * C. have an incidence of approximately 5% to 12% following spinal anaesthesia - false o Note the numbers for B are post LP. * D. have an incidence which is independent of needle gauge - false o Bigger needle, more likely * E. presents with signs which are not posturally related - false

PZ86b ANZCA version [2004-Apr] Q129, [2004-Aug] Q13 The herbal medicine associated with enhanced bleeding is A. echinacae B. ginko C. golden seal D. kava-kava E. St. John's wort

ANSWER B * A. echinacae - false: "Echinacea appears to have a favourable adverse effects profile [9, 23]. Although echinacea preparations contains pyrrozolidine alkaloids [24], hepatotoxicity is unlikely because these alkaloids are structurally different from the pyrrozolidine alkaloids known to be hepatotoxic [12, 25]. Adverse effects include unpleasant taste, gastro-intestinal upset, headache and dizziness." * B. ginko - true: "Danshen, dong quai, jui, ginkgo biloba and garlic have anticoagulant effects and therefore increase the risk of bleeding during surgery." * C. golden seal - false: "Effects on Bleeding: None reported" (Uptodate: Golden seal (Hydrastis canadensis): Natural drug information) * D. kava-kava - maybe: "The herb may also affect platelets in an antithrombotic manner by inhibiting cyclo-oxygenase" (Curr Opin Anaesthesiol. 2007 Aug;20(4):294-9.) * E. St. John's wort - false: "In vitro studies have demonstrated that St John's Wort decreases the efficacy of warfarin by this mechanism [106]. A decreased anticoagulant effect of warfarin associated with concomitant use of St John's Wort has been recorded [103]. Therefore, INR should be closely monitored when starting or stopping St John's Wort" (Anaesthesia Volume 57, Issue 9, 2002. Pages: 889-899)

During one-lung ventilation, hypoxic pulmonary vasoconstriction in the non-ventilated lung A. is increased with high pulmonary artery pressures B. is decreased by applying CPAP (continuous positive airway pressure) to the non-ventilated lung C. reduces the shunt fraction by approximately 40% D. is reduced significantly by isoflurane E. is unaffected by hyperventilation

ANSWER B * A. is increased with high pulmonary artery pressures - false * B. is decreased by applying CPAP to the non-ventilated lung - maybe true; CPAP will decrease the amount of shunted blood definitely, but does its application actually recruit alveoli and improve V/Q ratio?? * C. reduces the shunt fraction by approximately 40% - maybe true: 50% is quoted figure * D. is reduced significantly by isoflurane - false * E. is unaffected by hyperventilation - probably false: pulmonary vessel tone is affected by hyper/hypocapnoea.

A patient with severe COPD on home oxygen is having an excision of a submandibular tumour under local anaesthesia. The best way to prevent fire in the operating room is: A. seal the surgical site from the patients airway with adhesive drapes B. use bipolar instead of monopolar diathermy C. decr FIO2 to maintain sats 97% D. use alcoholic chlorhex instead of iodine E. add nitrous oxide to the inhaled gases to reduce the FiO2 and provide sedation

ANSWER B * A. seal the surgical site from the patients airway with adhesive drapes - potentially correct: "Oxygen is heavier than air, and can therefore accumulate under surgical drapes. This accumulation may be reduced by the use of 'incise drapes' that protect the wound from high oxygen concentrations and by tenting surgical drapes to dilute oxygen with room air." (Muchatuta and Sale, Fires and explosions, Anaesth&IC, 2007, 8:11) The answer will depend on what the examiners mean by adhesive drapes * B. use bipolar instead of monopolar diathermy - definitely correct: "The cutting mode of diathermy is more likely to ignite fuels than the coagulation mode, and fires are more likely with monopolar diathermy than bipolar" (Muchatuta and Sale, Fires and explosions, Anaesth&IC, 2007, 8:11) * C. decr FIO2 to maintain sats 97% - almost certainly incorrect: "Strategies to reduce the risks posed by high oxygen concentrations include (the) judicious use of oxygen (using the lowest oxygen concentration that provides acceptable haemoglobin oxygen saturations" (Muchatuta and Sale, Fires and explosions, Anaesth&IC, 2007, 8:11) The key here is acceptable haemoglobin oxygen saturations. "Long term continuous oxygen therapy should be considered for patients with stable chronic lung disease, particularly COPD, who have an arterial PO2 (PaO2) consistently less than or equal to 55 mm Hg when breathing air, at rest and awake. ...Flow rate should be set to maintain PaO2 > 60mmHg (8 kPa) (oxygen saturation level, measured by pulse oximetry [SpO2]> 90%) during waking rest." (McDonald et al, Adult domiciliary oxygen therapy. Position statement of the Thoracic Society of Australia and New Zealand, MJA 2005; 182: 621-626) Note that PO2=55mmHg is equivalent to SaO2 88% * D. use alcoholic chlorhex instead of iodine - definitely incorrect: "Alcohol-based antibacterial skin preparations are one of the more common causes of surgical fires since the withdrawal of flammable anaesthetic agents. They can pool on the body surface (especially umbilicus and suprasternal notch), be wicked into surgical drapes and produce flammable vapours that can accumulate beneath the drapes." (Muchatuta and Sale, Fires and explosions, Anaesth&IC, 2007, 8:11) * E. add nitrous oxide to the inhaled gases to reduce the FiO2 and provide sedation - definitely incorrect: "Nitrous oxide also supports combustion and is broken down to produce oxygen, nitrogen and heat." (Muchatuta and Sale, Fires and explosions, Anaesth&IC, 2007, 8:11)

Forty patients are randomly dived into two groups - one to receive induction agent A and another to receive induction agent B. The next day they are asked to rate their anaesthetic experience on a scale of 1 (very bad) to 5 (very good). The most appropriate test to compare the anaesthetic experience of the two groups is the A. unpaired t-test B. Mann-Whitney test C. Chi-square test D. Kruskal-Wallis test E. paired t-test

ANSWER B * A. unpaired t-test - false: Whilst the two groups are independent, the data is non-parametric. The unpaired t-test test "if the population means estimated by two independent samples differ significantly" (Myles and Gin p.52) * B. Mann-Whitney test - true: The scale is ordinal. "If there is a natural order among categories, so that there is a relative value among them... then the data can be considered ordinal data... Ordinal data are... a type of categorical data." (Myles and Gin p 2-3). There are two independent groups and the equivalent of the unpaired t-test for non-parametric data is the Mann-Whitney U test. "Mann-Whitney U Test (identical to the Wilcoxon rank sum) is a non-parametric equivalent to the unpaired Student's t-test" (Myles and Gin p.63) "The Mann-Whitney U test is the recommended test to use when comparing two groups that have data measured on an ordinal scale. However, if the data represent a variable that is, in effect, a continuous quantity, then a t-test may be used if the data are normally distributed. This is more likely with large samples (say n>100)." (p. 64) * C. Chi-square test * D. Kruskal-Wallis test * E. paired t-test - false. The two groups are independent and data non-parametric. A paired T-test is used "to test if the population means estimated by two dependent samples differ significantly." (p.52) "Paired tests are used when the two samples are matched or paired ("dependent"). The usual setting is when measurements are made on the same subjects before and after a treatment." (p.55)

In hypercalcaemia associated with malignancy: A. Bone metastasis is an essential feature B. The ECG characteristically shows a shortened Q-T interval C. Emergency treatment includes fluid restriction to control the associated polyuria D. Calcitonin has no place in management

ANSWER B * A: false - not essential - 40% ectopic PTH producing tumours * B: true * C: false * D: false

Another pregnant lady ?39/40 with BP185/115 , 4+proteinuria, clonus. IDC placed, 10mLs of dark coloured urine only for the last few hours. Initial management A. 500mL Crystalloid bolus B. IV hydralazine C. IV Magnesium D. insert epidural

ANSWER B * By definition she has SEVERE pre-eclampsia. * A - False. Although they are usually intravascularly deplete, IV fluids should be given cautiously as these women are prone to develop pulmonary oedema. Volume expansion alone can reduce SVR and systolic BP. Oliguria should be treated with careful IV fluid challenge. The BJA CEACCP article (see below) states that a bolus of 250ml crystalloid should be given, but if no improvement (in urine output) a CVC should be inserted before any further fluid given. Then be guided by CVP and urine output. Crystalloid decreases plasma oncotic pressure, while colloids increase it and have a greater tendency to cause pulmonary oedema. However, if CVP low, can use colloid. I say false because 500ml is probably a bit too large as a bolus initially. * B - TRUE. Initial management should aim to reduce the BP. This is the best of the options, although the CEACCP article also states that careful volume expansion should precede the use of vasodilators, so as not to drop the BP too much. That is not one of the options though, and I think this is the best option. * C - False. Initial attempts to reduce BP and improve urine output should probably precede Mg administration. Anyway, giving someone with minimal urine output a large bolus of IV Mg increases the chances of Mg toxicity. Should give IV fluids before giving Mg. * D - False. Epidurals are desirable in pre-eclampsia but in severe pre-eclampsia you should try and reduce the BP first, and always check the platelet count +/- coags BEFORE placing epidural.

Carbon monoxide halflife in a person who has stopped smoking: A. 1 hour B. 4 hours C. 12 hours D. 24 hours E. 72 hours

ANSWER B * The half-life of carboxyhemoglobin is reduced from ~ 240 mins on Room Air to ~ 75 to 80 mins at an FIO2 of 100% and to 20 mins with Hyperbaric oxygen (HBO) at 2.0 atmospheres

Jaundice without bilirubin in the urine is seen in: A. Carcinomatosis B. Acquired haemolytic anaemia C. Obstruction of common bile duct D. Infective hepatitis E. Chlorpromazine hepatitis

ANSWER B * This is also known as acholuric jaundice * The other causes are due to obstruction or failure of excretion of conjugated bilirubin (ie. intra and extra hepatic obstruction) o Prehepatic jaundice In prehepatic jaundice, excess unconjugated bilirubin is produced faster than the liver is able to conjugate it for excretion. The liver can excrete six times the normal daily load before bilirubin concentrations in the plasma rise. Unconjugated bilirubin is insoluble and is not excreted in the urine. It is most commonly due to increased haemolysis--for example, in spherocytosis, homozygous sickle cell disease, or thalassaemia major--and patients are often anaemic with splenomegaly. The cause can usually be determined by further haematological tests (red cell film for reticulocytes and abnormal red cell shapes, haemoglobin electrophoresis, red cell antibodies, and osmotic fragility).

In children with upper respiratory tract infections (URTIs) presenting for general anaesthesia A. airway events are less likely if an endotracheal tube is used B. oxygen desaturation to under 90% is more likely (compared to children without URTIs) C. parental smoking does NOT affect the incidence of airway events D. peri-operative breath holding is NOT more likely (compared to children without URTIs) E. the site of surgery has no effect on the incidence of airway events

ANSWER B "Results showed that children with active and recent URIs (in the last 4 wk) had significantly more episodes of overall respiratory events, breath-holding, major arterial oxygen desaturation (Spo2 <90%), and severe coughing compared with children with no URIs." "Independent risk factors for adverse respiratory events in children with active URIs included use of an ETT in a child <5 yr old, prematurity (<37 wk), history of reactive airway disease, paternal smoking, surgery involving the airway, presence of copious secretions, and presence of nasal congestion."

In its pure form, arnica (a herbal medicine), can cause postoperative A. angina B. bleeding C. prolonged sedation D. oculogyric crises E. vomiting

ANSWER B # "Two sesquiterpene lactones of Arnica montana, helenalin and 11, 13-dihydrohelenal have been shown to inhibit collagen-induced platelet aggregation, thromboxane formation, and 5-hydroxytryptamine secretion." (Uptodate) # "Use with caution in individuals with a history of bleeding, hemostatic disorders, or drug-related hemostatic problems. Use with caution in individuals taking anticoagulant medications, including warfarin, aspirin, aspirin-containing products, NSAIDs, or antiplatelet agents (eg, ticlopidine, clopidogrel, dipyridamole). Discontinue use prior to dental or surgical procedures (generally at least 14 days before)." (

In hyperaldosteronism, the following may be seen: A. Malignant hypertension B. Severe muscle weakness C. Hyperkalaemia D. Skin pigmentation E. Acidaemia

ANSWER B # A - false - Hypertension is usually mild (Ref: Oxford handbook) # B - true - Hypokalaemia can be marked and cause muscle weakness # C - false - Hypokalaemia occurs due to increased urinary secretion (in exchange for the increased Na+ reabsorption) # D - false - This distractor refers to Addison's Disease # E - False - hypokalaemic metabolic alkalosis due to H+ ion loss

When providing anaesthesia for a patient who is a Jehovah's Witness, it is NOT acceptable to use A. erythropoietin B. albumin and clotting factors C. cardio-pulmonary bypass D. isovolaemic haemodilution E. blood products for children, if parents insist that they be witheld

ANSWER B # A. erythropoietin - false # B. albumin and clotting factors - true: These are blood products and often will be refused by JW patients. Always should be done on a case by case basis though as some JWs are more liberal than others. # C. cardio-pulmonary bypass - false # D. isovolaemic haemodilution - false # E. blood products for children, if parents insist that they be witheld - false From CEACCP 04 - Children under 16 yrs of age of Jehovah's Witness parents present a difficult legal management problem. For elective procedures, there should be full and frank discussion between the surgeon, anaesthetist, parents and child (if they are old enough to understand). Most parents will accept that while every attempt will be made to avoid blood, a doctor will not allow a child to die for lack of transfusion. Children under 16 can legally give consent themselves if they can understand the issues involved (Gillick Competence). However, the courts have proved willing to overrule the refusal of specific procedures by children.

If a tingling sensation is felt when touching an active electrical device the leakage current is approximately: A. 80-120 micro Amp B. 300-400 micro Amp C. 1-2 milli Amp D. 10-15 milli Amp E. 100-200 milli Amp

ANSWER B # Tingling 0.5 mA # Pain at 1 mA # 2-15 mA feel pain and muscles contract but can let go # 100 - 200 mA will likely produce VF, but current density at heart depends on path of current. Also frequency important : 50 HZ is optimal (but very similar VF thresholds thru wide range 10 - 500 Hz)

AZ55b ANZCA version [2005-Apr] Q79 Application of cricoid pressure with a force of 40 newtons will resist reflux with an intra-oesophageal pressure of A. 30 mmHg B. 40 mmHg C. 50 mmHg D. 60 mmHg E. 70 mmHg

ANSWER B 40N resists 38mmHg and 40mmHg OESOPHAGEAL pressure depending on study. Halves LOS tone from 24 to 12.

Heparin induced thrombocytopenia (HITS) A. is associated with antibodies to complexes of anti-thrombin 3 (ATIII) and heparin B. is associated with a more rapid drop in platelet count if the patient has been exposed to heparin within the last 3 months C. is not associated with the use of low molecular weight heparins D. results in the maintenance of heparin-dependent antibodies indefintely after their development E. results in thrombotic complications in most patients

ANSWER B A - False * Clinically, there are two types of HIT that can result from heparin administration: HIT type I, a benign non-immune condition in which no heparin-dependent antibodies are present; and HIT type II, a immune-mediated syndrome caused by an antibody to the heparin-platelet factor 4 (PF4) complex B - True Thirty percent of patients present with 'rapid onset' HIT type II. They show an abrupt drop in platelet count when heparin is administered. These patients have often received heparin within the last 3 months. C - False * Less binding to PF4 occurs with low molecular weight heparin (LMWH) and pentassacharide, and consequently, these are less antigenic and cause HIT less frequently. D - False * HIT-IgG antibodies are transient and cannot be detected several weeks or months following the episode of HIT type II. HIT-IgG usually are not restimulated despite re-exposure to heparin. In the event that they are re-stimulated, the antibodies develop gradually usually after a 5-day interval E - False (?) * Of the patients who develop HIT-IgG seroconversion, 30-50% will develop thrombocytopenia; and of these, 30-80% will demonstrate isolated thrombotic events, 0.01-0.1% of whom will experience multiple thromboses or 'white clot syndrome' o I don't know if this is most...but only refers to HIT II...so overall maybe not that common

Thalassemia major:- A. Affects HbF more then HbA B. Most die around 25-30 years C. Liver failure is the most common cause of death D. Associated with microcytic normochromic anaemia E. Growth retardation is fixed by transfusion

ANSWER B A - False - No HbA, HbF normal B - True - 25-30 years due to cardiac siderosis (Oxford Handbook of Med) * Chronic transfusions with RBCs improves oxygen delivery, suppresses the excessive ineffective erythropoiesis, and prolongs life, but the inevitable side effects, notably iron overload, usually prove fatal by age 30...Harrison's C - False - CCF and chronic hypoxia D - False - microcytic, hypochromic * Hypochromia and microcytosis characterize all forms of beta thalassemia because of the reduced amounts of hemoglobin tetramers...Harrison's E - True - Development normal with adequate transfusion (OxHM) * If these children are transfused, the marrow is "switched off", and growth and development may be normal...ABC of Clinical Haematology (BMJ Books) * according to Harrison's 14th Ed 651, death in untreated pts is in 2nd decade (i.e. 20-30). Also says that bone deformity can be "largely prevented" by maintaining normal Hb level throughout.

Chronic post-operative pain A. in a phantom limb is reduced in incidence by administration of chemotherapy B. after thoracotomy has an incidence of approximately 50% C. following thoracotomy does NOT have its severity predicted by the severity of acute post-operative pain D. following mastectomy combined with implantation of a prosthesis is LESS likely to occur than following mastectomy alone E. following cholecystectomy is MORE likely if there is a history of classic gallbladder pain pre-operatively

ANSWER B A - False: "Administration of chemotherapy increases the incidence of phantom limb pain." B - True: "Long-term pain after thoracotomy, the postthoracotomy pain syndrome (PTPS), may have an incidence of more than 50%." C - False, this is the only predictor: "The intensity of acute postoperative pain is a statistically significant predictor of post-thoracotomy pain syndrome (PTPS) (36 vs. 56% PTPS for minor vs. moderate to severe acute pain)." D - False: "......found that mastectomy combined with implantation of a breast prosthesis yielded a higher incidence of pain (53%) than did mastectomy alone (31%)." E - False, has not been adequately investigated "A history of classic gallbladder attack symptoms is associated with reduced risk of chronic pain and symptoms."

Acute renal failure. Which is not an indication for dialysis ? A. Hyperkalaemia B. Metabolic alkalosis C. Hypernatraemia D. Uraemic pericarditis E. APO

ANSWER B A = acidosis E = electrolytes inc hyperkalaemia, hyper/hyponatraemia when assoc with ARF (see google books ref below) I = Ingestion toxins/drugs O = oedema U = uraemia

In hyperparathyroidism one finds: A. High serum calcium and low urine calcium B. High serum calcium and high urine calcium C. Serum calcium lowered by corticosteroids D. Serum calcium raised by corticosteroids

ANSWER B A False B True. Even though there is increased renal tubular calcium reabsorption, the very high amount of calcium filtered still results in an overall high urinary calcium concentration. C FALSE - "plasma calcium in hyperparathyroidism is resistant to suppression by steroids; this also occurs with some malignancies. Hypercalcaemia due to sarcoidosis, vitamin D-mediated hypercalcaemia and some malignancies, suppression to normal or near normal levels is seen" AKA "the hydrocortisone suppression test" Ref Kumar & Clarke "Clinical Medicine" D False

The left recurrent laryngeal nerve A hooks around the arch of the aorta anterior to the attachment of the ligamentum arteriosum B passes under cover of the lower border of the inerior constrictor muscle before entering the larynx C supplies the cricothyroid muscle D supplies sensation to the whole of the laryngeal mucosa on the left side E contains motor fibres derived from the spinal root of the accessory nerve

ANSWER B A false posterior to ligamentum arteriosum B TRUE C False innervates all intrinsic muscles of the larynx except the cricothyroid D False supplies sensation to mucosa BELOW the cords (Superior laryngeal nerve above cords) E False Vagus nerve.

Causes of inverted P waves in Lead II of the electrocardiogram include A. transposed lower limb leads B. junctional rhythm C. hypothermia D. left axis deviation E. inferior myocardial infarction

ANSWER B A negative P wave in lead I may be due to 1. transposition of left and right arm electrodes 2. dextrocardia 3. abnormal atrial rhythms A. FALSE : LIMB LEADS : I, II, III -these leads are bipolar I = LA - RA II = LL - RA III = LL - LA -transposing lower limb leads should not invert P waves B. TRUE : -The presence of P waves indicates that the atria have depolarised. -If P waves are upright in lead II and aVF they have originated from the SA node -Inverted P indicates that the retrograde depolarization is occuring through atria : junctional or ventricular C. FALSE : Hypothermia -bradycardia -long QT -J waves (rounded waves above the isoelectric point that immediately follow the R wave) D. FALSE E. FALSE

Arterial blood gases in a patient: pH 7.12, PO2 100, PCO2 65, HCO3 20.3, BE -10. These results are consistent with? A. Chronic renal failure B. Malignant hyperthermia C. Diabetic ketoacidosis D. End-stage respiratory failure E. Ethylene glycol toxicity

ANSWER B A significant mixed acidosis is present (high CO2 AND low HCO3). This makes A/C/E unlikely as they will usually cause hyperventilation and a LOW PCO2. D is also unlikely to be correct, as chronic respiratory failure will cause the HCO3 to rise. The best answer is B - consistent with Malignant Hyperthermia.

Effect of Injecting 5 mL of saline into the epidural space: a. increase incidence of patchy block b. decreased risk of epidural vein catheterisation c. no effect d. increased ease of threading catheter e. ? decreased effectiveness of block

ANSWER B A&A August 2007 (105, 2, 460-464) -Reduced risk of cannulation -Reduced risk of patchy block -Reduced time of onset -Easier to thread

The below would increase A-a oxygen gradient Except A. Increase FIO2 B. Decrease FIO2 C. Decrease cardiac output D. Increase shunt

ANSWER B A-a gradient * increases 5-7mmHg for every 10% increase in FiO2 * increases with age * increases with increased shunt * V/Q mismatch * defect in diffusion

MN38 ANZCA version [2004-Apr] Q124, [2005-Apr] Q100, [2005-Sep] Q94, [Jul07] [Apr08] [Aug08] [Aug09][Mar10] [Aug10] Respiratory function in quadriplegics is improved by A. abdominal distension B. an increase in chest wall spasticity C. interscalene nerve block D. the upright position E. unilateral compliance reduction

ANSWER B A. Abdominal distention? This doesn't make sense "as is". It does occur, reduced ab tone improves diaphragmatic effect, but abdo distention eg constipation, will reduce diaphragm effect. B. Spasticity of intercostal muscles may improve VC after a period of weeks the chest no longer collapses outwards. However how does compliance change? presumably more bang for your buck (incr V change/P change) From JCA [1] (http://www.sciencedirect.com.ezproxy.anzca.edu.au/science?_ob=ArticleURL&_udi=B6T83-49JFPP2-J&_user=2850510&_coverDate=08%2F31%2F2003&_rdoc=16&_fmt=full&_orig=browse&_srch=doc-info(%23toc%235075%232003%23999849994%23453360%23FLA%23display%23Volume)&_cdi=5075&_sort=d&_docanchor=&view=c&_ct=17&_acct=C000056830&_version=1&_urlVersion=0&_userid=2850510&md5=9daf80181f2b6676679ec23c07dcaa69) "Lesions below the C5 level (T1-T12) affect intercostal and abdominal muscles. In this situation, respiratory impairment results predominately from inspiratory and expiratory muscle weakness; intercostal muscle function is altered and there is a resulting paradoxic inward rib cage movement on inspiration. These patients most often do not require mechanical ventilatory support unless there are also associated injuries, such as cerebral involvement, cardiac or pulmonary contusions, pulmonary edema, or respiratory insufficiency from pneumonia. In this type of injury, improvement in ventilatory mechanics begins as early as 3 to 5 days after the initial insult, because muscle function improves as a result of the transition from flaccid paralysis to muscle spasticity. During this transition, there is an increase in muscular tone that promotes rib cage stability and decreases paradoxical chest wall movement during the inspiratory phase, improving vital capacity, and overall inspiratory function for up to 6 months. This gradual improvement is more prominently noted for inspiratory than expiratory muscles." Hence I think B is correct, increased spasticity causes increased chest wall compliance. C. Anyone with the ability to do an interscalene block hopefully isn't stupid enough to do it in a quad. D. when patients are nursed in the upright position the diaphragm is at a disadvantage. If the diaphragm is able to contract effectively in this position, the patient will ventilate supine as well, but not vice versa. E. FALSE: presumably if quadraplegia is present compliance would be affected on both sides. This may be an incomplete answer.

In patients giving a history of penicillin allergy A. cross-sensitvity to cephalosporins occurs in approximately 30% of patients B. there is an increased liklihood of allergic reactions to neuromuscular blocking drugs C. beta-lactamase resistant penicillins are less likely to provoke a response D. a 'test dose' of one tenth the planned dose of penicillin should be given with a 5 minute delay before the main dose

ANSWER B A. FALSE : 8-10% cross reactivity B. TRUE : patients with penecillin allergy are 3 times more likely to experience anaphylaxis to other drugs C. FALSE : immunological reaction is against b-lactam ring D. FALSE : test dose is not given if they are known to be allergic.

When considering an acute myocardial infarction: A. Aspirin and clopidogrel should not be given together B. PCI is better than thrombolysis C. Aspirin should not be given prior to MI confirmed with cardiac enzyme rise D. Reperfusion can be delayed for 24hrs E. something about confirming cardiac enzyme elevation before instituting reperfusion strategies

ANSWER B A. FALSE : All patients with AMI should be loaded with aspirin and continued life long. Clopidogrel loading depends on whether the institution offers CABGs, if CABGs are not offered then clopidogrel is commenced. Clopidogrel is continued 1. Angioplasty no clopidogrel 2. BMS 1 month 3. DES 1 year

Regarding patients aged 65 years or older with recurrent atrial fibrillation (AF) A. amiodarone and digoxin have similar efficacy in restoring sinus rhythm B. patients who have been reverted to sinus rhythm should still remain on warfarin therapy C. patients who remain in atrial fibrillation with heart rates less than 80 beats per minute do NOT require long term warfarin therapy D. peri-operative therapy with a beta-blocker will commonly lead to restoration of sinus rhythm E. restoration of sinus rhythm with electrical DC cardioversion improves long-term survival in comparison to controlling heart rate alone

ANSWER B A. FALSE : Amiodarone and sotolol are used for chemical cardioversion, digoxin is used for rate control B. TRUE : a patient who undergoes cardioversion should at least remain on anticoagulative therapy for a short while after DC reversion because of the risk of atrial thrombus formation in the stunned atrium. C. FALSE : not part f CHADS2 Score or HAS-BLED D. FALSE : perioperative beta blockers will not restore SR with recurrent AF, beta blockers are mainly used for rate control E. FALSE : AFFIRM trial showed no survival benefit

Anaphylaxis is least likely in the following situations: A. Muscle relaxants given to patients with documented IgE antibodies to thiopentone B. Protamine given to patient on protamine linked insulin C. Exposure to latex allergy in patient with fruit allergy D. Women vs men E. Adults vs neonates

ANSWER B A. FALSE : Antibodies are known to cross-react between muscle relaxants and the pyrimidine nucleus of STP. Specific IgE against quartenary ammonium ions. B. TRUE : theoretic link, however, no documented evidence of it occurring. C. FALSE : Latex allergy is indeed more common in those with fruit allergy such as grapefruit allergy, also reported with kiwi fruit, bananas and avocados. D. FALSE : Females overwhelmingly > males, especially to neuromuscular blockers E: FALSE : Adults >> neonates

In the treatment of phantom limb pain A. calcitonin infusion is NOT effective B. gabapentin reduces the pain C. intravenous lignocaine reduces the pain D. ketamine provides long-term pain relief E. opiates are NOT effective

ANSWER B A. FALSE : Calcitonin by IV infusion is effective in the treatment of acute phantom limb pain (Level II). Calcitonin may also be given subcutaneously or intranasally. It was not effective for chronic phantom limb pain (Level II). B. TRUE : Gabapentin was effective in reducing phantom limb pain C. FALSE : IV lignocaine has no effect on phantom pain but is effective in reducing stump pp ain (level II) D. FALSE : Ketamine, an NMDA-receptor antagonist (see Section 4.3.2), provided short-term relief of stump and phantom limb pain. Perioperitve ketamine may orevent severe phantom limb pain E. FALSE : multiple modal therapy inclusive of opiates. KEY MESSAGES 1. Continuous regional blockade via nerve sheath catheters provides effective postoperative analgesia after amputation, but has no preventive effect on phantom limb pain (U) (Level II). 2. Calcitonin, morphine, ketamine, gabapentin, amitriptyline and tramadol reduce phantom limb pain (S) (Level II). 3. Sensory discrimination training and motor imagery reduce chronic phantom limb pain (S) (Level II). Examples include sensory discrimination training, mental imagery of limb movement and motor imagery. 4. Ketamine, lignocaine (lidocaine), tramadol and amitriptyline reduce stump pain (S) (Level II). 5. Perioperative epidural analgesia reduces the incidence of severe phantom limb pain (U) (Level III-2).

Carcinoid tumours A. arise in the gastrointestinal tract in approximately 40% of cases B. result in the carcinoid syndrome in approximately 50% of cases C. release vasoactive agents including histamine, kallikrein and prostaglandins D. are NOT associated with hypergylcaemia E. do not usually result in intraoperative hypotensive episodes

ANSWER B A. FALSE : Gastrointestinal carcinoids are slow growing neuroendocrine tumors that may arise anywhere in the gastrointestinal tract. *small intestine (30%) *rectum (12%) *colon (8%) *appendix (8%) *stomach (10-30%) B. FALSE : Carcinoid syndrome occurs in less than 10%; hepatic metastases are generally present. C. TRUE : Carcinoid tumors can contain numerous GI peptides including gastrin, insulin, somatostatin, motilin, neurotensin, tachykinins (substance K, substance P, neuropeptide K), glucagon, gastrin-releasing peptide, vasoactive intestinal peptide, pancreatic peptide, other biologically active peptides (corticotropin, calcitonin, growth hormone), prostaglandins, and bioactive amines (serotonin). D. FALSE E. FALSE : manipulation of the tumour you may get carcinoid hypotnesive crises no matter where the tumour lies

MR48 [Aug08] COPD patient with pulmonary hypertension and acute RHF. Which is TRUE? a. 100% oxygen will decrease the pulmonary artery pressure b. Sildenafil will be useful for treating RHF c. Noradrenaline is an appropriate inotrope for this patient d. ? e. ?

ANSWER B A. FALSE : HPV is most prominent with PAO2<70mmHg. -Alveolar PAO2>100mmHg there is little change in vascular resistence B. TRUE : Oral sildenafil has been used successfully to manage acute RV dysfunction in heart transplant recipients, wean patients from nitric oxide, prevent rebound hypertension in patients on nitric oxide, reduce the duration requirement for mechanical ventilation, and prevent the occurrence of pulmonary endothelial cell dysfunction C. FALSE : Noradrenaline will partially constrict pulmonary vasculature increasing pulmonary vascular resistance. Furthermore, noradrenaline is almost never used for its apparent inotropy, rather for its pressor action. If an inotrope is required Milronone would be a better agent.

PP98 ANZCA version Jul07 With respect to gastric volumes and fasting in children A. casein-predominant milks empty faster than whey-predominant milks B. children have a higher incidence of aspiration than adults C. solids rely on first order kinetics for gastric emptying but liquids follow zero order kinetics D. the rate of gastric emptying is NOT related to the energy content of the meal E. unlimited clear fluid ingestion 2 hours before surgery does NOT affect volume, but does affect the pH of stomach contents

ANSWER B A. FALSE : Human milk and whey predominant formula empty faster than casein predominant formula and cow's milk. Probably due to lower protein content in human and whey based milk. B. TRUE : Paediatric aspiration is slightly more common than in adults (1 per 1200-2600 compared to 1 per 2000-3000 in adults) C. FALSE : Solids follow zero order (linear decay) kinetics and liquids first order (exponential decay), in regards to stomach emptying. D. FALSE : fats slower than CHO E. FALSE : Clear fluids up to 2 hrs preop do not increase gastric volume or signicantly change gastric pH (there may be an increase in pH with clear fluids up to 2 hrs)

The most correct statement concerning thyroid diseases is that A. thyrotoxicosis has the same incidence in men and women B. thrombocytopaenia is associated with thyrotoxicosis C. the half life of T 4 (thyroxine) is 2 days D. beta-blockers are contraindicated in acute thyrotoxicosis E. carbimazole acts within 2-3 days to reduce T4 synthesis

ANSWER B A. FALSE : Men: Female is 9:1 B. TRUE C. FALSE : T3 half life is 7 days D. FALSE: esmolol is the first drug used for symptom control E. FALSE : improvement in 1-2 days Carbimazole • Onset of action: Antithyroid: Oral: 12-18 hours • Duration: 36-72 hours • Distribution: Concentrated in thyroid gland; crosses placenta; enters breast milk (1:1) • Protein binding, plasma: None • Metabolism: Hepatic • Bioavailability: 80% to 95% • Half-life elimination: 4-13 hours • Excretion: Urine (80%)

An INCORRECT statement regarding the autonomic nervous system is that A. autonomic dysfunction is a predictor for worse long term survival after myocardial infarction B. heart rate responses are primarily mediated through the sympathetic nervous system C. inhalation anaesthetics all impair autonomic reflex responses D. autonomic dysfunction is a predictor for haemodynamic instability following anaesthetic induction E. low heart rate variability is associated with worse cardiac outcomes following non-cardiac surgery

ANSWER B A. FALSE : There also is strong evidence in patients that autonomic balance and vagal reflexes influence morbidity and mortality after MI. Patients surviving MI who had either low HRV or low baroreflex sensitivity had an increased risk of subsequent sudden cardiac death B. TRUE : The sympathetic component is primarily involved in adjusting peripheral vascular tone and plays a lesser role in the reflex regulation of HR and cardiac output. The vagal component is primarily involved in regulating HR. C. FALSE : All inhalational agents impair the autonomic nervous system D. FALSE : it is well documented that patients with impaired autonomic reflexes (e.g., patients with diabetes) have greater intraoperative BP lability compared with autonomically intact patients. [2,3] In addition, it now is clear that many of the sedative, hypnotic drugs used for induction of anesthesia and all of the potent inhaled anesthetic gases in clinical use impair autonomic reflex responses. E. FALSE : low HR variability (HRV), an index of impaired cardiac-vagal tone, is an independent predictor of mortality after non-cardiac surgery

A permanent dual-chamber transvenous pacemaker set to a DDDR mode will A. NOT be affected by diathermy B. increase the ventricular pacing rate if the minute volume is increased C. usually require reprogramming to VVO mode for surgery D. revert to DDD mode if a magnet is placed over the generator site E. result in a tachyarrhyrthmia if the patient develops atrial flutter

ANSWER B A. FALSE : XXDX will both sense and therefore trigger or inhibit based on the detected rhythm. Only in VOO or DOO (asynchronous pacing) B. TRUE : XXXR Rate Responsive Pacemakers -detect increases in metabolic rate by movement via piezoelectric crystals, minute ventilation via thoracic impedance, QT interval and stroke volume C. FALSE : depends if the patient is pacemakers dependent -yes? then set to VOO -no? leave it alone or disable but put on pacing pads -if the patient has an underlying rhythm, there is a risk of R on T. D. FALSE : PPM revert to VOO or DOO, AICD turns off shock function E. FALSE : This is a dual chamber pacing and sensing pacemaker. Whilst the atrial pacing may increase, the ventricular pacing will still function. Hence a tachyarrhythmia is only possible if all atrial impulses are conducted normally. Often this is not the case as the AV node aids to slow down conduction.

In the treatment of phantom limb pain A. calcitonin infusion is NOT effective B. gabapentin reduces the pain C. intravenous lignocaine reduces the pain D. ketamine provides long-term pain relief E. opiates are NOT effective

ANSWER B A. FALSE : calcitonin SC or intranasal is effective in treating acute phantom limb pain B. TRUE C. FALSE : IV lignocaine is effective for treatment of stump pain but not phantom limb pain D. FALSE : periopertive ketamine may prevent severe phantom limb pain, but does not reduce the incidence of phantom limb pain E. FALSE : Calcitonin, morphine, ketamine, gabapentin, amitriptyline and tramadol reduce phantom limb pain

The ascending aorta A has no branches B begins at the semilunar valve C arises from right ventricle D occupies the superior mediastinum E lies inferior to the SVC

ANSWER B A. FALSE : coronary arteries B. TRUE C. FALSE : left ventricle D. FALSE : middle mediastinum E. FALSE : left and anterior

Regarding patients aged 65 years or older with recurrent atrial fibrillation (AF) A. amiodarone and digoxin have similar efficacy in restoring sinus rhythm B. patients who have been reverted to sinus rhythm should still remain on warfarin therapy C. patients who remain in atrial fibrillation with heart rates less than 80 beats per minute do NOT require long term warfarin therapy D. peri-operative therapy with a beta-blocker will commonly lead to restoration of sinus rhythm E. restoration of sinus rhythm with electrical DC cardioversion improves long-term survival in comparison to controlling heart rate alone

ANSWER B A. FALSE : digoxin is used for rate control not for cardioversion B. TRUE : must remain on warfarin for 3-6 months, unless a thrombus has been excluded by TOE C. FALSE D. FALSE E. FALSE

In providing anaesthesia for a patient with Eisenmenger's syndrome, it is NOT true that A. an important goal is to maintain an optimal shunt, by preventing changes to pulmonary vascular resistance (PVR) or systemic vascular resistance (SVR) B. the patient's high haemoglobin should be maintained and blood loss monitored closely C. a gaseous induction with sevoflurane presents an effective method for anaesthesia and avoids cardiovascular compromise D. if general anaesthesia is required, ketamine is an - appropriate choice of drug E. careful attention to intravenous infusions and drug administration is needed to prevent paradoxical air embolism

ANSWER B A. FALSE : fine balance of PVR and SVR to maintain the optimal shunt. B. TRUE : Compensate for hypoxia by erythrocytosis with increase in Hct. HCT>65% should have vensection before to minimize risk of thromboembolism. Blood loss should be closely monitored. C. FALSE : Even though gaseous induction theoretically will drop SVR and increase R to L shunt. D. FALSE : Ketamine has been the most popular agent for anesthetic induction in patients with cyanotic conditions because it increases SVR and cardiac output, thereby diminishing the magnitude of R-L shunting. E. FALSE : care should be taken to avoid iatrogenic air emoblism as there is a clear right to left shunt Eisenmenger's syndrome -patients who left to right shunt has been reversed resulting from increased PVR -explosure of pulmonary vasculature to increased blood flow and pressure (VSD or ASD) results in pulmonary obstructive disease -obliteration of pulmonary vascular bed progresses with increases in PVR -when PVR = SVR : eisenmenger's syndrome -shunt reversal in 50% untreated VSD and 10% untreated ASD Signs and symptoms -cyanosis -decreased exercise tolerance -paplitations are common due to AF/AFlu -arterial hypoemia resulting in polycythemia : visual disturbances, headache, dizziness, paresthesia -hemoptysis -thrombosis -CVA and brain abscess -sudden death Treatment -mainly prevention -phlebotomy with isovolmetric replacement to reduces risks of hyperviscosity -lung transplant with repair of heart defect in selected cases Anaesthetic Management * Preop assessment: Cardiac function ? failure, degree of shunt (SpO2, echo), rhythm, antiarrhythmics, hyperviscosity syndrome (HCT > 65% ie Hb 210), associated congenital problems. * Monitoring: Art, CVL, ? TOE, ? PICCO. ? Monitor sats in both arms (PDA). * Preinduction: antibiotic prophylaxis(amoxycillin and gentamicin), ? phlebotomy (haemodilution). * Physiological goals: maintain SVR:PVR ratio, maintain cardiac output. Prevent air/paradoxical embolus. * Induction: anything is OK, maintain SVR. * Maintenance: anything is OK except N2O. Consider selective pulm vasodilator (NO, PGI2). * Neuraxial: slow onset and watch SVR. * Postop: ICU. DVT prophylaxis. Effective analgesia.

Heparin Induced Thrombocytopenia (HITS) A. is associated with antibodies to complexes of Antithrombin 3 (ATIII) and heparin B. is associated with a more rapid drop in platelet count if the patient has been exposed to heparin within the last three months C. is not associated with the use of low molecular weight heparins D. results in the maintenance of heparin-dependent antibody levels indefinitely after their development E. results in thrombotic complications in most patients

ANSWER B A. FALSE : heparin binds to platelet factor 4 which together forms a neoantigen which causes a humoral immune response. ATIII has nothing to do with it. B. TRUE : 'Rapid onset' HIT is assoc with a drop in platelet count within minutes to hours of heparin exposure and affected patients will usually have been exposed to heparin within the last 3 months. C. FALSE : Low molecular weight heparins can cause HIT, and when used in its treatment there is a significant risk of recurrent or progressive thrombocytopenia with or without thrombosis D. FALSE : The antibodies to heparin-platelet factor 4 become undetectable within 50 to 85 days on stopping heparin exposure. E. FALSE : Less than 10% of those who develop an antibody to the heparin-PF4 complex will exhibit a thrombotic event. However the risk varies considerably with the clinical situation and can reach 40% or more in the postop setting when high circulating levels of both activated platelets and thrombin are present, for example, following orthopedic surgery

IC85 [Apr07] Q115 A young woman has an MVA when driving home from a nightclub. She was known to have moderate MDMA (ecstasy) consumption. Which of the following complications is least likely to be due to MDMA? A. Hyponatraemia B. Hypotension C. Pneumothorax D. Sweating E. Tachycardia

ANSWER B A. FALSE : multifactorial due to dehydration, excessive sweating, increase water intake, surge in ASH -can result in seziures and coma B. TRUE : MDMA causes tachycardia and hypertension. Any hypotension in a trauma situation is hypovolaemia until proven otherwise. C. FALSE : there are case reports of pneumonthorax attributed to estasy as the result of excessive physical activity against a closed glottis (valsalva) => alveolar rupture D. FALSE E. FALSE 2 Minor clinical symptoms and signs seen with MDMA Tachycardia Elevated mood Hypertension Confusion Mydriasis Ataxia Dry mouth Nystagmus Sweating Bruxism (jaw clenching)

A patient with severe COPD on home oxygen is having an excision of a submandibular tumour under local anaesthesia. The best way to prevent fire in the operating room is: A. seal the surgical site from the patients airway with adhesive drapes B. use bipolar instead of monopolar diathermy C. decr FIO2 to maintain sats 97% D. use alcoholic chlorhex instead of iodine E. add nitrous oxide to the inhaled gases to reduce the FiO2 and provide sedation

ANSWER B A. FALSE sealing may cause gas / oxygen entrapment, creating a reservoir. This accumulation may be reduced by the use of 'incise drapes' that protect the wound from high oxygen concentrations and by tenting surgical drapes to dilute oxygen with room air.. B. TRUE : fires are more likely with monopolar diathermy than bipolar C. FALSE :Strategies to reduce the risks posed by high oxygen concentrations include (the) judicious use of oxygen (using the lowest oxygen concentration that provides acceptable haemoglobin oxygen saturations. Aim for SatO2 88% or PO2 55mmHg. D. FALSE : alcoholic chlorhex more flammable than traditional iodine in potassium iodide E. FALSE : nitrous oxide supports combustion

SF29e ANZCA version [2005-Sep] Q76 When magnesium sulphate is used in the treatment of preeclampsia A. its tocolytic action commonly delays the progress of labour B. it potentiates neuromuscular blockade by depolarising muscle relaxants C. therapeutic blood levels are 6 - 8 mmol.l-1 D. when given as an infusion it frequently produces hypotension E. none of the above

ANSWER B A. FALSE: In term nulliparous women, neither preeclampsia nor magnesium prophylaxis affected labor duration. B. TRUE C. FALSE 2-3.5mmol/L D. TRUISH E. FALSE

Bronchospasm may be induced in the asthmatic patient by: A. Suxamethonium B. Intubation C. Tubocurarine D. Halothane E. Opiate premed

ANSWER B A. Suxamethonium - false: Administration of neuromuscular-blocking drugs relieves the difficulty of ventilation due to light anestheis but has no effect of bronchospasm B. Intubation - true: During induction and maintenance of anesthesia in asthmatic patients, it is necessary to suppress airway reflexes to avoid bronchoconstriction in response to mechanical stimulation of these hyperreactive airways C. Tubocurarine - false: Administration of neuromuscular-blocking drugs relieves the difficulty of ventilation due to light anestheis but has no effect of bronchospasm D. Halothane - false: The lesser pungency of halothane and sevoflurane (compared with isoflurane and desflurane) may make coughing, which can trigger bronchospasm, less likely E. Opiate premed - false

Protamine side effects include all except A. Impaired platelet function B. Systemic hypertension C. Pulmonary hypertension D. Hypotension E. Anaphylaxis

ANSWER B A. TRUE : Protamine administered IV in the absence of heparin interacts with platelets and proteins, including fibrinogen B. FALSE : histamine release causing hypotension C. TRUE : See question above D. TRUE : Too rapid administration of protamine sulfate may cause severe hypotension and anaphylactoid reactions E. TRUE : Hypersensitivity reactions and fatal anaphylaxis have been reported

Immunologically mediated heparin-induced thrombocytopaenia is characterised by A. onset within a few days of first starting heparin B. intravascular thromboses C. platelet count rarely reduced below 100x10^9/L D. continuation of thrombocytopaenia after cessation of heparin E. presence of non-specific (heparin-independent) platelet antibodies

ANSWER B A. onset within a few days of first starting heparin - FALSE : "A second form of HIT, HIT type II or immune-mediated HIT, demands more attention. In patients receiving heparin for more than 5 days, antibodies to the heparin-platelet factor 4 complex can form, which are capable of binding to platelet Fc receptors and inducing platelet activation and aggregation." (Stoelting Ch17) B. intravascular thromboses - TRUE : "In vivo, this leads to both an increased clearance of platelets with resultant thrombocytopenia and venous and/or arterial thrombus formation, with the potential for severe organ damage (loss of limbs, stroke, myocardial infarction) as well as unusual sites of thrombosis (adrenal, portal vein, skin)." C. platelet count rarely reduced below 100x10^9/L - FALSE D. continuation of thrombocytopaenia after cessation of heparin - FALSE : will not continue indefinitely, and once the heparin concentration drops the process of platelet activation stops allowing platelet counts to recover. E. presence of non-specific (heparin-independent) platelet antibodies - FALSE

With respect to gastric volumes and fasting in children, A. casein-predominant milks empty faster than whey-predominant milks B. children have a higher incidence of aspiration than adults C. solids rely on first order kinetics for gastric emptying but liquids follow zero order kinetics D. the rate of gastric emptying is NOT related to the energy content of the meal E. unlimited clear fluid ingestion 2 hours before surgery does NOT affect volume, but does affect the pH of stomach contents

ANSWER B A: FALSE: Human milk and whey predominant formula empty faster than casein predominant formular and cow's milk. Probably due to lower protein content in human and whey based milk. B. Paediatric aspiration is slightly more common than in adults (1 per 1200-2600 compared to 1 per 2000-3000 in adults) C. FALSE: Solids follow zero order (linear decay) kinetics and liquids first order (exponentail decay) D. FALSE E. FALSE: Clear fluids up to 2 hrs preop do not increase gastric volume or signiicantly change gastric pH (there may be an increase in pH with clear fluids up to 2 hrs)

Correct statements regarding Hypertrophic Cardiomyopathy include each of the following EXCEPT that it A. results in a left ventricular wall thickness of greater than 12 mm B. is a condition where the left ventricle is hypertrophied and dilated C. is predominantly a non-obstructive disease D. is most easily and reliably diagnosed with 2-dimensional echocardiography E. the hypertrophy is characteristically asymmetrical

ANSWER B A: True. usually >15mm (12mm or less is considered normal) B: False. hypertrophic yes, dilated no C: True. "75%" have no outflow obstruction D: This is an excellent way to diagnose. E: True. Asymmetric septal hypertrophy.

A patient with known ischaemic heart disease undergoes percutaneous coronary artery stenting (with a non drug-eluting stent) prior to major elective surgery. Ideally the surgery should be postponed for A. 1-4 weeks B. 4-8 weeks C. 8-12 weeks D. 12-24 weeks E. over 24 weeks

ANSWER B ACC/AHA guidelines [1] (http://www.americanheart.org/downloadable/heart/1013454973885perio_update.pdf) (see page 31) * case series show significant incidence of perioperative death and major bleed for patients who had surgery within 14 days of stent placement. * recommend waiting a minimum of 2 weeks, but 4 weeks is preferable. * waiting longer than 6 to 8 weeks puts you into the time period when restenosis [if it is to occur] will begin occuring. * pts are usually on antiplatelet rx for 4 weeks, so optimal time period seems to be 4-6 weeks, allowing time for stoppage of antiplatelet agent preop if surgeon desires [but ideally not stopping before the 4 week mark.

Timing of peak respiratory depression post intrathecal 300 mcg morphine: A. < 3.5 hours (think it was one hour) B. 3.5 - 7.5 hours (then three hours) C. 7 - 12.5 hours (then 7.5 - 12.5 hrs) D. 12.5 -18 hours E. > 18 hours

ANSWER B ANZCA Acute Pain: Scientific Evidence 3rd ed p195: "Respiratory depression occurs in up to 1.2% to 7.6% of patients (Meylan et al, 2009 Level I) given intrathecal morphine. When measured in opioid-naive volunteers, respiratory depression peaked at 3.5 to 7.5 hours following intrathecal morphine at 200 to 600 mcg doses (Bailey et al, 1993 Level IV). Volunteers given 600 mcg had significant depression of the ventilatory response to carbon dioxide up to 19.5 hours later.

Most common cause of maternal cardiac arrest A. PE B. AFE C. Haemorrhage D. Preeclampsia E. cardiomyopathy

ANSWER B ANZCA blue book 2009 Maternal morbidity and mortality, Table 1 p 17. Major causes of cardiac arrest during pregnancy: VTE, AFE, haemorrhage, HTN, sepsis, trauma, cardiac disease, iatrogenic.

AZ62 ANZCA version [2001-Aug] Q63, [2002-Mar] Q58 A 91 year old farmer presents for anaesthesia for a total hip replacement. He gives a history of failed endotracheal intubation with his last two anaesthetics, probably due to arthritis of the cervical spine. The rest of the history and examination is unremarkable. His American Society of Anesthesiologists Physical Status Score (ASA Status) is A. ASA 1 B. ASA 2 C. ASA 3 D. ASA 4 E. ASA 5

ANSWER B ASA I: * patient's health: excellent with no systemic disease * limitations on activity: none * danger of death: none * excluded: persons at extremes of age (very young very old) ASA II: * patient's health: disease of one body system * status of underlying disease: well-controlled * limitations on activity: none * danger of death: none ASA III: * patient's health: disease of more than one body system or one major system * status of underlying disease: controlled * limitations on activity: present but not incapacitated * danger of death: no immediate danger ASA IV: * patient's health: poor with at least one severe disease * status of underlying disease: poorly controlled or end-stage * limitations on activity: incapacitated * danger of death: possible ASA V: * patient's health: very poor moribund * limitations on activity: incapacitated * danger of death: imminent ASA VI: * brain-dead patient undergoing organ harvest

Petit mal epilepsy - Which is true? (or words to that effect) A: Most common in child <2 years old B: Can precipitate seizures by hyperventilating C: Often seizures last for more than 30 seconds D: Rarely familial E: Isoelectric EEG during seizure (burst suppression)

ANSWER B Absence seizures -abrupt and sudden onset of impairment of consciousness (blank stare, possible upward rotation of eyes) -usually brief <20sec -EEG generalised spike and slow wave discharges -hyperventilation triggers in 90% -treatment mainly valproate (second line lamotrigine) -should not use carbamazepine, vigabatrin, tiagabine, phenytoin, gabapentin and pregabalin --> not effective in treatment

72 year old has had hip replacement surgery and 3 days postop has a pulmonary embolus. He is fully heparinised, but still dyspnoeic, clammy, BP 80/40, pulse 120 and CVP 18. The most appropriate next step is A. IVC filter B. Refer him for a pulmonary embolectomy C. Supportive (fluids and inotropes) D. Thrombolysis E. Warfarin

ANSWER B Absolute Previous intracranial bleeding at any time, stroke in less than 3months,closed head or facial trauma within 3 months,suspected aortic dissection ,ischemic stroke within 3 months(except in ischemic stroke within 3hours time), active bleeding diathesis, uncontrolled high blood pressure (>180 systolic or >100 diastolic),known structural cerebral vascular lesion viz av malformations. [edit] Relative Current anticoagulant use, invasive or surgical procedure in the last 2 weeks, prolonged cardiopulmonary resuscitation (CPR) defined as more than 10 minutes, known bleeding diathesis, pregnancy, hemorrhagic or diabetic retinopathies, active peptic ulcer, controlled severe hypertension.

Which patient do you not put a left-sided Robert-Shaw DLT into? A. Left pneumonectomy B. Left main bronchial lesion C. There is a right-sided broncho-pleural fistula D. The patient has shunt > 10% E. The left lung is to be collapsed

ANSWER B Acc to Miller's, Left pneumonectomy can be done with L sided tube, but needs to be retracted prior to stapling, therefore, B is right. Both according to Laurence talk Always a left: exceptions Anatomical distortion of the LMS (left mains stem bronchus) (TB, tumour) Sleeve resection LMS Left pneumonectomy Extrinsic compression of LMS by mediastinal tumour or aortic arch (dissection)

Recognised factors that predispose upper limb nerves to compression, under anaesthesia include A. forearm extension and supination B. extreme flexion of elbows across chest C. internal rotation of abducted arm D. lateral position - uppermost arm flexed in arm support E. prone position - arms by side and fingers flexed

ANSWER B According to the Anaesthesia article (Peripheral nerve injuries associated with anaesthesia), the following positions are a problem: * abduction of shoulder > 90 degrees, especially if elbow fully extended or externally rotated * forearm should not be extended and pronated * extremes of elbow flexion should be avoided

Acromegaly is associated with ONE of the following: A. Aortic incompetence B. Postop respiratory obstruction C. Aspiration D. hypohidrosis E. Renal cell carcinoma

ANSWER B Acromegaly -rare clinical syndrome caused by excessive Growth Hormone Production (Anterior Pituitary) CVS : hypertension 30%, IHD, cardiomyopathy, heart failure, valvular disease Airway : large jaw, head, tongie, lips, hypertrophy of larynx and trachea, OSA, enlarged thyroid Drugs : -somatostatin causing nausea and vomiting -bromocriptine : postural hypotension Neurological -raised ICP -nerve entrapment Endocrine -25% diabetic

Burns dressings. The following is proven to be of analgesic benefit: A. Morphine gel B. Biosynthetic dressings (the answer per pain book) C. Dexmedetomidine IV D. Lignocaine IV E. Cognitive/Distraction technique

ANSWER B Acute Burn Injury Key Messages 1. Chronic pain reported 35-60% of burn patients 2. Immediately after injury : cooling, covering, elevating and immobilising may provide analgesia -severe pain is best achieved by titration of IV opiods -conversation to oral opiods is possible once normal GI function has returned 3. Opiods adjuncts a. ketamine may reduce hyperalgesia b. clonidine is opiods sparing c. gabapentin reduces pain, is opiod sparing following acute burns and reduced neuropathic pain 4. The use of biosynthetic dressings is associated with a decrease in time to healing and a reduction in pain during burn dressings changes 5. Short‐acting opioids such as fentanyl administered via PCA or target‐controlled infusions 6. IN fentanyl was a viable alternative to oral morphine in children for burn dressings 7. N2O, ketamine and IV lignocaine infusions have also been used to provide analgesia for burn procedures 8. PCA with a ketamine and midazolam mixture was effective and well‐tolerated when used for analgesia and sedation during burn dressings 9. Augmented reality techniques virtual reality or distraction techniques reduce pain during burn dressings.

Which of the following is NOT seen in acute haemoglobinaemia? A. Fever B. Increased bilirubin in the urine C. Increased stercobilinogen in the gut D. Increased urobilinogen E. Reticulocytosis

ANSWER B Acute haemoglobinaemia = intravascular haemolysis Features * Fever * Pallor (if anaemic) * Jaundice * Splenomegaly * Urine: * haemoglobinuria * haemosiderinuria * increased urobilinogen * no increased urine bilirubin * Haemoglobinaemia * Methaemalbuminaemia

15kg child found fitting on paeds ward ?24h ?48h postop while on infusion of 60ml/h ½ NS + Dextrose. Now intubated. Na is 119, next management step: A) frusemide B) hypertonic saline C) Normal saline at 20ml/hr D) Water restrict E) Phenytoin

ANSWER B Acute symptomatic hyponatraemia. Aim to raise serum Na by 2mmol/hr until symptom resolve. Complete correction is unnecessary, although not unsafe. Infuse hypertonic saline (3% NaCl) at a rate of 1.2-2.4ml/kg/hr through a large vein... If there are severe neurological symptoms, eg seizure, coma, 3%NaCl may be infused at 4-6ml/kg/hr. Electrolyte should be carefully monitored.

Which of the following electrolyte patterns may be found in Addison's Disease: A. High Na+ and Cl- B. Low Na+ and Cl- C. High Na+ and K+ D. Low Na+ and high Cl- E. Low K+ and high Cl-

ANSWER B Addisons Disease : Primary adrenocortical insufficiency. The adrenal cortex produces a number of hormones * Mineralocorticoids e.g. aldosterone * Glucocorticoids e.g. Cortisol * Sex hormones/Androgens Failure of the adrenal cortex leads to a decrease in these hormones. Destruction of 90% of the cortex is required before insufficiency occurs. Causes Destruction by antibodies (autoimmune disorder) is the cause in 70-80% of cases Other causes of Addisons: * TB (previously a common cause but no longer so in Western countries) (eg For anaesthetic-related case report see [1] * Metastases * Bilateral adrenalectomy * Haemorrhage e.g. following meningococcal sepsis Effects of Addisons Disease Skin * Pigmentation due to increased ACTH by anterior pituitary gland. The ACTH cross stimulates Melanocyte Stimulating Hormone (MSH) causing pigmentation Fluid Balance * Hypovolaemia with postural hypotension is typical Acid Base * Metabolic acidosis Electrolytes * Loss of aldosterone results in loss of Na+ reabsorption by the kidney * Hence urinary Na+ is high * Serum Na+ is low * Serum K+ is usually high * Haemoconcentration is often present "Hyponatraemia, hyperkalaemia, hypoglycaemia and haemoconcentration are often present." "...Hyponatraemia, Hypovolaemia, Hypotension, Hyperkalaemia and metabolic acidosis..." Anaesthesia * Volume is a key issue. These patients need intravascular reexpansion * Minimise neuromuscular blockade as hyperkalaemia can cause skeletal muscle weakness

Addison's disease lab features: Which is NOT correct A. Ca 2.50 mmol/L B. BSL 12.0 mmol/L C. Na 128 mmol/L D. K 6.1 mmol/L E. Urea 15mmol/L

ANSWER B Addisons Disease : Primary adrenocortical insufficiency. -insidious, progressive hypofunctioning of the adrenal cortex The adrenal cortex produces a number of hormones * Mineralocorticoids e.g. aldosterone * Glucocorticoids e.g. Cortisol * Sex hormones/Androgens Failure of the adrenal cortex leads to a decrease in these hormones. Destruction of 90% of the cortex is required before insufficiency occurs. Causes -Destruction by antibodies (autoimmune disorder) is the cause in 70-80% of cases -Other causes of Addisons: * TB (previously a common cause but no longer so in Western countries) (eg For anaesthetic-related case report see [1] * Metastases * Bilateral adrenalectomy * Haemorrhage e.g. following meningococcal sepsis Effects of Addisons Disease Skin * Pigmentation due to increased ACTH by anterior pituitary gland. The ACTH cross stimulates Melanocyte Stimulating Hormone (MSH) causing pigmentation Fluid Balance * Hypovolaemia with postural hypotension is typical Acid Base * Metabolic acidosis (low HCO3) Electrolytes * Loss of aldosterone results in loss of Na+ reabsorption by the kidney * Hence urinary Na+ is high * Hyponatraemia * Hyperkalaemia * Haemoconcentration is often present * Hypoglycaemia * high urea Hormonal * Elevated ACTH (>50pg/mL) * Low cortisol (<5ug/dL) Anaesthesia * Volume is a key issue. These patients need intravascular reexpansion * Minimise neuromuscular blockade as hyperkalaemia can cause skeletal muscle weakness

Retrobulbar block is least likely to block which muscle? A. Lateral rectus B. Superior oblique C. Levator palpebrae superioris D. Inferior rectus E. Medial rectus

ANSWER B All nerves except the trochlea nerve (CN IV) pass through the muscle cone. Therefore with retrobulbar blocks, the superior oblique is spared (incomplete akineses).

Epidural analgesia in labour A. typically increases uterine perfusion in healthy women B. can result in lateral rectus muscle palsy if complicated by dural puncture C. is particularly indicated for mothers with aortic stenosis D. may cause hyperglycaemia in the presence of diabetes mellitus E. may be lethal in the presence of maternal mitral stenosis if, following delivery, the block is prolonged

ANSWER B Answer: B (C - epi can be done, but not particularly indicated for) • A. False - "Continuous epidural analgesia with bupivacaine 0.075% increases the resistance of uterine artery and therefore possibly reduces the uterine blood flow" • B. True - We believe that a dural puncture during an attempted epidural anesthetic resulted in cerebrospinal fluid (CSF) leakage with a consequent headache. The CSF leak caused traction on the sixth cranial nerve resulting in lateral rectus muscle palsy. An epidural blood patch performed after the onset of symptoms did not acutely resolve the abducens nerve palsy." o 'The complications of accidental dural puncture include headache, high or total spinal anaesthesia, 6th cranial nerve palsy and subdural haemorrhage. • C. True -traditionally contraindicated but now considered safe - epidural for labour indicated in AS. Epidural, CSE and spinal also been safely used in patients with AS for caesarian section • D. False - Epidural reduces the stress in labour and enables better glucose control, therefore is indicated. • E. ?False - epidural indicated for labour in MS. Epidural and CSE also used for C/S. No evidence found for mortality after prolonged block, no mention early removal.

After a procedure with an LMA in situ a patient complains of loss of sensation to the anterior part of the tongue. What nerve is likely damaged? A. Facial B. Lingual C. Greater palatine D. Glossopharyngeal E. Facial

ANSWER B Anterior 2/3: Somatic by lingual (V3 of trigeminal) Taste by chorda tympani (facial nerve) Posterior 1/3: Somatic and taste glossopharyngeal (IX) Motor hypoglossus (XII)

Post scoliosis repair, decreased movement bilaterally in the legs with decreased pain and temperature sensation but spared joint position sense and vibration. What is at fault? A: Posterior spinal arteries B: Anterior spinal arteries C: Epidural haematoma D: Misplaced pedicle screw E: Lateral cord syndrome

ANSWER B Anterior Spinal Cord Syndrome # Complete motor paralysis below the level of the lesion due to interruption of the corticospinal tract # Loss of pain and temperature sensation at and below the level of the lesion due to interruption of the spinothalamic tract # Retained proprioception and vibratory sensation due to intact dorsal columns Typical causes include acute disc herniation or ischemia from anterior spinal artery occlusion.

AZ02 ANZCA Version [Jul07] A healthy female patient is undergoing a laparoscopic sterilisation under a relaxant based general anaesthetic. Which of the following monitors does NOT have to be in continuous use? A. Capnograph B. Electrocardiogram C. Oximeter D. Oxygen analyser E. Ventilator disconnect alarm

ANSWER B As per ANZCA document PS18: * ECG: correct "must be available for every anaesthetised patient" * Pulse oximeter "A pulse oximeter must be in use for every anesthesied patient" * Breathing system disconnection or ventilation failure alarm: "must be in continuous operation" * Oxygen analyser: "must be in continuous operation for every patient when an anaesthesia delivery system is used" * Carbon dioxide monitor: "must be in use for every patient under GA"

Patient with aortic dissection. Blood pressure 150/90. Best drug to control BP: A. Captopril B. Esmolol C. GTN D. Hydralazine E. SNP

ANSWER B B-blockade has most desired effects -reduce the force of LV ejection (dP/dt) SNP/GTN can be used, but always need to be combined with beta blockers, as a reflex tachycardia and contractility can result in increased sheer damage to the dissection.

Following one-lung ventilation there is an increased risk of lung injury if plateau airway pressure (during one-lung ventilation) exceeds A. 20 cm water B. 30 cm water C. 40 cm water D. 50 cm water E. lung injury is not related to ventilation pressure

ANSWER B BJA CEPD article on OLV states that " tidal volume of 10ml/kg suggested , if airway pressure is excessively high (>30cmH20), the tidal volume should be reduced and respiratory rate increased...permissive hypernapnia to spare barotrauma should be considered

The MAIN indication for biventricular pacing is A. complete heart block B. congestive cardiac failure C. VF D. ? E. ?

ANSWER B Biventricular pacing is defined as a lead in the RV to pace the interventricular septum, and a lead in the coronary sinus which can pace the LV lateral free wall. This is apparently most commonly used in patients with LBBB which can cause dyssynchronous contraction of the LV leading to impaired systolic function. The biventricular pacing "resynchronises" LV contraction and improves systolic function. Indications: (i) severe cardiomyopathy (EF<35%), and (ii) LBBB with NYHA class III or IV symptoms despite maximal medical therapy.

What is NOT a contraindication to MRI? A. Pulmonary artery catheter B. Arterial line C. Scissors D. Coiled ECG cable E. Laryngoscope

ANSWER B Blue Book 2005 "Invasive blood pressure transducers: These are not ferromagnetic, and are safe to use. Transducer cables should be kept out of the magnet bore, so as to avoid imagedistortion" MRIsafety.com "There is at least one report of a cardiovascular catheter (Swan-Ganz Triple Lumen Thermodilution Catheter) that "melted" in a patient undergoing MR imaging. This catheter contained a wire made from a conductive material that was considered to be responsible for this problem. Thus, there are realistic concerns pertaining to the use of similar devices in patients undergoing MR examinations"

PL05 [Aug93] [Mar94] Placental transfer of bupivacaine: A. Lipid solubility B. Low F/M ratio of bupivacaine C. High F/M ratio of bupivacaine metabolites D. Molecular weight ?? E. Detectable levels of bupivacaine & PPX in the neonate for several days

ANSWER B Bupivacaine is 96% protein bound, so less free drug is available for transfer than with lidocaine (67% bound). The mean F/M ratio for bupivacaine is approximately 0.3 and for lignocaine 0.5, confirming lignocaine's greater placental transfer

Comparing two anti-hypertensive agents (or test on a new antihypertensive agent). Study reports 95% confidence interval for the difference between the two drugs is 2 to 10 mmHg. The conclusion is: A. There is no statistically significant difference because the confidence interval is small B. There is a statistically significant difference (p< 0.05) because the confidence interval does not include zero. C. There is a statistically significant difference (p< 0.01) because the confidence interval does not include zero D. There is a clinically significant difference

ANSWER B C would be true for a 99% CI

A six-week-old baby is booked for elective right inguinal hernia repair. An appropriate fasting time is A. 2 hours for formula milk B. 2 hours for clear fluids C. 5 hours for breast and formula milk D. 6 hours for solids E. 6 hours for solids and 3 hours for all fluids

ANSWER B C. is also true, but a 6 week infant would not be on solids

A six-week-old baby is booked for elective right inguinal hernia repair. An appropriate fasting time is A. 2 hours for breast milk B. 4 hours for formula C. 5 hours for breast and formula D. 6 hours for solids E. 8 hours for solids and 4 hours for all fluids

ANSWER B C. is also true, but a 6 week infant would not be on solids College Guidlines: Fasting Fluid : 2 hours Milk: 4 hours Solids: 6 hours

A patient is ventilated and invasively monitored in intensive care following a traumatic brain injury. Systemic arterial blood pressure is 140/80 mmHg with a mean pressure of 100 mmHg. Central venous pressure is 8 mmHg and intracranial pressure is 15 mmHg. The cerebral perfusion pressure (CPP) for this patient is: A. 72 mmHg B. 85 mmHg C. 92 mmHg D. 125 mmHg E. 132 mmHg

ANSWER B CPP = MAP - (ICP or CVP (whichever is greater))

A patient is ventilated and invasively monitored in intensive care following a traumatic brain injury. Systemic arterial blood pressure is 140/80 mmHg with a mean pressure of 100 mmHg. Central venous pressure is 8 mmHg and intracranial pressure is 15 mmHg. The cerebral perfusion pressure (CPP) for this patient is: A. 72 mmHg B. 85 mmHg C. 92 mmHg D. 125 mmHg E. 132 mmHg

ANSWER B CPP = MAP - (ICP or CVP (whichever is greater))

The carotid sinus derives its nerve supply from the A. vagus nerve B. glossopharyngeal nerve C. ansa cervicalis (hypoglossi) D. middle cervical ganglion E. stellate ganglion

ANSWER B Carotid sinus innervation: A. T vagus (CN X) (efferent) B. T glossopharyngeal (CN IX) (afferent) Glossopharyngeal -> Nucleus of the tractus solitarius of the medulla (also true for baroreceptors in aortic arch) carotid sinus : baroreceptor at the origin of the ICA; rich nerve supply from IX (Glossopharyngeal), stimulation à fall in BP carotid body : chemoreceptor : small, oval, reddish brown structure deep to bifurcation of CCA, also innervated by IX

Cerebral palsy is associated with each of the following EXCEPT A. gastro-oesophageal reflux B. increased sensitivity to non-depolarising muscle relaxants C. malnutrition D. recurrent aspiration E. scoliosis

ANSWER B Cerebral Palsy associated with increased sensitivity to depolarising NMBDs and resistance to non-depolarising NMBDs. (Morgan & Mikhail)

Labour epidurals increase maternal and foetal temperature. This results in neonatal: A. Increased sepsis B. Increased investigations for sepsis C. increased non shivering thermogenesis D. Increased need for resuscitation E. Cerebral palsy

ANSWER B Chestnut's Obstetric Anesthesia: Principles and Practice (4th Edn), Chestnut et al. 2009; p457. Labour epidural analgesia is associated with an increase in maternal core body temperature, but also with an increased neonatal temperature and fetal heart rate. Several studies have shown that labour epidural analgesia is associated with increased neonatal neonatal sepsis evaluations, but no increase in neonatal sepsis.

A 12-year-old boy with spastic cerebral palsy and painful muscle spasms presents for multilevel osteotomies of his legs. The most effective option for post-operative analgesia would be A. a combination of NSAIDs (non-steroidal anti-inflammatory drugs) and regular tramadol B. continuous lumbar epidural analgesia with a mixture of local anaesthetic and clonidine C. regular paracetamol and oxycodone D. patient controlled morphine E. spinal morphine

ANSWER B Children undergoing single event multilevel surgery present specific difficulties. Our initial experience was with continuous epidural infusion of 0.125% bupivacaine and fentanyl 2 mg.ml at 0.2 - 0.3 ml.kg.h postoperatively. Analgesia was good, but spasm was frequent, necessitating 4-hourly administration of diazepam. Patients would often have postoperative nausea and vomiting resistant to anti-emetics, resulting in plain bupivacaine 0.125% replacing the combined bupivacaine/fentanyl solution. Subsequently we have been using bupivacaine 0.125% with clonidine 2.5 mg.ml at 0.2 -0.3 ml.kg.h

Complications of mediastinoscopy include all of the following EXCEPT A. air embolism B. cardiac laceration C. pneumothorax D. recurrent laryngeal nerve palsy E. tracheal compression

ANSWER B Complications as follows : DEATH MAJOR complications -Major haemorrhage -Tracheobronchial laceration -Oesophageal perforation -Recurrent nerve paralysis -Phrenic nerve paralysis -Thoracic duct injury -Cerebrovascular accident -Mediastinitis -Venous air embolism -Tumour implantation MINOR complications -Pneumothorax -Superficial wound infection -Recurrent nerve paresis -Minor bleeding -Autonomic reflex braydcardia -Mediastinal lymph node necrosis

Which of the following is not included in the CHADS2 AF thromboembolic risk scoring system A. Age B. Gender C. Diabetes D. Heart failure E. Previous TIA

ANSWER B Congestive heart failure Hypertension Age >75 Diabetes mellitus Stroke/Tia/Thromboembolism To complement the CHADS2 score, by the inclusion of additional 'stroke risk modifier' risk factors, the CHA2DS2-VASc score has been proposed. These additional non-major stroke risk factors include age 65-74, female gender and vascular disease. In the CHA2DS2-VASc score score, 'age 75 and above' also has extra weight, with 2 points. The CHA2DS2-VASc score has been used in the new European Society of Cardiology guidelines for the management of atrial fibrillation. The European Society of Cardiology (ESC) guidelines recommend that if the patient has a CHADS2 score of 2 and above, oral anticoagulation therapy (OAC) such as warfarin (target INR of 2-3) or one of the new OAC drugs (such as dabigatran) should be prescribed.

The cardiac valvular lesion identified as the MOST significant risk factor for non-cardiac surgery is severe A. aortic incompetence B. aortic stenosis C. mitral incompetence D. mitral stenosis E. tricuspid incompetence

ANSWER B Consider that most general anaesthesia results in a decrease in cardiac contractility, which can be detrimental to cardiac output with severe AS. From ACC/AHA Guideline Update on Perioperative Cardiovascular Evaluation for Noncardiac Surgery [1] (http://www.acc.org/clinical/guidelines/perio/clean/perio_index.htm) "Severe aortic stenosis poses the greatest risk for noncardiac surgery. If the aortic stenosis is severe and symptomatic, elective noncardiac surgery should generally be postponed or canceled. Such patients require aortic valve replacement before elective but necessary noncardiac surgery. On the other hand, in patients with severe aortic stenosis who refuse cardiac surgery or are otherwise not candidates for aortic valve replacement, noncardiac surgery can be performed with a mortality risk of approximately 10%. In rare instances, percutaneous balloon aortic valvuloplasty may be justified when the patient is not a candidate for valve replacement."

Hypertensive female at 38 weeks gestation BP 180/110. CTG shows no foetal distress. First Hb 110 and second is 109. First plt count 90 then drops to 40. AST increases from 50 to ? 120. Most appropriate management is a. deliver the baby b. various antihypertensive medication options c. 20mg frusemide d.? e.?

ANSWER B Control BP first then deliver baby. No foetal distress. Delivery is definitve treatment but stabilising patient imperative. Drugs of choice :Antihypertensive drugs that can be safely used include labetalol, nifedipine and hydralazine. The choice should be made on clinician familiarity and experience with a particular agent.

Cryoprecipitate contains each of the following clotting factors EXCEPT A. factor VIII B. factor IX C. factor XIII D. fibrinogen E. von Willebrand factor

ANSWER B Cryoprecipitate is a human blood component obtained from fresh frozen plasma (FFP) prepared from a unit of whole blood (WB). When FFP is thawed in the cold, a cryoprecipitate forms which is rich in fibrinogen, factor VIII, von Willebrand factor, factor XIII, and fibronectin. One unit of cryoprecipitate derived from a unit of whole blood contains: * a volume of 10-20 mL, * 80-100 units of factor VIII (which consists of both the procoagulant activity and the von Willebrand factor), * 150-250 mg of fibrinogen, * 50-100 units of factor XIII, and * 50-60 mg of fibronectin. It can be stored at -18° C for a maximum of one year. When ordered, cryoprecipitate is thawed in a 37° C waterbath and issued in individual bags or as a pooled product. Once thawed it must be kept at room temperature and has an expiration time of 6 hours for unpooled cryoprecipitate, and 4 hours for the pooled product.

Infusion of 1 litre of Hartmanns solution over 30 minutes in a healthy adult results in A. hypercoagulability due to platelet activation B. hypercoagulablity due to a fall in antithrombin III (ATIII) levels C. hypocoagulablilty due to a fall in thrombin levels D. hypocoagulablility due to a fall in platelet levels E. no change in coagulation status

ANSWER B Crystalloid hemodilution was associated with hypercoagulability in in vitro and in vivo studies. Increased thrombin activity due to dilutional effects on AT3.

Which is NOT a disadvantage of drawover vaporiser versus plenum vaporiser? A. Temperature compensation B. Cannot use sevoflurane C. Small volume reservoir D. Flow compensation E.

ANSWER B Drawover vaporisers are * "simple to assemble and use, and can operate without fresh gas supplies. They are lightweight and portable" * not well compensated for temperature o "the OMV...suffers a reduction in vapour output at lower temperatures, with a maximum output varying from 2-4% with halothane between 0-30OC, and higher above this * very difficult to use with sevoflurane" o "Sevoflurane (Sevorane) has been used in draw-over, but its use is hampered by a need to deliver high percentages which are at the upper limits of simple vaporiser performance capabilities...using additional wicks to maximise output can be helpful, but latent heat of vaporisation rapidly cools the system and lowers performance...two OMV vaporisers are required to provide adequate concentrations for induction." * equipped with a small volume reservoir o the OMV...contains 50mls of volatile agent, empties quite rapidly when in use" * variable with regards to flow compensation o for instance..."the OMV...OMV is reasonably accurate over a wide range of flow rates and tidal volumes and, in particular it performs well at small tidal volumes, making it suitable for paediatric anaesthesia" o but..."EMO only begins to perform reasonably accurately with flow rates around 10 l/min, and is therefore not ideal for paediatric use with a T piece, although circuit adaptations can be made. If used in "pushover' fashion, with a ventilator or bellows placed upstream, the output can significantly exceed the dial setting."

Which drug is NOT used in the preoperative management of phaeochromocytoma? A. Phenoxybenzamine B. Aprotinon C. Atenolol D. Alpha-methyl tyrosine E. Prazosin

ANSWER B Drugs used in phaeochromocytoma * α blockade (first) o Phentolamine o Phenoxybenzamine o Prazosin o Doxazocin (doesn't need subsequent β blockade because it doesn't block α2 receptors) * β blockade (only after α blockade established) o Atenolol o Labetolol * α-Methyl-p-Tyrosine - blocks tyrosine hydroxylase * Calcium channel antagonists

The incidence of spinal headache may be reduced by: A. The use of Quincke point spinal needles B. Use of the paramedian approach rather than midline C. Strict bed rest for 24 hours D. Alignment of the bevel of the spinal needle transverse to the long axis of the spine E. .....?? facial n block ??

ANSWER B Entering the dura at an angle (either paramedian, cephalad angulation, use of fine guage needles which bend more in the interspinous ligaments) all decrease risk. Theorie is that it creates a flap valve defect which self-seals.

Contraindications to the use of Ergometrine: A. Vasopressors B. Pre-eclampsia C. Current use of Oxytocics D. Halothane

ANSWER B Ergometrine is contraindicated: * in patients who have previously displayed hypersensitivity or idiosyncratic reactions to ergometrine, other ergot alkaloids or any of the ingredients in the Ergometrine Injection preparation * for the induction of labour and during the first and second stages of labour * if there is any suspicion of retained placenta * in eclampsia or preeclampsia, and in cases of threatened spontaneous abortion. * in severe or persistent sepsis * in patients with peripheral vascular disease or heart disease and in patients with hypertension or a history of hypertension * where impaired hepatic or renal function exists

During laparoscopic surgery, pneumoperitoneum usually results in a fall in cardiac output when intra-abdominal pressure exceeds A. 10 mmHg B. 20 mmHg C. 30 mmHg D. 40 mmHg E. 50 mmHg

ANSWER B FROM CEACCP ARTICLE * IAP < 10 mm Hg: increases VR, increases CO * IAP 10-20 mm Hg decreases VR, decreases CO BUT increases SVR and therefore BP unchanged or increased * IAP > 20 mm Hg greater decrease VR, greater decrease CO, so decreases BP

During laparoscopic surgery, pneumoperitoneum usually results in a fall in systemic blood pressure when intra-abdominal pressure exceeds A. 5 mmHg B. 10 mmHg C. 20 mmHg D. 30 mmHg E. 40 mmHg

ANSWER B FROM CEACCP ARTICLE * IAP < 10 mm Hg: increases VR, increases CO * IAP 10-20 mm Hg decreases VR, decreases CO BUT increases SVR and therefore BP unchanged or increased * IAP > 20 mm Hg greater decrease VR, greater decrease CO, so decreases BP

An 18 yo with Fontan circulation undergoing exploratory laparotomy. On ICU ventilation, saturation is 70%. Which ventilator parameter would you INCREASE to improve his saturation? A. Bilevel pressure B. Expiratory time C. Inspiratory time D. Peak inspiratory pressure E. PEEP

ANSWER B Fontan circulation Main determinants of success -SVR -PVR -AV valve function -sinus rhythm -ventricular function (preload, SVR, contractility)

PP05b [1985] [Aug93] The fluid requirements of a one day old full term neonate would be: A. 5 ml/hr B. 10 ml/hr C. 20 ml/hr D. 30 ml/hr E. 40 ml/hr

ANSWER B Frank Shann's book has * 2ml/kg/h first day * 3ml/kg/h second day * 4ml/kg/h subseqently 3 ml/kg/hr x 3.3 kg = 10 ml/hr

The current required for microelectrocution with a connection onto the endocardium is: A. 1-10 microamps B. 10-100 microamps C. 0.1-1.0 milliamps D. 1-10 milliamps E. 10-100 milliamps

ANSWER B From Miller "10 microA is the max allowable leak, but usually needs to be ~ 50 microA to cause VF" Cardiac-protected: must LEAK < 50 microamps through the cardiac catheter or wire EVEN with a SINGLE FAULT. Need 50 microamps THROUGH THE HEART to produce VF. TOTAL working current limited to <10 mAmp. [Ref for B] Parbrook p 218; RDM para 33615; Rosewarne p 136

Which LMA has highest seal pressure? A. Classic B. Disposable supreme C. Flexible D. Intubating E. Proseal

ANSWER B From the LMA website / product information: * LMA Classic - Seal pressure up to 20cm H20 * LMA Supreme™ - measured oropharyngeal leak pressures up to 37 cm H2O * LMA Flexible™ - oropharyngeal seal pressures up to 20 cm H2O * LMA Fastrach - Seal pressures up to 20 cm H2O * LMA ProSeal™ - leak pressures up to 32 cm H2O

Estimate GCS post head-trauma. E - Response to pain V - Mumbling incoherently M - Withdraws to pain (attempted IV cannulation) A. 7 B. 8 C. 9 D. 10

ANSWER B GCS 8 E2 V2 M4 Best Eye Response. (4) 1. No eye opening. 2. Eye opening to pain. 3. Eye opening to verbal command. 4. Eyes open spontaneously. Best Verbal Response. (5) 1. No verbal response 2. Incomprehensible sounds. 3. Inappropriate words. 4. Confused 5. Orientated Best Motor Response. (6) 1. No motor response. 2. Extension to pain. 3. Flexion to pain. 4. Withdrawal from pain. 5. Localising pain. 6. Obeys Commands.

A 60-year-old diabetic has had a below knee amputation for an ischaemic leg. He has neuropathic pain being managed with oxycodoiie 40 mg bd and paracetamol 1 g qid. He is also on omeprazole 20 mg daily for reflux. You decide to commence gabapentin. Before deciding on a dosage regimen and commencing therapy it is most important that you A. cease his omeprazole B. check his hepatic transaminase level C. check his renal function D. check his QT interval on a resting EGG E. reduce his oxycodone dose

ANSWER B Gabapentin, pharmacology and its use in pain management.' Anaesthesia 2002 57, p 451-462 Not metabolised, no induction or inhibition of hepatic enzymes. Excreted unchanged, renal impairment will decrease clearance. Lack of clinically relevant drug interactions. Well tolerated, few adverse effects. Need dose adjust in renal failure

Conditions which are more commonly associated with exomphalos, compared to gastroschisis include A. amniotic fluid peritonitis B. cardiac abnormalities C. fluid and electrolyte disturbances D. hypothermia E. prematurity

ANSWER B Gastroschisis involves a defect in the anterior abdominal wall (usually on the right) causing herniation of the abdominal contents without a covering sac In exomphalos there is a failure of the gut to return to the abdominal cavity during fetal embryonic development, resulting in persistent herniation through the extra embryonal part of the umbilical cord which covers it. other abdominal organs may be included. there is also an increased incidence of associated abnormalities including cardiac disease. Foetal gastrointestinal abnormalities associated with polyhydramnios are generally those with impaired/absent swallowing - ie. atresias. Exomphalos: failure to return to cavity, associated with other abnormalities particularly cardiac and renal (possibly oligohydramnios). Gastroschisis, herniation, usually to right of umbilicus. Coverings in exomphalos make a difference to fluid/heat loss. Hypovolaemia, hypothermia, infection, adhesions, irritation by amniotic fluid, blood loss during surgery. Abdo compartment syndrome on closure prediced by abdo pressure > 20mmHg or CVP increase of 4mmHg. Reoperation for ischaemic gut, anuria, staged abdo closure. Exomphalos as a central defect of abdo wall is assoc with other midline developmental anomalies - including cardiac. Gastroschesis is thought to arise from vascular disruption of the right omphalmomesenteric artery and is more commonly an isolated defect. Both may be complicated by all the problems of external bowel - infection, fluid and electrolytes, temp, prematurity

Modified Cormack and Lehane grade - You cannot see beyond the epiglottis and there is a little space between the epiglottis and the posterior pharyngeal wall (? remembered as epiglottis touching posterior pharyngeal wall) A. 2a B. 2b C. 3a D. 3b E. 4

ANSWER B Grade I: Complete glottis visible. Grade IIa: Partial view of the glottis is visible Grade IIb: Only arytenoids or very posterior portion of vocal cords are visible. Grade III: Only epiglottis is visible, not the glottis Grade IV:Neither epiglottis nor glottis seen.

You have anaesthetised a patient with a partially obstructing right main bronchus tumour. You are using Heliox (28% 02/72% He) and sevoflurane. You secure the airway with an endotracheal tube and continue with the same gas mixture. After 20 minutes the end-tidal CO2 is 28 mmHg. The capnogram waveform is normal. This probably means the patient Is A. appropriately ventilated with a low cardiac output B. appropriately ventilated with a normal cardiac output C. hypothermic (340 C) D. hyperventilated E. hypoventilated

ANSWER B Helium absorption of IR light interferes with measured ETCO2. Therefore it underestimates.

The estimated risk of infection following percutaneous exposure (needlestick injury) to human immunodeficiency virus (HIV) is approximately A. 1 in 30 B. 1 in 300 C. 1 in 3,000 D. 1 in 30,000 E. 1 in 300,000

ANSWER B Hep C : 30% risk Hep B : 3% risk HIV : 0.3% risk

Histamine release in anaphylaxis does NOT cause: A. Tachycardia B. Myocardial depression C. Coronary artery vasodilatation D. Prolonged PR interval E. Decreased impulse conduction

ANSWER B Histamine is arrythmogenic: * increased PR o ventricular irritability * decreased VF threshold * shifts in pacemaker site * H1 via phospholipase C: o coronary constriction; o bronchoconstriction; o slowing at AV node. o release of prostacyclin * H2 via cAMP: o inotropy; o coronary dilation; o b'dilation; o tachy. o arrythmias o CNS stimulation o increased H+ secretion by parietal cells * Both increase capillary leak. H3 presynaptic.

A 25-year-old man with Marfan's syndrome is scheduled for aortic arch reconstruction under circulatory arrest. Cooling to 18 degrees is planned. The maximum time for circulatory arrest at this temperature after which the risk of long term neurological injury increases markedly is A. 15 minutes B. 25 minutes C. 35 minutes D. 45 minutes E. 55 minutes

ANSWER B Hypothermic cardiac arrest at18°C provided electrocerebral silence is obtained, rewarmed correctly and hemodynamic stablity <30 minutes : safe <40 minutes : safe or transient neurological dysfunction >40 minutes : neurological deficit is prone to occur especially in high risk (elderly, diabetes, hypertension) -further cooling of the brain to 13°C to 15°C reduces the risk again how careful rewarming with close monitoring must occur Oxford states -45 min in adults -60 min in neonates

Salicylate poisoning: A. Respiratory acidosis B. Metabolic acidosis (/ don't think this was an option - ak )(yep i think it was- mm) C. Increased CO2 (production) D. High output renal failure E. Hyperthermia (/ pretty sure this option was HYPOthermia - too late)

ANSWER B In high doses, leads to toxicity a. uncoupling of oxidative phosphorylation (mainly in skeletal muscle) increases oxygen consumption, and carbon dioxide production, thus decreasing PaO2 & increasing PaCO2 b. hyperventilation via direct effect, and increased PaCO2, leads to respiratory alkalosis, with renal compensation c. with even larger doses, - depress respiratory centre, causing further CO2 accumulation, resulting in uncompensated respiratory acidosis - metabolic acidosis with accumulation of pyruvic, lactic and aceto-acetic acid, and absorption of salicylic acid - hyperpyrexia due to increased metabolic rate - initial CNS stimulation, then later coma

Electrocardiographic changes in acute hyperkalaemia include A. the appearance of a J wave B. loss of P waves C. flattening of the T wave D. a prominent U wave E. a prolonged QT interval

ANSWER B In orders of appearance 1. Tall peaked T waves with shortened QT 2. Lengthening of PR and QRS 3. Loss of P wave 3. Lengthening of QRS to Torsades J-wave mainly with hypothermia and hypercalaemia U-wave seen in mainly precordial leads V2-V4. -?early repolarization of the His-Purkinje system -typically less than 0.2mV and is clearly separate from the T wave -Prominent U waves are most often seen in hypokalemia, but may be present in hypercalcemia, thyrotoxicosis, or exposure to digitalis, epinephrine, and Class 1A and 3 antiarrhythmics, as well as in congenital long QT syndrome and in the setting of intracranial hemorrhage.

The most commonly reported cause of awareness during general anaesthesia for a non-obstetric procedure is A. equipment failure B. human error C. lack of premedication D. recreational drug use E. the use of total intravenous anaesthesia

ANSWER B Incidence -1:1000 who receive general anaesthesia (0.1-0.2%) -higher incidence among obstetric (0.4%) and cardiac (1.5%) cases -higher in children (1%) Caused by administering an amount of anesthesia that is inadequate to maintain unconsciousness and prevent recall during surgical stimulation 1. selection of inadequate dose : frequently associated with poor anaesthetic techinique -errors include omission or late commencement of volatile agent, inadequate dosing or failure to recognize the signs of awareness -under dosing due to hypotension : cardiac, emergency surgery or LUSCS -TIVA : higher risk of underdosage, unfamiliarity, no ability to monitor concentrations in blood Resistance of anaesthetic agents -Factors associated with increase in MAC include: pyrexia; hyperthyroidism; obesity; anxiety; young age; tobacco smoking; regular, heavy alcohol use; use of recreational drugs (e.g. opioids, amphetamines, cocaine); chronic use of sedatives (e.g. temazepam); and previous and repeated exposure to anaesthetic agents -Factors associated with a reduction in MAC include: hypocapnia, pregnancy, hypothyroidism, hypothermia, hypotension, increased atmospheric pressure and old age. No effect : Increased atmospheric pressure does not alter brain sensitivity to anaesthetic agents, but increases the inspired and brain partial pressures for given inspired concentration. Equipment Malfunction -empty vaporizer -miscalibration -impurities in volatile agent -disconnection from machine

The most commonly reported cause of awareness during general anaesthesia for a non-obstetric procedure is A. equipment failure B. human error C. lack of premedication D. recreational drug use E. the use of total intravenous anaesthesia

ANSWER B Incidence -1:1000 who receive general anaesthesia (0.1-0.2%) -higher incidence among obstetric (0.4%) and cardiac (1.5%) cases -higher in children (1%) Caused by administering an amount of anesthesia that is inadequate to maintain unconsciousness and prevent recall during surgical stimulation 1. selection of inadequate dose : frequently associated with poor anaesthetic techinique -errors include omission or late commencement of volatile agent, inadequate dosing or failure to recognize the signs of awareness -under dosing due to hypotension : cardiac, emergency surgery or LUSCS -TIVA : higher risk of underdosage, unfamiliarity, no ability to monitor concentrations in blood Resistance of anaesthetic agents -Factors associated with increase in MAC include: pyrexia; hyperthyroidism; obesity; anxiety; young age; tobacco smoking; regular, heavy alcohol use; use of recreational drugs (e.g. opioids, amphetamines, cocaine); chronic use of sedatives (e.g. temazepam); and previous and repeated exposure to anaesthetic agents -Factors associated with a reduction in MAC include: hypocapnia, pregnancy, hypothyroidism, hypothermia, hypotension, increased atmospheric pressure and old age. No effect : Increased atmospheric pressure does not alter brain sensitivity to anaesthetic agents, but increases the inspired and brain partial pressures for given inspired concentration. Equipment Malfunction -empty vaporizer -miscalibration -impurities in volatile agent -disconnection from machine

Which of the following strategies reduces the incidence of severe phantom limb pain? A. NSAIDS B. Ketamine perioperatively C. Regional nerve catheter with local anaesthetic D. morphine PCA E. intravenous lignocaine

ANSWER B Ketamine bolus followed by infusion for 72 hours. Key messages : PREVENTION 1. Perioperative epidural analgesia effective in reducing severe phantom limb pain NNT6 (Level 3) 2. Perioperative ketamine bolus prior to skin incision followed by infusion for 72 hours reduced incidence of severe phantom limb pain -no effective with epidural ketamine 3. Perioperative gabapentin is ineffective in reducing incidence and severity of phantom limb pain 4. Continuous regional blockade via nerve sheath catheters provides effective postoperative analgesia after amputation, but has no preventive effect on phantom limb pain (U) (Level II). TREATMENT 1. Calcitonin intranasal or subcutaneously is effective in treating acute phantom pain but inffective for chronic phantom pain 2. Ketamine provided short-term relief from stump and phantom limb pain 3. Oral slow-release morphine and IV infusions of morphine reduced phantom limb pain 4. Gabapentin is effective in reducing phantom limb pain 5. IV lignocaine reduced stump pain but no effect on phantom pain 6. amitriptyline and tramadol provided good control of stump and phantom pain 7. Sensory discrimination training and motor imagery reduce chronic phantom limb pain. 8. Perioperative epidural analgesia reduces the incidence of severe phantom limb pain (U) (Level III‐2).

Fit lady for elective laparoscopic cholecystectomy seen in PreAdmission Clinic. ECG shows LAD, RSR in V1, wide slurred S in V6 and QRS duration 0.2 msec. Your options a. Refer to cardiology b. Crack on c. Place permanent pacemaker d. Place temporary pacing wires e. Give atropine premedication

ANSWER B LAD and RBBB = bifascicular block. As per AHA/ACC go forth. Unless patient described intermittent syncope, indicating an intermittent trifasicular (complete) heart block.

An anaesthetised patient is ventilated and has standard monitoring plus a central venous line. As the surgeon is commencing the case, the line isolation monitor (LIM) alarms indicating a potential leakage current of greater than 5 milliamps from one of the power circuits in use. The most appropriate action is to A.check the diathermy return plate B.disconnect the central line to electrically isolate the patient till the fault is identified C.make sure the patient is properly "earthed" or "grounded" D.sequentially unplug non-vital equipment from the circuit until the fault is identified E.suspend the operation and move the patient to a safe environment

ANSWER B LIM alarms it warns of the existence of a single fault, but 2 faults are required for a shock to occur. The last piece of equipment that was plugged in is suspect and should be removed from service until it is repaired.

A child weighing 8 kg would be expected to need a Laryngeal Mask Airway of size: A. 0 B. 1 C. 1.5 D. 2 E. 2.5

ANSWER B LMA Size Weight 1 <5kg 1.5 5-10kg 2 10-20kg 2.5 20-30kg 3 >30kg

What is the appropriate LMA size for an 8kg child: A. 1 B. 1.5 C. 2 D. 2.5 E. 3

ANSWER B LMA sizes (Drug Doses Frank Shann Book) <5kg = 1, 5-10kg = 1.5, 10-20kg = 2, 20-30kg = 2.5, 30-50kg = 3.0, 50-70kg = 4.0, 70-100kg = 5.0, >100kg = 6

The plasma half-life of low molecular weight heparin is A. increased in conditions with raised plasma proteins B. 2 to 4 times that of unfractionated heparin C. much less predictable than unfractionated heparin D. dependent upon a saturatable mechanism for clearance E. longer than unfractionated heparin because of a higher affinity for plasma protein

ANSWER B LMW Heparin * Higher bioavailability * Bioavailability is greater. Protein binding is much less compared to unfractionated heparin, giving superior bioavailability at lower doses. * Not fully reversible with protamine unlike unfractionated heparin * Less HITS and less platelet function affect * Longer T 1/2 and more predictable kinetics * clearance by first order kinetics, through the renal route

MR25 [1989] [Sep90] [Mar91] [Mar92] Legionnaire's disease - Which is NOT true? A. Gram negative bacillus as causative organism B. Aminoglycosides effective C. Endocarditis D. Haemoptysis in 20% E. Relative bradycardia F. A rapidly rising fever

ANSWER B Legionella are gram negative bacilli. They are intracellular organisms. Legionella bacteria cause 2 distinct clinical conditions: 1. Legionnaires disease, a severe multisystem disease which includes pneumonia 2. Pontiac fever, a self-limiting flu-like illness without pneumonia The natural environment for legionella is water, and they multiple at temperatures between 25 and 42C, with 35C being their optimal temperature meaning that hot water systems, whirlpools etc are ideal breeding grounds. Many outbreaks have also been associated with air-conditioning systems or with travel, especially cruise ships. Symptoms of Legionnaire disease include * Fever * Dry cough * Dyspneoa * Nausea, vomiting and abdominal pain * Confusion Relative bradycardia may occur in up to 66% of patients. The vital signs may reveal high fever' and tachypnoea. Other pulmonary manifestations include dyspnea, pleuritic chest pain, and hemoptysis, which may be present in as many as one third of cases. Pericarditis and endocarditis may be present. Legionnaire disease is also associated with * Hyponatraemia (SIADH is associated) * Deranged LFTs (very common, may help distringuish Legionnaires disease from other pneumonias) * Thrombocytopeaenia *pleural effusion (up to50%) Diagnosis may be by * Urinary antigen * Sputum culture * Serology Most patients have abnormal CXR at presentation, most commonly patchy basal consolidation. Up to 30% have pleural effusions. Treatment is with antibiotics with a high intracellular concenration, such as azithromycin, clarithromycin, fluroquinolone or rifampacin. Note that Legionnaires disease is a notifiable disease.

A dopamine infusion of 3 mcg/kg/min is likely to A. increased myocardial contractility B. increased renal blood flow C. increased heart rate D. increased systemic vascular resistance E. all of the above

ANSWER B Low dose infusion = renal dose "Dopamine has significant adverse effects that should limit its use. Low-dose dopamine has been clearly shown not to be beneficial. Dopamine is predominantly a beta agonist in doses of 5 to 10 g/kg per minute; at doses exceeding 10 g/kg per minute, dopamine is an alpha agonist and therefore increases arterial resistance. Dopamine increases Pms and, hence, venous return increases, resulting in increased left ventricular filling pressures. The increased afterload at large doses and increased filling pressures associated with dopamine are often undesirable in treating decreased contractility." (Principles of Critical Care)

Man with peripheral vascular disease, post unilateral lumbar sympathectomy injection - most likely Cx: A. orthostatic hypotension B. genitofemoral nerve neuralgia C. ?L2-L4 paraesthesia D. psoas haematoma E.

ANSWER B Lumbar Sympathectomy Anatomy -sympathetic outflow to lower extremities originates from spinal cord segments T10-L3 and conveys to L1-4 ganglia Physiology -abolishing basal and reflex constriction of arterioles and precapillary sphinters -flow increase 20-200% has been observed -maximal vasodilation noted immediately after, tapers within 5-7days, resting vasomotor tone returns after 6 months (due to incomplete denervation, cross over, hyper reactivity to circulating catecholamines -altered pain transmission, decreases noxious stimulus Indications -inoperable arterial occlusive disease -urogential pain -renal colic -complex regional pain syndromes -frostbite -stump pain, phantom limb pain Contraindications Patients on anticoagulant therapy Hemorrhagic disorder Allergies to medications injected Local infection Local neoplasm Local vascular anomalies Method Equipment : Sterile sutff, long needle, XR image intensifier, La or phenol Positioning : Prone or Lateral Technique : 1. needle inserted 10-12cm from middle aiming needle at L2 vertebral body 2. when needle strikes bone, walk need off vertebral body anteriorly to slide of antero-lateral surface of the vertebral body 3. contrast injection should be easy after aspiration to exclude intravascular placement, spread up and down vertebral column should be demonstrated (between vertebral body and psoas muscle) 4. 15-20 injected Complications •Genitofemoral Neuralgia occurs in 5% of all blocks. This causes pain in the L1 groin area and is thought to be due to bruising of the L1 nerve root by the needle passing by it. More than 90% of cases recover spontaneously after 6 weeks. Treatment with amitriptyline and gabapentin / pregabalin can help greatly. *Bleeding due to aorta and inferior vena cava injury by the needle. •Intravascular injection (should be prevented by checking the needle position with radio-opaque dye). •Upper abdominal organ puncture with abscess / cyst formation. *Paraplegia from injecting phenol into the arteries that supply the spinal cord (should be prevented by checking the needle position with radio-opaque dye).

In a patient with aortic stenosis, A. aortic regurgitation is rarely seen B. loss of sinus rhythm is poorly tolerated C. presentation is usually with a cerebrovascular accident D. the characteristic compensation of the left ventricle is dilatation E. the most common aetiology is rheumatic calcification

ANSWER B Maintenance of sinus rhythm and adequate intravascular volume is vital to ensure ventricular filling. Arrhythmias must be treated promptly. New onset atrial fibrillation may require cardioversion Degenerative calcific aortic stenosis - This is now the most common form of aortic stenosis in the UK and tends to occur over the age of 70 yr. Mechanical stress over time leads to progressive fibrosis and calcification of a previously normal tri-leaflet valve Anaesthetic techniques that reduce systemic vascular resistance (e.g. regional neuroaxial techniques) must be used with extreme caution although successful cases of carefully titrated epidural and spinal blocks using catheters have been reported

Acromegaly due to excess of growth hormone. Why is it difficult to do a direct laryngoscopy? A: Distorted facial anatomy B: Macroglossia C: Glottic stenosis D: Prognathe mandible E: Arthritis of the neck

ANSWER B Many typical acromegalic features are suggested to cause a difficult airway in these patients. The most discussed changes are: *macroglossia, *prognathism, *enlargement and distortion of glottic structures with additional folds, and * hypertrophy of laryngeal and pharyngeal soft tissue.

In a patient undergoing a femoro-popliteal bypass, the most predictive independent risk factor for the development of post-operative myocardial infarction would be A. an acute myocardial infarct 3 months ago B. an episode of intra-operative myocardial ischaemia C. an episode of post-operative myocardial ischaemia D. 50% blood volume blood loss intra-operatively E. poorly controlled diabetes mellitus

ANSWER B Most (>80%) Perioperative MIs occur early after surgery, are asymptomatic, of the non-Q-wave type (60-100%), and are most commonly preceded by ST-segment depression rather than ST-segment elevation

During surgery for tracheostomy insertion surgical diathermy is being used at the tracheal incision. You are ventilating with 100% oxygen. As the trachea is opened you notice a blue flame shooting up from the incision. Your first action should be to A. disconnect the breathing circuit from the endotracheal tube B. douse the wound with saline C. insert a tracheostomy tube D. remove the endotracheal tube E. turn off oxygen and ventilate with air

ANSWER B Muchatuta N & Sale S - Fires & explosions Management of airway fire • Disconnect the breathing circuit • Flood oropharynx with sterile water or saline • Consider flushing saline down tracheal tube to extinguish intraluminal fire • Consider removing tracheal tube (potential source of further thermal injury or toxic products) • Re-intubate and ventilate (airway swelling may make intubation difficult) • Perform bronchoscopy to inspect for damage and remove foreign bodies (e.g. tube debris) • Consider steroids and antibiotics • T ransfer to critical care if patient at risk of upper airway swelling or acute lung injury The following anaesthetic and surgical recommendations are suggested. * Anaesthetic 1. Use the minimum concentration of oxygen to maintain arterial saturation. 2. Use other non-oxidizing agents such as helium, air or nitrogen in place of nitrous oxide. 3. Position the ETT near the carina to minimize exposure of the cuff to injury in opening the trachea. 4. A good seal on the cuff is essential to prevent leaking of anaesthetic gases. 5. Fill the cuff with water to act as a fire retardant. 6. Consider using fire resistant ETT (silicone/metallic) often used in laser surgery. * Surgical 1. Minimize use of diathermy particularly once the trachea is opened. 2. Use bipolar diathermy if bleeding occurs once the trachea is opened. 3. Use suction to remove oxygen rapidly from the field. 4. Surround the operating field with moist pack if diathermy has to be used. 5. Always have sterile saline at hand in case of fire.

The anterior branch of the femoral nerve supplies everything but: A: pectinius B: rectus femoris C: Medial thigh D: anterior thigh E: sartorius

ANSWER B Muscular Branches Anterior : Pectineus and Sartorius Posterior : Quadriceps femoris (4 Parts : Rectus Femoris, Vastus Lateralis, Vastus Medialis, Vastus Intermedius) Cutaneous Branches Anterior : * Intermediate cutaneous nerve of thigh * Medial cutaneous nerve of the thigh * Lateral cutaneous nerve Posterior : * terminal branch of saphenous nerve Articular Branches to knee joint

Which of the following is NOT a feature of primary myxoedema? A. Bradycardia B. Pretibial myxoedema C. Macrocytic anaemia D. Flattened ECG complexes

ANSWER B Myxoedema = Severe Hypothyroidism -rare form of decompensated hypothroidism -hypothyroidism in 2% women and 0.2% men -myxedema in 0.1% of all cases of hypothyroidism -mortality 15-20% -typically seen in elderly women -precipitated by infection, medication, environmental exposure or metabolic-related stresses ECG changes -bradycardia -prolonged QT -low P, T, QRS amplitude -complete heart block -BBB Treatment ABC approach 1. rehydrate with IV glocose and saline 2. inotropes and vasopressors if required 3. warming 4. IV liothronine 5. Hydrocortosone (100mg TDS) 6. ICU

EM31 ANZCA version [2001-Apr] Q39, [2001-Aug] Q38, [2003-Apr] Q40, [2003-Aug] Q45, [Mar06]Q37 The end-tidal CO2 partial pressure (PetCO2) may be greater than the arterial CO2 partial pressure (PaCO2). Causes of this do NOT include A. transitory variations in V/Q matching and deadspace B. breathing with small tidal volumes C. PaCO2 fluctuations during the respiratory cycle D. exercise E. slow emptying of long time constant alveoli containing CO2 levels approximating mixed venous CO2 levels

ANSWER B NEAGATIVE Pa-ET CO2 Where can they occur? * Healthy subjects during low frequency high tidal volume ventilation * Pregnant subjects * Infants and Children * After coming off cardiac bypass * During and after exercise. What are the reasons for negative values? * Experimental errors * Rebreathing * Inadvertent addition of CO2 to the inspired gases * Physiological reasons * Smaller alveolar dead space and inherent upward slope of phase III * Increase in the slope of phase III * Exaggerated alveolar PCO2 fluctuations during respiratory cycle due to increased CO2 production and decreased FRC which make it more likely to sample higher alveolar PCO 2 (sampled as PETCO2 ) during expiration greater than mean PaCO2 * Occurrence of phase IV

Neurofibromatosis is associated withall EXCEPT A. Phaeochromocytoma B. Diabetes mellitus C. Spinal nerve root compression D. Pulmonary fibrosis / (or Interstitial lung disease) E. Macrocephaly

ANSWER B NEUROFIBROMATOSIS Group of hereditary diseases characterized by tendency to form tumors of ectodermal and mesodermal tissues. AETIOLOGY • incidence 1 in 3000, prevalence of 1 in 5000 • autosomal dominant, 100% penetrance, defect of tumor suppressor gene (Chromosome 17) • 50% sporadic mutation CLINICAL FEATURES * Two or more neurofibromas on or under the skin, or one plexiform neurofibroma (a large cluster of tumors involving multiple nerves); neurofibromas are the subcutaneous bumps characteristic of the disease, and increase in number with age. * Freckling of the groin or the axilla (arm pit). * Café au lait spots (pigmented, light brown macules located on nerves, with smooth edged * Skeletal abnormalities, such as sphenoid dysplasia or thinning of the cortex of the long bones of the body * Lisch nodules (hamartomas of iris), freckling in the iris * Tumors on the optic nerve, also known as an optic glioma * Macrocephaly in 30-50% of the pediatric population without any hydrocephalus * Epilepsy * Juvenile posterior lenticular opacity ANAESTHETIC CONSIDERATIONS • Airway : - Neurofibroma of tongue, pharynx or larynx may interfere with tracheal intubation - Suspicion raised by history of dysphagia, dysarthria, stridor or change of voice • Respiratory system - Intrapulmonary neurofibroma, pulmonary fibrosis may produce cough and dyspnoea - RV failure - Scoliosis, kyphosis • Cardiovascular system - Hypertension (consider pheochromcytoma or renal artery stenosis) - HOCM - Mediastinal tumors leading to SVC obstruction • Central nervous system - Cerebral and spinal neurofibromas are common - Increased incidence of epilepsy and learning disorders - CVA - Seizures • Gastrointestinal tract - Intestinal tumors may present with pain, GI haemorrhage or perforation - Carcinoid tumors/syndrome • Genitourinary system - Neurofibromas may cause ureteric/urethral obstruction • Musculoskeletal - Vertebral deformities or spinal cord tumors may make spinal/epidural techniques difficult

Numbers needed to treat is the inverse of: A. Difference of the Variance B. reduction of absolute risk C. absolute decrease in relative risk D. relative risk E. odds ratio

ANSWER B NNT = 1/ARR ARR = CER - EER

Numbers needed to treat is the inverse of: A. ? B. reduction of absolute risk C. absolute decrease in relative risk D. relative risk E. odds ratio

ANSWER B NNT = 1/Absolute Risk Reduction

Maternal cardiac arrest. In making the diagnosis of amniotic fluid embolism, large amount of PMNs surrounding foetal squamous cells are A. Pathonomonic B. Supportive C. Only found at postmortem D. Irrelevant E. Incidental

ANSWER B No diagnostic test for AFE, it is diagnosis of exclusion. Non specific tests FBE : anaemia, thrombocytosis Coag : consumptive coagulopathy ABG : hypoxemia, low Pa02 to FiO2 ratio CXR : early ARDS ECG : strain, arrythmias Diagnostic test -cytological analysis of central venous blood and broncho-alveolar fluid -Sialyl tn antigen test -zinc coproporphyrin -serum tryptase levels all are non specific and only suggestive of AFE

The patient most likely to desaturate significantly during one lung anaesthesia is one who is having A. a left sided thoracotomy, has reasonable PaO2 values during two-lung ventilation, but poor pre-operative spirometry B. a right-sided thoracotomy, has relatively poor PaO2 values during two-lung ventilation but good pre-operative spirometry C. a left-sided thoracotomy and has diminished perfusion but not ventilation to the operative lung on a V/Q scan D. a right-sided thoracotomy, has relatively poor PaO2 values during two-lung ventilation and poor pre-operative spirometry

ANSWER B OLV used for almost all thoracic operations : lung, esophageal, aortic or mediastinal surgery, to improve surgical access. Although one lung is ventilated, both lung are perfused. Resulting in -trans pulmonary shunting -impairment of oxygenation -hypoxemia Prediction of Hypoxemia during OLV 1. Side of operation -size R>L -therefore operation on left lung (ei larger right lung is dependent and ventilated) results in better oxygenation -FiO2 1, results in 280mmHg for left sided operation and 170mmHg for right sided operations 2. Lung Function Abnormalities a. airway obstruction with low FEV1 results in better oxygenation during OLV, perhaps due to autoPEEP resulting in reduced atelecatasis and improving oxygenation b. low PaO2 on pre-operative ABG is a predictor for hypoxemia during OLV. 3. Distribution of perfusion - less perfusion of non ventilated lung and more to ventilated lung results in high PaO2 during OLV a. large tumors in non ventilated lung = better PaO2 as they present for lobectomies or pneumonectomies b. gravity, if ventilated lung is dependent = better PaO2; in one study in COAD pts, FiO2 1.0 resulted in PaO2 300mmHg in supine versus PaO2 490mmHg in lateral

Effects of volatile anaesthetic agents on the brain include A. maintenance of cerebral blood flow when used with hypocapnia B. uncoupling of autoregulation, with a consequent rise in intracranial pressure C. reduction of cardiac output and cerebral blood volume when used at concentrations of 1.3 MAC D. maintenance of cerebral metabolic rate, but reduction of cerebral electrical activity E. equal depression of all neurons of the brain at 1.3 MAC concentration

ANSWER B Option A - Incorrect. Cerebral blood flow is increased Option B - Correct. Autoregulation is uncoupled with volatiles, leading to a rise in ICP despite lowering cerebral metabolic rate. Option C - Incorrect. Cerebral blood volume icnreases. Option D - Incorrect. Cerebral metabolic rate decreases. Option E - Incorrect. Not all volatiles depress equally e.g. enflurane is capable of producing excitation and seizures.

Osteogenesis imperfecta is associated with A. blindness B. cleft palate C. hypothermia D. mitral stenosis E. thrombocytopaenia

ANSWER B Osteogenesis imperfecta is a autosomal recessive disorder of collagen Presents with a triad of * Multiple bone fractures with easy fracturing of bone * Easy bruising * Deafness Clinical signs * Delayed bone age * Mutiple fractures * Phosph./calcium metabolism abn. * Restricted joint mobility * Teeth anomalies * Wormian bones * Blue sclerae * Cleft soft palate/bifid uvula * Lordosis * Scoliosis * Tendency for hyperthermia * AV and MV insufficency * Qualitative platelet defect

In performing an awake fibreoptic intubation it is MOST important that care is taken to avoid: a. Causing any bleeding that will obstruct view b. Oversedation as leads to posterior pharyngeal wall collapse c. Trauma to nasal turbinates d. Touching vocal cords as will induce coughing e. Oral route as may bite the fibreoptic scope

ANSWER B Oversedation results in posterior wall collapse and reduction in cough (which maybe a problem if bleeding)

To improve oxygenation in a patient intubated and ventilated for a laparotomy you adjust the ventilator settings to apply 10 cm H2O of PEEP (positive end-expiratory pressure). The patient's blood pressure falls from 130/80 to 90/50 mmHg. The addition of PEEP may result in a fall in blood pressure because PEEP causes A. decreased myocardial contractility B. decreased venous return C. increased left ventricular afterload D. increased left ventricular compliance E. increased right ventricular afterload

ANSWER B PEEP may reduce cardiac output by reducing venous return, by increasing pulmonary vascular resistance, and by shifting the interventricular septum to the left, thus reducing the left ventricular end diastolic volume.

MR21 A superior sulcus tumour (Pancoast tumour) is usually: A. Metastatic B. Bronchogenic C. Frontal lobe of brain D. Osteosarcoma of upper rib E. Tumour involving lymph nodes of neck

ANSWER B Pancoast Syndrome - also called a pulmonary sulcus tumor or superior sulcus tumor, is a tumor of the pulmonary apex Clinical Presentation 1. Arm pain 2. Horner's Syndrome 3. Weakness/atropy of hand muscles Pathology -mostly non-small cell lung cancer (squamous cell) -small cell ca 5% Treatment is dependent on malignant cause.

TURP - patient under spinal. Confused. ABG: Na+ 117 / normal gas exchange. Treatment ? A. 10 ml 20% Saline as fast push IV B. 3% NS 100 ml/h C. Normal saline 200 ml/h D. Frusemide 40 mg IV E. Fluid restrict 500 ml/day

ANSWER B Patient is symptomatic -3% Saline at 1-2ml/kg/hr (stop as soon as symptoms resolve) Correction should be no more than 2 mmol/L per hour initially for 3 to 4 hours, then about 1 mmol/L per hour afterwards. In 24 hours, correction should be no more than 12 mmol/L. -use frusemide -fluid restriction

When compared with intra-muscular or subcutaneous opioid regimens, patient controlled analgesia (PCA} with opioids A. is equally preferred by patients B. provides better analgesia C. results in less opioid-related adverse effects D. results in lower opioid consumption E. results in shorter hospital stay

ANSWER B Patient-controlled analgesia 1. Intravenous opioid PCA provides better analgesia than conventional parenteral opioid regimens (S) (Level I [Cochrane review]). 2. Opioid administration by intravenous PCA leads to higher opioid consumption (R), a higher incidence of pruritus (R), and no difference in other opioid-related adverse effects (S) or hospital stay (S) compared with traditional methods of intermittent parenteral opioid administration (Level I [Cochrane review]). 3. In settings where there are high nurse-patient ratios there may be no difference in effectiveness of PCA and conventional parenteral opioid regimens (N) (Level I). 4. Patient preference for intravenous PCA is higher when compared with conventional regimens (U) (Level I). 5. The addition of ketamine to PCA morphine does not improve analgesia or reduce the incidence of opioid-related side effects (U) (Level I). 6. Iontophoretic fentanyl PCA may not be as effective as intravenous morphine PCA, with more patients withdrawing from studies because of inadequate pain relief (Level I). 7. There is little evidence that one opioid via PCA is superior to another with regards to analgesic or adverse effects in general; although on an individual patient basis, one opioid may be better tolerated than another (U) (Level II). 8. There is no analgesic benefit in adding naloxone to the PCA morphine solution; however in ultra-low doses the incidence of nausea and pruritus may be decreased (U) (Level II). 9. The addition of a background infusion to intravenous PCA does not improve pain relief or sleep, or reduce the number of PCA demands (U) (Level II). 10. Subcutaneous PCA opioids can be as effective as intravenous PCA (U) (Level II). 11. Intranasal PCA opioids can be as effective as intravenous PCA (U) (Level II). 12. The risk of respiratory depression with PCA is increased when a background infusion is used (U) (Level IV).

All of the following should be irradiated before they are given to an immunocompromised patient EXCEPT: A. Whole Blood B. Peripheral stem cell reinfusions C. Granulocyte transfusions D. Apheresis platelets E. Leukocyte-reduced RBCs

ANSWER B Patients who are immunocompromised are at risk for Transfusion Associated Graft vs. Host Disease (TA-GVHD). Any product that contains viable lymphocytes should be irradiated prior to transfusion to a patient at risk. This would include all of the choices listed above, but stem cell reinfusions shouldn't be irradiated. Granulocytes CAN be irradiated, because they still function OK following irradiation, and the incompatible lymphs can attack host tissues and cause TA-GVHD. Granulocytes CANNOT be filtered .

Features of anorexia nervosa include each of the following EXCEPT A. dental caries B. increased gastric emptying C. mitral valve prolapse D. sinus bradycardia E. superficial parotitis

ANSWER B Perioperative Considerations for Anorexia Nervosa Cardiovascular -hypotension : resulting from dehydration -bradycardic : decreased basal metabolic rate -ECG : AV block, ST depression, T wave inversion and QT prolongation; >80% will have an ECG abnormality -QT prolongation maybe caused by hypocalcaemia, hypomagnaesemia, drugs or starvation -hypophosphataemia reduces arrythmia threshold -Other arrhythmias : sinus arrest, wandering atrial pacemakers, nodal escape beats, SVT or VT; Reported incidence of 16-62% under anaesthesia. -reduced myocardial contractility : due to malnutrition, also ipecac is directly cardiomytoxic, avoid fluid overload -mitral valve prolapse Respiratory -metabolic alkalosis from self induced vomiting and laxatives produces compensatory bradypnoea -pneumothorax -aspiration pneumonia -pneumomediastinum Renal -hyponatraemia, hypokalaemia, hypochloraemia, hypomagnesaemia, hypocalaemia metabolic alkolosis -renal calculi from chronic ingestion of magnesium hydroxide

Recognised factors that predispose upper limb nerves to compression, under anaesthesia include A. forearm extension and supination B. extreme flexion of elbows across chest C. internal rotation of abducted arm D. lateral position - uppermost arm flexed in arm support E. prone position - arms by side and fingers flexed

ANSWER B Position (ANAESTHETIC FACTORS) 1. internal rotation (pronation) with arm abducted >90 degrees 2. pronation when by side (should be neutral) 3. compression or stretch 4. Elbow flexion greater than 90 degrees increases risk.

LUSCS for failure to progress. Spinal is inserted uneventfully. Next day the patient has foot drop. The most likely cause is? A. epidural haematoma B. lumbosacral palsy C. sciatic nerve palsy D. common peroneal palsy E. ?

ANSWER B Postpartum foot drop is caused by damage to the lumbosacral trunk or, less frequently, the common peroneal nerve. Risk factors include 1. prolonged labor 2. forceps 3. lithotomy 4. LUSCS Common peroneal nerve damage may occur due to improper or prolonged positioning during lithotomy and the sensory deficit may be limited to the dorsum of the foot

During interpretation of a report of a clinical trial the: A. Most important factor in determining the statistical power of the study is the size of the expected difference between the groups being compared B. Statistical power of the study is the probability of not making a type II error C. 99% confidence interval for the population mean is given by the sample mean +/- 1.96 x standard error of the mean D. Designs least subject to bias are cohort and case control studies E. Validity of the results is questionable if serial comparisons were made between the two samples using ANOVA (analysis of variance).

ANSWER B Power = 1 - β where β = probability of type II error

Why does a proseal LMA provide a better airway seal? A. More stable position due to oesophageal tube B. Doral cuff pushes ventral cuff...? C. Presence of oesophageal lumen D. Higher cuff pressure E. ?

ANSWER B Proseal LMA has a 2nd dorsal cuff, which pushes the LMA anteriorly to provide a better seal at the glottic apeture. It also helps anchor the LMA in place, and the Ventral cuff is larger than the classic LMA to help improve the seal.

Mitral valve replacement for Mitral stenosis. Pulmonary artery catheter in situ. Following separation from bypass, copious frank haemoptysis via ETT. Next step in management" A) Insert double lumen tube B) Go back on bypass C) Give protamine D) Deflate and pull back PAC E) Perform fibreoptic bronchoscopy

ANSWER B Pulmonary artery rupture as a consequence of PAC. In this case going back on bypass to circumvent the lungs. Non surgical -Immedate treatment involves deflating the PAC ballon, withdrawing 1-2 cm and then reinflating to tamponade the bleed -A double lumen tube should be inserted for isolation and ventilation of the good lung. -Bronchoscopy and insertion of Fogerty cather tamponade -Injection of autologous blood clot through catheter to seal puncture Surgical options -thoracotomy with lobectomy, pneumonectomy, hilar clamping with direct arterial repair -haemothroax is a strong indication for surgical repair Risk factors for rupture ->60 years old -pulmonary hypertension -improper balloon inflation -improper catheter positioning -cardiopulmonary bypass -anticoagulation

Leukocyte reduction is indicated for prevention of all of the following EXCEPT: A. Febrile nonhemolytic transfusion reactions B. Transfusion-associated Graft vs Host Disease C. HLA alloimmunization D. Transmission of Cytomegalovirus (CMV) E. All of the above are prevented by leukocyte reduction

ANSWER B Reduction of the transfused white blood cell load is well accepted for prevention of recipient antibody formation vs. donor HLA antigens, and works well for prevention of febrile reactions. In recent years, prevention of CMV transmission has become a widely accepted use for leukocyte reduction, as well (though there remains some controversy about this). However, TA-GVHD is a brutal, usually fatal complication of blood transfusion that may NOT be prevented by leukocyte reduction. The big problem is that no one really knows the minimum white cell load required to cause this horrific complication. So, most everyone feels that it is better to just deactivate the transfused lymphs in cases where a patient is at risk. This is done with irradiation.

SG53 During laparoscopic surgery, pneumoperitoneum usually results in a fall in cardiac output when intra-abdominal pressure exceeds A. 10 mmHg.... B. 20 mmHg C. 30 mmHg D. 40 mmHg E. 50 mmHg

ANSWER B Reference CEACCP 2004, V4,107: IAP < 10 mm Hg: increases VR, increases CO IAP 10-20 mm Hg decreases VR, decreases CO BUT increases SVR and therefore BP unchanged or increased IAP > 20 mm Hg greater decrease VR, greater decrease CO, so decreases BP

A patient undergoing liver surgery has a venous air embolism, what is the most appropriate position to place them in: A. Reverse trendelenburg, right side up B. Reverse trendelenburg left side up C. Reverse trendelenburg, neutral D. Trendelenburg right side up e. Trendeleburg left side up

ANSWER B Relative risk of air embolism in hepatic surgery if low (<5%) Factors predisposing include -Surgical technique: inflow vascular occlusion techniques VAE occurs during parenchymal transection, reperfusion or mobilization of air bubbles trapped in open veins -Size and place of tumor: right lobe, close to IVC -Blood loss: >5L -Low CVP anaesthesia Aim is to prevent further air entry, and to overcome RV mechanical obstruction. To prevent further air entry: -Position surgical site below level of RA -IV fluid bolus -Increase ITP eg Valsalva Overcoming obstruction from RV to RVOT; -Left lateral decubitus and Trendelenburg -Inotropic support -CP bypass To come back to question, 1st maneuvers should be to minimize air entry, then if this period is over and HD instability, steps to decrease obstruction is important. http://www.hindawi.com/journals/hpb/2012/720754/

What splitting ratio gives a 3% concentration of isoflurane A. 1/3 B. 1/9 C. 1/13 D. 1/20 E. 1/27

ANSWER B SVP 238mmHg +31% at saturation

If a test is negative, what proportion will not have the disease: A. Sensitivity B. Specificity C. Positive Predictive Value D. Negative Predisctive Value

ANSWER B Sensitivity and specificity are a function of a test, but positive and negative predictive values change depending on prevelance. * Sensitivity = TP/(TP+FN), "proportion of positive cases correctly identified" * Specificity = TN/(TN+FP), "proportion of negative cases correctly identified" * PPV = TP/(TP+FP) * NPV = TN/(TN+FN)

What raises intra-ocular pressure (IOP)? A. metabolic acidosis B. respiratory acidosis C. miosis D. reverse trendelenberg (head up) E. carbonic anhydrase inhibitor

ANSWER B Similar to the factors which increase ICP.

The COMMONEST organism causing meningitis post spinal is: A. Staph epidermidis B. Strep salivarius C. Staph aureus D. Strep pneumoniae E. ?

ANSWER B Sources of bacterial introduction 1. intrinsic or extrinsic contamination of needles, syringes, or injected medications 2. inadequately decontaminated patient skin 3. inadequately cleaned health-care provider hands 4. a contaminated sterile field 5. droplet transmission from the health-care provider's upper airway Most common organisms identified 1. Strep Salivarious 50-60% 2. Haemophilus influenzae, 3. Neisseria meningitides 4. Streptococcus pneumoniae.

Post scoliosis repair, decreased movement bilaterally in the legs with decreased pain and temperature sensation but spared joint position sense and vibration. What is at fault? A: Posterior spinal arteries B: Anterior spinal arteries C: Epidural haematoma D: Misplaced pedicle screw E: Lateral cord syndrome

ANSWER B Spinal injuries may, by damaging the radicular arteries, seriously compromise the function of the spinal cord and cause weakness and paralysis of muscles. The areas that are most vulnerable to deprivation of blood supply are T1-3, T5 and L1. Blockage of a posterior spinal artery may have little effect owing to the extensive anastomoses Anterior spinal artery often produces ischaemia of the anterior central part of the cord causing flaccid paralysis and loss of pain and temperature sensation. Thrombosis of the great radicular artery of Adamkiewicz may produce a paraplegia because it makes a major contribution to the blood supply of the lower two-thirds of the spinal cord.

What transfusion related complication is the commonest cause of mortality A. Bacterial infection B. TRALI C. ABO incompatibility D. Hyperkalemia E. Graft versus host

ANSWER B TRALI CEACCP Transfusion-related acute lung injury is the most common cause of major morbidity and death after transfusion.

Transfusion related acute lung injury (TRALI) occurs A. almost immediately B. within 4 hours C. within 8 hours D. within 24 hours E. within 48 hours

ANSWER B TRALI is defined as acute onset of hypoxia and bilateral pulmonary infiltrates that is temporally related to a blood transfusion, it must occur within 6 hours following transfusion.

TRALI more likely if blood donor has been A. diabetic? B. multiparous woman C. male over 50 years old D. stayed in UK for few yrs

ANSWER B TRALI is defined as acute onset of hypoxia and bilateral pulmonary infiltrates that is temporally related to a blood transfusion, it must occur within 6 hours following transfusion. Typically associated with platelets and FFP. Etiology is not currently understood, but it is thought to be immune mediated. TRALI is more likily when blood donor is 1. Multiparous women : exposure to fetal blood 2. Previous transfusions 3. Previous transplantation Reciept RF include 1. Duration of cardiopulmonary bypass (cardiac surgery) 2. older age 3. repeated FFP/Platelets

During interscalene block placement get medial movement of the scapula. This is secondary to stimulation of: A: long thoracic nerve B: dorsal scapula nerve C: suprascapular nerve D: supraclavicular nerve E: accessory nerve

ANSWER B TROUBLE SHOOTING Needle contacts bone at 10cm depth, no twitches -needle stopped by transverse process -insertion point is too posterior -reinsert needle more anteriorly Twitches the diahragm -stimulation of phrenic nerve -needle inserted anteriorly -reinsert more posteriorly Arterial blood -carotid artery puncture -angled too anterior -withdraw and apply pressure -redirect posteriorly Twitch of scapula -direct stim of serratus anterior muscle -direct stim of thoracodorsal nerve (supplied lat dorsi) -direct stim of dorsal scapular nerve (supplies rhomboid muscles and levator scapulae) -re-direct needle anteriorly

The weight of a child can be estimated using the formula A. (age + 2) x 3 B. (age + 4) x 2 C. (age x 2) + 4 D. (age x 3) + 2 E. age x 4

ANSWER B The "standard" anaesthetic formula are: * From age 1-8: wt= (age in years x 2) + 8 [which is equal to (age+4)x2] * From age 8-12 wt = (age in years) x 3 For practical use though, a simple easy-to-remember guide that I made up is the "1-6-10 rule": * At age 1 yrs -> 10kgs * At age 6 yrs -> 20kgs * At age 10yrs -> 30kgs And for the in-between years: * From 1->6, add 2kgs/yr * From 6->10, add 2.5kgs/yr Another rule of thumb for >1yr old is (2x Age)+9 until 9yrs old, and 3x Age thereafter.

PP87 [Jul06] Which is an estimate of the weight (in kgs) of a child? (where age is in years) A. (age+4)x2 B. (agex2)+4 C. (age+2)x3 D. (age+3)x2 E. ?

ANSWER B The "standard" anaesthetic formula are: From age 1-8: wt= (age x 2) + 8 From age 8-12 wt = (age) x 3 For practical use though, a simple easy-to-remember guide that I made up is the "1-6-10 rule": * At age 1 yrs -> 10kgs * At age 6 yrs -> 20kgs * At age 10yrs -> 30kgs And for the in-between years: * From 1->6, add 2kgs/yr * From 6->10, add 2.5kgs/yr Another rule of thumb for >1yr old is (2x Age)+9 until 9yrs old, and 3x Age thereafter.

At what age in children is the Hb level at its lowest? A. Neonate B. 3 months C. 6 months D. 12 months E. 24 months

ANSWER B The Children's Hospital Westmead Handbook says lowest at 3-12 months. The Children's Hospital notes emphasise that 3 months is the lowest value. * The nadir is at 3 months where the Hb is 10g/dl.

Desflurane TEC6 vaporiser, unable to turn dial on. This is NOT because A. vapouriser is tilted B. Hotter than 39C C. On battery power D. Interlock not engaged, or not seated properly (or something like that) E. other vapouriser is already on

ANSWER B The TEC 6 desflurane vaporizer needs to warm up and the "operational" LED light needs to be illuminated before you can turn the dial to the ON position. If you try to turn it on before the "operational" light is on it will not work. According to Graham (BJA 1994; 72:470-73), causes of vaporizer shutdown include: o a tilt of about 10 degrees or more - although extremely unlikely given the design + excessive tilting of the vaporizer activates auditory and visual alarms and causes cessation of desflurane output. o it will not work on battery power + the 9-volt battery does not supply the heaters with power or maintain the vaporizer in an operational mode. o the vaporizer will not work unless locked into the selectatec mount, and the selectatec mount wil not allow the use of multiple vaporizers simultaneously

Desflurane TEC6 vaporiser, unable to turn dial on. This is NOT because A. vapouriser is tilted B. Hotter than 39C C. On battery power D. Interlock not engaged, or not seated properly (or something like that) E. other vapouriser is already on

ANSWER B The TEC 6 desflurane vaporizer needs to warm up and the "operational" LED light needs to be illuminated before you can turn the dial to the ON position. If you try to turn it on before the "operational" light is on it will not work. I assume that any of the alarms which cause the "operational" light to go off (and subsequently halt delivery of desflurane) mean that if you turn the dial to 0% (i.e. OFF) then you will not be able to turn it back on, but I have not read that anywhere. According to Graham (BJA 1994; 72:470-73), causes of vaporizer shutdown include: * a tilt of about 10 degrees or more - although extremely unlikely given the design o excessive tilting of the vaporizer activates auditory and visual alarms and causes cessation of desflurane output...Pharmacology of Inhaled Anaesthetics, p212 * it will not work on battery power o the 9-volt battery does not supply the heaters with power or maintain the vaporizer in an operational mode...Pharmacology of Inhaled Anaesthetics, p212 * the vaporizer will not work unless locked into the selectatec mount, and the selectatec mount wil not allow the use of multiple vaporizers simultaneously o nb. note the "Tec" refers to the use with the Select-a-Tec manifold

Xenon concentration can be measured by each of the following EXCEPT A. piezoelectric adsorption B. infra-red absorption spectrometry C. mass spectrometry D. thermal conductivity E. ultrasound techniques

ANSWER B The measurement of xenon is difficult because it is diamagnetic and does not absorb infrared radiation. Its low reactivity precludes the use of specific fuel cell or electrode-type devices. Mass spectrometry works but is expensive. Physical properties such as its high thermal conductivity have also been used. Because of its density and atomic mass, xenon alters the speed of sound such that when an ultrasonic beam is passed through a sample, an estimation of the percentage of xenon can be made. A recent study [6] has demonstrated the accuracy of such a device, with a maximum difference between readings of 1.22% when compared with a reference method of laser refractometry.

60yo with history of hypertension. Presents with chest pain, hoarse voice, left arm weakness. Has lateral T-wave changes on ECG, also present on an old ECG. Heart rate 110, BP 130/80 (definitely this value), SpO2 96% or something. First drug to give: A. aspirin B. metoprolol C. GTN D. nitroprusside E. Heparin

ANSWER B The question is whether this represents dissection or AMI. Didn't think hoarse voice and left arm weakness are typically associated with AMI; thus I think the question is getting at dissection. The history of HTN is also associated. The first drug to give would be a beta-blocker of some description; thus B

The muscles of the upper eyelid receive a somatic nerve supply from the A. oculomotor nerve and a parasympathetic supply from the superior vagus nerve B. oculomotor nerve and a sympathetic supply from the superior cervical ganglion C. ophthalmic division of the trigeminal nerve and a parasympathetic supply from the superior vagus nerve D. ophthalmic division of the trigeminal nerve and a sympathetic supply from the superior cervical ganglion E. ophthalmic division of the facial nerve only

ANSWER B The superior branch (of CNIII, the oculomotor) passes lateral to the optic nerve to supply the superior rectus muscle and levator palpebrae superioris; the inferior branch supplies three muscles: the medial rectus, the inferior rectus and the inferior oblique" (Ellis) and "The clinical manifestation of this is a Horner's syndrome: the pupil is small (paralysis of the dilator pupillae), there is ptosis (paralysis of the sympathetic supply to levator palpebrae)

The muscles of the upper eyelid receive a somatic nerve supply from the A. oculomotor nerve and a parasympathetic supply from the superior vagus nerve B. oculomotor nerve and a sympathetic supply from the superior cervical ganglion C. ophthalmic division of the trigeminal nerve and a parasympathetic supply from the superior vagus nerve D. ophthalmic division of the trigeminal nerve and a sympathetic supply from the superior cervical ganglion E. ophthalmic division of the facial nerve only

ANSWER B The superior branch (of CNIII, the oculomotor) passes lateral to the optic nerve to supply the superior rectus muscle and levator palpebrae superioris; the inferior branch supplies three muscles: the medial rectus, the inferior rectus and the inferior oblique" (Ellis) and "The clinical manifestation of this is a Horner's syndrome: the pupil is small (paralysis of the dilator pupillae), there is ptosis (paralysis of the sympathetic supply to levator palpebrae)"

Hoarseness following a total thyroidectomy may be due to damage to the: A. Deep cervical plexus B. External branch of superior laryngeal nerve C. Hypoglossal nerve D. Internal branch of superior laryngeal nerve E. Superficial cervical plexus.

ANSWER B The superior laryngeal nerve (SLN) has 2 divisions: internal and external. * The internal branch provides sensory innervation to the larynx. It enters the larynx through the thyrohyoid membrane and therefore should not be at risk during thyroidectomy. * The external branch provides motor function to the cricothyroid muscle and is at risk during thyroidectomy. This muscle is involved with elongation of the vocal folds. Trauma to the nerve results in an inability to lengthen a vocal fold and thus to create a higher-pitched sound. The external branch of the SLN is probably the most commonly injured nerve in thyroid surgery. The rate of injury to the external branch of the SLN has been estimated at 0-25%. This rate is probably underestimated, because the diagnosis is frequently missed.

You are performing a Level 2 check on the anaesthetic machine. The suction bulb at the common gas outlet does not stay compressed after 10 seconds. The cause of this could be: A. leak in CO2 absorber B. loosely seated vaporiser C. leak in pipeline O2 D. leak in cylinder attachment to anaesthetic machine E. malfunction in one of the valves of the ventilator circuit

ANSWER B This is a low pressure test : -Bulb is connected to common gas outlet -squeezed repeatedly -must stay deflated > 10 seconds -tests flow meter to common gas outlet NOT the circle system (tubing, valves, CO2 absorber, reservior, vaporizer)

Immediately following delivery by caesarean section under regional anaesthesia a previously healthy primigradiva complains of chest pain and breathlessness, and then becomes unconscious. The most likely diagnosis is A. accidental administration of suxamethonium B. air embolism C. amniotic fluid embolism D. anaphylaxis to syntocinon E. pulmonary thromboembolism

ANSWER B This is a previously healthy patient who suddenly develops chest pain and breathlessness. I don't have any specific references, other than memory of a tutorial at the Royal Hospital for Women, at which the presenter emphasised that the time immediately following delivery has a high risk for venous air embolism, especially if the uterus is exteriorised, due to the large number of vessels open to atmosphere at this stage

Following a cadaveric renal transplant under general anaesthesia, your patient's plasma K+ increases to 6.0 mmol.I-1 in recovery after being 5.0 mmol.I-1 pre and intra-- operatively. This patient requires A. an intravenous infusion of CaCl2 (10 mls over 20 minutes) B. arterial blood gases to ascertain the acid/base status C. potassium exchange resins rectally D. sodium bicarbonate infusion (50-10O mEq over 5-10 minutes) E. urgent haemodialysis

ANSWER B This patient is likely to have chronically elevated K+ and hence be at low risk of arrythmias with a K+ 6.0. An ABG is reasonable to evaluate. However : * If cardiac changes (ie emergent treatment) -> CaCl2 or CaGluconate (up to 0.5ml/kg) * If acidosis & urgent treatment required -> NaHCO3 * If urgent treatment required - Insulin dextrose or urgent dialysis. I generally avoid salbutamol infusions. * If it can wait or using other treatments in the meantime -> resonium (sodium polystyrene sulphonate)

TURP - patient under spinal. Confused. ABG: Na+ 117 / normal gas exchange. Treatment ? A. 10 ml 20% Saline as fast push IV B. 3% NS 100 ml/h C. Normal saline 200 ml/h D. Frusemide 40 mg IV E. Fluid restrict 500 ml/day

ANSWER B Treatment according to OHA 1. Alert surgeon: Stop further resection and diathermy bleeding points. Then terminate surgical procedure. 2. Stop IV fluids 3. Give frusemide 40mg (to reduce chance of APO) and check Na+ and Hb 4. Support respiration and ventilation if required 5. Administer IV anticonvulsants if fitting 6. Central pontine myelinolysis may result from BOTH over-rapid correction of chronic hyponatraemia AND acute severe hyponatraemia If the serum Na+ has fallen acutely ,120mmol/L and is assoc. with neurological signs, consider giving hypertonic saline (2N/S or 3%) to restore Na to around 125mmol/L Give 1.2-2.4 mL/kg/hr of 3% Saline until symptoms improve or Na+ >125mmol/L. This should produce a rise in serum Na+ of 1-2 mmol/L/hr Beware of compounding effects on Na+ by other simultaneous treatments (eg diuretics, colloids etc) 7. Admit to ICU/HDU for management including regular Measurement of Na+

Correct statements regarding trigeminal neuralgia include A. Associated sensory loss is common. B. lt most frequently occurs in the second or third division of the nerve (V2 or V3). C. Oedema of the trigeminal ganglion is usually seen on a cranial CAT scan. D. The pain is often bilateral. E. The pain is usually described as an intense deep ache.

ANSWER B Trigeminal neuralgia is a neuropathic disorder characterized by episodes of intense pain in the face, originating from the trigeminal nerve. -1 or all 3 branches of trigeminal branches may be affect -Distribution * V2 + V3 commonest → V3 → V2 → V1 rarest * Usually unilateral R>L (about 3% bilateral) -10% bilateral -intense facial pain last from a few seconds to hours Well established link to multiple sclerosis (MS) * MS occurs in 2-4% of patients with Trigeminal neuralgia * Trigeminal neuralgia diagnosed in 1-5% of MS patients Treatment 1. Medical -first line : carbemazepine (NNT=2.6) -second line : baclofen, lamotrigine, phenytoin, gabapentin (NNT=3.6), valproate -low doses of amytriptyline (NNT=2.1) is thought to be effective, [pprer side effect profile and dangerous if overdosed -opiates 2. Surgical -evidence is poor -microvascular decompression resluts in longest pain releif -percutaneous radiofrequency thermorhizotomy → 20% relief at 10 years -glycerol injections

In relation to foetal outcome, which of the following is most important: A. Induction-delivery time B. Displacement of the uterus C. Minimal Thiopentone on induction D. The use of 100% oxygen E. Keeping inhalational agent use to a minimum

ANSWER B Uterine incision to delivery (U-D) time is more important than induction to delivery (I-D) time. A U-D >3mins is assoc with lower umbilical pH. Regional: prolonged I-D doesn't adversely affect Apgars provided hypotension is treated. GA: prolonged I-D >8mins may worsen neonatal acidosis, but not if hypotension is treated, and maternal FIO2at least 65%. Prolonged exposure to volatile/N2O may decrease 1min apgar, but not 5min.

The observed fall in cardiac output induced by carbon dioxide pneumoperitoneum (with intra- abdominal pressures below 12 mmHg), during laparoscopic cholecystectomy is primarily a result of A. a fall in venous return to the heart B. an increase in systemic vascular resistance C. increased pressure transmitted to intra-thoracic baroreceptors D. increased pulmonary vascular resistance E. reflex bradycardia

ANSWER B VR will fall if pressure > 15mmHg SVR will increase over all ranges

When the calcium antagonists diltiazem, verapamil or nifedipine are used in patients with cardiovascular disease A. diltiazem causes the greatest depression of atrioventricular nodal conduction B. nifedipine is the most likely to increase the heart rate C. nifedipine causes the least systemic vasodilation D. diltiazem causes the greatest depression of myocardial contractility E. verapamil is the most likely to cause lower limb oedema

ANSWER B Verapamil has most effect on the heart, nifedipine most on the blood vessels; diltiazem in between A. Probably Verapamil B. True - reflex tachy - Nifedipine incr; dil no change; ver dec C. False - most D. False - veramamil E. False - nifedipine

An alcoholic man has pale stools, dark urine, RUQ pain, AST 2000, ALP 100, Bilirubin 50, vomiting and diarrhoea. The most likely diagnosis is: A. Drug-induced hepatitis B. Viral hepatitis C. Obstructive jaundice D. Chronic active hepatitis E. Alcoholic cirrhosis

ANSWER B Viral hepatitis on the background of alcoholism * Mildly elevated ALT level (less than 1.5 times normal) o ALT value could be normal for gender, ethnicity or body mass index. o Consider muscle injury or myopathy. * Alcoholic hepatitis o Laboratory values can appear cholestatic, and symptoms can mimic cholecystitis. o Minimal elevations of AST and ALT often occur. * AST level greater than 500 U per L o The AST elevation is unlikely to result from alcohol intake alone. o In a heavy drinker, consider acetaminophen toxicity. * Isolated rise in AST o consider extrahepatic source such as cardiac muscle * Common bile duct stone o Condition can simulate acute hepatitis. o AST and ALT become elevated immediately, but elevation of AP and GGT is delayed. * Isolated elevation of GGT level o This situation may be induced by alcohol and aromatic medications, usually with no actual liver disease. * Isolated elevation of AP level (asymptomatic patient with normal GGT level) o Consider bone growth or injury, or primary biliary cirrhosis. o AP level rises in late pregnancy. * Isolated elevation of unconjugated bilirubin level o Consider Gilbert syndrome or hemolysis. * Low albumin level o Low albumin is most often caused by acute or chronic inflammation, urinary loss, severe malnutrition or liver disease o It is sometimes caused by gastrointestinal loss (e.g., colitis or some uncommon small bowel disease). o Normal values are lower in pregnancy. * Blood ammonia level o Blood ammonia values are not necessarily elevated in patients with hepatic encephalopathy. o Determination of blood ammonia levels is most useful in patients with altered mental status of new onset or unknown origin. ALT=alanine aminotransferase; AST=aspartate aminotransferase; AP=alkaline phosphatase; GGT=gamma-glutamyltransferase.

Von Willebrand's disease: A. Autosomal recessive B. Abnormal bleeding time C. Usually abnormal APTT D. Treatment of choice for bleeding is factor VIII

ANSWER B Von Willebrands disease * Autosomal dominant trait (for Type I and II) and autosomal recessive (Type 3) affecting <3% of population * Deficit or defective amounts of VWF (von Willebrand factor) which is required for adherence of platelets to endothelium * Binding protein for Factor VIII in plasma * Three forms: 1. Type I - Quantitative decrease but normal vWF and Factor VIII. Usually mild clinical symptoms with variable penetrance. 70-80% of VWD is this type. 2. VWD type 2 has a number of different forms but is primarily a qualitative disease of vWF (ie non functional vWF). This is 15-20% of VWD 3. Type 3 - Undetectable vWF; therefore most severe. Types 2 and 3 do NOT respond to DDAVP. 25% of patients with type I have APTT outside reference range, and pts with type II or III who have abnormal vWF and therefore abnormal factor 8 complex will have abnormal aPTT. Replacing factor 8 may help, but vWF is needed as well. Cryoprecipitate or plasma derived concentrates of Factor VIII (doses vary depending on clinical situation) are used. DDAVP may stimulate release of vWF. Note that recombinant preparations of FVIII do not contain vWF and are therefore not effective. * DDAVP (0.3 mcg/kg IV) for mild bleeding * Factor VIII-vWF concentrate recommended for severe or surgical bleeding (40 - 75 IU/kg) - not purified factor VIII * Cryoprecipitate may be used but carries risk of transfusion-transmitted infection * Aim for VIII levels 50 - 70 %

SF61 ANZCA version [2004-Apr] Q144, [2005-Sep] Q88 The optimal patient position for anaesthetising a woman at term with an umbilical cord presenting externally is A. knee-chest (i.e. on hands and knees) B. left lateral (i.e. on side with left side down) C. supine with head down (and lateral tilt) D. supine with head elevated (and lateral tilt) E. supine and level (with lateral tilt)

ANSWER B Well the best way to deal with this is as follows: 1. Get obstetrician to calm down 2. Ask assistant to place hand on head of baby, and at the same time feel cord. 3. Ask assistant if cord is warm and pulsatile. 4. If cord is warm and pulsatile, take your time. You are safer placing the patient on their side, and performing a spinal. Don't listen to the howling obstetric staff, your priority is the safety of the mother at all times. You do no favours inducing GA and failing to get an airway, when there is time to perform a safer manouvre. 5. If cord is cold and non-pulsatile, I would then induce GA. The safest method for mother is lateral tilt and horizontal. Some people advocate head down to relieve pressure on the cord, but this decreases FRC and endangers the mother. I think there is no great sense in this if an assistant is pushing the head back already.

The LEAST desirable position for the tip of a central venous catheter which has been inserted into the left internal jugular vein is A. mid-way along the left brachiocephalic vein B. at the junction of the left brachiocephalic vein and the superior vena cava (SVC) C. in the SVC at the level of the carina D. at the junction of the SVC and the right atrium E. in the right atrium

ANSWER B Whenever the catheter negotiates a sharp bend, the catheter must be passed a reasonable distance beyond the bend such that the axes of the catheter and vein are aligned. In the case of insertion from the left side, this may be achievable only by siting the tip in the right atrium. 'The carina as a radiological landmark for central venous catheter tip position' P. A. Stonelake and A. R. Bodenham British Journal of Anaesthesia 2006 96(3):335-340 * Zone A represents the lower SVC and upper RA. In this zone CVCs placed from the left side are likely to lie parallel to the vessel walls. However, a part of this zone lies within the RA and therefore within the pericardial reflection. This may represent a necessary compromise for left-sided CVCs to ensure they lie parallel to the vessel wall. * Zone B represents the area around the junction of the left and right innominate veins and the upper SVC. This is a suitable area for CVCs placed from the right side, however left-sided CVCs will enter this area at a steep angle and are at risk of abutting the lateral wall of the SVC and should ideally be advanced into zone A. * Zone C represents the left innominate vein proximal to the SVC. CVCs in zone C are probably suitable for short-term fluid therapy and CVP monitoring, but not for inotrope infusions or long-term use. * A high proportion of left-sided lines above the carina had a steep (≥40°) angle to the vertical...These can be further divided into two groups: those lying within the left innominate vein proximal to the SVC (zone C) and those where the tip abuts the right internal wall of the SVC (zone B). The latter group has a higher chance of vessel wall perforation and associated morbidity.

Recognised weaknesses of systematic reviews include all of the following EXCEPT A. publication bias B. duplicate publication C. study heterogeneity D. inclusion of outdated studies E. systematic review author bias

ANSWER B http://cjasn.asnjournals.org/content/3/1/253.full

A 38-year-old primigravida presents with progressive dyspnoea in late pregnancy. The strongest indicator for further investigation would be A. a 2/6 systolic ejection murmur B. a raised JVP (jugular venous pressure) C. a third heart sound D. orthopnea E. peripheral oedema

ANSWER B normal changes -increase in end-diastolic chamber size -increase in total L ventricular wall thickness -CVP unchange -asymtomatic pericardial effusion -an innocent grade I or II systolic heart murmur -S3, or S4 in late pregnancy -ECG: increase in benign dysrhythmias, reversible ST, T and Q wave changes and some L axis deviation. indication of heart disease: -systolic murmur greater than grade III -any diastolic murmur -severe arrthymias -unequivacol cardiac enlargement on x-ray. -presence of congestive heart failure is suggested by hepatomegally and jugular venous distension paroxysmal nocturnal dyspnoea, chest pain, nocturnal cough, new regurgitant murmurs, pulmonary crackles, elevated jugular venous pressure and hepatomegaly.

What is the chance of having the disease if the test is positive? Disease Yes No Test Positive 0.5 0.2 Negative X 0.8 A. 0.89 B. 0.71 C. ? D. ?

ANSWER B refers to the PPV which is 0.71

Paget's disease of bone is LEAST likely to affect the A. heart B. respiratory system C. renal system D. neurological system E. vertebral column

ANSWER B PAGETS DISEASE Chronic disorder characterised by breakdown & regrowth of bone, resulting in enlarged & deformed bones AETIOLOGY • men & women equally affected • familial tendency • increased incidence in patients > 50yrs, present in ~10% of those > 80yrs PATHOPHYSIOLOGY • Excessive osteoblastic and osteoclastic activity • Abnormally thick but weak bones • Bones most commonly involved: Pelvis, leg bones, vertebrae, humerus & clavicle • cause unknown, believed to be caused by a progressive infection, present for many years before symptoms appear TREATMENT • calcitonin • bisphosphonates (inhibit osteoclasts, treatment of choice) CLINICAL FEATURES • often asymptomatic (up to 70% patients with Pagets will be asymptomatic) • Bones: - bone pain (most common symptom) - enlarged bones - fractures - deformity - arthritis - sarcoma (rare - < 1% of patients with Pagets disease) • Neurological (nerve compression from enlarged bone) - hearing loss o may be sensorineural (VIII n compression) o may be conductive (involvement of middle ear ossicles) o or mixed - other CN palsies - headaches • High output cardiac failure • Renal calculi • Loose teeth (Pagets affecting the facial bones) REFERENCES Stoelting Anaesthesia & co-existing disease 'Diagnosis & Treatment of Pagets disease of bone' American family physician 2001

The pressure in the epidural space: A. Lower in the lumbar region sitting then supine B. Is negative in the thoracic area due mainly to tenting of the dura by the needle C. Is higher in the thoracic region then lumbar when sitting D. ?

ANSWER B The epidural space is slightly positive, but large negative pressures are induced by tenting of the epidural space from the Tuohy needle and account for the rapid inward entry of saline using the hanging-drop method.

AM43 ANZCA version [2004-Aug] Q119, [2005-Apr] Q82, [Jul07] A 13-year-old boy with Duchenne's muscular dystrophy A. is at increased risk of malignant hyperthermia B. is likely to have significant scoliosis C. is more likely to be cachectic than obese D. is unlikely to have cardiac involvement E. may develop rhabdomyolysis when non-depolarising muscle relaxants are used

ANSWER B * A. False - "It is now thought that DMD and other dystrpphinopathies are not associated with malignant hyperthermia", although prolonged exposure to volatiles or sux produces a MH like experience * B. true - once wheelchair bound (8-12yrs) scoliosis progresses rapidly * C. no - usually overweight (apparently) * D. FALSE Cardiac failure often present by age of 6 * E. FALSE - sux and prolonged volatile exposure

Acromegaly due to excess of growth hormone. Why is it difficult to do a direct laryngoscopy? A: Distorted facial anatomy B: Macroglossia C: Glottic stenosis D: Prognathe mandible E: Arthritis of the neck

ANSWER B CEACCP Airway maintenance and manual 'bag and mask' ventilation is usually straightforward. Laryngoscopy and tracheal intubation may be more difficult, due to a combination of macrognathia, macro- glossia, and expansion of upper airway soft tissues; however, tra- cheal intubation is achievable with standard techniques such as external laryngeal pressure and the use of a gum elastic bougie or airway exchange catheter.4 Awake fibreoptic intubation is increasingly regarded as the favoured solution for difficult intuba- tion in these patients. Up to 70% of patients will have significant obstructive sleep apnoea (OSA) as a result of soft tissue enlarge- ment of the upper airway. This is associated with airway difficul- ties, cardiac instability, and postoperative cardiorespiratory failure

In preparation for a posterior-lateral incision for a thoracotomy, a patient is placed in the left lateral decubitus position. In this position the A. axillary roll is placed under the chest to aid surgical exposure B. dependant leg should be extended and the superior leg flexed C. neck should be extended to allow access to the airway D. radial nerve is the most commonly injured nerve E. right brachial plexus is at risk of injury if the right arm is flexed at the shoulder to more than 90deg

ANSWER B E A. FALSE : axillary roll is placed to remove pressure of the humoral head and reduce the risk of brachial plexus injury B. FALSE : both hips flexed 30-40deg C. FALSE D. FALSE : Ulnar nerve E. TRUE : shoulders should be kept at 60-80deg RF in positioning for ulnar nerve injury include -abduction ->90deg shoulder flexion -shoulder extension

AM01 [1986] [1987] In the laboratory test for malignant hyperpyrexia, the following are used: A. Suxamethonium B. Caffeine C. Halothane D. Potassium E. Dibucaine

ANSWER B and C

You are performing epidural anaesthesia on an adult patient. To minimize the chance of inserting the epidural catheter into a blood catheter into a blood vessel you could: A. avoid using a combined spinal-epidural technique B. establish loss-of-resistance with saline rather than air C. inject saline prior to threading the catheter D. perform the procedure in the sitting rather than the lateral position E. use a midline rather than a paraspinous (paramedian) approach

ANSWER B and C

Which of the following is NOT a normal pressure measurement? A. Pulmonary artery: 25/10 mmHg B. Aortic root: 120/0 mmHg C. Right ventricle 25/8 mmHg D. Right atrium: 5 mmHg E. Left atrium: 3 mmHg

ANSWER B and C RAP 5-10/0 RVP 25/0 PAP 25/8 LAP 5-10/0 LVP 120/0 MAP 120/80

AM34 [Mar00] [Jul00] Myotonia: A. (?Not) Reversed with neuromuscular blockade B. Autosomal dominant appearing 2nd-3rd decade C. Not reversed with deep inhalational anaesthesia D. Not reversed with regional anaesthesia E. Give volatiles & NM blockers to help contractures

ANSWER B and D

Known adverse effects of intrathecal pethidine include: A. Hypertension B. Hypotension C. Tachycardia D. Sedation E. Increased sweating

ANSWER B and D * A. False. * B. True. "Following a dose of 0.5 mg/kg, the reported incidence of hypotension ranged from 0 - 17% . . ." * C. False. "Bradycardia after intrathecal pethidine has been observed in a number of other reports." * D. True. "Other adverse effects that are associated with intrathecal pethidine include nausea and vomiting, pruritus, sedation and respiratory depression." * E. False.

AM32 [Apr99] In comparing the serotonergic syndrome with the neuroleptic malignant syndrome: A. Abnormal ocular signs with serotonin but not NMS B. Muscular rigidity of the lower limbs different to that seen in NMS C. Can be triggered by dopamine agents unlike NMS which can be triggered by dopamine antagonists like bromocriptine D. More rapid onset & shorter duration with serotonin syndrome

ANSWER B and D A - False (?) * Mydriasis with serotonin syndrome (? also ping-pong gaze) B - True * lead-pipe rigidity in NMS and hyperreflexia, tremors and clonus in SS. C - False * Bromocriptine is a dopamine agonist used in the treatment of NMS. * Bromocriptine can cause SS D True * NMS has a slow onset (days to weeks) and a slow progression of 24-72 hours, whereas serotonin syndrome has a more rapid onset and progression. SS and NMS can be difficult to distinguish -mydriasis -hyperreflexia -tremors -myoclonus -diarrhoea

Regarding atrial fibrillation A. aspirin is as effective as warfarin for the prevention of embolic stroke in elderly patients B. patients should not receive warfarin therapy unless they have risk factors for embolic stroke C. patients who are converted to sinus rhythm should be placed on rhythm maintenance therapy D. rhythm control has NOT been shown to be superior to rate control E. the best drug for rate control during exercise is digoxin

ANSWER B and D Embolic Risk of AF : Application of the CHADS2 CHF - 1 HT - 1 Age > 75 - 1 DM - 1 CVA - 2 Score of *0 : 0.5-1.7% Risk per year of stroke, Commence Aspirin only *1 : Unsure if stroke benefit outweights bleeding risk if patient is female; greater >65 OR has any artherosclerotic disease (PVD/CAD) they should be started on warfarin (CHA2S2VASc score) *>2 : 4-18% Risk per year of stroke, commence warfarin AFFIRM, PIAF, RACE, STAF which show that rate control is at least as effective as rhythm control in improving symptoms and functional capacity, particularly in those over 65yrs. AFFIRM -the Atrial Fibrillation Follow-up Investigation of Rhythm Management -designed to determine whether restoration and maintenance of SR is associated with a lower all-cause mortality when compared to the alternative strategy of anticoagulation and rate control in patients with AF -The sample size for this trial was calculated at 5300, but after screening 7400 patients, enrollment was stopped at just over 4000 -Sotalol and amiodarone (for rhythm control) -ventricular rate control was achieved with digoxin, beta-blockers, and rate-lowering calcium-channel blockers -no difference in the all cause mortality Rhythm Control Limb -at least 80% of the patients could be stabilized into sinus rhythm, and 60% remained in sinus rhythm after 5 years -strangely, a larger number were hospitalized -all cause mortality was greater when analyzed with intention to treat

SF76 ANZCA version [2004-Apr] Q140 Concerning backache in obstetrics A. gestational backache occurs in 10-20% of pregnant women B. posterior pelvic pain is much more common than lumbar pain during pregnancy C. relaxin production is decreased during pregnancy D. risk factors include smoking history and ethnicity E. the principal diagnostic tool is magnetic resonance imaging (MRI)

ANSWER B and D This question appears to come from Curr Opin Anaesthesiol. 2003 Jun;16(3):269-73 * A - incidence 40-90% * B - The locations of backache can be thoracic or the low back. The pain occurring in the low back can further be classified into lumbar pain (LP) and posterior pelvic pain (PPP). PPP is four times more common than LP during pregnancy [1]. * C - The etiology of GBP is multifactorial [1-3]. The most widely accepted explanations relate to the secretion of relaxin, a hormone that facilitates the loosening of the supporting structure, tendons, and ligaments, making the spine and sacroiliac joints (SIJs) 'less stable', and the expansion of the uterus causing pregnant women to shift their center of gravity resulting in back strain [1-3]. * D - The risk factors that contribute to the development of GBP are: a history of backache, ethnic background, smoking, parity, type of work, age, fetal weight, and rapid weight gain over a short period of time. Other pathological changes such as lumbosacral disc herniation, spondylolisthesis, coccydynia can also cause GBP [1-3]. * E - The evaluation of pregnancy-induced backache begins with an in-depth history and a thorough physical examination. Once a medical history has been completed, a physical examination, with emphasis on the back and a detailed neurological examination, should be performed. It is a relative contraindication to utilize radiographs as part of a backache work-up during early pregnancy. Magnetic resonance imaging is a safe means of assessing the spine and pelvis [2].

Best evidence in obstetric anaesthesia supports each of the following assertions EXCEPT A. colloid prevents hypotension from regional anaesthesia more effectively than crystalloid B. fentanyl added to spinal bupivacaine for caesarean section has no influence on the incidence of intraoperative nausea C. high doses of ephedrine (>15 mg) are more likely to cause hypertension than prevent hypotension D. in labour, combined spinal-epidural analgesia is associated with faster onset and greater maternal satisfaction than epidural analgesia E. in nulliparous women, epidural analgesia in labour, compared with intravenous opioid analgesia, does not increase caesarean section rate

ANSWER B and D (both wrong)

\A patient with pulmonary hypertension secondary to lung disease presents for laparotomy. The anaesthetic technique LEAST likely to exacerbate the pulmonary hypertension is endotracheal intubation and controlled ventilation with A. high dose opioids, N2O and O2 B. isoflurane and O2 / air C. isoflurane, N2O and O2 D. ketamine and O2 / air E. propofol and O2 / air

ANSWER B and E MEDICAL MANAGEMENT of PPHN 1) Minimize Pulmonary Hypertension/Vasoconstriction AVOID: HYPOXIA, HYPOTHERMIA, ACIDOSIS, ANEMIA, HYPOTENSION AND STIMULATION ! 2) Maximize Pulmonary Vasodilatation (Decrease pulmonary vascular resistance) a) OXYGEN (FiO2 = 1.0) b) ALKALINIZATION - METABOLIC ALKALOSIS (pH > 7.55) 3) Support Cardiac Output and Blood Pressure a) VOLUME b) INOTROPIC AGENTS: Dobutamine, Dopamine and Epinephrine 4) Relieve Pain and Anxiety a) ANALGESIA: Morphine or Fentanyl b) SEDATION: Lorazepam, Chloral Hydrate, Phenobarbital, Midazolam and Thorazine c) PARALYSIS: Pavulon 5) Administer Pulmonary Vasodilating Agents a) "INHALED NITRIC OXIDE" b) TOLAZOLINE c) PROSTAGLANDIN E1 d) ISOPROTERENOL 6) Avoid Barotrauma a) Small tidal volumes with high rates (i.e., HFOV) b) Avoid hyperventilation (pCO2 ² 30) to minimize barotrauma

Jehovah's witness refused blood- you have told him you refuse to do the surgery/anaesthesia for his own good. Ethical principle: A. Paternalism B. Maleficience C. Autonomy D. Beneficience

ANSWER B? Paternalism would be to proceed and give him blood regardless of his autonomy because we know best Non maleficence would be first to do no harm. In this case not proceeding with surgery. Autonomy would be to proceed and not give him blood even if he exsanguinates. Beneficience ?? action that is done for the benefit of others. Examples of beneficent actions: Resuscitating a drowning victim, providing vaccinations for the general population, encouraging a patient to quit smoking and start an exercise program, talking to the community about STD prevention.

A 35yr old African-American with sickle cell and fractured ankle for ORIF. Hb 90, Hct 0.3. A. T/F 2 units packed cells (?pre-op) B. Let him cool passively to low/normal temperature C. Spinal is safe D. Avoid thiopentone E. Tourniquet is absolutely contra-indicated

ANSWER C

A 35yr old African-American with sickle cell and fractured ankle for ORIF. [Hb] 90g/l, Haematocrit 0.3. A. Transfuse 2 units packed cells (?pre-op) B. Let him cool passively to low/normal temperature C. Spinal is safe D. Avoid thiopentone E. Tourniquet is absolutely contra-indicated

ANSWER C

A 4y.o. boy presents for insertion of grommets. His maternal great-grandfather is know to have had an episode of malignant hyperthermia. Which of the following is the strongest evidence that the boy is NOT susceptible to MH? A. the boy was exposed to halothane at age 2 with no sequelae B. the boy has recently been shown to have a normal serum creatinine kinase C. the boy's grandfather has had a negative muscle contracture test for MH D. the boy's mother has had negative molecular genetic testing for MH E. there have been NO other episodes of MH in the family despite exposure to known triggers on multiple occasions

ANSWER C

A 55 year old man suddenly develops left sided chest pain 18 hours after an inguinal hernia repair. He has a nonproductive cough, dyspnoea, a fever of 38C, pulse rate of 110/min and tachypnoea. The ECG shows non-specific ST changes. Provisional diagnosis: A. Atelectasis B. Pneumothorax C. Pulmonary embolism D. Myocardial infarction E. Bronchopneumonia

ANSWER C

A 6-year-old child is referred via the orthopaedic clinic for closed reduction under general anaesthesia of a forearm fracture, sustained 2 days earlier. He is an asthmatic, usually well controlled, but has a current upper respiratory tract infection. He is on salbutamol 2 puffs twice a day, and on auscultation his chest is clear. He is adequately fasted and can proceed to theatre immediately. The best choice for airway management of this child is A. endotracheal intubation and controlled ventilation B. endotracheal intubation and spontaneous ventilation C. facemask and spontaneous ventilation D. laryngeal mask airway and spontaneous ventilation E. ProsealTM laryngeal mask airway and spontaneous ventilation

ANSWER C

A 9 year old boy for an appendix operation is induced with thiopentone, fentanyl, suxamethonium. Two minutes after induction, he is pulseless & ECG shows sinus arrest. The most likely cause is: A. Allergy to suxamethonium B. Bradycardia secondary to suxamethonium C. Duchenne muscular dystrophy (?undiagnosed) D. Dystrophica myotonia E. Malignant hyperthermia

ANSWER C

A child with intra-operative blood loss. A cardiac arrest is most likely because of A. A delay in delivery of blood from the blood bank B. Inadequate intravenous access C. Underestimated intra-operative blood loss D. Underestimated pre-operative hypovolaemia E. Complication of transfusion

ANSWER C

A line isolation monitor protects against microshock A. only if the warning current is set at 10mA B. only if the warning current is set at 30mA C. under no circumstances D. only if the equipment used is grounded E. only if it monitors all the equipment in the region

ANSWER C

A one year old child arrests with VT. Has had 2x DC shocks, and 100 mcg adrenaline. Further 1x DC shock given. What is next step? A. 20 J DC shock B. 40 J DC shock C. 50 mg amiodarone D. 100 mcg adrenaline E. 1000 mcg adrenaline

ANSWER C

A patient has suffered flash burns of the upper half of the left upper limb, all of the left lower limb and the anterior surface of the abdomen. The approximate percentage of the body surface which has been burned is A. 18% B. 23% C. 32% D. 41% E. 48%

ANSWER C

A patient has suffered flash burns of the upper half of the left upper limb, all of the left lower limb and the anterior surface of the abdomen. The approximate percentage of the body surface which has been burned is: A. 18% B. 23% C. 32% D. 41% E. 48%

ANSWER C

A patient is having an electrophysiological study and ablation for atrial fibrillation. Suddenly the blood pressure drops to 76/38 mmHg, with the heart rate at 110 in sinus rhythm. What is the best investigation to confirm the cause of hypotension? A. Troponin B. ST-segment elevation C. Transoesophageal echocardiography D. Coronary Angiogram E. Electrocardiogram F. MRI

ANSWER C

AM02 Which of the following are useful in the treatment of malignant hyperpyrexia? A. 100% 02 B. Phenytoin C. Sodium dantrolene D. Dextrose E. Sodium bicarbonate

ANSWER C

AM31 ANZCA version [2004-Apr] Q11 (Similar question reported in [Apr99] [Mar00]) Malignant hyperthermia: A. May be confirmed by a muscle biopsy showing contracture in response to halothane, caffeine & calcium B. Is an autosomal recessive disorder C. Should be treated with an initial dose of dantrolene of 2.5mg/kg D. Had a mortality as high as 50% prior to dantrolene E. Is consistently associated with a rapidly rising temperature as one of the early signs

ANSWER C

ANZCA Version [Jul 07] Q.147 A 4y.o. boy presents for insertion of grommets. His maternal great-grandfather is know to have had an episode of malignant hyperthermia. Which of the following is the strongest evidence that the boy is NOT susceptible to MH? A. the boy was exposed to halothane at age 2 with no sequelae B. the boy has recently been shown to have a normal serum creatinine kinase C. the boy's grandfather has had a negative muscle contracture test for MH D. the boy's mother has had negative molecular genetic testing for MH E. there have been NO other episodes of MH in the family despite exposure to known triggers on multiple occasions

ANSWER C

ASA grading was introduced to A. predict intraop anaesthetic risk B. Predict intraop surgical and anaesthetic risk C. Standardise the physical status classification of patients D. Predict periop anaesthetic risk E. Predict periop anaesthetic and surgical risk

ANSWER C

ASA grading was introduced to A. predict intraop anaesthetic risk B. Predict intraop surgical and anaesthetic risk C. Standardise the physical status classification of patients D. Predict periop anaesthetic risk E. Predict periop anaesthetic and surgical risk

ANSWER C

ASD murmur heard at A. ASD B. Tricuspid valve C. Pulmonary valve D. Mitral valve E. Aortic valve

ANSWER C

AZ71 ANZCA version [2003-Apr] Q99 The most common reason for patients complaining about their medical treatment is A. charging of excessive fees B. patient's predisposition to com plain C. poor communication with medical staff D. poor outcome of medical treatment E. poor physical environment

ANSWER C

Anaphylaxis is least likely in the following:: (a) adults (b) latex exposure in a patient with a fruit allergy (c) muscle relaxants given to someone with a documented IgE antibody to thiopentone (d) protamine given to a patient using protamine insulin

ANSWER C

Appropriate postoperative maintenance fluid in a child [can't recall situation, but something to do with head injury]: With a serum sodiaum of 142 A. 3% and 1/3 NS B. 1/2 NS C. Normal Saline D. Hartmanns E. Hartmanns with glucose

ANSWER C

C-reactive protein: A. is a reliable marker of metastatic breast cancer B. is a chronic phase protein produced in the lungs C. is usually increased dramatically in response to infection D. is found only in the cerebrospinal fluid (CSF) E. decreases in response to tissue trauma

ANSWER C

Carbon dioxide is the most common gas used for insufflation for laparoscopy because it A. is cheap and readily available B. is slow to be absorbed from the peritoneum and thus safer C. is not as dangerous as some other gases if inadvertently given intravenously D. provides the best surgical conditions for vision and diathermy E. will not produce any problems with gas emboli as it dissolves rapidly in blood

ANSWER C

Compared to lignocaine, bupivacaine is A. Twice as potent B. Three times as potent C. Four times as potent D. Five times as potent E. Same potency

ANSWER C

Compared to lignocaine, bupivacaine is: A. Twice as potent B. Three times as potent C. Four times as potent D. Five times as potent E. Same potency

ANSWER C

During elective major vascular surgery the best way to reduce the risk of acute renal failure is to maintain a normal A. central venous pressure B. mean arterial blood pressure C. renal blood flow D. systemic vascular resistance E. urine output

ANSWER C

During transfusion of platelets a patient develops fever, rigors and vomiting and becomes hypotensive and tachycardic. The most likely diagnosis is A. ABO incompatibility B. Anaphylaxis C. bacterial contamination of the platelets D. leukocyte mediated transfusion reaction E. viral contamination of the platelets

ANSWER C

During transfusion of platelets a patient develops fever, rigors and vomiting and becomes hypotensive and tachycardic. The most likely diagnosis is A. ABO incompatibility B. anaphylaxis C. bacterial contamination of the platelets D. leukocyte mediated transfusion reaction E. viral contamination of the platelets

ANSWER C

Exponential decline / definition of time constant A. time for exponential process to reach log(e) of its initial value B. Time until exponential process reaches zero C. Time to reach 37% of initial value D. Time to reach half if its initial value E. 69% of half life

ANSWER C

Exponential decline / definition of time constant (with various options) A. time for exponential process to reach log(e) of its initial value B. Time until exponential process reaches zero C. Time to reach 37% of initial value D. Time to reach half if its initial value E. 69% of half life

ANSWER C

Factors associated with post-operative ulnar nerve palsy include all of the following EXCEPT A. male gender B. sternal retraction for cardiac surgery C. cardiopulmonary bypass for cardiac surgery D. internal jugular vein catheterisation E. diabetes mellitus

ANSWER C

Finding on haemophilia A patient A. Female haemarthrosis B. Male haemarthrosis C. Normal PT, abnormal APTT D. Abnormal PT, normal APTT

ANSWER C

How quickly does the CO2 rise in the apnoeic patient ? A. 1 mmHg per min B. 2 mmHg per min C. 3 mmHg per min D. 4 mmHg per min E. 5 or ?8 mmHg per min

ANSWER C

In a neonate the main resistance in a circle system with CO2 absorber A. APL valve B. Expiratory and inspiratory unidirectional valves tubing C. ETT D. HME filter

ANSWER C

In a patient requiring fresh frozen plasma (FFP), where the patient's blood group is unknown, it is ideal to give FFP of group A. A B. B C. AB D. O E. blood group of FFP in this situation doesn't matter

ANSWER C

In asthmatic patients having anaesthesia, the most significant factor in bronchospasm production is: A. Thiopentone B. Suxamethonium C. Intubation D. Opiate premedication

ANSWER C

Indication for percutaneous closure of ASD A. Ostium primum < 3cm B. Ostium primum > 3cm C. Ostium secundum < 3 cm D. Ostium secundum > 3cm E. Sinus venosus ASD

ANSWER C

Indication for percutaneous closure of ASD A. Ostium primum < 3cm B. Ostium primum > 3cm C. Ostium secundum < 3 cm D. Ostium secundum > 3cm E. Sinus venosus ASD

ANSWER C

Indications for steroids in neurosurgery a. Cerebral abscess b. Subdural haematoma c. Meningioma d. SAH e. Traumatic brain injury

ANSWER C

Indicative of severe Aortic Stenosis? A. Palpitations B. Fatigue C. Paroxysmal Nocturnal Dyspnoea D. Angina E. Syncope

ANSWER C

Infra-renal aortic cross-clamping usually results in A. decreased cardiac contractility B. decreased coronary blood flow C. decreased renal blood flow D. minimal change in cardiac output E. increased heart rate

ANSWER C

Initial metastatic deposits in the liver are best indicated by: A. Increased bilirubin B. Decreased serum albumin C. Increased alkaline phosphatase D. Increased gamma globulins E. Increased INR

ANSWER C

OLV hypoxaemia. After 100% O2 and FOB next step is: A. CPAP 5cm top lung B. CPAP 10cm top lung C. PEEP 5cm bottom lung D. CPAP 5cm top + PEEP 5cm bottom

ANSWER C

PAC seeing patient with thyroid disease. Most reassuring factor for normal thyroid function is: A. Absence of 'hot' nodules on nuclear scan B. Normal ocular signs C. Normal heart rate D. Normal temperature E. Absence of any antithyroid medications

ANSWER C

PL27 ANZCA version [2004-Apr] Q141 All of the following may be useful in the treatment of ventricular fibrillation due to bupivacaine cardiotoxicity EXCEPT A. adrenaline B. diazepam C. intralipid D. propofol E. vasopressin

ANSWER C

PP88 ANZCA Version [Jul06] Q131, [Jul07] The glomerular filtration rate of a paediatric patient (in ml.min-1.m-2) is similar to that of an adult by the age of approximately A. 1 month B. 6 months C. 2 years D. 4 years E. 6 years

ANSWER C

PZ112 ANZCA version [2004-Apr] Q105, [Mar06] Q131, [Jul06] Q2 Hepatotoxicity from paracetamol overdose is enhanced in A. chronic renal failure B. concomitant ingestion of benzopdiazepines C. conditions associated with glutathione deficiency D. obese patients E. patients with hepatitis C antibody

ANSWER C

Post thyroidectomy patient, patient in PACU for 30 minutes. Develops respiratory distress. MOST likely cause? A. Hypercalcemia from taking parathyroids B. Bilateral laryngeal nerve palsies C. Bleeding and haematoma D. Tracheomalacia E. ?

ANSWER C

Prior to surgical stimulation, which of the following drugs, when added to propofol or volatile anaesthesia, will decrease the bispectral index (BIS)? A. 70% nitrous oxide B. ketamine (0.5mg. kg-1) C. midazolam (0.2 mg.kg-1) D. remifentanyl infusion (0.25 microg.kg-1.min-1) E. all of the above

ANSWER C

Required for diagnosis of Neuroleptic Malignant Syndrome A. Diaphoresis B. ↑ CK C. Rigidity D. Hypertenion E. ↑ HR

ANSWER C

Separation anxiety in most infants begins at A. 2 - 4 months B. 4 - 6 months C. 6 - 8 months D. 8 - 10 months E. 10 - 12 months

ANSWER C

Sudden cardiovascular collapse 48 hours post gastrectomy, Temp 39C, HR 130/min, BP 80 systolic, CVP 2cm above sternal angle. Cause? A. Anastomotic breakdown B. Acute gastric dilatation C. septicaemia D. haemorrhage E. PE

ANSWER C

The QT interval may be prolonged by each of the following EXCEPT A. high intra-thoracic pressure B. hypothermia C. magnesium sulphate D. suxamethonium E. volatile anaesthetic agents

ANSWER C

The average expected depth of insertion of an oral endotracheal tube, from the lip, in a normal newborn infant is A. 7.5 cm B. 8.5 cm C. 9.5 cm D. 10.5 cm E. 11.5 cm

ANSWER C

The commonest initial presenting feature in anaphylaxis is A. coughing B. desaturation C. hypotension D. rash E. wheeze

ANSWER C

The largest and most direct branch of the internal carotid artery is the A. ophthalmic B. anterior cerebral C. middle cerebral D. posterior cerebral E. choroidal

ANSWER C

The main rationale for using CPAP (continuous positive airway pressure) in the management of acute left ventricular failure is to A. increase the inspired concentration of oxygen (FiO2) B. recruit alveoli C. reduce the afterload D. reduce the preload E. reduce the work of breathing

ANSWER C

The normal haemoglobin (g.dl-1) for age during infancy is A. 16 at 3 months B. 12 at 3 months C. 10 at 3 months D. 8 at 3 months E. 9 at 12 months

ANSWER C

The normal range of the QRS axis in the ECG is A. 0 to +180 degrees B. 0 to +90 degrees C. -30 to +90 degrees D. -90 to +90 degrees E. -90 to +30 degrees

ANSWER C

The percentage of the population which is heterozygous as regards pseudocholinesterase, thus having a dibucaine number between 30 and 80, is A. 0.04% B. 0.4% C. 4.0% D. 14.0% E. 40.0%

ANSWER C

Tramadol A. may be used with caution in patients receiving monoamine oxidase inhibitors (MAOI) B. is useful in the treatment of narcotic withdrawal C. has no clinically significant effect on heart rate, left ventricular function or cardiac index at usual therapeutic doses D. use in patients on selective serotonin reuptake inhibitors has NOT been associated with signs of serotonin syndrome E. needs to be given in reduced doses to patients with severe hepatic disease, but not to patients with impaired renal function

ANSWER C

Transfusion related acute lung injury (TRALI) A. can be caused by all homologous blood components, but particularly FFP (fresh frozen plasma) B. is associated with significantly elevated pulmonary artery pressure C. is the commonest cause of morbidity associated with blood transfusion D. should be treated with high dose steroids E. typically presents 24 hours following transfusion

ANSWER C

Very sick patient on CVVHF. On norad, changed to adrenaline with no improvement in haemodynamic variables. What is your next step? A: change to another inotrope B: check their response to a synacthen test C: give hydrocortisone

ANSWER C

When intubating over a bougie / awake fibreoptic, which direction do you rotate the tube to stop it catching on structures in the glottis A. no change from normal B. 90 degrees clockwise C. 90 degrees counterclockwise D. 180 degrees E. try either direction

ANSWER C

When performing laryngoscopy using a Macintosh blade, your best view is of the patient's epiglottis touching the posterior pharyngeal wall. Using the Cormack and Lehane scale this is grade A. 1 B. 2 C. 3a D. 3b E. 4

ANSWER C

When stimulating the ulnar nerve with a nerve stimulator, which muscle do you see twitch? A. opponens abducens B. abductor pollicis brevis C. adductor pollicis D. extensor pollicis E. flexor pollicis brevis

ANSWER C

When stimulating the ulnar nerve with a nerve stimulator, which muscle do you see twitch? A. opponens abducens B. abductor pollicis brevis C. adductor pollicis brevis D. extensor pollicis E. flexor pollicis brevis

ANSWER C

Which of the following does NOT occur following bilateral lung transplant? A. Impaired mucociliary clearance B. Impaired lymphatic drainage C. Impaired HPV D. the response to increased pCO2 is lost

ANSWER C

Which of the following is most likely to be the first sign of hypoparathyroidism following total thyroidectomy? A. Carpopedal spasm B. Positive Chvostek's sign C. Circumoral tingling D. Tachycardia E. All of the above

ANSWER C

You are asked by an Obstetrician to help relax a uterus in labour and deliver for manual removal of placenta. What is a safe and effective dose of IV GTN to be delivered? A. 5 mcg B. 50 mcg C. 250 mcg D. 500 mcg E. 750 mcg

ANSWER C

You are called to see a labouring 24 year old primigravida with preeclampsia. She is convulsing. Following initial management, the best drug to prevent further convulsions is A. phenytoin B. diazepam C. magnesium sulphate D. carbamazepine E. hydralazine

ANSWER C

You are doing an awake fibreoptic intubation and having difficulty identifying the anatomy of where you are. Then you observe a trifurcation. The lobe of the lung to which this airway is connected is A. LUL B. Lingula C. RUL D. RML E. RLL

ANSWER C

You intubate a young male patient for a left thoracotomy with a 39FG Robert Shaw tube. When you inflate both cuffs and ventilate the bronchial lumen you get left sided ventilation. When you attempt to ventilate the tracheal lumen the pressures are very high and you get no air entry. Yet when you deflate both cuffs you can ventilate the patient through the tracheal lumen. The most appropriate step to take next is: A. Change to a 41FG tube B. Change to a 37FG tube C. Deflate both cuffs and insert further cm and recheck D. Deflate both cuffs and withdraw a few cm and recheck E. Pull ETT out and start again.

ANSWER C

You intubate a young male patient for a left thoracotomy with a 39FG Robertshaw tube. When you inflate both cuffs and ventilate the bronchial lumen you get left sided ventilation. When you attempt to ventilate the tracheal lumen the pressures are very high and you get no air entry. Yet when you deflate BRONCHIAL cuff you can ventilate BOTH left and right lungs through the tracheal lumen. The most appropriate step to take next is: A. Change to a 41FG tube B. Change to a 37FG tube C. Deflate both cuffs and insert further cm and recheck D. Deflate both cuffs and withdraw a few cm and recheck E. Pull ETT out and start again.

ANSWER C

You see a man in his 60s in clinic 1 week prior to laparoscopic cholecystectomy. He has dilated cardiomyopathy with an ejection fraction of 30%, but does not get dyspnoeic with normal activities of daily living. What is the most appropriate management of his heart failure? A. amiodarone 100mg bd B. digoxin 250mcg daily C. enalapril 2.5mg bd D. metoprolol 100mg bd E. diltiazem slow release 240mg daily

ANSWER C

Young asthmatic male in emergency department. RR 26, pCO2 27, SAO2 92%, struggling talking in sentences. Given nebulised salbutamol, and ipratropium, 200mg IV hydrocortisone. After 30 minutes - no improvement. Further management: A. IV salbutamol B. IV aminophylline C. IV magnesium D. Intubate and ventilate E. ???IV adrenaline?

ANSWER C

Satisfactory sedation after 0.5 mg.kg-1 of oral midazolam as a premedication in children usually occurs after A.5 minutes B.15 minutes C.25 minutes D.35 minutes E.45 minutes

ANSWER C "The dose of oral midazolam... usually results in a satisfactorily sedated child in approximately 10-15 min with a peak effect occurring at approximately 20-30 min, with minimal to no delay in recovery, even for brief procedures."

Clinical features of autonomic neuropathy include all of the following EXCEPT A. increased risk of gastro-oesophageal reflux B. postural hypotension C. pronounced sinus arrhythmia D. resting tachycardia E. silent myocardial ischaemia

ANSWER C -Hypertension -Painless MI -Orthostatic hypotension -Lack of heart rate variability with breathing (normal heart rate variability during voluntary deep breathing is > 10 beats/min) -Resting tachycardia -Early satiety (due to delayed gastric emptying) -neurogenic bladder -lack of sweating and impotence. -impaired thermoregulation (intraoperative hypothermia) -sudden death syndrome -gastroparesis -bladder atony -asymptomatic hypoglycaemia

During a laparotomy for resection of a liver tumour, a 25 kg 8-year-old has received a rapid transfusion of one unit of packed cells. Her central venous pressure (CVP) is now 5mmHg. The most likely cause of haemodynamic instability related to this transfusion is A. ABO incompatibility B. coagulapathy C. hyperkalaemia D. hypocalcaemia E. hypothermia

ANSWER C A. ABO incompatibility : FALSE B. coagulapathy : FALSE C. hyperkalaemia : TRUE, red cell lesion D. hypocalcaemia : packed cell contain no citrate. *PRBC contains dextrose, sodium chloride, mannitol and adenine *Platelets contains Sodium chloride, Sodium acetate trihydrate, Sodium citrate dihydrate, sSodium dihydrogenophospate, Disodium hydrogenophosphate, Potassium chloride, Magnesium chloride E. hypothermia: FALSE

A six year old boy requires return to theatre for arrest of post-tonsillectomy haemorrhage. When anaesthetising for this procedure it is important to: A. avoid sedative premedication prior to induction B. avoid volatile anaesthesic agents C. have duplicate suction apparatus and ETTs available D. use an uncuffed endotracheal tube in this age group E. RSI

ANSWER C A. Truish B. False C. True - CCEAP article "Before induction, in addition to the standard equipment, a selection of laryngoscope blades, smaller than expected tracheal tubes, and two suction catheters should be immediately available" D. False. Not necessarily. E. False - can do RSI or SV gas induction on side

Primary hyperparathyroidism, INCORRECT A. Hypercalcaemia B. Hypophosphataemia C. Hypochloraemic acidosis D. Increased alkaline phosphatase E. Raised urinary calcium

ANSWER C Hyperparathyoidism a. hight PTH b. high Ca c. low phosphate -85% primary hyperparathyroidism from ademoma of single parathyroid gland -15% from hypertrophy of all parathyroid glands -parathyroid malignancy is rare -Associated with MEN1 and MEN2 Presentation -50% asymptomatic -discovered incidently from hypercalaemia -may present with anorexia, dyspepsia, nausea/vomiting, constipation -hypertension, shorten QT, polyuria, polydipsia, renal calculi -depression, poor memory, drowsiness Diagnosis -elevated parathyroid hormone in the setting of hypercalaemia -other causes of hypercalaemia will suppress parathryoid hormone Action -PTH is released from the parathyroid glands. It acts principally to increase the plasma calcium level, principally by three mechanisms: 1. Increased osteolysis. Increased osteoclastic activity causes calcium to be released from the bones into plasma. More than 99% of body Ca is stored in bones. This release can occur within minutes. 2. Renal conservation of calcium. The kidneys reabsorb filtered calcium in the ascending loop, distal tubule and collecting tubules. Renal conservation occurs via two mechanisms: * Increased reabsorption of filtered calcium * Increased secretion of PO4. This causes decreased plasma PO4, hence increasing the unbound plasma calcium level. 3. PTH increases synthesis of 1,25 dihydrocholecalciferol. This in turn increases calcium absorption from the GI tract.

The signs of primary myxoedema except: A. Bradycardia B. Macrocytic anaemia C. Pretibial oedema D. Flattening of T wave E. Loss of hair

ANSWER C Myxoedema = Severe Hypothyroidism -rare form of decompensated hypothroidism -hypothyroidism in 2% women and 0.2% men -myxedema in 0.1% of all cases of hypothyroidism -mortality 15-20% -typically seen in elderly women -precipitated by infection, medication, environmental exposure or metabolic-related stresses ECG changes -bradycardia -prolonged QT -low P, T, QRS amplitude -complete heart block -BBB Treatment ABC approach 1. rehydrate with IV glocose and saline 2. inotropes and vasopressors if required 3. warming 4. IV liothronine 5. Hydrocortosone (100mg TDS) 6. ICU

A 5kg baby is 5% dehydrated. The body water deficit: A. 150mls B. 200mls C. 250mls D. 300mls E. 350mls

ANSWER C The % dehydrated is a % of total body weight. Five percent of 5,000g is 250g.

Baby with Tracheo-oesophageal fistula found by bubbling saliva and nasogastric tube coiling on Xray. BEST immediate management? A. Bag and mask ventilate B. Intubate and ventilate C. position head up, insert suction catheter in oesophagus (or to stomach?) D. Place prone, head down to allow contents to drain E. Insert gastrostomy

ANSWER C * A - FALSE. Not unless the baby is in respiratory distress and/or hypoxic. May inflate stomach by ventilating through fistula. * B - FALSE. Just because the baby has been diagnosed with TOF is not an immediate indication for intubation in and of itself. * C - TRUE. Neonates with TOF should have a "nasogastric" tube inserted into the oesophageal stump to drain secretions and prevent accumulation in the blind-end pouch. The NGT should be connected to continuous suction. The infant should be nursed prone or in the lateral position with 30 degrees head up tilt to decrease the risk of aspiration. See A Practice of Anesthesia for Infants and Children - 4th edition by Cote, Lerman, Todres; p.755. Saunders (2009) * D - FALSE. Can nurse prone, but lateral with head up tilt seems to be the recommended and most commonly cited method. * E - FALSE. Initial management as above (see C - TRUE), and then repair. Gastrostomy may be performed, but not best immediate management.

PZ119 ANZCA VERSION [Mar06] Q144, [Apr07], [Jul07] Fondaparinux Sodium (Arixtra) A. activates platelet B. cross reacts with sera from patients with heparin induced thrombocytopaenia C. has a mechanism of action that is antithrombin (ATIII) dependent D. is associated with thrombocytopaenia E. can be safely used in patients with severe renal failure

ANSWER C * A. activates platelet - false: "Fondaparinux does not inactivate thrombin (activated factor II) and has no effects on platelet aggregation." * B. cross reacts with sera from patients with heparin induced thrombocytopaenia - false: "It does not cross react with sera from patients with heparin induced thrombocytopenia." (MIMS online) * C. has a mechanism of action that is antithrombin (ATIII) dependent - true: "Fondaparinux is a synthetic and specific inhibitor of activated factor X (Xa) with no animal sourced components. The antithrombotic activity of fondaparinux is the result of antithrombin III (ATIII) mediated selective inhibition of factor Xa. By binding selectively to ATIII, fondaparinux potentiates (about 300 times) the innate neutralisation of factor Xa by antithrombin. Neutralisation of factor Xa interrupts the blood coagulation cascade and inhibits both thrombin formation and thrombus development." (MIMs Online) * D. is associated with thrombocytopaenia - false: "To date a causal association between treatment with fondaparinux and the occurrence of HIT has not been established." (MIMS online) * E. can be safely used in patients with severe renal failure - false: "Fondaparinux is almost completely excreted by the kidney as unchanged compound. The elimination half-life (t1/2) is about 17 hours in healthy young subjects and about 20 hours in healthy elderly subjects." Also "Arixtra is contraindicated in severe renal impairment" (Both from MIMS online)

PL25 ANZCA version [2005-Apr] Q114 Prilocaine is superior to lignocaine for intravenous regional anaesthesia because prilocaine A. exhibits no cardiotoxicity B. has a higher pKa C. has a larger volume of distribution D. is an ester, metabolised in the bloodstream E. is more highly protein bound

ANSWER C * A. exhibits no cardiotoxicity - false * B. has a higher pKa * C. has a larger volume of distribution - true o Prilocaine: "Distribution: Vd: 0.7-4.4 L/kg; crosses blood-brain barrier" o Lignocaine: "Distribution: Vd: 1.1-2.1 L/kg" * D. is an ester, metabolised in the bloodstream - false: Prilocaine is an amide anaesthetic. o "Prilocaine is a membrane stabilising agent and a local anaesthetic of the amide type" (Mims online) * E. is more highly protein bound - false o Prilocaine: "Protein binding: 55%" (uptodate) o Lignocaine: "Protein binding: 60% to 80% to alpha1 acid glycoprotein" (Uptodate)

Prilocaine is superior to lignocaine for intravenous regional anaesthesia because prilocaine A. exhibits no cardiotoxicity B. has a higher pKa C. has a larger volume of distribution D. is an ester, metabolised in the bloodstream E. is more highly protein bound

ANSWER C * A. exhibits no cardiotoxicity - false * B. has a higher pKa * C. has a larger volume of distribution - true o Prilocaine: "Distribution: Vd: 0.7-4.4 L/kg; crosses blood-brain barrier" o Lignocaine: "Distribution: Vd: 1.1-2.1 L/kg" * D. is an ester, metabolised in the bloodstream - false: Prilocaine is an amide anaesthetic. o "Prilocaine is a membrane stabilising agent and a local anaesthetic of the amide type" (Mims online) * E. is more highly protein bound - false o Prilocaine: "Protein binding: 55%" (uptodate) o Lignocaine: "Protein binding: 60% to 80% to alpha1 acid glycoprotein" (Uptodate)

Correct statements regarding expiratory-inspiratory flow-volume loops include all of the following EXCEPT A. in obstructive disease the expiratory curve has a scooped out or concave appearance B. in restrictive disease expiratory flows are usually decreased in relation to lung volume C. in restrictive disease the expiratory curve has a convex appearance D. the expiratory curve is largely effort independent E. the inspiratory curve is effort dependent

ANSWER C * A. in obstructive disease the expiratory curve has a scooped out or concave appearance - true * B. in restrictive disease expiratory flows are usually decreased in relation to lung volume - some people are saying this is incorrect: Expiratory flows are decreased, as are lung volumes... this seems to be true to me. It would also make sense because in restrictive disease, the compliance of the lungs is decreased, so exp flow (a passive process) would be decreased more so than the actual lung volumes. * C. in restrictive disease the expiratory curve has a convex appearance - this sounds the most incorrect to me: the shape of the curve is similar to a normal flow-volume loop, just with a smaller magnitude * D. the expiratory curve is largely effort independent - true * E. the inspiratory curve is effort dependent - true

In elderly patients A. opioid requirements are decreased, primarily due to age-related changes in physiology B. pain thresholds are decreased C. self-rated pain scores are lower than in younger patients D. there is a decrease in the density of unmyelinated but not myelinated nerve fibres E. there is impairment of pain inhibitory systems

ANSWER C * A. opioid requirements are decreased, primarily due to age-related changes in physiology - true: "Older patients require less opioid than younger patients to achieve the same degree of pain relief (Macintyre & Jarvis, 1996 Level IV; Woodhouse & Mather, 1997 Level IV; Gagliese et al, 2000 Level IV; Upton et al, 2006), however, a large interpatient variability still exists and doses must be titrated to effect in all patients. The decrease is much greater than would be predicted by age related alterations in physiology and seems to have a significant pharmacodynamic component" AND "The physiological changes associated with ageing are progressive. While the rate of change can vary markedly between individuals, these changes may decrease the dose (maintenance and/or bolus) of drug required for pain relief and may lead to increased accumulation of active metabolites" (Acute pain book 3rd ed) * B. pain thresholds are decreased - false and true: "Examples of differences in reports of acute pain are commonly related to abdominal pain (eg associated with infection, peptic ulcer, cholecystitis, or intestinal obstruction) or chest pain (eg myocardial ischaemia or infarction; pneumonia) and are in general agreement with the experimental finding of increased pain thresholds in the older person" BUT "Experimental pain thresholds to a variety of noxious stimuli are altered in older people; there is also a reduction in tolerance to pain" * C. self-rated pain scores are lower than in younger patients - perhaps: "Older men undergoing prostatectomy reported less pain on a present pain intensity scale and McGill Pain questionnaire (but not a visual analogue scale [VAS]) in the immediate postoperative period and used less PCA opioid than younger men undergoing the same procedure (Gagliese & Katz, 2003 Level III-2). In a study of pain following placement of an IV cannula (a relatively standardised pain stimulus), older patients reported significantly less pain than younger patients" AND "Reported frequency and intensity of acute pain in clinical situations may be reduced in the older person" * D. there is a decrease in the density of unmyelinated but not myelinated nerve fibres - false: "The peripheral nerves show a decrease in the density of both myelinated and, particularly, unmyelinated peripheral nerve fibres, an increase in the number of fibres with signs of damage or degeneration and a slowing of the conduction velocity" * E. there is impairment of pain inhibitory systems

Regarding tracheobronchial disruption following chest trauma - Which of the following is true? A. Haemoptysis is the most common symptom B. Should all have double lumen tubes C. Dyspnoea is the most common symptom D. Stridor is the most common symptom E. Mediastinal emphysema on CXR confirms diagnosis if suspected

ANSWER C * A: "Haemoptysis occurs in 25% of cases and is usually due to mucosal tear" * B: "Several choices of endotracheal tubes can be used for isolation of TBI" (tracheobronchial injury). Double lumen tubes have the remote possibility of extending the lesion because of their size * C: "Dyspnoea occurs in more than 60% or patients". * D: "Stridor, however, is not a prominent feature of TBI" * E: "Radiological findings in TBI are mostly non specific", "Direct inspection of the tracheobronchial tree is the most definitive method to diagnose TBI" Trauma divided into penetrating vs non-penetrating (ie blunt) Usual mechanism of TBI is a shearing force, with blunt trauma to the chest. This would typically occur with a deceleration type injury. Clinical signs of TBI: * Dyspnoea * Cough * Haemoptysis * Cyanosis * Subcutaneous emphysema * Tracheal shift * Persistent pneumothorax following successful placement of ICC * Signs of airway obstruction

Which of the following statements regarding the use of epidurally administered adjuvant drugs with epidural analgesia for acute postoperative pain is FALSE? A. adrenaline added to the local anaesthetic improves thoracic epidural analgesia B. clonidine added to epidural opioids improves analgesia C. clonidine prolongs the effects of epidural local anaesthetics D. ketamine added to opioid based epidural analgesia improves analgesia E. neostigmine combined with an epidural opioid reduces the dose of opioid required

ANSWER C * A: True (distinction between thoracic vs lumbar epidural) * B: True ("the addition of clonidine to PCEA with ropivacaine and morphine after total knee arthroplasty decreased opioid requirements and improved analgesia") * C: False (most of the info about clonidine is regarding intrathecal rather than epidural, and that seems to be the distinction here. It prolongs intrathecal block, but no reference made of effect on duration of epidural, so this option is false by a process of elimination as all others are true) * D: True ("Epidural ketamine...added to opioid-based epidual analgesia regimens improves pain relief") * E: True ("Epidural neostigmine combined with an opioid reduces the dose of epidural opioid that is required")

PL08 Bupivacaine ('Marcaine'): A. Possesses an ester type chemical formula B. Is unsuitable for paediatric caudal anaesthesia C. Is biotransformed in the liver D. Has poor plasma binding properties E. Has a shelf-life of one year

ANSWER C * Aminoamide local anaesthetic (NOT an ester) * Higher toxicity so better to use something else for caudal. Small but significant risk of IV injection with caudal injection. * Metabolised in the liver (as are all the amide LAs) * Lipid soluble -> high plasma protein binding Amides have a shelf life of up to 2 yrs

Spinal anesthesia in infants A. eliminates the risk of postoperative apnea B. has a lower failure rate than in adults C. lasts for a shorter time than in adults D. may be performed at a higher level than in adults E. often causes hypotension if the infant is awake

ANSWER C * Proven to reduce risk of apnoeas * Failure rate of 20% * Duration 20-60 minutes (the answer) * Spinal cord terminates more inferiorly (L3)

Cerebrospinal fluid: A. Lower sodium then plasma B. Pressure ? 8-10 mmHg in normal supine person C. Hypochloraemia in tuberculous meningitis D. Glucose level rises with bacterial meningitis

ANSWER C * Sodium is 144-152mmol/L * ICP is measured in millimeters of mercury (mmHg) and, at rest, is normally less than 10-15 mmHg. * Low CSF chloride is classic of TB meningitis. * glucose is normal in viral meningitis but decreased in all other causes. CSF composition -Reference values for CSF are as follows: * Protein 0.15-0.45 g/L Electrolytes: * Osmolality 280-300 mmol/L * Sodium 135-150 mmol/L * Potassium 2.6-3.0 mmol/L * Chloride 115-1 30 mmol/L * Carbon dioxide 20-25 mmol/L * Calcium 1.00-1 .40 * Magnesium 1.2-1.5 mmol/L * Lactate 1.1-2.4 mmol/L * pH 7.28-7.32 * PCO2 44-50 mm Hg * PO2 40-44 mm Hg Other constituents: * Creatinine 50-110 umol/L * Glucose 2.8-4.4 mmol/L * Iron 0.2-0.4 umol/L * Phosphorus 0.4-0.6 umol/L * Urea 3.0-6.5 mmol/L A comparison of the composition of CSF and plasma reveals that: 1. CSF proteins are -1 % that of plasma; 2. CSF calcium levels are -50% that of plasma; 3. CSF glucose levels are -60% that of plasma; 4. CSF chloride and magnesium levels are higher than plasma; 5. CO2 diffuses rapidly, and HCO3 slowly, from the plasma into the CSF.

25 yo primip ?38/40 gestation with beta thalassemia trait for epidural. BP 140/95, mild proteinuria ...something else... Best test before you will put in epidural A. Coagulation screen B. Hb C. Platelet count D. skin bleeding time E.

ANSWER C * Thalassemia trait is a red-herring. No effect on clotting/epidural placement. Money is on pre-eclampsia Main thing to look at is the platelet count. Answer is C. * See Oxford Handbook 2nd edn - p.744. - If plt>100, proceed. - If plt<100, do coags. - If plt 80-100, and coags normal - regional is OK.

Ventricular fibrillation (VF) following caudal anaesthesia in 20kg six year old child. The recommended dose of of Intralipid 20% is: A. 10mls B. 20mls C. 30mls D. 40mls E. 50mls

ANSWER C * Weight = (Age(y) + 4) x 2 which is 20kg * Dose of Intralipid 20% is 1.5 ml/kg = 30ml

Propofol A. is contra-indicated in all patients who give a history of egg allergy B. requires a higher dose, when given slowly for induction of anaesthesia C. may be diluted with 5% dextrose solution D. causes no pain on injection if 20 mg of lignocaine is pre-mixed with 200mg prior to intravenous injection E. has a metabolic half-life of 8 minutes

ANSWER C A False B False C True - according to PI D False - pain on injection occurs in <10% of patients and the incidence can be decreased by potent short acting opioid or lignocaine 1% E False - elimination t1/2 is 0.5-1.5 hours

The knee jerk is A. mediated through spinal segments L4-6 B. mediated through receptors in the patellar tendon C. a monosynaptic reflex D. diminished in a patient with an upper motor neuron lesion

ANSWER C A. False - L2-4 B. False - Quadriceps femoris C. True D. False - Increased

A 50 year old diabetic man is admitted to intensive care with pneumonia. Intubated and ventilated. (Extensive results given): BP 80/-, HR 120, CVP 4, PCWP 6, SvO2 69% PaO2 80, BE -4 pH 7.2. Management: A. Blood transfusion B. Bicarbonate infusion C. Fluid resuscitation D. Adrenaline infusion E. Insulin infusion

ANSWER C He's hypotensive, tachycardiac, has low CVP and PCWP consistent with hypovolaemia. Acidosis and BE might be related to DM or lactate. Some intitial thoughts: * You wouldn't give blood unless indicated by low [Hb] or active bleeding * A bicarbonate infusion is contraindicated in DKA; or at least it doesn't help * Fluid resuscitation - Hypotension and tachycardia consistent with need to give fluids. If DKA, then expect significant fluid deleted. * Adrenaline infusion - not initially. * Insulin - Definitely if hyperglycaemia & DKA (Infection is major cause of "metabolic decompensation" - i.e. DKA - in diabetics. No plasma glucose in results above. The pH and BE result don't indicate a severe DKA so check blood for glucose. Also urine for glucose and ketones.

The peak effect of oral midazolam as a premedication in children occurs after A. 10 - 20 minutes B. 20 - 30 minutes C. 30 - 40 minutes D. 40 - 50 minutes E. 50 - 60 minutes

ANSWER C "The dose of oral midazolam... usually results in a satisfactorily sedated child in approximately 10-15 min with a peak effect occurring at approximately 20-30 min, with minimal to no delay in recovery, even for brief procedures."

A young man following a motor vehicle accident is lying in respiratory distress on the roadside. He has a penetrating chest wound. Which ONE of the following measures would most benefit the man? A. Mouth to mouth respiration B. Tracheostomy C. Seal the wound during expiration D. Lie the man in a lateral posture with the wounded thorax lowermost

ANSWER C "Treatment of sucking chest wounds in the field begins with coverage using a sterile occlusive dressing. The dressing is taped on three sides so that it can act as a one-way valve, allowing air to exit the chest during exhalation, but preventing air entrainment during inhalation. Alternatively, commercially manufactured "Heimlich valves" can be placed. If the patient's state deteriorates after placing the dressing, it must be removed immediately. In the case of a large open wound that cannot be occluded, the patient must be intubated and ventilated to survive the injury"...Trama, Vol 1, p 143

Serious post-operative epidural infection A. is rarely due to Staphylococcal species B. is associated with epidural catheter disconnection C. occurs with an incidence in the range 1-2 per 10,000 D. is usually reported in obstetric cases E. mandates surgical drainage if an abscess is present

ANSWER C # Incidence 0.2 - 1.2 per 10000 hospital admissions # Staph > 90% # Conservative management if: * Poor surgical candidate * Abscess over too many segments * No spinal cord symptoms * Complete paralysis > 3 days

The signs of exposure to a nerve agent such as Sarin or VX include A. bronchodilation B. dry skin C. muscle fasciculation D. pupillary dilatation E. tetany

ANSWER C # People exposed to a low or moderate dose of VX by inhalation, ingestion (swallowing), or skin absorption may experience some or all of the following symptoms within seconds to hours of exposure: * Runny nose * Watery eyes * Small, pinpoint pupils * Eye pain * Blurred vision * Drooling and excessive sweating * Cough * Chest tightness * Rapid breathing * Diarrhea * Increased urination * Confusion * Drowsiness * Weakness * Headache * Nausea, vomiting, and/or abdominal pain * Slow or fast heart rate * Abnormally low or high blood pressure # Even a tiny drop of nerve agent on the skin can cause sweating and muscle twitching where the agent touched the skin. # Exposure to a large dose of VX by any route may result in these additional health effects: * Loss of consciousness * Convulsions * Paralysis * Respiratory failure possibly leading to death

The following methods can be used to prevent postoperative hypoxia in patients having upper abdominal surgery: A. Avoid hyperventilation and hypocarbia B. IPPV with PEEP C. Sit up postoperatively D. Continuous epidural postoperatively E. Increased inspired oxygen

ANSWER C * Likely causes of hypoxia: atelectasis and FRC<CC à shunt and decreased V/Q units. * Risk factors include: age, body habitus, pulmonary disease, general anaesthesia rather than regional, supine/trendelenberg, * Best prevented by maintaining a clear airway, supplemental oxygen and ensuring adequate ventilation. * Intraoperative manoeuvres to prevent atelectasis: benefits of PEEP are quickly lost, high FiO2 increase atelectasis, * Postoperative treatment of atelectasis: physiotherapy (deep breathing exercises, incentive spirometry, intermittent PPV etc) have questionable evidence according to review in Chest 2001 but the review on atelectasis 2005 says they are the most important. Simply changing from supine to seated can improve gas exchange. Sternal or thoracic traction and aminophylline infusions have been used (with effect). (Atelectasis review 2005) Epidural anaesthesia increases FRC (despite intercostal paralysis) because of caudad diaphragm displacement and decreased central blood volume. See: Human chest wall function during epidural anesthesia * Hyperventilation and hypocarbia cause apnoea which can cause hypoxia.

The magnesium concentration at which loss of deep tendon reflexes typically occurs is: A. 2 mmol/l B. 3.5 mmol/l C. 5 mmol/l D. 8 mmol/l E. 12 mmol/l

ANSWER C 0.8 - 1.0 : Normal physiological range 1.7 - 3.5 : Therapeutic 2.5 - 5.0 : Early ECG changes, widen QRS, increase PQ 5.0 - loss of deep tendon reflexes 7.5 -SA and AV block, respiratory paralysis 12 - cardiac arrest

Liposuction. Infiltration of lignocaine with 1:200,000 adrenaline. Peak plasma concentration of lignocaine occurs at: A. 1 hour B. 3 hours C. 18 hours D. 24 hours E. 30 mins

ANSWER C 5. Tumescent anesthesia: This is a technique of local anesthesia which involves the subcutaneous injection of large volumes of dilute LA in combination with epinephrine and other agents. It is used for dermabrasion, skin grafting, rhinophyma correction, liposuction and hair transplant procedures. The injection consists of 1000 ml of normal saline, 50 ml of 1% lidocaine, 1 ml of 1:1000 adrenaline and 12.5 ml of sodium bicarbonate (1 mEq/L). Total doses of lidocaine range between 35-55 mg/kg, and the plasma concentrations may peak more than 8-12 hours after infusion.[13] Clinicians are advised to exercise great caution in administering additional local anesthesia by infiltration or other routes for at least 12-18 hours following the use of this technique. Burk RW 3rd, Guzman-Stein G, Vasconez LO. Tumescent anesthesia with a lidocaine dose of 55 mg/kg is safe for liposuction. Dermatol Surg 1996:22:921 Back to cited text no. 13

Investigation of a suspected anaphylactic reaction requires measurement of tryptase levels. Correct statements regarding tryptase include each of the following EXCEPT A. 99% of body tryptase is in mast cells B. a concentration of greater than 20 ng/mL suggests an anaphylactic reaction C. blood samples should be repeated 24 to 48 hours after the reaction D. maximum blood concentrations occur within 1 hour of the reaction E. tryptase concentrations rise after both anaphylactic and anaphylactoid reactions

ANSWER C A - 99% of body tryptase is in mast cells - true B - concentration >20ng/ml suggests anaphylaxis - true C - blood samples at 24-48 hours - false - hence the answer as 6-24hrs is best D - max blood concs within 1 hour - true (perhaps slightly tempered as max conc within 1 hour of degranulation and question says within 1 hour of reaction, but the reaction is due to degranulation of course . . . E - tryptase up after anaphylaxis and anaphylactoid - true

Sickle cell anaemia: A. Occurs in people of central European descent B. Sex-linked inheritance C. Decreased Hb solubility at low pO2 D. OxyHb dissociation curve shifted to left

ANSWER C A - FALSE * Central Africa - 25 % carry the allele o SCD originated in West Africa, where it has the highest prevalence * Also occurs in parts of India and the Mediterreanean (Central Europe is typically Germany / Austra / Czech Rep...but this may be just a poorly worded question) B - FALSE * Autosomal C - TRUE * HbS polymerizes at low PaO2 o Also ppt. by cold, acidosis, infection, dehyadration D - FALSE * OxyHb curve shifted right (P50=31mmHg)...

The pain of chronic herpetic neuralgia is best controlled by A. ipsilateral stellate ganglionectomy B. intrathecal alcohol injection C. analgesic drugs D. dorsal rhizotomy E. topical capsaicin

ANSWER C A - False Stellate ganglion block is used as a treatment for CRPS. Is ALSO used in post-herpetic neuralgia, though not obviously the first or best choice. CRPS is defined by the IASP as follows: CRPS Diagnosis triad * Allodynia, Hyperalgesia or pain after original injury has passed * Oedema and vasomotor instability * Absence of other causes B - False. C - Best answer The drug of choice for post-herpetic neuralgia is controversial. D - False. This is the treatment for intractable pain, and is not without risk. There are better and less invasive options available. E - True but not best answer. Capsaicin has been used for treatment of post-herpetic neuralgia. Herpes Zoster-associated pain -reactivation of varicella-zoster virus -dormant in DORSAL root and cranial nerve ganglia -increased risk with increasing age and diseases and drug which impair immunity Characteristics -80% prodromic pain (burning, throbbing, shooting, dysaesthesia and allodynia) -rash -self limiting Post-herpetic neuralgia -pain persists > 3 months -followed by postherpetic neuralgia Prevention -live attentuated VZV vaccine reduces incidence of 1. HZ infection 50% 2. PHN 65% -recommended for all person >60 yo Treatment 1. antiviral agents started within 72 ours of onset of herpes zoster rash accelerate the resolution of acute pain -does not reduce the incidence of postherpatic neuralgia 2. Multimodal anaglesia with regarular paracetamol with oxycodone or tramadol as required. 3. Corticosteriods : little effect on pain intensity or skin healing 4. Anticonvulsants : single dose of gabapentin reduced acute pain intensity and aloodynia in first 6 hour -no difference in long term 4.Amitriptyline (used in low doses for 90 days from onset of the herpes zoster rash) reduces the incidence of postherpetic neuralgia 6. Topical aspirin, topical lignocaine patch or oxycodone controlled release, provide analgesia in herpes zoster

Increased bleeding risk is associated with all of the following EXCEPT A. aspirin B. hypothermia C. lupus anticoagulant D. renal failure E. von Willebrand's disease

ANSWER C A - True B - True C - False (lupus antiphospholipid → hypercoaguable) therefore an increased clotting risk, not an increased bleeding risk D - True E - True

A 35yr old African-American with sickle cell and fractured ankle for ORIF. Hb 90, Haematocrit 0.3. A: Transfuse 2 units packed cells (?pre-op) B: Let him cool passively to low/normal temperature C: Spinal is safe D: Avoid thiopentone E: Tourniquet is absolutely contra-indicated

ANSWER C A - WRONG: "Although widely practiced, prophylactic erythrocyte transfusion remains a treatment with appreciable complications whose potential benefits have not been clearly demonstrated by a prospective, randomized clinical trial". Also a study in orthopaedic patients "did not detect a prophylactic effect from preoperative transfusion" (Sickle Cell Disease and Anesthesia. Anesthesiology 2004; 101:766-85) B - WRONG: "Although hypothermia would tend to retard sickling because of a left shift of the oxygen dissociation curve, hypothermia is often identified as a precipitant of perioperative SCD complications". "avoidance of patient hypothermia is a basic objective for most anesthetics" (Same reference) C - CORRECT: "The use of regional anesthesia therefore does not appear to be contraindicated in SCD" (Same reference) and OHA p202 agrees D - ??? but seems unlikely E - WRONG according to OHA p202 -JC

The first heart sound on auscultation is A. characteristically quiet in mitral stenosis B. due equally to mitral and tricuspid valve closure C. louder than usual with a premature beat D. loudest at the left sternal edge E. unchanged in mitral regurgitation

ANSWER C A - false Loud S1 in MS because ↓ valve orifice → impaired LV filling ∴ mitral valve wide open when systole starts → valve snaps shut loudly B - false - with the higher L-sided pressures the mitral valve closure contributes more to S1, than does tricuspid closure C - true - akin to MS, the early arrival of ventricular contraction before ventricular filling tapers off means the valve is widely open and will make a louder sound upon closure with the VEB D - false - fitting w the MV contributing more to the S1, the ideal place to auscultate is the apex (MCL, 5th ICS) E - false - soft or absent

PZ120 [Mar06] Q134 Gabapentin used for acute postoperative pain A. has its analgesic effect reduced by concurrent use of COX-2 inhibitors B. increases anxiety if administered preoperatively C. may require high doses to be effective D. reduces the incidence of dizziness when compared with opioids alone E. works by binding at GABA receptors

ANSWER C A - sounds like nonsense B - incorrect, it is an anxiolytic. C - Doses of 300mg reduced morphine consumption post-op in this study Acta Anesthesiogica Scandanavica Volume 48 Issue 3, Pages 322 - 327. However the same (and clearly the ref for this Q) BJA editorial states "Unfortunately, it may be that smaller doses are ineffective." D - When used with PCEA in the BJA study (ref below) "There was a significant increase in the incidence of dizziness (35% vs 5%) and a non-significant increase in somnolence (25% vs 10%)". But no mention of PCA. Could it be the examiners did not read the article poperly? E - BJA ref below says this is not its bag, "despite its name gabapentin does not bind at the GABAA or GABAB receptor."

In Wolff-Parkinson-White (WPW) syndrome, A. the delta wave is caused by a delay in conduction in the accessory pathway B. the PR interval is of normal duration but the QRS complex is widened C. a narrow complex supraventricular tachycardia (SVT) is the most common form of arrhythmia D. central venous cannulation is unlikely to trigger SVT E. cardioversion of arrhythmias is seldom effective

ANSWER C A : False. Accessory pathway is not delayed, it is fast (ventricular preexcitation) B : False. Short PR. Apparent long QRS as P wave slurs into the QRS complex via delta wave C : True. So-called 'orthodromic' tachycardia via the rapid normal pathway (with retrograde conduction via the accessory pathway) is commonest. D : FALSE. CVC insertion is not likely to cause SVT but it could "theoretically predispose" to the the arrhythmia according to Stoelting's textbook. E : False. Cardioversion or procainamide are standard therapies for antidromic (broad-complex) tachycardia. Can also be used for orthodromic tachycardia.

Carbon dioxide is the most common gas used for insufflation for laparoscopy because it A. is cheap and readily available B. is slow to be absorbed from the peritoneum and thus safer C. is NOT as dangerous as some other gases if inadvertently given intravenously D. provides the best surgical conditions for vision and diathermy E. will NOT produce any problems with gas emboli as it dissolves rapidly in blood

ANSWER C A : although CO2 is cheap and readily available, it is not the reason we use CO2. B : CO2 is absorbed quickly across peritonium, resulting in high PaCO2 and respiratory acidosis C : CO2 is highly soluable in blood, therefore gas emolization is less severe and with treatment rapid dissolution of emboli. He, Air and N2 dissolve less rapidly and carry a higher risk of lethal venous emobli. D : false, argon or helium would be better. E : false, it will produce problems, but severity will be less and emobli dissolve quicker, CO2 is 50 times more soluable then He. Ideal insufflating agent : colorless, non combustable, non explosive, physiolgocially inert and readily soluable HELIUM and N2 Pro : inert, no acid-base effects Cons : poor blood gas solubility making embolism life threatening, require special inline set up N20 Pro : less post op pain, well tolerated hemodynamically Con : combustion ARGON Pro : inert, slightly less soluable than CO2 Con : rare gas, difficult set up

MM Carcinoid syndrome - finding on examining heart: A. Fine inspiratory crepitations B. Systolic murmur at apex C. Systolic murmur at left sternal edge D. Murmur at apex with opening snap E. Pericardial rub

ANSWER C A carcinoid tumor is a tumor that secretes large amounts of the hormone serotonin. These tumors usually: * arise in the gastrointestinal tract and * from there may migrate (metastasize)to the liver. Carcinoid tumors also sometimes develop in the lung. Only about 10% of the people with carcinoid tumors will develop the carcinoid syndrome. Major symptoms of this syndrome include: * hot, red facial flushing, * diarrhea and * wheezing. Carcinoid syndrome occurs when the tumors produce excessive amounts of serotonin or other substances. The presence of carcinoid syndrome suggests: * the presence of liver secondaries * a non-GI carcinoid tumour * massive mediator release by tumour The liver normally breaks down the mediators (vasoactive compounds) secreted by the tumour. As most carcinoid tumours are in the GIT, the mediators are released into the portal veins, and broken down in the liver. To cause carcinoid syndrome then indicates the liver is being bypassed or overwhelmed. Carcinoid Heart Disease * Develops in 50% of patients with carcinoid syndrome o hence in about 5% of people with a carcinoid tumour * It results in fibrosis of myocardial tissues, especially on the Right side of the heart o tricuspic regurgitation is the most common finding o pulmonary regurg and/or stenosis also occurs o less than 10% of patients with carcinoid have mitral or aortic regurg o constrictive pericarditis has also been described

You are on a humanitarian aid mission in the developing world. Drawover vaporiser apparatus described being used. Given 400 mm tubing, OMV or diamedica vaporiser, 200mm tubing attached to self-inflating bag. Which ONE other piece of equipment is ESSENTIAL to make this system functional? A. Halothane B. In-line Waters' Cannister C. Non-rebreathing valve D. Oxygen source E. Ventilator

ANSWER C A drawover system (figure 2) is designed to provide anaesthesia without requiring a supply of compressed gases. Atmospheric air is used as the main carrier gas and is drawn by the patient's inspiratory effort through the vaporizer, where the volatile agent, normally ether or halothane, is added. The mixture is then inhaled by the patient via a non-rebreathing valve. Features of drawover apparatus: 1. Robust, compact and portable 2. Low purchase price and running costs 3. Straightforward maintenance 4. Not dependent on compressed gases%%//d3cgb598vs7bfg.cloudfront.net/images/upload-flashcards/back/5/9/49395192_m.jpg

A 20 kilogram child has a haemoglobin of 60 g.l-1. The child is normovolaemic and there is no on-going blood loss. The volume of packed cells required to increase the haemoglobin level to 100 g.l-1 is A. 80 ml B. 160 ml C. 320 ml D. 500 ml E. 750 ml

ANSWER C A simple solution is to use Frank Shann's equation: 4ml/kg of packed cells will increase Hb by 10g/L. Thus 4x20x4 = 320 mls (which is option C).

Paralysed with atracurium. TOF is 1(25%). You give a dose of 0.1 mg/kg mivacurium to close the abdomen. When will you be back to TOF 1(25%)? A. 5 min B. 10 min C. 30 min D. 60 min E. 90 min

ANSWER C A study by Naguib et. al. showed that after an initial intubating dose of atracurium, and spontaneous recovery of the first twitch (T1) to 10% of its control height, a maintenance dose of 0.1mg/kg of mivacurium resulted in a time of 25 minutes to regain T1=10%. My guess would be the answer is C, but it's still a guess

Cauda equina syndrome A. involves large sensory fibres initially B. is not associated with back pain C. results from compression of nerve roots D. results in loss of sensation confined to the foot E. results predominantly in an upper motor neuron lesion

ANSWER C A- False: Usually affects small nerve fibres first, ie autonomic nerves. B False: Is characteristically associated with low back pain. C True: Spinal cord finishes at L1-2 D False: Nerve root distribution, mostly saddle/perineal anaesthesia E False: LMN Lesion, see below

Black bank March 2011 ET02 An 85y.o for open AAA repair. Refuses blood because of risk of vCJD. You tell him you won't anaesthetise him as the risk is too high. This is an example of: A: Autonomy B: Beneficence C: Malevolence D: Coercion E: Paternalism

ANSWER C A. Autonomy would be to respect the patients decision to refuse blood and continue with the case after discussing the risks, optimising and considering alternatives B. FALSE C. TRUE : Maleflicence : first do no harm D. FALSE : Coercion would be to pursuade him to accept blood E. FALSE : Paternalism would be to anaesthetise him and given him blood regardless of his views.

With regard to fire in OT A. Mainly caused by laser surgery B. Decreased incidence since cessation of use of cyclopropane and ether C. Need fuel, ignition source and oxidizing agent D ? E. ?

ANSWER C A. FALSE B. FALSE : drastic fall in explosion, but not fires C. TRUE : fire triad : oxidizer, ignition, fuel Diathermy 70% and Laser 10% of OT fires

Epidural blood patch for severe post-dural puncture headache A. is contraindicated in patients with Acquired Immunodeficiency Syndrome (AIDS) B. has NOT been shown to be associated with a higher success rate if performed more than 24 hours after dural puncture C. is associated with a higher success rate if more than 20 ml of blood is used D. is rarely associated with back pain during injection E. is most effective when given immediately following accidental dural puncture

ANSWER C A. FALSE - HIV NOT a contraindication, unless active bacterial or viral infection B. FALSE - see above C. TRUE - best results with 20-30ml blood D. FALSE - it's always associated since that's your indication to stop injecting more blood! E. FALSE - More effective if delayed by >24 hours

You are seeing a 60yo man in the pre-anaesthetic clinic before his right total knee replacement. He weighs 70kg and apart from his osteoarthritis is fit and well. You discuss with him the options of a general anaesthetic with multi-modality analgesia and enoxaparin postoperatively as well as the option of an epidural for both the anaesthetic and post operative pain management. What is incorrect regarding the epidural? A. It will shorten his hospital stay and accelerate his rehabilitation B. It will give him better pain relief particularly for the CPM machine (the continuous pain machine) C. It will reduce his risk of myocardial ischaemia D. There will be little difference in his risk of thromboembolism. E. If he has no sedation, his risk of post-operative delirium and cognitive impairment will be reduced

ANSWER C A. FALSE : B. FALSE : After hip or knee replacement, epidural analgesia provides better pain relief than parenteral opioids, in particular with movement (Choi et al, 2003 Level I). C. TRUE : Only thoracic epidruals have been should to reduce the risk of AMI D. FALSE : There is little difference in risk for thromboembolism E. FALSE :

The standard error of the mean is A. dependent for its validity on a normal distribution in the population B. an indication of the likelihood of making a type II error C. about 2, if the standard deviation is 15 and the sample size 50 D. NOT necessary for calculating the confidence interval for the mean E. the variance of the population of sample means

ANSWER C A. FALSE : The formula for SEM (=SD/ square root n) does not assume a normal distribution. However, many of the uses of the formula do assume a normal distribution. SEM will be higher in non-normal distributions, but just as valid. B. FALSE C. TRUE : SEM=SD/SQRT N for large samples, for small samples use the t-distribution D. FALSE : 95% CI =mean +/- 1.96 SEM E. FALSE :SD is a measure of variability of results from the mean, whilst SEM is a measure of precision and relates the sample mean to the population mean. It is the standard deviation of the means of multiple samples, but not the variance, since this is the square of SD

In patients with mitral regurgitation A. left ventricular damage does NOT occur before the onset of symptoms B. mitral valve replacement has a lower operative mortality and incidence of late adverse outcomes compared to mitral valve repair C. an ejection fraction of less than 60% may be an indication for surgery, even in the absence of symptoms D. long term vasodilators have been shown to delay the need for surgery E. the presence of a third heart sound always indicates heart failure

ANSWER C A. FALSE : Unlike stenotic cardiac valve lesions, regurgitant cardiac valve lesions often progress insidiously, causing left ventricular damage and remodeling before symptoms have developed B. FALSE : Mitral valve repair is preferred to mitral valve replacement because it restores valve competence, maintains the functional aspects of the mitral valve apparatus, and avoids insertion of a prosthesis C. TRUE : Survival may be prolonged if surgery is performed before the ejection fraction is less than 60% or before the left ventricle is unable to contract to an end-systolic dimension of 45 mm (normal < 40 mm). Symptomatic patients should undergo mitral valve surgery even if they have a normal ejection fraction D. FALSE : Although vasodilators are useful in the medical management of acute mitral regurgitation, there is no apparent benefit to long-term use of these drugs in asymptomatic patients with chronic mitral regurgitation E. FALSE : S3 can be heard and recorded in healthy young adults. However, it is usually abnormal in patients over the age of 40 years, suggesting an enlarged ventricular chamber

A patient with pulmonary hypertension secondary to lung disease presents for laparotomy. The anaesthetic technique LEAST likely to exacerbate the pulmonary hypertension is endotracheal intubation and A. controlled ventilation with isoflurane, N2O and O2 B. spontaneous breathing with isoflurane and O2 / air C. controlled ventilation with total intravenous anaesthesia and O2 / air D. controlled ventilation with ketamine and O2 / air E. controlled ventilation with high dose opioids, N2O and O2

ANSWER C A. FALSE : addition of N2O increases PVR and RAP B. FALSE : aim to prevent hypoxia and hypercarbia, spontaneous ventilation will not achieve this C. TRUE : TIVA is acceptable, controlled ventilation is important D. FALSE : ketamine increases PVR especially in spontaneous ventilation E. FALSE

With regard to accidental electrocution, which of the following statements is true? A. all electrical equipment in the operating room should be earthed B. risk of ventricular fibrillation increases with increasing current frequency C. risk of ventricular fibrillation is greater with alternating current (cf. direct) D. risk of electrocution is reduced by earthing the patient E. use of battery operated equipment avoids the risk of ventricular fibrillation

ANSWER C A. FALSE : electrical equipment in OR should have isolated circuit B. FALSE : between 30-100Hz C. TRUE D. FALSE : patients should not be earthed E. FALSE

Features of anaphylactoid reactions to anaesthetic drugs include: A. known but rare reactions to inhalational agents B. a lower frequency of occurrence and severity following pretreatment with H1 and H2 receptor blockers C. difficulty in establishing that a colloid plasma volume expander is the causative agent D. rapid synthesis and release of histamine from basophil and mast cell membranes E. lower morbidity in beta-blocked patients

ANSWER C A. FALSE : inhalational agents have never been shown to cause anaphylaxic/anaphylactoid reactions B. FALSE : Pretreatment with H1 or H2 antagonists will markedly attenuate severity of anaphylactoid reactions but not frequency. Arguments/evidence for pretreatment * reduces histamine-mediated adverse effects in various studies: * was effective in reducing the incidence of o opioid-induced anaphylactoid reaction o adverse effects of non-immune histamine release following muscle relaxant or vancomycin administration. o anaphylactoid reactions provoked by urea-linked gelatin solutions used as volume expanders, including a prospective study conducted in patients undergoing standard general anaesthesia. * even in the absence of any well-documented studies concerning anaphylaxis, some authors propose pretreatment with H1, or H1 and H2 antagonists as useful in the management of the patient with a history of anaphylaxis or at risk of non-immune histamine release Arguments/evidence against: * beneficial effects have mainly been obtained during clinical manifestations associated with non-immune mediated histamine release. histamine detected during alarming immune-mediated reactions is merely a marker of co-release of more dangerous mediators. * may blunt the early signs of anaphylaxis, leaving a full-blown episode as the presenting sign. * Many authors consider that pretreatment with corticosteroids or antihistamines, or both, do not provide for a reliable prevention of immune-mediated reaction. * proven anaphylactic reactions even in the wake of preoperative H1-H2-receptor antagonists and steroids have been documented in epidemiological surveys. C. TRUE : by definition, causative agents for anaphylactoid reactions are difficult to isolate as they are non immune mediated. There skin testing and RAST will be un-interpretable as these relay on IgE reactivity. D. FALSE : synthesis of histamine of slow, taking 4-6 weeks to restore E. FALSE : can have refractory anaphyaxis with poor response to supportive therapy

Post-transfusion hepatitis in Australia is associated with A. jaundice in over 50% of patients B. development of chronic disease in less than 10% of patients C. hepatitis B in the majority of patients D. the presence of antigen or antibody to hepatitis C E. elevation of serum alkaline phosphatase

ANSWER C A. FALSE : less than 1/3 will have jaundice B. FALSE : 25% will develop chronic liver disease, of these * 20% cirrhosis after 20 years * 10% hepatocellular carcinoma after 30 years C. and D. From Australian Red Cross HIV 1 in 5.4 million Hepatitis C 1 in 2.7 million Hepatitis B 1 in 739,000 HTLV Approx 1 in 17.5 million Malaria 1 in 4.9 to 1 in 10.2 million Variant CJD Possible and cannot be excluded E. FALSE : ALT and AST 400-4000

Foetal thiopentone: A. Peaks (?within 2 minutes/?at 10 mins) of intravenous administration to the mother B. Effects are proportional to maternal peak level C. Level is reduced if given during a contraction D. Has an increased effect due to foetal BBB being more permeable

ANSWER C A. FALSE : peaks within 30-45 seconds, fetal-maternal ration 0.40-1.0 B. FALSE effects are not proportional to maternal peak level. why? C: TRUE levels are decreased during contraction D: FALSE why?

A patient is undergoing forearm surgery under axillary nerve block. What would indicate a failure of proximal spread within the axillary sheath?? A. Preserved sensation in the 5th and medial 4th digits B. Preserved sensation of the lateral forearm C. Preserved sensation of the medial forearm D. Preserved sensation of the palmar surface of the hand excluding the medial 4th and 5th digits

ANSWER C A. FALSE : this supplied by ulnar nerve B. FALSE : this is supplied by the radial nerve C. TRUE : the medial cutaneous nerve branches early from the medial cord D. FALSE : median nerve

In a patient who is HIV antibody positive: A. CD4 count commonly 200-400 (units ?cells/ml) B. Cardiomyopathy is a recognised feature C. Toxoplasmosis infection is a common initial presenttion D. Contraindication to epidural blood patch E. Regional anaesthesia should not be used

ANSWER C A. FALSE : typically 300-600 in a healthy HIV, <200 is AIDS B. FALSE : toxoplasmosis is one of the AIDS-defining opportunistic infections, but occurs in 8% of AIDS patients C. TRUE D. FALSE : relative contraindication E. FALSE

Hyperventilation during neurosurgery A. only vasoconstricts intracerebral vessels B. protects the brain from retractor injury C. is relatively contraindicated in severe coronary artery disease D. can reduce cerebral blood flow by 15% of normal at its peak effect E. may cause a "steal" phenomenon

ANSWER C A. FALSE : vasoconstricts most vessels B. FALSE : vasoconstriction may exacerbate retractor injury by limiting blood flow to compressed areas C. TRUE D. FALSE : can reduced CBF by 30% E. FALSE : reverse steal occurs, vasoconstrict normal brain with normal CO2 responsiveness while the injured brain stays vasodilated due to "vasomotor paralysis".

AZ82 ANZCA version [2005-Sep] Q105, In the elderly patient A. renal dysfunction can be excluded if the serum urea and creatinine concentration are within normal limits B. sedation does not negate the benefits of regional anaesthesia with respect to postoperative cognitive dysfunction C. shivering may be less effective in restoring body temperature, than in younger adults D. the effect of ephedrine in treating hypotension is similar to younger adults E. the uptake of sevoflurane is similar to younger adults, because of its low blood gas solubility

ANSWER C A. False B. False "The advantages of regional anaesthesia with respect to postoperative cognitive dysfunction are only maintained if sedation is avoided" C. True "elderly or debilitated patients shiver less effectively than younger patients" Perioperative hypothermia in the high-risk surgical patient. Best Practice & Research Clinical Anaesthesiology, Volume 17, Issue 4, Pages 485-498 K. Leslie, D. Sessler D. False "The elderly have a poor response to exogenous b-agonists therefore a-agonists should be used". E. False, newer insoluble volatiles(eg Sevo) are ass with a slower uptake in the elderly dt increased V/Q Mismatch and shunting.

Post-transfusion hepatitis in Australia is associated with A. jaundice in over 50% of patients B. development of chronic disease in less than 10% of patients C. hepatitis B in the majority of patients D. the presence of antigen or antibody to hepatitis C E. elevation of serum alkaline phosphatase

ANSWER C A. False : Less than one third B. FALSE : 23% have chronic hepatitis, 51% have chronic active hepatits C. TRUE HIV 1 in 5.4 million Hepatitis C 1 in 2.7 million Hepatitis B 1 in 739,000 HTLV Approx 1 in 17.5 million Malaria 1 in 4.9 to 1 in 10.2 million Variant CJD Possible and cannot be excluded D. FALSE E. inc ALT and AST 400-4000

The oculocardiac reflex is: A. More common with LA than GA B. Caused by retrobulbar block C. Caused by pulling on medial rectus muscle D. Most commonly caused by pulling on lateral rectus muscle E. Not in children

ANSWER C A. More common with LA than GA - False Uncommon with LA B. Caused by retrobulbar block - Rare with regional technique (Using belows reference) C. Caused by pulling on medial rectus muscle - True D. Most commonly caused by pulling on lateral rectus muscle - False E. Not in children - False, more common in children

Regarding TURP syndrome, all are correct EXCEPT A. May occur as quickly as 15 minutes B. May occur 24 hours post operatively C. Mortality 10% D. lacks sterotypical presentation E. has been reported to occur with endometrial ablation and ureteroscopic procedures

ANSWER C A. TRUE B. TRUE C. FALSE : 0.2 to 0.8% D. TRUE : lacks sterotypical presentation resulting in delayed diagnosis E. TRUE

In patients with acute symptomatic post-operative hyponatraemia A. men and women are equally susceptible and have similar risks for severe morbidity and mortality B. rapid correction at a rate greater than 12 mmol.l-1/day carries a risk of osmotic demyelination syndrome C. treatment should consist of hypertonic saline alone and frusemide is relatively contra-indicated D. mortality is less than 10% and is lower in children compared withadults E. sodium deficit is calculated using the formula: mmol Na required = (desired Na - present Na) x 0.4 x weight (kg)

ANSWER C A. men and women are equally susceptible and have similar risks for severe morbidity and mortality FALSE. They are equally susceptible, but women appear to be at dramatically increased risk of severe morbidity and mortality. B. rapid correction at a rate greater than 12 mmol.l-1/day carries a risk of osmotic demyelination syndrome FALSE. It does carry this risk even in acute situations. The risk is very small however if the correction is less than 1mmol/hr in the setting of acute hyponatraemia. C. treatment should consist of hypertonic saline alone and frusemide is relatively contra-indicated TRUE. intracellular overhydration coupled with extracellular and intravascular fluid deficit. Conc Na fixes both. Pulmonary oedema is a potential risk but is actually quite rare unless the patient is particularly prone to it. Frusemide compounds the extracellular deficit without aiding the hypo-osmolar state. A hydro-retic agent (ie anti-desmopressin!) would work but we haven't got them yet. D. mortality is less than 10% and is lower in children compared withadults FALSE. If only SEVERE postoperative hyponatraemia is considered then the mortality is 25%. If all symptomatic hyponatraemia is considered, it is less than 5%. E. sodium deficit is calculated using the formula: mmol Na required = (desired Na - present Na) x 0.4 x weight (kg) False. The formula is based on TBW, ie 0.6 x weight.

AM42 ANZCA version [2004-Apr] Q146 In patients with myasthenia gravis A. aminoglycosides have no effect on the skeletal muscle weakness B. corticosteroids are useful as first line therapy C. myocarditis can result in heart block D. regular plasmapheresis produces long term benefits E. thyroid function is not altered

ANSWER C A: False * Aminoglycosides DO cause aggravated skeletal muscle weakness B: False * Anticholinesterases (eg. pyridostigmine) are first line C: True * Myocarditis can result in AF, heart block or cardiomyopathy D: False * Used in crisis or to prepare patient for theatre * Plasmapheresis can be used as reserved therapy, and is often instituted in patients who have disease progression that is unresponsive to steroids E: False * As an associated auto-immune disease...hyperthyroidism is present in approx 10 %

Improved neurological outcome has been demonstrated with the use of hypothermia soon after:: A. asystolic cardiac arrest B. Cerebrovascular accident C. Perinatal complications causing ischaemic encephalopathy D. rupture of an inracranial aneurysm E. traumatic brain injury

ANSWER C A: False * For VF arrest B: False * There is currently no evidence from randomised trials to support routine use of physical or pharmacological strategies to reduce temperature in patients with acute stroke. Large randomised clinical trials are needed to study the effect of such strategies...[1] C: True * There is evidence that induced hypothermia (cooling) of newborn babies who may have suffered from a lack of oxygen at birth reduces death or disability, without increasing disability in survivors....Cooling for newborns with hypoxic ischaemic encephalopathy D: False * No evidence E:False * The jury is still out on TBI... * Reductions in risk of mortality were greatest and favorable neurologic outcomes much more common when hypothermia was maintained for more than 48 h. However, this evidence comes with the suggestion that the potential benefits of hypothermia may likely be offset by a significant increase in risk of pneumonia.

Which of the following statements regarding the use of epidurally administered adjuvant drugs with epidural analgesia for acute postoperative pain is FALSE? A. adrenaline added to the local anaesthetic improves thoracic epidural analgesia B. clonidine added to epidural opioids improves analgesia C. clonidine prolongs the effects of epidural local anaesthetics D. ketamine added to opioid based epidural analgesia improves analgesia E. neostigmine combined with an epidural opioid reduces the dose of opioid required

ANSWER C A: True (distinction between thoracic vs lumbar epidural) B: True ("the addition of clonidine to PCEA with ropivacaine and morphine after total knee arthroplasty decreased opioid requirements and improved analgesia") C: False (most of the info about clonidine is regarding intrathecal rather than epidural, and that seems to be the distinction here. It prolongs intrathecal block, but no reference made of effect on duration of epidural, so this option is false by a process of elimination as all others are true) D: True ("Epidural ketamine...added to opioid-based epidual analgesia regimens improves pain relief") E: True ("Epidural neostigmine combined with an opioid reduces the dose of epidural opioid that is required")

Hypercalcaemia associated with malignancy, EXCEPT A. Commonest cause: tumour in bone B. Corrected by fluid loading/diuresis C. No response to calcitonin D. Results from increased PTH E. Narrow QT interval

ANSWER C A: true B: true C: false : calcitonin (3-4U/kg IV followed by 4U/kg SC BD) rapid decrease in skeletal release of Ca and PO4 D: true 40% ectopic PTH production E: True

AZ72 ANZCA version [2003-Apr] Q121 [Mar10] The American Society of Anaesthetists (ASA) Physical Status Classification rating system was designed for categorising A. intra-operative adverse anaesthetic event risk based on physical status B. intra-operative adverse surgical event risk based on physical status C. physical status in a standardised manner D. peri-operative adverse anaesthetic event risk based on physical status E. peri-operative adverse surgical event risk based on physical status

ANSWER C ASA classification was always meant to globally assess the degree of "sickness" or "physical state" prior to selecting the anesthetic or prior to performing surgery. It is not an measure of risk.

A one-month-old infant requires analgesia following simple inguinal hernia repair. The most appropriate paracetamol dosing regimen for the first 48 hours is Route Loading Subsequent Maximum daily dose doses dose (mg.kg-1) (mg.kg-1) (mg.kg-1.day-1) A. Oral 20 15 45 B. Oral 20 20 90 C. Oral 20 20 60 D. Rectal 20 20 90 E. Rectal 40 20 90

ANSWER C According to ANZCA APM (2nd ed) p210: * Max daily dose for infants less than 6 months is 60mg/kg/day - so answer is C * Over 6 months: 90 mg/kg/day * Under 6 months: 60 mg/kg/day * 34 wk prem: 45 mg/kg/day

AZ40 [Aug93] A fit healthy 70kg adult has a laparotomy & a morphine infusion is ordered postoperatively. After a loading dose of ..(.?.)... mg/kg. what infusion rate would you order? A. 200 mcg/hr B. 500 mcg/hr C. 2000 mcg/hr D. 5000 mcg/hr

ANSWER C Acute pain management guidelines: 10-40mcg/kg/hr --> 2mg/h seems good.

SG (Q103 Aug 2008) After a difficult thyroidectomy for thyroid carcinoma, a 63 year old woman develops stridor immediately following extubation. The most likely cause is A hypocalcaemia B neck oedema C recurent laryngeal nerve palsies D tracheomalacia E vocal cord oedema

ANSWER C Although all possible causes for stridor post thyroidecotomy, RLN palsy is most likely.

The proportion of adult patients with a probe-patent foramen ovale is A. 0-10% B. 10-20% C. 20-35% D. 35-50% E. greater than 50%

ANSWER C Anatomic closure of the foramen ovale occurs between 3 months and 1 year of age, although 20% to 30% of adults have probe-patent foramen ovales. -Stoelting

SF36 [Mar93] In pregnant patients with severe chronic respiratory disease A. The presence of cor pulmonale is associated with a maternal mortality of more than 70% B. Asthma typically gets worse C. There is usually a similar change in the severity of asthma with each pregnancy D. Chronic maternal hypoxia is poorly tolerated by the foetus

ANSWER C Asthma (Adelaide ICU notes) a. incidence ~ 1:20 persons ~ 1% of pregnant women ~ 10-15% of these will require hospitalisation ~ 50% will have no change in their asthma with pregnancy ~ 25% will improve & 25% worsen b. conditions associated with maternal asthma preterm delivery, low birthweight infants & perinatal death occur more frequently haemorrhage, PIH, requirement for induced labour also more common c. factors which affect asthma in pregnancy reduction in FRC ~ 20%, increased MRO2, increased progesterone - increased RR & MV

At what level of intra-abdominal does cardiac output fall? A. 10 mmHg B. 20 mmHg C. 30 mmHg D. 40 mmHg E. 50 mmHg

ANSWER C At IAP of approximately 30mmHg, CVP falls significantly from previous levels but remains high in comparison with preinsufflation levels. Trendelenburg's position may not overcome the decreases in VR and CO presumably because of pressure on the inferior vena cava. Cardiac index falls to 50% of preoperative values in 5 min

When administering adrenaline and atropine via ETT dose compared with IV should be A. Same dose B. Double C. Quadruple D. Six times E: Ten times

ANSWER C Australian resus guideline Adults Endotracheal route: If IV/IO access cannot be attained and an endotracheal tube is present, endotracheal administration of some medications is possible, although the absorption is variable and plasma concentrations are substantially lower than those achieved when the same drug is given by the intravenous route (increase in dose 3-10 times may be required). There are no benefits from endobronchial injection compared with injection of the drug directly into the tracheal tube. Dilution with water instead of 0.9% saline may achieve better drug absorption. Adrenaline, lignocaine and atropine may be given via endotracheal tube, but other cardiac arrest drugs should NOT be given endotracheally as they may cause mucosal and alveolar damage. Paeds: 100mcg/kg (10x). Atropine 30mcg/kg (1.5x)

Autonomic hyperreflexia: A. Can be seen any time after an acute spinal injury B. Does not occur if lesion is below T10 C. Lesion is above T6 D. ? E. ?

ANSWER C Autonomic dysreflexia is a syndrome of massive imbalanced reflex sympathetic discharge occurring in patients with spinal cord injury (SCI) above the splanchnic sympathetic outflow (T5-T6).

A patient is suffering from a head injury. The first sign of an increase in intracranial pressure is: A. Bradycardia B. Increase in BP C. Decrease in level of consciousness D. Tachycardia E. Vomiting F. Pupil size increase

ANSWER C Bradycardia and hypertension (ie Cushings response) are late stages of a raised ICP. Pupil size increase is usually due to uncal herniation so is a sign of serious injury with ICP high enough to cause herniation. Decreased level of consciousness is a serious sign but is not typically considered the "first" or even an early sign. Vomiting is a common symptom in patients with raised pressure but is pretty non-specific in trauma patients. If an HI patient in ICU vomited then they should be neurologically assessed via GCS and decisions based on that. No point is rushing off for an emergency CT or to OT on the basis of some vomiting alone.

A 60-year-old man with BMI (body mass index) of 30 is having an inguinal hernia repair. He has a LMA-Proseal® in place and you plan positive pressure ventilation. If the device is properly seated the maximum pressure in cm of water before leaking during positive pressure ventilation should be A. 15 B. 20 C. 30 D. 40 E. 50

ANSWER C C. Classics are 20cmH2O. BUT for proseal LMA it's about 30cmH2O!

The pain of the first stage of labour is transmitted by: A. Grey rami communicantes B. T10-L1 anterior roots C. The hypogastric plexus D. Inhibitory nerves to the internal vesical sphincter E. Parasympathetic nerves

ANSWER C C. hygogastric px Update in anaesthesia 2004. The uterus and cervix are supplied by afferents accompanying sympathetic nerves in the uterine and cervical plexuses, the inferior, middle and superior hypogastric plexuses and the aortic plexus. The small unmyelinated 'C' visceral fibres3 transmit nociception through lumbar and lower thoracic sympathetic chains to the posterior nerve roots of the 10th, 11th and 12th thoracic and also to 1st lumbar nerves to synapse in the dorsal horn4. The chemical mediators involved are bradykinin, leukotrienes, prostaglandins, serotonin, substance P and lactic acid5. As the labour progresses severe pain is referred to the dermatomes supplied by T10 and L1. In the second stage, the direct pressure by the presenting part on the lumbosacral plexus causes neuropathic pain. Stretching of the vagina and perineum results in stimulation of the pudendal nerve (S2,3,4) via fine, myelinated, rapidly transmitting 'A delta' fibres3. From these areas, the impulses pass to dorsal horn cells and finally to the brain via the spino-thalamic tract

An adult male patient requires general anaesthesia. He admits to long-standing substance abuse with central nervous system (CNS) stimulants. Compared with a patient who is not a substance abuser, he is likely to require an A. increased dose of induction agent and increased dose of opioid B. increased dose of induction agent and reduced dose of opioid C. increased dose of induction agent and unchanged dose of opioid D. unchanged dose of induction agent and increased dose of opioid E. unchanged dose of induction agent and unchanged dose of opioid

ANSWER C CEACCP (2002) says "the main effects of stimulant drugs will be an increased MAC for anaesthetic agents and cardiovascular instability. Generous premedication and good analgesia may help to obtund major changes in heart rate and blood pressure in response to surgical pain" - suggests to me an increased dose of induction agent and increased or unchanged opiate. Combined with ANZCA pain book "these drugs do not exhibit any cross-tolerance with opioids" and "no data from the clinical setting of any difference in opioid requirements compared from patients who do not use these drugs"

Carbon dioxide is the most common gas used for insufflation for laparoscopy because it A. is cheap and readily available B. is slow to be absorbed from the peritoneum and thus safer C. is not as dangerous as some other gases if inadvertently given intravenously D. provides the best surgical conditions for vision and diathermy E. will not produce any problems with gas emboli as it dissolves rapidly in blood

ANSWER C Carbon dioxide is the most frequently used gas for insufflation of the abdomen as it is colourless, non-toxic, non-flammable and has the greatest margin of safety in the event of a venous embolus (highly soluble). It is absorbed readily from the peritoneum, causing an increase in PaCO2.

Possible mechanisms of post operative visual loss include each of the following EXCEPT A. atherosclerosis causing decreasing blood flow to the optic nerve B. fluid overload C. Isovolaemic haemodilution D. Postually induced raised venous pressure E. Variations in the number of posterior ciliary arteries

ANSWER C Causes of postoperative visual loss Ischemic optic neuropathy Anterior ischemic optic neuropathy Non-arteritic Arteritic Posterior ischemic optic neuropathy Central retinal artery occlusion Central retinal vein occlusion Cortical blindness Hypotensive Embolic Miscellaneous Glycine toxicity Complications of eye surgery Etiology of anterior ischemic optic neuropathy Predisposing factors Variable blood supply (posterior ciliary arteries) Small optic disc size Age Hypertension Smoking Diabetes Vascular disease Precipitating factors Acute systemic hypotension Venous obstruction Raised intraocular pressure Lowered hematocrit Increased blood viscosity

Cephalothin has NO significant activity against A. escherichia coli B. proteus mirabilis C. pseudomonas aeruginosa D. staphylococcus aureus E. streptococcus pneumoniae

ANSWER C Cephalothin : * 1st generation cephalosporin * Good activity against Gram-positive organisms * Poor activity against Enterococci and Gram-negative organisms * good activity against all except P. Aeruginosa (MIMS online)

With regard to Digoxin toxicity which of the following is NOT a feature? a. ventricular bigeminy b. sinus arrest c. atrial flutter d. atrial tachycardia with variable block

ANSWER C Clinical Features • GIT: Nausea, vomiting, anorexia, diarrhoea • Visual: Blurred vision, yellow/green discolouration, haloes • CVS: Palpitations, syncope, dyspnoea • CNS: Confusion, dizziness, delirium, fatigue Electrocardiographic Features • Digoxin can cause a multitude of dysrhythmias, due to increased automaticity (increased intracellular calcium) and decreased AV conduction (increased vagal effects at the AV node) • The classic dysrhythmia associated with digoxin toxicity is the combination of a supraventricular tachycardia (due to increased automaticity) with a slow ventricular response (due to decreased AV conduction), e.g. 'atrial tachycardia with block'. Other arrhythmias associated with digoxin toxicity are: • Frequent PVCs (the most common abnormality), including ventricular bigeminy and trigeminy • Sinus bradycardia or slow AF • Any type of AV block (1st degree, 2nd degree & 3rd degree) • Regularised AF = AF with complete heart block and a junctional or ventricular escape rhythm • Ventricular tachycardia, including polymorphic and bidirectional VT

A woman has an epidural placed for forceps delivery after a prolonged second stage of labour. The next day she has a right foot drop, and numbness over the anterior part of her lower leg and the dorsal surface of her right foot. The most likely cause is A. right common peroneal nerve lesion from the use of stirrups in the lithotomy position B. right L5 nerve root lesion from the epidural placement C. right lumbar plexus lesion from compression by the fetal head D. L5 nerve root lesion from a disc protrusion E. transient neurological symptoms (TNS) syndrome

ANSWER C Common peroneal nerve * dorisiflexion * sensation to anterior calf and dorsum of foot * prolonged lithotomy Lumbar sacral plexus * quadraceps * dorsiflexion * hip abduction Risk factors - primip, vertex presentation, forceps, prolonged labour and CPD Femoral nerve (lithotomy damage) * quadraceps * loss of patella reflex * hyperalgesia ant thigh and med calf Anterior spinal syndrome * loss of motor function, pain and temperature, but touch retained TNS - pain and dysaesthesia in the bum, thigh or calf, usually within 24hrs and resolves in 72hrs Risk factors - lignocaine, small needles, hyperbaric solution, high conc, lithotomy

RH31 [Apr07] Q70 In performing a retrobulbar block, the complication of brainstem anaesthesia: A. Is not associated with contralateral amaurosis B. Has an incidence of approximately 1 in 2000 blocks C. Becomes clinically apparent within 2 to 15 minutes D. Usually takes 4 to 6 hours to resolve E. Is associated with increased auditory acuity

ANSWER C Complication from Brain stem anaesthesia -LA spread along optic nerve sheath -mostly likely with retobulbar -prevalence of 0.1%-0.3% -rare if needle is less than 31mm Symptoms and signs -drowsiness -lightheadness -confusion -loss of verbal contact -incranial nerve palsies -convulsions -respiratory depression -respiratory arrest -cardiac arrest *onset within 10minutes *lasy up to 2-3 hours Prevention -minimised using short, fine needles -eye looking straight ahead

AM46 ANZCA Version [Jul06] Q150 Duchenne muscular dystrophy is a contra-indication to the use of A. corticosteroids B. non-depolarising neuromuscular blockers C. suxamethonium D. tramadol E. volatile anaesthetic agents

ANSWER C Corticosteroids are the only available treatment, although beneficial effects are modest ' Response to nondepolarizing muscle relaxants is normal Potent volatile anesthetics have been linked in these patients to massive elevations in creatine kinase, gross myoglobinuria, and cardiac arrest succinylcholine has been observed to induce severe hyperkalemic cardiac arrest

Stellate ganglion blockade causes A. Conjunctival injection B. Dry eyes C. Decreased axillary sweating D. Paralysis of hemidiaphragm E. Reduced bromage score

ANSWER C Decreased sweating of upper extremity is one of the indications. Other complications: Nerve damage: Horner's syndrome, Rec laryngeal nerve, phrenic nerve, brachial plexus. Intra-thecal injection. Seizures secondary to LA. Damage to structures: Trachea, oesophagus, larynx, pleura, lung.

AM38 ANZCA version [2003-Aug] Q121, [2004-Apr] Q25, [Mar06] The earliest sign in the development of malignant hyperthermia is A. acidosis B. hyperthermia C. increased end-tidal carbon dioxide concentration D. muscle rigidity E. myoglobinuria

ANSWER C Diagnosis of MH (Table 2 from Ref) 1. Unexplained, unexpected increase in end-tidal carbon dioxide (most sensitive indicator of potential MH) 2. Unexplained, unexpected tachycardia and masseter muscle spasm usually follow the carbon dioxide increase 3. Mixed respiratory and metabolic acidosis 4. Skeletal muscle rigidity 5. Temperature elevation (often a late sign) 6. Laboratory abnormalities: blood—coagulation profile, electrolytes, arterial blood gas, creatine kinase; urine—myoglobin

PL29 [Aug09] Ventricular fibrillation (VF) following caudal anaesthesia in 20kg six year old child. The recommended dose of of Intralipid 20% is: A. 10mls B. 20mls C. 30mls D. 40mls E. 50mls

ANSWER C Dose of Intralipid 20% is 1.5 ml/kg = 30ml AAGBI guidelines 2010 - Immediately give iv 1.5ml/kg over 1 min and start and iv infulsion of 15 ml/kg/hr

Regarding the use of adrenergic drugs to maintain normotension during regional anesthesia for elective caesarean section A. alpha-adrenergic agonists are associated with increased fetal acidosis B. alpha-adrenergic agonists are associated with reduced uteroplacental perfusion C. ephedrine increases fetal heart rate and catecholamine levels D. phenylephrine is associated with increased nausea and vomiting compared with ephedrine E. prophylactic ephedrine decreases the incidence of fetal acidosis

ANSWER C Either phenylephrine, metaraminol or ephedrine may be used for the management (prevention and treatment) of hypotension during spinal anaesthesia in obstetrics.A recent quantiative systematic review of controlling trials comparing ephedrine with phenylephrine found NO difference between the 2 drugs in their ability to manage hypotension, except for a higher incidence of maternal bradycardia with phenylephrine. There was NO difference between the two vasopressors in the incidence of foetal acidosis (umbilical pH <7.2)

The output of a diathermy machine does NOT cause patient electrocution because the A. current is too low B. current travels on the surface of the body C. frequency is too high D. return electrode is never placed between the heart and the operating site E. voltage is too low

ANSWER C Electrocution related to the voltage and the current density. Patients are not electrocuted because 1. every attempt is made to make sure the return electrode does not 'cross' the heart, 2. low current as possible is used, combined with the lowest voltage to make the resistance work (ie humans are more sensitive to voltage as a sensation) 3. frequency is high, too high to cause VF, frequency>100kHz produce entirely heat

75 year old with non-valvular AF usually on warfarin has their warfarin stopped for one week. What is their daily risk of stroke? A: 1% B: 0.1% C: 0.01% D: 4% E: 10%

ANSWER C Embolic Risk of AF : Application of the CHADS2 CHF - 1 HT - 1 Age > 75 - 1 DM - 1 CVA - 2 Score of *0 : 0.5-1.7% Risk per year of stroke, Commence Aspirin only *1 : Unsure if stroke benefit outweights bleeding risk if patient is female; greater >65 OR has any artherosclerotic disease (PVD/CAD) they should be started on warfarin (CHA2S2VASc score) *2 = 4% *3 = 6% *4 = 8.5% *5 = 12.5% *6 = 18% 2%/365 = 0.005%

The percentage of diabetic patients with autonomic neuropathy is reported to be A. <5% B. 10 - 20% C. 20 - 40% D. 60 - 80% E. >80%

ANSWER C Estimated to be present in 20% to 40% of patients with long-standing diabetes . Autonomic dysfunction, which is of par-ticular importance to the anaesthetist, is detectable in up to 40% of type 1 and 17% of type 2 diabetic patients.

The most appropriate intravenous fluid volume to administer to a one-year-old 10 kilogram infant, fasted for five hours having minor peripheral surgery of two hours duration (with minimal blood loss) is A. 200 ml B. 260 ml C. 320 ml D. 410 ml E. 490 ml

ANSWER C Fluid requirements per hour: * 4ml/kg/hr first 10 kg * 2ml/kg/hr next 10 kg * 1ml/kg/hr each further kg 10kg child 7hr x 40ml/hr = 280ml Third space losses = 0-2ml/kg/hr for relatively atraumatic surgery or 6-8ml/kg/hr for traumatic procedures, so 2ml/kg/hr x 10kg x 2hr = 40ml Total = 320ml = C

A 7 kg Infant with tetralogy of Fallot, post Blalock-Taussig shunt. Definitive repair at later date. Paralysed and vetilated. Baseline saturation 85%, now 70%, BEST treatment: A. Increase FiO2 from 50 to 100% B. Esmolol 70 mcg C. Phenylephrine 35 mcg D. Morphine 1 mg E. 1/2 NS with 2.5% dextrose 70 mls

ANSWER C Frank Shann's dose recommendation for phenylephrine is 2-10 mcg/kg as a bolus, which would fit nicely with C (5mcg/kg). It'd be nice to do A too, but I'd say C is more effective.

A 'sleep dose' of thiopentone in a healthy adult is likely to result in: A. A fall in peripheral resistance followed by a rise in cardiac output B. A fall in cardiac output followed by a rise in peripheral resistance C. A fall in peripheral vascular resistance followed by a fall in cardiac output D. A rise in peripheral vascular resistance followed by a fall in cardiac output E. A fall in cardiac output followed by a fall in peripheral vascular resistance

ANSWER C From Miller The primary cardiovascular effect of barbiturate induction is peripheral vasodilation resulting in pooling of blood in the venous system.[344] A decrease in contractility is another effect and is related to reduced availability of calcium to myofibrils. In addition, the heart rate is increased.[343] Mechanisms for the decrease in cardiac output include (1) direct negative inotropic action, (2) decreased ventricular filling because of increased capacitance, and (3) transiently decreased sympathetic outflow from the CNS.[332][345] The increase in heart rate (10% to 36%) that accompanies thiopental administration probably results from baroreceptor-mediated sympathetic reflex stimulation of the heart in response to the drop in output and pressure. Thiopental produces dose-related negative inotropic effects that appear to result from a decrease in calcium influx into the cells with a resultant diminished amount of calcium at sarcolemma sites. The cardiac index is unchanged or reduced, and mean arterial pressure is maintained or slightly reduced. Thiopental infusions and lower doses tend to be accompanied by smaller hemodynamic changes than those noted with rapid bolus injections. In the dose ranges studied, no relationship between plasma thiopental level and hemodynamic effect has been found. A sympathetic discharge in response to intubation increases the heart rate and blood pressure and can be attenuated by the administration of fentanyl (1 to 3 µg/kg)."

The BEST agent to decrease gastric volume AND increase gastric pH before semi-urgent procedure A. Omeprazole B. Cimetidine C. Ranitidine D. Sodium citrate E. ?

ANSWER C From the ANZCA Obstetric SIG Ranitidine has been the most extensively studied (Escolano et al 1996, Level II; Lin et al 1996, Level II; Rout et al 1992, Level II). To date, no other agent has been consistently demonstrated to be more effective than ranitidine and most are more expensive. A number of studies have been done in the non-obstetric elective surgery setting. Maltby et al (1990, Level II) showed that 150 mg of oral rantitidine when given two to three hours before surgery resulted in a mean gastric pH of 5.86 (+/- 1.73) and with only one out of 49 patients having an 'at risk' combination of pH <2.5 plus gastric volume of >25 mL. According Stoelting's Co-exisiting disease - p281 rantitidine is the most potent agent for reducing volume and increasing pH within the time span, ie 1-1.5 hours. Omeprazole needs to be given the night before. There are other PPI than can be give immediately prior to surgery, but this was not an option.

AZ27 [Mar94] Lowest venous oxygen saturation during anaesthesia occurs in blood from: A. Liver B. Brain C. Heart D. Lung E. Kidney

ANSWER C Heart

The treatment of carbon monoxide poisoning in the unconscious patient includes: A. Dexamethasone B. Mannitol C. Hyperbaric oxygen D. Methylene blue E. Bicarbonate

ANSWER C Hyperbaric Oxygenation Displaces CO from Hb, after which it can be expelled through the lungs. The half-life of carboxyhemoglobin is reduced from ~ 240 mins on Room Air to ~ 75 to 80 ms at an FIO2 of 100% and to 20 mins with Hyperbaric oxygen (HBO) at 2.0 atmospheres * Controversial as to whether it reduces neurologic sequelae, and obvious problems with Mx of critically ill, often with other injuries , so if mild / no neurologic sequelae - probably not indicated * If > 40% burns don't delay Mx of other more life-threatening problems - i.e airway compromise or haemodynamic instability CO Poisoning Smokers may have up to 10% 'normally'; Coma or levels > 40% always indicate severe poisoning * Impaired oxygen delivery to tissues esp brain and heart * Neuro signs vary from mild confusion to fitting or coma * ECG : may see ST segment changes * PaO2 is normal (in absence of hypoventilation/aspiration) * Cherry-pink skin and mucosa is RARELY seen, cyanosis is more common Management 1. 100% O2 - elimination depends more on alveolar partial pressure of O2 and less on alveolar ventilation - therefore O2 as soon as possible 2. Hyperbaric Oxygen : probably beneficial for those with coma, neuro deficits, or carboxyHb > 30%

Neonate born to known drug abusing mother brought to emergency department by grandmother, unwell lethargic, slightly jaundiced, ABG shows following: pH 7.52 Na 135 Cl 87 K 3 pCO2 38 Which of the following is the Diagnosis? A) Septicaemia B) Hepatitis C) Pyloric stenosis D) Pneumonia E) Opioid withdrawl

ANSWER C Hypokalaemic, hypochloraemic metabolic alkalosis; classic for pyloric stenosis. The other stuff is probably a distraction.

What is the best predictor of severe bleeding in cirrhosis? A. Thrombocytopaemia B. Hypofibrinogenaemia C. Prolonged PT D. Hypoalbuminaemia E. Pulmonary hypertension

ANSWER C I assume this question refers to the risk of transfusion in liver resection. Predictors of tranfusion include -Redo resection -Tumor size >3.5cm -Metastatic disease -Preopertive anaemia -Extended resection (5+ segments) -Laparaoscopic technique -Elevated PT http://www.ncbi.nlm.nih.gov/pmc/articles/PMC2814404/

An elderly lady has a closed neck of femur fracture and presents to ED. She is in chronic AF and on warfarin. INR is 2.6 and she is not bleeding. It is 9am and she is scheduled for repair the following day. According to current guidelines, how should her warfarin be reversed? A. Prothrombinex 25IU/kg immediately and then 2 units FFP immediately prior to surgery B. No immediate treatment then 2 units FFP immediately prior to surgery C. Vitamin K 1mg IV immediately D. Vitamin K 10mg IV immediately E. Withhold warfarin

ANSWER C INR <1.5 = proceed with surgery URGENT SURGERY INR <1.9 FFP INR <5.0 FFP + Vit K 1-3mg slow IV INR < 9 FFP Vit K 2-5mg slow IV SEMI URGENT INR <1.9 Vikt K 1mg oral INR< 5.0 Vit K 1.25mg oral, repeat INR INR <9.0, Vit K 2.5-5.0mg, repeat INR ELECTIVE SURGERY Bridging therapy if high risk of VTE Risk factors 1. Acute thrombosis, on treatment > 1month 2. Mechanical heart valve 3. Severe myocardial dysfunction 4. Atrial fibrillation Acute thrombosis <1month IVC filter

IC77 ANZCA version [2005-Sep] Q115, [Apr07] Q84, [Jul07] The signs of exposure to a nerve agent such as Sarin or VX include A. bronchodilation B. dry skin C. muscle fasciculation D. pupillary dilatation E. tetany

ANSWER C Inhibition of acetylchonlinesterase Causing a cholinergic crisis People exposed to a low or moderate dose of VX by inhalation, ingestion (swallowing), or skin absorption may experience some or all of the following symptoms within seconds to hours of exposure: * Runny nose * Watery eyes * Small, pinpoint pupils * Eye pain * Blurred vision * Drooling and excessive sweating * Cough * Chest tightness * Rapid breathing * Diarrhea * Increased urination * Confusion * Drowsiness * Weakness * Headache * Nausea, vomiting, and/or abdominal pain * Slow or fast heart rate * Abnormally low or high blood pressure # Even a tiny drop of nerve agent on the skin can cause sweating and muscle twitching where the agent touched the skin. # Exposure to a large dose of VX by any route may result in these additional health effects: * Loss of consciousness * Convulsions * Paralysis * Respiratory failure possibly leading to death

A strategy shown to reduce the incidence of severe phantom limb pain is the use of A. continuous regional blockade using nerve sheath catheters B. patient controlled analgesia with opioids post-op C. perioperative ketamine D. perioperative NSAIDs E. spinal anaesthesia for the amputation

ANSWER C Ketamine bolus followed by infusion for 72 hours. Key messages : PREVENTION 1. Perioperative epidural analgesia effective in reducing severe phantom limb pain NNT6 (Level 3) 2. Perioperative ketamine bolus prior to skin incision followed by infusion for 72 hours reduced incidence of severe phantom limb pain -no effective with epidural ketamine 3. Perioperative gabapentin is ineffective in reducing incidence and severity of phantom limb pain 4. Continuous regional blockade via nerve sheath catheters provides effective postoperative analgesia after amputation, but has no preventive effect on phantom limb pain (U) (Level II). TREATMENT 1. Calcitonin intranasal or subcutaneously is effective in treating acute phantom pain but inffective for chronic phantom pain 2. Ketamine provided short-term relief from stump and phantom limb pain 3. Oral slow-release morphine and IV infusions of morphine reduced phantom limb pain 4. Gabapentin is effective in reducing phantom limb pain 5. IV lignocaine reduced stump pain but no effect on phantom pain 6. amitriptyline and tramadol provided good control of stump and phantom pain 7. Sensory discrimination training and motor imagery reduce chronic phantom limb pain. 8. Perioperative epidural analgesia reduces the incidence of severe phantom limb pain (U) (Level III‐2).

With respect to latex allergy A. immediate IgE mediated hypersensitivity is thought to be due to polysaccharides retained within finished latex products B. it is possible to distinguish between contact dermatitis and IgE mediated hypersensitivity on the basis of history C. no further testing is required if a latex RAST or EAST (enzyme linked) test is positive D. intradermal tests are used to make a diagnosis, because anaphylaxis has been reported with skin prick testing E. less than 5% of atopic health workers are likely to skin test positive to latex

ANSWER C LATEX ALLERGY IgE mediated to natural latex proteins Incidence : in a Norwegian series, NRL accounts for 3.6% of perioperative anaphylaxis 0.8% non health care workers 5-15% of all health care workers At risk individuals : 1. genetically predisposed - atopics (Type I - IgE mediated) 2. significant exposure - healthcare workers, children requiring multiple/repetitive surgical and medical interventions (neural tube defects, spinal cord trauma, urogenitial malformations) (delayed Type IV) **adult spinal cord injuries are not at increased risk Associated with serological and/or clinical cross-reactivity for numerous plant allergens (banana, chest nut, avocado) In most patients, diagnosis of NRL anaphylaxis can readily be established with RAST and skin tests.

TMP-121 PC Levosemendin: A. Increases contractility and myocardial oxygen consumption B. Increases SVR C. Binds to troponin C and induces a conformational change D. Increases contractility by increasing calcium influx E. Causes coronary vasodilation but NOT peripheral vasodilation

ANSWER C LEVOSIMENDAN -Calcium sensitizor used for inotropic support in acute severe decompensated cardiac failure. Mode : binds to cardiac Troponin C in a calcium-dependent manner -positive inotropic effect -vasodilatory effect by opening ATP-sensitive K+ channels in vascular SM -therefore decreasing preload and afterload Contraindications -renal impairment -hepatic impairment -hypotension -tachycardia Adverse effects -tachyarrthymias -myocardial ischemia

Levosimendan: A. Increases contractility and myocardial oxygen consumption B. Increases SVR C. Binds to troponin C and induces a conformational change D. Increases contractility by increasing calcium influx E. Causes coronary vasodilation but NOT peripheral vasodilation

ANSWER C Levosimendan causes conformational changes in cardiac troponin C during systole, leading to sensitisation of the contractile apparatus to calcium ions It increases contractility without increasing oxygen requirements Causes coronary and systemic vasodilation

Levosimendan: A. Increases contractility and myocardial oxygen consumption B. Increases SVR C. Binds to troponin C and induces a conformational change D. Increases contractility by increasing calcium influx E. Causes coronary vasodilation but NOT peripheral vasodilation

ANSWER C Levosimendan is a calcium sensitiser that can be administered intravenously (IV) to patients with acute decompensated congestive heart failure (CHF). At therapeutic dosages levosimendan enhances myocardial contractility without increasing oxygen requirements, and causes coronary and systemic vasodilation.---medscape The drug works via a dual mechanism of action which enhances cardiac contractility and vasodilatation without affecting intracellular free calcium, and so should have reduced proarrhythmic potential. It can be administered intravenously (IV) which makes it a therapeutic option for acute decompensated CHF.[----medscape

You are called to the cardiac catheter lab to assist when a 55-year-old man with unstable angina becomes restless during difficult placement of a right coronary artery stent. When you arrive he is conscious and responding to voice. He is sweating with a pulse of 60 beats per minute in sinus rhythm, blood pressure measured from arterial catheter of 80/50 mmHg and Sp02 of 97%. The arterial pressure wave has an exaggerated fall with inspiration. The most appropriate next clinical intervention would be to A. administer atropine B. commence an adrenalin infusion C. perform a quick transthoracic echocardiograph D. sedate and intubate E. transfer to operating theatre immediately

ANSWER C Likely diagnosis is tamponade secondary to RCA rupture with ?AV node bradycardia? Differential * Cardiac Causes -Tamponade/effusion secondary to RCA rupture, although would expect a higher CVP -Cardiogenic shock due to infarction or ischaemia - Pericarditis/cardiomyopathy -Pulmonary embolism * Pulmonary Causes -Tension Pneumothorax -Severe Asthma * Non-cardiac causes -Hypovolaemic shock -Diuretic overdose -Occult haemorrhage -Anaphylaxis (rare, but would explain low CVP and tachycardia may be blocked by beta-blockers) -Drug error (but not an anaesthetic overdose, because he's anxious)

AZ63 ANZCA version [2002-Aug] Q123, [Mar06] Q59 [Apr07] Q10 Regarding extracorporeal shockwave lithotripsy (ESWL) for renal calculi in patients with a permanent pacemaker in situ, A. ESWL is contraindicated B. ESWL pulses should be timed to coincide with the ST interval C. rate modulation of the pacemaker should be deactivated during the ESWL procedure D. there is a risk that the ESWL pulse will reprogramme the pacemaker E. regional anaesthesia is relatively contraindicated

ANSWER C Lithotripsy may be used in patients with a pacemaker insitu, but the lithotriptor must be kept at least 6 inches away from the pacemaker. The lithotripsy pulses should be timed with the ECG and rate-modulation should be de-activated

Young female patient for tonsillectomy. Which causes INCREASED tendency for bleeding? A. Factor V Leiden B. Protein C deficiency C. Haemophilia B (Christmas disease) D. Antithrombin III deficiency E. Lupus anticoagulant

ANSWER C Lupus anticoagulant (http://en.wikipedia.org/wiki/Lupus_anticoagulant) is asociated with a tendency to thrombosis. It is a nuisance on coag testing as its leads to an abnormal result which may unnecessarily delay surgery. Factor V Leiden confers resistance to breakdown by protein C & S system. It results in hypercoagulability, and is therefore the wrong answer. Protein C deficiency results in reduced FV & VIII breakdown and hypercoagulability, and is therefore the wrong answer. Haemophilia B, is X-linked but the phenotype in females is variable. It does cause increased bleeding. And is the answer to choose. ATIII deficiency is associated with a hypercoagulable state, and is the wrong answer. Lupus anticoagulant, causes in vitro prolongation of the APTT but in vivo, results in hypercoagulability, and is the wrong answer.

Young pregnant patient with moderate mitral stenosis, normal LV function. The best delivery method A. Epidural anaesthesia LSCS B. Spinal with LSCS C. Epidural analgesia and normal vaginal delivery D. GA LSCS E. Normal vaginal delivery with remifentanil PCA

ANSWER C MS is a fixed output valvular disease -transmitral gradient is proportional to CO squared, therefore increasing CO by 50%, will increase the gradient 2.3 fold -generally pregnancy will increase NYHA class by One Aims are to keep the patient at they are -maintain preload -normal HR -maintain afterload MS should be evaluated before pregnancy -prophylactic percutaneous mitral balloon valvotomy -NHYA 2-4 or high PTH high risk of complications and death Treat antenatal with diuretic and beta blockers -care should be taken to avoid hypovolaemia -ACEI are contraindicated in pregnancy Labour and post partum are the most dangerous times -Epidural to blunt sympathetic response (attenuate HR, CO, therefore minimised increasing transmitral gradient) -Most delivery, sudden increase in preload with autotransfusion can lead to APO -Cautious use of Syntocinon during 3rd stage. Vasodilation can cause hypotension with compensatory tachycardia leading to overdistention of LA (due to MS) and AF => APO -AF should be electrically cardioverted.

During elective major vascular surgery the best way to reduce the risk of acute renal failure is to maintain a normal A. central venous pressure B. mean arterial blood pressure C. renal blood flow D. systemic vascular resistance E. urine output

ANSWER C Maintain renal flow Limit AoX time Avoid supra renal AoX

The QT interval may be prolonged by each of the following EXCEPT A. high intra-thoracic pressure B. hypothermia C. magnesium sulphate D. suxamethonium E. volatile anaesthetic agents

ANSWER C MgSO4 does not prolong the QTI, it is treatment for Torsades (30mg/kg).

AM45 ANZCA version [2005-Apr] Q101, [2005-Sep] Q34 Spinal correction is planned for a twelve-year-old girl with idiopathic scoliosis. Potential problems associated with this condition include' A. diabetes insipidus B. laryngeal abnormalities C. mitral valve prolapse D. phaeochromocytoma E. renal artery stenosis

ANSWER C Mitral valve prolapse has a strong association with idiopathic scoliosis. Its one of the common viva questions when asked about your assessment of scoliosis in a young female patient.

In a patient undergoing a femoro-popliteal bypass, the most predictive independent risk factor for the development of post-operative myocardial infarction would be A. a previous history of coronary artery bypass grafts B. diabetes mellitus C. an episode of intra-operative myocardial ischaemia D. hypercholesterolaemia E. an episode of post-operative myocardial ischaemia lasting over 60 minutes

ANSWER C Most (>80%) Perioperative MIs occur early after surgery, are asymptomatic, of the non-Q-wave type (60-100%), and are most commonly preceded by ST-segment depression rather than ST-segment elevation

DC cardioversion - LEAST likely indicated for A atrial fibrillation B atrial flutter C multifocal atrial tachycardia D paroxysmal atrial tachycardia E ventricular tachycardia

ANSWER C Multifocal AT are resistent to DC cardioversion Indications for DC 1. narrow or wide QRS complex tachycardia (ventricular rate >150) who is unstable (eg, chest pain, pulmonary edema, lightheadedness, hypotension) should be immediately treated with synchronized electrical cardioversion 2. Synchronized electrical cardioversion may be used to treat stable VT that does not respond to a trial of intravenous medications 3. In hemodynamically stable patients with atrial fibrillation, atrial flutter, or other supraventricular tachycardia (SVT), synchronized electrical cardioversion can also be used to electively restore sinus rhythm. Contraindications 1. known digitalis toxicity -associated tachycardia. 2. Atrial Tachycardias (including multifocal) 3. AF with risk of developing or known left sided thrombus

AM06 ANZCA version [2003-Apr] Q132 In patients with hypokalaemic periodic paralysis one should avoid A. anxiolytic drugs B. beta-adrenergic antagonists C. intravenous dextrose D. regional anaesthesia E. suxamethonium

ANSWER C Mutation on chromosome 17 -autosomal dominant pattern with a penetrance that is about 90% in males, but around 50% in females. -1/3 no family history (sporadic) Pathophysiology - disturbance in ATP-sensitive potassium channel in skeletal muscle. Seems to be excessive potassium transport into muscle, especially in response to insulin. Total body potassium is normal Clinical features - usually presents in adolescence - attacks vary widely in frequency and duration - may be precipitated by high CHO or sodium meals - affects bulbar & and respiratory muscles rarely - +/- cardiac arrhythmias during death - examination between attacks is normal Triggers of hypokalaemic periodic paralysis -mental stress -cold -beta-agonists -carbohydrate load -sodium load Managment - acute attacks respond to potassium -oral preferred -IV should be given with 5% mannitol (avoid dextrose and normal saline) -potassium not effective in prophylaxis. -Best agent appears to be acetazolamide: may block flux of potassium into muscle. -Effectiveness may be related to metabolic acidosis Individuals with HypoPP should be considered as susceptible to malignant hyperthermia and managed with a nontriggering anesthetic technique

The Neonatal Facial Coding Scale (NFCS) used to assess pain in neonates includes all of the following except A. Brow bulge B. Chin quiver C. Closed mouth D. Deep nasolabial fold E. Eyes squeezed shut

ANSWER C Neonatal Facial Coding System (NFCS) actions monitored: * (1) brow lowering (lowering and drawing together of the brow can result in brow bulge) * (2) eyes squeezed shut * (3) deepening of the naso-labial furrow (fold) * (4) open lips (any separation of the lips is an occurrence) * (5) vertical mouth stretch * (6) horizontal mouth stretch * (7) taut tongue (cupping of the tongue) * (8) chin quiver (high frequency vibration of the chin and lower jaw) * (9) lip pursing (tightening the muscles around the lips to form an "oo") In addition a tenth activity was monitored in preterm infants: * (10) tongue protrusion (this is a "no pain" response in full term infants)

PP93 [Apr07] Q124 The Neonatal Facial Coding Scale (NFCS) used to assess pain in neonates includes all of the following except A. Brow bulge B. Chin quiver C. Closed mouth D. Deep nasolabial fold E. Eyes squeezed shut

ANSWER C Neonatal Facial Coding System (NFCS) actions monitored: 1. Brow lowering (lowering and drawing together of the brow can result in brow bulge) 2. eyes squeezed shut 3. deepening of the naso-labial furrow (fold) 4. open lips (any separation of the lips is an occurrence) 5. vertical mouth stretch 6. horizontal mouth stretch 7. taut tongue (cupping of the tongue) 8. chin quiver (high frequency vibration of the chin and lower jaw) 9. lip pursing (tightening the muscles around the lips to form an "oo") In addition a tenth activity was monitored in preterm infants: 10. Tongue protrusion (this is a "no pain" response in full term infants)

SF27 ANZCA version [Sep90] [Mar91] [Mar93] [Aug93] [2003-Apr] Q48, [2003-Aug] Q83 The pain of the first stage of labour is transmitted by: A. Grey rami communicantes B. T10-L1 anterior roots C. The hypogastric plexus D. Inhibitory nerves to the internal vesical sphincter E. Parasympathetic nerves

ANSWER C Neural pathway of pain * The uterus and cervix are supplied by afferents accompanying sympathetic nerves in the uterine and cervical plexuses, the inferior, middle and superior hypogastric plexuses and the aortic plexus. * The small unmyelinated 'C' visceral fibres transmit nociception through lumbar and lower thoracic sympathetic chains to the posterior nerve roots of the 10th, 11th and 12th thoracic and also to 1st lumbar nerves to synapse in the dorsal horn. * The chemical mediators involved are bradykinin, leukotrienes, prostaglandins, serotonin, substance P and lactic acid. * As the labour progresses severe pain is referred to the dermatomes supplied by T10 and L1. * In the second stage: the direct pressure by the presenting part on the lumbosacral plexus causes neuropathic pain. Stretching of the vagina and perineum results in stimulation of the pudendal nerve (S2,3,4) via fine, myelinated, rapidly transmitting 'A delta' fibres3. From these areas, the impulses pass to dorsal horn cells and finally to the brain via the spino-thalamic tract.

The most appropriate method for improving oxygenation during one lung anaesthesia, after institution of an FiO2 of 1.0, is application of A. 5cm H2O CPAP to the non-dependent lung B. 10cm H2O CPAP to the non-dependent lung C. 5cm H2O PEEP to the dependent lung D. 5cm H2O CPAP to the non-dependent and 5cm H2O PEEP to the dependent lung E. intermittent re-inflation to the non-dependent lung

ANSWER C OLV used for almost all thoracic operations : lung, esophageal, aortic or mediastinal surgery, to improve surgical access. Although one lung is ventilated, both lung are perfused. Resulting in -trans pulmonary shunting -impairment of oxygenation -hypoxemia Treatment of hyoxemia during OLV 1. Increasing FiO2 -effective in immediately treating -will not improve with shunt fractions > 40% 2. Check position of DLT (using FOB) 3. ensure adequate CO, reduce volatile to <1MAC 4. Apply recruitment maneouvre 5. Apply 5cm H2O PEEP to dependent lung 6. Apply 1-2cm H2O CPAP to non-dependent lung after recruitment maneouvre 7. Intermittent Reinflation 8. Partial ventilation techniques including oxygen insufflation, HFJV, mechanical obstruction of blood flow to non-dependent lung 9. clamp pulmonary artery of non ventilated lung (rare)

MR46 ANZCA version [2004-Aug] Q93, [Jul06] Physiological consequences of obstructive sleep apnoea usually include each of the following EXCEPT A. stimulation of erythropoiesis B. pulmonary vasoconstriction C. tachycardia followed by bradycardia D. systemic vasoconstriction E. loss of deep sleep

ANSWER C Obstructive sleep apnoea -cessation of airflow for over 10 seconds, despite continuing ventilatory effort, 5 or more times per hour of sleep, and is usually associated with a decrease in arterial oxygen saturation (SaO2) of more than 4% Obstructive sleep hypopnea (OSH) is "as a decrease in airflow of more than 50% for more than 10 seconds, 15 or more times per hour of sleep, and is usually associated with snoring and may be associated with a decrease in SaO2 of greater than 4% AHI = apnoea hyponea index. This index is the number of apnoeas and hyponeas per hour of sleep. 5-20=mild, 21-50=moderate, above 51 severe LAST (lowest saturation) <= 80% --> these highly predict perioperative airway complications. Consequences 1. Respiratory -CO2 insensity 2. Cardiac -systemic hypertension -pulmonary hypertension -right heart strain / RHF / LVF -IHD -CCF Neurological -daytime solmnence Haematological -polycythmia Autonomic -sympathetic hyperstimulation

Off-label use of a drug refers to all of the following EXCEPT: A. Different age-group B. Different indication C. Different concentration D. Different route of administration

ANSWER C Off-label use of medicines: consensus recommendations for evaluating appropriateness. MJA 2006; 185 (10): 544-548 "Examples include use in a different indication, patient age range, dose or route" Use of "Off Label" or Drugs beyond Licence in Pain Medicine. ANZCA Faculty of Pain Medicine "The term "off-label use" may pertain to an unapproved indication, route of administration, age group, or dose. The term does NOT relate to any prescribing conditions outlined by the PBS"

Insertion of a pulmonary artery catheter is relatively contraindicated if the patient has an ECG (electrocardiogram) showing A. anterior fascicular block B. atrial fibrillation C. left bundle branch block D. posterior fascicular block E. right bundle branch block

ANSWER C Passing PAC through RV can cause RBBB in 5% of patients -therefore patients with LBBB can result in complete heart block PAC contraindicated -TV OR PV mechanical prothesis -right heart mass (thrombus or tumor) -TV or PV endocarditis Relative contraindications to PAC -LBBB -WPW -Ebstein's malformation

Patient with aortic stenosis, the signs indicate poor prognosis A. Palpitation B. Radiation to carotid arteries C. Paroxysmal nocturnal dyspnoea D. Angina E. Syncope

ANSWER C Peri op course: Onset of symptoms to death: CCF: 2 years Syncope 3 years Angina: 5 years.

A 34 week prem neonate in recovery post hernia. At 90 seconds after extubation, HR is 75. Most important intervention: A. CPR B. Atropine C. Ensure oxygenation D. Reintubate E. ?

ANSWER C Preterm infant post GA, bradycardia most likely due to hypoxia, either apnea or obstruction - therefore ensuring oxygenation is most important,

Correct statements regarding desmopressin (DDAVP) include each of the following EXCEPT A. it improves platelet function in patients with liver disease B. it improves platelet function in patients with renal disease C. it is contraindicated in Type 1 von Willebrands disease D. it is given in a dose of 0.3 micrograms.kg' intravenously E. it is used in the treatment of diabetes insipidus

ANSWER C Product information for desmopressin: 1. Dose -DI is 1-4mcg/day (10-40mcg/day nasal). Haemophilia or vWD 0.3-0.4mcg/kg 2. Indications: DI, Haemophilia A, vWD except IIB, platelet disorders 3. May induce platelet aggregation in vWD IIB 4. Congenital or acquired platelet disorders including uraemia, drug induces, CPB induced may respond. No evidence for platelet dysfunction of liver disease and use is not supported.

When pulmonary oedema is due to raised capillary hydrostatic pressure, the most reliable radiological evidence of this aetiology is derived from A. heart size B. mediastinal width C. upper lobe vascular markings D. the presence of basal effusions E. oedema migration with postural change

ANSWER C Pulmonary edema may be classified as: * increased hydrostatic pressure edema, * permeability edema with diffuse alveolar damage (DAD), * permeability edema without DAD, or * mixed edema. Pulmonary edema has variable manifestations: * Postobstructive pulmonary edema typically manifests radiologically as septal lines, peribronchial cuffing, and, in more severe cases, central alveolar edema. * Pulmonary edema with chronic pulmonary embolism manifests as sharply demarcated areas of increased ground-glass attenuation. * Pulmonary edema with veno-occlusive disease manifests as large pulmonary arteries, diffuse interstitial edema with numerous Kerley lines, peribronchial cuffing, and a dilated right ventricle. * Stage 1 near drowning pulmonary edema manifests as Kerley lines, peribronchial cuffing, and patchy, perihilar alveolar areas of airspace consolidation; stage 2 and 3 lesions are radiologically nonspecific. * Pulmonary edema following administration of cytokines demonstrates bilateral, symmetric interstitial edema with thickened septal lines. * High-altitude pulmonary edema usually manifests as central interstitial edema associated with peribronchial cuffing, ill-defined vessels, and patchy airspace consolidation. * Neurogenic pulmonary edema manifests as bilateral, rather homogeneous airspace consolidations that predominate at the apices in about 50% of cases. * Reperfusion pulmonary edema usually demonstrates heterogeneous airspace consolidations that predominate in the areas distal to the recanalized vessels. * Post-reduction pulmonary edema manifests as mild airspace consolidation involving the ipsilateral lung, whereas pulmonary edema due to air embolism initially demonstrates interstitial edema followed by bilateral, peripheral alveolar areas of increased opacity that predominate at the lung bases.

LSCS for foetal distress, meconium stained liquor. Management of baby A. Intrapartum suctioning B. Intrapartum suctioning and post partum tracheal suction C. Post partum tracheal suctioning D. Routine neonatal care E. Intubate

ANSWER C Rapid assessment: -pink and breathing-> routine care -apnoea/flat -> tracheal suction then CPAP 5 PIP 30 with air with neopuff mask -continue neopuff is adequate ventilation, HR>100, pink,Preductle Sat>90% -HR<60: commence CPR, atropine -Consider intubation if prem, difficulty with neopuff mask, expected prolonged respiratory support, HR<60

Factors associated with post-operative ulnar nerve palsy include all of the following EXCEPT A. male gender B. sternal retraction for cardiac surgery C. cardiopulmonary bypass for cardiac surgery D. internal jugular vein catheterisation E. diabetes mellitus

ANSWER C Risk factors for ulnar neuropathy Preexisting problems (PATIENT FACTORS) 1. neuropathy 2. malnutrition 3. cervical rib 4. male sex identified in Mayo clinic study 5. peripheral vascular disease (according to the practice advisory) 6. alcohol dependency 7. arthritis Surgical injury (SURGICAL FACTORS) 1. direct including CVL 2. sternal retraction 3. tourniquet Intraoperative conditions (?ANAESTHETIC FACTORS) 1. hypothermia - particularly surface cooling 2. hypotension 3. anaemia 4. electrolyte abnormalities Position (?ANAESTHETIC FACTORS) 1. internal rotation (pronation) with arm abducted >90 degrees 2. pronation when by side (should be neutral) 3. compression or stretch 4. Elbow flexion greater than 90 degrees increases risk.

PZ122 ANZCA version [2006-Mar] Q132 Ginseng (Panax Ginseng) has been associated with an increased risk of each of the following EXCEPT A. agitation with concurrent mono amine oxidase inhibitors B. bleeding with concurrent aspirin C. brochospasm D. hypoglycaemia in fasting patients E. Stevens-Johnson syndrome

ANSWER C SIDE EFFECTS OF GINSENG -Stevens Johnson -Hypertension -Hypoglycaemia -Bleeding -Mania with MAOI

Patient with IV in right arm, has mediastinal mass and SVC compression undergoing mediastinal biopsy, suddenly uncontrolled surgical bleeding in mediastinum. Next step in management prior to thoractomy: A. insert femoral cannulae and place on bypass B. insert IV in left arm C. insert IV into foot D. insert jugular CVC

ANSWER C SVC is obstructed

Patient having TURP becomes restless, dyspnoeic and hypertensive. Which test would be most useful? A. Haematocrit B. Arterial blood gases C. Sodium D. Plasma Hb

ANSWER C Serum Na usually drops 5-8 mmol/L during TURP.

For serum tryptase levels, blood should be collected in a: (a) EDTA tube (b) Li heparin tube (c) plain tube

ANSWER C Serum tryptase analysed. Take three blood tests, each 5 to 10 ml (i) immediately after the reaction has been treated, and; (ii) about 1 hour after the reaction; (iii) about 6 hours or up to 24 h after the reaction It is essential to state the time on samples (and time from onset of reaction) and record this in the notes. Separate serum (or plasma) and store at 4degC if the sample can be analysed within 48 hours. Otherwise store the sample at - 20degC until it can be sent for measurement of serum tryptase.

Middle-aged male with severe MS having general anaesthesia for repair of fractured ulna / radius. 10 minutes into the case you notice a tachyarrythmia with his HR 130 and BP 70. He is normally in sinus. What do you do? A. Adenosine B.Amiodarone C. Shock D. Volume E. Metaraminol

ANSWER C Shock. MS pt very reliant on atrial kick for LV filling, with acute decompensation when SR lost. This sounds like AF with unstable HD.

Intraoperative hypoglycaemia in children is LEAST likely in A. small-for-age children B. Beckwith-Wiedeman Syndrome C. infants following an overnight fast D. infants on propanolol peri-operatively E. intra-operative interruption of pre-operative total parenteral nutrition (TPN)

ANSWER C Small for age, young, prolong fasting, propranolol, Beckwith (organomegaly, glossomegaly, hypoglycaemia), Silver-Russell (small for age, failure to thrive, poor feeding) all associated with hypoglycaemia. Overnight fast causes less hypo than a daytime fast because of morning cortisol, maybe this is the answer. Interruption of preop TPN may cause hypo but add the stress of surgery and I think that they'd probably be OK. All that's required to prevent hypoglycaemia is 2% glucose at the maintenance rate.

Intraoperative hypoglycaemia in children is LEAST likely in A. small-for-age children B. Beckwith-Wiedeman Syndrome C. infants following an overnight fast D. infants treated with propanolol E. Russell-Silver syndrome

ANSWER C Small for age, young, prolong fasting, propranolol, Beckwith (organomegaly, glossomegaly, hypoglycaemia), Silver-Russell (small for age, failure to thrive, poor feeding) all associated with hypoglycaemia. Overnight fast causes less hypo than a daytime fast because of morning cortisol, maybe this is the answer. Interruption of preop TPN may cause hypo but add the stress of surgery and I think that they'd probably be OK. All that's required to prevent hypoglycaemia is 2% glucose at the maintenance rate.

MR45 ANZCA version [2003-Aug] Q99, [2005-Sep] Q90, [Apr07] Q99, [Jul07] In chronic obstructive pulmonary disease (COPD), the variable most closely associated with prognosis is A. arterial carbon dioxide partial pressure (PaC02) B. arterial oxygen partial pressure (Pa02) C. forced expiratory volume in one second (FEV1) D. forced vital capacity (FVC) E. response to bronchodilators

ANSWER C Spirometry is the most reproducible, standardised and objective way of measuring airflow limitation, and FEV1 is the variable most closely associated with prognosis. 30% of patients (with FEV <35% of predicted) are dead within 1 year and 95% within 10 years On the other hand, pCO2 >45mmHg or more (OR 61.0)" was a statistically significant (and best) predictor of post-operative pulmonary risk NOT prognosis in general.

Reverse splitting of the 2nd heart sound is caused by A. acute pulmonary embolism B. ASD C. complete LBBB D. severe MR E. pulmonary HT

ANSWER C Split during inspiration : Normal Split during expiration = Reverse splitting *Aortic stenosis *hypertrophic cardiomyopathy *left bundle branch block (LBBB) * ventricular pacemaker Split during both inspiration and expiration = fixed split S2 * atrial septal defect (ASD) * ventricular septal defect (VSD).

Anaphylaxis, which is wrong: A. higher incidence in females (females have a higher incidence of anaphylaxis to neuromusclar drugs) B. cross-reactivity between ??avocados, bananas and latex C. vecuronium - more likely to cause an anaphylactoid reaction than anaphylaxis D. 99% within mast cells E. peak tryptase in 1hr

ANSWER C Steroidal cause anaphylaxis, benzylisoquinoliums cause anaphylactoid.

A 5kg baby is 5% dehydrated. The body water deficit: A. 150mls B. 200mls C. 250mls D. 300mls E. 350mls

ANSWER C The % dehydrated is a % of total body weight. Five percent of 5,000g is 250g. One ml of water weighs one gram, so 250 grams is 250mls of water. A general description of dehydration signs & symptoms: * Mild dehydration produces thirst and mild oliguria but no detectable physical signs; it equates with 5% acute weight loss. * Moderate dehydration (5-10%) : tachycardia, slightly sunken eyes, reduced skin turgor and peripheral perfusion/capillary return, some loss of ocular tension and sunken fontanelles, and more thirst and oliguria * Severe dehydration (>10%): marked tachycardia, obvious loss of skin tone and tissue turgor, sunken eyes, loss of ocular tension, severe oliguria/anuria, restlessness or apathy. Also circulatory collapse, delirium, coma, fever and cyanosis

Hetastarch 130/0.4. The 0.4 means: A. 40 mg/l B. 40 g/l C. 4 hydroxylations of every 10 glucose residues D. every 4th glucose is hydroxyethylated E. ?

ANSWER C The 130 is the molecular weight of the starch 0.4 is the MS (molar substitution) is the ratio of hydroxyethyl groups to glucose residues

The most frequently reported cause of mortality associated with transfusion of blood and blood products is A. anaphylaxis B. bacterial sepsis C. haemolytic reaction D. transfusion associated graft versus host disease E. Transfusion Related Acute Lung Injury (TRALI)

ANSWER C The ARC handbook has mortality figures listed: Bacterial sepsis 1: 4mill- 8mill Hemolytic reactions acute 1:600000- 1.5mill TRALI 1:5 mill GVHD rare but 90% fatal (CEACCP states "TRALI is the most common cause of major morbidity and death after transfusion. It presents as an acute respiratory distress syndrome (ARDS) either during or within 6 h of transfusion.3")

In regards to anaphalaxis what is INCORRECT, A. 8% risk cross-reactivity of cephalosporins with penicillin B. Colloid use in anaphalaxis often makes situation worse C. Tryptase elevated post anaphylactic but not anaphylactoid reaction D. Repeat tryptase at 24 hours E.

ANSWER C The CEACCP article from Aug 04 answers these questions, it states A. 8% cross reactivity between cephalosporins and penicillin B. Colloids can precipitate histamine release and make the situation worse and are recommended to avoid C. Tryptase is elevated in both- making this option incorrect D. Collect blood immediately, at 1hr and a third sample between 6 and 24 hrs

Ehlers-Danlos syndrome is associated with each of the following EXCEPT: A. blood vessel fragility B. glaucoma C. mental retardation D. mitral valve prolapse E. resistance to local anaesthetics

ANSWER C The Ehlers-Danlos family of disorders is a group of related conditions that share a common decrease in the tensile strength and integrity of the skin, joints, and other connective tissues. Patients can often perform "amazing, almost unnatural, contortions" and worked in circuses. All forms of Ehlers-Danlos syndrome share the following primary features to varying degrees: * Skin hyperextensibility * Joint hypermobility and excessive dislocations * Tissue fragility * Poor wound healing, leading to wide thin scars ("cigarette paper scars") * Easy bruising

In the Revised Trauma Score includes GCS, Blood pressure and what other parameter? A. HR B. Saturation C. Respiratory rate D. Urine output E. CRT<2sec

ANSWER C The Revised Trauma Score is made up of a three categories: Glasgow Coma Scale, Systolic blood pressure, and respiratory rate. The score range is 0-12. In triage, a patient with an RTS score of 12 is labeled delayed, 11 is urgent, and 10-3 is immediate. Those who have an RTS below 3 are declared dead and should not receive certain care because they are highly unlikely to survive without a significant amount of resources.

The average expected depth of insertion of an oral endotracheal tube, from the lip, in a normal newborn infant is A. 7.5 cm B. 8.5 cm C. 9.5 cm D. 10.5 cm E. 11.5 cm

ANSWER C The approximate depth of insertion measured from the centre of the lips for an oral tube: * in a newborn is 9.5 cm, * 11.5 cm for a 6 months old infant * 12 cm for a 1 year old. * Thereafter, the approximate depth of oral insertion is given by the formula: age (years)/2 + 12 cm

Supine hypotension during late pregnancy is associated with A. a rise in the systemic vascular resistance B. a rise in the cardiopulmonary blood volume C. increased heart rate D. stable stroke volume E. a rise in the cardiac index

ANSWER C The concept of aortocaval compression & supine hypotension are different. While all pregnant women compress the aorta & vena cava on supine position, not all women become hypotensive. "Supine hypotension" (or supine hypotension syndrome - SHS ) only occurs in 8% of women. It occurs because these women have not developed sufficient pelvic collaterals to assist venous return on caval compression. SHS is defined as a 15-30 mmHg reduction in systolic BP with a SUSTAINED ELEVATION OF HR of > 20 bpm. Ref: "Hemodynamic changes & baroreflex gain in the supine hypotension syndrome"; American Journal of Obstetrics & Gynaecology 2002: 187; 1634-4 SHS is most commonly due to CAVAL compression. Aortic compression and neurogenic etiologies are less common causes. Ref: Uptodate - Maternal cardiovascular & hemodynamic adaptation to pregnancy" Hence aortic compression is not a uniform feature of SHS, but caval compression is. The increased in SVR due to increased SNS outflow may not occur in pregnant women because they are maximally vasodilated & the aorta may not be compressed.

There is evidence to avoid BIS <40 for more than 5minutes because A. Safe cost B. Increase incident of hypotension C. Increase post op mortality D. Decrease volatile (?) for poor cardiac output patient E. Decrease the incidence of awareness

ANSWER C The effect of bispectral index monitoring on long-term survival in the B-Aware Trial. Anesth Analg 2010 BACKGROUND: When anesthesia is titrated using bispectral index (BIS) monitoring, patients generally receive lower doses of hypnotic drugs. Intraoperative hypotension and organ toxicity might be avoided if lower doses of anesthetics are administered, but whether this translates into a reduction in serious morbidity or mortality remains controversial. The B-Aware Trial randomly allocated 2463 patients at high risk of awareness to BIS-guided anesthesia or routine care. We tested the hypothesis that the risks of death, myocardial infarction (MI), and stroke would be lower in patients allocated to BIS-guided management than in those allocated to routine care. METHODS: The medical records of all patients who had not died within 30 days of surgery were reviewed. The date and cause of death and occurrence of MI or stroke were recorded. A telephone interview was then conducted with all surviving patients. The primary end point of the study was survival. RESULTS: The median follow-up time was 4.1 (range: 0-6.5) years. Five hundred forty-eight patients (22.2%) had died since the index surgery, 220 patients (8.9%) had an MI, and 115 patients (4.7%) had a stroke. The risk of death in BIS patients was not significantly different than in routine care patients (hazard ratio = 0.86 [95% confidence interval {CI}: 0.72-1.01]; P = 0.07). However, propensity score analysis indicated that the hazard ratio for death in patients who recorded BIS values <40 for >5 min compared with other BIS-monitored patients was 1.41 (95% CI: 1.02-1.95; P = 0.039). In addition, the odds ratios for MI in patients who recorded BIS values <40 for >5 min compared with other BIS-monitored patients was 1.94 (95% CI: 1.12-3.35; P = 0.02) and the odds ratio for stroke was 3.23 (95% CI: 1.29-8.07; P = 0.01). CONCLUSIONS: Monitoring with BIS and absence of BIS values <40 for >5 min were associated with improved survival and reduced morbidity in patients enrolled in the B-Aware Trial.

The artery NOT supplied by the internal carotid is: A. Middle cerebral artery B. Anterior cerebral C. Posterior cerebral D. Choroidal E. Retinal

ANSWER C The following are the branches of the internal carotid artery, listed by segment: * C1: Branches from the cervical portion - none. * C2: Branches from the petrous portion o Caroticotympanic arteries o vidian artery * C3: Branches from the lacerum portion - none * C4: Branches from the cavernous portion o Branches of the meningohypophyseal trunk: + Tentorial basal branch + Tentorial marginal branch + Meningeal branch - helps supply blood to the meninges of the anterior cranial fossa + Clivus branches - tiny branches that supply the clivus + Inferior hypophyseal artery o Capsular branches - supplies wall of cavernous sinus o Branches of the inferolateral trunk: + Branches to trigeminal ganglion - provide blood to trigeminal ganglion + Artery of the foramen rotundum + Branches to nerves * C5: Branches from the clinoid portion - none * C6: Branches from the ophthalmic portion o Ophthalmic artery o Superior hypophyseal artery * C7: Branches from the communicating portion o Posterior communicating artery o Anterior choroidal artery o Anterior cerebral artery (a terminal branch) o Middle cerebral artery (a terminal branch)

In anaphylaxis during anaesthesia, bronchospasm is the sole feature in approximately: (a) 0.34 % of cases (b) 1 % of cases (c) 3 % of cases (d) 9 % of cases (e) 25 % of cases

ANSWER C The most common clinical features of anaphylaxis during anaesthesia are cardiovascular (73.6%), cutaneous (69.6%), bronchospasm (44.2%) cardiovascular collapse (53.7%), angio-oedema (11.7%), arterial hypotension (17.8%), bradycardia (2.1%) and cardiac arrest (4%). Cardiovascular collapse was the sole feature in 8.4% hypotension in 2%, bronchospasm in 3.1% and cutaneous symptoms in 7% cases.

A 16 kilogram, 3 year old boy for unilateral inguinal herniotomy has general anaesthesia and a caudal with 8 mls of 0.25% plain bupivacaine. The LEAST correct statement regarding this anaesthetic management is that A. the required level of the block is T12 B. the volume of local anaesthetic used is inadequate C. the block duration is the same for all segments D. peripheral nerve blocks are safer than neural axis blocks in children E. use of adrenaline in the local anaesthetic would prolong the block

ANSWER C The question is asking about the least correct or probably better rephrased, the most false. D is clearly contentious so could be either T/F in a debate. E is probably true but causes spinal ischaemia and so is not ideal in a practical sense. B is correct. The volume is inadequate. (should be 1ml/kg) A is sort of correct. The caudal is for post op analgesia as the child is having a GA, so T12 block height is probably just OK. This leaves C as being 'the most false'

Thoracodorsal nerve arises from? A. Upper trunk B. Middle trunk C. Posterior Cord D. Medial Cord E. Lateral Cord

ANSWER C The thoracodorsal nerve is a branch of the posterior cord of the brachial plexus, and is made up of fibres from the posterior divisions of all three trunks of the brachial plexus. It derives its fibers from the sixth, seventh, and eighth cervical nerves. Anatomy for Anaesthetist - 'Branches of the Posterior Cord' "The nerve to latissimus dorsi (thoracodorsal nerve) (C6-8) arises between the upper and lower subscapular nerves. It accompanies the subscapular vessels along the posterior axillary wall and supplies latissimus dorsi

cTnI remains elevated for up to? A. 1-2 days B. 2-5 days C. 5-14 days D. 1 month E. 1 year

ANSWER C The troponins have a prolonged release pattern remaining present in the blood for up to 10 days after infarction" Because troponins are only released from the cardiac cells and are therefore very sensitive, they are not specific for ischaemic injury (can ↑with myocarditis, PE, etc) From black bank - the universal definition of myocardial infarction says 7-14 days

Conditions which predispose patients to deep venous thrombosis , EXCEPT A. antiphospholipid syndrome (lupus anticoagulant) B. Protein C deficiency C. Plasminoginogen deficiency D. Factor V Leiden mutation E. Protein S deficiency

ANSWER C There is an increased risk of thrombosis with: * Protein C deficiency * Protein S deficiency * Antithrombin III deficiency * Lupus anticoagulant and/or anticardiolipin antibody * Factor V Leiden mutation * Thrombocytosis * Polycythemia

Nonsteroidal anti-inflammatory drugs given during pregnancy, have been associated with all of the following EXCEPT: A. foetal cardiac complications if given in late pregnancy B. foetal renal complications if given in late pregnancy C. increased production of amniotic fluid D. increased risk of miscarriage E. persistent neonatal pulmonary hypertension

ANSWER C This is directly form the ANZCA Acute Pain Book. All the answers there under section "The Pregnant Patient". Foetal cardiac and renal problems with late use of nsaids, realtively safe in early pregnancy, should be discontinued by 32nd week. Definitely wrong one is increased amniotic fluid production.

Each of the following is effective in the treatment of pain from acute herpes zoster EXCEPT A. acyclovir B. amitriptyline C. carbamazepine D. corticosteroids E. topical aspirin

ANSWER C Treatments for acute herpes zoster pain: antivirals (treats acute and reduces PHN), analgesics (paracetamol, nsaids, opioids), topical aspirin (not oral), corticosteroids (treats acute but no effect on PHN), epidural local anaesthesia + steroids (treats acute and reduces PHN). Anticonvulsants will have no effect on acute. TCAs: no effect on acute but may reduce PHN. Carbemazepine no place in the treatment of acute HZ infection.

The line between the iliac crests in a neonate corresponds to: A. L2/3 B. L3/4 C. L4/5 D. L5/S1 E. S1/2

ANSWER C Tuffiers line

Post-operative ulnar neuropathy A. usually presents within 48 hours of surgery B. symptoms usually persist for over two years C. is more common in men D. can usually be avoided by careful positioning E. can usually be avoided by protective padding of the elbow

ANSWER C Usually present after 24hrs & within seven days. 53% asymptomatic within 1 year. Another study showed 50% improved within 8 weeks, the rest had symptoms after 2years. The 2000 practice advisory says that there is insufficient evidence to prove that positioning strategies work. Mayo Clinic Study: male, body habitus (thin or obese) and length of hospital stay (increased chance during hospitalisation).

Immunological evidence of immunity to hepatitis B is the presence of A. hepatitis B core antibodies B. hepatitis B core antigen C. hepatitis B surface antibodies D. hepatitis B surface antigen E. any of the above

ANSWER C Vaccine : + for HBV surface antibody Infected : + HBV surface antigen and Core Antigen Chronic : HBV antigen but no core Acute : Core Antigen

In a patient who has a plasma tricyclic antidepressant level greater than 1000 ng.mL-1 the most likely E.C.G. finding is A. a prolonged PR interval B. deepened Q waves C. a prolonged QRS duration D. a prolonged ST segment E. the appearance of U waves

ANSWER C With increasing TCA dose, the ECG changes are: 1. prolonged QRS and QT 2. prolonged PR 3. VT/VF "The tricyclic antidepressants (TCAs) are probably the leading cause of life-threatening overdoses worldwide Signs and symptoms suggestive of TCA overdose include: * tachycardia, * characteristic QRS widening on EKG, * orthostatic hypotension, * dry mouth, * dilated pupils, * hyperthermia, * hypertonicity, * seizures, * ataxia, and * delirium. The preponderance of anticholinergic findings best may be remembered as: "Hot as a Hare, Blind as a Bat, Dry as a Bone, Red as a Beet, Mad as a Hatter."

Milrinone A. prolongs conduction in Purkinje fibres B. is contraindicated with ACE (angiotensin converting enzyme) inhibitors C. inhibits a phosphodiesterase iso-enzyme D. causes nodal tachycardia E. increases systemic and pulmonary vascular resistance

ANSWER C YENTIS A: Faster conduction B: Caution as can cause hypotension, but not contraindicated C: True D: Generally no change in HR, Ventricular and SV arrythmias can occur E: Reduces SVR

An infant is anaesthetised and ventilated using an endotracheal tube and circle breathing system with CO2 absorber. The item which causes the most resistance to breathing is the A. airway pressure limiting (APL) valve B. circuit hosing C. endotracheal tube D. heat and moisture exchange filter E. inspiratory and expiratory valves

ANSWER C a size 3.5 ETT in a 3 month old infant has 10 times the resistance of a circle system.

When using osmotic agents to reduce intracranial pressure (ICP), the patient's serum osmolality should not be allowed to exceed: A. 280 mosmol/L B. 300 mosmol/L C. 320 mosmol/L D. 340 mosmol/L E. 360 mosmol/L

ANSWER C http://www.trauma.org/archive/neuro/icpcontrol.html Controlling ICP for intensive treatment of traumatic brain injury -maintenance of adequate cerebral perfusion pressure vs raised ICP Ventilation -carbon dioxide dilates the cerebral blood vessels, increasing the volume of blood in the intracranial vault, increasing ICP -aim for normocapnia -no studies have shown improved outcome with hyperventilation -however, hyperventilation can be used for short periods when immediate ICP control is necessary : eg acute deterioration, CT scanning, surgical intervention -transcranial doppler and PET have shown excessive vasoconstriction which may reduce cerebral blood flow to below ischaemic threshold -extended periods of hyperventilation maybe instituted with intensive neuromonitoring : jugular venous oxygen saturation and transcranial doppler assessments Intravenous therapy -should be normovolaemic, hypovolaemia inceases risk of decreased cerebral perfusion -use isotonic fluids to maintain volume -osmolality is most important -avoid all free water -hypertonic solutions and osmotic diuretics : mannitol Mannitol -6 carbon sugar, works via 2 mechanisms 1. expands circulating volume 2. decreases blood viscosity -therefore increasign creebral flow and oxygen delivery -takes 15-30 minutes to draw fluid from brain -mannitol itself directly breakdown BBB -wholly excreted in the urine and will cause rise in serum and urine osmolality -osmolality > 320mOsm are at risk of ATN Anaesthesia -TBI should be intubated -intubation mat induce a rise in ICP especially in those on the cusp of the pressure volume curve -RSI is indicated -continued sedation to allow ventilation and avoid coughing or fighting the ventilator -avoid PEEP

NNT is the number of patient who need to be treated to prevent 1 additional bad outcome. The NNT is the reciprocal of the A. absolute odds of a bad outcome B. absolute risk of a bad outcome C. absolute risk reduction in the bad outcome (due to the treatment) D. odds ratio of the bad outcome (due to the treatment) E. relative risk of the bad outcome (due to the treatment)

ANSWER C new antiemetic reduces risk of vomiting by 1/5th. Thus absolute risk reduction of bad outcome is 1/5th. Thus NNT is 5 inorder for 1 patient to not vomit.

Patient for pneumonectomy. Pre op FEV1 2.4. (Predicted 4.5L) FVC given as well. For R lower lobectomy. Postoperative predicted FEV1 ? A. 1.3 B. 1.5 C. 1.7 D. 1.9 E. 2.2

ANSWER C ppoFEV1 = 2.4 x (44-12)/44 = 1.7

A patient comes to see you in clinic for a pneumonectomy for SCLS. His spirometry shows an FEV1 of 2.5L (>40% predicted). What do you advise about his fitness for surgery? A. He is not fit for the procedure B. He is fit for the procedure C. He needs referral for formal exercise testing D. He needs a blood gas

ANSWER C ppoFEV1 = 2.5 x 0.5 = 1.25L -he requires further testing Algorithms from the British Thoracic Society and American College of Chest Physicians. -initial screening tool is preoperative FEV1 -FEV1 >2L for pneumonectomy -FEV>1>1.5L for lobectomy If these threshold lung volumes are not present -full respiratory testing to calculate ppoFEV1 and DCLO -both need to be >40% and O2 sat>90% on RA If both ppoFEV1 and DLCO are <40%, then CPX testing -VO2 max <15 ml/kg/min delineates between high and medium risk patients%%//d3cgb598vs7bfg.cloudfront.net/images/upload-flashcards/back/4/5/49054099_m.jpg

Black Bank March 2011 24. NEW. When compared to a non-obese patient, in an obese patient (BMI >35) when fasted for an elective procedure the gastric secretion will have: A: more volume, higher pH B: more volume, lower pH C: same volume, same pH D: less volume, lower pH E: less volume, higher pH

ANSWER C the study of Vaughan, in a study with 256 patients undergoing elective surgical interventions, did not observe statistically significant differences in gastric volume and pH between obese and non-obese patients.

Trauma patient with GCS 6 with hard collar. HR and BP unstable. What is the best way of clearing neck? A. CT B. Cervical spine trauma series C. MRI D. Leave had collar indefinitely?? E. Clinically

ANSWER C NEXUS CRITERIA • National Emergency X-Radiography Utilization Study • NPV 99.9% and PPV 2.7% • Presence of any one of these findings requires radiological evaluation 1. Midline tenderness C-spine 2. Focal neurological deficit 3. Decreased level of alertness 4. Evidence of intoxication 5. Distracting injury CANADIAN C-SPINE • NPV 100%, PPV 40% • Includes 20 clinical findings and 5 demographic variables • High-Risk YES? → Radiography o Age>65 o Dangerous mechanism • Fall from elevation 3m • Axial load to head • MVA>100km/hr rollover ejection • Motorized recreational vehicles • Bicycle struck or collision o Paresthesia in extremities • Low Risk NO? → Radiography o Simple rearend MVA o Sitting position in ED o Ambulatory at any time o Delayed onset of neck pain o Absence of midline C-spine tenderness • Able to actively rotate neck 45° left or right NO? → Radiography RADIOLOGICAL EVAULATION • Plain radiographs 1. Cross table lateral view 2. Anteriorposterior view 3. Open mouth view of odontoid 4. Swimmer view • CT or MRI for obtunded patients • CT unable to define unstable ligmentus injury in less than 5% of cases • MRI is no better than CT

Regarding extracorporeal shockwave lithotripsy (ESWL) for renal calculi in patients with a permanent pacemaker in situ, A. ESWL is contraindicated B. ESWL pulses should be timed to coincide with the ST interval C. rate modulation of the pacemaker should be deactivated during the ESWL procedure D. there is a risk that the ESWL pulse will reprogramme the pacemaker E. regional anaesthesia is relatively contraindicated

ANSWER C Pacemakers relatively contraindicated * Dual chamber pacemakers o Reprogram to VVI o Turn rate response off * Continuous ECG monitoring * Keep lithotripter more than 15cm from pacemaker * Time shock wave with R wave Reprogramming a pacemaker to asynchronous pacing at a rate greater than the patient's underlying rate usually ensures that no over- or undersensing during EMI will take place, protecting the patient. Miller p1422

Stellate ganglion block associated with all except: A. Ptosis B. Miosis C. Sweating D. Facial flushing E. Nasal stuffiness

ANSWER C Stellate Ganglion produces Horners syndrome * Ptosis * Miosis * Anhidrosis In addition, several other eye signs are present: * Conjunctival injection * Lacrimation

Which statement regarding Addison's Disease is INCORRECT A. Presents with hyperkalaemia B. Results in water and sodium depleted C. Hypernatraemia is common D. Hyperpigmentation is usually present E. Metabolic acidosis

ANSWER C Addisons Disease : Primary adrenocortical insufficiency. -insidious, progressive hypofunctioning of the adrenal cortex The adrenal cortex produces a number of hormones * Mineralocorticoids e.g. aldosterone * Glucocorticoids e.g. Cortisol * Sex hormones/Androgens Failure of the adrenal cortex leads to a decrease in these hormones. Destruction of 90% of the cortex is required before insufficiency occurs. Causes -Destruction by antibodies (autoimmune disorder) is the cause in 70-80% of cases -Other causes of Addisons: * TB (previously a common cause but no longer so in Western countries) (eg For anaesthetic-related case report see [1] * Metastases * Bilateral adrenalectomy * Haemorrhage e.g. following meningococcal sepsis Effects of Addisons Disease Skin * Pigmentation due to increased ACTH by anterior pituitary gland. The ACTH cross stimulates Melanocyte Stimulating Hormone (MSH) causing pigmentation Fluid Balance * Hypovolaemia with postural hypotension is typical Acid Base * Metabolic acidosis (low HCO3) Electrolytes * Loss of aldosterone results in loss of Na+ reabsorption by the kidney * Hence urinary Na+ is high * Hyponatraemia * Hyperkalaemia * Haemoconcentration is often present * Hypoglycaemia * high urea Hormonal * Elevated ACTH (>50pg/mL) * Low cortisol (<5ug/dL) Anaesthesia * Volume is a key issue. These patients need intravascular reexpansion * Minimise neuromuscular blockade as hyperkalaemia can cause skeletal muscle weakness

Improved neurological outcome has been demonstrated with the use of hypothermia soon after:: A. asystolic cardiac arrest B. Cerebrovascular accident C. Perinatal complications causing ischaemic encephalopathy D. rupture of an inracranial aneurysm E. traumatic brain injury

ANSWER C (maybe E) A: False * For VF arrest B: False * There is currently no evidence from randomised trials to support routine use of physical or pharmacological strategies to reduce temperature in patients with acute stroke. Large randomised clinical trials are needed to study the effect of such strategies...[1] (http://www.cochrane.org/reviews/en/ab001247.html) C: True * There is evidence that induced hypothermia (cooling) of newborn babies who may have suffered from a lack of oxygen at birth reduces death or disability, without increasing disability in survivors....Cooling for newborns with hypoxic ischaemic encephalopathy (http://www.cochrane.org/reviews/en/ab003311.html) D: False * No evidence E:False * The jury is still out on TBI... * Reductions in risk of mortality were greatest and favorable neurologic outcomes much more common when hypothermia was maintained for more than 48 h. However, this evidence comes with the suggestion that the potential benefits of hypothermia may likely be offset by a significant increase in risk of pneumonia...Journal of Neurotrauma

AM22 [Aug95] [Jul98] [Apr99] [Aug99] (type A) Patient with suspected malignant hyperpyrexia. Which of the following is/are true? A. Chance of affected sibling is 25% B. CK level is elevated at all times C. Report of a case occurring 48hrs after anaesthesia D. It is possible for the patient to have had a previous anaesthetic & not triggered a reaction

ANSWER C and D A ? False Close to 50% (OHA)Originally thought to be dominant but genetics not really understood. (Blue Book 2005) B False Peak CK at 24hrs. (CEPD 2003) CK levels have been considered as screening tool. "... some families in which people susceptible to MH have high CK levels." (Australian Anaesthesia 2005, p42) C True may rarely present 2-3 days post op with ARF (OHA) D True 50-75% of patients have had a prev anaesth without problems (OHA) Two-thirds of MHS patients manifest during their first GA One-third of MHS patients manifest during subsequent GAs

You intubate a young male patient for a left thoracotomy with a 39FG Robert Shaw tube. When you inflate both cuffs and ventilate the bronchial lumen you get left sided ventilation. When you attempt to ventilate the tracheal lumen the pressures are very high and you get no air entry. Yet when you deflate both cuffs you can ventilate the patient through the tracheal lumen. The most appropriate step to take next is: A. Change to a 41FG tube B. Change to a 37FG tube C. Deflate both cuffs and insert further cm and recheck D. Deflate both cuffs and withdraw a few cm and recheck E. Pull ETT out and start again.

ANSWER C or D C. is true if the bronchial cuff has herniated into the carina. It needs to be pulled back. D. is true if the entire tube is endobronchial. It needs to be pulled back.

A patient with pulmonary hypertension secondary to lung disease presents for laparotomy. Regarding this patient's anaesthetic management: A. an alpha-agonist is the inotrope of choice B. hypothermia is protective against rises in pulmonary artery pressure C. isoflurane will tend to decrease pulmonary artery pressure D. ketamine is an appropriate anaesthetic agent E. right heart failure is not a concern

ANSWER C?

A 30-year-old man presents to the emergency department following a high speed motor vehicle accident. His blood pressure is 70/50 mmHg with a strongly positive FAST (focused abdominal sonography for trauma). His chest X-ray shows a widened mediastinum. The most appropriate method to assess the widened mediastinum in this patient is A. aortogram B. CT angiogram of the chest C. repeat chest X-ray D. intraoperative TOE (transoesophageal echocardiography) E. transthoracic echocardiography

ANSWER D

A 62 year old man has chronic renal failure. You notice his total serum calcium is 2.05 mmol/L. This is because he has A. high serum vitamin D B. hypoparathyroidism C. primary hyperparathyroidism D. secondary hyperparathyroidism E. tertiary hyperparathyroidism

ANSWER D

A 65 year old man one week after an appendectomy suddenly experiences left sided (?pleuritic) chest pain with dyspnoea. The previous night he had been restless with a temperature of 37.2C. ECG was normal. No leg signs. Immediate therapy would be: A. Intranasal oxygen and serial chest xrays B. Heparinisation C. Antibiotics D. Observe closely for further signs E. Chest physiotherapy

ANSWER D

A 75yo male with moderate aortic stenosis (valve area 1.1cm2). Gets mild dyspnoea on exertion but otherwise asymptomatic. Needs hip replacement. A. Continue with surgery B. Beta block then continue C. Get myocardial perfusion scan D. Postpone surgery awaiting AVR E. Postpone surgery awaiting balloon valvotomy

ANSWER D

A man is working with electrical appliances at home with a residual current device. If he touches the active and the neutral (was it neutral or earth) wire he will suffer a. A microshock b. A macroshock c. Nothing happens because the fuse blows d. The RCD will protect him from macroshock

ANSWER D

A patient presents with a herniated disc complaining of pain in the left buttock and posterior thigh, numbness on the left lateral malleolus and on exam has decreased left Achilles reflex. What level is the herniation? A. L2-L3 B. L3-L4 C. L4-L5 D. L5-S1

ANSWER D

A three-week old infant presents with pyloric stenosis and 3 days of vomiting. A typical electrolyte profile would be A. Na+ 130 mmol.l-1 K+ 5.8 mmol.l-1 Cl- 98 mmol.l-1 HCO3- 17 mmol.l-1 B. Na+ 148 mmol.l-1 K+ 4.1 mmol.l-1 Cl- 108 mmol.l-1 HCO3- 13 mmol.l-1 C. Na+ 135 mmol.l-1 K+ 4.5 mmol.l-1 Cl- 90 mmol.l-1 HCO3- 30 mmol.l-1 D. Na+ 130 mmol.l-1 K+ 2.8 mmol.l-1 Cl- 90 mmol.l-1 HCO3- 28 mmol.l-1 E. Na+ 130 mmol.l-1 K+ 3.9 mmol.l-1 Cl- 98 mmol.l-1 HCO3- 17 mmol.l-1

ANSWER D

AM48 [Jul07] Shown a family tree... 6 year old boy coming for routine operation. Maternal Great Grandfather has had a malignant hyperthermia reaction under GA. Which of the following is most likely to rule out that the 8 year old will NOT suffer a MH reaction A. 8 year old has negative resting CK level B. 8 year old has had a previous operation before with no problems C. mother has had negative genetic testing D. grandfather has had negative muscles testing E. father has had an operation before with no problems

ANSWER D

An 8-year-old boy who is wheelchair bound with cerebral palsy and spastic quadriplegia is booked for an MRI (magnetic resonance imaging), under general anaesthesia for a rapid deterioration in his epilepsy control. On arrival to the MRI suite (with no premedication) he is distressed and inconsolable. Immediate management of this situation should include A. an inhalation induction with parental/carer presence B. cancellation and rescheduling with sedative premedication C. cancellation of procedure until epilepsy is better controlled D. detailed discussion with parents/carer regarding anaesthetic options E. intramuscular or oral ketamine induction

ANSWER D

An infant born at 32 weeks gestational age comes at 6 weeks for elective bilateral inguinal hernia repair. The parents expect to take him home that day. What do you tell them? A. He cannot have surgery until he is 3 months old B. They can take him home that day C. They can take him home with apnoea monitoring overnight D. He needs to stay in hospital for apnoea monitoring

ANSWER D

Blunt liver trauma can be treated non surgically if A. No peritoneal signs B. Low Grade injury on CT scan C. Severe COPD D. Haemodynamically stable E. US confirms <500mls peritoneal fluid collection

ANSWER D

Blunt liver trauma can be treated non surgically if A. No peritoneal signs B. Low Grade injury on CT scan C. Severe COPD D. Haemodynamically stable E. US confirms <500mls peritoneal fluid collection (i thought this was a paracentesis result)

ANSWER D

Circuit disconnection during spontaneous breathing anaesthesia A. will be reliably detected by a fall in end-tidal carbon dioxide concentration B. will be detected early by the low inspired oxygen alarm C. will be most reliably detected by spirometry with minute volume alarms D. may be detected by an unexpected drop in end-tidal volatile anaesthetic agent concentration E. can be prevented by using new, single-use tubing

ANSWER D

Circuit disconnection during spontaneous breathing anaesthesia A. will be reliably detected by a fall in end-tidal carbon dioxide concentration B. will be detected early by the low inspired oxygen alarm C. will be most reliably detected by spirometry with minute volume alarms D. may be detected by an unexpected drop in end-tidal volatile anaesthetic agent concentration E. can be prevented by using new, single-use tubing

ANSWER D

Compared to retrobulbar block, peribulbar block is associated with A. More bleeding B. More risk to optic nerve C. More akinetic eye D. Less block to orbicularis oculi

ANSWER D

During prolonged trendelenburg positioning there is: A. No change in ICP B. No change in IOP C. Increased pulmonary compliance D. Increased myocardial work E. No increased pulmonary venous pressures

ANSWER D

During prolonged trendelenburg positioning there is: A. No change in ICP B. No change in IOP C. Increased pulmonary compliance D. Increased myocardial work E. No increased pulmonary venous pressures

ANSWER D

Eisenmengers syndrome: A: decrease Hb to <180 with venesection B: Give high FiO2 C: Pulmonary vasodilators will treat the pulmonary hypertension D: terminal RV failure usually occurs in the 3rd-4th decade

ANSWER D

Hyperkalaemia is associated with A. Conn's syndrome B. Bartter's syndrome C. Cushing's syndrome D. Addison's disease

ANSWER D

Hypoxaemia may be relieved by assisted ventilation with: A. Rapid early inspiratory phase B. Steady rise in inspiratory pressure C. Inspiratory phase of less than one second D. PEEP

ANSWER D

In assessing the adequacy of oxygen delivery to meet the body's oxygen demands the best indicator is A. arterial PO2 B. arteriovenous oxygen content difference C. oxygen flux calculation D. mixed venous PO2 E. cardiac output

ANSWER D

In patients undergoing thoracotomy, techniques which reduce the incidence of intraoperative atrial fibrillation include A. hyperventilation B. pre-operative loading with digoxin C. rocuronium, rather than pancuronium D. thoracic epidural bupivacaine E. thoracic epidural morphine

ANSWER D

In the diagnosis of postoperative myocardial infarction A. echocardiography has little role to play B. infarct size can be judged from cumulative enzyme release C. new left bundle branch block on ECG confirms acute infarction D. serum troponins have high sensitivity and are detectable for up to 5 days E. ST segment elevation is extremely common

ANSWER D

In the placement of a needle for a coeliac plexus block, the correct position can be ascertained by: A. The use of lignocaine immediately prior to phenol B. Patient reporting immediate warmth C. Use of a nerve stimulator D. Use of an image intensifier

ANSWER D

Laparoscopic cholecystectomy patient with hyperparathyroidism and ionised calcium of 2.0 mmol/l. The BEST initial treatment is: A. Calcitonin B. Magnesium C. Dialysis D. IV fluids E. Frusemide

ANSWER D

Long duration of surgery, arms stretched out, head turned 30 degrees to right. On waking patient has a neurological deficit. Sensory loss over ventral lateral palm and 3 fingers, some weakness of the hand, weakness of the wrist, some paraesthesia of the forearm and weak elbow flexions. Most likely injury is A. Median nerve B. Ulnar nerve C. C5 nerve root D. Upper cervical trunk E. Musculocutaneous

ANSWER D

Management of rhabdomyolysis - best option? A. Haemodialysis B. Bicarbonate C. Frusemide D. IV fluids E.

ANSWER D

Multiple attempts to place ETT during difficult intubation causing pharyngeal and oesophageal perforations. Most likely cause of death? A: Failure to intubate B: Failure to ventilate C: Bleeding D: Sepsis

ANSWER D

Muscles which abduct the vocal cords include the A. aryepiglottic B. cricothyroid C. lateral cricoarytenoid D. posterior cricoarytenoid E. thyroarytenoid

ANSWER D

Myaesthenia gravis - features predicting need for post op ventilation EXCEPT A. Prolonged disease B. High dose Rx C. Previous respiratory crisis D. Increased sensitivity to NMB's E. bulbar dysfunction

ANSWER D

National labelling standards endorsed by ANZCA. What colour should the label on a brachial plexus catheter infusion be? A. Red B. Blue C. Beige D. Yellow E. Pink

ANSWER D

Neonate and intubation: A. use 2.5mm internal diameter ETT B. uncuffed tube has no Murphys eye C. external diameter of a reinforced ETT is the same as a normal ETT D. use same size for oral and nasal E. Something about a 4.0 ETT has an external diameter of 4mm F. Age/4 + 4 is formula for external diameter

ANSWER D

Obstructive sleep apnoea: upper airway muscles vs diaphragm; not as sensitive to: A. IV induction agents B. Inhalation agents C. Opioids D. Neuromuscular blockers

ANSWER D

PZ102 ANZCA version [2005-Sep] Q122, [Mar06] Q27 Clopidogrel A. acts via the glycoprotein IIb/IIIa receptors on platelets B. binds reversibly to its receptor on the platelet C. blocks thrombin mediated platelet activation D. inhibits ADP induced platelet activation E. is inhibited by concurrent aspirin usage

ANSWER D

PZ78b ANZCA version [2001-Apr] Q34, [2004-Aug] Q53, [2005-Apr] Q40, [Mar06] When paracetamol is used in infants and children, A. a dose of 10 mg.kg-I is more effective than placebo for relief of symptoms of tonsillitis B. the bioavailability of rectal suppositories is less than 50% of that from an equivalent oral dose C. a rectal loading dose of 45 mg.kg-I will reliably produce therapeutic plasma levels with a peak concentration after 1 hour D. the elimination half life is 2 - 2.3 hours E. a far greater proportion of unmetabolised paracetamol is excreted by the kidney, compared to adults

ANSWER D

Patho-physiological features of patients with morbid obesity include A. a blood volume:body weight ratio which is similar to that of patients with normal body weight B. an increased blood pressure and systemic vascular resistance compared to that of patients with normal body weight C. decreased gastric motility due to increased gastrin secretion D. cardiac pathology resulting from excess body mass and increased metabolic demand E. cardiac pathology resulting mainly from fatty infiltration or fatty change of the heart

ANSWER D

Patient on ward, collapsed. Nurse calls code blue when finds pt unresponsive. No signs of life. After ensuring airway is clear, 1st action A. DCR x3 200J B. DCR x1 200J C. precordial thump D. CPR E. Adrenaline 1mg

ANSWER D

Patient post anterior cervical fusion. Patient in recovery. Confused and combative. Nurse concerned about haematoma. Taken to theatre: Most appropriate way of securing airway: A. Gas induction / laryngoscopy / intubate B. Awake tracheostomy C. Awake fibreoptic intubation using minimal sedation D. Thiopentone, suxamethonium, direct laryngoscopy and intubation E. Retrograde intubation

ANSWER D

Patient presents with carcinoid syndrome and developes hypotension intraoperatively. Best drug to treat it is: A. Noradrenaline B. Adrenaline C. Metaraminol D. Octreotide E. Ephedrine

ANSWER D

Plasma clearance of non-depolarising muscle relaxants in pregnant patients (when compared with matched non-pregnant controls) is: A. reduced because the distribution half-life is prolonged due to the increased circulating blood volume in late pregnancy B. reduced because the elimination half-life is prolonged due to delayed hepatic & renal clearance brought about by hormonal changes in pregnancy C. increased because the distribution half-life is shortened due to changes in cardiac output in pregnancy D. increased because the elimination half-life is shortened due to increased hepatic and renal clearance brought about by hormonal changes in pregnancy E. increased because the distribution half-life is shortened due to the placental transfer of the relaxant to fetal & placental tissue.

ANSWER D

Pneumoperitoneum to a pressure greater than 10 mmHg for laparoscopic surgery usually results in A. a decreased arterial to end-tidal CO2 difference B. a decrease in airway resistance C. an increase in pulmonary compliance D. an increase in physiological dead space E. an increase in functional residual capacity (FRC)

ANSWER D

Post dural punture headache A. 24hour bed rest B. Prone position worst C. Increase incidence with insertion of spinal catheter D. Hearing loss

ANSWER D

Post epidural and LSCS, the next day patient have persistent paraesthesia anterior thigh. What other injuries would indicate more of nerve roots instead of peripheral nerve injuries A. Weakness on hip flexion and thigh adduction B. Weakness on knee flexion and plantar flexion C. Urinary incontinence D. Foot drop

ANSWER D

Post operative left pneumonectomy. What to do with underwater seal drain? A: Nurse patient in R lateral decubitus position B: Expect to see bubbles C: Suction every hour for 5 minutes D: Unclamp drain once an hour for 5 minutes, leave clamp on for the rest of the time E: Leave on free drainage

ANSWER D

Post operative left pneumonectomy. What to do with underwater seal drain? A: Nurse patient in R lateral decubitus position B: Expect to see bubbles C: Suction every hour for 5 minutes D: Unclamp drain once an hour for 5 minutes, leave clamp on for the rest of the time E: Leave on free drainage

ANSWER D

Postoperative diffusion hypoxia is associated with: A. Prolonged operation with N20 B. High concentrations of N20 C. Low cardiac output D. Postoperative hypoventilation

ANSWER D

Pre-ganglionic sympathetic fibres pass to the A. otic ganglion B. carotid body C. ciliary ganglion D. coeliac ganglion E. all of the above

ANSWER D

SF57 ANZCA version [2003-Aug] Q143, [2004-Apr] Q66, [Mar06] During elective caesarean section under spinal anaesthesia A. maternal hypotension requiring intervention is infrequent B. the duration that the maternal systolic blood pressure is below 100 mmHg is of less importance for producing adverse cord blood acid-base measures than the degree of fall in systolic blood pressure C. there is a significant difference between the use of rescue boluses compared to infused prophylactic ephedrine on the status of cord blood acid-base measures D. there is less nausea and vomiting if ephedrine is prophylactically infused compared to using it as a rescue bolus to treat any maternal hypotension when it does occur E. there is no adverse effect from maternal hypotension on cord blood acid-base values

ANSWER D

The MAC (minimal alveolar concentration) and blood/gas partition coefficient of desflurane are A. 2% and 0.47 respectively B. 4% and 0.47 respectively C. 6% and 0.67 respectively D. 6% and 0.42 respectively E. 8% and 1.4 respectively

ANSWER D

The depth of unconsciousness in a patient with diabetic ketoacidosis correlates best with the degree of: A. Hyperglycaemia B. pH alteration C. Hyponatraemia D. Hyperosmolarity E. Hypokalaemia

ANSWER D

The following blood gas result is from a patient breathing room air who has been unwell for three days pH 7.56 PCO2 46 mmHg PO2 90 mmHg HCO3 39 mEq.L-1 B.E. +16.2 mEq.L-1 These results can best be described asall A. primary respiratory acidosis with metabolic compensation B. uncompensated primary metabolic alkalosis C. compensated secondary respiratory alkalosis D. primary metabolic alkalosis with partial respiratory compensation E. mixed metabolic and respiratory alkalosis

ANSWER D

The following statements regarding an epidural blood patch is FALSE A. Better results if rest in bed for several hours after patch B. May cause neck and back pain C. Is reported to be very effective D. Always reduces the effect of the next block because obliterates space E. Wait until resolution of sensory blockade

ANSWER D

The most common complication of blunt cardiac trauma is A. myocardial ischaemia B. valve rupture C. right ventricular rupture D. arrhythmias E. tamponade

ANSWER D

The most direct branch of the internal carotid artery: A. Ophthalmic artery B. Choroidal artery C. Anterior cerebral artery D. Middle cerebral artery E. Posterior cerebral artery

ANSWER D

The most frequently reported clinical sign in malignant hyperpyrexia is A. arrhythmia B. cyanosis C. sweating D. tachycardia E. rigidity

ANSWER D

The most important factor in reducing peri-operative morbidity in diabetic patients undergoing peripheral vascular surgery is A. tight control of blood sugar level in the peri-operative period B. frequent blood sugar level estimations C. the use of regional rather than general anaesthesia D. stabilisation of co-existing disease the use of an insulin infusion rather than a E. subcutaneous sliding scale regimen

ANSWER D

The most important factor in reducing peri-operative morbidity in diabetic patients undergoing peripheral vascular surgery is A. tight control of blood sugar level in the peri-operative period B. frequent blood sugar level estimations C. use of regional rather than general anaesthesia D. stabilisation of co-existing disease E. the use of an insulin infusion rather than a subcutaneous sliding scale regimen

ANSWER D

The muscles of the larynx which separate the vocal cords are the A. thyro-arytenoids B. lateral crico-arytenoids C. oblique arytenoids D. posterior crico-arytenoids

ANSWER D

The prevalence of latex positive latex skin prick in atopic individuals with increased exposure to latex is about: (a) 0.28 % (b) 1.4 % (c) 7 % (d) 35%

ANSWER D

The site of a lesion producing blindness with preservation of the pupillary reflexes is: A. Optic chiasma B. Optic tract C. Frontal lobe D. Occipital lobe E. None of the above

ANSWER D

The use of epidural opioids for analgesia after caesarean section is NOT associated with A. respiratory depression B. activation of herpetic lesions C. facial pruritis D. a higher incidence of nausea and vomiting than occurs with other techniques of administration E. an increased incidence of urinary retention

ANSWER D

Thermoneutral zone in 1 month old infant ? A. 26 - 28 degrees Celcius B. 28 - 30 degrees Celcius C. 30 - 32 degrees Celcius D. 32 - 34 degrees Celcius E. 34-36 degrees celcius

ANSWER D

Trauma patient with GCS 6 with hard collar. HR and BP unstable. What is the best way of clearing neck? A. CT B. Cervical spine trauma series C. MRI D. Leave had collar indefinitely E. Clinically

ANSWER D

Troponin is elevated post-infarct A. 1-2 days B. 2-5 days C. 5-14 days D. 7-21 days E ?

ANSWER D

What is the most common way to measure end tidal gas concentrations on our anaesthetic machines? A: mass spectometry B: Raman scattering C: ultrasonic D: infrared E: piezoelectric

ANSWER D

When a circle absorption system and tracheal intubation are in use with an adult subject, rebreathing of expired gas becomes practically negligible if A. spontaneous breathing is replaced by intermittent positive pressure breathing B. the minute volume is 8 L.min-1or more C. the carbon dioxide absorbent is fresh and has an intergranular space of at LEAST one litre D. a fresh gas flow rate of at LEAST 8 L.min-1 is used

ANSWER D

When performing cardioversion for new onset atrial fibrillation, what is the safe maximum time between onset and cardioversion without anticoagulation? A. 12 hours B. 24 hours C. 36 hours D. 48 hours E. 72 hours

ANSWER D

Which does NOT minimise intravenous cannulation with epidural insertion? A. Injection saline through epidural needle before catheter insertion B. Lie patient lateral C. Do combined spinal-epidural technique (CSE) D. Thread catheter slowly E. ?

ANSWER D

Which of the following best describes the number of cases of a disease in a given population at a specific time: A. Incidence B. Occurrence C. Frequency D. Prevalence E. Rate

ANSWER D

Which of the following is NOT associated with serotonin syndrome A. phenelzine B. pethidine C. ondansetron D. chlorpromazine E. sumatriptan

ANSWER D

Which of the following is most effective way to reduce renal failure in AAA surgery A. Fluid bolus prior to aortic clamping B. Fluid bolus after aortic clamp release C. Frusemide D. Minimize cross-clamp time E. Mannitol

ANSWER D

Why should NSAIDs be avoided in pregnant women >30 weeks gestation? A. cause neonatal acute renal failure B. increased antepartum haemorrhage C. increased rate of pre-eclampsia D. cause closure of the fetal ductus arteriosus E. increase preterm labour

ANSWER D

With an opioid addict, which would you recommend for postop analgesia following an operation (?cholecystectomy) ? A. Oral pentazocine B. Pethidine 100 mg 4th hourly IMI C. PCA 1 mg bolus, 5 minute lockout D. PCA 2 mg bolus with background infusion E. Oral analgesia

ANSWER D

You are called to see a 30 year old man with rapidly deteriorating asthma. Following appropriate medical management an endotracheal tube is inserted and he is ventilated with a mechanical ventilator with a tidal volume of 600 ml and a rate of 12 breaths per minute. Five minutes later the blood pressure is unrecordable and external cardiac massage is commenced. Arterial blood is taken and shows pH 7.08, pCO2 96 mmHg, pO2 36 mmHg, oxygen saturation 46% and bicarbonate 27 mmol.L-1. He is administered adrenaline, salbutamol, pancuronium, bicarbonate and calcium gluconate. The ECG shows sinus rhythm at a rate of 60 beats per minute. The patient remains pulseless and cyanosed with fixed dilated pupils and distended neck veins. The most appropriate continuing management is A. cease resuscitation B. administer further adrenaline C. insert bilateral intercostal drains D. cease ventilation for 30 seconds and resume at a slower rate E. increase peak inspiratory pressure

ANSWER D

You are on call for a maternity hospital. Your junior registrar calls you after having inserted a labour epidural in an extremely anxious 19 yo parturient, and obtained blood in the catheter. He informs you the epidural space was found by LOR at 6cm and the catheter has been inserted to 12 cm. Your first instruction should be: A. Flush with saline then check again for blood (NOT an option - option was just flush with saline and secure and use) B. Aspirate again for blood C. Give 3mls 2% lidnocaine with 1:200 000 adrenaline D. Pull back 2cm and check again for blood E. Remove epidural and start again

ANSWER D

The most common diagnosis of the cause of blindness following anaesthesia in the prone position is: A. drug effect B. emboli C. globe compression D. ischaemic optic atrophy E. psychiatric disturbance

ANSWER D Ischemic optic neuropathy is the most common diagnosis in postoperative visual loss. Ischemic optic neuropathy is divided into anterior and posterior, depending upon the location of the lesion on the optic nerve. Anterior ischemic optic neuropathy -53% cardiopulmonary bypass procedures -12% prone Posterior ischemic optic neuropathy -48% neck, nose or sinus operations -16% prone -11% cardiopulmonary bypass procedures

Complications of coeliac plexus block include A. hypertension B. failure of erection C. constipation D. paraplegia E. dysaesthesia along L3-4

ANSWER D * 3 common transient adverse effects 1. Local pain (96%) 2. Diarrhoea (44%) 3. Hypotension (38%) * Other complications include o Lower extremity weakness o Paraplegia o Parasthesia o Adjacent organ puncture o Infection o Bleeding → retroperitoneal haematoma o Epidural injection o Subarachnoid injection o Intravascular injection o Pneumothorax o Chylothorax * Neurology caused by o Direct injury to spinal cord or somatic nerves o Spinal cord ischaemia

In patients with phaeochromocytoma A. paroxysmal hypertension is the most common presentation B. initial treatment should be with beta blockade if tachycardia is present C. excesses of adrenaline and noradrenaline occur with equal frequency D. urinary Vanillyl Mandelic Acid (VMA) studies may be normal E. extra-adrenal tumours occur in 2% of patients

ANSWER D * A False - 85% have sustained hypertension. Whilst I don't know where 85% comes from there is discrepency with serum peaks of NA and Adr with BP due to receptor sensitivity and vascular reactivity (CEACCP) o 50 % have paroxysmal hypertension, 30 % have sustained hypertension, 20 % have none (Stoelting 5th ed.) - True * B False - α blockers first then β blockers o A distractor, as in text from Stoelting...if tachycardia (>120) after α blockade, then non-specific β blockers added * C False - Some Ad > Norad, Some Norad > Ad * D True - not the most sensitive and specific test o VMA's are oldest and least expensive test but non-specific o The glucagon stimulation test is the most specific test * E False - 6% in other sites in sympathetic system o 20 % extra-adrenal (Stoelting 5th ed.)

SF29b [Mar91] Magnesium for pre-eclampsia by IV infusion: A. Lengthens QT B. Produces hypotension C. Prolongs labour D. Prolongs AV conduction E. Side effects include AV bradycardia

ANSWER D * A True edit disagree. if it did would you use it for Torsades? Magnesium has no effect on QT intervals. * B ?True - can → hypotension. * C False * D True * E ?True

In preeclampsia A. once delivery of the placenta takes place, the condition improves B. in the absence of other risk factors a platelet count of greater than 50 is adequate for epidural anaesthesia C. corticosteroid therapy has no effect on the severity of thrombocytopenia D. magnesium sulphate halves the incidence of eclampsia E. spinal anaesthesia contraindicated

ANSWER D * A, truish... the condition usually/eventually improves... but not always * B, false... 70 seems to be the current line in the sand, but how low is really safe?? Obstetric Anaesthesia Scientific Evidence from ANZCA gives 75 as a safe figure. * C, having wasted far too much time in the literature, this is probably false (eg [1] (http://www.ncbi.nlm.nih.gov/entrez/query.fcgi?db=pubmed&cmd=Retrieve&dopt=AbstractPlus&list_uids=15802415&itool=iconabstr&query_hl=9&itool=pubmed_docsum)), especially in HELLP [2] (http://bmj.bmjjournals.com/cgi/content/full/329/7460/270)... but it could be true [3] (http://www.ncbi.nlm.nih.gov/entrez/query.fcgi?db=pubmed&cmd=Retrieve&dopt=AbstractPlus&list_uids=14973983&itool=iconabstr&query_hl=9&itool=pubmed_docsum) * D, TRUE as abstract says: "Magnesium sulphate halves the risk of eclampsia, and probably reduces the risk of maternal death. There do not appear to be substantive harmful effects to mother or baby in the short term." From MAGPIE * E. false

AZ18 [Aug95] Pupillary dilatation may be seen with: A. Metaraminol B. Trimetaphan C. Cardiac asystole D. Stellate ganglion block

ANSWER D * A. Metaraminol - maybe true: Metaraminol is a mixed-acting drug but "has relatively stronger direct α1-adrenergic-stimulating effects" (Miller Ch 12). Note that alpha 1 effects are to constrict, so any pupillary dilation will be due to indirect effects acting on beta receptors. * B. Trimethaphan - true: Trimethaphan is a ganglion-blocking drug. "The ganglion-blocking drugs cause a predictable cycloplegia with loss of accommodation because the ciliary muscle receives innervation primarily from the parasympathetic nervous system. The effect on the pupil is not so easily predicted, since the iris receives both sympathetic innervation (mediating pupillary dilation) and parasympathetic innervation (mediating pupillary constriction). Ganglionic blockade often causes moderate dilation of the pupil because parasympathetic tone usually dominates this tissue." (Katzung, Basic and Clinical Pharmacology, 11E, Chapter 8, Cholinoceptor-Blocking Drugs) * C. Cardiac asystole - maybe true: cardiac asystole per se should not cause dilated pupils. However, presumably a sympathetic response to the asystole will cause pupillary dilation, as will brain death secondary to asystole. * D. Stellate ganglion block - false: Horner's syndrome: meiosis, ptosis, enophthalmos, anhidrosis

Regarding decontamination of anaesthetic equipment A. alcohol is sporicidal B. disinfection is sporicidal C. phenol is sporicidal D. sterilisation with ethylene oxide requires 5-12 hours to work E. sterilisation with glutaraldehyde requires 5-8 exposure hours to work

ANSWER D * A. alcohol is sporicidal - false: " A few disinfectants have sporicidal activity with prolonged exposure and are called high-level disinfectants. It is this level of disinfection that is commonly used for semi-critical anaesthetic and respiratory therapy equipment. High-level disinfectants include gluteraldehyde, stabilized hydrogen peroxide, peracetic acid, chlorine and chlorine-releasing compounds. Low-level disinfectants will kill most vegetative bacteria and some viruses and fungi. Examples include alcohols, sodium hypochlorite and iodophore solutions." * B. disinfection is sporicidal - false: See above * C. phenol is sporicidal - false: See Table 2 * D. sterilisation with ethylene oxide requires 5-12 hours to work - true: "In the US, ethylene oxide is used commonly for the sterilization of heat- and moisture-sensitive devices. It is a colourless gas and very flammable; risks of fire are reduced by dilution with inert gases such as carbon dioxide or hydroflurocarbons. Microbiocidal activity is thought to be the result of alkylation of protein, DNA and RNA. Temperatures of 29-65°C are employed and cycles are 5-12 h." * E. sterilisation with glutaraldehyde requires 5-8 exposure hours to work - false: "Immersion in gluteraldehyde 2% is a form of sterilization and is used for optical instruments such as cytoscopes or bronchoscopes as it is non-corrosive and has no deleterious effects on lens cement. Immersion must be >10 h; less time will result in disinfection, not sterilization."

In infants with congenital pyloric stenosis A. dehydration is associated with early hyponatraemia B. plasma chloride levels seldom fall below 85 mmol.l-1 C. renal conservation of hydrogen and potassium ions occurs D. the urine is initially alkaline, then may become acidic E. vomiting causes a loss of potassium ions

ANSWER D * A. dehydration is associated with early hyponatraemia * B. plasma chloride levels seldom fall below 85 mmol.l-1 * C. renal conservation of hydrogen and potassium ions occurs - false: Hydrogen exchanged for potassium leading to hypokalaemia * D. the urine is initially alkaline, then may become acidic - true and most correct answer: "Vomiting normally results in equal loss of gastric acid and alkaline duodenal fluid. However, with gastric outlet obstruction, gastric acid along with a variable amount of Na+ and K+ is lost in vomitus without loss of HCO3− from the duodenum. The resultant systemic alkalosis overwhelms the capacity of the proximal convoluted tubule to reabsorb HCO3−, producing an alkaline urine. As extracellular fluid volume depletion increases, aldosterone is secreted, leading to renal conservation of Na+ in exchange for K+. Eventually hypokalaemia forces Na+ exchange preferentially with H+ instead of K+ in the renal tubules. This produces the characteristic 'paradoxical acid urine' in the face of systemic alkalosis." (Anaesthesia & Intensive Care Medicine Volume 9, Issue 4, April 2008, Pages 133-141) * E. vomiting causes a loss of potassium ions - true, but a variable amount: "Vomiting normally results in equal loss of gastric acid and alkaline duodenal fluid. However, with gastric outlet obstruction, gastric acid along with a variable amount of Na+ and K+ is lost in vomitus without loss of HCO3− from the duodenum" (Anaesthesia & Intensive Care Medicine Volume 9, Issue 4, April 2008, Pages 133-141)

PL28 ANZCA version [2004-Apr] Q137, [2004-Aug] Q59, [Jul 06] Q41 Intra-nasal topical cocaine used in nasal surgery A. has a duration of action of the order of 6 hours B. is metabolised more quickly by the liver if the patient is using ecothiopate eye drops C. is typically used in a dose of approximately 5 ml of 5% solution in an adult D. may be metabolised more slowly in patients with liver disease E. reaches a peak plasma concentration in 3 hours

ANSWER D * A. has a duration of action of the order of 6 hours - false o "There has been an increase in both IV administration and inhalation of pyrolyzed cocaine via smoking. Following intranasal administration, changes in mood and sensation are perceived within 3-5 min, and peak effects occur at 10-20 min. The effects rarely last more than 1 h. "(Harrisons Ch389) o "Peak anesthetic effect following topical application of cocaine or lidocaine occurs within 2 to 5 minutes (3 to 8 minutes with tetracaine), and anesthesia lasts for 30 to 45 minutes (30 to 60 minutes with tetracaine)." (Goodman and Gilman) o "Peak venous plasma concentrations of cocaine are achieved at approximately 30 to 40 minutes after intranasal administration... The duration of effects is approximately 60 minutes or longer after peak effects." (Stoelting p.178) * B. is metabolised more quickly by the liver if the patient is using ecothiopate eye drops - false: "Demecarium, echothiophate, and isoflurophate are indirect-acting parasympathomimetic agents, which are also known as cholinesterase inhibitors and anticholinesterases. {37} {60} {63} {88} Cholinesterase inhibitors prolong the effect of acetylcholine, which is released at the neuroeffector junction of parasympathetic postganglion nerves, {37} by inactivating the cholinesterases that break it down" (http://www.drugs.com/mmx/ecothiopate-iodide.html) * C. is typically used in a dose of approximately 5 ml of 5% solution in an adult - false: Beyond safe dose. Dose = 50mg/ml*5ml=250mg; "Maximal safe total dosages for topical anesthesia in a healthy 70-kg adult are 300 mg for lidocaine, 150 mg for cocaine, and 50 mg for tetracaine." (Goodman and Gilman Ch 14) * D. may be metabolised more slowly in patients with liver disease - true: "Cocaine is metabolised by plasma and liver cholinesterases to water-soluble metabolites that are excreted in urine. Plasma choliesterase activity is decreased in parturients, neonates, the elderly, and patients with severe underlying hepatic disease." (Stoelting Pharmacology p 165) * E. reaches a peak plasma concentration in 3 hours - false: See answer A

SF75 ANZCA version [2003-Aug] Q23 Oxytocin, when administered intravenously in a bolus dose of 10 units immediately postpartum A. has a half life of approximately 30 minutes B. commonly induces vomiting C. commonly causes premature atrial contractions D. often lowers blood pressure E. causes transient bradycardia

ANSWER D * A. has a half life of approximately 30 minutes----FALSE "The relative ease with which the rate and force of uterine contractions can be regulated by the intravenous infusion of Syntocinon is due to the short half-life of oxytocin. Values reported by various investigators range from 3 to 20 minutes." (Mims online) * B. commonly induces vomiting----Is a SE, but occuring commonly? * C. commonly causes premature atrial contractions-----Can do, but commonly? * D. often lowers blood pressure---- TRUE---Best answer "Another pharmacological effect observed with high doses of oxytocin, particularly when administered by rapid intravenous bolus injection, is a transient direct relaxing effect on vascular smooth muscle, resulting in brief hypotension, flushing and reflex tachycardia" (Mims online) * E. causes transient bradycardia----Possible. It causes Fetal bradycardia.

AC18b ANZCA version [2002-Aug] Q68, [2004-Apr] Q85 & [2004-Aug] Q98, [Jul06] Q92 (Similar question reported in [Jul98] [Apr99] [Apr07]) When providing general anaesthesia to a patient with a history of asthma A. thiopentone should not be used as it may cause bronchospasm B. intravenous and topical tracheal lignocaine are equally effective in preventing bronchial hyperreactivity C. ketamine provides little benefit in a patient with active wheezing D. induction with propofol is effective in reducing the incidence of wheezing following intubation E. isoflurane is as effective a bronchodilator as halothane when given in MAC equivalent doses

ANSWER D * A. thiopentone should not be used as it may cause bronchospasm - false: o "When general anesthesia is selected, induction of anesthesia is most often accomplished with an intravenous induction drug. The incidence of wheezing is higher in asthmatic patients receiving thiopental for induction than in those given propofol. Thiopental itself does not cause bronchospasm, but it may inadequately suppress upper airway reflexes so airway instrumentation may trigger bronchospasm." (From Stoelting's Anesthesia and Co-existing Diease 5E Ch 9 p.167) * B. intravenous and topical tracheal lignocaine are equally effective in preventing bronchial hyperreactivity - unsure but probably incorrect: o "An alternative method to suppress airway reflexes prior to intubation is the intravenous or intratracheal injection of lidocaine 1 to 3 minutes before tracheal intubation" (Stoelting's Anesthesia and Co-existing Disease, 5E, Ch 9, p. 167) * C. ketamine provides little benefit in a patient with active wheezing - false: o "Ketamine may produce smooth muscle relaxation and contribute to decreased airway resistance, especially in patients who are actively wheezing." (Stoelting's Anesthesia and Co-existing Disease, 5E, Ch 9, p. 167) * D. induction with propofol is effective in reducing the incidence of wheezing following intubation - true: o "When general anesthesia is selected, induction of anesthesia is most often accomplished with an intravenous induction drug. The incidence of wheezing is higher in asthmatic patients receiving thiopental for induction than in those given propofol... The mechanism of propofol's relative bronchodilating effect is unknown." (Stoelting's Anesthesia and Co-existing Disease, 5E, Ch 9, p. 167) * E. isoflurane is as effective a bronchodilator as halothane when given in MAC equivalent doses - false: o "The lesser pungency of halothane and sevoflurane (compared with isoflurane and desflurane) may make coughing, which can trigger bronchospasm, less likely" (Stoelting's Anesthesia and Co-existing Disease, 5E, Ch 9, p. 167)

To make the diagnosis of diabetes mellitus: A. Need 2 fasting BSLs > 6.0 mmol/l B. Need 2 fasting BSLs > 8 mmol/l with symptoms C. Patient must fast for 4 hours before a glucose tolerance test D. Patient must have at least 150g/day of carbohydrate for 3 days prior to a glucose tolerance test

ANSWER D * A: FALSE No. WHO: random BSL > 11.1 mmol/L (200 mg/dL) ONCE with symptoms or MORE THAN ONCE without symptoms. * B: FALSE No. WHO: random BSL > 11.1 mmol/L (200 mg/dL) ONCE with symptoms or MORE THAN ONCE without symptoms. * C: FALSE No. GTT involves overnight fast. (>8hours) Harrison's Ch. 334 * D: TRUE Diagostic Criteria - WHO DIABETES * Must confirm the diagnosis on second test if patient is asymptomatic * Random plasma glucose': >11.1 mmol/L * Fasting plasma glucose: >7 mmol/L Oral glucose tolerance test (OGTT) * 2 hours post 75g load plasma glucose > 11.1 mmol/L * Note: 3 days of > 150g/day carbohydrate. Fast 8 - 16hrs then give 75g glucose. This is because marked carbohydrate depletion can impair glucose tolerance. (Red book RCPA) HbA1c * >7% DM is likely (sensitivity 99%, specificity 99.6%) IMPAIRED GLUCOSE TOLERANCE Impaired Fasting Glucose * plasma glucose 6.1 -7.0 mmol/L Impaired Oral glucose tolerance test (OGTT) * 2 hours post 75g load plasma glucose 7.8 - 11.1 mmol/L

In the management of exposure to toxic nerve agents (highly potent anticholinesterases) A. early treatment with glycopyrolate is more effective than atropine B. ketamine is contra-indicated for endotracheal intubation C. oximes (eg.pralidoxime) in normal doses have no endogenous anticholinergic effects D. oximes reactivate AChE by cleavage of phosphorylated active sites E. pre-treatment with pyridostigmine prevents the effects of these agents

ANSWER D * A: False. "Atropine and oximes are effective antidotes if administered early after exposure. Atropine antagonizes muscarinic side-effects and is more beneficial than glycopyrrolate, which has a shorter half-life and does not cross the blood-brain barrier." * B: False. "Ketamine may be beneficial for intubation. But there is an increase in salivation. * C: False. "They (oximes) in normal doses have an endogenous anticholinergic effect." * D: True. "Administered early, oximes reactivate AChE by cleavage of phosphorylated active sites." * E: False. Does not prevent the effects. "Pyridostigmine bromide may be used as a pretreatment . . . (though it is not a true pretreatment, which would protect against nerve agents, but an "antidote enhancer") . . . The rationale of pyridostigmine pretreatment is that pyridostigmine carbamylation of AChE binding sites produces a reservoir of temporarily inactivated AChE. After exposure, nerve agents are unable to bind to the carbamylated enzyme. Later dissociation of pyridostigmine reactivates AChE, which, by hydrolysing acetylcholine, reduces the incidence of cholinergic crisis (in conjunction with atropine and pralidoxime).

Which one of the following is most likely to be associated with a high mixed venous oxygen saturation (SvO2)? A. acute myocardial infarction B. acute pulmonary embolism C. cardiac tamponade D. sepsis E. severe liver disease

ANSWER D * MIXED VENOUS pO2 (pVO2):"A normal or high pVO2 can also coexist with tissue dysoxia especially in high flow states such as sepsis". * MIXED VENOUS ARTERIAL SATURATION: "SvO2 values exceeding 0.8 are genserally seen in nhigh flow states such as sepsis, hyperthyroidism and severe liver disease".

Acromegaly is associated with the followig A. Superior laryngeal nerve palsy B. Glossoptosis C. Fixed vocal cords D. Prognathism

ANSWER D A False B False * Glossoptosis = abnormal downward or back placement of the tongue * Acromegaly → macroglossia C False * Abnormal cord movement, not fixed * Deepening of voice D True Acromegaly -rare clinical syndrome caused by excessive Growth Hormone Production (Anterior Pituitary) CVS : hypertension 30%, IHD, cardiomyopathy, heart failure, valvular disease Airway : large jaw, head, tongie, lips, hypertrophy of larynx and trachea, OSA, enlarged thyroid Drugs : -somatostatin causing nausea and vomiting -bromocriptine : postural hypotension Neurological -raised ICP -nerve entrapment Endocrine -25% diabetic

Which drug is contra-indicated in a patient with chronic renal failure presenting with a fractured femur? A. fentanyl B. morphine C. paracetamol D. pethidine E. oxycodone

ANSWER D Acute Pain Scientific Evidence * Fentanyl - no dose adjustment required * Oxycodone - no dose adjustment required * Morphine - reduced dose, alternative agent if high doses anticipated * Pethidine - dose reduction, altenative agent recoomended as norpethidine accumulation None contra-indicated but Pethidine best avoided

LUSCS for failure to progress. Spinal is inserted uneventfully. Next day the patient has foot drop. The most likely cause is? A. epidural haematoma B. lumbosacral palsy C. sciatic nerve palsy D. common peroneal palsy E. ?

ANSWER D "Peripheral nerve lesions are usually associated with a long labour and the use of forceps" "Postpartum foot drop is caused by damage to the lumbosacral trunk or, less frequently, the common peroneal nerve. The lumbosacral trunk (L4, L5) is compressed between the ala of the sacrum and the descending fetal head. It may also occur during a forceps delivery. The result is a unilateral foot drop with loss of sensation and/or paraesthesia along the lateral calf and foot." "Common peroneal nerve damage may occur due to improper or prolonged positioning during lithotomy and the sensory deficit may be limited to the dorsum of the foot." Epidural haematomas extremely rare (1:168,000 from review in Anaesthesiology 2006; 105: 394)and obstetric palsies are much more common than complications related to neuraxial blocks. Sciatic nerve injury would cause a foot drop but would also affect knee flexion (hamstrings) and all muscles in lower leg and foot. Common peroneal nerve palsy less likely in this case as there is no mention of stirrups or 'excessive knee holding'

Adverse effects of a remifentanil infusion for surgery under general anaesthesia include: A. 10% incidence of PONV B. Delayed respiratory depression after prolonged infusion C. Increased sensitivity to morphine postoperatively D. Muscle rigidity with an infusion rate 0.5 mcg/kg/min E. Pain on bolus injection

ANSWER D "Skeletal muscle rigidity can be caused by remifentanil and is related to the dose and speed of administration. Remifentanil may cause chest wall rigidity (inability to ventilate) after single doses of > 1 microgram/kg administered over 30 to 60 seconds or infusion rates > 0.1 microgram/kg/minute. Administration of doses < 1 microgram/kg may cause chest wall rigidity when given concurrently with a continuous infusion of remifentanil."

Tracheo-oesophageal fistula in a neonate is associated with all of the following EXCEPT: A. VSD B. Vertebral column abnormalities. C. Anal atresia (or: imperforate anus) D. Pulmonary hypoplasia. E. Renal abnormalities. F. ? Radial abnormalities

ANSWER D "VACTERL Association" V stands for vertebral anomalies, like hemi or block vertebrae. A stands for imperforate anus or anal atresia. C denotes cardiac anomalies such as VSD. TE stands for tracheoesophageal fistula. R stands for renal or anomalies and radial or arm anomalies. L is often added to stand for limb anomalies in case you forget.

A young woman with type 1 von Willebrand disease presents for a dilatation and curettage. She is a Jehovah's Witness. You consider administering intravenous desmopressin in an attempt to reduce haemorrhage. Which of the following statements regarding desmopressin is FALSE? A. it is a synthetic substance and is acceptable to Jehovah's Witnesses B. it is likely to reduce haemorrhage in this patient C. it should be given 30 minutes prior to surgery as an infusion D. its duration of effect is approximately 5 days E. the intravenous dose is 0.3 mcg.kg-1

ANSWER D * A - Clearly true, acceptable to JWs * B - Useful for type 1 vWD, so it is likely to reduce bleeding * C - Infuse in 50mL N saline over 30 min, 30 min prior to surgery sounds about right * D - False - elimination half-time 2.5-4.4 hours * E - Correct dose

A 30 year old female requires drainage of a large wound abscess following an abdominoplasty 1 week ago. She has been on enoxaparin (Clexane) 40 mg once a day for 8 days. She would probably have A. a prolonged prothrombin time (PT) B. a prolonged activated partial thromboplastin time (aPPT) C. normal coagulation, 12 hours after the last dose of enoxaparin D. a risk of bleeding, which would NOT be predicted by her antiXa activity E. her haemostatic function restored to normal after administration of 6 units of FFP (fresh frozen plasma) preoperatively

ANSWER D * A. False (Mosby) * B. False (Mosby + Rang and Dale) * C. False - some residual (although minor) effects at 12hrs (t1/2 is 7hrs, longer in renal impairment). In fact, Factor Xa effects at 12hrs are 50% of peak [2] (http://www.ncbi.nlm.nih.gov/pubmed/17215700) * D. Best answer - see below - Xa level is not predictive of bleeding complications unfortunately - see the above reference also. * E. False - Partial reversal is by protamine (FFP indicated for warfarin reversal)

AZ50 [Aug96] [Apr97] [Jul97] [Apr99] ANZCA guidelines for care of patients recovering from anaesthesia in the recovery area: A. Recovery bed must be tiltable from one end but not both (OR: Tilt head down but not head up) B. Ratio of registered nurses to patients must provide no less than one nurse to two patients, and one to any patient who remains unconscious C. Each bed space must be provided with a means of inflating the lungs with oxygen in a ratio of one per bed D. Recording of postop observations including SaO2 & temperature

ANSWER D * Observations should be recorded at appropriate intervals and should include state of consciousness, oxygen saturation, respiratory rate, pulse rate, blood pressure and temperature. * Bed must tilt from one or both ends both head up and head down at least 15 degrees * The ratio of registered nurses to patients needs to be flexible so as to provide no less than one nurse to three patients, and one nurse to each patient who has not recovered protective reflexes or consciousness. * Must be means for manual ventilation with oxygen in a ratio of one unit per two bed spaces, but with a minimum of two such devices

ANZCA 2007 Q72. The American Heart Association guidelines for pre-operative cardiac risk assessment define poor functional capacity as being only able to exercise at a level of less than 4 METS (metabolic equivalents). An exercise capacity of 4 METS would correspond to A. light housework such as dishwashing B. heavy work around the house such as moving heavy objects C. jogging for 2 kilometres D. walking briskly on level ground (6 kilometres per hour) E. walking slowly on level ground (3 kilometres per hour)

ANSWER D 1 MET = 3.5 ml O2 per kg/min = 1 kCal/kg/hr =resting metabolic rate

The American Heart Association guidelines for pre-operative cardiac risk assessment define poor functional capacity as being only able to exercise at a level of less than 4 METS (metabolic equivalents). An exercise capacity of 4 METS would correspond to A. light housework such as dishwashing B. heavy work around the house such as moving heavy objects C. jogging for 2 kilometres D. walking briskly on level ground (6 kilometres per hour) E. walking slowly on level ground (3 kilometres per hour)

ANSWER D 1 MET = 3.5 ml O2 per kg/min = 1 kCal/kg/hr =resting metabolic rate

In patients undergoing vascular surgery, myocardial ischaemia is most likely to occur A. in the pre-operative period B. during induction of anaesthesia C. intra-operatively, prior to revascularisation D. in the first 48 hours post-operatively E. 48 to 72 hours post-operatively

ANSWER D 2/3 of myocardial ishcemia begins in the first 24 hours post operatively.

What is the fluid replacement for maintenance of a 3kg neonate in the first 24 hours of life? A. 50mls B. 100mls C. 150mls D. 200mls E. 250mls

ANSWER D 60 mls/kg/day for first 24 hrs so 180 mls/day for a 3 kg newborn Maintenance Water Requirements Age ml/kg/day 1st day of life 60 2nd day 90 3rd day 120 up to 9 months 120-140 12 months 90-100 2 years 80-90 4 years 70-80 8 years 60-70 12 years 50-60 [edit] Factors modifying water requirements Extra Required * Fever (add 12% for each °c above 37°C) * Hyperventilation * High ambient temperature * Extreme activity * Any other abnormal losses e.g. diarrhoea, polyuria Less required * Hypothermia (subtract 12% for each°c below 37°C) * Very high humidity * Oliguria or anuria * Extreme inactivity * Fluid retention e.g. cardiac failure

In aspirin overdose, which is common: A. Respiratory acidosis B. Increased bleeding due to fibrinolysis C. Hypothermia D. Increased CO2 production E. Renal failure

ANSWER D A - FALSE * "Common: Respiratory alkalosis (subsequently complicated by a metabolic acidosis)." Peck, Hill & Williams, 2nd. ed., p.140. o Respiratory Alkalosis [2ndry to Resp Centre stimulation] o Metabolic Acidosis - increase in pyruvate and lactate that causes a elevated anion gap metabolic acidosis B - FALSE * "Rare: Coagulopathy" Peck, Hill & Williams, 2nd. ed., p.140. * laboratory abnormalities may include a prolonged prothrombin time, thrombocytosis...Current Critical Care (but not common) * ↓ Plt Function C - FALSE * "Common: Pyrexia" Peck, Hill & Williams, 2nd. ed., p.140. D - TRUE * Uncouples oxidative phosphorylation which leads to an increased metabolic rate with a resulting increase in glucose utilization, oxygen consumption, and heat production. Clinical effects of this uncoupling include hypoglycemia and fever...Current Critical Care, 2008 * Also...Inhibition of enzymatic components of the Krebs cycle occurs, leading to an increase in pyruvate and lactate that causes a elevated anion gap metabolic acidosis. As a result of their stimulatory effects on lipid metabolism, salicylates increase ketone formation...Current Critical Care, 2008 o Does uncouple ox-phos but also inhibits krebs cycle. So CO2 production should be reduced. (Yes, but overall increased...Kingfed) * "Aspirin also has effects on the metabolic state, which are usually of little significance but in overdose become significant. It uncouples oxidative phosphorylation, thereby increasing oxygen consumption and carbon dioxide production." Peck, Hill & Williams, 2nd. ed., p.140. E - FALSE

Heparin-induced thrombocytopenia (HITS): A. Rarely see platelets less than 100 B. Thrombocytopaenia will continue when heparin is ceased C. Onset within 2 to 3 days of starting heparin D. Is associated with intravascular thrombosis E. Non-specific antibodies

ANSWER D A - FALSE the clinically important form of the disease frequently does result in much lower counts B - FALSE - will not continue indefinitely, and once the heparin concentration drops the process of platelet activation stops allowing platelet counts to recover. C - FALSE - it implies if the counts are ok on day 3 you're out of the woods which is definitely untrue and once again the clinically relevant thing the examiner wants you to know True - 90% of HIT is type I which has an onset of 2-3 days D - True/False - 10% of HIT is type II which has 30-80% thrombosis. Type I not associated with thrombosis. E - FALSE o HIT Type II + Immunogenic heparin-PF4 complexes which cause an immunologic response + Antibodies are generated resulting in a complex forming between antibodies, heparin, and PF4 (mediated through the FcyIIa portion of the platelet). This complex leads to further platelet activation resulting in formation of microparticles and thrombin generation. + Antibodies also recognize PF4 bound to heparin on the endothelial surface and this surface becomes activated leading to another route of thrombin production.

Multiple sclerosis in pregnancy is A. a contraindication for epidural anaesthesia in labour B. a contraindication for the use of suxamethonium C. associated with an increased caesarean section rate D. associated with an increase in relapse rate postpartum E. associated with a worse fetal outcome

ANSWER D A - False - Neuraxial blockade is associated with exacerbations but epidural analgesia for labour is not contraindicated as long as local anaesthetic concentrations are kept to a minimum B - False - Relative contraindication and is dependent on the magnitude of denervation/disability as to the K efflux from sux. Sux should be avoided. D - True o Pregnancy appears to have a relatively protective effect on women with MS. The number of MS exacerbations is reduced during pregnancy, especially in the second and third trimesters o Exacerbation rates may rise in the first three to six months postpartum, and the risk of a relapse in the postpartum period is estimated to be 20-40%

Regarding chemotherapy agents, A. azathioprrne is a cholinesterase inhibitor and may interact with suxamethonium B. bleomycin may have an idiosyncratic (i.e. not dose related) association with progressive respiratory fibrosis C. cyclophosphamide has NO known interactions with neuromuscular blocking agents D. high cumulative doses of doxorubicin are associated with cardiomyopathy E. nonsteroidal anti-inflammatory drugs (NSAIDs) do NOT interact with methotrexate

ANSWER D A - False: Azathioprine is a purine analog, main interaction is with Allopurinol B - Maybe true: Blue Book 2007 says "potential for idiosyncratic reactions such as pneumonitis from Bleomycin has drawn interest." Later also says risk factors for pneumonitis include... total cumulative dose." Doses <450mg have 3% chance, whereas doses 450-550mg have 15% chance. 2 questions here: 1 - is "pneumonitis" different enough to "progressive pulmonary fibrosis", and 2 - is this a predictable dose-related effect or an unpredictable idiosyncratic reaction? C - False: Cyclophosphamide inhibits pseudocholinesterase activity, so it would effect suxamethonium and mivacurium D - True and possibly best answer: Doxorubicin - "chronic use causes dose-dependent cardiomyopathy" E - False: NSAID's when used with Methotrexate reduce methotrexate clearance and enhance methotrexate-related toxicity

Haemophilia A is commonly associated with: A. a haemarthrosis in a female infant B. a haemarthrosis in a male infant C. low levels of Factor IX D. normal prothrombin time (PT) and prolonged activated partial thromboplastin time (APTT) E. prolonged prothrombin time (PT) and prolonged activated partial thromboplastin time (APTT)

ANSWER D A - X-linked, females are usually carriers, B - True for severe haemophilia A, but not exclusively male C - Congenital factor VIII deficiency D - True (as extrinsic pathway is unaffected, PT is normal) E - Normal PT

SF (Q105 August 2008) A 25yo primipara with an uncomplicated pregnancy presents to delivery suite in labour at term. Her membranes spontaneously rupture soon after, and it is blood-stained. At the same time, a severe foetal bradycardia appears on the CTG. What is the most likely cause of this? A. Placenta accreta B. Placental abruption C. Uterine rupture D. Vasa praevia E. True knot in the umbilical cord

ANSWER D A : Placenta Accreta -abnormally deep attachment of the placenta, through the endometrium and into the myometrium -bleeding occurs when placenta removed post birth **Cause of PPH -1:2,500 pregnancy -very rarely recognized before birth, and is very difficult to diagnose Placenta Increta - invasion past myoemtrium Placenta Percreta - invasion through uterine serosa into neighbouring organs (bladder) RF - uterine surgery, LUSCS, myomectomy B : Placental Abruption -abnormal separation of placenta from uterine wall 20 weeks after -symptoms : pain, pallor, fetal distress, raising fundus (continued bleeding) -in severe cases PV bleeding -RF : maternal hypertension, abdominal trauma, short umbilical cord, prolonged ruptured of membranes, <20 >35, prev abruption C : Uterine rupture -usually during labor -integrity of myometrium breached -similar presentation to placental abruption, but pain and bleeding follow fetal distress -RF : LUSCS, previous uterine surgery, induction, high parity D : Vasa Praevia The classic triad are membrane rupture followed immediately by painless vaginal bleeding and fetal bradycardia E : True Knot Painless Fetal distress

Patient Controlled Analgesia (PCA) is an analgesic technique which is particularly suited to patients with severe and prolonged pain. Desirable features and acceptable practices include: A. A lockout period greater than 10 minutes B. The selection of methadone as an analgesic C. A constant background infusion D. Limitation of the 4 hourly dose of morphine to a maximum of 0.4 mg/kg E. Use of the sub-cutaneous route for long term infusions

ANSWER D A = False. Transient or long lasting damage to peripheral nerves depending on how low the temp is. One month is more accurate. pg 996 Cousins B = False. Unmyelinated axons blocked at a lower temp. pg 995 Cousins C = ? D = ? "initial local anaesthetic effect is mentioned on pg 1024 Cousins - but no mention of waiting 24hrs. E = False. It is hypobaric, phenol is hyperbaric. pg 1024 Cousins * Phenol in glycerin is hyperbaric * Hypertonic saline (10%) causes severe pain on injection so inject local first. Phenol has local anaesthetic effect so have to wait 24-36 hrs before deciding in neurolysis has been successful.

AM15b [Mar91] [Mar93] [Aug93] Myasthenia gravis: A. Is associated with antibodies affecting acetylcholine release B. Best treated with thymectomy and/or steroids in the young C. Becomes self-limiting in old age D. Affects the babies of mothers with the disease E. Is not an antibody mediated disease

ANSWER D A = false - is referring to myasthenic syndrome B = ?false - see page 524 S&D, ?anticholinesterase drugs are prob still FIRST line treatment. C = presumably false - given duration of disease is a predictive for requiring post op ventilatory support. "clinical course is marked by exacerbations and remissions" - pg 524 S&D D = true - see above E = false

A device that detects a 10mA difference in active and neutral leads and causes turning off of the circuit within 40 ms. This is a: A. Class 1 device B. Equipotential earthing C. Line isolation monitor D. Residual Current Device E. Fuse

ANSWER D A Class 1 device = earthed, insulated wires, fuses within equipment B Equipotential earth is a mechanism where the casings of all the equipment in theatre are earthed to the same potential so there can't be a potential difference between two live casings which would be potential source for macro or microshock (green and yellow cabels) C Line isolation monitor detects leakage currents >5mA and sounds an alarm, it does not automatically shut anything off D = detects current leakages and then shuts off supply- not so good if there is life saving equipment being used, e.g. CPB

A device that detects a 10mA difference in active and neutral leads and causes turning off of the circuit within 40 ms. This is a: A. Class 1 device B. Equipotential earthing C. Line isolation monitor D. Residual Current Device E. Fuse

ANSWER D A Class 1 device = equipment is fitted with a three core mains cable containing a protective earth wire. Exposed metal parts on class I equipment are connected to this earth wire. Should a fault develop inside the equipment and the exposed metal comes into contact with the mains, the earthing conductor will conduct the fault current to ground. Regular testing procedures ensure that earthing conductors are intact, as the integrity of the earth wire is of vital importance. B Equipotential earth is a mechanism where the casings of all the equipment in theatre are earthed to the same potential so there can't be a potential difference between two live casings which would be potential source for macro or microshock (green and yellow cabels) C Line isolation monitor detects leakage currents >5mA and sounds an alarm, it does not automatically shut anything off D = detects current leakages and then shuts off supply- not so good if there is life saving equipment being used, e.g. CPB

Which of the following statements is INCORRECT? Recombinant Factor VIla A. directly activates Factors IX and X on the surface of activated platelets, leading to thrombin formation B. has a half-life of three and a half hours C. has been used "off-label" for bleeding in trauma patients D. is best monitored by the prothrombin time, which is shortened in a dose-dependent manner at therapeutic doses E. was developed for patients with haemophilia A and B who have inhibitors to Factors VIII and IX respectively

ANSWER D A-. TRUE : Factor VIIa is also involved in the "thrombin burst". This is a direct activation of factor VIII and IX on the platelet surface, in the absence of TF. This is thought to be the major mechanism of action in thrombus formation. B. TRUE : The half life of Novoseven is 2-6 hrs depending on your source. 2.8-3.1 in Novoseven PI C. TRUE : Recombinant factor VII (RF7, or Novoseven (TM)) has been used for major trauma cases. It was first documented for this use in 1999, when used on a soldier with coagulopathy. Its use in trauma is on a 'compassionate' basis, and so it is used off license. D. FALSE : Whilst it is true that lab coagulation profiles can be used to monitor efficacy of treatment, these parameters have not shown a direct correlation to level of haemostasis that is achieved. TEG as well as clinical cessation of bleeding seems to be the most effective method at present but they both obviously also have limitations. See Novoseven web site for more information. E. TRUE Licensed uses of Novoseven include: * Congenital Haemophilia A or B * Haemophilia with inhibitors * Factor VII deficiency * Glanzmanns thrombocytopaenia Unlicensed uses of Novoseven * Major trauma * Bleeding post cardiac bypass * Obstetric - major haemorrhage.

AZ44b ANZCA version [2002-Mar] Q29, [2002-Aug] Q1 (A-type but only 4 options) (Similar question reported in [Jul00]) According to current legal recommendations in Australia regarding the obtaining of consent for anaesthesia A. it is reasonable to withhold risk information from the very anxious patient B. it is adequate to use the standard of risk disclosure set by a responsible body of medical opinion C. only risks which would be considered significant by a reasonable person need to be discussed D. significant risks with a likelihood of less than 1% may need to be discussed

ANSWER D A. FALSE B. FALSE : Bolam principle superceded by principles of Rogers v Whitaker. C. FALSE : Whitaker case D. TRUE : Whitaker case Even if an extremely unlikely risk is of concern to the patient it does need to be discussed. Roger's vs Whitaker : Duty of Disclosure -Whitaker was almost total blind in right eye since 9yo, but lives a normal life -Roger's, opthalmic surgeon, advised her that an operation on the injured eye would not only improve its appearance but would probably restore sight to it -Whitaker developed 'sympathetic ophthalmia' in her left eye with no improvement in her right and therefore was totally blind -Rogers defence was the "Bolan" principle, that a medical practitioner is not negligent if he acts in accordance with a practice accepted at the time as proper by his peers, even though other medical practitioners adopt a different practice. However, High Court Decision : -all medical treatment is preceded by the patient's choice to undergo it. -The choice is meaningless unless it is made on the basis of relevant information and advise. -description of risks should be tailored in the proposed risk, and the patient -a reasonable person in the patient's position, if warned of the risk, would be likely to attach significance to it

The systolic blood pressure may be overestimated by the auscultatory method of blood pressure measurement if: A. the cuff is deflated too slowly B. the patient has severe arteriosclerosis C. the patient's arm is very thin D. there is severe peripheral vasoconstriction E. too wide a cuff is used

ANSWER D A. FALSE - Inappropriately small cuff leads to BP overestimation B. ? FALSE - With an appropriately sized cuff shouldn't matter... * The maximal pressure under the cuff is proportional to the inflation pressure and cuff width. A cuff that is too narrow generates lower tissue pressures than indicated and thus overestimates blood pressure. Conversely, a very wide cuff slightly underestimates blood pressure. A correctly fitted cuff should cover two-thirds of the upper arm or should be roughly 20% wider than the diameter of the limb. (AICM 2005 6 (12); 405-7) C. FALSE - Intense vasoconstriction leads to BP underestimation D. TRUE - Calcified, noncompressible arteries lead to BP overestimation E. FALSE - Overly rapid cuff deflation leading to BP underestimation OVERESTIMATE -non compliant vessels -small cuff -periperal oedma -cuff to loose -too distal -too frequent

With respect to acute myocardial infarction A. true posterior infarcts manifest on the ECG as a tall R wave in Lead V3 B. aspirin should NOT be given unless myocardial infarction is confirmed C. regional anaesthesia is NOT recommended if thrombolytic therapy has been used within the previous 2 weeks D. Angiotensin Converting Enzyme (ACE) inhibitors may be indicated in the acute period E. Troponin I testing is unsuitable in the perioperative period

ANSWER D A. FALSE : Dominant R in V1 B. FALSE : AHA/ACC guidelines suggest aspirin within 10 minutes of acute chest pain C. ??? ASRA guidelines : Patients who have received fibrinolytic and thrombolytic drugs "should be cautioned against receiving spinal or epidural anesthetics except in highly unusual circumstances. Data are not available to clearly outline the length of time neuraxial puncture should be avoided after discontinuation of these drugs. But regional anaesthesia does not necessarily include neuroaxial blockade D. TRUE : ACC suggest all AMI's if no hypotension. "Start ACE inhibitor orally in patients with anterior infarction, pulmonary congestion, or LVEF less than 0.40 if the following are absent: hypotension (SBP less than 100 mm Hg or less than 30 mm Hg below baseline) or known contraindications to this class of medications." E. FALSE

Epidurals in the elderly: A. Increased mass of drug required B. Increased volume required C. Increased incidence of dural puncture D. Increased incidence of hypotension

ANSWER D A. FALSE : Factors affecting spread: *Major: dose (concentration x volume), age, thoracic > lumbar. *Minor: morbid obesity, pregnancy, position. *No effect: gender, height, weight (except as above), direction of needle, speed of injection, atherosclerosis, mode of injection (bolus vs fractionated) B. TRUE C. FALSE : Risk Factors for Post-Dural Puncture Headache * Beveled (Quincke) needle (pencil-point needles are preferable) * Larger needle * Female gender * Pregnancy * Younger age * History of headache prior to the dural puncture D. TRUE

During elective caesarean section under spinal anaesthesia A. maternal hypotension requiring intervention is infrequent B. the duration that the maternal systolic blood pressure is below 100 mmHg is of less importance for producing adverse cord blood acid-base measures than the degree of fall in systolic blood pressure C. there is a significant difference between the use of rescue boluses compared to infused prophylactic ephedrine on the status of cord blood acid-base measures D. there is less nausea and vomiting if ephedrine is prophylactically infused compared to using it as a rescue bolus to treat any maternal hypotension when it does occur E. there is no adverse effect from maternal hypotension on cord blood acid-base values

ANSWER D A. FALSE : Hypotension is defined as SBP<100 or decrease in SAP 10-30%. Incidence of hypotension during spinal LSCS is 80-90%. It is associated with hypotension which influences uterine blood flow. Roberts et al and Mueller et al have documented higher rate of retal acidemia after regional anaesthesia for elective LUSCS. B. FALSE : marked or prolong hypotension are likely to be assocated with fetal asphyxia and/or acidosis. However, the degree and duration of hypotension that is likely to be harmful to the fetus in humans is undetermined C. FALSE : fetal cord blood pH shown to be lower in a dose dependant manner. However, rescue compared to infused showed no difference in cord pH D. TRUE D. TRUE E. FALSE

In hemiplegic patients A. neuromuscular blockade should be monitored on the affected side B. suxamethonium may cause a hyperkalaemic response 3 months to one year after the stroke C. there is resistance to non-depolarizing muscle relaxants in the unaffected muscle groups D. the response to non-depolarizing muscle relaxants is similar to that seen in paraplegic patients E. ventricular fibrillation after suxamethonium has been documented

ANSWER D A. FALSE : NMB monitoring on the non affected side. The effected side may have contractures and as such will probably be abnormal. B. FALSE : 1wk to 6 months C. FALSE : resistence on the affected side, furthermore when compared to controls the unaffected arm shows resistence. D. TRUE : both are UMN diseases with reduction in peripheral ACh release leading to immature nAChR upregulation. E. TRUE : Ventricular fibrillation in a normokalemic hemiplegic patient during induction of anesthesia.

The most useful finding to confirm the diagnosis of aortic stenosis in an adult with a systolic murmur is: A. increasing murmur intensity with inspiration B. decreasing murmur intensity with passive leg elevation C. increased second heart sound D. effort syncope E. a slow rate of rise of the carotid pulse

ANSWER D A. FALSE : RILE, louder with expiration B. FALSE : with most murmurs passive leg raising causes them to get louder (except HOCM and MVP) C. FALSE : A soft or absent A2 or reversed splitting of S2 may denote severe AS D. TRUE : this is a cardinal symptoms of AS E. TRUE : A slow-rising, diminished arterial pulse suggests severe AS in a patient with a grade 2/6 midsystolic murmur at the upper intercostal spaces, however has a poor sensitivity.

When anaesthetising a patient with a ventricular assist device (VAD) for noncardiac surgery A. anticoagulation should be temporarily discontinued during surgery B. electrocautery is well tolerated by these devices C. malignant arrhythmias should be treated with defibrillation if indicated D. the most important causes of decreased pump output are hypovolaemia and increased afterload E. volatile anaesthetic agents should be avoided.

ANSWER D A. FALSE : With the exception of the Heartmate, maintenance of therapeutic levels of anticoagulation is imperative for extracorporeal circulation through these devices (because of the risk of thromboembolism), and both the surgeon and anesthesiologist must be prepared for increased intraoperative bleeding...At no time should anticoagulation ever be fully reversed in a VAD-supported patient B. FALSE : Extracorporeal devices (the Abiomed and the Thoratec) and the well shielded Novacor will not be affected by defibrillation or the electrocautery. Unfortunately, the Heartmate is not well shielded and may be reset to a fixed-rate mode by the electrocautery and potentially damaged by external defibrillation. When feasible, the use of bipolar electrocautery is recommended. C. FALSE : patients on biventricular support will often remain hemodynamically stable in the face of otherwise malignant arrhythmias. Regardless, malignant ventricular arrhythmias should be terminated whenever possible. D. TRUE : The two most important factors leading to decreased pump output are hypovolemia and increased afterload E.

In preeclampsia A. once delivery of the placenta takes place, the condition improves B. in the absence of other risk factors a platelet count of greater than 50 is adequate for epidural anaesthesia C. corticosteroid therapy has no effect on the severity of thrombocytopenia D. magnesium sulphate halves the incidence of eclampsia E. spinal anaesthesia contraindicated

ANSWER D A. FALSE : after delivery of the placenta, pre-eclampsia usually improves, can take 3 months to improve. B. FALSE : Obstetric Anaesthesia Scientific Evidence from ANZCA gives 75 as a safe figure, but must be balanced against operator experience and other clinical risk factors for coagulopathy C. FALSE : Consistent with observational studies, dexamethasone was shown to significantly increase the platelet count. This however did not translate to improvement in outcomes and the clinical relevance of this is unclear. D. TRUE : Magnesium sulphate halves the risk of eclampsia, and probably reduces the risk of maternal death. There do not appear to be substantive harmful effects to mother or baby in the short term." From MAGPIE. E. FALSE : Regional blockade is the preferred method of anaesthesia for caesarean section (Level II). There is currently insufficient evidence to support any specific type.

Regarding ball flowmeters the A. flow control knob cannot stop gas leakage if the glass chamber is broken B. flowmeter maintains accuracy when tilted C. flowmeter will over estimate gas flow if connected to a high resistance device such as a nebuliser D. gas flow rate is read at the centre of the ball E. gas flow lifts the ball up in a parallel sided tube in the glass chamber

ANSWER D A. FALSE : depends if flowmeter is pressurized with needle valve downstream B. FALSE : important to keep vertical, 10 deg over reads 5% 25 deg over reads 10% 30 deg over reads 20% C. FALSE - The chamber is pressurized by the inlet pressure and minor degrees of outlet occlusion do not alter the flow. D. TRUE - middle of the ball E. FALSE - The ball is lifted up a conical tube, meaning there is variable flow but constant pressure across the ball

Regarding the use of epidurals for post-operative analgesia, A. epidural analgesia following bowel surgery may increase the risk of anastomotic leakage B. epidurals have similar analgesic efficacy to intravenous PCA (patient controlled analgesia) with morphine C. studies show the overall failure rate of epidurals to be less than 2% D. the incidence of pulmonary infections with epidural local anaesthetics is LESS than that with parenteral opioids E. the incidence of significant epidural haematoma is NOT affected by concomitant anti-thrombotic therapy

ANSWER D A. FALSE : there have been numerous case reports, however, there is no statistically significant evidence from the small RCT's available in bowel surgery. Thoracic epidural analgesia is protective for anastomotic leakage after oesophagectomy. B. FALSE : epidurals provide superior analgesia as compared to PCA C. FALSE : large prospective study showed 22% premature termination of epidural treatment D. TRUE : Epidural local anaesthetics improve oxygenation and reduce pulmonary infections and other pulmonary complications compared with parenteral opioids E. FALSE : Anticoagulation is the most important risk factor for the development of epidural haematoma after neuraxial blockade

Circuit disconnection during spontaneous breathing anaesthesia A. will be reliably detected by a fall in end-tidal carbon dioxide concentration B. will be detected early by the low inspired oxygen alarm C. will be most reliably detected by spirometry with minute volume alarms D. may be detected by an unexpected drop in end-tidal volatile anaesthetic agent concentration E. can be prevented by using new, single-use tubing

ANSWER D A. FALSE: Gas analyzer is still connected to patient despite disconnection of circuit, therefore ETCO2 will be normal B. FALSE: Disconnection allows entrainment of room air which will not be detected by low inspired O2 alarm C. FALSE : Depends where the spirometry is taken from, but some spirometry taken near the filter and so a disconnect at the machine will not be detected D. TRUE : Any disconnect will allow entrainment of room air into the circuit and allow escape of the volatiles, thus a drop in the ET-agent. E. FALSE: Single use tubing will not prevent a disconnection. Apnea/disconnect alarms may be based on 1. Chemical monitoring (lack of end tidal carbon dioxide) 2. Mechanical monitoring * Failure to reach normal inspiratory peak pressure, or * Failure to sense return of tidal volume on a spirometer 3. Visual monitoring * Failure of standing bellows to fill during mechanical ventilator exhalation * Failure of manual breathing bag to fill during mechanical ventilation (machines with fresh gas decoupling- the Apollo, Fabius GS, Narkomed 6000) 4. Auditory monitoring - lack of breath sounds in precordial, lack of sound from ventilator cycling, etc. 5. Optic monitoring - Failure of the hanging bellows to fill completely (the "garage door" electronic eye sensor on the Julian)

PL28 ANZCA version [2004-Apr] Q137, [2004-Aug] Q59, [Jul 06] Q41 Intra-nasal topical cocaine used in nasal surgery A. has a duration of action of the order of 6 hours B. is metabolised more quickly by the liver if the patient is using ecothiopate eye drops C. is typically used in a dose of approximately 5 ml of 5% solution in an adult D. may be metabolised more slowly in patients with liver disease E. reaches a peak plasma concentration in 3 hours

ANSWER D A. FALSE: Intranasal adminstration: onset in 3-5min, peak at 10-20min, rarely last >1 hour B. FALSE: ecothiopate is a plasma cholinesterase inhibitor, cocaine is metabolised extensively by the liver, only 1% excreted unchanged in the urine C. FALSE: max dose is 3mlkg (150mg is typical adult dose) D. TRUE: Cocaine is metabolised by plasma and liver cholinesterases to water-soluble metabolites that are excreted in urine. Plasma choliesterase activity is decreased in parturients, neonates, the elderly, and patients with severe underlying hepatic disease. E. FALSE: Peak venous plasma concentration in 30-40 minutes after IN adminstration

In the management of drug-induced anaphylaxis A. the first priority is the correction of diminished intravascular volume using intravenous colloid B. adrenaline is contra-indicated in the presence of a ventricular arrhythmia or the concurrent administration of halothane C. metaraminol is the treatment of choice for hypotension D. calcium is contra-indicated because of potential enhancement of mediator release E. lignocaine is the treatment of choice for arrhythmias

ANSWER D A. FALSE: No. Withdraw causative agent. 100% O2. "Fluid therapy should be an early resuscitative measure, but pharmacological resuscitation is the priority" - and in any case adrenaline would be the initial treatment for the hypotension. B. FALSE: Articles mentions the higher chance of reaction, but it is EXPLICITLY STATED that this should not preclude its use. C. FALSE Adrenaline is the drug of choice in the management of hypotension, bronchospasm and angio-oedema D. TRUE "Calcium salts, as inotropes, should be avoided because of the potential enhancement of mediator release" E. FALSE "Calcium antagonists are probably the drugs of choice, as they are known to antagonize some of the arrhythmogenic effects of histamine in vitro"

After transfusion of 5 unit of FFP what is least likely to occur A. Haemolytic reaction B. Hypocalcaemia C. Infection D. Hyperkalaemia

ANSWER D A. FALSE: can occur if not matched B. FALSE: likely to occur if tranfused >1ml/kg/min C. FALSE: viral transmission possible D. TRUE: FFP does not contain potassium

Patho-physiological features of patients with morbid obesity include: A. A blood volume:body weight ratio which is similar to that of patients with normal body weight B. An increased blood pressure and systemic vascular resistance compared to that of patients with normal body weight C. Decreased gastric motility due to increased gastrin secretion D. Cardiac pathology resulting from excess body mass and increased metabolic demand E. Cardiac pathology resulting mainly from fatty infiltration or fatty change of the heart

ANSWER D A. FALSE: less than normal may be as low as 47ml/kg B. FALSE: BP increases but SVR declines because of increased CO C. FALSE: gastrin, HCl, pepsin, trophic action on stomach and intestine. However the decreased motility seen with gastrin is related to intragastric acid that can be inhibited by proton pump inhibitors. Lastly motility may be increased in obesity making the association incorrect. D. TRUE : Obesity ⇒ Increased blood volume ⇒ Increased stroke volume ⇒ Increased cardiac output ⇒ LV enlargement ⇒ LV wall stress ⇒ LV systolic and diastolic dysfunction ⇒ LV failure E. FALSE: Stoelting 447: "Increases in epicardial fat are common in obese individuals, but fatty infiltration of the myocardium is uncommon and not responsible for congestive heart failure."

You are asked to see a 60 y.o. male 2 days following a cervical laminectomy because he has new new neurological symptoms in his right arm. The surgical team think these may be due to poor patient positioning. The sign that would most help differentiate c C8-T1 nerve root injury from an ulnar nerve injury is A. loss of sensation in the index finger B. loss of sensation in the little finger C. weakness of the abductor digiti minimi muscle D. weakness of the abductor pollicis brevis m E. weakness of the first dorsal interosseous m.

ANSWER D A. FALSE: neither ulnar nerve or C8T1 supply, therefore non differentiating B. FALSE. both ulnar and C8T1 supply therefore non differentiating C. FALSE: both affected by C8T1 and ulnar therefore non differentiating D. TRUE: T1 and median nerve supply, but ulnar does not E. FALSE. both supplied by c8T1 and ulnar nerve therefore non differentiating

Regarding endotracheal tubes for paediatric patients A. a 2.5 mm endotracheal tube is the appropriate size for a term neonate B. armoured (wire spiral) endotracheal tubes have the same outside diameter as non-armoured endotracheal tubes (of the same internal diameter) C. the outside diameter (in mm) of an appropriately sized tube is given by the formula (Age/4) +4 D. the same diameter tube is used for nasal and oral intubation in a child E. uncuffed, paediatric endotracheal tubes do NOT have a Murphy's eye

ANSWER D A. False - term neonate would be 3.0-3.5 - preterms would be 2.5 tubes B. False - armoured tubes are thinner, so for the same internal diameter, armoured tubes are smaller C. False - this formula refers to the INTERNAL diameter. D. TRUE - albeit at different lengths of insertion E. False - they do have a Murphy eye!

When providing general anaesthesia to a patient with a history of asthma A. thiopentone should not be used as it may cause bronchospasm B. intravenous and topical tracheal lignocaine are equally effective in preventing bronchial hyperreactivity C. ketamine provides little benefit in a patient with active wheezing D. induction with propofol is effective in reducing the incidence of wheezing following intubation E. isoflurane is as effective a bronchodilator as halothane when given in MAC equivalent doses

ANSWER D A. False. When compared with propofol it seems to; this however may just be because propofol is a bronchodilator. Thiopental itself does not cause bronchospasm. B. Probably false. Depends on your source. Both sides are presented in Burburan et al. C. Definitely false. It is a widely recommended and used treatment for severe asthma not responding to normal Rx. D. Probably true. It depends on what you compare it to! Compared with thio and etomidate, yes. Compared with ketamine and volatiles, no. Go figure. E. Maybe true.

When a reaction to an anaesthetic drug is suspected A. The single most useful test to confirm a reaction is mast cell tryptase sampled 20-24 hour the event B. The single most useful test to confirm a reaction is mast cell tryptase sampled 8-12 hours C. The most valuable test to identify the drug responsible is radioimmunoassay (RAST) D. The morphine IgE radioimmunoassay is a sensitive and efficient test for the detection of IgE antibodies to neuromuscular blocking drugs E. The abscence of an elevated mast cell tryptase (sampled at an appropriate time) excludes a Type I hypersensitivity reaction

ANSWER D A. False: When there has been extreme mast cell degranulation, as occurs in systemic anaphylaxis, tryptase levels rise within 1 hour and remain elevated for 4-6 hours. [1] (http://www.haps.nsw.gov.au/edrsrch/edinfo/tryptase.html) B. False: Serum (clotted sample, eg crossmatch tube) needs to be taken 1-4 hours after onset of reaction. [2] (http://www.usyd.edu.au/su/anaes/anaphylaxis.html) C. False: Negative mast cell tryptase tests are only rarely associated with positive skin tests and antibody tests. While, if taken at the correct time, they usually mean anaphylaxis has not occurred, a small number of patients who are probably having reactions related to basophils rather than mast cells, will have MCT levels that are not elevated in association with anaphylaxis. The three we have documented all had a convincing clinical history. Unfortunately these few patients mean that MCT levels cannot be used to show that skin testing is unnecessary. D. Best Answer: (If implying morphine itself, and not some other IgE generated by morphine) Morphine, which has a single substituted ammonium group, avidly binds in vitro to antibodies that react with NMBDs - (see reference below) E. - From RCPA Manual (http://www.rcpamanual.edu.au/sections/pathologytest.asp?s=33&i=350) on collecting specimens for tryptase analysis: "Post-mortem blood is also suitable, if collected within 4 days of death, provided that death did not follow the event by less than 30 minutes." "A negative result does not exclude an allergic basis especially with anaphylaxis following food ingestion."

Correct statements regarding atrial fibrillation include all of the following EXCEPT A. A history of hypertension, associated with atrial fibrillation, increases stroke risk. B. A history of poor left ventricular systolic function, associated with atrial fibrillation, increases stroke risk. C. Anticoagulation should be continued for one month following successful cardioversion of atrial fibrillation. D. Anticoagulation therapy (to an INR of 2.0) decreases the risk of stroke by up to 50%. E. Atrial fibrillation becomes an increasingly important cause of stroke with advancing age.

ANSWER D A. TRUE : CHADSII B. TRUE : CHADSII C. TRUE : The first is to anticoagulate for at least 3 weeks before and 4-6 weeks after elective cardioversion. The second strategy obviates the need for prior anticoagulation, and involves a pre-procedure transoesophageal echo (TOE) to exclude the presence of LA/LAA thrombus. After cardioversion, patients must be anticoagulated with heparin or enoxaparin followed by warfarin or enoxaparin as above (IIa/B). D. FALSE : therapeutic range is 2.0-3.0 E. TRUE : increasing ages increases the annual risk of stroke from 1.5% at 50yo to 23% at 80yo.

AM09d ANZCA version [2005-Apr] Q92 Which of the following statements is NOT true of the neuroleptic malignant syndrome? A. it is a rare complication of dopaminergic antagonist drugs B. it may be successfully treated with bromocriptine C. it may be successfully treated with dantrolene D. unlike malignant hyperpyrexia it does not have a genetic component E. the syndrome includes extra-pyramidal signs, hyperthermia peripheral vasoconstriction, tachycardia, dyspnoea and other autonomic manifestations

ANSWER D A. it is a rare complication of dopaminergic antagonist drugs - true: dopamine receptor blockade is central to most theories of its pathogenesis B. it may be successfully treated with bromocriptine - true C. it may be successfully treated with dantrolene - true D. unlike malignant hyperpyrexia it does not have a genetic component - false: o "Familial clusters of NMS suggest a genetic predisposition to the disorder. Genetic studies have shown that the presence of a specific allele of the dopamine D2 receptor gene is over represented in NMS patients. This allele is associated with reduced density and function of dopamine receptors as well as decreased dopaminergic activity and metabolism." (Uptodate) E. the syndrome includes extra-pyramidal signs, hyperthermia peripheral vasoconstriction, tachycardia, dyspnoea and other autonomic manifestations - true

Bowel preparation for colonoscopy A. may cause hypophosphaturia, hypercalcaemia and dehydration B. can be avoided in high risk patients C. no longer causes significant fluid loss with modern formulations D. may cause Trousseau's sign to be positive E. may result in arrhythmias and a shortened QT interval

ANSWER D A. may cause hypophosphaturia, hypercalcaemia and dehydration * False - hyperphosphataemia and uria due to high absorption and secondary hypocalcaemia. Dehydration yes, of course. B. can be avoided in high risk patients * False - unfortunately not because then you can't see. No prep = no colonoscopy (according to my wife, a gen surg reg). C. no longer causes significant fluid loss with modern formulations * False - much less with polyethylene glycol granted but 'Fleet' is still being used a lot. D. may cause Trousseau's sign to be positive * True - classic sign of hypocalcaemia, the physiologic derangement which tends to predominate (see Tan's article below) E. may result in arrhythmias and a shortened QT interval * False - very unlikely; the hypocalcaemia causes a long QT.

You are asked to see a 60 y.o. male 2 days following a cervical laminectomy because he has new new neurological symptoms in his right arm. The surgical team think these may be due to poor patient positioning. The sign that would most help differentiate c C8-T1 nerve root injury from an ulnar nerve injury is A. loss of sensation in the index finger B. loss of sensation in the little finger C. weakness of the abductor digiti minimi muscle D. weakness of the abductor pollicis brevis m E. weakness of the first dorsal interosseous m.

ANSWER D A. would both be affected by both C8/T1 lesion (C8 dermatome) AND ulnar nerve lesion - so not differentiating B. would be affected but neither (index finger = C7 and median nerve) - so not differentiating C. would both be affected by both C8/T1 lesion AND ulnar nerve lesion - so not differentiating D. abductor pollicus brevis = T1 myotome and median nerve T1 and ulnar nerve have very similar sensory and motor distribution. HOWEVER, the one difference is that the LOAF muscles are supplied via the median nerve and provide a point of differentiation. E. would both be affected by both C8/T1 lesion AND ulnar nerve lesion - so not differentiating

All are causes of goiter, except: A. Decreased iodine intake B. Thioureas C. Hashimoto's disease D. Hypopituitarism

ANSWER D A: FALSE B: FALSE - Thioureas have goitrogenic effect, so B is true (See thioureas (http://en.wikipedia.org/wiki/Thioureas)) C: FALSE D: TRUE

Each of the following statements regarding cardiac tamponade as a complication of central venous lines is true, EXCEPT: A. catheters with multiple lumens carry greater risk B. for a left sided catheter, placement of the tip at the mid-point of the brachiocephalic vein is safe C. it usually occurs within the first week following insertion D. placement of the catheter tip above the junction of the right atrium and superior vena cava will avoid this complication E. visceral chest pain with drug infusion is an early symptom

ANSWER D A: TRUE..."Stiffer catheters are more likely to perforate. Stiffness is a function of the composition of the catheter (Silastic probably being safest) and the number of the lumen (a function of the greater diameter and presence of 'septa' within the catheter)." B: TRUE..."Zone C (mid‐point, left innominate vein). This is a suitable site for the tip when the catheter is introduced from the left internal jugular or subclavian vein, and reduces the risk of SVC perforation." (In reference to a diagram of the heart and great vessels showing various "zones" for the CVC tip) C: TRUE..."Most cases occur in the first week after insertion" D: FALSE (and answer to choose)..."the pericardium may ascend alongside the medial wall of the SVC by up to 5 cm (mean 3 cm). Thus, placement just proximal to the atrium does not obviate the risk of tamponade". E: TRUE..."This results in two warning signs: visceral‐type chest pain on infusion of drugs or parenteral nutrition solutions and a curved appearance of the distal catheter seen on chest x‐ray"

Regarding atrial fibrillation A. aspirin is as effective as warfarin for the prevention of embolic stroke in elderly patients B. patients should not receive warfarin therapy unless they have risk factors for embolic stroke C. patients who are converted to sinus rhythm should be placed on rhythm maintenance therapy D. rhythm control has NOT been shown to be superior to rate control E. the best drug for rate control during exercise is digoxin

ANSWER D AFFIRM, PIAF, RACE, STAF : -this RCT have shown rate control is at least effective as rhythm control in improving symptoms and fucntional capacity

Highest likelihood of motor block with labour epidural analgesia: A)Nurse initiated epidural topups B)Anaesthetist initiated epidural topups C)PCEA D)Continuous epidural infusion E)All associated with same motor block

ANSWER D ANZCA pain book page 189: comparison with continuous epidural (vs PCEA) first study quoted found higher incidence motor block with continuous (but also better pain scores)

The APGAR score of a neonate who is blue in colour with weak respirations, weak cry, pulse of 84, flo[[PPy with slight flexion to stimulation is: A. 1 B. 2 C. 3 D. 4 E. 5

ANSWER D APPEARANCE 0 - blue or pale 1 - pink body, blue extremities 2 - completely pink PULSE 0 - abscent 1 - slow <100 2 - normal >100 GRIMACE 0 - no response 1 - Grimace 2 - Cough or sneeze ACTIVITY 0 - Limp 1 - some flexion 2 - active movement RESPIRATIONS 0 - absent 1 - slow irregular 2 - crying A=0, P=1, G=1, A=1, R=1

Recognised features of a true anaphylactic reaction do NOT include A. Previous exposure to the drug. B. Mediation by IgE antibody. C. Histamine relase from mast cells. D. Activation of the complement system. E. Intracellular influx of calcium ions.

ANSWER D Activation of complement is not part of Type 1 hypersensitivity reaction. In anaphylactoid reactions : classical and alternate pathways may be activated particularly after protamine, contrast media, and althesin.

In a haemodynamically stable 20-year-old male presenting with blunt chest trauma, the best screening test for diagnosis of cardiac injury requiring treatment is A. chest X-Ray B. serum CK-MB levels C. serum Troponin levels D. standard 12 lead ECG E. transthoracic echocardiography

ANSWER D An admission 12 lead ECG should be performed on all patients in whom there is suspected BCI. If the admission ECG is normal, the risk of having a BCI that requires treatment is insignificant, and no further testing is indicated. Echocardiography is the diagnostic method of choice in patients with ECG abnormalities or unexplained cardiovascular instability following blunt cardiac trauma. If the patient is hemodynamically unstable, a TTE should be obtained. If an optimal TTE cannot be performed, then the patient should have a TOE. Neither creatine phosphokinase with isoenzyme analysis nor measurement of circulating cardiac troponin T are useful in predicting which patients have or will have complications related to BCI.

The Pringle manoeuvre (a surgical intervention when faced with exanguinating haemorrhage from the liver) involves clamping the A. aorta above the coeliac axis B. hepatic artery only C. hepatic vein only D. portal pedicle E. splenic artery only

ANSWER D An important advance is the use of the Pringle maneuver (clamping of the hepatoduodenal ligament only) for vascular inflow occlusion as an alternative to total vascular occlusion (clamping of the portal vein, hepatic artery and supradiaphragmatic as well as infradiaphragmatic portion of the inferior vena cava) The Pringle manoeuvre was performed by placing a clamp on the hepatic artery and portal vein.

A lady with a Fontan's circulation for tricuspid atresia presents for caesarian section. What is the best way of maintaining her cardiac output? A. Trendelenburg B. Epidural contraindicated C. Allow pCO2 to rise to 50 to vasodilate her D. Short inspiratory time E. Allow hypovolaemia

ANSWER D CEACCP: In the single-ventricle circulation, increases in PVR and intrathoracic pressure can compromise PBF. This has implications for ventilatory strategy. Spontaneous breathing causes negative intrathoracic pressures and augments PBF; however, positive pressure ventilation may allow greater control of oxygenation and minute ventilation, thus avoiding hypoxia and hypercapnia. Positive end-expiratory pressure should be optimized; peak inspiratory pressures and inspiratory times minimized to facilitate PBF.

Chronic alcohol use. Which is not an associated complication ? A. Pancreatitis B. Atrial fibrillation C. Macrocytosis D. Nephrotic syndrome E. Hypertriglyceridaemia

ANSWER D CNS Wernicke-Korsakoff syndrome Peripheral neuropathy Autonomic dysfunction Metabolic Obesity Hyperlipidaemia Hypokalaemia Hypomagnesaemia Hyperuricaemia Hypoglycaemia CVS Cardiomyopathy Heart failure Hypertension Arrhythmias (e.g. AF, SVT, VT) Haematological Macrocytosis Thrombocytopenia Leucopoenia GI Alcoholic liver disease Pancreatitis Gastritis Oesophageal and bowel carcinoma Musculoskeletal Myopathy Osteoporosis Osteomalacia

AZ38 [Mar93] [Aug93] [Apr98] [Jul98] CPAP during spon resps: A. Increased insp. capacity B. IIncreased vital capacity C. Increased closing volume D. Increased (FRC-CV) difference

ANSWER D CPAP * ↑ FRC * ↓ extravascular lung water * ↓ CV ∴ if ↑ FRC and ↓ CV → ↑ FRC-CC

Which drug has the best evidence for reducing blood loss in trauma? A. Aminocaproic acid B. Novo 7 C. Prothrombinex D. Tranexamic acid E. Aprotinin

ANSWER D CRASH-2 trial (Lancet 2010]: Tranexamic acid safely reduced the risk of death in bleeding trauma patients in this study. On the basis of these results, tranexamic acid should be considered for use in bleeding trauma patients. Early (within 8 hours of injury) tranexamic acid trauma pt reduces mortality and transfusion rates)

Endocarditis prophylaxis A Bicuspid valve B Congenital repair > 12 months ago C Rheumatic heart valve D Uncorrected cyanotic heart disease E MVP + ?MR

ANSWER D Cardiac conditions that need prophylaxis -Prosthetic valve or prosthetic material -Previous IE -Congenital HD with - unrepaired cyanotic defects -completely repaired defects with prosthetic material <6months ago -repaired defects with residual defects at or adjacent to site of - -prosthetic patch or device (inhibit endothelialisation) -cardiac TP with subsequent development of valvulopathy -RHD in indigenous Australians only.

Patient burns during MRI can be associated with each of the following EXCEPT A high intensity changing magnetic fields B looped monitoring lines ... C non ferromagnetic material in contact with the patient D cosmetics worn by the patient (which do not contain metals) E temperature monitoring with thermister probes

ANSWER D Causes of contact burn with MRI due to induction of current by high intensity magnetic field. -contact with monitoring cable, equipment -ferromagnetic items : bracelets -ferromagnetic containing : tattoos, fentanyl/nicotine patchs (aluminium) -looping Recommendations 1. Monitors and sensors for MRI use nonconducting paths (e.g., fiberoptic cable and plastic tubing) or high-resistance paths (e.g., carbon ECG leads) are preferred, provided that they meet monitoring needs. 2. Do not loop leads or cables. 3. Place the sensor well away from the RF coil, and run the cables away from the coil whenever possible. 4. Check all sensors and cables to ensure that the electrical insulation around them is intact. Be sure that no other bare metal surface is in contact with the patient. 5. Keep the cables off the patient and run them over blankets whenever possible. 6. Remove all unused sensors, cables, and surface coils from the MRI system. 7. Instruct conscious patients to call out if they experience uncomfortable levels of warming anywhere, especially at sites of sensor application. Heed such calls. Install intercom systems and check them regularly. Operators should periodically check the sites of sensor locations on unconscious patients.

Conditions which cause a metabolic acidosis with normal anion gap include A. chronic renal failure B. diabetic ketoacidosis C. salicylate poisoning D. renal tubular acidosis E. ethylene glycol poisoning

ANSWER D Causes of metabolic acidosis with increased anion gap * Lactic acidosis * Circulatory arrest * Pulmonary oedema * Gram negative sepsis * ketoacidosis * toxins * nitroprusside * fructose * acid-citrate-dextrose blood Causes of metabolic acidosis with a normal anion gap: * renal tubular acidosis * diarrhoea * biliary / pancreatic fistulae * excessive infusion of normal saline A nice easy way to remember all of the common causes of metabolic acidosis - PKUSMAL (patients with DKA have Kussmall respiration) * Paraldehyde * Ketones * Uraemia * Salicylates * Methanol * Alcohol * Lactate

Circuit disconnection during spontaneous breathing anaesthesia A. will be reliably detected by a fall in end-tidal carbon dioxide concentration B. will be detected early by the low inspired oxygen alarm C. will be most reliably detected by spirometry with minute volume alarms D. may be detected by an unexpected drop in end-tidal volatile anaesthetic agent concentration E. can be prevented by using new, single-use tubing

ANSWER D Circuit disconnection during spontaneous breathing anaesthesia * A. will be reliably detected by a fall in end-tidal carbon dioxide concentration - false: If circuit disconnect at the machine, ETCO2 will be normal. * B. will be detected early by the low inspired oxygen alarm - false: Disconnection allows entrainment of room air which will not be detected by low inspired O2 alarm * C. will be most reliably detected by spirometry with minute volume alarms - false: Depends where the spirometry is taken from, but some spirometry taken near the filter and so a disconnect at the machine will not be detected * D. may be detected by an unexpected drop in end-tidal volatile anaesthetic agent concentration - true: Any disconnect will allow entrainment of room air into the circuit and allow escape of the volatiles, thus a drop in the ET-agent. * E. can be prevented by using new, single-use tubing - false: Single use tubing will not prevent a disconnection.

Adults with chronic renal insufficiency can maintain a normal plasma potassium concentration via tubular secretion until glomerular filtration rate falls below A. 100ml/min B. 80ml/min C. 50ml/min D. 20ml/min E. 8ml/min

ANSWER D Classification * Chronic renal failure (CRF) is defined as a permanent reduction in glomerular filtration rate (GFR) sufficient to produce detectable alterations in well-being and organ function. This usually occurs at GFR below 25 ml/min. * Four stages of decreased renal function may be visualized: 1. Silent - GFR up to 50 ml/min. (=3000ml/hr) 2. Renal insufficiency - GFR 25 to 50 ml/min. (=1500-3000ml/hr) 3. Renal failure - GFR 5 to 25 ml/min (=300-1500ml/hr) 4. End-stage renal failure - GFR less than 5 ml/min. (<300ml/hr)

The half life of the active metabolite of levosimendan (OR-1896) is: A: 1hr B: 8hr C: 24hr D: 3 days E: 7 days

ANSWER D Clin Pharmacokinet. 2007;46(7):535-52. Levosimendan has been developed for the treatment of decompensated heart failure and is used intravenously when patients with heart failure require immediate initiation of drug therapy. It increases cardiac contractility and induces vasodilatation. The pharmacokinetics of levosimendan are linear at the therapeutic dose range of 0.05-0.2 microg/kg/minute. The short half-life (about 1 hour) of the parent drug, levosimendan, enables fast onset of drug action, although the effects are long-lasting due to the active metabolite OR-1896, which has an elimination half-life of 70-80 hours in patients with heart failure (New York Heart Association functional class III-IV). Although levosimendan is administered intravenously, it is excreted into the small intestine and reduced by intestinal bacteria to an amino phenolpyridazinone metabolite (OR-1855). This metabolite is further metabolised by acetylation to N-acetylated conjugate (OR-1896). The circulating metabolites OR-1855 and OR-1896 are formed slowly, and their maximum concentrations are seen on average 2 days after stopping a 24-hour infusion. The haemodynamic effects after levosimendan seem to be similar between fast and slow acetylators despite the fact that the enzyme N-acetyltransferase-2, which is responsible for the metabolism of OR-1855 to OR-1896, is polymorphically distributed in the population. Levosimendan reduces peripheral vascular resistance and has direct contractility-enhancing effects on the failing left ventricle. It also improves indices of diastolic function and seems to improve the function of stunned myocardium. Despite an improvement in ventricular function, levosimendan does not increase myocardial oxygen uptake significantly. An increase in coronary blood flow and a reduction in coronary vascular resistance have been observed. Levosimendan reduces plasma brain natriuretic peptide (BNP) and N-terminal pro-BNP (NT-proBNP) levels substantially, and a decrease in plasma endothelin-1 has been observed. Levosimendan also exerts beneficial effects on proinflammatory cytokines and apoptosis mediators. The effects of a 24-hour levosimendan infusion on filling pressure, ventricular function and BNP, as well as NT-proBNP, last for at least 7 days.

The half life of the active metabolite of levosimendan (OR-1896) is: A: 1hr B: 8hr C: 24hr D: 3 days E: 7 days

ANSWER D Clin Pharmacokinet. 2007;46(7):535-52. Levosimendan has been developed for the treatment of decompensated heart failure and is used intravenously when patients with heart failure require immediate initiation of drug therapy. It increases cardiac contractility and induces vasodilatation. The pharmacokinetics of levosimendan are linear at the therapeutic dose range of 0.05-0.2 microg/kg/minute. The short half-life (about 1 hour) of the parent drug, levosimendan, enables fast onset of drug action, although the effects are long-lasting due to the active metabolite OR-1896, which has an elimination half-life of 70-80 hours in patients with heart failure (New York Heart Association functional class III-IV). Although levosimendan is administered intravenously, it is excreted into the small intestine and reduced by intestinal bacteria to an amino phenolpyridazinone metabolite (OR-1855). This metabolite is further metabolised by acetylation to N-acetylated conjugate (OR-1896). The circulating metabolites OR-1855 and OR-1896 are formed slowly, and their maximum concentrations are seen on average 2 days after stopping a 24-hour infusion. The haemodynamic effects after levosimendan seem to be similar between fast and slow acetylators despite the fact that the enzyme N-acetyltransferase-2, which is responsible for the metabolism of OR-1855 to OR-1896, is polymorphically distributed in the population. Levosimendan reduces peripheral vascular resistance and has direct contractility-enhancing effects on the failing left ventricle. It also improves indices of diastolic function and seems to improve the function of stunned myocardium. Despite an improvement in ventricular function, levosimendan does not increase myocardial oxygen uptake significantly. An increase in coronary blood flow and a reduction in coronary vascular resistance have been observed. Levosimendan reduces plasma brain natriuretic peptide (BNP) and N-terminal pro-BNP (NT-proBNP) levels substantially, and a decrease in plasma endothelin-1 has been observed. Levosimendan also exerts beneficial effects on proinflammatory cytokines and apoptosis mediators. The effects of a 24-hour levosimendan infusion on filling pressure, ventricular function and BNP, as well as NT-proBNP, last for at least 7 days.

PZ101 ANZCA version Mar06 Q146 Regarding Placebos A. for every intervention, a fixed fraction of the population responds to placebo, whatever the outcome B. randomisation of different numbers of patients to active and placebo groups can affect the response to placebo C. the more invasive the method of delivering a treatment, the higher the response to placebo D. The placebo effect can be eliminated by using active treatments in both study groups E. the placebo response is a fixed fraction of the maximum effect of treatment

ANSWER D Common misconceptions about the response to placebo: Misconception 1: for every intervention, a fixed fraction of the population, usually a third, responds to placebo, whatever the outcome Misconception 2: the placebo response is a fixed fraction (about a third) of the maximum effect of treatment—the bigger the treatment effect the bigger the placebo response Misconception 3: the more invasive the method of delivering a treatment, the higher will be the response to placebo—injection will give bigger response than tablets Misconception 4: randomisation of different numbers of patients to active and placebo can affect the response to placebo

After coronary artery bypass graft surgery, the FRC is A. increased 40% B. increased 20% C. unchanged D. decreased 20% E. decreased 40%

ANSWER D Compared to preop -open sternotomy 55% increase -closure decrease 10% -day 1 decrease 20%

Complications of severe anorexia nervosa (body weight < 40% ideal) include all of the following EXCEPT A. cardiomyopathy B. delayed gastric emptying C. hypokalaemia D. hypercalcaemia E. prolonged QT interval

ANSWER D Complications CVS -sinus Brady -peripheral cyanosis -arrythmias ECG - av block, long QT, ST depression, T inversion -contractility only mildly impaired (but ipecac induced cardiomyopathy common) -MV proplapse Resp -Vomiting - aspiration, pneumothorax, pneumomediastinum hypovent due to alkalosis Renal -decreaesed GFR -proteinuria -calculi -hypo - K, Na, Cl, Ca GI -dental caries -parotitis -oesophageal injury -delayed gastric emptying Haem, Endo and skin -anaemia -leucopaenia -thrombocytopaenia -panhypopituitarism -high cortisol and GH -poor thermoregulation -osteoporosis -dry, cracked skin -perioral dermatitis

Complication of celiac plexus block A. Hypertension B. Failure of erection C. Constipation D. Paraplegia E. L3,4 lumbar pain

ANSWER D Complications • Severe hypotension may result, even after unilateral block. • Local pain during procedure • Diarrhoea • Intramuscular injection into the psoas muscle. • Bleeding due to aorta or inferior vena cava injury by the needle. • Intravascular injection (should be prevented by checking the needle position with radio-opaque dye). • Upper abdominal organ puncture with abscess/cyst formation. • Paraplegia from injecting phenol into the arteries that supply the spinal cord (prevented by checking the needle position with radio-opaque dye). • Lumbar nerve root irritation (injected solution tracks backwards towards the lumbar plexus). • Sexual dysfunction (injected solution spreads to the sympathetic chain bilaterally) = inability to ejaculate • Pneumothorax • Shoulder/chest/pleuritic pain/hiccupping - diaphragmatic irritation • Haematuria from renal pouncture

Black bank April 2009 Young man on the ward post ORIF # tib/fib. On morphine PCA, high demands/bolus given ratio, used 40mg morphine in last 2 hours (or something else high). Is a bit drowsy but has severe constant leg pain. Next step in management A. admit to intensive care B. increase bolus dose morphine PCA C. decrease lockout interval of PCA D. organise urgent orthopaedic review E. give more morphine until comfortable (or something else rubbish)

ANSWER D Concerned that this is compartment syndrome.

ANZCA April 2007 [60] A 25-year-old male presents for ECT (electroconvulsive shock therapy) at a free-standing facility. He has a life- threatening depressive illness that has not responded adequately to medication, however he is still taking tranylcypramine (Parnate). You should A. cancel the procedure, cease the tranylcypramine and perform the ECT in 2 weeks B. proceed with the ECT, but induce with midazolam and remifentanil C. proceed with the ECT, but pre-treat with esmolol D. proceed with the ECT with caution, but with your usual drugs E. transfer the patient to a tertiary centre for their ECT

ANSWER D Concerns regarding MOAI 1. sympathetic hyperreactivity 2. ECT results in vagus stimulus followed by overwhelming sympathetic surge Therefore there is a significant risk of hypertensive crisis with MOAIs in ECT J ECT. 2004 Dec;20(4):258-61. Electroconvulsive therapy in patients taking monoamine oxidase inhibitors. -Reviewed the literature, presented 4 new cases and concluded there is no evidence of a dangerous interaction between ECT and MAOI use. In general, a cautious approach would be to discontinue MAOIs before ECT if the medication has not been helpful; however, there is no need for a washout interval before starting ECT. Furthermore, if there is otherwise a reason for continuing the MAOI, it can be continued during index ECT or initiated during maintenance ECT.

Haemophilia A is commonly associated with: A. a haemarthrosis in a female infant B. a haemarthrosis in a male infant C. low levels of Factor IX D. normal prothrombin time (PT) and prolonged activated partial thromboplastin time (APTT) E. prolonged prothrombin time (PT) and prolonged activated partial thromboplastin time (APTT)

ANSWER D D - True (as extrinsic pathway is unaffected, PT is normal)

PI80 ANZCA version [2005-Sep] Q109, [Mar06] Q83, [Apr07] Q26 [Jul07] The MAC (Minimum Alveolar Concentration) of volatile anaesthetics is DECREASED by A. hypercarbia (pCO2 55 mmHg) B. hypernatraemia C. hyperthermia D. lithium E. tricyclic antidepressants

ANSWER D Decrease MAC Nitrous oxide Hypothyroid/myxoedema Hypocapnia Hypothermia-decrease is roughly linear Hyponatraemia Increasing age - (MAC Hal < 3 months ~ 1.1 % MAC Hal > 60 years ~ 0.64 %) Hypoxaemia Hypotension Anaemia Pregnancy CNS depressant drugs - opioids, benzodiazepines, major tranquilizers, TCA's Other drugs - lithium, lidocaine, magnesium, methyl dopa, clonidine Acute alcohol abuse

A 40-year-old man with Marfan's syndrome has undergone successful thoracoabdominal aortic reconstruction for aortic dissection. He has a CSF (cerebrospinal fluid) drain in situ. 48 hours post-operatively the CSF is noted to be bloodstained and he becomes obtunded. The most appropriate urgent investigation is A. CSF microscopy and culture B. MRI (magnetic resonance imaging) brain C. MRI spine D. non-contrast head CAT scan E. serum electrolytes

ANSWER D Decreased CSF pressure from drainage causing stretching then tearing of fragile subdural veins and consequent development of SDH Marfan is associated with intracranial aneurysms and lowering pressure in the CSF compartment via lumbar drain increases the transmural pressure gradient in the aneurysm (if there is one), hence increases the risk of rupture.

A young woman with type 1 von Willebrand disease presents for a dilatation and curettage. She is a Jehovah's Witness. You consider administering intravenous desmopressin in an attempt to reduce haemorrhage. Which of the following statements regarding desmopressin is FALSE? A. it is a synthetic substance and is acceptable to Jehovah's Witnesses B. it is likely to reduce haemorrhage in this patient C. it should be given 30 minutes prior to surgery as an infusion D. its duration of effect is approximately 5 days E. the intravenous dose is 0.3 mcg.kg-1

ANSWER D Elimination half life 2.4-4-4 hrs

Which statement about the use of Entropy depth of anaesthetic monitoring is true? A. artefacts caused by cardiac pacemakers produce a high RE (Response entropy) value B. RE only assesses the EEG (electroencephalogram) range from 32-47Hz (high frequency) C. SE (State Entropy) values range from 0 - 100 D. SE is a stable indicator of the effects of hypnotics on the cortex E. the RE algorithm filters the EMG (electromyogram) component

ANSWER D Entropy provides information on the depth of anaesthesia -measures the irrgularity of EEG and FEMG signs -EEG from irregular to more regular patterns as anaesthesia deepens Response entropy : fast-reacting -0 to 47Hz -display 0-100 -EMG -sensitive to the activation of facial muscles State Entropy : more steady and robust -0 to 32Hz -display 0-91 -EEG -NMBA are not known to affect EEG -state of hypnosis N2O does not affect BIS Pacemakers and diathermy are filtered

A nulliparous woman in labour for 8 hours with epidural analgesia has a fever 37.6 degrees. The most likely reason for this is A. altered thermoregulation B. chorioamnionitis C. urinary tract infection D. inflammatory response E. neuraxial infection

ANSWER D Epidural associated fever is common, ranging from mild hyperthermia to overt fever. Risk Factors 1. Nullparious 2. Prolonged labor 3. PROM Mechanisms postulated 1. Inflammation: most accepted explanation, unknown if it is infectious or non infectious (women with fever and epidural do not have evidence of chorioamonitis on histology) 2. Altered thermoregulation: hyperventilation during labor is diminished, resulting in reduced heat loss. 3. Effect of opioids: opioids suppress IL-2 formation www.anesthesia-analgesia.org/content/111/6/1467.full.pdf

Patients with ankylosing spondylitis usually exhibit A. skin rash B. raised Erythrocyte Sedimentation Rate (ESR) C. positive test for Rheumatoid Factor D. HLA B27 antigen E. urethritis

ANSWER D Extra-articular findings in ankylosing spondylitis * Acute anterior uveitis * Aortic insufficiency (about 3%) * Inflammatory bowel disease (symptomatic in 5-10%) * Slowly progressive upper lobe pulmonary fibrosis * Cardiac conduction defects including 3rd degree AV block * Prostatitis * Amyloidosis * Peptic ulcer disease secondary to treatment (steroids, NSAIDs) B27 >90%. ESR up in 75%

Regional anaesthesia for modified radical mastoidectomy requires blockade of the following nerves EXCEPT A. greater auricular B. auriculotemporal C. auricular branch of the vagus D. facial E. tympanic branch of the glossopharyngeal

ANSWER D Facial nerve block is not required. This nerve can become damaged during the operation. Greater auricular nerve block

Patient for fempop bypass (i believe it said "angioplasty"), history of CCF. Also has diabetes on oral hypoglycaemics, controlled hypertension and atrial fibrillation at rate of 80bpm. A. Medium risk surgery, medium risk patient B. Medium risk surgery, high risk patient C. High risk surgery low risk patient D. High risk surgery, medium risk patient E. High risk surgery, high risk patient

ANSWER D Fem-pop bypass is HIGH risk surgery This patient has 2 intermediate risk factors: a history of prior CCF, and diabetes. The controlled HT and AF<80bpm are minor risk factors * High risk pt = An "active cardiac condition" - Unstable coronary syndromes - Unstable or severe angina (CCS class III or IV) - Recent MI - Decompensated HF - NYHA class IV - Worsening or new-onset HF - Significant arrhythmias - High-grade A-V block - Mobitz type II A-V block - 3rd degree A-V block - Symptomatic ventricular arrhythmias - SVT (incl AF) with uncontrolled ventricular rate (HR>100bpm @ rest) - Symptomatic bradycardia - Newly recognised VT - Severe valvular disease - Severe AS (mean P gradient > 40mmHg; valve area < 1.0cm2; symptomatic) - Symptomatic mitral stenosis (progressive SOBOE, exertional presyncope, or HF) * Medium risk pt = Presence of "clinical risk factors" (presumably 1 or more) - ischaemic heart disease - compensated or prior heart failure - diabetes mellitus - renal insufficiency (creatinine >2mg/dL, which is >177umol/L) - cerebrovascular disease * Low risk pt = Minor predictors - Age >70 - Abnormal ECG (LVH; LBBB; ST-T abnormalities) - Rhythm other than sinus - Uncontrolled systemic hypertension

PZ45b ANZCA version [2001-Aug] Q5, [2003-Aug] Q34, [2004-Apr] Q30, [Mar06] Q17 When used to prevent aspiration pneumonitis A. ranitidine has a similar onset time and a shorter duration of action than cimetidine B. anticholinergic drugs increase gastric acidity and increase lower oesophageal sphincter tone C. non-particulate antacids have a longer duration of action than particulate antacids D. metoclopramide increases the lower oesophageal sphincter tone but relaxes the pyloric sphincter E. suxamethonium reduces the barrier pressure in most adults

ANSWER D For the ANZCA version, D is the best answer. As regards the other options: * A: Ranitidine has a LONGER duration (10-12hrs) than cimetidine (4-8hrs) although onset times are similar (1-2hs). (Morgan and Mikhail p244) * B: Anticholinergic drugs DECREASE gastric acidity. * C: Non-particulate antacids have a MORE rapid action and shorter duration than particulate antacids (Morgan & Mikhail pg 245) * E: Suxamethonium increases LOP pressure AND intragastric pressure such that barrier pressure is little affected. (Peck & Williams) LOS tone increased by: * metoclopramide * domperidone * neostigmine * pancuronium and decreased by: * anticholinergics * opioids * volatile anaesthetics * most IV anaesthetics * ganglion blocking drugs and unaffected by: * H2 receptor blockers Suxamethonium increases intragastric pressure but also increases LOS tone so that barrier pressure is maintained Ref: Yentis p322 * Cimetidine: onset 1-2hrs, duration 4-8hrs; * Ranitidine: onset 1-2hrs, duration 10-12hrs (Morgan & Mikhail pg 244) Non particulate antacids mix more rapidly with gastric contents and lose effectiveness within 30-60 minutes. (Morgan & Mikhail p245) Maxolon increases the tone and amplitude of gastric (especially antral) contractions, relaxes the pyloric sphincter and the duodenal bulb, and increases peristalsis of the duodenum and jejunum resulting in accelerated gastric emptying and intestinal transit. It increases the resting tone of the lower oesophageal sphincter. It has little, if any effect on the motility of the colon or gall bladder. MIMS

The American Heart Association guidelines for pre-operative cardiac risk assessment define poor functional capacity as being only able to exercise at a level of less than 4 METS. An exercise capacity of 4 METS would correspond to: A. Light housework such as dishwashing B. heavy work around the house such as moving heavy objects C. jogging for 2 kms D. walking briskly on level ground (6kph) E. walking slowly on level ground (3kph)

ANSWER D From AHA exercise standards Table 1. Clinically Significant Key Metabolic Equivalents for Maximum Exercise 1 MET = resting 2 METs = level walking at 2 mph 4 METs = level walking at 4 mph (6.4 kmph) <5 METs = poor prognosis; usual limit immediately after MI; peak cost of basic activities of daily living 10 METs = prognosis with medical therapy as good as coronary artery bypass surgery 13 METs = excellent prognosis regardless of other exercise responses 18 METs = elite endurance athletes 20 METs = world class athletes

In morbidly obese patients drugs can be administered on a per kilogram of total body weight (TBW) or a per kilogram of ideal body weight (IBW) basis. It is correct to say, A. atracurium should be administered on an IBW basis B. induction dose of propofol should be administered on an IBW basis C. induction dose of thiopentone should be administered on a TBW basis D. suxamethonium should be administered on a TBW basis E. vecuronium should be administered on a TBW basis

ANSWER D HIGHLY LIPOPHILIC DRUGS : ↑ VD IN OBESE. DRUG DOSES CALCULATED ON THE BASIS OF PATIENT'S TOTAL BODY WEIGHT (TBW): THIOPENTAL, PROPOFOL, BENZODIAZEPINES, FENTANY, SUFENTANYL, DEXMEDETOMIDINE, SUCCINYLCHOLINE, ATRACURIUM AND CISATRACURIUM WEAKLY LIPOPHILIC OR LIPOPHOBIC DRUGS : UNCHANGED VD. DRUGS DOSES ARE CALCULATED ON BASIS OF THE PATIENT'S LEAN BODY WEIGHT (LBW) = IDEAL BODY WEIGHT (IBW) + 20% TO 40% IBW: ALFENTANYL, KETAMINE, VECURONIUM,ROCURONIUM AND MORPHINE EXCEPTION : LIPOPHILIC DRUGS WITH UNCHANGED VD: REMIFENTANYL DOSES ACCORDING TO LBW reference yao and artusiao-gb

A 28 yr old patient with hypertrophic obstructive cardiomyopathy (HOCM) presents for an emergency laparotomy for suspected ectopic pregnancy. In anaesthetising this patient it is important to A. maintain a pulse rate of at least 100 beats.min-1 B. increase cardiac contractility C. reduce preload D. maintain sinus rhythm.

ANSWER D Haemodynamics with HOCM Hypertrophic obstructive cardiomyopathy is a dynamic condition, which is associated with the following: * Hypertrophy in an asymmetrical manner, with the septum and aortic outflow tract being hypertrophied to a greater extent than the rest of the LV * Dynamic motion of the hypertrophied portion of the heart leading to a function outflow resistance * Systolic anterior motion of the mitral valve, leading to a functional mitral regurgitation due to the Bernoulli effect on the mitral valve. Haemodynamic aims with HOCM * Slow heart rate * Normal to mildly reduced contractility * Sinus rhythm * Maintain afterload to ensure good coronary perfusion pressure * Full preload

During surgery there is increased secretion of each of the following hormones EXCEPT A. aldosterone B. glucagon C. growth hormone D. thyroid stimulating hormone E. vasopressin

ANSWER D INCREASED -GH -ACTH -B endorphins -Prolactin -ADH -Catecholamines -Cortisol -Aldosterone -Glucogon -Renin SAME -TSH -LH -FSH DECREASED -insulin -testerone -estrogen -T3

New onset atrial fibrillation in a 10 week pregnant lady. BP 150/90, HR 160, SaO2 92%. Moderate mitral stenosis on TTE, no thrombus seen. Emergency doctor gave her anticoagulant (not specified what). Most appropriate management: A. Verapamil B. Labetalol 20mg iv to 300mg C. Amiodarone 300mg IV D. Synchronised biphasic cardioversion with 70-100 Joule E. Oral digoxin -1000mcg then 500mcg 6 hrs later

ANSWER D In a pregnant woman who develops AF -diagnosis and treatment of the underlying condition causing the arrhythmia -ventricular rate should be controlled with digoxin, a beta blocker, or a nondihydropyridine calcium channel antagonist -All currently available antiarrhythmic drugs have the potential to cross the placenta and enter breast milk and should therefore be avoided if possible. -Quinidine, sotalol, flecainide, and amiodarone have all been used successfully during pregnancy, however, in relatively small numbers of cases. -Quinidine has the longest record of safety in pregnant women and remains the agent of choice for pharmacological cardioversion of AF in this situation. **In the event of hemodynamic embarrassment, direct-current cardioversion can be performed without fetal damage This patients Sats 92%, ?progressing failure and should be cardioverted

Patients with a history of anaphylaxis to penicillin should not receive: (a) ciprofloxaxin (b) clindamycin (c) gentamicin (d) imipenem (e) metronidazole

ANSWER D In patients with a history of a rash, the rate of cross-reactivity with cephalosporins is about 2-5%. However, in patients with a history of anaphylaxis to penicillins extreme caution is required with all β-lactams such as cephalosporins and carbepenems.

AM09e ANZCA version [Apr08] Q79 The diagnosis of neuroleptic malignant syndrome requires the presence of: A. Diaphoresis B. elevated plasma creatinine kinase C. hypertension D. muscle rigidity E. tachycardia

ANSWER D In the DSM-IV, there are criteria for diagnosis as follows (from Crit Care Med 2010 Vol. 38, No. 6 (Suppl) S244-52 (http://ovidsp.tx.ovid.com.ezproxy.anzca.edu.au/sp-3.2.4a/ovidweb.cgi?WebLinkFrameset=1&S=KPGAFPFAAJDDDOJKNCCLNBDCBBAJAA00&returnUrl=http://ovidsp.tx.ovid.com/sp-3.2.4a/ovidweb.cgi%3f%26Titles%3dS.sh.44%257c1%257c100%26FORMAT%3dtitle%26FIELDS%3dTITLES%26S%3dKPGAFPFAAJDDDOJKNCCLNBDCBBAJAA00&directlink=http://graphics.tx.ovid.com/ovftpdfs/FPDDNCDCNBJKAJ00/fs046/ovft/live/gv025/00003246/00003246-201006001-00017.pdf&filename=Adverse+drug+reactions+resulting+in+hyperthermia+in+the+intensive+care+unit.&navigation_links=NavLinks.S.sh.44.1&link_from=S.sh.44|1&pdf_key=B&pdf_index=S.sh.44)) A - Development of severe muscle rigidity and elevated temperature associated with the use of neuroleptic medication B - Two or more of; Diaphoresis, dysphagia, tremor, incontinence, changes in level of consciousness ranging from confusion to coma, mutism, tachycardia, elevated or labile blood pressure, leukocytosis, laboratory evidence of muscle injury (ie, elevated creatine phosphokinase) C - The symptoms in criteria A and B are not due to another substance (eg, phencyclidine) or a neurologic or other general medical condition (eg, viral encephalitis) D - The symptoms in criteria A and B are not better accounted for by a mental disorder (eg mood disorder with catatonic features)

PR56 ANZCA version [March 2006] Q97 The duration of action of suxamethonium may be increased by A. betamethasone B. bleomycin C. carvedilol D. neostigmine E. all of the above

ANSWER D Increases phase II block.

Which statement regarding a successful inferior dental nerve block is INCORRECT A. Associated with loss of taste on the ipsilateral anterior 2/3rds of the tongue B. Associated with sensory loss on the ipsilateral anterior 2/3rds of the tongue C. Associated with cutaneous sensory loss over the chin and lip of the ipsilateral side D. Unsuited for extraction of the incisor teeth of the ipsilateral side

ANSWER D Inferior Alveolar Nerve Block - Successful execution of this technique results in anesthesia of the mandibular teeth on the ipsilateral side to the midline, associated buccal and lingual soft tissue, lateral aspect of the tongue on the ipsilateral side, and lower lip on the ipsilateral side.

Iron deficiency anaemia: A. Increased serum ferritin, decreased serum Fe B. Decreased serum ferritin, increased serum Fe C. Decreased serum ferritin, increased TIBC D. Absence of Fe in bone marrow E. Decreased TIBC

ANSWER D Iron Deficency Anaemia -hypochromic and micocytic Decreased -plasma iron -ferritin -marrow depleted of iron stores Increased -transferrin

Iron deficiency is characterised by A. high serum ferritin level and low serum iron B. high serum ferritin level and absent bone marrow iron C. increased serum ferritin level and normal serum iron D. low serum ferritin level and low serum iron E. low serum iron level and lowered total iron binding capacity

ANSWER D Iron Deficency Anaemia -hypochromic and micocytic Decreased -plasma iron -ferritin -marrow depleted of iron stores Increased -transferrin

The segmental nerve supply to the renal pelvis and the ureter gives an anatomical basis for the surface representation of the pain of renal colic. The segments concerned are A. T11 and T12 B. L1 C. L1 and L2 D. T11, T12, L1 and L2 E. T12 and L1

ANSWER D KIDNEY Sympathetic preganglionic from T8-L1 converge onto celiac plexus and aorticorenal ganglia; post ganglionic to the kidney Parasympathetic from the Vagus nerve Pain T10-L1 URETER Sympathetic from the T10 to L2, post ganglionic fibres to aorticorenal and superior and inferior hypogastric plexuses Parasympathetic from the S2-S4 Pain T10-L2 BLADDER/URETHRA Sympathetic from T11 to L2 converge on the superior hypogastric plexus and supply the bladder through the right and left hypogastric nerves Parasympathetic from S2-S4 and form the pelvic parasympathetic plexus Pain dome : T11-L2 Neck S2-S4 Prostate Sympathetic T11-L2 Parasympathetic S2-S4 Pain T11-12, S2-4 Penis Sympathetic L1 and L2 Parasympathetic S2-4 Pain S2-4 from the pudendal nerve Scrotum is innervated anteriorly by the ilioinguinal and genitofemoral nerve (L 1 and L2), posteriorly by perineal branches of the pudenal nerve (S2 and S4) Testes (similar to kidneys where the begin during fetal development) Sympathetic T10-L2 Pain T10-L1

An anaesthetised patient is ventilated and has standard monitoring plus a central venous line. As the surgeon is commencing the case, the line isolation monitor (LIM) alarms indicating a potential leakage current of greater than 5 milliamps from one of the power circuits in use. The most appropriate action is to A.check the diathermy return plate B.disconnect the central line to electrically isolate the patient till the fault is identified C.make sure the patient is properly "earthed" or "grounded" D.sequentially unplug non-vital equipment from the circuit until the fault is identified E.suspend the operation and move the patient to a safe environment

ANSWER D LIM alarm warns that there is a single fault with one device of >5mA. Two faults are required to complete the circuit and cause a shock. The CVC transducer should be plugged into a type CF cardiac protected monitor. This means that even if the patient was to come into contact with a live source, less than 50microamps will flow through the CVL cable (due to either the module being isolated, or having very high internal resistance), hence preventing microshock. This means that theoretically, unplugging the CVL is not required, and may be actually harmful.

Which nerve is not blocked for ear surgery? A. Auriculotemporal nerve B. Greater auricular nerve C. Auricular branch of vagus D. Greater occipital nerve E. Lesser occipital nerve

ANSWER D Main Cutaneous nerves are * (Cervical plexus) greater auricular -> entire cranial surface + lower part of lateral surface * (Mandibular division) auriculotemporal -> upper part of lateral surface + most of meatal skin Other much smaller contributors are: * lesser occipital nerve (C2 root) -> small contribution of upper part of cranial surface * auricular br of vagus -> tympanic membrane + posterior meatal skin * facial nerve (via tympanic plexus) -> tympanie membrane + posterior meatal skin

What is the fluid replacement for maintenance of a 3kg neonate in the first 24 hours of life? A. 50mls B. 100mls C. 150mls D. 200mls E. 250mls

ANSWER D Maintenance Water Requirements Age ml/kg/day 1st day of life 60 2nd day 90 3rd day 120 up to 9 months 120-140 12 months 90-100 2 years 80-90 4 years 70-80 8 years 60-70 12 years 50-60 Frank Shann's book has * 2ml/kg/h first day * 3ml/kg/h second day * 4ml/kg/h subseqently Factors modifying water requirements Extra Required * Fever (add 12% for each °c above 37°C) * Hyperventilation * High ambient temperature * Extreme activity * Any other abnormal losses e.g. diarrhoea, polyuria Less required * Hypothermia (subtract 12% for each°c below 37°C) * Very high humidity * Oliguria or anuria * Extreme inactivity * Fluid retention e.g. cardiac failure

In a patient with aortic stenosis, A. aortic regurgitation is rarely seen B. the characteristic compensation of the left ventricle is dilatation C. presentation is usually with a cerebrovascular accident D. a valve area of < 0.8 cm2 represents severe disease E. atrial arrhythmias are commonly seen

ANSWER D Maintenance of sinus rhythm and adequate intravascular volume is vital to ensure ventricular filling. Arrhythmias must be treated promptly. New onset atrial fibrillation may require cardioversion Degenerative calcific aortic stenosis - This is now the most common form of aortic stenosis in the UK and tends to occur over the age of 70 yr. Mechanical stress over time leads to progressive fibrosis and calcification of a previously normal tri-leaflet valve Anaesthetic techniques that reduce systemic vascular resistance (e.g. regional neuroaxial techniques) must be used with extreme caution although successful cases of carefully titrated epidural and spinal blocks using catheters have been reported

70 year old post TKJR. On sub-cut heparin. Develops clinical DVT and platelets 40 (sounds like HITS type-II). Management A. Enoxaparin B. Fondoparinux C. Heparin by infusion D. Lepirudin E. Warfarin

ANSWER D Management of HIT: * First task is to discontinue unfractionated heparin from ALL sources (including heparin-coated lines, etc). LMWH can also cause HIT, therefore not suitable as a replacement. Fondaparinux is an indirect Factor-Xa inhibitor (synthetic pentasaccharide), and there are some reports of it being used in HIT successfully. Warfarin (Vit K antagonist) is contraindicated in acute HIT (or if suspected HIT), as it can cause skin necrosis or venous limb gangrene. * Current recommendations are to treat with DTI's (lepirudin, argatroban, bivalirudin) or danaparoid. Although danaparoid is a LMW heparinoid, there is an extremely low cross-reactivity rate with HIT antibodies, and this is rarely clinically significant. o As danaparoid is not an option, the best answer is therefore a direct thrombin inhibitor (DTI), and lepirudin is the only one listed, so answer is D.

A dock-worker is extricated from beneath a fallen lift platform and has signs of respiratory distress. He has a jagged wound of the chest wall with blood-stained froth alternately oozing from, and sucking back into the wound. The best initial treatment at the scene should be to A. apply 100% oxygen B. apply a CPAP (continuous positive airway pressure) mask C. apply a sterile dressing and seal the wound completely with adhesive tape D. apply a sterile dressing to the wound and seal on three sides only E. insert a 14G cannula in the 2nd intercostal space at the mid-clavicular line

ANSWER D Management of an open pneumothorax ("sucking chest wound") 100% oxygen should be delivered via a facemask. Consideration should be given to intubation where oxygenation or ventilation is inadequate. Intubation should not delay placement of a chest tube and closure of the wound. The definitive management of the open pneumothorax is to place an occlusive dressing over the 'wound and immediately place an intercostal chest drain. Rarely, if a chest drain is not available and the patient is far from a definitive care facility, a bandage may be applied over the wound and taped on 3 sides. This, in theory, acts as a flap-valve to allow air to escape from the pneumothorax during expiration, but not to enter during inspiration. This dressing may be difficult to apply to a large wound and it's effect is very variable. As soon as possible a chest drain should be placed and the wound closed. from Trauma.org. (Also has a video of an open pneumothorax) In summary: 1. Occlude wound 2. Place intercostal catheter (with seal i.e. underwater seal if in a hospital)

A 60yo Man with anterior mediastinal mass, during induction for mediastinoscopy....lose cardiac output, decreased saturations, drop in ETCO2. Management A. Adrenaline B. CPR C. CPB D. Place prone

ANSWER D Management of mediastinal mass Preoperative -establish anatomy : CT chest -establish pathology : most likely lymphoma but also thymoma, germcell tumor granuloma, bronchogenic carcinoma, thryoid tumor, bronchogenic cyst and cystic hydroma -intended surgery : diagnostic or therapeutic; sternotomy, thoracotomy, cervical mediastinoscopy, anterior parasternal mediastinoscopy, VATS -occasionally maybe for ermergenct surgery such as AAA or LUSCS -signs or symptoms that include chest pain or fullness, dyspnea, cough, sweats, superior vena cava obstruction, hoarseness, syncope or dysphagia -increased perioperative risk are increased dyspnea (orthopnea) or cough when supine (increased risk of airway complications) and syncopal symptoms or pericardial effusion (increased risk of cardiovascular complications). -does this patient need a GA? LA for anterior mediastinoscopy, needle biopsy, thoracentesis Intraoperative -step-by-step induction and continuous monitoring of gas exchange and haemodynamics (IAL) -maintain breathing with inhalation induction with sevoflurane or propfol-remi TIVA -possible AFO if there is non compressed trachea -development of airway or vascular compression requires the patient to be awakened as rapidly as possible -intraoperative lifesaving techniques Airway : repositioning or rigid bronchoscopy and ventilation distal to obstruction CVS : immediate sternotomy and surgical elevation of mass off he great vessel

What is the maximum recommended dose of Intralipid in local anesthetic toxicity (ml/kg) A. 6 B. 8 C. 10 D. 12 E. 14

ANSWER D Max cumulative dose =12ml/kg. ANZCA endorsed guideline. http://www.aagbi.org/sites/default/files/la_toxicity_2010_0.pdf

AM17d ANZCA version [2005-Apr] Q26 Myotonic dystrophy A. usually presents before puberty B. is frequently associated with cardiac failure C. is associated with obstructive lung disease D. contractions are not relieved by nondepolarising neuromuscular blockers and deep anaesthesia E. is associated with mitral valve in over 50% of patients

ANSWER D Myotonic dystrophy: Cataracts, subcapsular deposits, diabetes, cardiac conduction defect (30-40%), diastolic cardiac dysfunction (can occur) but cardiac failure is rare, no mention of valve disease, increased sensitivity to anaesthetic drugs, mental retardation may occur, frontal balding, testicular atrophy, ovarian dysfunction, respiratory limitation is restrictive, central and obstructive sleep apnoea, decreased smooth muscle motility à bowel and uterus (atony). Usually presents in young adulthood.

The most important factor in reducing peri-operative morbidity in diabetic patients undergoing peripheral vascular surgery is A. tight control of blood sugar level in the peri-operative period B. frequent blood sugar level estimations C. use of regional rather than general anaesthesia D. stabilisation of co-existing disease E. the use of an insulin infusion rather than a subcutaneous sliding scale regimen

ANSWER D NICE SUGAR tight control in ICU does worse

In patients with chronic renal failure there is A. increased calcium absorption B. increased phosphate excretion C. increased vitamin D3 production D. increased osteoclastic activity E. decreased osteoblastic activity

ANSWER D Nephron Destruction -as GFR fall, less PO4 is filtered and excreted -serum PO4 does not raise initially because of increased PTH secretion and therefore renal excretion -As GFR falls towards stages 4 and 5, hyperPO4 develops -HyperPO4 suppresses the hydroxylation of 25-hydroxyvitamin D to calcitriol (reduced Vit D) -Reduced Vit D -Reduced intestinal Ca reabsorption -Hypocalaemia Low serum calcitriol levels, hypocalcemia, and hyperphosphatemia have all been demonstrated to independently trigger PTH synthesis and secretion. As these stimuli persist in CKD, particularly in the more advanced stages, PTH secretion becomes maladaptive and the parathyroid glands, which initially hypertrophy, become hyperplastic. The persistently elevated PTH levels exacerbate hyperphosphatemia from bone resorption of phosphate.

Consider the following blood gases. Normal ranges are in brackets. pH 7.28 PaCO2 36 Bicarbonate 18 mmol.l-1 (18-25) Base excess -7 mmol.l-1 (-4- +3) Na+ 142 mmol.l-1 (135-145) Cl- 112 mmol.l-1 (98-110) These blood gases are consistent with A. acute renal failure B. diabetic ketoacidosis C. ethylene glycol overdose D. intraoperative infusion of 6 litres of normal saline E. salicylate overdose

ANSWER D Normal abion gap Metabolic Acidosis Causes include 1. Diarrhoea leading to HCO3 loss 2. ATN leading to HCO3 loss 3. Acetazolamide (carbonic anhydrase inhibitor) 4. TPN 5. hyperchloremic (NS) 6. Addisons 7. Hyperparathyroidism

According to the American Heart Association Guidelines on Perioperative Cardiac Evaluation for Noncardiac Surgery, which of the following operations has a high risk (>5%) of cardiac death or non-fatal myocardial infarct? A. carotid endarterectomy B. trans-urethral resection of the prostate (TURP) C. total knee joint replacement D. femoral-popliteal bypass E. right hemicolectomy

ANSWER D PROCEDURE RELATED RISK 1. Vascular (>5% risk of MI and/or death). Includes all vascular surgery except carotid endarterectomy 2. Intermediate (Less than 5%, more than 1%). Carotid endarterectomy, abdominal, thoracic, neuro, head and neck, major orthopaedic, prostatic, etc. 3. Low risk (less than 1%). Ambulatory, eyes, superficial, endoscopy, breast

Patho-physiological features of patients with morbid obesity include A. a blood volume:body weight ratio which is similar to that of patients with normal body weight B. an increased blood pressure and systemic vascular resistance compared to that of patients with normal body weight C. decreased gastric motility due to increased gastrin secretion D. cardiac pathology resulting from excess body mass and increased metabolic demand E. cardiac pathology resulting mainly from fatty infiltration or fatty change of the heart

ANSWER D Patho-physiological features of patients with morbid obesity include A. FALSE : "Total blood volume is increased in the obese but on a volume/weight basis is less than that in non-obese individuals B. FALSE : "hypertension is presumably caused by an increased cardiac output forced into an unaltered peripheral resistance" C. FALSE : gastrin is a prokinetic hormone D. TRUE : cardiac pathology resulting from excess body mass and increased metabolic demand - true by default E. FALSE : The morbidly obese individual is at risk of a specific form of obesity-induced cardiac dysfunction, although the belief is that this is secondary to fatty infiltration of the heart ('cor adiposum') is no longer valid

The risk of latex allergy is increased in all of the following patients EXCEPT those A. who work in hospitals B. with chronic urological conditions C. with food allergy D. with sarcoidosis E. with spina bifida

ANSWER D People at increased risk for developing latex allergy include people who have: * Congenital urologic abnormalities * A history of multiple surgical procedures * Intermittent catheterization * Dental dams for endodontic care * Atopy, asthma or eczema * Food allergies to banana, avocado, kiwi or chestnuts

Complications of severe anorexia nervosa (body weight < 40% ideal) include all of the following EXCEPT A. cardiomyopathy B. delayed gastric emptying C. hypokalaemia D. hypercalcaemia E. prolonged QT interval

ANSWER D Perioperative Considerations for Anorexia Nervosa Cardiovascular -hypotension : resulting from dehydration -bradycardic : decreased basal metabolic rate -ECG : AV block, ST depression, T wave inversion and QT prolongation; >80% will have an ECG abnormality -QT prolongation maybe caused by hypocalcaemia, hypomagnaesemia, drugs or starvation -hypophosphataemia reduces arrythmia threshold -Other arrhythmias : sinus arrest, wandering atrial pacemakers, nodal escape beats, SVT or VT; Reported incidence of 16-62% under anaesthesia. -reduced myocardial contractility : due to malnutrition, also ipecac is directly cardiomytoxic, avoid fluid overload -mitral valve prolapse Respiratory -metabolic alkalosis from self induced vomiting and laxatives produces compensatory bradypnoea -pneumothorax -aspiration pneumonia -pneumomediastinum Renal -hyponatraemia, hypokalaemia, hypochloraemia, hypomagnesaemia, hypocalaemia metabolic alkolosis -renal calculi from chronic ingestion of magnesium hydroxide

Reverse splitting of the 2nd heart sound is caused by A. acute pulmonary embolism B. ASD C. complete LBBB D. severe MR E. pulmonary HT

ANSWER D Physiology of the Second Heart Sound -Audible component of the closure of aortic and pulmonary valves -A2 followed by P2, separated only by 0.02 - 0.08 seconds. -Although contraction of both ventricles occurs at the same time, difference in opening and closure are due to differences in impedance of each vascular bed -as PVR is low, PV opens sooner and closes later -as SVR is high, AV opens later and closes sooner -therefore A2-P2 -this interval is widened by deep inspiration, known as Physiological Splitting. Abnormal Splitting of Second Heart Sound (1) persistently single; -S2 remains single throughout respiratory cycle -most commonly due to inaudibility -in setting of CHD : absence of PV : Fontans (pulmonary atresia, severe PV stensosis, dysplastic PV, transposition of great arteries) (2) persistent (audible expiratory) splitting, with normal respiratory variation; -most common cause if RBBB (3) persistent splitting without respiratory variation (fixed splitting); -A2 and P2 is constantly wide and remains unchanged during respiratory cycle -Hallmark of ASD (4) reversed (paradoxical) splitting. -Reversed sequence of semilunar valve closure P2-A2 -Common causes : LBBB and Right ventricular PPM (activation of the RV before LV) -Other causes : RV ectopic beats, prolonged LV mechanical systole, HOCM, Severe hypertension, PDA -

The PiCCO monitor (Pulsion Medical Systems) combines pulse contour analysis and transpulmonary thermodilution to provide a continuous measurement of: A. cardiac output B. cardiac output and intermittent assessment of intrathoracic blood volume C. cardiac output and intermittent assessment of extravascular lung water D. cardiac output and intermittent assessment of intrathoracic blood volume and extravascular lung water

ANSWER D PiCCO : Pulse Contour Cardiac Output Monitoring Uses CVC and femoral or axilliary IAL Employs patented algorithms to combines real-time continuous monitoring through pulse contour analysis with intermittent thermodilution via the transpulmonary method. PiCCO technology provides clinicians with the following clinical measurements, many of which can be displayed as absolute or indexed values: via continuous pulse contour analysis * Continuous pulse contour cardiac analysis (PCCO) * Arterial blood pressure (AP) * Heart rate (HR) * Stroke volume (SV) * Stroke volume variation (SVV) * Systemic vascular resistance (SVR) * Index of left ventricular contractility via intermittent transpulmonary thermodilution * Transpulmonary cardiac output (C.O.) * Intrathoracic blood volume (ITBV) * Extravascular lung water (EVLW) (Not Available in United States) * Cardiac function index (CFI)

MN02 ANZCA version [2002-Mar] Q25 A five-year-old child presents with headache, morning vomiting and double vision for three weeks. On examination nystagmus is present when the eyes are turned to either side. The most likely diagnosis is A. tuberculous meningitis B. craniopharyngioma C. aseptic meningitis D. posterior fossa tumour E. acoustic neuroma

ANSWER D Posterior fossa tumour Symptoms and their association: * early morning vomiting = raised ICP * double and blurred vision, nystagmus = infratentorial lesion * problems with equilibrium, gait, and coordination = infratentorial lesion * focal problems (eg motor or sensory deficit, speech change, seizures) = supratentorial lesion * strong hand preference = supratentorial lesion * neuroendocrine problems (DI, hypothyroidism) = suprasellar lesion * change in visual acuity, visual field defect, Marcus Gunn pupil (afferent papillary defect), nystagmus = visual pathway lesion * long nerve tract motor and/or sensory deficits, bowel and bladder deficits, and back or radicular pain = spinal cord lesion

The power of a statistical test can be expected to decrease, if there is an increase in A. the sample size B. the size of the treatment effect C. the chance of making a Type 1 error D. the variability of the population E. none of the above

ANSWER D Power is the likelihood of detecting a specificed difference if it exists. It is equal to 1-beta error. Factors influencing power 1. Significance criterion : increasing alpha error will increase the chance of obtaining a statistically significant result (reject the null hypothesis) 2. Magnitude of effect : larger the effect, the greater the chance of obtaining a significant result 3. Sample size : easiest way to boost the power of a test 4. Precision (ie SD) this is influenced by the sample size 5. Design of the test , for example RCTs have the highest power Sample size =

AM28a ANZCA version [2002-Mar] Q35, [2002-Aug] Q24 (Similar question reported in [Apr98] [Jul98] [Apr99] [Aug99]) In a patient with myasthenia gravis undergoing a laparotomy for large bowel obstruction, the need for post-operative ventilation is significantly increased by a A. daily dose of pyridostigmine > 450mg B. known history of resistance to suxamethonium C. past history of prednisolone treatment > 10 mg.day-1 D. recent history of dysphagia E. past history of thymectomy

ANSWER D Preoperative predictors of postoperative need for ventilation: * duration of disease of greater than 6 years * history of coexisting chronic resp disease * dose requirements of pyridostigmine > 750 mg/day less than 48 H prior to surgery * preoperative VC < 2.9L

Conn's syndrome (primary aldosteronism) is characterised by all except A. Hypertension B. Oedema C. Hypokalaemia D. Oliguria

ANSWER D Primary hyperaldosteronism is one of the more common causes of secondary hypertension. The term Conn's syndrome is used for primary hyperaldosteronism secondary to an adrenal adenoma secreting aldosterone. The syndrome is due to: * bilateral idiopathic adrenal hyperplasia 70 % * unilateral idiopathic adrenal hyperplasia 20 % * aldosterone-secreting adrenal adenoma (benign tumor, < 5%) * rare forms, including disorders of the renin-angiotensin system Aldosterone acts in kidney to increase Na+ reabsorption and increase K+ secretion. -Water follows Na, therefore increase in extracellular fluid volume -hypokalaemia -metabolic alkalosis -Hypomagnesemia -hypoglycaemia Clinical Features -refractory hypertension, hypervolaemia, metabolic alkolosis -hypokaelaemia -muscle weakness or paralysis -nephrogenic diabetes insipidus -impaired glucose tolerance Treatment is adrenalectomy 1. Medical optimisation a. spironoloctone (400mg/day, compeditively inhibits aldosterone) b. correct K+, HCO3 c. Treat hypertension d. exclude LVH, treat as for CCF if present

PI76 ANZCA version [2005-Apr] Q138, [2005-Sep] Q4 When comparing propofol (induction and infusion for maintenance) with desflurane (propofol induction only) anaesthesia for ambulatory surgery, which statement is INCORRECT? A. anti-emetic use is higher with desflurane B. incidence of postoperative nausea is higher with desflurane C. time to eye opening in recovery is shorter with desflurane D. time to home readiness is shorter with desflurane E. time to obeying commands in recovery is shorter with desflurane

ANSWER D Propofol Versus Desflurane There were 13 studies (2,5,8,25-34) with extractable data that were included in the meta-analysis (Table 2). Time to eye opening was significantly quicker with desflurane compared with propofol (1.3 min), as was the time to obey commands (1.3 min), with significant heterogeneity between the anesthetics (Table 5). No differences were found in home readiness or discharge between the anesthetics. The RR for postoperative complications including PON (NNT, 7) and POV (NNT, 10) was significantly greater with desflurane compared with propofol and the need for antiemetics was also increased with desflurane (RR, 3.9) (Table 5). No other differences were seen between the anesthetics. Gupta (et al) Comparison of Recovery Profile After Ambulatory Anesthesia with Propofol, Isoflurane, Sevoflurane and Desflurane: A Systematic Review Anesth Analg 2004;98:632-41

Patient over-warfarinised and is for surgery. Prothrombinex 50U/kg may NOT reverse an INR of 5.5 because it contains: A. Citrate B. Heparin C. Anti-thrombin III D. Not enough Factor VII E. Not enough Factor X

ANSWER D Prothrombinex has 2, 9 and 10 only. Warfarin inhibits 2, 7, 9 and 10. It does contain human plasma proteins < 500mg (incl VII), but VII levels are low and unquantified PTX administration without FFP is recommended only when FFP is unavailable, as PTX factor VII levels are low and unquantified. MIMS Australia.

Abnormal Q waves are NOT a feature of the ECG in A. an old myocardial infarction B. left bundle branch block C. recent transmural myocardial infarction D. digitalis toxicity E. Wolff-Parkinson-White syndrome

ANSWER D Q waves transmural MI, LBBB, hyperkalaemia, WPW, amyloid, HOCM, cardiac contusion, myocarditis, dextrocardia, reverse limb leads

Chest Xray findings in a child who has inhaled a foreign body: A. Opaque mass overlying the airway B. Hyper-expanded lung fields C. Unilateral pulmonary oedema D. Collapsed lung base E. Mediastinal shift

ANSWER D RCH website Radiology Request inspiratory and expiratory chest films. Look for: ▪ an opaque foreign body ▪ segmental or lobar collapse Hyperinflation - localised emphysema in expiration (ball valve obstruction) CXR may be normal. (Other sites states hyperinflation important sign- Loren also thinks its hyperinflation??)

Features of paracetamol administration in children include A. limitation of the daily dose to a maximum of 150 mg.kg-1 because of the risk of hepatotoxicity B. reliable absorption when administered rectally with most patients achieving a therapeutic concentration with a loading dose of 20 mg.kg-1 C. peak blood levels being reached approximately 1 hour following rectal administration D. a one hour delay between peak plasma concentration and maximum analgesia E. a faster absorption of high dose rectal paracetamol compared to oral

ANSWER D Rectal administration may be erratic and prolonged. Therapeutic plasma concentrations (>10mcg/mL) are rarely achieved with a rectal loading dose of 20mg/kg but more commonly achieved with a rectal loading dose of 40-60mg/kg. Peak plasma levels following rectal administration are at 2.6hrs and following oral administration at 1.6-1.9 hrs. Rectal absorption is thus slower than oral absorption. Toxicity occurs with plasma levels greater than 120mg/ml. The maximum daily dose of oral paracetamol is 90-100mg/kg because at 150mg/kg hepatotoxicity has occurred. The maximum daily dose for rectal paracetamol has not been elucidated in children

Factors associated with post-operative ulnar nerve palsy include all of the following EXCEPT A. male gender B. sternal retraction for cardiac surgery C. cardiopulmonary bypass for cardiac surgery D. internal jugular vein catheterisation E. diabetes mellitus

ANSWER D Risk factors * Male * Diabetes (due to preexisting neuropathy) * Positioning * Operations e.g. cardiac surgery with midline sternotomy * Direct trauma e.g. needles such as IJ cannulation * Thermal injury e.g. diathermy * Surgical trauma e.g. orthopaedics

The risk of seroconversion after a needlestick injury, with a hollow needle from a HIV positive patient, is: A. 30% B. 15% C. 3% D. 0.3% E. 0.03%

ANSWER D Risk of HIV seroconversion 1. Hollow needle : 0.3% 2. Mucous membrane : 0.003%

A mixture of 20% oxygen and 80% helium is passed through an oxygen rotameter: A. Flow will be independent of the rotameter setting B. At 1 l/m flow setting, the real flow is 500 mls/min C. At 10 l/m flow setting, the real flow is 20 l/m D. At 10 l/m flow setting, the real flow is 15 l/m E. At 5 l/m flow setting, the real flow is 15 l/m

ANSWER D Rotameters * Rotameters have a mixture of laminar and turbulent flow o Tend to be laminar with low flows, turbulent with high flow o Tend to laminar at bottom of rotameter, turbulent at top * Rotameters are constant pressure, variable orifice flowmeters * Laminar flow o Q = ΔPπr4 / 8nl * Turbulent flow o Q ∞ r2 o Q ∞ sqrt(ΔP) o Q ∞ 1 / length o Q ∞1/ sqrt(ρ) (density) Calculations The actual flow rate is obtained by multiplying the flow rate on the meter by the square root of the density ratios. Density Helium = 0.164 kg/m3 Density Oxygen = 1.31 kg/m3 Heliox 70/30, multiply reading by 1.6 (oxygen-calibrated meter) and 1.5 (air-calibrated one). For heliox 80/20, the multiplying factors are 1.8 and 1.7 for oxygen and air-calibrated devices respectively.

In a rotameter the: A. Bobbin spins inside a tube that has parallel sides B. Flow is laminar at high flow rates C. Height of the bobbin is proportional to the pressure drop across the bobbin D. Pressure drop across the bobbin is constant at varying flows E. Resistance increases with increasing gas flow

ANSWER D Rotameters are widely used in gas delivery systems for continuous flow measurement. -vertical tapered tube containing a bobbin or ball which is supported by the gas flow as it passes upwards through the tube -weight of the bobbin (and thus the pressure drop required to support it) is constant, but as the flow increases its position in the tube rises, lowering the resistance as a larger pathway is created alongside the bobbin. -laminar flow but at the top of the tube produce turbulent flow. -The physical characteristics of the gas that determine the resulting flow are therefore viscosity at the bottom and density at the top of the tube. -calibration of rotameters is gas-specific and for accuracy its use must be restricted to that gas -the flow rate from the top of a bobbin but the centre of a ball. The range of gas flow measurements can be increased by using two tubes (one for low and one for high flow rates), or by varying the taper so that a greatly increased diameter results at the top of the tube. Inaccuracy results from anything that causes the bobbin to stick in the tube, including dirt or static electricity. To prevent build-up of static, the inside walls of a rotameter and its mounting points are made of conductive material. To demonstrate that the bobbin is not stuck, it has angled flutes to produce rotation, which is made easier to see by appropriate colouring. Back pressure caused by downstream resistance also leads to an inaccurately low reading on a rotameter, though the actual flow is the same as that shown before the resistance was applied.

A 50-year-old female patient presents with a 12 hour history of feeling unwell and is found to fulfill the criteria for Systemic Inflammatory Response Syndrome. Her blood pressure is 80/45 mmHg, her pulse rate is 90 beats.min1 and her central venous pressure is 12 mmHg. The hypotension is most appropriately managed with A. adrenaline B. dobutamine C. dopamine D. ephedrine E. noradrenaline

ANSWER D SIRS = vasodilated with hyperdynamic heart. Resusitation Goal from Surviving Sepsis Guidelines 1. CVP 8-12 2 MAP >65 3. UO >0.5ml/kg/hr 4. CVP O2 sat>70% or mixed venous >65%

Lumbar epidural analgesia in labour using 0.125% bupivacaine A. improves FVC (forced vital capacity) if the upper sensory level is kept below T12 B. improves FVC and FEV1 (forced expiratory volume in one second) if upper sensory level is kept below T12 C. improves FVC, even if the sensory level is above T10 D. improves FVC and FEV1, even if the sensory level is above T10 E. reduces FVC and FEV1 if the sensory level is above T10

ANSWER D Seems to be based on a study from Anaesthesia Volume 59 Page 350 - April 2004 The effect of epidural analgesia in labour on maternal respiratory function. The respiratory function measurements were taken with mothers completely PAIN FREE, so must have had a block to T10: "As soon as a sensory blockade above T10 was obtained, we started a continuous infusion of 10 ml.h−1 bupivacaine 0.125% with fentanyl 0.0001%...The upper sensory level of epidural analgesia was T8 (T6-T8[T4-T10])" from above article. FVC, FEV1, and PEF all improved.

Findings in a patient with serotonin syndrome include each of the following EXCEPT A. clonus B. diaphoresis C. hyperreflexia D. miosis E. tachycardia

ANSWER D Serotonin toxicity has now been more clearly characterized as a triad of neuro-excitatory features. 1. Neuromuscular hyperactivity -tremor -clonus -myoclonus -hyper-reflexia -pyramidal rigidity (advanced) 2. Autonomic hyperactivity; -diaphoresis -fever -tachycardia -tachypnoea. 3. Altered mental status -agitation -excitement -confusion (advanced)

A 60yo man with anterior mediastinal mass, during induction for mediastinoscopy....lose cardiac output, decreased saturations, drop in ETCO2. Management: A. Adrenaline B. CPR C. CPB D. Place prone E.

ANSWER D Slinger describes 2 scenaros 1. airways compression -reposition lateral or prone (this should be determined before, which position is best for patient) -ventilate with rigid bronch inserted distal to obstruction 2. Cardiovascular compression -lighten anaesthesic -if this does not work : immediate sternotomy and CPB -can consider placing prone

A 26-year-old female with ulcerative colitis has a total colectomy with J pouch formation. Preoperatively she was on regular oxycodone, fluoxetine and prednisolone orally. She has normal renal function and liver function. Her postoperative pain management consists of PCA (patient controlled analgesia) with morphine, regular intravenous tramadol (100 mg every 6 hrs) and regular intravenous paracetamol (1 gm every 6 hrs). When you assess her 24 hrs postoperatively she is agitated, confused and sweaty with a pulse of 120, BP 150/95 and temperature of 38°C. You should: A. administer naloxone B. administer 100 mg of hydrocortisone C. cease her paracetamol D. cease her tramadol E. change her PCA morphine to PCA fentanyl

ANSWER D Sounds like serotonin syndrome - hence stopping tramadol is the best answer.

MR35 ANZCA version [2001-Aug] Q16 (Similar reported question in [Jul97] [Jul98] [Apr99] [Mar00]) The lobar collapse which is most easily missed on chest X-ray, is A. right upper lobe B. left lower lobe C. right lower lobe D. left upper lobe E. right middle lobe

ANSWER D Structures behind the heart are obscured, and left lower lobe pathology may be easily missed.

A patient experiences parasthesia in the little finger during supraclavicular brachial plexus block, the needle is in proximity to the A. posterior cord B. middle trunk C. Ulnar nerve D. lower trunk E. medial cord

ANSWER D Supraclav block blocks the trunks of brachial plexus. Ulna nerve (little finger) from lower trunk.

Infra-renal aortic cross-clamping usually results in A. decreased cardiac contractility B. decreased coronary blood flow C. decreased renal blood flow D. minimal change in cardiac output E. increased heart rate

ANSWER D Systemic haemodynamic response to aortic cross-clamping -preload does not always increase -infra-renal aortic cross-clamping blood volume shifts into splanchnic circulation and preload does not increase MAP : no change PAP : no change End-diastolic area : increase 9% End-systolic area : increases 11% Abnormal wall motion : 0 New AMI : 0%%//d3cgb598vs7bfg.cloudfront.net/images/upload-flashcards/back/1/1/49011489_m.jpg

The J-point of the ECG waveform is altered by all of the following EXCEPT A. hypothermia B. tachycardia C. myocardial ischaemia D. calcium channel blockers E. digoxin

ANSWER D The "J-point" is the end of the QRS complex and serves at the take-off point of the ST segment. The J Wave deflection, aka Osborn Wave. It represents the approximate end of depolarization and the beginning of repolarization as determined by the surface ECG. J point elevation is very common in young healthy individuals. Has no pathological implications whatsoever. But must be distinguished from ST elevation (J point elevation, the P wave maintains its own shape) Causes of J point depression from baseline 1. hypothermia 2. hypercalcaemia 3. brain injury, SAH 4. early repolarization, tachycardia 5. epicardial or endocardial ischemia 6. percarditis 7. RBBB/LBBB 8. RVH / LVH 9. Digitialis Clinical Significance? Early sign of tachyarrthmias has been reported%%//d3cgb598vs7bfg.cloudfront.net/images/upload-flashcards/back/3/3/47033811_m.jpg

Cerebral oximetry measures? A. Arterial saturation B. Mostly arterial saturation and some venous saturation C. Capillary saturation D. Mostly venous saturation and some arterial saturation E. Venous saturation

ANSWER D The Official Journal of the Anesthesia Patient Safety Foundation. Spring 2009 "Cerebral oximetry differs from pulse oximetry in that tissue sampling represents primarily (70-75%) venous, and less (20-25%) arterial blood" Cerebral Oximetry: Monitoring the Brain as the Index Organ. Anesthesiology 2011 "Sco2 is weighted for approximately 70-75% venous blood" http://anesthesiologynews.com/download/CerebralOximetry_AN0412_WM.pdf

Expected adverse drug effects in a geriatric population receiving a high dose of a selective serotonin reuptake inhibitor for depression would include all of the following EXCEPT A. hyponatraemia caused by inappropriate secretion of ADH B. impairment of platelet aggregation caused by depletion of 5HT (serotonin) stores C. withdrawal symptoms characterised by anxiety, agitation and increased sweating D. sedation, dry mouth, orthostatic hypotension and cardiac conduction defects E. gastro-intestinal effects (nausea, vomiting, diarrhoea)

ANSWER D The SSRIs do NOT cause sedation, dry mouth, orthostatic hypotension and cardiac conduction defects, all of which are seen with the tricyclics." (Selective serotonin reuptake inhibitors Pharmacology and clinical implications in anaesthesia and critical care medicine in Anaesthesia Volume 52, Issue 10, Date: October 1997, Pages: 982-988)

If a 50% mixture of helium and oxygen is passed through an oxygen rotameter which of the following statements are most nearly true: A. If rotameter reads 10 l/m, the actual flow is approximately 20 l/m. B. If rotameter reads 2 l/m, actual flow is approximately 1 l/m C. If rotameter reads 2 l/m, actual flow is approximately 0.5 l/m D. A direct relationship exists between reading and actual flow E. None of the above

ANSWER D The actual flow rate is obtained by multiplying the flow rate on the meter by the square root of the density ratios. Density Helium = 0.164 kg/m3 Density Oxygen = 1.31 kg/m3 Heliox 70/30, multiply reading by 1.6 (oxygen-calibrated meter) and 1.5 (air-calibrated one). For heliox 80/20, the multiplying factors are 1.8 and 1.7 for oxygen and air-calibrated devices respectively.

The sciatic nerve supplies the following muscles EXCEPT A. biceps femoris B. semitendinosus C. semimembranosus D. gluteus maximus E. adductor magnus

ANSWER D The branches of the sciatic nerve can be grouped into the following (from Ellis p200): 1 Muscular to: BASS * (a) bicep femoris * (b) adductor magnus * (c) semitendinosis * (d) semimembranosis. 2 Articular to: * (a) the hip joint. 3 Terminal to: * (a) common peroneal (lateral popliteal) nerve; * (b) tibial (medial popliteal) nerve.

The leading cause of anaphylaxis during anaesthesia is: (a) antibiotics (b) induction drugs (c) latex (d) muscle relaxants

ANSWER D The causes in decreasing order frequency are: muscle relaxants (70%), latex (12%), antibiotics (8%) induction drugs (4%), colloids (3%) and opioids (1.5%)

Prior to elective surgery, an increased bleeding risk is most likely to be identified by A. a platelet count B. a prothrombin time C. a skin bleeding time D. an accurate clinical history E. an examination for bruising

ANSWER D The following extract is from Up ToDate: Preoperative assessment of hemostasis (http://www.utdol.com/utd/content/topic.do?file=coagulat/12865&type=A&selectedTitle=3~44) The clinical history — The clinical history remains the cornerstone of the preoperative assessment. However, four reasons have been cited why the history alone may be insufficient, suggesting a role for laboratory testing in some patients [1]: * The forgetful physician — simply put, this is a surgeon who does not take an adequate history. In our current medicolegal climate, this may be a very compelling argument for laboratory screening tests. * The unreliable patient — a common example of the unreliable patient is one who claims to have no prior surgical history but neglects to mention wisdom teeth extractions or tonsillectomy. Both procedures are quite common, often occurring during the childhood years, and hence are often forgotten by the time of adulthood. Both operations are true challenges to hemostasis, since the oral cavity is rich in fibrinolytic substances. Alternatively, patients may recall these procedures but do not consider the bleeding they may have experienced as excessive. Having no basis for comparison, they may inadvertently overlook this complication. Other relevant items often overlooked include circumcision, childbirth and associated surgical procedures (eg, episiotomy), epistaxis, and the extent of menstrual blood losses. * The unprovoked patient — a patient with an inherited bleeding disorder may not have experienced excessive bleeding in the past, especially if there has not been any prior surgery. In such patients the history will be uninformative unless there is a family history of bleeding (eg, mild hemophilia). * The acquired disorder — patients with an acquired bleeding disorder may have no history of excessive surgical bleeding because their hemostatic defect is relatively recent in onset (eg, acquired von Willebrand disease).

A 4y.o. boy presents for insertion of grommets. His maternal great-grandfather is know to have had an episode of malignant hyperthermia. Which of the following is the strongest evidence that the boy is NOT susceptible to MH? A. the boy was exposed to halothane at age 2 with no sequelae B. the boy has recently been shown to have a normal serum creatinine kinase C. the boy's grandfather has had a negative muscle contracture test for MH D. the boy's mother has had negative molecular genetic testing for MH E. there have been NO other episodes of MH in the family despite exposure to known triggers on multiple occasions

ANSWER D The ideal anaesthestic if you decide to do the child with URTI is "minimally invasive". I.e. if you can use a face mask use one, if you can use an LMA use one as instrumentation of the reactive airway can get you into trouble.

Post-operative nausea and vomiting in children A. is more common in females (than males) before puberty B. is more common in children under two years of age (than older) C. is rare following orchidopexy D. is twice as frequent in children over the age of three, compared to adults E. should not be treated with ondansetron

ANSWER D The incidence of PONV above preschool age is higher than in adults. It depends on age: * 34-50% in school children * 20% in preschool children * 5% (lowest incidence) in infants There is a greater incidence in girls but this only begins after 13 years of age

The intercostobrachial nerve: A. Arises from T2 trunk B. Is usually blocked in brachial plexus block C. Supplies antecubital fossa D. Can be damaged by torniquet E. Arises from inferior trunk

ANSWER D The intercostobrachial nerve is the (modified) lateral cutaneous branch of T2. It supplies inner side of upper arm, so in under the torniquet in arm surgery. * A - False. Arises from lat. cut. branch of 2nd intercostal nerve (from T2 originally, but not directly from the trunk) * B - False. It joins the medial cutaneous nerve of the arm which comes from the medial cord, but does not form part of the brachial plexus, and is not blocked in brachial plexus blocks. * C - False. Supplies medial side of upper arm, and joins medial cutaneous nerve of arm which supplies medial side of upper arm down to the elbow. * D - TRUE. Any nerve compressed by a tourniquet can be damaged. Would have to be high up the arm/close to axilla to compress it. * E - False. Not part of brachial plexus, or a branch from it. Arises from lat. cut. branch of 2nd intercostal nerve.

Which distinguishes C8-T1 from an ulnar nerve lesion at elbow? A. Paraethesia of the 5th digit B. Paraesthesia over index finger C. Flexor carpi ulnaris function D. Paraesthesia/sensory loss over medial forearm E. Adductor pollicis function

ANSWER D The medial cutaneous nerve of the forearm arises from the medial cord of the brachial plexus which is prior to the formation of the ulnar nerve.

Patient presents with carcinoid syndrome and developes hypotension intraoperatively. Best drug to treat it is: A. Noradrenaline B. Adrenaline C. Metaraminol D. Octreotide E. Ephedrine

ANSWER D The occurrence of intraoperative carcinoid crisis manifesting as bronchospasm or hypotension is treated with IV octreotide 100-200 mcg.

Pre-ganglionic sympathetic fibres pass to the A. otic ganglion B. carotid body C. ciliary ganglion D. coeliac ganglion E. all of the above

ANSWER D The otic ganglion is supplied by pre-ganglionic parasympathetic fibres The carotid body is supplied by the glossopharyngeal nerve (NMS, The ciliary ganglion is supplied by pre-ganglionic parasymathetic and post-ganglionic sympathetic fibres Somatic nerves all get postganglionic. Head and neck receives postganglionic fibres from the sympathetic chain. The visceral plexuses: coeliac, hypogastric and pelvic are receive preganglionic via splanchnic nerves to supply postgangionic to viscera (except adrenal medulla).

MR42 ANZCA version [2001-Aug] Q18 A 30 year old male presents with a single sudden shaking chill with a rapid rise in temperature. Within a few hours agonizing left pleuritic pain and cough with rust coloured sputum supervene. On examination the man appears acutely ill with rapid shallow respiration. With no treatment he continues to run a temperature up to 40.5 deg C for a week at which time he experiences a dramatic improvement. Physical examination on day 2 of the illness would probably show all of the following over the left chest EXCEPT A. restricted hemithorax movement B. flat percussion sound C. bronchial breathing D. decreased fremitus E. whispering pectoriloquy

ANSWER D The presumptive diagnosis in this case is left sided pneumonia. Typical signs of pneumonia include: * Decreased chest wall movement on the affected side * Decreased percusion sounds * Bronchial breath sounds * Increased fremitus (fremitus is the condition where consolidation increases the passage of sound through the chest) * Whispering pectoriloquy

PZ106 ANZCA Version [Jul06] Q136, [Apr07] A 25-year-old male presents for ECT (electroconvulsive shock therapy) at a free-standing facility. He has a life-threatening depressive illness that has not responded adequately to medication, however he is still taking tranylcypramine (Parnate). You should A. cancel the procedure, cease tranylcypramine and perform the ECT in 2 weeks B. proceed with the ECT, but induce with midazolam and remifentanil C. proceed with the ECT, but pre-treat with esmolol D. proceed with the ECT with caution, but with your usual drugs E. transfer the patient to a tertiary centre for their ECT

ANSWER D The problem with ECT is the vagal stimulus followed by an overwheling sympathetic surge. -significant risk of a hypertensive crisis with MAOI -avoid sympathomimetic drugs and stimulating sympathetic NS -discontinuing MAO inhibitors at least 2 weeks prior to elective surgery is no longer recommended -with the expection of trachlcypromine (irreversible enzyme inhibition, 2 weeks to regernate new enzymes) However, this is an emergency and should go ahead with precautions -avoid pethidine : hyperthermia, seziures and coma -exaggerated response to vasopressors and sympatheometics -consider use of esmolol to blunt sympathetic surge but this may worsen brady -avoid ketamine and pancuronium as they enhance the sympathetic response

Which is not a feature of tricyclic anti-depressant (?amitriptyline) overdosage: A. Cardiac arrhythmias B. Convulsions C. Postural Hypotension D. Metabolic acidosis E. Coma

ANSWER D The toxic manifestation of the tricyclic antidepressants result primarily from their anticholinergic actions and their ability to block the reuptake of neurotransmitters such as norepinephrine. * CNS: Early manifestations include fever, agitation and pupillary dilatation. Advanced cases are characterised by delerium, coma, and generalised seizures. The neurotoxic manifestations are attributed to anti-cholinergic effects * Cardiovascular: Early signs include tachycardia and hypertension, which are attributed to blockade of norepinephrine reuptake. Depletion of norepinephrine in nerve terminals leads to postural and supine hypotension. Cardiac conduction abnormalitis (prolonged QRS interval), arrhythmias and a decreased cardiac output are seen in advanced cases. No mention of metabolic acidosis.

AZ15 Increased alveolar-arterial oxygen gradient can be caused by: A. Endotracheal intubation B. Decreased cardiac output C. Second gas effect D. Atelectasis

ANSWER D The two main factors that affect A-a gradient are: * Shunt --> increased A-a gradient * PAO2 - higher PAO2 (due to higher FiO2) results in a high A-a gradient for any shunt The Cardiac output story: decreased CO - decreases mixed venous O2 so shunted blood has lower PaO2, BUT also reduces the shunt fraction, so that the NET effect is no change in A-a gradient

A patient has a history of polyuria and has an elevated serum calcium, normal parathyroid hormone level and an elevated angiotensin converting enzyme level. The most likely diagnosis is: A. milk-alkali syndrome B. primary hyperparathyroidism C. malignancy D. sarcoidosis E. multiple myeloma

ANSWER D There are three diseases which elevate SACE 1. sarcoidosis, 2. Gaucher disease, 3. leprosy Physiological ACE overproduction in 1. childhood growth, 2. hyperthyroidism, 3. diabetes mellitus with retinopathy, 4. hepatitis and liver cirrhosis undergoing active liver regeneration, and, 5. possibly HIV disease.

Which condition is not associated with a high risk of thromboembolism post-operatively A. protein C deficiency B. protein S deficiency C. antithrombin III deficiency D. Thrombocytopenia E. lupus anticoagulant

ANSWER D There is an increased risk of thrombosis with: * Protein C deficiency * Protein S deficiency * Antithrombin III deficiency * Lupus anticoagulant and/or anticardiolipin antibody * Factor V Leiden mutation * Thrombocytosis * Polycythemia

ANZCA Version [Apr 07] A 60y.o. man with a 45 pack year history of smoking and heavy drinking presents with stridor. He has had a hoarse voice for about 4 months. He has been waking at night for a week with difficulty breathing relieved by sitting up. He is sitting upright in bed with moderate inspiratory stridor. His SaO2 on air is 95%. The next step in his management should be A. awake fibreoptic intubation B. CT scan of the neck C. examination under anaesthesia following gaseous induction D. nasenoscopy under topical anaesthesia E. tracheostomy under LA

ANSWER D This patient has upper airway obstruction -currently stable -but likely to progress to complete obstruction Options to secure airway include 1. ETT (Inhalational, AFOI) 2. Tracheostomy (LA or GA) This will depend on evaluation, diagnosis, and level of skill and speed. Fibre-optic nasendoscopy must be performed by the ENT surgeon in the out-patient clinic or on the ward. It may require no local anaesthesia or simple preparation of the nose with lidocaine and epinephrine. A CT scan may help to locate the tumor and aid tracheostomy placement.

In patients undergoing thoracotomy, techniques which reduce the incidence of intraoperative atrial fibrillation include A. hyperventilation B. pre-operative loading with digoxin C. rocuronium, rather than pancuronium D. thoracic epidural bupivacaine E. thoracic epidural morphine

ANSWER D Thoracic Epidural Bupivacaine Attenuates Supraventricular Tachyarrhythmias After Pulmonary Resection (Anesth Analg 2001;93:253-9) This reference only compares epidural bupivacaine and epidural morphine, no mention of any of the other options however.

child for grommets with clear runny nose, coryza. dry cough, clear chest, otherwise well, no fever. management: A. delay 2 weeks B. delay 1 week C. proceed with ETT D. proceed with face mask E.

ANSWER D Traditionally, children who present for elective surgery with RTI are postoned at least until asymptomatic Paedatric Anaesthesia 2001 11:29-40 Clinical predictors of anaesthetic complications in children with respiratory tract infections -Prospective case control study -Highlighted 8 Variables 1. Airway management 2. Parent states the child has a 'cold' 3. Child has nasal congestion 4. Child Snores 5. Child is a passive smoker 6. Induction agent used 7. Child produces sputum 8. Reversal agent used RTI in preceding 6 weeks did not increase probability of complications Risk factors for airway complications during general anaesthesia in paediatric patients Pediatric Anesthesia Volume 12, Issue 9, pages 762-769, November 2002 3 Independent Risk Factors 1. Age <6 2. RTI 3. Airway manipulation FM>LMA>ETT Anesthesia for the Child with an Upper Respiratory Tract Infection: Still a Dilemma? Anesthesia & Analgesia Issue: Volume 100(1), January 2005, pp 59-65

During controlled ventilation of a neonate prior to repair of a truncus arteriosus which of the following is MOST likely to be deleterious? A. controlled hypoventilation B. ventilation with air C. ventilation with an FiO2 of 0.18 D. ventilation with nitric oxide E. ventilation with PEEP of 3 cmH2O

ANSWER D Truncus arteriosis results from failure of division of pulmonary artery from the aorta -resulting in too much blood flow to the lungs Anaesthetic Management -involves maintaining a high PVR to reduce the gradient across the truncus -minimise further pulmonary shunting -PVR/SVR ratio 1. Minimise SVR -heart failure medication -mainly diuretics 2. Maximise PVR -hypoventilation -hypoxia -PEEP

36. 4 yr old presents for elective surgery, otherwise fit healthy, murmur at left sternal edge on auscultation heard in systole and diastole, disappears on lying down. Most likely cause: A. HOCM B. VSD C. PDA D. venous hum E. ASD

ANSWER D Venous Hum - systolic + diastolic components, not really a murmur. Benign medical condition where 20% of the blood flow travels to the brain and back to the heart. Due to the large amount of blood it can move quite fast causing the vein walls to vibrate which can create a humming noise to be heard by the patient.The murmur disappears when the patient is in the supine position.

Von Hippel-Lindau disease is associated with A. increased risk of malignant hyperthermia B. meningiomas C. peripheral neuropathy D. pheochromocytomas E. poor dentition

ANSWER D Von Hippel-Lindau disease (VHL)? von Hippel-Lindau disease (VHL) is a rare, genetic multi-system disorder characterized by the abnormal growth of tumors in certain parts of the body (angiomatosis). The tumors of the central nervous system (CNS) are benign and are comprised of a nest of blood vessels and are called hemangioblastomas (or angiomas in the eye). Hemangioblastomas may develop in the brain, the retina of the eyes, and other areas of the nervous system. Other types of tumors develop in the adrenal glands, the kidneys, or the pancreas. Symptoms of VHL vary among patients and depend on the size and location of the tumors. Symptoms may include headaches, problems with balance and walking, dizziness, weakness of the limbs, vision problems, and high blood pressure. Cysts (fluid-filled sacs) and/or tumors (benign or cancerous) may develop around the hemangioblastomas and cause the symptoms listed above. Individuals with VHL are also at a higher risk than normal for certain types of cancer, especially kidney cancer. Inheritance * Familial * Autosomal dominant Features * Retinal angiomas * Haemangioblastomas * Cerebellar and visceral tumours (usually benign but can cause pressure effects) Associations An increased incidence of * Phaeochromocytoma - apparently 20% * Renal cysts * Renal cell carcinoma Anaesthesia * Treat hypertension occurring with phaeochromocytoma * Haemangioblastoma of spinal cord may limit use of spinal although epidural has been used for LSCS * Exagerrated hypertension with surgical stimulation or laryngoscopy o Treat with β blockers and/or SNP

The use of Xenon in anaesthesia is limited because it A. has a blood/gas solubility coefficient similar to that of ether B. is a myocardial depressant C. is less potent than nitrous oxide D. is expensive to extract E. lacks analgesic properties

ANSWER D Xenon: A little more detail on the drug, just to tempt us! Despite the high cost of xenon, xenon anaesthesia will become possible at a reasonable cost. The hourly uptake of xenon decreases rapidly and with full recycling of the gas during closed circuit anaesthesia, xenon anaesthesia will be economically viable. * Physical properties o Colourless, odourless, tasteless. o Monatomic gas, atomic number = 54, molecular weight = 131,3 o Gas under normal temperature and pressure o 9 stable isotopes. o Freezing point -111,9oC, Boiling point -108,1oC. o 4 times more dense than air. o Nonflammable and will not support combustion. o Diffuses freely through rubber and silicone components. o Manufactured by separation from liquid air, often as a by product of steel making. * Anaesthetic agent o First used in 1951 by Cullen on an 81yr old man having an orchidectomy.. o Very close to the 'ideal agent' o The very large electron shell of xenon ca be distorted and polarised by nearby molecules, creating a dipole. o Xenon inhibits the plasma membrane Ca2+ pump, altering excitability. It inhibits the nociceptive responsiveness of spinal dorsal horn neurons. o MAC = 71%. (With more widespread usage the Russians have noticed the MAC to be closer to 60%) o Minimal haemodynamic effects. o Lowest blood/gas partition coefficient = 0,115 of currently available inhalational agents. o Low oil/water partition co-efficient of 20. o Rapid induction and eduction regardless of duration of administration o Progressively decreasing xenon requirements during closed circuit anaesthesia makes it an economic viability. o 4 stages of anaesthesia noted with 70% Xenon/ 30% oxygen. i. Whole body paraesthesia & hypo-algesia. ii. Euphoria & increased psychomotor activity. iii. Analgesia with partial amnesia (after 3-4min). iv. Surgical anaesthesia with a degree of muscle relaxation. * Equivalent analgesia when compared with equipotent doses of N2O * The analgesia produced by both gases is not reversible by naloxone. * The analgesic action of nitrous oxide involves the descending inhibitory system from the brainstem. Xenon probably has a direct suppressive action on spinal dorsal horn neurons * No occupational/ environmental disadvantages. * Specific effects on the body o Respiratory + Central depression causes a decrease in respiratory rate with a compensatory increase in tidal volume and can progress to apnoea + Higher density and viscosity (compared with oxygen, air and N2O) theoretically makes xenon more likely to increase airway resistance. Clinically the airway resistance is slightly less than that seen with N2O and it can be used safely in lung disease + Diffusion hypoxia is very mild as the blood/gas partition of Nitrogen (0.014) is only almost 10 times less than that of Xenon (0.115) as opposed to the almost 40 times less than Nitrous Oxide (0.47) o Cardiovascular + No inhibitory effects on cardiac ion channels i.e. calcium, sodium and inward potassium channels. + No significant change in contractility, blood pressure and systemic resistance. + No sensitisation of the myocardium to adrenaline + May attenuate the myocardial depressant effects of isoflurane. + In an animal study, xenon anaesthesia produced the highest regional blood flow to brain, liver, kidneys and GIT. The control groups were 1% halothane in Nitrous oxide and thiopentone with fentanyl. o Central nervous system + Xenon does not alter regional cerebral blood flow and autoregulation (Fink H, Blobner M, Bogdanski R, et al. Effects of xenon on cerebral blood flow and autoregulation: an experimental study in pigs. Br J Anaesth 2000; 84:221-225) + Inhaled xenon when used as a tracer for the measurement of cerebral blood flow increases cerebral blood flow, increases intracranial pressure and decreases cerebral perfusion pressure in acute head injury patients. This is not associated with cerebral oligaemia or ischaemia. This increase in cerebral blood flow is reversed by mild hyperventilation and so might have been a result of poorly controlled physiological variables. Hypercarbic induced vasodilatation induced by the sedative effects of the inhaled xenon, and the stress effect of breathing a dense gas mixture. + At present its use in neurosurgery is controversial o Renal + No data available. o Endocrine/neurohumoral + Attenuates surgical stress due to analgesia. Does not have any short or long term cortisol suppression effects. o Toxicity + Platelet aggregation potentiated at 2atm (relevant to deep-sea divers). + No reported haematological toxicity. + Halted Cell Division + Mitosis is blocked at the metaphase-anaphase stage. This is reversible on withdrawing xenon and administering calcium to the cell. + The most likely site for inhibition is the enzyme complex called Ca MK II o Malignant hyperthermia + Seems not to trigger malignant hyperthermia. o Metabolism and elimination + Unlikely to be involved in any biochemical events in the body. + Eliminated via the lungs. Why Xenon is going to become a potential volatile anaesthetic. * o Decreasing manufacturing costs. + All new liquid air production plants have xenon extraction apparatus fitted + Loss of xenon from the atmosphere from satellite engines will cease from next year + SASOL has completed a very large production facility in South Africa o Use in a fully closed breathing system + A computer controlled closed circuit machine is now commercially available - The Draeger/Physio partnership Physioflex machine + The ability of xenon to alter the speed of sound allows an reasonably accurate measurement + Measurement is generally difficult as # Xenon diamagnetic and does not absorb infrared radiation # Low reactivity precludes the use of specific fuel cell or electrode-type device # Mass spectrometry is expensive # Rotating vane meters are available to measure gas flow # Xenon causes Pitot tube and the variable orifice flowmeters to under - read # Hot-wired anemometers under-read. o Medical license + Russian Federation in November 1999 + German is expected in 2001 Practical usage 1. Nitrogen must be washed out by giving a high flow of pure oxygen for at least 5 minutes 2. Normal induction and muscle relaxation 3. After intubation connect the patient to an appropriate anaesthesia delivery system 4. The hypnotic concentration of 40-45% is achieved after 1.5min 5. The anaesthetic concentration of 60-70% takes approximately 8 minutes. Summary * Colourless and odourless gas with no irritation to the respiratory tract. Well tolerated with gas induction * Low blood/gas and oil/water partition co-efficients allowing rapid induction and eduction * The central nervous system effects are readily reversibly and there is no stimulant activity. * Produces unconsciousness with analgesia and a degree of muscle relaxation * MAC of 60-70% allows a reasonable inspired oxygen concentration * It does cause respiratory depression, to the point of apnoea. * It is cardiac stable. * Not metabolised in the body and is eliminated rapidly and completely via the lungs. * It is non toxic, has inherent molecular stability and does not form flammable mixtures. * Has no long-term adverse effects with chronic exposure at low dose. * Stable in storage, no interaction with anaesthesia circuits or soda lime. Should not be used with rubber anaesthesia circuits as there is a high loss through the rubber * Expensive - Routine usage will only be possible with a closed circuit delivery system that recycles xenon. Owing to environmental concerns there may be no alternative but to use xenon even if it incurs an increase in cost.

Plasma glucose concentration is approximately A. 32% lower than blood glucose concentration B. 14% lower than blood glucose concentration C. the same as blood glucose concentration D. 14% higher than blood glucose concentration E. 32% higher than blood glucose concentration

ANSWER D a BSL of 5.6-6.1 = pl glucose 6.1-7

A 55 year old male has a gastrointestinal hemorrhage and drops his hematocrit from 45% to 19%. A bleeding arteriovenous malformation is resected, and the patient stabilizes. The clinician calls you to ask what level he should expect the patient's hematocrit to rise to if he gives him 4 units of red blood cells. You say: A. About 23% B. About 25% C. About 27% D. About 31% E. About 35%

ANSWER D absence of bleeding, each unit of red cells should raise the hematocrit approximately 3% in an average sized person (that translates to about a 1 g/dL per unit hemoglobin increase).

A 25yo primipara with an uncomplicated pregnancy presents to delivery suite in labour at term. Her membranes spontaneously rupture soon after, and it is blood-stained. At the same time, a severe foetal bradycardia appears on the CTG. What is the most likely cause of this? A. Placenta accreta B. Placental abruption C. Uterine rupture D. Vasa praevia E. True knot in the umbilical cord

ANSWER D asa praevia (vasa previa AE) is an obstetric complication defined as "fetal vessels crossing or running in close proximity to the inner cervical os. These vessels course within the membranes (unsupported by the umbilical cord or placental tissue) and are at risk of rupture when the supporting membranes rupture. These vessels may be torn at the time of labor, delivery or when the membranes rupture. It has a high fetal mortality because of the bleeding that follows. [2] The bblood lost is foetal not maternal blood hence the high mortality. The classic triad are membrane rupture followed immediately by painless vaginal bleeding and fetal bradycardia.

Which type of aortic dissection is typically managed non-operatively? A. Debakey Type I B. Debakey Type II C. Stanford A D. Stanford B E. Stanford C

ANSWER D eMedicine - Emergent Management of Acute Aortic Dissection Stanford classification "The Stanford classification divides dissections into 2 types, type A and type B. Type A involves the ascending aorta (DeBakey types I and II); type B does not (DeBakey type III). This system helps to delineate treatment. Usually, type A dissections require surgery, while type B dissections may be managed medically under most conditions."

Patient indicated for prophylaxis of infective endocardititis A. amoxicillin orally 2 hours prior B. amoxicillin IV 1 hourly prior C. amoxicillin IV just before incision D. cefazolin IV 1 hour prior

ANSWER D http://www.heart.org/idc/groups/heart-public/@wcm/@hcm/documents/downloadable/ucm_307684.pdf ORAL Amoxillin 2g orally or 50mg/kg Allergic to penicillin: Clindamycin 600mg orally or 20mg/kg OR Cephazolin 2g orally or 50mg/kg OR Azithromycin or Clarithromycin 500mg orally or 15mg/kg INTRAVENOUS Ampicillin 2g IV or 50mg/kg Allergic to penicillin: Clindamycin 600mg IV or 20mg/kg IV OR Cefazolin or Ceftriaxone 1g IV/IM or 50mg/kg IV/IM

New national labelling standards endorsed by ANZCA. What colour should the label on a subcutaneous ketamine infusion be? A. Red B. Blue C. Beige D. Yellow E. Pink

ANSWER D http://www.safetyandquality.gov.au/wp-content/uploads/2012/02/ANZCA-and-Commission-Joint-statement-on-user-applied-labelling-standardisation.pdf

Black bank August 2010 TMP-106 A 75yo male with moderate aortic stenosis (valve area 1.1cm2).. Gets mild dyspnoea on exertion but otherwise asymptomatic. Needs hip replacement. A. Continue with surgery B. Beta block then continue C. Get myocardial perfusion scan D. Postpone surgery awaiting AVR E. Postpone surgery awaiting balloon valvotomy

ANSWER D if the aortic stenosis is symptomatic, elective noncardiac surgery should generally be postponed or cancelled. Such patients require aortic valve replacement before elective but necessary noncardiac surgery Indications for AVR -mean P gradient > 40mmHg -valve area < 1.0 cm2 -symptomatic

MAC of halothane in children compared to adult: A. Higher in neonate B. Lower at 2 months C. Lower at 6 months D. Higher in 2 yr old E. Unchanged <5 years old regardless of age

ANSWER D n general - neonatal MAC = Adult MAC * Prems are less * Peaks at age 1 ~ 1.5 Adult MAC * Falls back after this to puberty and adult values MAC is higher in neonates than 40 year olds.

Patient presents with carcinoid syndrome and developes hypotension intraoperatively. Best drug to treat it is: A. Noradrenaline B. Adrenaline C. Metaraminol D. Octreotide E. Ephedrine

ANSWER D the occurrence of intraoperative carcinoid crisis manifesting as bronchospasm or hypotension is treated with IV octreotide 100-200 mcg...Stoelting...drstitch

AZ29 ANZCA version [2001-Aug] Q26, [2002-Mar] Q39 (Similar question reported in [Sep90] [Aug91] [Mar92] [Aug92] [Apr96] [Aug96] [Apr97]) When using low-flow circle absorber techniques, the uptake of nitrous oxide must be considered. In a healthy 70 kg adult the expected uptake of nitrous oxide, with a 70% inspired concentration, after 1.5 hours would be about A. 1000 ml.min-1 B. 500 ml.min-1 C. 250 ml.min-1 D. 100 ml.min-1 E. <50 ml.min-1

ANSWER D uptake of N2O (in mls/min) is equal to 1000 divided by the square root of the time elapsed (in mins) * the time elapsed is 100 minutes. * the square root of this is 10 * 1000 divided by 10 is 100 so the N2O uptake at 100 mins into the case would be about 100mls/min.

Isoflurane is administered in a hyperbaric chamber at 3 atmospheres absolute pressure using a variable bypass vaporizer. At a given dial setting and constant fresh gas flow, vapour will be produced at: A. the indicated vapour concentration B. three times the indicated vapour concentration C. one third the partial pressure obtained at 1 atmosphere D. the same partial pressure as is obtained at 1 atmosphere E. three times the partial pressure obtained at 1 atmosphere

ANSWER D vaporisers use saturated vapour pressure, which is a function of temperature and not ambient pressure. The concentration delivered is inversely proportional to the ambient pressure. Concentration of agent = SVP/Atmospheric pressure. For example at altitude and ambient P of 380mmHg, a vaporiser calibrated to deliver 1% at sea level will therefore deliver 2% (or in the case of iso SVP becomes 2/3 of 1 atmosphere), which is still 7.6mmHg (2% of 380mmHg). Thus although the output of the vapouriser in volume percent increases, the partial pressure is unchanged and it is this that is responsible for the clinical effect. Thus clinically at altitude vaporisers can be used as normal.

How long prior to a spinal anaesthetic should dabigatran be ceased? A. 4 hours B. 12 hours C. 1 day D. 7 days E. 14 days

ANSWER D • Dabigatran - direct thrombin inhibitor. Renally Excreted. Higher risk of bleeding: age >75, weight <50kg, CrCl 30-50ml/min (don't use <30). Check APTT as trough should be 1.5x normal (good). Can reverse with Novo7 in emerg. Cease 48hrs pre surg 4-5days high risk (eg neuro) if normal kidneys, 4-5 if abnormal: 72 hrs to 4-5 days.

The most sensitive monitor for air embolism is A. Respiratory pattern B. Precordial /?oesophageal stethoscope C. ECG D. End-tidal pCO2 ("capnography") E. Precordial doppler device F. SpO2 ("pulse oximeter")

ANSWER D TTE -sensitivity 0.2ml/kg -moderately specific -non invasive -not qualitative, difficult placement with high false negatives But TOE is better -Sensitivity of 0.02ml/kg -able to see VAE, microembolii in pulmonary capillaries and PAE -most sensitive and semi-quantitative -requires continuous observation, potential glottic injury, expensive, operator dependent Sensitivity TOE > ETN2 >doppler > ETCO2 Specificity ETN2 > TOE > Doppler >ETCO2 > PA Quantitative PA

BP measurement - overestimates with: A. big (wide) cuff B. skinny arm C. severely peripherally vasoconstricted D. atherosclerosis (it was arteriosclerosis) E. slow cuff deflation

ANSWER D Overestimates 1. calcified non compressible arteries 2. inappropriately small cuff 3. Peripheral oedema (sound travesl much better though water than tissue) 4. Cuff too loose Underestimate 1. Vasoconstricted 2. Slow cuff INflation : venous congestion leading to low systolic and high diastolic (narrow pulse pressure) Other conditions that will OFTEN render inaccurate measurements are; -Certain heart diseases -Cardiac arrhythmia -Poor circulation -Arterial sclerosis -Preeclampsia -Pulsus alternans -Pulsus paradoxus

At what valve area do you begin to get symptoms, at rest, with mitral stenosis? A. 4.5 cm2 B. 3.5 cm2 C. 2.5 cm2 D. 1.5 cm2 E. 1.0 cm2

ANSWER D - 1.5cm^2 The challenge of valvular heart disease. Cleveland Clinic Journal Of Medicine, Volume 71, Number 6, June 2004 When is it time to operate? "Mitral stenosis is most commonly caused by damage to the mitral valve from rheumatic fever, after which there typically is a long period of asymptomatic progressive valve narrowing. Symptoms at rest are rare until the mitral valve area is less than 1.5 cm2"

Precautions when providing anaesthesia for surgery using a Nd-YAG laser should include all of the following EXCEPT A. avoidance of nitrous oxide B. scavenging of smoke particles because they may contain viral DNA C. awareness that gas embolism is possible from cooling gas down laser conduit D. use of sunglasses to protect eyes E. avoidance of an FiO2 above 0.4

ANSWER D : NdYAG requires special lens to protect retina from direct laser trauma. LASER HAZARDS include 1. atmospheric contamination 2. perforation of vessel or structure 3. embolism 4. inappropriate energy Atmospheric Contamination -vaporization of tissue causes plume of smoke and fine particles (0.3um) which are efficiently transported to alveoli -The laser plume may cause headache, tearing, nausesa, interstitial pneumonia, emphysema, mutogenic, teratogenic or a vector viral infection. -Viral DNA has been detected in plume of warts and condylomas -No HIV RNA has been detected, but HIV DNA (from host DNA) has been detected -No cancer cells detected -CO2 lasers produced more smoke than NdYAG Tissue or vessel perforation -misdirected laser may perforate any viscus or large blood vessel -vessels>5mm are not coagulable by laser Embolism -NdYAG has been associated with venous gas embolism, as gas coolant is used to cool tip can enter through perforated vessels (similar to TURP syndrome), -Reported in uterine, tracheal, laparoscopic and endoscopic procedures -CO2 gas coolant is prefered Inapproprate laser transfer -eye protection -ignition to tissue, endotracheal tube, drape Eye protection -requires special glasses/googles specific for the specific laser wavelength in use. Using the wrong filter provided no protection. Must wraparound to protect from reflected light. -CO2 laser : any clear glass or plastic, they are opaque for infrared -NdYAG laser : green-tinted lens -Argon or kypton : amber-orange lens Ignition -avoid nitrous as it support combustion -aim FiO2<0.4 as O2 supports combustion Other strategies to avoid airway fire 1. reduction in flammability of ETT 2. removal of flammable materials using Metallic Jet Insufflation or intermittant extubation 3. reduction in FiO2 PVC ETT -PVC is transparent and immune to NdYAG and laser light -but thin coating of blood or mucus can absorb energy and cause ETT ignition Flammablity PVC>red rubber >silicon ETT

AZ03 [Jul07] The BEST indication of a difficult intubation in morbid obesity: A. Mallampatti Score B. Neck circumference C. Limited neck movement D. TMD E. Body weight

ANSWER D, neck circumference From Morbid Obesity and Tracheal Intubation (Anesth Analg 2002) * Factors looked at included: "height, weight, neck circumference, width of mouth opening, sternomental distance, thyromental distance and Mallampati score" * "Logistic regression identified neck circumference as the best single predictor of problematic intubation. Mallampati score inclusion did not further improve the model in our limited study with only 12 problematic intubations. In patients with a large neck, the view during direct laryngoscopy was poorer."

In an arterial line system A. Overdamping exaggerates mean B. Underdamping increases mean C. Underdamping underestimates systolic D. ? multiple damping coefficients in an optimal system E. Compliant tubing is good

ANSWER D? Damping of the pressure waveform due to poor positioning of the cannula, or the use of overly compliant tubing, underestimates systolic pressure and overestimates diastolic pressure. The mean pressure is still reasonably accurate.

When a circle absorption system is in use with an intubated 70kg adult patient, rebreathing of expired gas becomes practically negligible if A. a fresh gas flow rate of > 9L/min is used B. intermittent PPV is used, rather than spontaneous ventilation C. the carbon dioxide absorbent is fresh and has an intergranular space of at least one litre D. the minute volume is > 9L E. the FGF enters the circle circuit on the absorber side of the inspiratory valve

ANSWER D??

20 kilogram child suffered 15% full thickness burns 6 hours ago. Optimum crystalloid fluid volume resuscitation for the first hour is A. 160 ml B. 260 ml C. 360 ml D. 460 ml E. 660 ml

ANSWER E

38yo obese female with DM, other comorbidities, undergoing LSCS with spinal anaesthetic with a 27G whitacre needle, having this and that, blah, blah. then two days later complains of numbness on a small patch on lateral aspect of mid-thigh. On full neuro exam - no other signs/symptoms. A. conus medullaris injury B. L2 nerve root compression C. L3 root lesion D. L4 root lesion E. meralgia paraesthetica

ANSWER E

A 30-year-old man presents to the Emergency Department following a high speed motor vehicle accident. He has a Glasgow Coma Score of 7 and arrives with a cervical collar in situ and an 18 gauge intravenous cannula in his right hand. You first priority in managing this patient would be to A. insert a large bore intravenous cannula B. perform a CAT scan of the brain C. perform a cervical spine X-ray D. perform a plain chest and pelvic X-ray E. secure the airway with an endotracheal tube

ANSWER E

A 4 year old child with VSD (repaired when 2 years old) for dental surgery. What antibiotic prophylaxis do the guidelines recommend? A. Amoxycillyn orally B. Amoxycillin IV C. Cephazolin IV D. Amoxycillin / gentamicin E. No antibiotics required

ANSWER E

A 50 year old male with sleep apnoea presents for a laparoscopic cholecystectomy. Factors which may worsen this patient's tendency to airway obstruction post-operatively include A. multiple attempts at intubation B. supine position C. narcotic analgesia D. presence of a nasogastric tube E. All of the above

ANSWER E

A male patient has a haemoglobin of 8g/dL and reticulocyte count 10%. The most likely diagnosis is A. Untreated pernicious anaemia B. Aplastic anaemia C. Acute leukaemia D. Anaemia of chronic disease E. Hereditary spherocytosis

ANSWER E

A recently published large Australian multicentre trial of patients undergoing major abdominal surgery (The MASTER Trial) concluded that the use of epidural anaesthesia (with general anaesthesia) and postoperative epidural analgesia, compared with general anesthesia alone, was associated with a decrease in: A. mortality at 30 days B. both mortality at 30 days and adverse cardiovascular events C. both mortality at 30 days and respiratory failure D. adverse cardiovascular events E. respiratory failure

ANSWER E

A woman has an epidural placed for forceps delivery after a prolonged second stage of labour. The next day she has a right foot drop, and numbness over the anterior part of her lower leg and the dorsal surface of her right foot. The most likely cause is A. L4 nerve root lesion from trauma during epidural placement B. L5 nerve root lesion from trauma during epidural placement C. L5 nerve root lesion from an acute disc protrusion D. right common peroneal nerve lesion from compression by lithotomy stirrups E. right lumbar plexus lesion from compression by the fetal head

ANSWER E

An 86yo with severe dementia and multiple medical problems.. Surgeons want to operate for faecal peritonitis/bowel perforation, and believe he will die without the surgery. Your decision NOT proceed with surgery is supported by which ethical principle? A. Dignity B. Competence C. Non-maleficience D. Paternalism E. Futility

ANSWER E

An elderly gentleman on warfarin has suffered a subdural haematoma. His INR on admission was 4.5. The resident in Ed has already given him 2.5mg of Vit K. To reverse his coagulopathy prior to urgent surgery you should give him A. Factor VIIa B. FFP C. Cryoprecipitate D. Prothrominex E. Prothrombinex and FFP

ANSWER E

Anaemia in chronic renal failure is characteristically A. due to haemolysis in the renal vascular bed B. normochromic and microcytic C. due to defective haemoglobin synthesis D. responsive to ion and folate therapy E. associated with increased 2,3-DPG levels in blood cells

ANSWER E

Apnoeic oxygenation in obese patient can be increased by A. Sniffing position B. Prone C. Supine D. Lateral E. Head up

ANSWER E

Best single predictor of difficult intubation in obese patient? A. Mallampati score B. Interincisor distance C. Severe OSA D. BMI E. Neck circumference >44cm

ANSWER E

Called to emergency department to review a 20 year old male punched in throat at a party. Some haemoptysis / hoarse / soft voice. Next step in management: A. CT to rule out thyroid cartilage fracture B. XR to rule out fractured hyoid C. Rapid sequence induction / laryngoscopy / intubation D. Awake fibreoptic intubation E. Nasendoscopy by ENT in emergency department

ANSWER E

Cause of generalised convulsions 20 minutes postop: A. Pre-existing Grand mal epilepsy B. Local anaesthesia with lignocaine C. Enflurane D. Intraoperative use of vasoconstrictors E. Post intracranial surgery

ANSWER E

Ciliary ganglion A sympathetic from inferior cervical ganglion B located inferiorly within orbit C may be damaged during a peribulbar block D preganglionic parasympathetic supply from the supra trochlear nerve E preganglionic parasympathetic originates from the Edinger Westpal nucleus

ANSWER E

During elective major vascular surgery the best way to reduce the risk of acute renal failure is to maintain a normal A. cardiac output B. central venous pressure C. mean arterial blood pressure D. pulmonary capillary wedge pressure E. renal blood flow

ANSWER E

During surgery there is increased secretion of each of the following hormones EXCEPT A. aldosterone B. glucagon C. growth hormone D. thyroid stimulating hormone E. vasopressin

ANSWER E

Each of the following statements regarding the haemodynamic changes during pneumoperitoneum for laparoscopy is true EXCEPT: A. in patients with severe cardiac disease changes are qualitatively similar to those in normal patients B. right atrial pressure is NOT a reliable indicator of cardiac filling C. they are well tolerated by morbidly obese patients D. they are well tolerated in cardiac transplant patients with good ventricular function E. they are well tolerated in patients with low cardiac output secondary to low preload

ANSWER E

Each of the following statements regarding the haemodynamic changes during pneumoperitoneum for laparoscopy is true EXCEPT: A. in patients with severe cardiac disease changes are qualitatively similar to those in normal patients B. right atrial pressure is NOT a reliable indicator of cardiac filling C. they are well tolerated by morbidly obese patients D. they are well tolerated in cardiac transplant patients with good ventricular function E. they are well tolerated in patients with low cardiac output secondary to low preload

ANSWER E

Following a left sided pneumonectomy, a left intercostal drain is placed and connected to an underwater drainage system. In the postoperative period A. A leakage of air is expected from the drain B. The patient should be nursed in the right lateral decubitus position C. The underwater seal drain should be left on continuous free drainage D. The underwater seal drain should be left on continuous free drainage, and connected to wall suction for 5 minutes every hour E. The underwater seal drain should remain clamped and be released for a short period every hour

ANSWER E

Fontan patient having an open appendicectomy. What do you want? A: long I time and PEEP B: long I time C: short I time D: raised ETCO2 E: spontaneous ventilation

ANSWER E

In an acute malignant hyperthermia episode A. the serum creatine kinase level peaks within one hour B. the peak serum creatine kinase level is a good indicator of the amount of muscle involved C. elevated creatine kinase levels contribute to acute renal failure D. the serum myoglobin level does NOT peak for at LEAST 24 hours E. muscle rigidity occurs in 75% of cases

ANSWER E

In clinically used doses levosimendan, a novel inotrope used in heart failure, A. acts by increasing levels of intracellular cyclic AMP B. increases myocardial intracellular calcium C. increases myocardial oxygen consumption D. increases the rate of fatal ventricular arrhythmias E. increases the sensitivity of intracellular contractile proteins to calcium

ANSWER E

Incarcerated inguinal hernia in a child with a mild URTI. Most appropriate course of action? a. postpone for two weeks b. continue but with a spinal anaesthetic only c. continue but without using an endotracheal tube d. he requires antibiotics e. continue, but with careful monitoring

ANSWER E

Long stem about an old #NOF patient with aortic stenosis. What is a sign/ investigation/ symptom that shows the most severity? (ie Which one of these would indicate that the lesion was severe?) A: Thrill in Aortic area B: Murmur in lower left sternal edge C: Murmur radiating to carotids D: History of ischaemic heart disease or coronary artery disease E: history of angina/ syncope

ANSWER E

MR33c ANZCA version [2003-Aug] Q42 The most useful tool for evaluation of the condition of a patient with an acute asthmatic attack and in assessing the response to therapy is A. chest radiography B. arterial blood gas measurement C. measurement of pulsus paradoxes D. observation of the degree of use of accessory muscles of respiration E. measurement of peak expiratory flow or FEV1

ANSWER E

Man in MVA is hypotensive with distended neck veins. Which is not a cause: A. Tension pneumothorax B. Cardiac tamponade C. Myocardial contusion D. Air embolus E. HOCM

ANSWER E

PZ86d Serotonin syndrome has been reported following SSRI coadministration with: A. Gingko B. Garlic C. Ginger D. St John's wort E. Vallerian

ANSWER E

Pain continuing well after the rash of Herpes Zoster has disappeared (Post-Herpetic Neuralgia): A. Has an incidence of approximately 25% B. Is more common after ophthalmic than after spinal nerve involvement C. Is more common in the immunocompromised but is independent of age and gender D. is best treated with high dose opioids E. Has been shown to occur less commonly after acyclovir (Zovirax)

ANSWER E

Patient with subdural hematoma and PPM for ?AV ablation. PPM technician >1 hour away. Surgeon wishes to proceed immediately. Do you? A. Postpone and await a cardiologist review B. Postpone and await arrival of PPM technician C. Postpone and insert a transvenous temporary PM D. Proceed after institution of transcutaneous pacing. E. Proceed with a magnet handy.

ANSWER E

Phosphate-containing bowel preparations for colonoscopy are contra-indicated in frail, elderly patients because of the A. frequent nausea, bloating and abdominal pain B. risk of hypercalcaemia C. risk of hypokalaemia D. risk of hypomagnesaemia E. risk of large fluid shifts

ANSWER E

Post partum sudden collapse, suspected amniotic fluid embolism. The consistent finding is: A. Low C3, C4 B. Increase complement C. Increase tryptase D. Increase histamine? E. petechial rash

ANSWER E

Pulsus paradoxus in constrictive pericarditis: A. Decrease in BP Every second beat B. Decrease in BP In expiration when increase is normal C. Decrease in BP In expiration more than normal subjects D. Decrease in BP In inspiration when increase is normal E. Decrease in BP In inspiration more than normal subjects

ANSWER E

SF34 [Aug92] [Mar93] [Aug96] The epidural dose of local anaesthetic that is required to produce the same mean height of block in a pregnant woman at term, when compared with a non-pregnant woman of the same age requires on average: A. 30% more B. 15% more C. The same D. 15% less E. 30% less

ANSWER E

SF70 ANZCA version [2003-Apr] Q115 The most important characteristic of non-particulate antacids in obstetrics is their A. inability to cross the placenta B. low cost C. pleasant taste D. reduced incidence of nausea E. speed of onset of action

ANSWER E

Significant differences between the LMA-ProSeal™ and the standard laryngeal mask (LMA-Classic™) include all of the following EXCEPT A. a built in bite-block B. a double cuff arrangement C. an improved seal pressure at a given cuff pressure D. an independent oesophageal drain tube E. improved aperture bars to prevent the epiglottis occluding the airway tube

ANSWER E

Signs consistent with cocaine overdose include all of the following except A. arrhythmias B. dysphoria C. Hyperglycemia D. Hyperthermia E. Miosis

ANSWER E

The BEST clinical indicator of SEVERE aortic stenosis is: A. Presence of thrill B. Mean Gradient 30mmHg C. Valve area of 1.2 cm2 D. Slow rising pulse and ESM radiating to carotids E. Shortness of breath

ANSWER E

The MOST correct statement concerning lignocaine toxicity is that A. extremely high concentrations of lignocaine will produce persistent ventricular fibrillation B. hypercarbia increases the convulsive threshold of the drug C. the first sign of lignocaine toxicity is cardiovascular collapse D. the initial treatment of lignocaine induced convulsions is phenytoin E. tonic-clonic convulsions may be preceded by symptoms such as auditory disturbances

ANSWER E

The active metabolite of ketamine is: a. Hydroxyketamine b. Hydroxynorketamine c. Ketamine glucuronide d. Ketamine sulphonamide e. Norketamine

ANSWER E

Thrombocytopaenia occurs in all except A. Hypersplenism B. Massive transfusion C. Disseminated intravascular coagulation D. B12 deficiency E. Iron deficiency anemia

ANSWER E

Typical physiological changes in pregnancy at term, compared to the non-pregnant state include a twenty percent A. Increase in alveolar ventilation B. Increase in tidal volume C. Increase in vital capacity D. Reduction in arterial pH E. Reduction in functional residual capacity

ANSWER E

What is the first sign/symptom seen with an inadvertent total spinal whilst performing caudal anaesthesia in a neonate A. Hypotension B. Bradycardia C. Desaturation D. Tachycardia E. Loss of consciousness

ANSWER E

When instructing ward staff on monitoring for respiratory depression in a patient using PCA (patient controlled analgesia) you would advise that early respiratory depression is best detected by monitoring A. frequency of boluses on PCA machine B. pulse oximetry C. pupil size D. respiratory rate E. sedation scores

ANSWER E

Which of the following does cause a crisis in a patient with sickle cell anaemia? A. Hypothermia B. Respiratory acidosis C. Metabolic acidosis D. Hypotension E. Hypokalaemia

ANSWER E

Which of the following statements regarding anaphylactic and anaphylactoid reactions is FALSE? A. cross-sensitivity between latex and bananas, chestnuts and avocado has been reported B. cross-sensitivity of cephalosporins with penicillin is about 8% C. gelatin solutions used for resuscitation can worsen any reaction D. reactions to neuromuscular blocking agents are more common in females E. vecuronium is more likely to cause an anaphylactoid rather than an anaphylactic reaction

ANSWER E

Which of the following statements regarding pre-operative oral midazolam in children (in appropriate dosage) is INCORRECT? A. Distress at induction is reduced by approximately one half B. Peak effect is at 20 - 30 minutes C. Pre-operative anxiety is reduced D. Post-operative maladaptive behaviour is reduced E. Significantly delays recovery and discharge

ANSWER E

Which two nerves are most reliably blocked in a fascia iliaca block? A. Femoral and obtruator nerve B. Lateral cutaneous and sciatic nerve C. Sciatic and Femoral nerve D. Femoral, Sciatic and Lateral cutaneous E. Femoral and lateral cutan

ANSWER E

With regard to surgical antibiotic prophylaxis, which of the following statements is LEAST correct? A. cephalosporins can usually be safely administered to penicillin allergic patients B. clindamycin can be indicated in penicillin allergic patients C. they should be administered at the time of surgical incision D. the ideal timing of administration is less than 30 minutes prior to surgical incision E. vancomycin should be given routinely in MRSA (methicillin resistant staphylococcus aureus) prevalent areas

ANSWER E

A 4-year-old child with obstructive sleep disorder presenting for tonsillectomy A. is likely to suffer from daytime somnolence B. is unlikely to have a history of snoring C. is suitable for day-case surgery D. has a 40% chance of postoperative vomiting without antiemetic treatment E. is likely to lose 5% of their blood volume during surgery

ANSWER E • A - False . ' in contrast to adults where daytime somnolence is common, this is not typical presenting complaint in children' • B - False. 'Most common symptoms in children with sleep apnoea are nocturnal snoring and noisy breathing' • C - False. • D - True. o Miller says 70% PONV. The presence of blood in the stomach & the need for opioids are a potent stimulus to vomiting, as are swelling and inflammation of the posterior pharynx & uvular. o The incidence of emesis after tonsillectomy can be as great as 60%.' (Cote CJ et al (2001) A Practice of Anesthesia for Infants and Children, 3rd ed, p465) ref Ferrari LR, Donlon JV: Metoclopramide reduces the incidence of vomiting after tonsillectomy in children. Anesth Analg 1992; 75:351-354. • E - ? True o Miller p2187: "Blood loss during tonsillectomy is difficult to estimate and may reach 5% of estimated blood volume. Blood loss should be replaced if this exceeds 10-15% of blood volume." o For a 4 year old: • Weight = 2x(Age + 4) = 2x(4 + 4) = 16kg • Blood volume of child = 80mls/kg = 80x16 = 1280mls • 5% blood volume = 0.05 x 1280 = 64 mls • This blood volume is entirely conceivable given surgical blood loss. o Quote from the Australian Journal of Oto-Laryngology states "Overall intra-operative blood loss was an average of 18mls (range 0-150mls) or 1% of blood volume (range 0-8%)".

A 50-year-old female patient presents with a 12 hour history of feeling unwell and is found to fulfill the criteria for Systemic Inflammatory Response Syndrome. Her blood pressure is 80/45 mmHg, her pulse rate is 90 beats.min1 and her central venous pressure is 12 mmHg. The hypotension is most appropriately managed with A. adrenaline B. dobutamine C. dopamine D. ephedrine E. noradrenaline

ANSWER E * "Norepinephrine raises SVRI, MAP, regional blood flow, oxygen extraction and urine output and is the usual agent of first choice." * "Epinephrine may have undesirable effects on the splanchnic circulation" * "If norepinephrine maintains an adequate MAP but CI remains low, then a separate infusion of dobutamine maylead to improvement."

Intravenous paracetamol has A. a duration of antipyretic effect of 4 hours B. an antipyretic effect within 10 minutes C. an onset of analgesic effect at 30 minutes D. a peak analgesic effect at 30 minutes E. a peak analgesic effect at 60 minutes

ANSWER E * A - False - lasts at least 6 hours * B - False - ↓ fever within 30 minutes * C - False - Onset within 5-10 minutes * D - False - peak effect in one hour * E - True

With regard to non-depolarising muscle relaxants: A. NDMR actions are increased with hyperkalemia B. Vecuronium exclusively (or primarily) undergoes renal elimination C. Mivacurium half-life 10 minutes D. Cisatracurium is metabolised to laudanosine by the liver E. May require larger doses if treated with phenytoin and theophylline

ANSWER E * A - False. (Hypokalaemia prolongs action, not hyperkalaemia) * B - False. Vecuronium undergoes elimination also by the liver (ie non-exclusive elimination) * C - False. (about 2 min) * D - False. Cisatracurium undergoes spontaneous metabolism to produce laudanosine, not organ dependent. * E - True. In Sasada and Smith under "Phenytoin" ... "it appears to increase the dose requirements of all of the non-depolarising relaxants (with the exception of atracurium) by 60-80%. No entry for Theophylline, but under "Aminophylline" says "in high concentrations, the drug will antagonise non-depolarising neuromuscular blockade caused by pancuronium or tubocurarine"

When using NSAIDs (non-steroidal anti-inflammatory drugs) and COX-2 (cyclo-oxygenase 2) inhibitors for postoperative analgesia, A. COX-2 inhibitors are more effective analgesics than NSAIDs B. COX-2 inhibitors have less effect on renal function than NSAIDs C. COX-2 inhibitors impair platelet function D. COX-2 inhibitors trigger aspirin induced respiratory disease with a similar likelihood to NSAIDs E. NSAIDS increase the risk of peri-operative bleeding after some types of surgery

ANSWER E * A - False. COX-2 inhibitors are as effective as NSAIDs for postop pain...NNT's comparable with those for conventional NSAIDs for the treatment of mod-severe acute pain (APMSE 2005 Section 4.2.3)--Anaestheasy Tiger 01:15, 27 Jun 2009 (EDT) * B - False. "Renal and cardiovascular effects of COX-2 inhibitors are similar to conventional NSAIDs." Stoelting, 4th. ed., p.281. * C - False. "These drugs (COX-2 inhibitors) lack effects on platelets at therapeutic doses and may be associated with decreased gastrointestinal side effects in patients with arthritis, compared with nonspecific NSAIDs." and "COX-2 inhibitors have no effect on platelet aggregation, bleeding time or postoperative blood loss." Stoelting, 4th. ed., p.277 & 281. Also "COX-2 inhibitors do not impair platelet function; this leads to reduced perioperative blood loss in comparison with NSAIDs (level II - APMSE 2007 update) * D - False. "A history of asthma exacerbation on exposure to aspirin is a reason to avoid NSAIDs that inhibit COX-1 and COX-2. Nevertheless, the COX-2 inhibitor rofecoxib may be safe in aspirin-sensitive patients." Stoelting, 4th. ed., p.282. * E - True. "Preoperative NSAID therapy significantly increases intraoperative and postoperative blood loss in children undergoing tonsillectomy."Stoelting, 4th. ed., p.281, and "aspirin and some NSAIDs increase the risk of perioperative bleeding after tonsillectomy, except in paediatric patients. (level I (Cochrane Review) - APMSE update 2007)

PL24 ANZCA version [2003-Aug] Q133, [2004-Apr] Q4 The MOST correct statement concerning lignocaine toxicity is that A. extremely high concentrations of lignocaine will produce persistent ventricular fibrillation B. hypercarbia increases the convulsive threshold of the drug C. the first sign of lignocaine toxicity is cardiovascular collapse D. the initial treatment of lignocaine induced convulsions is phenytoin E. tonic-clonic convulsions may be preceded by symptoms such as auditory disturbances

ANSWER E * A - False. Miller, 6th ed. p 594: "Ventricular arrhythmias were rarely seen with lidocaine, mepivacaine, or tetracaine." and "Ventricular arrhythmias and fatal ventricular fibrillation may occur more often after rapid intravenous administration of a large dose of bupivacaine but far less frequently with lidocaine." * B - False. "Hypercarbia can lower seizure threshold by several mechanisms: (1) Increased CBF with increased drug delivery to the CNS; (2) increased conversion of the drug base to the active cation in the presence of decreased intracellular pH; and (3) decreased plasma protein binding, which increases the amount of free drug available for diffusion into the brain." Goldfrank's toxicologic emergencies (Caffeine: Think hypercarbia ie acidosis actually said to decrease seizure threshold: hyperventilation test as used in the past for dx petit-mal and diamox used as adjunct for Rx of intractable seizures by inducing acidaemia. Agree with increased blood flow delivering more drug to brain with hypercapnoea though. E best answer.) * C - False. "Initial symptoms are subjective and include tinnitus, lightheadedness, circumoral numbness, disorientation, confusion, auditory- and visual disturbances and lethargy." Goldfrank's toxicologic emergencies * D - False. "Treatment: supportive, with oxygenation / cardiovascular support as for CPR. Thiopental or diazepam / midazolam may be used for convulsions, although hypotension may be exacerbated." Yentis, 3rd ed., p 321. * E - True. Sequence of: "Toxicity . . . tingling . . . unconsciousness and or convulsions." Yentis, 3rd ed., p 321.

An INCORRECT statement regarding the management of hypocalcaemia is that A. Correcting a respiratory or metabolic alkalosis increases the level of ionised calcium B. Calcium should be administered via a central vein as it is irritant to peripheral veins C. Acidosis will decrease calcium binding to albumin and therefore increase ionised calcium D. When calcium alone is NOT sufficient for control of hypocalcaemia, Vitamin D metabolites can be added E. Calcium chloride has been shown to be superior to calcium gluconate

ANSWER E * A - True o Reducing alkalosis → ↑ H+ → displaces calcium from albumin → ↑ ionised calcium * B - False - although "should be" is true - if type A question then ?E the least correct? o Calcium causes tissue necrosis with extravasation so CVC or large vein preferable * C - True (see A) * D - calcitriol (also calcidiol and dihydrotachysterol) are Vit D metabolites. Calcitriol is the preferred because of its rapid onset (hours) * E - False - just more concentrated o CaCl2 + 5ml ampoule + 1mmol/ml o Ca Gluconate + 10ml ampoule + 0.22mmol/ml

The incidence of an intra-cranial aneurysm is increased by each of the following EXCEPT A. Ehlers-Danlos syndrome B. hypertension C. neurofibromatosis D. positive family history of intra-cranial aneurysms E. type 2 diabetes mellitus

ANSWER E * A. Ehlers Danlos disease - true: o " A number of inherited conditions are associated with increased risk of cerebral aneurysm and SAH. These include autosomal dominant polycystic kidney disease, glucocorticoid-remediable aldosteronism, and Ehler Danlos syndrome." (Uptodate) * B. hypertension - true: o "Hypertension is a major risk factor for SAH" * C. neurofibromatosis - true: o "Several retrospective series have noted abnormal cerebral vasculature (eg, moyamoya syndrome, intracranial aneurysm) in 2 to 6 percent of children with NF1 who underwent neuroimaging" (Uptodate) * D. positive family history of intra-cranial aneurysms - true: o "A family history of SAH also increases the risk of SAH in individuals without one of these conditions. As an example, one case-control study found that patients with a family history of SAH had an odds ratio of 4.0 (95% CI 2.0-8.0) for SAH compared with controls" (Uptodate) * E. type 2 diabetes mellitus - false and answer to choose Multiple conditions have been associated with cerebral aneurysms; they include the following: * Autosomal dominant inherited polycystic kidney disease * Fibromuscular dysplasia * Arteriovenous malformations * Osler-Weber-Rendu syndrome * Coarctation of the aorta * Other vascular anomalies * Moyamoya syndrome * Marfan syndrome * Ehlers-Danlos syndrome, type IV * Other collagen type III disorders * Pseudoxanthoma elasticum * Alpha1-antitrypsin deficiency * Systemic lupus erythematosus * Sickle cell anemia * Bacterial endocarditis * Fungal infections * Neurofibromatosis type 1 * Tuberous sclerosis

Norpethidine toxicity A. Is only seen if renal function is abnormal B. Develops because the half life of norpethidine is twice that of pethidine C. Is not seen unless treatment is prolonged D. only seen if dose of pethidine in excess of 1.2g/day E. May manifest early as anxiety and mood changes

ANSWER E * A. Is only seen if renal function is abnormal - false * B. Develops because the half life of norpethidine is twice that of pethidine - maybe: "Although the elimination half-life of pethidine varies from 3 to 6 h, that of norpethidine is around 17 h in healthy patients, but it may be much longer if renal function is compromised." * C. Is not seen unless treatment is prolonged - false: "Numerous cases of seizures have been reported in patients receiving pethidine over a period of days or in some cases only hours." * D. only seen if dose of pethidine in excess of 1.2g/day - false * E. May manifest early as anxiety and mood changes - true: "toxic side effects such as seizures, agitation, irritability, tremors, twitches and myoclonus"

Which one of the following statements concerning Tramadol is FALSE? A. It has an active metabolite B. It inhibits serotonin and noradrenaline reuptake C. It is LESS likely (at normal doses) to cause respiratory depression than other opioid agonists D. It is metabolised in the liver and excreted in the kidneys E. It structurally resembles codeine

ANSWER E * A. It has an active metabolite TRUE, THEREFORE NOT THE ANSWER = O desmethyltramadol * B. It inhibits serotonin and noradrenaline reuptake TRUE AND THEREFORE NOT THE ANSWER [2] * C. It is less likely to (at normal doses) to cause respiratory depression than other opioid agonists TRUE AND THEREFORE NOT THE ANSWER. APMS third edition 2010 page 63. * D. It is metabolised in the liver and excreted in the kidneys TRUE AND THEREFORE NOT THE ANSWER [3] * E. It structurally resembles codeine FALSE AND THEREFORE THE ANSWER.

A 32-year-old patient is admitted with early acute liver failure (unrelated to paracetamol ingestion). Management should include: A. avoidance of intubation to monitor encephalopathic progress B. consideration for liver transplant if the INR (international normalised ratio) is over 3 C. limited use of sodium containing fluids during acute resuscitation D. N-acetyl-cysteine as a generall hepatoprotective agent E. prophylactic antibiotics

ANSWER E * A. avoidance of intubation to monitor encephalopathic progress - partly true, partly false: If they need intubation, intubate them. You do however want to monitor the patient's encephalopathy, which is a bit harder when intubated and sedated. * B. consideration for liver transplant if the INR (international normalised ratio) is over 3 - false * C. limited use of sodium containing fluids during acute resuscitation - can be bad: "Patients with ALF often require aggressive fluid resuscitation at presentation to correct commonly associated hypovolaemia. There is a tendency to avoid excessive sodium in patients with chronic liver disease, and this is often translated into the care of patients with ALF. The overzealous use of 5% dextrose in the early stages of resuscitation results in severe hyponatraemia." * D. N-acetyl-cysteine as a generall hepatoprotective agent - false * E. prophylactic antibiotics - true: "Prophylactic antimicrobials with broad-spectrum coverage of gram-positive and gram-negative activity including an anti-fungal (e.g. piperacillin with tazobactam and fluconazole) should be administered on admission, as this halves the incidence of infective episodes when compared with commencement at the time of suspected infection. However, this benefit must be balanced against the risk of developing multi-resistant pathogens." (Continuing Education in Anaesthesia, Critical Care & Pain | Volume 4 Number 2 2004, p40f)

Transient Neurological Syndrome A. comprises pain localised to the back B. diagnosis is confirmed by typical findings on neurological examination C. is associated with consistent abnormalities on magnetic resonance imaging and electrophysiological studies (EPS) D. is associated with long term deficits in 5% of cases E. may occur with lignocaine, bupivacaine, prilocaine and procaine

ANSWER E * A. comprises pain localised to the back - false: "Follow-up of patients who received uncomplicated spinal anesthesia revealed that some of them developed pain in the lower extremities—transient neurologic symptoms (TNS)" (A & A June 2005 vol. 100 no. 6 1811-1816) * B. diagnosis is confirmed by typical findings on neurological examination - false: "In contrast to the lower extremity weakness and bowel and bladder dysfunction observed with cauda equina syndrome (8), neurologic examination, magnetic resonance imaging, and electropathological testing show no abnormalities in patients with TNS" * C. is associated with consistent abnormalities on magnetic resonance imaging and electrophysiological studies (EPS) - false: See B * D. is associated with long term deficits in 5% of cases - false: "There was no evidence that this painful condition was associated with any neurologic pathology; in all patients, the symptoms disappeared spontaneously by the 10th postoperative day." * E. may occur with lignocaine, bupivacaine, prilocaine and procaine - true: "The relative risk for developing TNS after spinal anesthesia with lidocaine was higher than with other local anesthetics (bupivacaine, prilocaine, procaine, and mepivacaine)"

The peri-operative use of beta-adrenergic antagonists A. exerts a cardioprotective effect entirely by reduction of heart rates B. is best started intra-operatively C. is contraindicated in patients with chronic airways limitation D. is contraindicated in patients receiving angiotensin converting enzyme inhibiting drugs (ACE inhibitors) E. is safe in patients with moderately impaired ventricular function

ANSWER E * A. exerts a cardioprotective effect entirely by reduction of heart rates - probably false * B. Is best started intra-operatively - false * C. Is contraindicated in patients with chronic airways limitation - maybe false: definitely false if there is no reversible component to the COAD, but if there is, beta-2 receptors cause bronchodilation and their blockage is relatively contraindicated * D. Is contraindicated in patients receiving ACEI - false * E. Is safe in patients with moderately impaired ventricular function - maybe true: bisoprolol and carvedilol are both beta blockers and are used to treat heart failure; however also note that beta-1 receptors increase cardiac contractility. o Beta-blockers and angiotension converting enzyme inhibitors should be used in all patients with a recent or remote history of MI regardless of EF or presence of HF (AHA guidelines)

Acute pancreatitis associated with all EXCEPT: A. ARDS B. Pleural effusions C. Ascites D. Hypocalcaemia E. Hepatocellular failure

ANSWER E * Acute respiratory distress syndrome (ARDS), acute renal failure, cardiac depression, hemorrhage, and hypotensive shock all may be systemic manifestations of acute pancreatitis in its most severe form. * Fat necrosis may cause hypocalcemia. Clinical History * The main presentation of acute pancreatitis is epigastric pain or right upper quadrant pain radiating to the back * Nausea and/or vomiting * Fever * Query the patient about recent surgeries and invasive procedures (ie, endoscopic retrograde cholangiopancreatography) or family history of hypertriglyceridemia. * Patients frequently have a history of previous biliary colic and binge alcohol consumption, the major causes of acute pancreatitis. Physical * Tachycardia * Tachypnea * Hypotension * Fever * Abdominal tenderness, distension, guarding, and rigidity * Mild jaundice * Diminished or absent bowel sounds * Because of contiguous spread of inflammation (effusion) from the pancreas, lung auscultation may reveal basilar rales, especially in the left lung. * Occasionally, in the extremities, muscular spasm may be noted secondary to hypocalcemia. * Severe cases may have a Grey Turner sign (ie, bluish discoloration of the flanks) and Cullen sign (ie, bluish discoloration of the periumbilical area) caused by the retroperitoneal leak of blood from the pancreas in hemorrhagic pancreatitis. Main causes of ascites * Hepatic - cirrhosis, veno-occlusive disease * Cardiac - right ventricular failure, constrictive pericarditis * Renal - nephrotic syndrome, renal failure * Malignancy - ovarian, gastric, colorectal carcinoma * Infection - tuberculosis * Pancreatitis * Lymphatic - congenital anomaly, trauma * Malnutrition * Myxoedema

Ischaemic preconditioning limits myocardial damage during a subsequent ischaemic insult. This process is inhibited by A. alpha-2 agonists B. insulin C. isoflurane D. opioids E. sulfonylureas

ANSWER E * Cardiac preconditioning is thought to be due to action of ATP K+ channels. * These are typically blocked by sulphonylureas, hence these agents would negate preconditioning. * Therefore option E is correct.

Serotonin syndrome A. delays clinical treatment (sic) B. has the specific antidote promethazine C. has signs and symptoms which are difficult to distinguish from neuroleptic malignant syndrome, but the distinction between the two syndromes is unnecessary for clinical management D. is self-limiting E. may be contributed to by pethidine

ANSWER E * No direct antidote * Cyproheptadine and chlorpromazine potentially antagonize serotonin in the CNS, but there are no controlled trials of these agents...chlorpromazine may be contraindicated in neuroleptic malignant syndrome because of its antidopaminergic properties (important as they are hard to distinguish)...Current Critical Care, 2008 * Benzodiazepines generally are considered useful for the serotonin syndrome. They are anticonvulsants, are not associated with serotonin release, and are anxiolytic and sedating. Dantrolene uncouples excitation-contract in skeletal muscles and has been used in malignant hyperthermia, neuroleptic malignant syndrome, and serotonin syndrome. There are case reports of benefit from dantrolene in serotonin syndrome but no controlled trials. * Meperidine-induced serotonin syndrome in a susceptible patient, Br. J. Anaesth., September 2009; 103: 369 - 370. * Tramadol, pethidine, fentani(y)l (and congeners), methadone, dextromethorphan, dextropropoxyphene, pentazocine...all possible

AZ66 ANZCA version [2003-Apr] Q124 The bibliographic database of LEAST relevance to anaesthetists is A. CINAHL B. Cochrane Library C. Embase D. Medline E. PsycINFO

ANSWER E * Psycinfo is specific for psychology. * Embase is an alternative to medline-it is relevant in that to FULLY search the literature you should search medline and embase- any decent meta-analysis will do this as some journals will not be referenced in both - contains many european & foreign language journals(although Medline is more than enough for our MCQ answering it includes BJA,CJA, A and A etc.) * Cochrane is a database of sytematic reviews and there are many others EBM etc. * CINAHL is a database of journals relevant to nursing; it has major anaesthesia journals such as Anesthesiology so should be useful.

The most effective method of deep venous thrombosis (DVT) prophylaxis for a fifty-year-old woman presenting for anterior resection for cancer of the colon would be A. electrical calf stimulation B. Dextran 70 infusion C. graduated compression stockings D. intermittent pneumatic leg compression E. low dose heparin (5000 units bd)

ANSWER E * heparin is the most studied, and seems to reduce the DVT rate by 66%-75% - LMWH are slightly better (ACCP) * mechanical methods reduce the DVT rate by 50%-66% (HTA) * dextran 70 reduces DVT rate by 50%, the same as warfarin (HTA)

Which of the following is the most frequent complication after use of LMA? A. dysphagia B. dysarthria C. sore throat D. hoarse voice E. dry mouth

ANSWER E Mr "LMA" Brimacombe again, chapter 21, p554-555 specifically lists all the problems with LMA: * Dry mouth 62-64% * Sore throat 13% * Dysphagia 11.5% * Dysarthria 5.3%

Anaemia in chronic renal failure is characteristically A. due to haemolysis in the renal vascular bed B. normochromic and microcytic C. due to defective haemoglobin synthesis D. responsive to iron and folate therapy E. associated with increased 2,3-DPG levels in blood cells

ANSWER E "The intracellular concentration of 2,3-bisphosphoglycerate is appropriately increased in response to anemia and hyperphosphatemia,9 with a moderate decrease in the affinity of hemoglobin for oxygen." (Williams Hematology)

Typical features of hyperparathyroidism include all of the following EXCEPT: A. Raised serum calcium B. Ectopic muscular calcification C. Raised urinary phosphate D. Staghorn calculi E. Lowered urinary calcium

ANSWER E "bones, groans, stones and moans" - most are diagnosed on random U+E's so asypmtomatic is 'typical' A: true B:? very unlikely to be 'typical' C: true D: true E: false - even though PTH decreases Ca excretion, urinary excretion actually high due to high plasma conc.

Codeine phosphate A. is converted by the liver to its active metabolite, oxycodone B. is not associated with tolerance on chronic use C. is not effective as an analgesic in approximately 20% of Causcasians D. is poorly absorbed from the gastrointestinal tract E. when given orally has approximately 5% of the analgesic potency of intramuscular morphine

ANSWER E # A - False, most to codeine-6-glucuronide, some to norcodeine, morphine, normorphine and hydrocodone. # B - False, tolerance does occur therefore oxycodone a better choice. # C - False,CYP2D6 exhibits genetic polymorphism in 9% UK and 30% Hong Kong population. # D - False, oral bioavailability of at least 50%. # E - True, see above or consider 60mg codeine oral goes to 30mg with first pass and then 10% metabolised to morphine is 3mg - 5% if you put oral codeine vs IM morphine.

Tracheo-oesophageal Fistula (TOF) A. is associated with cardiac anomalies in approximately 60% of cases B. is associated with oesophageal atresia in approximately 20% of cases C. is more common in males than females D. is usually left sided E. does not usually require contrast studies for diagnosis

ANSWER E # A. is associated with cardiac anomalies in approximately 60% of cases - false * Approx 27% (Table 2) # B. is associated with oesophageal atresia in approximately 20% of cases - false: * Higher association approx 96% # C. is more common in males than females - false # D. is usually left sided # E. occurs in approximately 1 in 3500 live births - true: * " The incidence of OS/TOF is 1 in 3500 live births."

TMP-103 [Mar10] [Aug10] What is the SVR in a patient with MAP 100mmHg, CVP 5, PCWP 15, CO 5L/min? A. ?0.8 B. ?3 C. 520 D. 1280 E. 1520 dynes.sec/cm-5

ANSWER E # SVR = (Systemic A-V Pressure difference) / Flow # Therefore SVR = (100-5)/5 = 95/5 = 19 mmHg/L/min # To convert to dynes.sec/cm5 then multiply by 80; this gives us 1520 dynes.sec/cm5.

Inserted DLT. FOB down tracheal lumen. What feature is most helpful in identifying Left vs Right main bronchus A. Trachealis muscle B. "there are 3 lobes in right lung" C. LMB longer than right D. Angle of RMB vs left E. Three segments of RUL

ANSWER E * A - False. Trachealis muscle divides at carina and continues in each main bronchus, so not particularly helpful. BUT ..I thought trachealis was only located posteriorly connecting the ends of the C-shaped cartilage. If you know whats the front and whats the back wouldnt this make it easy to figure out L from R ? - Isoma * B - False. While there are indeed 3 lobes in the right lung, that fact is not helpful to determine which is right or left main bronchus. * C - True. The LMB is about 5cm long before it gives off any subsequent lobar bronchi, whereas the RMB gives off a lobar bronchus (the RUL bronchus) about 2.5cm from the carina. This can help to determine between RMB and LMB. * D - ? False. While there is a difference in the angle (from the vertical) of the LMB and RMB, I don't know if this would be significantly appreciable bronchoscopically. Anyone? * E - True. The RUL bronchus has a trifurcation for each of the RUL segments, and this may also be useful in determining which side you are on.

Diabetics with cardiac denervation due to autonomic neuropathy develop: A. Supine hypertension B. Orthostatic hypotension C. Resting tachycardia D. Silent myocardial ischaemia E. All of the above

ANSWER E -Hypertension -Painless MI -Orthostatic hypotension -Lack of heart rate variability with breathing (normal heart rate variability during voluntary deep breathing is > 10 beats/min) -Resting tachycardia -Early satiety (due to delayed gastric emptying) -neurogenic bladder -lack of sweating and impotence. -impaired thermoregulation (intraoperative hypothermia) -sudden death syndrome -gastroparesis -bladder atony -asymptomatic hypoglycaemia

AM21 ANZCA version [2003-Aug] Q49, [2004-Aug] Q38, [2005-Apr] Q29 (Similar question reported in [Mar94] [Aug94] [Mar95] [Aug95] [Apr96] [Aug96] [Jul97] [Apr98] [Jul98]) The preferred technique of anaesthesia for reduction of a fractured wrist in a patient with myotonic disorder A. general anaesthesia with spontaneous ventilation using a volatile agent B. general anaesthesia with suxamethonium-induced muscle relaxation C. general anaesthesia with a non-depolarising relaxant reversed by neostigmine D. intravenous regional anaesthesia with lignocaine E. axillary brachial plexus block with lignocaine

ANSWER E 1. Perform an extensive preoperative evaluation. Organize a multi-disciplinary medical team. 2. Use regional anesthesia when appropriate. 3. Be cautious with premedications (benzodiazepines and opioids). 4. Keep the patient warm. 5. Consider applying defibrillator/pacer pads. 6. On induction, be aware of the high likelihood of aspiration and other airway complications. Avoid succinylcholine when possible. 7. Adhere to strict extubation criteria. Given the effects DM has on the pulmonary system, anticipate the need for supportive mechanical ventilation until extubation criteria are met. 8. Plan for the continuous SpO2 and EKG monitoring postoperatively. 9. Manage postoperative pain with NSAIDs, regional techniques, and acetaminophen when appropriate. Use opioids with extreme caution. 10. Encourage aggressive pulmonary toileting postoperatively.

Anaesthetising an obese patient. Acelerometer on TOF 0.9. Could dose suxamethonium on ideal body weight or total body weight. With respect to 1mg/kg IBW vs. TBW you will see: A: shorter onset and faster twitch recovery B: shorter onset and similar twitch recovery C: shorter onset and slower twitch recovery D: similar speed of onset with similar speed of twitch recovery E: similar onset and longer recovery

ANSWER E A & A February 2006 vol. 102 no. 2 438-442 The appropriate dose of succinylcholine (SCH) in morbidly obese patients is unknown. We studied 45 morbidly obese (body mass index >40 kg/m2) adults scheduled for gastric bypass surgery. The response to ulnar nerve stimulation of the adductor pollicis muscle at the wrist was recorded using the TOF-Watch SX® acceleromyograph. In a randomized double-blind fashion, patients were assigned to one of three study groups. In Group I, patients received SCH 1 mg/kg ideal body weight, in Group II 1 mg/kg lean body weight, and in Group III 1 mg/kg total body weight. After SCH administration, endotracheal intubating conditions were scored. The recovery from neuromuscular block was recorded for 20 min. There was no difference in the onset time of maximum neuromuscular blockade among groups, but maximum block was significantly less in Group I. The recovery intervals were significantly shorter in Groups I and II. In one third of the patients in Group I, intubating conditions were rated poor, whereas no patient in Group III had poor intubating conditions. Our study demonstrates that for complete neuromuscular paralysis and predictable laryngoscopy conditions, SCH 1 mg/kg total body weight is recommended.

The adverse effect of 6 units of Fresh frozen plasma LEAST likely to be a problem is A. an allergic reaction B. a febrile reaction C. a haemolytic reaction D. transmission of infectious disease (including hepatitis and HIV infection) E. citrate toxicity

ANSWER E A * Anaphylaxis only 1/30,000. Minor allergy 1/2000. B. a febrile reaction * This actually seems to be extremely rare with FFP (1/5000), compared with platelets or RBCs. And not to mention quite a minor 'problem' when it does occur. C. a haemolytic reaction * Rare but it does occur and is very serious when it does. It is due to the direct transfer of donor antibodies into the recipient's circulation causing haemolysis of the recipient's RBCs(ie the opposite to what occurs with transfused RBC haemolytic reactions in which the donor RBCs lyse) D. transmission of infectious disease (including hepatitis and HIV infection) * Viruses are the same as in RBCs - ie very very rare. Bacteria virtually unheard of. E. citrate toxicity * This totally depends on the context. If in the anhepatic phase of a liver Tx, sure, every time! Usually not much of an issue.

Regarding thoracic epidural analgesia for acute post-operative pain: A. it has NO role in improving preservation of total body protein after upper abdominal surgery B. opioids alone via the thoracic epidural provide significantly better analgesia than systemic opioids alone C. there is NO effect on the incidence of postoperative myocardial infarction D. the addition of adrenalin (epinephrine) to a local anaesthetic mixture has NO measurable benefit E. using local anaesthetics improves bowel recovery after abdominal surgery

ANSWER E A - FALSE : The combination of thoracic epidural analgesia with local anaesthetics and nutritional support leads to preservation of total body protein after upper abdominal surgery (U) (Level II). B - FALSE : epidural better than PCA except when epidural using hydrophilic opioids only C - FALSE : Thoracic epidural analgesia extended for more than 24 hours reduces the incidence of postoperative myocardial infarction D - FALSE: Addition of adrenaline improves block E - TRUE: reduced duration of ileus after colorectal surgery.

Each of the following findings is consistent with brain death EXCEPT A. absence of diabetes insipidus i.e. continued vasopressin secretion B. Babinski's reflex C. focal EEG (electroencephalogram) activity D. limb movement in response to touch E. nystagmus on injection of cold water into the ear canal

ANSWER E A - False * Absence of diabetes insipidus is consistent with a diagnosis of brain death. B - False * Presence of Babinski reflex is compatible with a diagnosis of brain death. C - False * EEG not part of the criteria in Australia. Has been used in the US. D - False * See IC24b - 2 and 4. E - True * An intact vestibulo-ocular reflex excludes brain death. Importantly: The following observations are compatible with the diagnosis of brain death: * spontaneous "spinal" movements of the limbs (not to be confused with a pathological flexion or extension response) * respiratory-like movements (shoulder elevation and adduction, back arching or intercostal expansion) without significant tidal volume * sweating, blushing, tachycardia. * normal blood pressure without pharmacological support * absence of diabetes insipidus (normal osmolar control mechanism) * deep tendon reflexes. * Babinski's reflex

When viewing the central venous pressure trace: A. an accentuated "a" wave supports the diagnosis of atrial fibrillation B. a steeper than normal "x" descent supports the diagnosis of tricuspid regurgitation C. blunted "a" and "v" waves are associated with extensive right ventricular infarction D. flattened "x" and "y" descents are associated with pericardial constriction E. a monophasic pattern with obliteration of the "y" descent supports the diagnosis of pericardial tamponade

ANSWER E A - False * In atrial fibrillation, a waves will be absent, and in atrioventricular dissociation, a waves will be dramatically increased ("cannon waves") as the atrium contracts against a closed tricuspid valve. B - False * In tricuspid regurgitation, the c wave and x descent will be replaced by a large positive wave of regurgitation as the blood flows back into the right atrium during ventricular contraction C - False * Prom a and v waves with RV infarction due to (a wave) increased atrial pressure from contraction into stiff RV (kind of like increased RA afterload) and v waves increased as often assoc TR hence increased filling RA. D - False * Pericardial constriction - tall a and v waves, steep x and y descents E - True In cardiac tamponade, all pressure will be elevated, and the y descent will be nearly absent

When viewing the central venous pressure trace: A. an accentuated "a" wave supports the diagnosis of atrial fibrillation B. a steeper than normal "x" descent supports the diagnosis of tricuspid regurgitation C. blunted "a" and "v" waves are associated with extensive right ventricular infarction D. flattened "x" and "y" descents are associated with pericardial constriction E. a monophasic pattern with obliteration of the "y" descent supports the diagnosis of pericardial tamponade

ANSWER E A - False * In atrial fibrillation, a waves will be absent, and in atrioventricular dissociation, a waves will be dramatically increased ("cannon waves") as the atrium contracts against a closed tricuspid valve. B - False * In tricuspid regurgitation, the c wave and x descent will be replaced by a large positive wave of regurgitation as the blood flows back into the right atrium during ventricular contraction C - False * Prom a and v waves with RV infarction due to (a wave) increased atrial pressure from contraction into stiff RV (kind of like increased RA afterload) and v waves increased as often assoc TR hence increased filling RA. D - False * Pericardial constriction - tall a and v waves, steep x and y descents E - True In cardiac tamponade, all pressure will be elevated, and the y descent will be nearly absent

When viewing the central venous pressure trace: A. an accentuated "a" wave supports the diagnosis of atrial fibrillation B. a steeper than normal "x" descent supports the diagnosis of tricuspid regurgitation C. blunted "a" and "v" waves are associated with extensive right ventricular infarction D. flattened "x" and "y" descents are associated with pericardial constriction E. a monophasic pattern with obliteration of the "y" descent supports the diagnosis of pericardial tamponade

ANSWER E A - False : In atrial fibrillation, a waves will be absent, and in atrioventricular dissociation, a waves will be dramatically increased ("cannon waves") as the atrium contracts against a closed tricuspid valve. B - False : In tricuspid regurgitation, the c wave and x descent will be replaced by a large positive wave of regurgitation as the blood flows back into the right atrium during ventricular contraction C - False : Prom a and v waves with RV infarction due to (a wave) increased atrial pressure from contraction into stiff RV (kind of like increased RA afterload) and v waves increased as often assoc TR hence increased filling RA. D - False : Pericardial constriction - tall a and v waves, steep x and y descents E - True :In cardiac tamponade, all pressure will be elevated, and the y descent will be nearly absent

Factors which do NOT contribute to the increased risk of aspiration pneumonitis during pregnancy include A. increased gastrin production B. a tendency for the stomach to be pushed up against the left diaphragm C. increased acidity of gastric secretion D. increased volume of gastric secretion E. decreased secretion of the hormone motilin

ANSWER E A - Gastrin is produced by placenta during pregancy. Causing hypersecretion of gastric acid. Nearly all parturients have gastric pH under 2.5, and over 60% have gastric volumes greater than 25 mL. B : stomach is pushed anterior and superior against left hemidiaphragm C : See A D : See A E : mixed reports in the literature, levels may or may not change, but they are not clearly associated with an increased risk of aspiration

What increases the risk of threading an epidural catheter into a blood vessel? A. not doing a CSE B. injecting saline prior to threading catheter C. LOR to saline instead of air D. paramedine instead of midline approach E. sitting position instead of lateral

ANSWER E A - it is thought that CSE decreases risk by ensuring catheter is midline, but no documented research. B : LOR NS decreases risk by distenting epidural space. OR 0.45 LOR NS vs LOR air. C : See B D : no difference (only 1 RCT showing this, ?insufficient data) E : epidural vein engorgement/ distention is greater when sitting instead of lateral position. Risk of epidural vein cannulation is higher in sitting position. Anesth Analg 2009;108:1232-42 : Metanalysis of 7 techniques in obstretric women 1. position : supine vs sittting OR 0.53 6 RCT 2. approach : paramedian vs midline no difference 1 RCT 3. touhy size : 16 vs 18 no difference 1 RCT 4. LOR technique : NS vs air, OR 0.45 8 RCT 5. oriface catheter : single vs multi OR 0.64 5 RCT 6. wire embbeded catheter 1 RCT 7. limiting catheter insertion to 6cm, >7cm OR 0.27 3 RCT

What increases the risk of threading an epidural catheter into a blood vessel? A. not doing a CSE B. injecting saline prior to threading catheter C. LOR to saline instead of air D. paramedine instead of midline approach E. sitting position instead of lateral

ANSWER E A - it is thought that CSE decreases risk by ensuring catheter is midline, but no documented research. B : LOR NS decreases risk by distenting epidural space. OR 0.45 LOR NS vs LOR air. C : See B D : no difference (only 1 RCT showing this, ?insufficient data) E : epidural vein engorgement/ distention is greater when sitting instead of lateral position. Risk of epidural vein cannulation is higher in sitting position. Anesth Analg 2009;108:1232-42 : Metanalysis of 7 techniques in obstretric women 1. position : supine vs sittting OR 0.53 6 RCT 2. approach : paramedian vs midline no difference 1 RCT 3. touhy size : 16 vs 18 no difference 1 RCT 4. LOR technique : NS vs air, OR 0.45 8 RCT 5. oriface catheter : single vs multi OR 0.64 5 RCT 6. wire embbeded catheter 1 RCT 7. limiting catheter insertion to 6cm, >7cm OR 0.27 3 RCT

38yo obese female with DM, other comorbidities, undergoing LSCS with spinal anaesthetic with a 27G Whitacre needle, having this and that, blah, blah. then two days later complains of numbness on a small patch on lateral aspect of mid-thigh. On full neuro exam - no other signs/symptoms. A. conus medullaris injury B. L2 nerve root compression C. L3 root lesion D. L4 root lesion E. meralgia paraesthetica

ANSWER E A : Conus medullaris ends at L1/L2 Sudden bilateral pain with dural puncture, Reflex : knee jerk preserved, ankles affected Radicular pain : minimal Lumbargo : severe Sensroy : saddle numbness, symmetrical and bilateral Motor : symmetric, hyperreflexic distal paresis Sphinter dysfunction E: Meralgia Paresthetica Mono neuropathy of lateral cutaneous nerve (purely sensory over anterolateral thigh, no motor) Focal entrapment as it passes through inguinal ligament Causes : DM, pregnancy, tight clothing, obesity, fetal position Treatment is conservative, lignicaine + steriod injection if paraesthesia is bad.

Visual loss post-operatively a. more common after external ocular compression b. incidence 1 in 200,000 c. most common after spinal surgery d. incidence independent of duration of surgery e. more common after isovolaemic haemodilution

ANSWER E A False: Ischaemic optic neuropathy(ION) more common(89%), bilateral, systemic causes( atherosclerosis, antihypertensives); Central retinal artery occlusion(CRAO) less common 11% and there is an association with external compression but it occurs less often than ION B False 1:125 000 overall surgery( CEACCP) C TRUE : spine operations (67 percent) followed distantly by cardiac bypass procedures (10 percent). The remaining 23 percent of cases are composed of liver transplants, thoracoabdominal aneurysm resections, peripheral vascular procedures, head and neck operations, prostatectomies and miscellaneous cases. D False association with surgery> 6hrs or blood loss > 1L( above reference) E FALSE : Hct does not play a factor Risk Factors include -large blood loss -systemic hypotension -anaemia -duration of surgery -smoker -atherosclerosis -diabetes -obesity External ocular compression is not a risk factor

Thoracic epidural blockade to T5 using bupivacaine 0.5% with adrenaline 1:200,000 involves: A. Blockade of resistance vessels B. Blockade of capacitance vessels C. Blockade of cardiac sympathetic nerves, causing bradycardia D. Beta-adrenergic stimulation E. All of the above

ANSWER E A True " Resistance: skin gets profound dilation. Autoregulated organs don't change blood flow much (eg kidney, heart, brain). Splanchic vasculature changes < muscle < skin. B True - venodilation > arteriodilation C True - Sympathetic blockade usually higher than sensory but it is highly variable - Cardiac accelerators are T1-4 D True - due to systemic absoption of adrenaline WITH SPINALS: Sympathetic block is approx 6 segments higher than sensory block. Temperature block is 2 segments higher than Touch block.

AM16 [Mar90] [Mar91] [Mar92] [Apr98] [Mar06] Aug10 Recognised clinical associations with Dystrophia myotonia include: A. development of diabetes mellitus B. abnormal intestinal motility C. cardiomyopathy D. ovarian dysfunction E. all of the above

ANSWER E A- Insulin Resistance B- May affect intestinal smooth muscle C- Does not necessarily correlate with severity of skeletal muscle involvement D- Females may be amenhorreic or have problems with infertility. Men have testicular atrophy. E- Hence all of above are true.

AM29 [Jul98] (type A) A 16 year old male patient with peritonitis undergoing appendicectomy has suspected MH. (History of ?grandmother/aunt died under anaesthetic). What would suggest/prove MH? (Or: MH reliably diagnosed if:) A. Elevated intraop temp > 39.5C B. Masseter spasm C. Intraop hypercapnia to >45mmHg D. Postop elevated creatinine kinase E. None of the above

ANSWER E A. Elevated intraop temp > 39.5C * Intermediately timed sign of MH B. Masseter spasm * Early Sign of MH C. Intraop hypercapnia to >45mmHg * Early sign of MH D. Postop elevated creatinine kinase * Late sign of MH E. None of the above

The LEAST useful investigation to diagnose early myocardial RE-infarction is A. CK-MB B. CK-MB as a fraction of total CK C. echo D. ECG E. troponins

ANSWER E A. FALSE : CK-MB is a very good test for acute myocardial injury. It is very specific, rises in quickly : 3-4 hours, peaks at 18-24 hours, returns to normal in 72 hours, therefore good test for re-infaraction B. FALSE : Total CK to CK-MB ratio known as the cardiac index, sensitive indicator of early MI C. FALSE : D. FALSE : E. TRUE : troponins more specific for infarction (more specific than CK-MB), rise within 2 hours, peak at 12hours, elevated for 5-10 days up to weeks, therefore ppor at detecting re-infarctions

AZ74 ANZCA version [2004-Apr] Q123, [2004-Aug] Q63, [Mar06] Q51, [Jul07] Intubation of patients with acute C5 spinal cord injury A. can only be safely performed using awake fibreoptic intubation B. is necessary in most patients for secretion clearance C. is necessary in most patients to avoid atelectasis D. is associated with a low risk of aspiration E. may be safely facilitated by the use of suxamethonium

ANSWER E A. FALSE : Options include (i) direct laryngoscopy and intubation in the presence of manual in-line immobilisation; (ii) blind nasal intubation if there is no compromise to the cribiform plate; (iii) blind oral intubation using the intubating laryngeal mask airway (ILMA); (iv) awake fibre-optic intubation; and (iv) surgical airway if intubation is not possible Direct laryngoscopy with in-line immobilisation is a safe and acceptable method B and C FALSE : A C5/C6 fracture will result in a VC 31% of predicted Note C5-C8-> complete intercostal muscle paralysis, and ineffective or no cough. Intubation indications with Cervical Spine injury * Prophylactic o VC < 15ml/kg (or <20% predicted) o Head or Chest Injuries * Therapeutic o Inability to clear secretions o Respiratory complications D. FALSE see above E. TRUE : Succinylcholine is the muscle relaxant of choice. The release of potassium associated with the use of succinylcholine in spinal cord injury has not been shown to be a problem until 3 days post-injury at the earliest.

AZ65a ANZCA version [2001-Apr] Q54, [2001-Aug] Q48, [2002-Aug] Q87, [2003-Apr] Q50 For personnel working in an operating room environment A. levels of volatile anaesthetic agents of less than 5 parts per million are considered safe B. chronic exposure to trace levels of anaesthetic agents causes significant cognitive impairment C. the risk of abnormal pregnancy is higher than that of smokers D. the risk of cancer is increased in males E. none of the above

ANSWER E A. FALSE : Recommended maximum concentrations in operating theatre (National Occupational Health and Safety Comission, Australia) • 25 ppm for nitrous oxide • 0.5 ppm for volatile agents when nitrous oxide is used, 2 ppm when it is not B. FALSE: The NIOSH recommendations arose from a study by Bruce and Bach in which healthy volunteers in the laboratory suffered a decrease in psychomotor performance when exposed to concentrations of nitrous oxide as low as 50 ppm, alone or in combination with 1 ppm of halothane. The same study showed that 25 ppm of nitrous oxide combined with 0.5 ppm of halothane had no effect. C. FALSE : This preliminary report showed the incidence of infertility, spontaneous abortion and children with congenital abnormalities in female anesthesiologists to be the same as that in other physicians D. FALSE : the prevalence of abortion was substantially higher in female anesthesiologists than in female physicians working outside the operating room. Studies in this era also concluded that the incidence of congenital anomalies in children of male and female anesthesiologists was higher than in the control groups of physicians. In addition to these reproductive effects, meta-analysis of six of these early studies linked the exposure to anesthetic gases to hepatic disease in males[10] and cervical cancer, liver disease, and kidney disease in females. E. TRUE

AZ56 ANZCA version [2001-Apr] Q53, [2001-Aug] Q40, [2002-Aug] Q32, [2003-Aug] Q67, [2005-Apr] Q72, [2005-Sep] Q82, [Apr07] Q67(Similar question reported in [Jul98]) A patient with severe liver disease is scheduled to have a portacaval shunt for portal hypertension. Recommended features of the anaesthetic management include A. use of vecuronium as the relaxant of choice B. care with the dose of suxamethonium because it may have a prolonged effect C. avoidance of fentanyl as an analgesic D. avoidance of all volatile anaesthetic agents including isoflurane E. reduction of the induction dose of thiopentone

ANSWER E A. FALSE : cis-atracurium is the drugs of choice B. FALSE The metabolism of succinylcholine may be slowed because of reduced pseudocholinesterase concentrations, but in practice this gives few problems C. FALSE D. FALSE E. TRUE : The dose of thiopental should be reduced because a reduction in plasma proteins results in an increased unbound fraction of drug; the distribution half-life and consequently the duration of action are also prolonged

The intercostal nerves A. arise as the sensory fibres of the anterior primary rami from their spinal segments B. give off a lateral cutaneous branch at the angle of the rib C. lie in the subcostal groove in only 50% of people D. lie between the intercostalis intimi and the pleura E. are enclosed in a dural sheath from their origin to the angle of the rib

ANSWER E A. FALSE : mixed sensory and motor nerve B. FALSE : the collateral motor branch is given off at the angle of the rib, while the lateral cutaneous branch arises in the mid axillary line C. FALSE : found in the classical subcostal position in 16.6%, in the midzone in 73% and in the inferior supracostal position in 10%. D. FALSE : Nerves lie between intermediate and inner most layers, ie between internal and transversus or intimi E. TRUE : the dura occasionally extends out along the intercostal nerve a variable distance before it adheres to the nerve as the neurolemma or nerve sheath' thus accounting for the rare occurrence of spinal anaesthesia following intercostal blocks.

Which statement regarding the use of opioids for the management of acute pain is true? A. in adults patient weight is the best predictor of opioid requirements B. metabolism to codeine-6-glucuronide produces the analgesic effect of codeine C. morphine produces more nausea and vomiting than pethidine D. pethidine is superior to morphine in the management of renal colic pain E. tramadol has a lower risk of respiratory depression than other opioids at equianalgesic doses

ANSWER E A. FALSE : patient's age is a better predictor than weight of opioid requirement but there is wide interindividual variability B. FALSE :10% demethylation in liver to morphine, remainder demethylated to inactive norcodeine C. pethidine induced more nausea and vomiting than morphine when used parenterally in the ED (level III-3) and in first 2 hours after gynaecological surgery (level II, APM 2010) D. not proven E. tramadol has a lower risk of respiratory depression than other opioids at equianalgesic doses - true: "Tramadol has a lower risk of respiratory depression and impairs gastrointestinal motor function less than other opioids at equianalgesic doses (U) (Level II)

When obtaining an image using ultrasound, the most effective way to improve the spatial resolution is to increase the: A. 2D gain B. frame rate C. single scatter D. time gain compensation E. transducer frequency

ANSWER E A. FALSE amplify signal intensity, brighter with increased noise B. FALSE : applies to color flow, increasing the frame rate will increase solution C. D. FALSE : selectively amplify weak signals from deeper structures E. TRUE - increase resolution and depth

Naltrexone A. given as a single usual dose antagonises the effects of opioids for approximately 8 hours B. mainly renally metabolised C. no hepatic side effects even at high doses D. is mixed opioid agonist-antagonist E. used for alcohol abuse

ANSWER E A. FALSE: duration of oral naltrexone is 72-108 hours B. FALSE: hepatic metabolism C. FALSE hepatocellular injury when given in excessive doses D. FALSE: opioid anatgonist E. TRUE: used for alcohol and opiate dependence

Post-operative ischaemic optic neuropathy A. has a good visual prognosis if optic nerve decompression is performed within 24 hours B. is an uncommon cause of post-operative blindness C. is commonly due to emboli to the ophthalmic artery D. is not associated with hypotension and low haematocrit E. may present as a confusional state

ANSWER E A. FALSe : Recovery is poor. ION is not associated with compression of the optic nerve B. FALSE : ION is the commonest cause of post operative blindness C. FALSE : can be use to emoblism, but ti is not the commonest cause D. FALSE E. TRUE Ischaemic Optic Neuropathy (ION) * Commonest cause of post-operative blindness * Divisible into anterior ION and posterior ION * Anterior & Posterior ION furthur divisible into arteritic and non-arteritic forms * Arteritic forms associated with inflammation of arteries. Diagnosis proven by temporal artery biopsy and steroid treatment * The non-arteritic form is much more common, and anterior ION is the commonest form of ION. Anterior ION usually arises from decreased O2 delivery to the watershed areas of the short posterior ciliary arteries. This has been associated with * Hypotension * Haemorrhage * Anaemia Prognosis varies, but recovery is usually poor

PI77b ANZCA Version [2006-Mar] Q121, [Jul06] Q67 Nitrous oxide anaesthesia may cause all of the following EXCEPT A. an increased incidence of myocardial ischaemia B. decreased leukocyte chemotactic response C. elevation of plasma homocysteine levels D. megaloblastic anaemia E. reversible inhibition of methionine synthetase

ANSWER E A. False * The ENIGMA trial (currently underway) is out to answer this question B. True * "The chemotactic migration of neulrophils and monocytes...a significant depression...was observed with enflurane, halothane, methoxyflurane and nitrous oxide (p < 0.05)...severity of depression of migration was maximal with nitrous oxide" CJA 84(6);631-37 C. True * Nitrous oxide anesthesia causes increased postoperative plasma homocysteine levels. Acute increases in plasma homocysteine are associated with impaired endothelial function and procoagulant effects. This nitrous oxide-induced plasma homocysteine increase may therefore affect the risk of perioperative cardiovascular events. D. True * exposure to nitrous causes megaloblastic changes in the bone marrow E. FALSE * the inhibition of methionine synthase is irreversible

Glycine, if absorbed into the circulation in large amounts during trans-urethral prostatic surgery, may cause: A. Hyperphosphataemia B. Nausea and vomiting C. Disseminated intravascular coagulation D. Anuria E. Temporary blindness

ANSWER E A. False B. True - Glycine's metabolism to ammonia can cause hyperammonaemia leading to N&V and coma C. False - DIC has been reported after TURP and generally is thought to be a result of the release of thromboplastins from the prostate into the circulation during surgery. As many as 6% of patients may develop subclinical intravascular coagulopathies D. False - Anuria may occur following absorption of irrigant fluid affecting the kidney's, but this effect is not due to glycine. no mention of renal effects in the big texts. Roizen says glycine causes a brisk osmotic diuresis. E. True- glycine acts endogenously as an inhibitory neurotransmitter in the retina. Blood levels >8mmol/L are associated with transient (<24hrs) blindness due to suppression of impulses from the retina to the cortex.

Von Willebrand's disease is A. A common acquired defect of platelet function B. Associated with an imbalance of thromboxane A2 and prostacyclin C. not responsive to DDAVP D. Often treated with epsilon aminocaproic acid (Amicar) E. Responsive to FFP

ANSWER E A. False. Inherited B. ? Not sure but I don't think that it affects production of either of these. C. Type 2B and 3 are not. These are however rare forms and 80% of VWD is type I which is responsive. D. Emedicine states that this is used for treatment of vWD. Often??? E. Responsive to DDAVP, cryoprecipitate or factor VIII concentrates can be used. FVIII is labile and therefore lower concentrations in FFP.

PZ53c ANZCA version [Apr97] [Jul97] [Apr98] [Apr99] [Aug99][2001-Aug] Q11, [2002-Mar] Q5, [2003-Aug] Q5, [2004-Apr] Q25, [2005-Sep] Q12, [Mar06] Q10 Nitric oxide, when used as supportive treatment for adult respiratory distress syndrome, A. potentiates the effect of the endothelial releasing factor in the lungs B. must be given by inhalation as it is rapidly metabolised by leucocytes C. may cause systemic hypotension when administered in a concentration of about 40 ppm D. must be administered continuously as the effect lasts only about 30 minutes E. will reduce the venous admixture when administered in a concentration of 20 ppm

ANSWER E A. False. Nitric Oxide (NO) is EDRF. Potentiate means one drug increases the action of another B. False. NO is metabolised by Oxy-Hb C. False. Stuedal et al "pulmonary vasoldilatation in the absence of systemic vasodilatation has been confirmed in numerous subsequent studies" p1096 - also confirmed in Miller p2815. D. False. The effect lasts seconds, not minutes E. True. All kinds of doses have been tried but 20ppm will do it. Stuedal at al "Inhaling 18ppm NO for 40min reduced the shunt fraction by 5% and increased the PaO2/FiO2 ration by 30% in patients with ARDS" p1102 NITRIC OXIDE * Synthesized from l-arginine by NOS. * Increases cGMP leading to smooth muscle relaxation. * t1/2 of 4-40 secs * Combines with Hb to form MetHb * Usual dose in adults 1-40ppm * Hypotension is not usually a significant problem * NO2 and MetHb are usually the dose limiting factors * In the body, nitric oxide (the 'endothelium-derived relaxing factor', or 'EDRF') is synthesized from arginine and oxygen by various nitric oxide synthase (NOS) enzymes and by sequential reduction of inorganic nitrate. Arginine is converted to citrulline by NOS. NO is a free radical. Nitric oxide can contribute to reperfusion injury when excessive nitric oxide produced during reperfusion (following a period of ischemia) reacts with superoxide to produce the damaging free radical peroxynitrite.

Features & management of the malignant neuroleptic syndrome do NOT include: A. Genetically determined response to dopamine antagonists B. Treatment with dantrolene C. Hyperthermia, tachycardia & peripheral vasoconstriction D. Raised white cell count E. Hyperthermia not often seen

ANSWER E A. Genetically determined response to dopamine antagonists - true: now appears to be a genetic link B. Treatment with dantrolene - true C. Hyperthermia, tachycardia & peripheral vasoconstriction - probably true D. Raised white cell count - true E. Hyperthermia not often seen - false NMS -rare idiosyncratic genetic disorder -reaction to antipsychotics -resembles malignant hyperthermia -hyperthermia -tachycardia -extrapyramidal signs : rigidity and dystonia -automonic dysfunction : labile BP,sweating, salivation, incontinence -CK and WCC increased -mortality 20%

For purposes of statistical analysis: A. The correlation coefficient relates association but not causation B. The unpaired t-test relates sample means C. The chi-squared test uses nominal values only D. Statistical significance does not mean clinical significance E. All of the above

ANSWER E A. TRUE The correlation coefficient relates association but not causation - "correlation is a measure of association" (Myles and Gin p.78) B. TRUE The unpaired t-test relates sample means - "Student's t-test is a parametric test used to compare the means of two groups" (Myles and Gin p.51) and it is used as an unpaired t-test "to test if the population means estimated by two independent samples differs significantly" (p.52) C. TRUE The chi-squared test uses nominal values only - "the Chi-square test is used to compare independent groups of categorical data" (Myles and Gin p.68). "Categorical data are nominal and can be counted" (p.1) D. TRUE Statistical significance does not mean clinical significance E. All of the above - best answer

Epidural sensory blockade to T2 in healthy adults is associated with each of the following EXCEPT A. bradycardia B. decreased circulating catecholamines C. dyspnoea D. vasodilatation E. raised PaCO2

ANSWER E A. bradycardia - true: will affect sympathetic cardiac plexus B. decreased circulating catecholamines - true: Decreased sympathetic response to pain C. dyspnoea - true : commonly results from loss of intercoastal muscle function, resulting in diaphragmatic breathing D. vasodilatation - true : loss of sympathetic tone E. raised PaCO2 - false : loss of intercoasts results in a rise in PaCO2 which is sensed by chemoreceptors to adjust minute ventilation. Hypercapnia does not result as the diaphragm is spared and aveolar ventilation can be maintained.

Which physical law relates to the function of rotameters? A. Avogadro's law B. Henry's law C. Poiseuille's law D. Graham's law E. Bernoulli's law

ANSWER E A: Equal volumes of gases, at the same temperature and pressure, contain the same number of particles, or molecules B: At equilibrium, the amount of gas (the mass) dissolved in a liquid, is directly proportional to the partial pressure of that gas in contact with the liquid at a given temperature. C: False - C isn't true for several reasons. Poiseuille's Law was based on straight, round, rigid tubes with laminar flow. Rotameters on the other hand: * Are variable orifice devices (ie tapered rather than straight tubes) * Flow is a combination of Turbulent and Laminar flow * The bobbin cannot be factored into Poiseuille's equation D: Graham's law; the rate of effusion of a gas is inversely proportional to the square root of the mass of its particles. E: The correct answer is Bernoulli's principle.

In relation to obstetric haemorrhage A. amniotic fluid embolism is unlikely to present as unexplained haemorrhage B. coagulopathy is uncommon, when severe abruption leads to maternal shock and fetal death C. the risk of placenta accreta, but NOT placenta previa, increases with an increasing number of caesarean sections D. treatment of uterine atony with prostaglandins is rarely assosciated with maternal adverse effects E. intravenous magnesium may facilitate replacement of an inverted uterus

ANSWER E A: FALSE: The classic presentation of AFE is characterized by sudden cardiovascular collapse, with profound systemic hypotension, cardiac dysrhythmia, cyanosis, dyspnea or respiratory arrest, pulmonary edema or the adult respiratory distress syndrome, altered mental status, and hemorrhage. B. FALSE 30% of patients with IUFD from abruption will have DIC. ( C. FALSE: Prior cesarean delivery increases the likelihood of placenta previa. Miller and associates (1996) cited a threefold increase of previa in women with prior cesarean delivery in over 150,000 deliveries at Los Angeles County Women's Hospital. The incidence increased with the number of previous cesarean deliveries; it was 1.9 percent with two prior cesarean deliveries and 4.1 percent with three or more. D. FALSE: PGF2alpha has significant side effects. PGE2 (misoprostil) has fewer side effects. E. TRUE: "Uterine relaxation may be necessary to replace the uterus; β-sympathomimetic agents, magnesium, and nitroglycerin all have been used to achieve this goal." (Miller 7th ed Ch 69)

Each of the following statements regarding cardiac tamponade as a complication of central venous lines is true, EXCEPT: A. catheters with multiple lumens carry greater risk B. for a left sided catheter, placement of the tip at the mid-point of the brachiocephalic vein is safe C. it usually occurs within the first week following insertion D. placement of the catheter tip above the junction of the right atrium and superior vena cava will avoid this complication E. visceral chest pain with drug infusion is an early symptom

ANSWER E A: TRUE..."Stiffer catheters are more likely to perforate. Stiffness is a function of the composition of the catheter (Silastic probably being safest) and the number of the lumen (a function of the greater diameter and presence of 'septa' within the catheter)." B: TRUE..."Zone C (mid‐point, left innominate vein). This is a suitable site for the tip when the catheter is introduced from the left internal jugular or subclavian vein, and reduces the risk of SVC perforation." (In reference to a diagram of the heart and great vessels showing various "zones" for the CVC tip) C: TRUE..."Most cases occur in the first week after insertion" D: FALSE (and answer to choose)..."the pericardium may ascend alongside the medial wall of the SVC by up to 5 cm (mean 3 cm). Thus, placement just proximal to the atrium does not obviate the risk of tamponade". E: TRUE..."This results in two warning signs: visceral‐type chest pain on infusion of drugs or parenteral nutrition solutions and a curved appearance of the distal catheter seen on chest x‐ray"

Regarding postoperative nausea and vomiting (PONV), it has been demonstrated that A. a repeat dose of a 5HT3 antagonist is useful if an initial prophylactic dose was not successful B. dexamethasone is best given at the end of surgery C. metoclopramide is effective for PONV prophylaxis D. 5HT3 antagonists are best given at the beginning of surgery E. the efficacy of droperidol is equivalent to ondansetron for PONV prophylaxis

ANSWER E AC143 ANZCA version [2005-Sep] Q146, [March 2006] Q40 Regarding postoperative nausea and vomiting (PONV), it has been demonstrated that A. a repeat dose of a 5HT3 antagonist is useful if an initial prophylactic dose was not successful B. dexamethasone is best given at the end of surgery C. metoclopramide is effective for PONV prophylaxis D. 5HT3 antagonists are best given at the beginning of surgery E. the efficacy of droperidol is equivalent to ondansetron for PONV prophylaxis

According to the AHA (American Heart Association) Guidelines intermediate clinical predictors of increased perioperative cardiovascular risk include each of the following EXCEPT A. prior heart failure B. diabetes mellitus C. previous myocardial infarct D. renal insufficiency E. uncontrolled systemic hypertension

ANSWER E ACTIVE CARDIAC CONDTIONS require investigation and likely intervention unless the surgury is an emergency. 1. Unstable coronary Sx 2. Decompensated heart failure 3. Significant arrythmias 4. Severe valvular disease CLINICAL RISK FACTOR to stratify other patients 1. Hx of heart disease 2. Prior heart failure 3. Cerebrovascular disease 4. Diabetes mellitus 5. Chronic renal impairment PROCEDURE RELATED RISK 1. Vascular (>5% risk of MI and/or death). Includes all vascular surgery except carotid endarterectomy 2. Intermediate (Less than 5%, more than 1%). Carotid endarterectomy, abdominal, thoracic, neuro, head and neck, major orthopaedic, prostatic, etc. 3. Low risk (less than 1%). Ambulatory, eyes, superficial, endoscopy, breast

AZ80 ANZCA version [2005-Sep] Q107, [Jul07] Mar10 Aug10 Which of the following is NOT an absolute contraindication to magnetic resonance imaging? A. cardiac pacemaker B. cerebral aneurysm clips C. cochlear implant D. implanted defibrillator E. prosthetic heart valve

ANSWER E Absolute Contraindications * electronically, magnetically, and mechanically activated implants: * cardiac pacemakers * ferromagnetic or electronically operated stapedial implants * hemostatic clips (CNS) * metallic splinters in the orbit Relative Contraindications * electronically, magnetically, and mechanically activated implants: * other pacemakers, e.g., - for the carotid sinus; - insulin pumps and nerve stimulators; - lead wires or similar wires * non-ferromagnetic stapedial implants * cochlear implants * prosthetic heart valves (in high fields, if dehiscence is suspected) * hemostatic clips (body) * makeup and tattoos * congestive heart failure * pregnancy (claustrophobia)

A patient with cancer pain has good analgesia with MS Contin 120 mg BD but has unmanageable nausea & vomiting and confusion. You would change the treatment to: A. Morphine syrup 40mg q4h B. Subcutaneous morphine 80mg over 24 hours C. Epidural morphine 80mg over 24 hours D. Intrathecal morphine 0.2 to 0.3 mg per 24 hours E. An alternative opioid because the patient has developed an allergy to morphine

ANSWER E According to Acute Pain Management - Scientific Evidence (2nd ed, 2005): * A - Apparently no difference with lockout period of 7 or 11 minutes (Level II) * B - FALSE * C - Nil benefit in routine use (Level II), useful in opiate tolerante (Level IV) * D - Dose limits have no evidence of benefit * E - As effective as IV Morphine PCA (Level II)

In primary adrenal insufficiency: A. Hypokalaemia is present B. ACTH level is normal C. Hypernatraemia is common D. Pigmentation is usually present E. Hypovolaemia with postural hypotension

ANSWER E Addisons Disease : Primary adrenocortical insufficiency. The adrenal cortex produces a number of hormones * Mineralocorticoids e.g. aldosterone * Glucocorticoids e.g. Cortisol * Sex hormones/Androgens Failure of the adrenal cortex leads to a decrease in these hormones. Destruction of 90% of the cortex is required before insufficiency occurs. Causes Destruction by antibodies (autoimmune disorder) is the cause in 70-80% of cases Other causes of Addisons: * TB (previously a common cause but no longer so in Western countries) (eg For anaesthetic-related case report see [1] * Metastases * Bilateral adrenalectomy * Haemorrhage e.g. following meningococcal sepsis Effects of Addisons Disease Skin * Pigmentation due to increased ACTH by anterior pituitary gland. The ACTH cross stimulates Melanocyte Stimulating Hormone (MSH) causing pigmentation Fluid Balance * Hypovolaemia with postural hypotension is typical Acid Base * Metabolic acidosis Electrolytes * Loss of aldosterone results in loss of Na+ reabsorption by the kidney * Hence urinary Na+ is high * Serum Na+ is low * Serum K+ is usually high * Haemoconcentration is often present "Hyponatraemia, hyperkalaemia, hypoglycaemia and haemoconcentration are often present." "...Hyponatraemia, Hypovolaemia, Hypotension, Hyperkalaemia and metabolic acidosis..." Anaesthesia * Volume is a key issue. These patients need intravascular reexpansion * Minimise neuromuscular blockade as hyperkalaemia can cause skeletal muscle weakness

An elective surgical patient with hypertrophic obstructive cardiomyopathy becomes hypotensive (systolic pressure 70 mmHg and heart rate 60 beats.min-1) during intravenous induction of anaesthesia. The most appropriate initial therapy would be aimed at increasing A. blood volume B. degree of myocardial depression C. heart rate D. myocardial contractility E. vasoconstriction

ANSWER E Aims with HOCM are the same as AS 1. Slow 2. Full 3. Tight 4. Avoid increases in contactility

Subtenon's block. What is the worst position to insert block? A. Inferonasal B. Inferotemporal C. Superonasal D. Supertemporal E. Medial / canthal

ANSWER E Answer E as might strike medial rectus insertion A sub-Tenon's block can be done in any of the 4 quadrants (options A-D above). These positions avoid the insertions of the rectus muscles which pierce the Tenon's capsule and insert into the sclera. Thus, for example, if you tried to do a block superiorly you would hit the superior rectus insertion.

Drug eluting stent 6 months old. On aspirin and prasugrel 10mg. Elective lap cholecystectomy for biliary colic. A. Do case while taking both. B. Do case while stopping both. C. Stop Prasugrel for 7 days, keep taking aspirin. D. Stop Prasugrel for some other different time E. Post-pone for 6 months

ANSWER E As per AHA/ACC Guidelines -recommended to continue dual anti-platelet therapy for 365 days, -Given that this is an elective procedure, and the potential for blood loss (while usually minimal) is significant (especially given dual anti-platelet therapy), the procedure should be delayed for another 6 months (i.e. 12 months from time of stent placement). The risk of an in-stent thrombosis is high if both anti-platelet drugs are not continued for the full 12 months.

Drug eluting stent 6 months old. On aspirin and prasugrel 10mg. Elective lap cholecystectomy for biliary colic. A. Do case while taking both B. Do case while stopping both C. Stop Prasugrel for 7 days, keep taking aspirin D. Stop Prasugrel for some other different time E. Post-pone for 6 months

ANSWER E As per AHA/ACC Guidelines, recommended to continue dual anti-platelet therapy for 365 days, and I would consider prasugrel the same as clopidogrel. Given that this is an elective procedure, and the potential for blood loss (while usually minimal) is significant (especially given dual anti-platelet therapy), the procedure should be delayed for another 6 months (i.e. 12 months from time of stent placement). The risk of an in-stent thrombosis is high if both anti-platelet drugs are not continued for the full 12 months.

Ciliary ganglion A sympathetic from inferior cervical ganglion B located inferiorly within orbit C may be damaged during a peribulbar block D preganglionic parasympathetic supply from the supra trochlear nerve E preganglionic parasympathetic originates from the Edinger Westpal nucleus

ANSWER E CILIARY GANGLION Mixed parasymp and sympathetic Located Posterior orbit, apex of orbit, between optic nerve and lateral rectus Parasympathetic Preganglionic efferent via Edinger Westphal Nucleus along CN III Postganglionic to short ciliary fibres : miosis and accomodation Sympathetic Carotid plexus causing vasoconstriction and midriaiss Sensory Nasociliary branch of ophthalmic nerve of trigeminal nerve Sensation of the globe

Campylobacter jejuni: A. Associated with meningitis B. May be treated with numerous single antibiotics C. Commonest cause of chronic gastritis D. Commonest colonizing organism after aspiration E. Causes diarrhoea in third world countries

ANSWER E Campylobacter jejuni -curved, helical shaped, non spore forming -Gram negative -microaerophillic bacteria Found in animal faeces -most common cause of human gastroenteritis -severely debilitating -rarely life-threatening -subsequent development in Guillian Barre Syndrome (2-3 weeks after) No antibiotics are usually given as the disease is self-limiting, however, severe or prolonged cases may require ciprofloxacin, erythromycin or norfloxacin. Rare cause of bacteremia (especially in immune compromised), meningitis and endocarditis.

A permanent pacemaker is indicated in the asymptomatic patient with A. complete heart block following acute inferior myocardial infarction B. sick sinus syndrome C. Wenckebach atrioventricular block D. Mobitz Type II atrioventricular block E. All the above

ANSWER E Class 1 - (most persuasive) 1. . Asystole. 2. . Symptomatic bradycardia (includes sinus bradycardia with hypotension and type I second-degree AV block with hypotension not responsive to atropine). 3. . Bilateral BBB (alternating BBB or RBBB with alternating LAFB/LPFB) (any age). 4. . New or indeterminate age bifascicular block (RBBB with LAFB or LPFB, or LBBB) with first-degree AV block. 5. . Mobitz type II second-degree AV block. Class 2a 1. . RBBB and LAFB or LPFB (new or indeterminate). 2. . RBBB with first-degree AV block. 3. . LBBB, new or indeterminate. 4. . Incessant VT, for atrial or ventricular overdrive pacing. 5. . Recurrent sinus pauses (greater than 3 seconds) not responsive to atropine. Class 2b 1. . Bifascicular block of indeterminate age. 2. . New or age-indeterminate isolated RBBB. Class 3 1. . First-degree heart block. 2. . Type I second-degree AV block with normal hemodynamics. 3. . Accelerated idioventricular rhythm. 4. . Bundle branch block or fascicular block known to exist before acute MI.

The most frequently reported cause of mortality associated with transfusion of blood and blood products is A. anaphylaxis B. bacterial sepsis C. haemolytic reaction D. transfusion associated graft versus host disease E. Transfusion Related Acute Lung Injury (TRALI)

ANSWER E Complications of blood transfusion - CEACCP Vol 6 Number 6 2006 TRALI is the most common cause of major morbidity and death after transfusion. It presents as an acute respiratory distresssyndrome (ARDS) either during or within 6 h of transfusion.

Correct statements regarding confidence intervals (CI) include all the following EXCEPT A. CI are derived from the standard error (of the mean). B. CI can be used to assess the precision of population parameter estimates. C. The width of the CI depends on the degree of confidence required. D. The width of the CI depends on the sample size. E. The width of the CI depends on the mean value of the sample

ANSWER E Confidence interval is a range with a particular assurance that the population parameter will fall within that range. 95%CI = mean +/- 1.96 SEM SEM = SD/SQRT(n) It is a measure of precision. 1. A narrow CI implies high precision. 2. CI that include zero are non significant ei no effect 3. Comparing CI that overlap imply non significant difference (ei clinical indifference)

Endocarditis prophylaxis in patient with MVR appropriate for? A. Dental procedure B. Rigid bronchoscopy C. Upper endoscopy with biopsy D. D&C E. Lithotripsy

ANSWER E Dental procedures, it really depends on which procedure.

PZ103 ANZCA version [2005-Sep] Q140, [Mar06] Q70 [Aug10] Intravenous paracetamol A. has a volume of distribution of 10 l.kg-1 B. is excreted 20% unchanged C. is highly protein bound D. is mainly excreted in the gut E. results in similar late plasma concentrations as oral paracetamol

ANSWER E Distribution 1L/kg Metabolism liver via 2 major pathways glucuronic acid and sulfuric acid conjugation -4% by cytochrome P450 -rapidly detoxified by reduced glutathione Elimination mainly excreted in the urine -60% as glucuronide -20% sulfate conjugates -<5% unchanged -Plasma 1/2 life 2.7 hour -Clearance 18L/hour

When obtaining an image using ultrasound, the most effective way to improve the spatial resolution is to increase the: A. 2D gain B. frame rate C. single scatter D. time gain compensation E. transducer frequency

ANSWER E E - Increased frequency increases resolution (at the expense of decreased penetration).

When obtaining an image using ultrasound, the most effective way to improve the spatial resolution is to increase the: A. 2D gain B. frame rate D. time gain compensation E. transducer frequency

ANSWER E E - Increased frequency increases resolution (at the expense of decreased penetration). Resolution determines the degree of image clarity : the ability to distinguish between two structures. It is influenced by : 1. Axial Resolution : ability to distinguish between two structures that are parallel, determined by frequency (wavelength), higher freq the higher the resolution 2. Lateral Resolution : resolution of 2 objects side by side (ei perpendicular to beam), determined by transducer beam width -inversely proportional to frequency -related to focal zone (depth) is best at narrowest part of beam

The best predictor of poor outcome for a peri-operative ulnar nerve injury is A. a delay in symptom onset to more than 48 hours postoperatively B. association with anaesthesia lasting more than 2 hours C. association with a brachial plexus block D. presence of bilateral injury E. presence of mixed sensory and motor deficit

ANSWER E E. Only 36% recovered at one year with a mixed sensory loss,

A patient with a head injury who has eye opening to speech, no verbal responses and withdrawalmotor responses would have a Glasgow Coma Scale score of A. 4 B. 5 C. 6 D. 7 E. 8

ANSWER E E=3/4 V=1/5 M=4/6 Eye Opening (E) 4=Spontaneous 3=To voice 2=To pain 1=None Verbal Response (V) 5=Normal conversation 4=Disoriented conversation 3=Words, but not coherent 2=No words (sounds only) 1=None Motor Response (M) 6=Normal 5=Localizes to pain 4=Withdraws to pain (Normal flexion) 3=Decorticate posture (Abnormal flexion) 2=Decerebrate (ie Extension) 1=None

Ebstein's abnormality describes a congenital defect of the A. aortic valve B. mitral valve C. pulmonary valve D. pulmonary veins E. tricuspid valve

ANSWER E Ebstein's anomaly is a rare congenital heart defect where there is apical displacement of the septal and posterior tricuspid valve leaflets with to atrialization of the right ventricle with a variable degree of malformation and displacement of the anterior leaflet. Usually associated with ASD. The leaflets are unusually deep in the right ventricle. The leaflets are often larger than normal. Leading to large TR and RHF. SVT and thrombus formation is common.%%//d3cgb598vs7bfg.cloudfront.net/images/upload-flashcards/back/9/4/47049443_m.jpg

AZ53 [Apr97] [Apr98] (type A) According to ANZCA policy documents which of the following emergency drugs does NOT need to be available where minor sedation is carried out: A. Lignocaine B. Atropine C. Adrenaline D. 50% dextrose E. Calcium chloride

ANSWER E Emergency drugs should include at least the following: 1. adrenaline 2. atropine 3. dextrose 50% 4. lignocaine 5. naloxone 6. flumazenil 7. portable emergency O2 supply

A term primip with meconium-stained liquor has a caesarian section. On initial assessment the neonate if pale and floppy with a heart rate of 90 bpm. Initial treatment should be: A. Positive pressure ventilation B. Dry and stimulate C. Suction the trachea D. Start CPR E. Suction the trachea

ANSWER E Endotracheal suctioning of non-vigorous infants born with meconium -stained amniotic should only be performed -Immediately after birth - Experienced practitioner and all needed equipment are immediately available - Before the onset of breathing or crying and in infants with decreased muscle tone. It should not be done if the infant is vigorous and breathing or crying. It should be done once with any subsequent resuscitation needed should be commenced expeditiously.

Epidermolysis bullosa may be associated with A. oesophageal stricture B. anaemia C. amyloidosis D. porphyria E. all of the above

ANSWER E Epidermolysis bullosa (EB) is an hereditary group of skin disorders, which may involve mucous membranes, especially particularly those of the oropharynx and oesophagus. The disease is classified into three types: * Simplex * Junctional * Dystrophic Incidence Dystrophic EB occurs in about 1 in every 300,000 births EB Dystrophica * Pseudosyndactlyly * Constriction of the mouth (microstomia) * Oesophageal strictures * Teeth are often poor * Malnutrition * Anaemia * Electrolyte derangements * Hypoalbuminaemia Diseases associated with EB * Porphyria * Amyloidosis * Multiple myeloma * Diabetes mellitus * Hypercoagulable states Management Patients with epidermolysis bullosa are extremely prone to frictional trauma, so all adhesives should be avoided. For example, the eyes should be lubricated rather than taped, special ECG electrodes without tape shoud be used, and any areas of potential friction should be well lubricated. Petroleum jelly gauze is useful for the sites where the anaesthetists fingers support the chin. Although carefully place endrotracheal tubes and laryngeal mask airways are not contraindicated, techniques that do not require their use are ideal. Therefore regional and ketamine based techniques are idea.

A 30-year-old man presents to the Emergency Department following a high speed motor vehicle accident. He has marked abdominal distenson, a pulse rate of 130 and a blood pressure of 80/50 mmHg. The most appropriate initial investigation would be A. abdominal angiogram B. abdominal paracentesis C. CAT scan of the abdomen D. plain X-ray of the abdomen E. FAST (focussed abdominal sonography for trauma) scan

ANSWER E FAST : Focused Abdominal Sonography in Trauma 4 Windows 1. Perihepatic : Pouch of Rutherford-Morison, most dependent abdominal compartment supine, looking at hepatorenal space Found at right mid-axillary line between 11th and 12th rib 2. Perisplenic : found left midaxillary line in 11th and 12th rib 3. Pelvis : visualise the cul de sac : Pouch of Douglas in women and rectovesicular pouch in men, midline superior to pubic symphysis 4. Pericardium : looking for pericardial effusion/haemorrhage, left xiphisternum angled cranially.

A 30-year-old man presents to the Emergency Department following a high speed motor vehicle accident. He has marked abdominal distenson, a pulse rate of 130 and a blood pressure of 80/50 mmHg. The most appropriate initial investigation would be A. abdominal angiogram B. abdominal paracentesis C. CAT scan of the abdomen D. plain X-ray of the abdomen E. FAST (focussed abdominal sonography for trauma) scan

ANSWER E FAST should be done first if available since it is rapid, sensitive and specific. Patient does not need to be transferred to obtain hence can be done in the department. No unstable patient should go for CT and AXR is not sensitive in picking up injury in blunt abdominal trauma. FAST when available would take initial precendent over a DPL which is used for unstable patients when the abdominal examination is equivocal. If free fluid/blood is seen on FAST, there is no need for a DPL.

What is the dose of FFP required to increase fibrinogen levels by 1 g/L A. 2 ml/kg B. 5 C. 10 D. 20 E. 30

ANSWER E FFP contains 2.5 g/l of fibrinogen Cryo contains 15g/l of fibrinogen http://onlinelibrary.wiley.com/doi/10.1111/j.1365-2141.2010.08208.x/pdf FFP given at 30ml/kg raised the fibrinogen by 1g/L in ICU patients http://www.medicine.wisc.edu/~williams/ffp.pdf This is a stupid question and probably is trying to say "Give FFP if the patient needs volume and fibrinogen, give cryp if the patient needs fibrinogen"

Emergence delirium in a kid in recovery. NOT used to treat a. Fentanyl 1mc/kg b. Midazolam 0.1 mg/kg c. Propofol d. Clonidine 1mc/kg e. Sucrose

ANSWER E Fentanyl IV 1-2 µg/kg (22), propofol IV 0.5-1.0 mg/kg (39), and midazolam IV 0.02-0.10 mg/kg (12,84) have all been used for the treatment of ED. A single bolus dose of dexmedetomidine 0.5 µg/kg was also shown to be efficient in the PACU for ED (85). Perhaps in the age group likely to get ED, sucrose probably isn't going to do the trick as usually that's reserved for the neonates.

Anaphylaxis to rocuronium. Which is most likely to cause coss-reactivity ? A. Vecuronium B. Pancuronium C. Atracurium D. Cisatracurium E. None of the above -cross reactivity too variable to predict

ANSWER E From AAGBI Cross-sensitivity between different NMBAs is relatively common, probably because they share a quaternary ammonium epitope. If anaphylaxis to an NMBA is suspected, the patient should undergo skin prick testing with all the NMBAs in current use. If a patient demonstrates a positive skin prick test (SPT) to an NMBA, the patient should be warned against future exposure to all NMBAs if possible. If it is mandatory to use an NMBA during anaesthesia in the future, it would seem appropriate to permit the use of an NMBA which has a negative skin test, accepting that a negative skin test does not guarantee that anaphylaxis will not occur.

Anaphylaxis to rocuronium. Which is most likely to cause coss-reactivity ? A. Vecuronium B. Pancuronium C. Atracurium D. Cisatracurium E. None of the above -cross reactivity too variable to predict

ANSWER E From Allergy 2007; 62: 471-487 Review Article: Anaphylaxis during anaesthesia:diagnostic approach "Cross Reactivity between NMBA is said to be common because of ubiquitous ammonium groups in these drugs. The estimated prevalence of cross-reactivity between NMBA is about 65% by skin tests and 80% by radioimmuno assay inhibition tests. While some pairings are common, the patterns of cross-reactivity vary considerably between patients." This was given to me in its paper form by our department allergy specialist. The fact that cross reactivity is unpredictable was also highlighted in the recent Allergy Podcast on the college website and that you cannot predict, without skin testing, which other NMBA you can use clinicall

Signs of severe dehydration in an infant include each of the following EXCEPT A. elevated pulse rate B. marked oliguria C. pale skin colour D. parched mucous membranes E. reduced blood pressure

ANSWER E From BJA review, ref below on assessment of dehydration Appearance: * Mild - Thirsty, restless, alert. * Moderate - Thirsty, restless or lethargic but rousable, pale (BP - Normal/low) * Severe - Drowsy to comatose, limp, cold, sweaty, grey, cyanosed ( BP - Low) Hypotension is very late sign.

The most important aspect of the peri-operative management of a patient with Gilbert's syndrome is A. avoidance of fasting B. avoidance of stress C. pre-operative transfusion of fresh frozen plasma (FFP) D. prophylaxis against hepato-renal syndrome E. recognition of aetiology of the laboratory abnormality

ANSWER E Gilbert syndrome is a metabolic disorder of little consequence. common hereditary cause of increased bilirubin and is found in up to 5% of the population It has no anaesthetic implications or clinical manifestations. This syndrome is considered harmless.

A patient suddenly collapses 36 hours after gastrectomy. His blood pressure is 80/30 mmHg, his pulse rate 100/minute, his central venous pressure +2 cm (from the sternal angle) and his temperature 39C. The most likely diagnosis is A. acute dilataton of the stomach B. anastomotic disruption C. haemorrhage D. pulmonary embolismm E. Septicaemia

ANSWER E Haemorrhage possible but less likely with fever 39° PE: - with temp 39°C less likely as mentioned above. - with sysBP 80 and HR 130, I'd expect a higher JVP due to R heart failure - no mention of resp symptoms/signs, chest pain or else in the stem Gastric dilatation: vagus response would result in bradycardia and hypotension Anastomotic breakdown: Will often cause sepsis, prob. more likely with colo-rectal surgery. Also, needs some time to develop sepsis. Septicaemia: Low BP, tachycardia, high temperature are all signs in sepsis, plus there's a good reason to be septic 2d postop. A CVP 2cm above sternal angle is not unusual, the fact that the CVP is not lower can have various reasons, eg sepsis-associated myocardial dysfunction, low RV compliance in high-CO state, incr PVR due to pulm pathology.

Male with a haemoglobin of 80%, and a reticulocyte count of 10%. Possible diagnosis: A. Untreated pernicious anaemia B. Aplastic anaemia C. Acute leukaemia D. Anaemia of chronic disease E. Hereditary spherocytosis

ANSWER E Hereditary spherocytosis (HS) is a genetically-transmitted (autosomal dominant) form of spherocytosis, an auto-hemolytic anemia characterized by the production of red blood cells that are sphere-shaped rather than donut-shaped, and therefore more prone to haemolysis There is a marked heterogeneity of clinical features, ranging from an asymptomatic condition to fulminant haemolytic anemia. The morphologic hallmark of HS is the microspherocyte, which is caused by loss of membrane surface area, and an abnormal osmotic fragility in vitro. Investigation of HS has afforded important insights into the structure and function of cell membranes and into the role of the spleen in maintaining red blood cell (RBC) integrity. An intrinsic genetic defect causes defects in membrane proteins. The major complications are aplastic or megaloblastic crisis, haemolytic crisis, cholecystitis and cholelithiasis, and severe neonatal haemolysis. Haemolysis in HS results from the interplay of an intact spleen and an intrinsic membrane protein defect that leads to abnormal RBC morphology. HS erythrocytes are caused by membrane protein defects resulting in cytoskeleton instability.

A female patient with a history of severe postoperative nausea and vomiting presents for abdominal surgery. If a volatile agent is used for maintenance of anaesthesia the most effective treatment to reduce her risk of postoperative nausea and vomiting would be A. avoidance of nitrous oxide B. prophylactic dexamethasone (4 mg) C. prophylactic droperidol (1.25 mg) D. propylactic ondansetron (4 mg) E. a combination of prophylactic dexamethasone and droperidol

ANSWER E IMPACT trial A Factorial Trial of Six Interventions for the Prevention of Postoperative Nausea and Vomiting by Apfel et al 5199 high risk patients for PONV Randomised into 1 of 64 combinations of 6 prophylatic interventions 1. 4mg ondancetron or no ondancetron 2. 4mg dexamethasone or no dexamethason 3. 1.25mg droperidol or no droperidol 4. propofol or volatile anaesthetic 5. nitrogen or nitrous oxide 6. remifentanyl or fentanyl Results 1. Ondancetron, dexamethasone and droperidol reduced PONV risk by 26% 2. Propofol reduced risk by 19% 3. Omitting nitrous oxide reduced risk by 12% 4. TIVA reduced risk by 25% 5. relative risks associated with the combined interventions can be estimated by multiplying the relative risks associated with each intervention 6. Absolute risk reduction is based on patient's baseline risk.

Ehlers-Danlos syndrome. Most important to specifically do all EXCEPT: A: Avoid hyperextension of the neck B: Damage to the teeth C: Avoid joint hypermobility D: Gastro oesophageal reflex E: Strict temperature regulation

ANSWER E ISSUES 1. GORD and gastritis 2. Early satiety and delayed gastric emptying 3. High, narrow palate and dental crowding 4. Peridontal disease (friability, gingivitis, gum recession) 5. Joint laxity. Subluxations and dislocations are common and represent the major manifestation of the condition. All sites can be involved, including the extremities, vertebral column, costo-vertebral and costo-sternal joints, clavicular articulations, and temporomandibular joints.

Intra-ocular pressure is increased by A. head-up B. hypothermia C. metabolic acidosis D. miosis E. respiratory acidosis

ANSWER E Increases ICP -Respiratory acidosis -MAP -Hypoxaemia -Straining -Blinking -Mydriasis Decreases ICP -metabolic acidosis -Miosis

The commonest cause of visual deficit following anaesthesia for major surgery is A. central retinal artery occlusion B. central retinal vein occlsuion C. cortical blindness D. glycine toxicity E. ischaemic optic neuropathy

ANSWER E Ischemic optic neuropathy is the most common diagnosis in postoperative visual loss. Ischemic optic neuropathy is divided into anterior and posterior, depending upon the location of the lesion on the optic nerve. Anterior ischemic optic neuropathy -53% cardiopulmonary bypass procedures -12% prone Posterior ischemic optic neuropathy -48% neck, nose or sinus operations -16% prone -11% cardiopulmonary bypass procedures

Increased risk of post-partum haemorrhage in: A. Nulliparous patient B. Patient < 20 years old C. Factor V Leiden deficiency D. Oligohydramnios E. Prolonged labour

ANSWER E Journal of Obstetrics 2004 While established risk factors can be identified among risk factors of PPH during labor after multivariate analysis: prolonged labor, oxytocin stimulation of labor, cesarean section, instrumental delivery, genital lacerations and episiotomy, prolonged third stage of labor, retained placenta; other risk factors are still uncertain: induction of labor, hyperthermia or chorioamniotitis, analgesia or anesthesia, macrosomia, various cesarean section techniques.

AM03b [Mar91] Which drug is known to trigger malignant hyperthermia? A. Droperidol B. Pancuronium C. Neostigmine D. Opioids E. None of the above

ANSWER E Known "triggering drugs" are: * ALL depolarising muscle relaxants (eg suxamethonium, suxethonium, decamethonium) * All potent volatile anaesthetic agents

In clinically used doses levosimendan, a novel inotrope used in heart failure, A. acts by increasing levels of intracellular cyclic AMP B. increases myocardial intracellular calcium C. increases myocardial oxygen consumption D. increases the rate of fatal ventricular arrhythmias E. increases the sensitivity of intracellular contractile proteins to calcium

ANSWER E Levosimendan, through its calcium-dependent binding to cardiac troponin C, exerts a positive inotropic effect on the myocardium. It also induces vasodilation by acting as an opener of ATP-dependent K-channels in vascular smooth muscle. Levosimendan itself has a short half-life, approximately 1 hour. Active metabolites of levosimendan that have longer half-lives have been identified.

Low molecular weight heparins A. are cleared principally by the liver B. are ineffective if only administered post-operatively C. cross the placenta D. have a dose-dependent half-life E. have a longer half-life than standard (unfractionated) heparin

ANSWER E Low molecular weight heparins * A. are cleared principally by the liver - false: "Metabolic breakdown of Clexane is slight and takes place mainly in the liver (desulfation and depolymerisation)." (MIMs online) * B. are ineffective if only administered post-operatively - false * C. cross the placenta - false: "The anti-Xa activity generated by Clexane does not cross the placental barrier during the second trimester of pregnancy." (MIMs online) Also "In contrast to warfarin, heparin does not cross the placenta and has not been associated with fetal malformations; therefore it is the drug of choice for anticoagulation during pregnancy." (Goodman and Gilman) * D. have a dose-dependent half-life * E. have a longer half-life than standard (unfractionated) heparin - true

The median nerve A. can be blocked at the elbow immediately lateral to the brachial artery B. can be blocked at the wrist between palmaris longus and flexor carpi ulnaris C. can be blocked at the wrist medial to flexor carpi ulnaris D. is formed from the lateral, medial, and posterior cords of the brachial plexus E. provides sensation to the radial half of the palm

ANSWER E MEDIAN NERVE Definition : mixed sensory and motor nerve of the brachial plexus Spinal nerve : C5-8, T1 * C5 to C7 roots from lateral cord of brachial plexus * C8 and T1 roots from medial cord of brachial plexus Anatomy 1. Inputs from the lateral cord and medial head of the medial cord 2. unite to form the median nerve in front of the 3rd part of the axillary artery 3. descends through the arm lateral to the brachial artery, cross at the midde point of the upper arm, then medial to the brachial artery 4. crosses in front of the elbow, lying on brachialis, passing medial to the brachial artery 6. dives between two head of pronator teres separating from ulnar artery 7. descends through the forearm between flexor digitorum superficialis (FDS) and flexor digitorum produndus (FDP) 8. emerging between flexor carpi radialis (lateral) and flexor digitorum and palmaris longus (medial) 9. passes through carpal tunnel under the flexor retinaculum SUPPLIES 1. Muscular branches Superficial group: * Pronator teres * Flexor carpi radialis * Palmaris longus Intermediate group: * Flexor digitorum superficialis muscle Deep group: * Flexor digitorum profundus (only the lateral half) * Flexor pollicis longus * Pronator quadratus Hand : LOAF * Lumbrical 1 and 2 * Opponens Pollicis * Abductor pollicus brevis * Flexor pollicus brevis (also innervated by ulnar nerve) 2. Cutaneous branches : anterior aspects of radial 3 digits and skin over the dorsal aspect of the distal phalanges 3. Articular branches : to elbow and wrist TESTS Motor : Opposition and flexion of the thumb are lost (LOAF) Sensory : Palm - 3 1/2 Fingers

Mastocytosis may result in elevation of serum: (a) ALT (b) AST (c) LDH (d) creatine kinase (e) tryptase

ANSWER E Mastocytosis is a group of rare disorders characterized by presence of mast cell proliferation and accumulation in multiple organs, most commonly the skin Incidence : less than 1:200,000 Diagnosis : 1 Major and 1 Minor Criteria OR 3 Minor Major criterion is multifocal dense infiltrates of mast cells in bone marrow and/or other extracutaneous organs. Minor criteria include -baseline total tryptase level of greater than 20 ng/mL -greater than 25% of the mast cells in bone marrow aspirate smears or tissue biopsy sections having spindle atypical morphology -mast cells in bone marrow, blood, or other lesional tissue expressing CD25 or CD2; -detection of a codon 816 c-kit point mutation in blood, bone marrow, or lesional tissue Presentation Most patients have pruritic cutaneous lesions. Some patients, especially those with extensive cutaneous disease, experience acute systemic symptoms exacerbated by certain activities or ingestion of certain drugs or foods. Possible systemic symptoms include flushing, headache, dyspnea, wheezing, rhinorrhea, nausea, vomiting, diarrhea, and syncope. Patients also may have chronic systemic symptoms involving various organ systems. o Involvement of the skeletal system may be manifested as bone pain or the new onset of a fracture. Long-term exposure to heparin and stem cell factor from degranulated mast cells is believed to put patients at risk for osteoporosis. o Involvement of the central nervous system may produce neuropsychiatric symptoms, as well as nonspecific changes such as malaise and irritability. o GI involvement may yield weight loss, diarrhea, nausea/vomiting, and abdominal cramps. o Cardiovascular effects can include shock, syncope (resulting from vascular dilatation), or angina. o Anaphylactic reactions to hymenoptera stings may be the first sign of mastocytosis.%%Treatment 1. avoid agents that precipitate mediator release, such as aspirin, nonsteroidal anti-inflammatory drugs, codeine, morphine, alcohol, thiamine, quinine, opiates, gallamine, decamethonium, procaine, radiographic dyes, dextran, polymyxin B, scopolamine, and D-tubocurarine 2. H1 and H2 anatgonists decrease pruritus, flushing, and GI symptoms 3. Oral disodium cromoglycate may ameliorate cutaneous symptoms, such as pruritus, whealing, and flushing, as well as systemic symptoms, such as diarrhea, abdominal pain, bone pain, and disorders of cognitive function.

In a 140kg obese patient, compared to a 70 kg person A. cardiac output >20% lower B. cardiac output 10% lower C. cardiac output no different D. cardiac output 10% higher E. cardiac output >20% higher

ANSWER E Miller : 10ml/min per kg of fat BJA article : 20-30 ml/min per kg fat Suggesting that E is the answer

The best predictor of poor outcome for a peri-operative ulnar nerve injury is A. a delay in symptom onset to more than 48 hours postoperatively B. association with anaesthesia lasting more than 2 hours C. association with a brachial plexus block D. presence of bilateral injury E. presence of mixed sensory and motor deficit

ANSWER E Miller Only 36% recovered at one year with a mixed sensory loss,

Which of the following is the most frequent complication after use of LMA? A. dysphagia B. dysarthria C. sore throat D. hoarse voice E. dry mouth

ANSWER E Mr "LMA" Brimacombe again, chapter 21, p554-555 specifically lists all the problems with LMA: Dry mouth 62-64% Sore throat 13% Dysphagia 11.5% Dysarthria 5.3%

AM04C ANZCA Version [Jul06] Q45 A 7 year old 30 kg. boy was booked for repair of an inguinal hernia. He was noted to have muscular looking arms and legs. Following midazolam premedication anaesthesia was induced with thiopentone intravenously and N2O/O2 by mask. Airway difficulties led to his being given 30 mg. of suxamethonium. He showed vigorous fasiculation followed by generalised muscular rigidity. The rigidity persisted despite another 30 mg. of suxamethonium intravenously. The probable cause of this response to suxamethonium is A. atypical pseudocholinesterase B. Duchenne's muscular dystrophy C. familial periodic paralysis D. hyperkalaemia E. myotonia congenita

ANSWER E Myotonia congenita is transmitted as an autosomal dominant trait and becomes manifest at birth or during early childhood. Skeletal muscle involvement is widespread, but there is not usually involvement of other organ systems. Muscle hypertrophy and myotonia are present. The disease does not progress nor does it result in a decreased life expectancy. Patients with myotonia congenita respond to phenytoin, mexiletine, or quinine therapy. The response to succinylcholine administration is abnormal.

Consider the following blood gases. Normal ranges are in brackets. pH 7.28 PaCO2 36 Bicarbonate 18 mmol.l-1 (18-25) Base excess -7 mmol.l-1 (-4- +3) Na+ 142 mmol.l-1 (135-145) Cl- 112 mmol.l-1 (98-110) These blood gases are consistent with A. acute renal failure B. diabetic ketoacidosis C. ethylene glycol overdose D. intraoperative infusion of 6 litres of normal saline E. salicylate overdose

ANSWER E Normal AG metabolic acidosis

At a late stage in severe septic shock A. the myocardium becomes increasingly sensitive to catecholamines B. adrenaline should not be infused as it is predominantly an alpha agonist at low doses C. the infusion of endogenously occurring catecholamines is futile as blood levels are already too high D. the infusion of dopamine is strongly recommended as it exerts its effects in high doses via different receptors than in low doses E. inadequate catecholamine synthesis may contribute to the poor circulation

ANSWER E Option A. False. With increasing time, the myocardial receptors become downregulated, hence actually become less sensitive. Option B. False. Adrenaline has predominantly beta effects at low doses, not alpha effects. Option C. False. Administration of catecholamines is necessary to maintain cardiovascular support. Option D. False. Whilst dopamine is an inotrope, it stands as a relatively poor relative to adrenaline or noradrenaline in the use of sepsis. Previous theories that low dose dopamine preserved renal function have now been soundly discredited. Dopamine itself is a diuretic, and its action is to increase urine output rather than preserve GFR. Option E. Correct. Endogenous catecholamine synthesis is often a factor in sepsis, hence the theory that supplementation with glucocorticoids can maintain or enhance production of catecholamines, leading to an improved outcome.

Abnormal Q waves are NOT a feature of the ECG in A. an old myocardial infarction B. left bundle branch block C. recent transmural myocardial infarction D. digitalis toxicity E. Wolff-Parkinson-White syndrome

ANSWER E Origin of the Q Wave The Q wave represents the normal left-to-right depolarisation of the interventricular septum Small 'septal' Q waves are typically seen in the left-sided leads (I, aVL, V5 and V6) Q waves in different leads Small Q waves are normal in most leads Deeper Q waves (>2 mm) may be seen in leads III and aVR as a normal variant Under normal circumstances, Q waves are not seen in the right-sided leads (V1-3) Pathological Q Waves Q waves are considered pathological if: > 40 ms (1 mm) wide > 2 mm deep > 25% of depth of QRS complex Seen in leads V1-3 Pathological Q waves usually indicate current or prior myocardial infarction. Differential Diagnosis Myocardial infarction Cardiomyopathies — Hypertrophic (HOCM), infiltrative myocardial disease Rotation of the heart — Extreme clockwise or counter-clockwise rotation Lead placement errors — e.g. upper limb leads placed on lower limbs

6 month old baby for VSD repair. Induced with 50% N2O, O2, sevoflurane 8%. While obtaining IV access, the patient desaturates to 85%. The manouevre to increase the O2 saturations is to A give a fluid bolus B change from sevoflurane to isoflurane C institute CPAP D decrease the FiO2 E reduce the sevoflurane concentration

ANSWER E PVR/SVR ratio determining R-->L shunt

6 month old baby for VSD repair. Induced with 50% N2O, O2, sevoflurane 8%. While obtaining IV access, the patient desaturates to 85%. The manouevre to increase the O2 saturations is to A give a fluid bolus B change from sevoflurane to isoflurane C institute CPAP D decrease the FiO2 E reduce the sevoflurane concentration

ANSWER E Poorly worded question, depends if examiner is looking at causes of laryngospam or reversal of shunt in VSD. The clinical situation will reveal which is more likely. VSD in children typically non cyanitic, restrictive CHD. Normally left to right shunt. Resulting in high pulmonary flow and therefore pressure. VSD most common isolated CHD 20% of all defects Classifed by anatomical location 1. Supracristal :above crista supraventricularis just under annulus of aorta 2. Infracristal : lower in membranous septum 3. Canal type : auriculo-ventricular canal 4. Muscular : anywhere in muscular septum Children with small defects have few symptoms due to restricted flow across defect, muscular type may close spontaneously Larger defects, heart failure develops due L-R shunt, increasing pulmonary hypertension and volume overload. Hypoxemia results when shunt is reversed R-L shunt Avoiding increases in PVR -Hypoxia -hypercapnia -acidosis -hypothermia -hyperinflation, CPAP, PEEP -atelectasis -sympathetic stimulation Maintain SVR -anaesthetic doses of fentanyl -

Following a left sided pneumonectomy, a left intercostal drain is placed and connected to an underwater drainage system. In the postoperative period A. a leakage of air is expected from the drain B. the patient should be nursed in the right lateral decubitus position C. the underwater seal drain should be left on continuous free drainage D. the underwater seal drain should be left on continuous free drainage, and connected to wall suction for 5 minutes every hour E. the underwater seal drain should remain clamped and be released for a short period every hour

ANSWER E Post-pneumonectomy an ICC is rarely placed However, when an ICC is placed it should be clamped and release intermittently. -it should NEVER be placed on suction as it will cause mediastinal shift and cardiac herniation. If cardiac hernation should occur, patient should be turned non operative side down and chest drain open to air.

A patient presents with headache, morning vomiting and double vision for three weeks. On examination nystagmus is present when the eyes are turned to either side. The most likely diagnosis is A. acoustic neuroma B. craniopharyngioma C. frontal glioma D. pituitary adenoma E. posterior fossa tumour

ANSWER E Posterior fossa tumour Symptoms and their association: * early morning vomiting = raised ICP * double and blurred vision, nystagmus = infratentorial lesion * problems with equilibrium, gait, and coordination = infratentorial lesion * focal problems (eg motor or sensory deficit, speech change, seizures) = supratentorial lesion * strong hand preference = supratentorial lesion * neuroendocrine problems (DI, hypothyroidism) = suprasellar lesion * change in visual acuity, visual field defect, Marcus Gunn pupil (afferent papillary defect), nystagmus = visual pathway lesion * long nerve tract motor and/or sensory deficits, bowel and bladder deficits, and back or radicular pain = spinal cord lesion

AM28b ANZCA version [2003-Apr] Q136, [2004-Aug] Q30, [2005-Apr] Q27 In a patient with myasthenia gravis undergoing a laparotomy for large bowel obstruction, the need for post-operative ventilation is significantly increased by a A. daily dose of pyridostigmine > 180mg B. known history of resistance to suxamethonium C. known history of sensitivity to non-depolarising muscle relaxants D. past history of prednisolone treatment > 10 mg.day-1 E. recent history of dysphagia

ANSWER E Preoperative predictors of postoperative need for ventilation: * duration of disease of greater than 6 years * history of coexisting chronic resp disease * dose requirements of pyridostigmine > 750 mg/day less than 48 H prior to surgery * preoperative VC < 2.9L

A fifty-five-year-old man on antihypertensive medication, including an ACE inhibitor, has a total knee replacement. Red cell transfusion is begun in recovery through a leukocyte reduction filter after brisk bleeding into his drains. A recognised complication of the use of this filter in this situation is A. air embolism B. clotting factor depletion C. haemolysis D. increased risk of postoperative infection E. severe hypotension

ANSWER E Profound hypotension has been reported in patients taking angiotension-converting-enzyme (ACE) inhibitors and receiving pretransfusion leukocytereduced blood products—platelets in particular (96). Presumably, ACE inhibitors decrease bradykinin degradation thereby prolonging its intravascular half-life.

AZ45 [Mar93] [Aug94] [Apr97] [Apr99] In a patient who is to receive a renal transplant: A. Suxamethonium is contraindicated if the patient is anephric B. Hypotension should be treated with phenylephrine or methoxamine C. Regional anaesthesia is contraindicated D. CVP is an inappropriate guide to fluid replacement E. Hypotension should be treated with dopamine

ANSWER E Promote renal perfusion by (a combination of...) * decreasing anaesthesia depth * maintaining high normal SBP * giving crystalloids * dopamine IV (3ug/kg/min)

Protamine anaphylaxis more common in: A. Men who have had vasectomies B. Infertile women C. Diabetics taking porcine insulin D. People with seafood allergy E. Patients previously exposed to protamine

ANSWER E Protamine sulfate is a polypeptide with molecular weights ranging from 4500 to 5000 Da that is used to reverse heparin anticoagulation and retard the absorption of insulin, often as neutral protamine Hagedorn (NPH). The polypeptide is extracted from salmon milt in a protein purification process. Protamine is a series of arginine-rich basic proteins (also called histones) in fish cell nuclei that provide structural integrity to chromatin.72 The basic guanidine groups of arginine allow it to bind to the acidic heparin molecule to reverse its activity. A. FALSE : After vasectomy, the blood-testis barrier exposes the tissues, and 55%-73% of men with vasectomies develop antibodies to sperm antigens of this group, 20%-33% develop autoantibodies against protamine. However, there have been no observed clinical reactions in a prospective evaluation of 16 vasectomized patients undergoing cardiac surgery with protamine reversal of heparin, but these were small numbers. B. FALSE : no documented increase in risk C. FALSE : no cross reactivity to porcine insulin. However, patients who receive protamine containing insulins, including NPH insulin, are at the greatest risk. Stewart et al. reported in 1984 that 4 of 15 NPH diabetics (27%) had anaphylaxis after protamine reversal of heparin after cardiac catheterization. Incidence of 0.6% (1 of 160) to 2% (1 of 50) in NPH insulin-dependent diabetics undergoing cardiac surgery, a rate 10-30 times more than other patients from 4796 patients evaluated. D. FALSE : fish-allergic patients are thought to be a theoretical risk. Reports are limited to a hand full of case reports only.. E. True : Previous exposure to IV protamine in vascular or cardiac procedures increase the risk of a reaction on subsequent protamine administration.

The condition of a neonate deteriorates twelve hours after repair of a diaphragmatic hernia. The most likely cause for deterioration: A. Hypoglycaemia B. Gastric dilatation C. Tension pneumothorax D. Haemorrhage E. None of the above

ANSWER E Pulmonary hypoplasia still exists and PTH is still sensitive to stressors (hypercarbia, hypoxia, acidosis, hypothermia, pain). The postoperative course, after surgical reduction of congenital diaphragmatic hernias, is often characterized by rapid improvement, followed by sudden deterioration with profound arterial hypoxemia, hypercapnia, and acidosis, resulting in death. The mechanism for this deterioration is the reappearance of fetal circulation patterns, with right-to-left shunting through the foramen ovale and ductus arteriosus. If shunting occurs through the ductus arteriosus, there is a 20-mm Hg or more difference in the PaO2 measured in samples obtained simultaneously from preductal and postductal arteries. If the shunting is predominantly through the foramen ovale, no such gradient exists. Proper sedation is necessary as any stressful stimulus can further exacerbate already elevated pulmonary pressures with resultant increases in shunt flow and further desaturation.

Stellate ganglion is where: A. at the level of the body of C6 (spine of C6) B. posterior to the brachial plexus sheath C. anterior to the dome of the pleura D. anterior to the thoracic duct E. anterior to scalenius anterior

ANSWER E Relations of stellate ganglion Anaesthesia UK website (http://www.anaesthesiauk.com/article.aspx?articleid=100538) * Anterior o The structures anterior to the ganglion include the skin and subcutaneous tissue, the sternocleidomastoid and the carotid sheath. The dome of the lung lies anterior and inferior to the ganglion. * Medial o The prevertebral fascia, vertebral body of C7, oesophagus and thoracic duct lie medially. * Posterior o Structures posterior to the ganglion include the longus colli muscle, anterior scalene muscle, vertebral artery, brachial plexus sheath and neck of the first rib

Post-operative delirium in the elderly is associated with all of the following factors EXCEPT A. pre-existing cognitive deficit B. pre-existing hearing impairment C. pre-existing visual impairment D. post-operative urinary tract infection E. use of general anaesthesia rather than regional anaesthesia

ANSWER E Risk factors In all likelihood, patient predisposition, type of surgery and postoperative factors may be more important to the development of delirium than the choice of anaesthesia. A painstaking prospective study of the risk factors for postoperative delirium has been carried out. Independent predictors of delirium were: 1. Age appears to place patients at greatest risk. Usually, elderly patients (more than 65 years) are on polypharmacy. However, they have decreased ability to metabolize drugs. Furthermore, visual and hearing impairments predispose elderly to disorientation. Postoperative hypoxia is more common in the elderly given the higher incidence of cardiovascular, respiratory and cerebrovascular diseases in this group. 2. Existence of underlying brain disease (cerebrovascular accident, dementia); psychiatric illness or other medical conditions (congestive heart failure, liver disease, renal failure). 3. History of taking medicines causing altered mental status (e.g., benzodiazepines, antidepressants). 4. Patients with preexisting central nervous system disorders (dementia, Parkinson's disease) have higher rates of postoperative delirium. 5. Type of surgery: a) Procedures, longer in duration place patients at increased risk of intraoperative hypoxia. b) Cardiac surgery can result in hypoperfusion and microemboli formation, causing cerebral ischaemia. c) Orthopaedic procedures (repair of femoral neck fracture) chances of fat embolism, may cause delirium. d) Cataract surgery is often associated with delirium due to age, loss of vision and anticholinergic side-effects of the used ophthalmic drugs. 6. Metabolic insults include dehydration, hyponatremia, hyperglycemia, hypoglycemia, acid - base disorders, hepatic disease, renal disease and endocrine disease can cause delirium. 7. Infections like pneumonia, urinary tract infections, intra-abdominal infections and wound infections all can cause confusion in susceptible patients. 8. The sensory overload in the ICU can lead to sleep deprivation, which is a risk factor for developing delirium. 9. Postoperative pain, either uncontrolled or unaddressed has been shown to increase delirium rates. 10. Abuse of alcohol/benzodiazepines, can become delirious secondary to withdrawal symptoms. 11. Type of anaesthesia - sometimes it is stated that delirium is less common after regional as opposed to general anaesthesia. In a large randomized controlled-study of patients undergoing elective total knee replacement, Williams - Russo et al found no statistical difference between the incidence of postoperative delirium in patients following general anaesthesia and that following epidural anaesthesia.

A 65 -year-old female patient with known chronic renal disease and a normal resting preoperative electrocardiogram (ECG) has undergone total hip replacement. Three days postoperatively she complains of chest pain and breathlessness. Her pulse rate is 110 min-1 and blood pressure 130/90 mmHg. The following ECG is recorded. The diagnosis is most likely to be: RBBB, R wave in VI, S wave in I, Q wave in III, t wave inversion in III A. Atrial fibrillation B. hyperkalaemia C. myocardial infarction D. pericarditis E. pulmonary embolus

ANSWER E S1Q3T3

A 65 -year-old female patient with known chronic renal disease and a normal resting preoperative electrocardiogram (ECG) has undergone total hip replacement. Three days postoperatively she complains of chest pain and breathlessness. Her pulse rate is 110 min-1 and blood pressure 130/90 mmHg. The following ECG is recorded. The diagnosis is most likely to be: (RBBB, R wave in VI, S wave in I, Q wave in III, t wave inversion in III A. Atrial fibrillation B. hyperkalaemia C. myocardial infarction D. pericarditis E. pulmonary embolus

ANSWER E S1Q3T3

Regarding postoperative nausea and vomiting (PONV), it has been demonstrated that A. A repeat dose of a 5HT3 antagonist is useful if an initial prophylactic dose was not successful B. Dexamethasone is best given at the end of surgery C. Metocloperamide is effective for PONV prophylaxis D. 5HT3 antagonists are best given at the beginning of surgery E. The efficacy of droperidol is equivalent to ondansetron for PONV prophylaxis

ANSWER E SAMBA 2011 Guidlines Pharmocological Prevention -Dexamethasone, 5-HT3 and droperidol shown to be all equally effective in preventing PONV -Risk reduction by 25% Dexamethasone -given at start of surgery -NNT=6 -no harm documentsed with single dose of dexa 5-HT3 -given at the end of surgery (except dolasetrong, long acting tron) -ondancetron most studies, better at preventing vomiting than nausea NNT Vomit =6, Nausea=7 NNH Headache = 36, LFT = 31, Constipation =26 -Ondancetron 4mg, Granistron 1mg, Dolesatrong 12.5mg, Tropisetron 2mg Butyrophenones -given at end of surgery -protective against headache -nausea > vomiting -NNT = 5 for PONV -NNT =3 for opoid induced nausea (used in combo with PCA, droperidol 2.5mg per 100mg morphine bag) -Haloperidol 0.5-2mg equally as effective, no sedation Combination Therapy Include 1. Dexamethasone + Droperidol 2. 5-HT3 + Droperidol 3. Dexamethasone + 5-HT3 4. Dexamethasone + 5-HT3 + Droperidol

The syndrome of inappropriate ADH secretion may be seen in all except A. carcinoma of the lung B. pulmonary tuberculosis C. septicaemia D. sarcoidosis E. hyperthyroidism

ANSWER E SIADH -characterised by excessive release of ADH from posterior pituitary or another source Presentation -fluid overload (signs of failure and hypertension) -hyponatraemia (symptoms of cerebral edema become prominent (irritability, confusion, seizures, and coma). Diagnosis -Hyponatraemia Na<135 -Urine hyponatramia >20 Causes * Meningitis * Head injury o Subarachnoid hemorrhage * Cancers o Lung cancer (especially small-cell lung cancer, as well as other small-cell malignancies of other organs) * Infections o Brain abscess o Pneumonia o Lung abscess * Guillain Barre syndrome * Drugs o Chlorpropamide o Clofibrate o Phenothiazine o Cyclophosphamide o Carbamazepine o Selective serotonin reuptake inhibitors (SSRIs, a class of antidepressants) o Methylenedioxymethamphetamine (MDMA, commonly called Ecstasy. SIADH due to taking ecstasy was cited as a factor in the death of Leah Betts) o Oxytocin o Vincristine morphine,amitriptaline * Hypothyroidism * sarcoidosis Management of SIADH includes: * Treating underlying causes when possible. * Fluid restriction of 1,200-1,800 mL/day to increase serum sodium. * Hypertonic saline * Drugs o Demeclocycline most potent inhibitor of Vasopressin (ADH/AVP) action. side effect profile, including but not limited to new onset Nephrogenic Diabetes Insipidus (70%), skin photosensitivity, and nephrotoxicity. o Urea: oral daily ingestion has shown favorable long-term results with protective effects in myelinosis and brain damage. o Conivaptan - an antagonist of both V1A and V2 vasopressin receptors. o Tolvaptan - an antagonist of the V2 vasopressin receptor.

The most sensitive monitor for detecting venous gas embolism during neurosurgery is a: A. Capnograph B. Praecordial Doppler transducer C. Praecordial stethoscope D. Pulmonary Artery Catheter E. Transoesophageal echocardiograph

ANSWER E Sensitive TOE>ETN2>Precord doppler>ETCO2 Specific ETN2>TOE>Precord doppler>PA>ETCO2 Quantitative : PA Semiquantitative : ETCO2>PA

Arrest in a 10 year old. Has ventricular tachycardia after a near drowning accident. Patient is intubated and is being ventilated with 100% O2 and has IV access. A single DC monophasic shock of 60J has been given. The next step is to give A adrenaline 10mcg/kg and DC shock 60J B adrenaline 10mcg/kg and DC shock 120J C amiodarone 5mg/kg D DC shock 60J E DC shock 120J

ANSWER E Sequence of actions for APLS UNWITNESSES 1. Establish BLS (ABC) 2. Oxygenate, ventilate, chest compressions 100% BMV or intubation early, resp rate 10-12/min Compression rate 100-120/min uninterrupted Ratio 15:2 IV access or intraosseous 3. Attach defibrillator and Assess rhythm Shockable VT/VF a. initial defibrillate with mono or biphasic 4J/kg -> resume CPR 2min, do not assess rhythm or pulse b. assess rhythm if VF/VT second shock 4J/kg ->resume CPR 2min c. assess rhythm if VF/VT give adrenaline 10mcg/kg IV/IO or 100mcg/kg ETT and shock 4J/kg d. repeat C -adrenaline should be given every second CPR cycle -Consider 4Ts and 4Ps -Refractory VT/VF consider : Amiodarone 5mg/kg bolus Lignocaine 1mg/kg bolus (do not give if amiodarone given) -Last resort attempts Sodium Bicarbonate 1mmol/kg Magnesium Sulphate 0.05-0.1 mmol/kg Potassium Chloride 0.05 mmol/kg 4J/KG Non shockable : continue CPR for 2min WITNESSED ei. cardiac cath lab, ICU, resus etc 3 stacked shocks at 4J 4J 4J

An INCORRECT statement regarding the management of hypocalcaemia is that A. correcting a respiratory or metabolic alkalosis increases the level of ionised calcium B. undiluted calcium should be administered via a central vein as it is irritant to peripheral veins C. acidosis will decrease calcium binding to albumin and therefore increase ionised calcium D. when calcium alone is not sufficient for control of hypocalcaemia, Vitamin D metabolites can be added E. calcium chloride has been shown to be superior to calcium gluconate

ANSWER E Serum ionised calcium < 0.50mmol/L is a medical emergency Management 1. Correct any coexisting respiratory or metabolic alkalosis 2. Avoidance of Calcium lowering drugs 3. Administration of calcium IV -bolus 100-200mg over 10minutes -maintenance of 1-2mg/kg/hr -Ca gluconate 10%, 10ml contains 9.3mg calcium -CaCl 10%, 10 contains 27.2mg calcium -Calcium is irritating to veins and should be diluted before administration and all solutions are best administered through a central vein. o Ionised Ca affected by pH o Acidosis ↓ Ca binding to albumin →↑ ionised Ca o Alkalosis ↑Ca binding (calcium shifts onto the albumin molecule), → ↓ionised calcium. Respiratory alkalosis may occur with acute hyperventilation, thus lowering ionised calcium.

Sickle cell disease (homozygous haemoglobin SS) is frequently associated with A. cardiomyopathy B. chronic respiratory dysfunction C. nephropathy D. peripheral neuropathy E. all of the above

ANSWER E Sickle cell disease -mutation of HbB to for HbS -HbS is biochemically unstable and can precipitate out of solution in the deoxygenated state Heteroxygous -sickle cell trait -30-40%HbS -benign clinical picture but can sickles is stressed -protective again Malaria Homozygous -100%HbS -debilitating -progressive multiorgan damage and early death Clinical presentation (SCD) CVS -cardiomegaly due to anaemia -pulmonary hypertension due to recurrent pulmonary infarcts RESP -dyspnoea, cough, haemoptysis, pleuritic chest pain caused by recurrent pulmonary infaractions -respiratory failure ENT/airway -marrow hyperplasia : front bossing and prominent maxilla -microvascular retinopathy, vitreous haemorrhage and retinal detachment -functional asplenim : hypertrophy of other lymphoid tissues (tonsils and adrenoids) leading to OSA URIN -priaprism -CRI GIT -infarction of spleen : immunie incompetence -gallstones -intrahepatic sicling Skeletal/skin -deformities from marrow hyperplasia -aseptic necrosis and leg ulcers NEURO -TIA -stroke -ACH HAEMO -anaemia -myelosuppresion

PZ86d ANZCA Version [Mar06]Q96 | [Jul06]Q20 | Aug10 Each of the following herbal treatments is associated with an increased risk of perioperative bleeding EXCEPT A. garlic B. ginger C. ginko D. ginseng E. St John's Wort

ANSWER E St John's wart NOT assoc with coagulopathy. Ginseng DOES increase risk of bleeding, so actually 4 G's do this - ginseng, gingko, ginger, garlic.

A 52 year old asthmatic female has a difficult intubation for laparascopy. After 4 attempts at intubation over three minutes, the saturation has decreased from 98% to 73%. The physiological change most likely to have occurred as a consequence is: A. decrease in systemic blood pressure B. decrease in cardiac output C. displacement of the haemoglobin-oxygen dissociation curve to the left D. decrease in pulse rate E. increase in pulmonary artery pressure

ANSWER E Summary from Nunn (p479): * Hyperventilation once PaO2 52.5mmHg * Increased pulmonary artery pressure * Increased cardiac output improves regional blood flow * Sympathetic activation * Vasodilation in every organ except pulmonary vasculature * Right shift of O2 curve by increased 2,3 DPG and acidosis if present * Anaerobic metabolism. Hypoxic Pulmonary vasoconstriction as a result of decreased oxygen tension within the alveoli leading to increased pulmonary artery pressure

Ehlers-Danlos syndrome. Most important to specifically do all EXCEPT: A: Avoid hyperextension of the neck B: Damage to the teeth C: Avoid joint hypermobility D: Gastro oesophageal reflex E: Strict temperature regulation

ANSWER E The Ehlers-Danlos family of disorders is a group of related conditions that share a common decrease in the tensile strength and integrity of the skin, joints, and other connective tissues. Patients can often perform "amazing, almost unnatural, contortions" and worked in circuses (eg the "The India Rubber Man," "The Elastic Lady," and "The Human Pretzel.) All forms of Ehlers-Danlos syndrome share the following primary features to varying degrees: * Skin hyperextensibility * Joint hypermobility and excessive dislocations * Tissue fragility * Poor wound healing, leading to wide thin scars ("cigarette paper scars") * Easy bruising * GORD/gastritis * high, narrow palate and dental crowding and peridontal disease

Significant differences between the LMA-ProSeal™ and the standard laryngeal mask (LMA-Classic™) include all of the following EXCEPT A. a built in bite-block B. a double cuff arrangement C. an improved seal pressure at a given cuff pressure D. an independent oesophageal drain tube E. improved aperture bars to prevent the epiglottis occluding the airway tube

ANSWER E The LMA ProSeal™ achieves a high seal pressure, ranging from 30-35cm H2O with no increase in mucosal pressure, supporting good ventilation. The double cuff design permits the walls of the cuff, when inflated, to match the contours of the pharyngeal and laryngeal surfaces, and improves the seal. The cuff design, with its wider and deeper bowl, produces a better perilaryngeal fit and higher seal pressures without increasing pressure against the pharyngeal mucosa. Mask Size Patient Size 1 Neonates up to 5kg 1.5 Infants 5-10kg 2 Infants/Children 10-20kg 2.5 10-20kg 3 30-50kg 4 50-70kg 5 70-100kg

Endocarditis prophylaxis is appropriate in? A. All patients undergoing dental surgery B. Aortic valve repair C. cardiac transplant D. Rheumatic heart diease E. recently repaired ASD

ANSWER E The new guidelines state that the following patients should be given BE prophylaxis; Prosthetic valve or valve repair material pHx of BE Unrepaired cyanotic congenital heart disease Partially repaired cynanotic CHD with defect over the repair Repaired CHD within the 1st 6 months of repair Heart transplant with valvulopathy

You are anaesthetizing a patient who is undergoing a posterior fossa craniotomy in the sitting position. The praecordial Doppler monitor sounds harshly and the end-tidal carbon dioxide falls. The mean arterial pressure falls from 90 mmHg to 60 mmHg and the central venous pressure rises from 5 mmHg to 20 mmHg. Your immediate management should include all of the following EXCEPT: A. Asking the surgeon to flood the wound with saline B. aspirating the central venous catheter C. compressing the neck veins D. infusing intravenous fluid E. instituting a Valsalva manoeuvre

ANSWER E The release of a valsalva manouver promotes paradoxical embolism. the impairment of systemic venous return caused by the sudden application of substantial PEEP may be undesirable in the face of the cardiovascular dysfunction already caused by the VAE

A patient is scheduled for emergency coronary artery bypass surgery (CABG) 2 hours after receiving tirofiban (Aggrastat) during coronary angiography and an unsuccessful coronary stenting procedure. The most useful strategy to treat or prevent excessive perioperative bleeding is: A. administration of concentrated Factor VIII B. administration of cryoprecipitate C. delaying surgery another 2 hours D. haemofiltration during cardiopulmonary bypass E. platelet transfusion

ANSWER E Tibrofiban (Aggrastat) is an antiplatelet agent -Gycoprotein IIb/IIIa inhibitor (block the final common pathway of platelet aggregation) -does not block platelet adhesion, screction of platelet products, inflammatory effects or thrombin activation Used in the management of STEMI or unstable angina in patients awaiting PCI Parental dosage -2.5mg in 250ml NS (50mcg/ml) -Loading dose 0.4mcg/kg/min for 30 minutes -Maintenance of 0.1mcg/kg/min for up to 72 hours -Infusion should continue through PCI and 12-24 hours after -Stop for 8 hours prior to CABGs Rapid onset and short duration of action -coagulation returns to normal after 4-8 hours of ceassation -half life 2 hours Contraindications -hypersensitivity -active bleeding -history of ICH, AV malformation, aneurysm -thrombocytopenia induced by tibrofiban in past -CVA in last 30 days -recent surgery -suggestive funding of aortic dissection Complications -major bleeding : ICH, retroperiotoneal, pulmonary, spinal-epidural haematoma -anaphylaxis -thrombocytopenia

A 50 year old man with multiple fractures. The BEST parameter to monitor volume resuscitation is: A. Heart rate B. LVEDV C. PCWP D. RVEDV E. Changes in R atrial pressure during inspiration

ANSWER E Traditionally "volume status" is assessed by a clinician considering a range of relevant clinical information and making a "clinical assessment". The relevant factors are generally: 1. heart rate (generally a high pulse rate of 100-160/min, esp if weak & "thready"(?) supports hypovolaemia); not specific so, for exanmple, tachycardia due to pain and anxiety in trauma patients. 2. arterial BP (including checking for postural drop) - low supports hypovolaemia, but of course can be other causes; also maintained by compensatory processes - tachycardia, vasoconstriction in certain vascular beds such as skin, renal, GIT) - so BP normal until volume loss exceeds as certain critical amount. 3. JVP - (Not useful for picking hypovolaemia) 4. urine output (The only "external non-invasive quantifier of tissue perfusion"; requires cathetisation for precise & serial measurements 5. peripheral perfusion assessed by warmth & colour of extremities - Cold & pale suggests "shutdown" - i.e. extreme vasoconstriction suggesting compensation for hypovolaemia). [NB: "Shutdown" = "peripheral circulatory failure"] Such clinical assessment is easy to do in the extreme cases. With invasive monitoring, the above is supplanted by a quantitative assessment of the preload of the ventricles, and this is traditionally assessed as "filling pressure" (and with of course the unstated assumption that ventricular compliance is unchanging). Cerebral perfusion is mostly not useful. Brain blood flow tends to be preferentially maintained (due metaboliccontrol and pressure autoregulation) so a volume depleted patient (assuming no head injury which confuses things) can still be talking with a BP of 50 systolic. Of course, less rousable as BP sinks from there. The right heart filling pressure (generally as CVP, or RAP measured using a central venous line) is useful in most patients, but is considered potentially misleading as an indicator of left heart filling pressures in certain situations. Then the emphasis is on an assessment of "left heart filling pressure" (thus PCWP, PAOP, LAP) but these require a balloon flotation catheter. Ultrasound (in suitable hands) allows a non-invasive assessment of LV volume. As LV volume is a better index of LV preload than LV filling pressure, and can be obtained non-invasively this can be very useful.

A 46 year old brewery worker has cirrhosis of the liver with oesophageal varices and has bled from these varices on one occasion. A portacaval shunt is being considered as definitive treatment for his portal hypertension. The operation would be contraindicated if he had A. a serum bilirubin greater than 50 micromol.litre-1 B. a serum albumin less than 30 gram.litre-1 C. ascites D. all of the above are true E. none of the above are true

ANSWER E Transjugular Intrahepatic Portosystemic Shunt INDICATIONS Absolute 1. Multiple episodes of variceal bleeding 2. Refractory variceal hemorrhage despite adequate endoscopic treatment 3. Refractory ascites Relative * Bleeding portal hypertensive gastropathy * Bleeding gastric varices * Gastric antral vascular ectasia * Refractory hepatic hydrothorax * Hepatorenal syndrome * Budd-Chiari syndrome * Veno-occlusive disease * Hepatopulmonary syndrome * Protein-losing enteropathy due to portal hypertension CONTRAINDICATIONS Absolute 1. RHF 2. Severe Encephalopathy 3. Pulmonary hypertension 4. Biliary obstruction 5. Sepsis 6. Hepatic cysts 7. CCF Relative 1. Haematoma 2. Coagulopathy INR >5 3. Thrombocytopenia 4. Hepatocellular carcinoma

PL30 [Mar10] Maximum dose (with low risk of toxicity) of lignocaine (with adrenaline 1:100000) for liposuction with tumescence technique: A. 3 mg/kg B. 7 mg/kg C. 15 mg/kg D. 25 mg/kg E. 35 mg/kg

ANSWER E Tumescent liposuction - Klein noted that the correct maximum safe dose of lidocaine was never investigated but rather extrapolated from procaine. He showed that infusion of lidocaine, by using the tumescent formula of 0.1% lidocaine with 1:1,000,000 epinephrine, into the subcutaneous tissues of a concentration of 35 mg/kg was safe. The maximum plasma level that was reached at 11-15 hours postoperatively was 0.8-2.7 mcg/mL, well below the toxic level of 5 mcg/mL. Tumescent anesthetic produces a delay in achieving the peak serum lidocaine level and does not produce as high a level compared with conventional local anesthetic.

The most correct statement describing the effect of the antithrombotic agents on the coagulation cascade is that A. low molecular weight heparin directly inhibits activated factor X B. low molecular weight heparin directly inhibits thrombin C. unfractionated heparin directly inhibits activated factor X D. unfractionated heparin directly inhibits thrombin E. ximelagatran directly inhibits thrombin

ANSWER E UFH and LMWH -activate ATIII (increasing it's acitvity 1000 fold) -UFH/ATIII inactivate thrombin and F10a equally -LWMH/ATIII inactivates F10a>thrombin Fondaparinoux -synthetic pentasaccharide -chemically related to heparin -binds and activates ATIII -fonda/ATIII exclusively inactivate F10a -can be used in HITS Ximelagtran -direct thrombin inhibitor -prodrug which is absorbed in SI and converted by the liver to melagatran -removed due to liver toxicity -replaced by dabigatran Dabigatran -oral anticoagulant -direct thrombin inhibitor RELY study -manufacturer sponsered phase III trial -assessor blinded RCT comparing warfarin to dabigatran 110 and 150mg bd in patients with AF -findings -110mg non inferior for stroke and systemic embolization to warfarin -110mg lower risk of bleeding to warfarin -150mg superior for stroke and systemic embolization to warfarin -150mg same risk of bleeding to warfarin RECOVER trial -large, randomized, double blind trial -dabigatran and warfarin in the treatment of acute VTE -non inferior -similar rates of major bleeding -however had more dyspepsia and therefore more drug discontinuation

What do C6/7 motor function do A. flex/extension of fingers B. flex /extend wrist C. shoulder ext rotation / abduction D. elbow pronation/supination E. flexion at elbow

ANSWER E UPPER ARM Nerve Muscle Roots Axillary Deltoid C5 Musculocutaneous Biceps C5,6 Radial Triceps C6,7,8 Radial Brachioradialis C5,6 Radial Wrist extensors C6,7 Radial Finger extensors C7,8 Median Wrist Flexor C6,7 Median Abduct Poll Brev C8,1 Ulnar Finger ABD/ADD C8,1 * all small muscles of the hand are innervated by C8 and Tl roots. * all small muscles of the hand are innervated by the ulnar nerve except for the muscles of the thenar eminence. * both wrist flexion and extension are innervated by C6,7. Nerve Muscle Roots Hip Extension L2,3,4 Femoral Knee Extension L2,3,4 Obturator Thigh Adduction L2,3,4 Sciatic Knee Flexion L5 S1 Sciatic Branches: Peroneal Ankle dorsiflex L4,5 Tibial Plantarflex S1,2 peroneal nerve everts the foot; tibial inverts it.

PC35 ANZCA version [Apr99] [2002-Aug] Q56, [2003-Apr] Q11, [2006-March] Q45, [Jul06] Q30 Side effects of digoxin are increased by A. hyperthyroidism B. hypocalcaemia C. hyperkalaemia D. hypermagnesaemia E. hypothyroidism

ANSWER E UpToDate: * Predispose to toxicity: advanced age, certain cardiac diseases (active ischemia, myocarditis, cardiomyopathy, cardiac amyloidosis, cor pulmonale) and metabolic factors (hypokalemia, hypomagnesemia, hypoxemia, hypernatremia, hypercalcemia, and acid-base disturbances) * In acute cardiac glycoside toxicity, hyperkalemia is an ominous sign and a predictor of morbidity and mortality. Hyperkalemia means much more severe blockade of Na/K ATPase and K is extracellular.

The best indicator of adequate fluid resuscitation in the trauma patient is: A arterial pH B blood pressure C core temperature D pulse rate E serum lactate level

ANSWER E Urine output is the best answer but is not available

In the management of pain for children A. a linear analogue scale is rarely used in assessment of pain B. non-steroidal anti-inflammatory drugs (NSAIDS) often cause renal dysfunction C. opioids are contraindicated under one month of age D. paracetamol is effective orally at a dose of 10 mg.kg-1 E. rectal paracetamol is recommended at doses of 20-30 mg.kg-1

ANSWER E VAS scores ARE useful in children, but probably limited to children seven years of age and older NSAIDs are clearly in widespread use, no reference seems necessary to prove this point. Morphine is relatively contraindicated in neonates due to increased risk of apnoeas, but if the neonate is being discharged from theatre to NICU for monitoring/ventilation, morphine would not be contraindicated. In small doses morphine is used in neonates. * A - "Older children (>5yo) with an understanding of numerical concepts may be able to use these; vertical formats such as ladders and thermometers seem to be most successful" * B - "Adequate hydration is required in the postoperative period. Consequently, after major surgery, oral NSAIDs are usally only started once the child is eating and drinking adequately, and prescribed for a set time period, eg 3 days" * C - "caution in infants under 6 months of age, as dose reduction may be required". Premature infants listed as CI in MIMS * D - "A dose of 10mg/kg is no more effective than placebo for minor pain in children" * E - "The recommended dose in children is...20mg/kg rectally every 6 hours"

After an infusion of normal saline causing isovolumetric haemodilution what does not occur? A. Increased cardiac output B. Increase oxygen extraction C. Capillary vasodilatation D. Increased venous return E. Increased mixed venous saturation

ANSWER E http://www.martilinstruments.com/files/Scientific%20Publications,%20Clinical%20Relevance%20of%20Blood%20Viscosity/6%20Blood%20viscosity%20and%20intensive%20care%20medicine/IsovolemichemodilutioninIC.pdf

What colour is the label for subcutaneously administered drugs A. Pink B. Yellow C. Brown D. Red E. Beige

ANSWER E http://www.safetyandquality.gov.au/wp-content/uploads/2012/03/Labelling-Recommendations-2nd-edition-February-2012.pdf Color coding for route of adminstration Red - Intra arterial Blue - Intravenous Yellow - Neural tissues Beige - subcutaneous Pink - Miscellaneous

In patients suffering from multiple sclerosis it is better to avoid the use of A. epidural lignocaine B. neostigmine C. pancuronium D. spinal anaesthesia E. suxamethonium

ANSWER E suxamethonium is associated with a large efflux of potassium in debilitated patients and should be avoided

Which of the following statements regarding patients with ankylosing spondylitis is FALSE? A. amyloid renal infiltration is rarely seen B. cardiac complications occur in less than 10% of cases C. normochromic anaemia occurs in over 85% of cases D. sacroileitis is an early sign of presentation E. uveitis is the most common extra-articular manifestation

ANSWER E ANKYLOSING SPONDYLITIS OVERVIEW • inflammatory arthritis of sacroiliac joints and spine, characterised by progressive stiffening and fusion of the axial skeleton • associated with HLAB27 in > 90% • male: female 4:1 ANAESTHETIC IMPLICATIONS ARTICULAR • Potentially difficult intubation: - kyphosis - fixation of the spine, risk of cervical spine fracture - atlanto-occipital subluxation - TMJ dysfunction - Crico-arytenoid arthritis • Spinal disease begins in the sacro-iliac joints and moves cranially • likely difficult mask ventilation also • Difficult neuraxial blocks - paramedian spinal most successful - increased incidence epidural haematomas • Limited chest expansion (arthritic involvement of costovertebral joints) • Can be difficult to position intraoperatively NON-ARTICULAR • anterior uveitis (most common extra-articular manifestation) synonymous with iritis. • fibrosing alveolitis, pulmonary fibrosis • Aortic Regurgitation (1%) • Rarely mitral valve involvement and conduction defects • Amyloidosis causing renal impairment • Cauda equina syndrome • Inflammatory bowel disease • normocytic anaemia TREATMENT • Anti-TNFa medication (infliximab, entanercept): - increases incidence of wound infection - avoid in severe heart failure

Which statement regarding the use of opiates for the management of acute pain is true? A. in adults weight is the best predictor of opioid requirements B. metabolism to codeine-6-glucuronide produces the analgesic effect of codeine C. morphine produces more nausea and vomiting than pethidine D. pethidine is superior to morphine in the management of renal colic pain E. tramadol has a lower risk of respiratory depression than other opioids at equianalgesic doses

ANSWER E A. FALSE : age is the best predictor of opiods requirements B. FALSE : codiene is metabolised to morphine via CYP2D6 (the main active metabolite) -Caucasians 5-10% and Asians 2% deficient; less effective in these patients -codeine-6-glucuronide produces some of its analgesic effect C. False - pethidine induced more nausea and vomiting than morphine when used parenterally in the ED (level III-3) and in first 2 hours after gynaecological surgery (level II, APM 2010) D. FALSE - not proven (level II, APM 2010) E. TRUE

The LEAST desirable position for the tip of a central venous catheter which has been inserted into the left internal jugular vein is A. mid-way along the left brachiocephalic vein B. at the junction of the left brachiocephalic vein and the superior vena cava (SVC) C. in the SVC at the level of the carina D. at the junction of the SVC and the right atrium E. in the right atrium

ANSWER E>B B. catheter tip will be against vessel wall and erode : causing mediastinal or pleural haemorrhage and drug deposition E. catheter tip erode into pericardium: causing pericardial haemorrhage and drug deposition

Thiopentone 2.5% precipitates with (when injected through the same cannula): A. Pancuronium B. d-Tubocurarine C. Vecuronium D. Suxamethonium E. Alcuronium F. All of the above

ANSWER F Thiopentone (highly alkaline solution) is incompatible with the following drugs and MUST NOT be mixed with: * Muscle relaxants o succinylcholine o pancuronium o vecuronium * Opioids o codeine o morphine o pethidine o alfentanil o sufentanil * Catecholamines o metaraminol o norepinephrine * Others o midazolam o atropine o amikacin o dimenhydrinate o diphenhydramine o insulin o penicillin G

Regarding perioperative use of processed salvaged red blood cells, A. malignant cells are removed by the washing process B. storage of salvaged cells should be limited to six hours C. the high free haemoglobin levels are associated with renal failure D. the salvaged cells have lower oxygen carrying capacity than banked blood E. the survival of the salvaged red blood cells is significantly impaired

Answer : C A - False. Some malignant cells are removed, but not all. Need irradiation and washing to guarantee removal, and even then controversial area B - FALSE : if collected under aseptic conditions with a saline-wash device, blood may be stored at room temp for up to 4 hours or at 1-6deg for up to 24 hours (provided that storage at 1-6deg is begun within 4 hours of ending the collection). Storage times are the same for recovered blood whether unwashed or washed. C - True. Free Hb in cell salvage can cause renal failure, but most removed during separation and washing. D - False. Banked blood has lower O2 carrying capacity due to depletion of 2,3-DPG over time. E - False. No difference in survival times.

PP95 [Jul07] A one year old child arrests with VT. Has had 2x DC shocks, and 100 mcg adrenaline. Further 1x DC shock given. What is next step? A. 20 J DC shock B. 40 J DC shock C. 50 mg amiodarone D. 100 mcg adrenaline E. 1000 mcg adrenaline

Answer is C In VF/VT: 1. Defibrillate the heart: Give 1 shock of 4 J kg-1 if using a manual defibrillator. 2. Resume CPR: Without reassessing the rhythm or feeling for a pulse, resume CPR immediately, starting with chest compression. 3. Continue CPR for 2 min. 4. Pause briefly to check the monitor: If still VF/VT, give a second shock at 4 J kg-1 if using a manual defibrillator 5. Resume CPR immediately after the second shock. 6. Consider and correct reversible causes (see above: 4Hs and 4Ts). 7. Continue CPR for 2 min. 8. Pause briefly to check the monitor: If still VF/VT: Give adrenaline 10 microgram kg-1 followed immediately by a (3rd) shock. 9. Resume CPR immediately and continue for 2 min. 10. Pause briefly to check the monitor: If still VF/VT: Give an intravenous bolus of amiodarone 5 mg kg-1 and an immediate further (4th) shock. 11. Continue giving shocks every 2 min, minimising the breaks in chest compression as much as possible. 12. Give adrenaline immediately before every other shock (i.e. every 3-5 min) until return of spontaneous circulation (ROSC). The scenario described in the question outlines steps 1-9. Next step if still in VT is amiodarone 5 mg/kg ~ 50mg. So answer = C.

In a Jehovah's Witness patient undergoing a revision of a total hip replacement, the most effective technique to minimise post-operative anaemia would be A. epidural anaesthesia B. induced hypotension C. intra-operative cell saving D. intra-operative intentional normovolaemic haemodilution E. pre-operative administration of recombinant erythropoietin

Hard to say which one is the most effective. * A. epidural anaesthesia - effective * B. induced hypotension - effective * C. intra-operative cell saving - effective if the patient allows its use * D. intra-operative intentional normovolaemic haemodilution - false and the odd one out as the others are all mentioned in the CEACCP article: "Acute normovolaemic haemodilution is often unacceptable to Jehovah's Witnesses, as it involves the removal and storage of blood before haemodilution." (Continuing Education in Anaesthesia, Critical Care & Pain | Volume 4 Number 2 2004) * E. pre-operative administration of recombinant erythropoietin - effective

A 24 year old female with mitral valve prolapse develops atrial flutter during a diagnostic laparoscopy. The drug most likely to revert this arrhythmia is A. Digoxin B. Amiodarone C. Verapamil D. Esmolol E. Adenosine

None of them? With regard to Atrial Flutter (not AF): Reversion best achieved with DC Cardioversion Reversion rate for pharmacological agents: * Ibutilide 38-76% * Sotalol 19% * Flecanide 13% * Verapamil 5% * Adenosine/Amiodarone -not of use in cardioverting Atrial Flutter ( but useful for rate control)

The sural nerve A. Is a branch of the posterior tibial nerve B. Supplies the skin of the anterior two thirds of the sole of the foot C. Lies anterior to the lateral malleolus at the ankle D. Reaches the foot in contact with the short saphenous vein E. Supplies the small muscles of the foot

SURAL NERVE Description : Purely sensory nerve supplying the sensation to the lateral foot Branch of the Tibial nerve Follows the long saphenous vien

What is the APGAR score? What the implications and management of APGAR scores?

Score 8 to 10 * This is 90% of all neonates. Normally, no treatment is required except nasal and oral suctioning, drying of the skin and maintenance of normal body temperature. * Carefully reevaluate the neonate at 5 minutes because some neonates hypoventilate when stimulation ceases. * As soon as stable, wrap the neonate in a warm blanket and return to the parents . [edit] Score 5 to 7 * This score suggests the neonate suffered mild asphyxia just before birth. They usually respond to vigorous stimulation with oxygen blown over the face. * If they are slow to respond and to become pink, ventilate with 80% to 100% oxygen by bag and mask. * By 5 minutes of age, patients who have Apgar scores of 5 to 7 at 1 minute are usually well. * At 2 minutes of age, PaO2 is usually 50 to 70 mm Hg (FIO2 = 0.21),PaCO2 is 40 to 50 mm Hg, pHa is about 7.15 and base deficit is approximately 12 mEq/L. * By 10 minutes of age, pHa increases to 7.30, PaCO2 decreases to below 40 mm Hg, and base deficit usually returns to normal (-3.0 to +3.0 mEq/L). Score 3 to 4 * These neonates are moderately depressed and are typically cyanotic with poor respiratory effort. They usually respond to bag-and-mask ventilation, breathe, and become pink. * If they have not breathed spontaneously, ventilating the lungs with a bag and mask may be difficult because the airway resistance exceeds that of the esophagus. If so, gas may preferentially enter and distend the oesophagus, stomach, and gut, which may interfere with ventilation and cause vomiting and regurgitation. Decompressing the stomach makes it easier to ventilate the lungs * If the neonate has not breathed or is breathing ineffectively, insert an endotracheal tube before ventilating the lungs. * Obtain umbilical artery and vein blood from a double-clamped segment of umbilical cord to measure blood gases and pH. The blood gases are frequently abnormal: PaO2 below 20 mm Hg, PaCO2 above 60 mm Hg, and pH below 7.15. * If the pH and base deficit are unchanged or worse on a sample of blood obtained from a radial artery, an umbilical artery catheter should be inserted, and if necessary, sodium bicarbonate should be administered. Score 0 to 2 * These neontates are severely asphyxiated and require immediate resuscitation. * It is not appropriate to wait to see how the patient will do as their condition usually worsens. Resuscitate immediately.

Type of aortic dissection - which classically NON-operative management: A. DeBakey Type I B. DeBakey Type II C. Stanford A D. Stanford B E. Stanford C

The Stanford classification divides dissections into 2 types, type A and type B. * Type A involves the ascending aorta (DeBakey types I and II); type B does not (DeBakey type III). * This system also helps delineate treatment. Usually, type A dissections require surgery, while type B dissections may be managed medically under most conditions. Type A ->Surgery Type B -> Medical The DeBakey classification divides dissections into 3 types. * Type I involves the ascending aorta, aortic arch, and descending aorta. * Type II is confined to the ascending aorta. * Type III is confined to the descending aorta distal to the left subclavian artery. o Type III dissections are further divided into IIIa and IIIb. o Type IIIa refers to dissections that originate distal to the left subclavian artery but extend both proximally and distally, mostly above the diaphragm. o Type IIIb refers to dissections that originate distal to the left subclavian artery, extend only distally and may extend below the diaphragm.

A two-year-old child sustains a simple fracture of the lower end of the forearm one hour after having a full meal. The most appropriate approach is to A. postpone surgical reduction for 12 hours and treat as an elective case B. allow reduction after 4 hours using a rapid sequence induction, cricoid pressure and intubation C. allow immediate reduction using a regional technique D. allow immediate reduction, after gastric emptying with a tube followed by rapid sequence induction, cricoid pressure and intubation E. wait 4 hours and treat as elective

A is best answer? (C,D,E are more incorrect) ?B - never heard of a four/five hours fast, but it's close to six. This is an area fought with most contention. My own approach is at least six hour fast and low threshold of RSI with any of the following: - same day injury - meal close to time of injury - unwell looking and distressed child - pain and opioid - afterhour with little help around Face mask/LMA if next day, pt fasted, hungry, etc. Main thing is ensure adequate depth of anaesthesia prior to any manipulation (airway or bones).

The incidence of fat embolism syndrome following a unilateral closed femoral fracture is A. 0 -3% B. 4 - 7% C. 8 -11% D. 12 - 15% E. 16 - 19

%ANSWER A

What percentage of the total greenhouse gas effect is due to the use of volatile anaesthetic agents? A. 0% B. less than 1% C. 2% D. 5% E. 10

%ANSWER B

The risk of transmission of Hepatitis C to a health care worker from an infected patient after a needle stick injury is approximately A. 0.5% B. 2-8% C. 8-15% D. 15-20% E. 20-30

%ANSWER B B. 2-8% - true: * "The average incidence of seroconversion to HCV after unintentional needle sticks or sharps exposures from an HCV-positive source is 1.8 percent (range, 0-7 percent) [40]. A study from Japan reported an incidence of HCV infection of 10 percent based upon detection of HCV RNA by reverse transcriptase polymerase chain reaction (RT-PCR) [41]. No incidence studies have documented transmission after exposure of nonintact skin. Transmission of HCV from blood splashes to the conjunctiva has been described. Hepatitis C virus has been demonstrated to survive on environmental surfaces for at least 16 hours but not four or seven days" (Uptodate, Management of healthcare workers exposed to hepatitis B virus or hepatitis C virus)

Regarding mixed venous blood oxygen saturation, which statement is correct? a. it is collected from the right atrium b. it is used to calculate cardiac output c. it can be used to accurately measure the mixed venous pO2 d. it has no impact on the A-a gradient e. it is usually 40

%ANSWER B Normal range for SvO2 is 60 - 80% Is collected from PA CO can be calculated by a modification of the Fick equation, where VO2 = (CO x CaO2) - (CO x CvO2 ) Where CO = Cardiac Output, Ca = Oxygen concentration of arterial blood and Cv = Oxygen concentration of mixed venous blood VO2 = Arterial Oxygen Transport - Venous, Oxygen Transport = (CO x CaO2 x 10) - (CO x CvO2 x 10) = CO x (CaO2 - CvO2) x 10 = CO (Hb x SaO2 x 13.8) - CO (Hb x SvO2 x 13.8) = CO x Hb x 13.8 x (SaO2 - SvO2) VO2 = CO x Hb x 13.8 x (SaO2 - SvO2) VO2 = Oxygen consumption CO = cardiac output SaO2 = arterial O2 sats SvO2 = venous O2 sats

Following superficial and deep cervical plexus blockade for carotid endarterectomy, the incidence of ipsilateral phrenic nerve block is A. less than 10% B. 20-30% C. 50-60% D. 80-90% E. nearly 100

%ANSWER C

When a new diagnostic test is evaluated in a population of subjects in whom the diagnosis is known, the following results are obtained Disease known Disease known to be present to be absent New test result positive 80 40 New test result negative 20 180 In this population the POSITIVE predictive value of this test is closest to A. 10% B. 33% C. 67% D. 80% E. 90

%ANSWER C

Relative humidity - air fully saturated at 20 %. What is the relative humidity at 37 degrees ? A. 20 B. 30 C. 40 D. 50 E. 60

%ANSWER C 1m3 of air at 20˚C, 100% saturated, contains about 17g of water. If it is warmed to 37˚C, the mass of water vapour or absolute humidity is still the same but the relative humidity is only 39%, as at 37˚C, 1m3 of air contains 44g of water vapour when fully saturated and the ratio of 17 to 44 gives a value of 39%

An eight-year-old boy presents with bleeding one week after a tonsillectomy. When you see him in the emergency room, he is conscious but restless, tachypnoeic, pale and cool peripherally, with a pulse rate of 135 and blood pressure of 80/60. What percentage of his blood volume is he most likely to have lost? A. 10 -15% B. 15-25% C. 25 - 45% D. 45-55% E. more than 55

%ANSWER C An 8 yr old is expected to have a systolic BP = 80 + (2 x age in yrs). So for this child expect SBP=95 The APLS guidelines suggest that a child with greater than 40% blood loss will be "responsive only to pain", clearly not this child. In fact, an agitated child falls into the <25% blood loss group. Also of note is that HR=135. A child of 8 years will have a HR in the range of 80-120. ATLS manual only has three (<25, 25-45, >45). Other references have different grading: * <15% will have mild tachycardia * <25% decreased pulse pressure but normal-ish BP * <35% decreased BP * >40% coma and severe decrease BP.

AM10 Which of the following would you NOT expect to find in a patient with malignant hyperpyrexia? A. Alkalosis B. Increased serum potassium C. Muscular rigidity D. Increased temperature

ANSWER A

Neonate desaturate faster than adult at induction because A. FRC decrease more B. Faster onset of induction agents C. More difficult to pre-oxygenation

??? Reasons for neonatal desat 1. High metabolic rate 2-3 x adult (required mainly for temperature control, high BSA to mass) 2. Small absolute FRC (same as adult 30ml/kg) therefore less oxygen 3. CC >FRC, neonate generates autoPEEP by partial closure of glottis, this is lost at induction, causing airway closure and V/Q mismatch

Magnesium for treatment of pre-eclampsia. What is the therapeutic level? (I think this may be a repeat of an old question, but i remember two of the options were- A: B: 3 - 5 C: 5 - 7 D: E:

??? THerapeutic range 1.7-3.5 mmol/L

Regarding College Professional Document PS9 - sedation for colonoscopy - the following equipment must be present (NB: The wording was 'present', not 'ready access to' as used for defib in PS9) a. Defibrillator b. Mechanical ventilator c. Anaesthetic machine d. Suxamethonium e. Dantrolene

?incorrectly remembered PS9 Sedation Guidlines : Facilities and Equipment Must be present 1. Appropiate lighting 2. Operating table, trolley or chair with head down capacity 3. Suction and suction catheters 4. Oxygen supply and masks 5. A means of inflating the lungs with oxygen : Air Veva and mask 6. Drugs : adrenaline, atropine, dextrose 50%, lignocaine, naloxone, flumazenil, portable emergency O2 7. Pulse oximeter and blood pressure 8. A means of summoning emergency assistance Must be readily available 1. Advanced airway management equipment : masks, oropharyngeal airways, LMA, ETT and larngoscopes) 2. ECG and debrilllator 3. device for measuring ETCO2

AM44 ANZCA version [2004-Aug] Q131, [2005-Apr] Q21 Which of the following blood gas results would be consistent with an episode of MH? A. pH 7.13 pCO2 55 HCO3 18 BE -9.1 B. pH 7.24 pCO2 63 HCO3 26 BE 0 C. pH 7.22 pCO2 30 HCO3 12 BE -13.8 D. pH 7.46 pCO2 45 HCO3 31 BE 7.5 E. pH 7.32 pCO2 44 HCO3 22 BE -3.1

ANSWER A

A patient with pulmonary hypertension secondary to lung disease presents for a laparotomy. Regarding this patient's anaesthetic management A. an alpha-agonist is the inotrope of choice B. hypothermia is protective against rises in pulmonary artery pressure C. isoflurane will tend to decrease pulmonary artery pressure D. ketamine is an appropriate anaesthetic agent E. right heart failure is not a concern

A patient with pulmonary hypertension secondary to lung disease presents for a laparotomy. Regarding this patient's anaesthetic management * A. an alpha-agonist is the inotrope of choice - probably true and best answer: o there are No α-1 adrenergic receptors are present in the pulmonary circulation (Blaise, Anaesthesiology, 2003, 99(6):1421) so α-1 agonists are fine and may assist RV function by increasing coronary perfusion pressure (although some prefer dobutamine initially becuase it increases contractility and may pulmonary vasodilate) o the wording is confusing and might subequently change now. Both the Blaise article and Stoelting 5th ed. suggest that causes of hypotension are multifactorial and should be treated accordingly. Specifically pulm HTN crisis requiring inotropy, the 'inotrope' of choice might be milrione (or possibly dobutamine), however R heart ischaemia and low SVR (with fixed PVR) are important causes of hypotension specifically treated with noradrenaline * B. hypothermia is protective against rises in pulmonary artery pressure - false o Hypothermia increases PVR (A & A ,Volume 96(6), June 2003, pp 1603-1616) * C. isoflurane will tend to decrease pulmonary artery pressure - false o PVR does not change with volatiles except N2O which does increase PVR (Stoelting Pharmacology p47) o Isoflurane has no effect on baseline pulmonary vessel tone. (Blaise, Anaesthesiology, 2003, 99(6):1421) * D. ketamine is an appropriate anaesthetic agent - false o 'In patients who have pulmonary artery pressure, ketamine seems to cause a more pronounced increase in pulmonary than systemic vascular resistance' (Miller, p348) o 'The sympathomimetic properties of ketamine may preclude use in the setting of pulmonary hypertension (Yao, p96) o In-vitro ketamine increases PVR in rat lung...(and)...ketamine attenuates endothelium-dependent pulmonary vasorelaxation in response to acetylcholine and bradykinin ...(and)...sympathetic innervation of the pulmonary circulation does exist (Blaise, Anaesthesiology, 2003, 99(6):1421) * E. right heart failure is not a concern - false

Normochromic normocytic anaemia is not associated with: A. Increased TIBI B. Rheumatoid arthritis C. Acute blood loss D. Polymyalgia rheumatica E. Hypothyroidism

ANSWER A

PC Clonidine is A. Alpha 2 agonist centrally that acts presynaptically B. Alpha 1 agonist C. Alpha 2 antagonist D. Alpha 1 antagonist E. ?

ANSWER A

A diagnosis of pulmonary embolism is most strongly suggested by A. intraluminal filling defects or vascular cutoffs on angiography B. PaO2 less than 85 mmHg and an abnormal lung perfusion scan C. PaO2 less than 85 mmHg and an elevated PaCOz D. right ventricular hypertrophy with right ventricular strain and right axis deviation on electrocardiography E. "unmatched" ventilation-perfusion defects

ANSWER A

The earliest indication of hypocalcaemia following thyroidectomy is usually: A. Tingling of face, nose & hands B. Positive Chvostek's sign C. Carpopedal spasm D. Hyperthermia E. Tachycardia

ANSWER A

A known alcoholic with anorexia and nausea has become jaundiced. His urine is dark and his faeces pale. He has discomfort in the right hypochondrium. The AST (SGOT) is 2000 IU.l-l, the alkaline phosphatase 100 IU.l-I and the serum bilirubin is 75 micromol.1-l. The best treatment would be A. withdrawal of alcohol B. expectant C. operation to remove obstruction (gallstones tumour) D. urgent liver biopsy E. endoscopy

ANSWER A

The features of Pierre Robin sequence include cleft palate, micrognathia and: A. Glossoptosis B. Craniosynostosis C. Macroglossia D. Microstomia

ANSWER A

TEG tracing given, post cardiac surgery. Had quite slim tail (ie fibrinolysis) but broader 'shoulders'. A: Fibrinolysis B: Hypofibrinogenaemia C: Platelet dysfunction D: Heparin effect E: Surgical bleeding

ANSWER A

The best clinical indicator of SEVERE Aortic stenosis A. Presence of thrill B. Mean Gradient 30mmHg C. Area 1.2 cm2 D. Slow rising pulse and ESM radiating to carotids E. Shortness of breath

ANSWER A

AM12 [Mar91] [Aug94] Dystrophia myotonica: A. The myotonia is abolished by regional (?spinal/epidural) anaesthesia B. The severity of the cardiomyopathy parallels that of the skeletal muscle C. Suxamethonium induces myotonia in all clinical cases D. Pulmonary aspiration is common E. Serum creatine phosphokinase is elevated

A: False * General anaesthesia, regional anaesthesia and neuromuscular blockers are not able to prevent or relieve this skeletal muscle * AKA Myotonia Dystrophica. A form of myotonic dystrophy. Not to be confused with muscular dystrophy. * Most common muscular dystrophy among Caucasians * AD with variable penetrance * Chromosome 19 locus q 12.3 * Defect is in Na conductance (via serine/threonin protein kinase) resulting in increased intra-cellular Na * incidence of 1 in 8,000. * A multisystem disorder, * onset 10 - 30 yrs (a congenital form exists) * distal muscles more involved than proximal muscles * Plasma CK normal or sl elevated. (M&M, Harrisons) * Progression: o myotonia early (persistent contraction after voluntary or mechanical stimulation) then o atrophy of face and neck, also o frontal balding and cataracts, o usually low intelligence, o Also possible dysarthria, dysphagia, and mild ophthalmoplegia. antagonist m. spasms when relaxes Drug effects * Prolonged respiratory depression with any resp depressant drug * Treatment: Na channel blockers: quinine, phenytoin and procainamide System effects * CVS : o conduction blocks (ECG : increased PR, ST and A flutter o Cardiomyopathy (<10%)-note that severe skeletal muscle disease is not necessary for a sever cardiomyopathy (see AIC 2001: Anaesthesia and myotonia-an Australian experience.) * Resp : o reduced VC, o exp. Effort/Pressure; o aspiration risk (common) * GIT : o Smooth mm is involved leading to dysmotility in stomach/oesophagus, colon, uterus * Endocrine : o Insulin resistance, o adrenal atrophy, o infertility in women, o testicular atrophy, primary testicular abnormalities * Early cataracts Diagnosis * Clincial * +ve FHx * EMG/Muscle bx * Genetic testing for mild cases useful Congenital form occurs in children born to mothers with myotonic dystrophy; some patients present with profound hypotonia at birth, with facial diplegia, feeding, respiratory difficulties, and skeletal deformities (such as clubfeet). Later, during childhood, delayed developmental progression is noted contraction...Stoelting B: False * Cardiac abnormalities have been well described in patients with myotonic dystrophy. The most prominent cardiac disorders include atrioventricualr conduction delays (A-V block), atrial flutter and fibrillation, and ventricular dysrhythmias. First-degree A-V block may actually precede the onset of skeletal muscle symptoms. Sudden death may be a result of the abrupt onset of third-degree A-V block. Other cardiac abnormalities associated with myotonic dystrophy include mitral valve prolapse, left ventricular diastolic dysfunction, and cardiac failure...Barash, Clinical Anaesthesia C: True (?) * Succinylcholine produces an exaggerated contracture and its use should be avoided. The myotonic response to succinylcholine can be so severe that ventilation and tracheal intubation are difficult or impossible...Barash, Clinical Anaesthesia * Succinylcholine will produce contractions lasting for several minutes, thus making intubation and ventilation a challenge...Miller D: True * Pharyngeal muscle weakness in conjunction with delayed gastric emptying increases the risk of aspiration of gastric contents...Barash, Clinical Anaesthesia E: True * MD patients may also have mildly elevated CK levels...Miller

Spinal anaesthesia in infants A. often causes hypotension if the infant is awake B. has a lower failure rate than in adults C. eliminates the risk of postoperative apnoea D. lasts for a longer time than in adults E. may be performed at a higher spinal level than in adults

ALL FALSE

Uterine relaxation might be required in: A. Breech delivery B. Manual removal of placenta C. Transverse lie for caesarian section D. Assisted delivery for breech

ALL TRUE

Side effects or complications of coeliac plexus block using a neurolytic agent include all of the following EXCEPT A. hypotension B. paraplegia C. intestinal hypermotility D. pneumothorax E. retroperitoneal haematoma

ALL TRUE * 3 common transient adverse effects 1. Local pain (96%) 2. Diarrhoea (44%) 3. Hypotension (38%) * Other complications include o Lower extremity weakness o Paraplegia o Parasthesia o Adjacent organ puncture o Infection o Bleeding → retroperitoneal haematoma o Epidural injection o Subarachnoid injection o Intravascular injection o Pneumothorax o Chylothorax * Neurology caused by o Direct injury to spinal cord or somatic nerves o Spinal cord ischaemia

A Hetastarch of intermediate plasma expansion and intermediate plasma duration is; A. 10% HES 250/0.6 B. 10% HES 200/0.5 C. 6% HES 450/0.7 D. 6% HES 130/0.4 E. 3% HES 200/0.5

ANSWER D

In a neonate the main resistance in a circle system with CO2 absorber A. APL valve B. expiratory and inspiratory unidirectional valves C. tubing D. ETT E. HME filter

ANSWER D

Side effect No side effect Drug given 5 2 Drug not given 1 5 Odds ratio is A. 1 B. 4 C. 8 D. 12.5 E. cannot calculate

ANSWER D

With regards to obstructive sleep apnoea (OSA), which of the following statements is INCORRECT? A. hypoxaemia is the main stimulus to arousal B. the main method of treating this syndrome is with Continuous Positive Airway Pressure (CPAP) C. this syndrome is the most likely diagnosis in patients presenting with excessive daytime sleepiness D. this syndrome occurs in up to 5% of adults E. this syndrome rarely has an obstructive component

ANSWER E

Popliteal block placed from the lateral approach: A: Passes through semimembranosus B: Has eversion of the foot as the end point C: Has increased failure rate compared to a posterior approach D: ? E: Can be performed supine or prone

ANSWER E A FALSE, semmembranosus is muscle on the back of thigh B. FALSE Common peroneal : dorsiflesion and eversion Tibial nerve: plantar flexion and inversion C FALSE intermediate block, easier to perform, higher success rate D ? E. TRUE

The risk of latex allergy is not increased by a history of: (a) allergy to avocados (b) multiple surgical procedures (c) allergy to kiwi fruit (d) atopy (e) dandruff

ANSWER E Cross reactivity to avocados banana kiwi fruit tomato grape passion fruit celery chest nut

Which of the following statements regarding anaphylactic and anaphylactoid reactions is FALSE A. cross-sensitivity between latex and bananas, chest nuts and avocardo has been reported B. cross-sensitivity of cephalosporins with penicillin is about 8% C. gelatin solutions used for resuscitation can worsen any reaction D. reactions to neuromuscular blocking agents are more common in females E. vecuronium is more likely to cause an anaphylactoid rather than an anaphylactic reaction

ANSWER E Steroidal cause anaphylaxis, benzylisoquinoliums cause anaphylactoid.

SF (Q141 Aug 2008) Patient with placenta acreta. Surgical management MOST likely to save her life A B lynch suture around the uterus for external tamponade B Rusch balloon in the uterus for internal tamponade C ligation of the internal iliac arteries D ligation of the uterine arteries E subtotal or total hysterectomy

ANSWER E Tricky question. A : B-Lynch Suture - Developed in 1997 by B Lynch. Heavy suture that envolopes and mechanically compress an atonic uterus in severe PPH, not yet used much in SE asia. No data so far to suggest usefulness in Placenta Acreta. B : Rusch balloon -used for atony or lower segment bleeding C : Ligation of internal iliac arteries -high rate of failure 50% as uterine arteries still bleeding D : Ligation of uterine arteries -high rate of failure 50% as internal iliac still bleeding E : subtotal or total hysterectomy -indicated if accreta is diagosed before delivery. -hysterectomy is performed with placenta still intact

Patient (age 60s) having a total knee replacement; on antihypertensives incl. an ACE Inhibitor. You have started a blood transfusion via a leukocyte-depleting filter in PACU after rapid loss into drains. What complication occurs with THIS filter in THIS situation? A. Air embolism B. Haemolysis C. Increased risk of postoperative infection D. Consumption of coagulation factors E. Severe hypotension

ANSWER E Relatively few adverse effects. 1. Profound hypotension : Negatively charged surfaces of leukoctye reduction filters may vause contact activation with release of bradykinin-like vasoactive substances. Profound hypotension has been reported in patients taking angiotension-converting-enzyme (ACE) inhibitors and receiving pretransfusion leukocytereduced blood products—platelets in particular. Presumably, ACE inhibitors decrease bradykinin degradation thereby prolonging its intravascular half-life. 2. 'Red Eye' Syndrome Allergic conjunctivitis linked to recipients of prestorage leukocyte-reduced RBCs. This condition is characterized by erythema (100%), periorbital edema (16%), eye pain (15%), and itching. In most patients, symptoms resolve within 48 hours, but erythema may require up to three weeks for complete resolution.

AZ19 [Aug92] Pressure inside the GIT is increased by: A. Nitrous oxide B. Suxamethonium C. Neostigmine D. T6 epidural

All true: * A. diffusion, * B. increased intragastric pressure, * C. augment vagal tone, * D. antagonise SNS so increased vagal effects

In a healthy person lying quietly on his back, the intracranial pressure (referred to the level of the interventricular foramen) is in the range: A. 0-5 cmH2O B. 5-15 cmH2O C. 15-30 cmH2O D. 2-3 mmHg E. 15-18 mmHg

Answer: B Conversion: 10.2cm H2O = 7.3mmHg, so 5-15cm H20 would be 3.5-11mmHg

Haemoglobin in infants: A. 30% fetal Hb at birth, adult levels by 3 months B. 70% fetal Hb at birth, negligible amounts by 6 months C. Hb 90 at 6 months normal doesn't need Ix D. Hb 200 at birth unless delayed cord clamping E. ?

At birth HbF 75-80% HbF deliver O2 effectively to tissues in hypoxic conditions but holds onto O2 after brith Compensated by polycythemia Hb 120-200 At 6/12 HbF replaced by HbA coupled with increases in 2,3 DPG there is right shift of curve which aids O2 offloading in tissues Although, neonates have high Hb, CO and low risk of arterial disease, their trigger for transfusion is higher due to the presence of HbF. TRIGGER FOR TRANSFUSION Neonate = Hb 120 Infant/Child = Hb 6.5 Adult = Hb 80

A young woman has an MVA when driving home from a nightclub. She was known to have moderate MDMA (ecstasy) consumption. Which of the following complications is least likely to be due to MDMA? A. Hyponatraemia B. Hypotension C. Pneumothorax D. Sweating E. Tachycardia

Minor clinical symptoms and signs seen with MDMA Tachycardia Elevated mood Hypertension Confusion Mydriasis Ataxia Dry mouth Nystagmus Sweating Bruxism (jaw clenching)

The optimal skin temperature of a premature neonate being nursed in an incubator is: A. 28C B. 30C C. 32C D. 34C E. 36C

Neutral Thermal Environment - environmental air temperature at which baby with a normal body temp has minimal metabolic rate and therefore minimal oxygen consumption Neonatal Core Normothermia 36.6-37.5 Guidelines from the Womens for Skin Temperature Neonates < 1750g : 36.8 degC Neonates > 1750g : 36.5 degC Optimal abdominal skin temperature control at 36.5°C (slightly warmer than previously reported but less than 37.5°C) is recommended for premature neonates nursed on radiant warmer beds. Frequent monitoring of core temperature for these infants is strongly recommended to avoid central hyperthermia. PEDIATRICS Vol. 79 No. 1 January 1987, pp. 47-54

AZ76b ANZCA Version [2006-Mar] Q123, [Jul06] Q50 A 70-yr-old man is to undergo removal of cataract and intraocular lens implantation. He has long-standing atrial fibrillation and is on warfarin. He has no other health problems. He has never had a stroke. A sub-tenon's block is planned for the procedure. His INR is 2.5. What should be the perioperative management of his warfarin therapy and anticoagulant status? A. Interrupting warfarin therapy is optional for this procedure. If warfarin is interrupted for 5 days to allow normalisation of INR (< 1.5) no other perioperative anticoagulant prophylaxis is necessary B. Warfarin therapy should be ceased 5 days preoperatively and no other perioperative anticoagulant prophylaxis is necessary. Surgery should proceed if INR is < 1.5 C. Warfarin therapy should be ceased 5 days preoperatively. He should commence daily low molecular weight heparin, omitting the dose on the day of surgery. Surgery should proceed if INR is < 1.5 D. Warfarin therapy should be ceased 5 days preoperatively. He should take daily clopidogrel till surgery. Surgery should proceed if INR is < 1.5 E. Warfarin therapy should be ceased 5 days preoperatively. He should take daily low dose aspirin till surgery. Surgery should proceed if INR is < 1.5

None. # Retrobulbar haemorrhage is rare with Subtenon's block # Retrobulbar haemorrhage is increased in people treated with warfarin even if it is stopped ? tissue fragility ? comorbidity # Retrobulbar haemorrhage is uncommonly sight threatening if a surgeon manages it immediately # The risks of thromboembolism are life threatening Continue warfarin. Test on the day of surgery. Don't stop aspirin or NSIADs Only for planned uncomplicated phaco INR<1.5 -peribulbar block INR <3.0 proceed with surgery -subtenon -topical LA INR >3.0 -postpone

Male 60's sudden onset of chest pain , L arm weakness and hoarse voice, ECG is unchanged from old (T inversion laterally), CXR normal, BP135/80, Pulse 110/min. Next step in management: A)Aspirin B)SNP infusion C)GTN infusion D)Metoprolol E)Heparin

Stupid question : next step would be to diagnose Sounds like aortic dissection therefore D

How do you calculate the inspiratory time constant for lungs A. resistance multiplied by compliance B. resistance divided by compliance C. compliance divided by resistance D. resistance minus compliance E. resistance plus compliance

answer a

Anaphylaxis presents with bronchospasm as SOLE feature in what percent of cases? A. 50% B. 30% C. 15% D. 5% E. 0.5

%ANSWER D

Complications of coeliac plexus block include A. hypertension B. failure of erection C. constipation D. paraplegia E. dysaesthesia along L3-4

ANSWER A 3 common transient adverse effects Local pain (96%) Diarrhoea (44%) Hypotension (38%) Other complications include Lower extremity weakness Paraplegia Parasthesia Adjacent organ puncture Infection Bleeding → retroperitoneal haematoma Epidural injection Subarachnoid injection Intravascular injection Pneumothorax Chylothorax Neurology caused by Direct injury to spinal cord or somatic nerves Spinal cord ischaemia Spasm or thrombosis of the artery of Adamkiewicz → spinal cord ischaemia T8 to L4

Stellate ganglion is located: A. At the level of the body of C6 (spine of C6) B. Posterior to the brachial plexus sheath C. Anterior to the dome of the pleura D. Anterior to the thoracic duct E. Anterior to scalenius anterior

ANSWER E Anatomy -ganglion formed by the fusion of the inferior cervical and the first thoracic ganglion -it is present 80% of subjects -located anterior to vertebral body of T7, above the neck of the 1st rib Relations of stellate ganglion *Anterior skin and subcutaneous tissue, the sternocleidomastoid and the carotid sheath. The dome of the lung lies anterior and inferior to the ganglion. *Medial The prevertebral fascia, vertebral body of C7, oesophagus and thoracic duct lie medially. *Posterior the longus colli muscle, anterior scalene muscle, vertebral artery, brachial plexus sheath and neck of the first rib. Indications *Pain syndromes -Complex regional pain syndrome type I and II -Refractory angina -Phantom limb pain -Herpes zoster -Shoulder/hand syndrome -Angina *Vascular insufficiency -Raynaud's syndrome -Scleroderma -Frostbite -Obliterative vascular disease -Vasospasm -Trauma -Emboli Contraindications -Coagulopathy -Recent myocardial infarction -Pathological bradycardia -Glaucoma%%Technique Equipment : image intensifier, resus equipment, assistant Position : supine with neck slightly extended and mouth open to relax neck muscles Landmarks : trachea, carotid sheath, sternocleidomastoid, cricoid cartilage and Chassaignac's tubericle Method 1. Retract SCM and carotid artery lateral 2. Palpate Chassaignac'ss tubercle (anterior tubercle of the transverse process of the 6th cervical vertebrae) 3. skin is pressed firmly onto the tubercle to reduce distance and push apex of lung out of the path of needle 4. needle is inserted onto the tubercle then redirectly medially and inferiorly towards the body of C6 until the body is contacted 5. withdraw 1-2mm to move needle out of belly of longus coli muscle 6. confirm position using fuoroscopy, contrast should spread cephalad and caudad along vertebral body 7. Aspirate and inject LA 3ml at a time looking for inadvertent arterial injection, total of 10-15ml 8. Onset of Horner's indicates successful block Complications *Misplaced needle -Haematoma from vascular trauma -Carotid trauma -Internal jugular vein trauma -Neural injury -Vagus injury -Brachial plexus roots injury -Pulmonary injury -Pneumothorax -Haemothorax -Chylothorax (thoracic duct injury) -Oesophageal perforation *Spread of local anaesthetic -Intravascular injection: -Carotid artery -Vertebral artery -Internal jugular vein *Neuraxial/brachial plexus spread: -Epidural block -Intrathecal -Brachial plexus anaesthesia or -injury (intraneural injection) *Local spread: -Horseness (recurrent laryngeal nerve) -Elevated hemidiaphragm (phrenic nerve) *Infection -Soft tissue (abscess) -Neuraxial (meningitis) -Osteitis

The drug LEAST likely to cause bronchospasm and/or anaphylactoid reactions: A. Protamine B. Labetalol C. Esmolol D. Vancomycin E. Ketamine

ANSWER E A. FALSE : Histamine release B. FALSE C. FALSE D. FALSE E. TRUE

PZ. The active metabolite of ketamine is: a. Hydroxyketamine b. Hydroxynorketamine c. Ketamine glucuronide d. Ketamine sulphonamide e. Norketamine

ANSWER E norketamine Metabolites of ketamine are norketamine and dehydronorketamin


Related study sets

Block 3 - Exam, Chapter 17 - Q/A -Copy COPY

View Set

Esthetics: Application & Removal procedures

View Set

Battle of the Books: I Am Malala

View Set

Sports Medicine Chapter 1- Careers in Sports Medicine

View Set